100% found this document useful (1 vote)
623 views720 pages

HY Systems

This document provides an overview of key points about different types of heart failure: - Left heart failure presents with pulmonary symptoms like dyspnea due to backup of pressure in the lungs. Right heart failure presents with systemic symptoms like jugular vein distension and edema. - Congestive heart failure involves both left and right heart failure. Cor pulmonale is right heart failure due to lung disease rather than left heart issues. - Peripheral edema can result from right heart failure, low albumin levels, kidney problems, certain drugs, or diet. Unilateral edema can occur from lymphatic issues, thyroid problems, or tissue deposits. The document also reviews murmurs and heart sounds in some cardiac abnormalities.
Copyright
© © All Rights Reserved
We take content rights seriously. If you suspect this is your content, claim it here.
Available Formats
Download as PDF, TXT or read online on Scribd
100% found this document useful (1 vote)
623 views720 pages

HY Systems

This document provides an overview of key points about different types of heart failure: - Left heart failure presents with pulmonary symptoms like dyspnea due to backup of pressure in the lungs. Right heart failure presents with systemic symptoms like jugular vein distension and edema. - Congestive heart failure involves both left and right heart failure. Cor pulmonale is right heart failure due to lung disease rather than left heart issues. - Peripheral edema can result from right heart failure, low albumin levels, kidney problems, certain drugs, or diet. Unilateral edema can occur from lymphatic issues, thyroid problems, or tissue deposits. The document also reviews murmurs and heart sounds in some cardiac abnormalities.
Copyright
© © All Rights Reserved
We take content rights seriously. If you suspect this is your content, claim it here.
Available Formats
Download as PDF, TXT or read online on Scribd
You are on page 1/ 720

MEHLMANMEDICAL

HY CARDIO
MEHLMANMEDICAL.COM

YouTube
@mehlmanmedical

Instagram
@mehlman_medical

MEHLMANMEDICAL.COM 2
MEHLMANMEDICAL.COM

HY Cardio – by Dr Mike Mehlman

Important basic heart failure points


- Presents as pulmonary findings (i.e., dyspnea, orthopnea, paroxysmal
nocturnal dyspnea).
- This is because left-heart problems cause a backup of pressure onto the
pulmonary circulation, leading to increased pulmonary capillary hydrostatic
pressure à transudation of fluid into the alveolar spaces (pulmonary
edema). Sometimes this can also cause pleural effusion.
- Left atrial pressure (LAP) = pulmonary capillary wedge pressure (PCWP).
Left heart failure - Therefore, if there is left heart pathology, PCWP is high (exceedingly HY).
- Conversely, if a Q gives you normal PCWP, you know there’s nothing wrong
with the left heart. Qs will often give high PCWP and low BP, where you
need to know immediately that means cardiogenic shock.
- What USMLE will do is give you some sort of left-heart pathology +
dyspnea, and then ask for the cause of the dyspnea à answer = “increased
pulmonary capillary hydrostatic pressure.” Another answer in this case is
“increase alveolar-arteriolar (A-a) oxygen gradient.”
- Presents as systemic findings – i.e., jugular venous distension (JVD) and
peripheral edema.
- Since blood cannot enter the right heart as easily, it backs up to the neck
veins (JVD) and venous circulation (increased hydrostatic pressure in veins
à transudation of fluid into legs). The Q might mention that central venous
Right heart failure pressure is high.
- Hepatosplenomegaly can also be seen in RHF but is very rare on USMLE.
- Normal jugular venous pressure (JVP) is 3cm above the sternal angle. JVD
would be higher than this. Sometimes questions can write that jugular
venous pulsations are seen 3cm above the sternal angle and the student
erroneously thinks this means JVD, but this is not the case.
- Congestive heart failure = left heart failure + right heart failure.
- The most common cause of right heart failure is left heart failure. Simply
adding the two together, we now call that congestive heart failure.
Congestive heart failure
- In congestive heart failure, we’ll see both left- and right-heart failure
findings – i.e., patient will have dyspnea, JVD, and peripheral edema.
- PCWP is elevated in these patients, since the left heart has pathology.
- Cor pulmonale is defined as right-heart failure due to a pulmonary cause.
In other words, the left heart is completely normal in cor pulmonale and
PCWP is normal.
- Cor pulmonale will be a patient who has JVD and peripheral edema in the
setting of obvious and overt lung disease, such as 100-pack-year smoking
history, cystic fibrosis, or pulmonary fibrosis. These can present with lung
findings such as wheezes, where you as the student need to say, “It just
doesn’t seem like they’re focusing on left-heart failure as the cause of the
right-heart failure here. It seems the 100-pack-year smoking Hx causing
Cor pulmonale COPD is why the right heart is failing.”
- The patient can have a “boot-shaped” heart colloquially, which refers to
right ventricular hypertrophy without left ventricular hypertrophy.
- If the patient has COPD, the massively hyperinflated lungs will push the
heart to the midline, causing a long, narrow cardiac silhouette, with a point
of maximal impulse in the sub-xiphoid space.
- You must know that pulmonary hypertension is the reason the right heart
decompensates. In both cor pulmonale and congestive heart failure, the
right heart experiences increased afterload because of pulmonary
hypertension.

MEHLMANMEDICAL.COM 3
MEHLMANMEDICAL.COM

- Endothelin 1 is vasoconstrictor and key mediator in pulmonary


hypertension. USMLE wants you to know this is increased in both cor
pulmonale and left heart failure. Bosentin is an endothelin 1 receptor
antagonist.
- Nitric oxide synthase, in contrast, USMLE wants you to know is decreased
in pulmonary hypertension (makes sense, since NO dilates).
- A loud P2 and tricuspid regurgitation are HY findings in cor pulmonale. I
discuss these in more detail in the tables below.

Hyper-quick causes of bilateral pitting peripheral edema


- Right heart failure (either due to cor pulmonale or congestive) à ¯ ability to fill right
heart à ­ central venous pressure à ­ systemic venous hydrostatic pressure à
transudation of fluid from systemic veins/venules into interstitium of legs.
- Tangential, but Step 1 NBME asks why HTN doesn’t automatically cause peripheral
Cardiac edema à answer = “increased pre-capillary resistance.” The USMLE wants you to
know that arterioles are responsible for the majority of peripheral resistance; in the
setting of high BP, the reason capillary hydrostatic pressure isn’t automatically high
enough where the transudation threshold is reached is because arterioles constrict,
thereby ­­ resistance and reducing excessive blood flow through the capillaries.
- Cirrhosis à ¯ hepatic production of albumin à ¯ intravascular oncotic pressure à
Hepatic
transudation of fluid from systemic veins/venules into interstitium of legs.
- Proteinuria à hypoalbuminemia à ¯ intravascular oncotic pressure à transudation
Nephrogenic
of fluid from systemic veins/venules into interstitium of legs.
- Dihydropyridine calcium channel blockers (i.e., amlodipine, nifedipine).
Drugs - Imatinib (targets BCR/ABL tyrosine kinase in CML).
- Miscellaneous mechanisms not important for USMLE. Just know above drugs do it.
- Strict vegetarianism or veganism à ¯ dietary protein consumption à ¯ intravascular
Dietary oncotic pressure à transudation of fluid from systemic veins/venules into interstitium
of legs.
Pregnancy - A little bit of peripheral edema is normal in pregnancy due to compression of IVC.

Hyper-quick causes of unilateral non-pitting edema


- Malignancy (e.g., peau d’orange of breast), Hx of surgery (e.g., mastectomy),
Lymphatic insufficiency
Wuchereria bancrofti (elephantiasis).
- Pretibial myxedema (Graves) à mucopolysaccharide deposition in skin +
surrounding edema.
- Myxedma (severe hypothyroidism) à despite the name, it refers to general
“severe hypothyroidism,” not just skin changes; can cause carpal tunnel
Thyroid
syndrome.
- “Pretibial myxedema” is only seen in Graves. Paradoxical hyperthyroidism seen
in Hashimoto causing pretibial myxedema is astronomically rare and will get you
questions wrong on USMLE.

HY Valvular / flow abnormalities on USMLE


- Fixed splitting of S2
- Can sometimes be associated with a systolic flow murmur, since more
blood L à R from the LA à RA means more blood flow across the
Atrial septal defect pulmonic valve. So Q might say “fixed splitting of S2 and a systolic
murmur.”
- Sometimes can be seen in Qs as “wide, fixed splitting.” I only mention
this because some students get pedantic / ask about this. “Wide splitting”

MEHLMANMEDICAL.COM 4
MEHLMANMEDICAL.COM

just means right ventricular hypertrophy. So if the Q says “wide, fixed


splitting,” they’re saying the patient has RVH due to an ASD.
- Patent foramen ovale = ASD on USMLE. Don’t confuse with patent ductus
arteriosus (discussed below).
- USMLE loves asking questions showing you change in oxygen in the
chambers of the heart and making you choose ASD, VSD, etc.

- For example, you can see above that somehow O2 increases from the
SVC to the RA, which is ordinarily impossible. The only way this could occur
is if an ASD is present, where oxygenated blood moved from LA à RA.
- ASDs can sometimes be responsible for “paradoxical emboli,” where a
DVT leads to stroke. This is ordinarily impossible, since a clot embolizing to
the lungs via the venous circulation has no way of reaching the arterial
circulation. But if an ASD is present, the clot can go RA à LA à LV à up to
the brain, causing stroke.
- Holosystolic (aka pan-systolic) murmur at lower left sternal border.
- Can be associated with a diastolic rumble or enlarged left atrium (if more
blood going L à R across VSD, then more blood is returning to the LA from
the lungs à LA dilatation).
- Seen as part of tetralogy of Fallot (VSD, RVH, overriding aorta, pulmonic
stenosis).
- If a VSD is repaired, USMLE wants ­ LV pressure, ¯ RV pressure, and ¯ LA
pressure as the changes now seen in the heart.
- VSD does not cause cyanosis at birth. Only years later after the higher
blood flow to the lungs results in pulmonary hypertension, followed by
right ventricular hypertrophy and reversal R à L (Eisenmenger) does the
patient become cyanotic.
Ventricular septal defect
- Murmur can be silent or soft at birth, followed by loud at 7 days of life.
The USMLE will ask why the murmur is louder now à answer = decreased
pulmonary vascular resistance – i.e., the lungs open up during the first
week of life, resulting in decreased RV pressure and an increase in the L à
R pressure gradient (louder murmur).
- Conversely, if they ask why the murmur was softer at birth compared to
now, the answer is “increased pulmonary vascular resistance,” where the
lungs were still closed at the time, so there was a lesser gradient L à R
(softer murmur).
- Similar to ASD Qs, USMLE loves giving you diagrams with changes in O2
between the chambers and then making you infer we have a VSD.

MEHLMANMEDICAL.COM 5
MEHLMANMEDICAL.COM

- NBME loves this style of Q. You can see O2 somehow increased from RA
to RV. The only way this is possible is if we have a VSD where oxygenated
blood moves from LV à RV.

- This one might initially appear a little more difficult. This is Eisenmenger
syndrome, where we have a reversal of flow from RV à LV across the VSD.
The NBME is known to show this diagram as well.
- Seen in Down syndrome.
Atrioventricular septal defect - Between the atrium and ventricle, aka “endocardial cushion defect,”
although this latter term can also apply to ASD and VSD in Downs.
- Holosystolic (pan-systolic) or just regular “systolic,” 29 times out of 30.
- Q on NBME 20 offline for Step 1 has MR as “mid-systolic,” but I contend
this is erratum.
- Most USMLE questions will not mention it radiating to the axilla.
- Highest yield cause of MR on USMLE is post-MI papillary muscle rupture.
USMLE is obsessed with this. They’ll say hours to days after an MI, patient
Mitral regurgitation
has new-onset systolic murmur à answer = MR.
- Seen acutely in rheumatic heart disease (valve scars over years later and
becomes mitral stenosis).
- Can be caused by general ischemia / dilated cardiomyopathy.
- Can cause JVD (i.e., back up all the way to the right heart); this is asked
multiple times on the new Step 1 NBMEs.
- Described as “rumbling diastolic murmur with an opening snap”; can also
be described as “decrescendo mid-late diastolic murmur” (i.e., following
the opening snap).
- Can cause a right-sided S4 if the pressure backs up all the way to the right
Mitral stenosis
heart (seen on NBMEs sometimes; this confuses students because they
think S4 must be LV, but it’s not the case). An S4 is a diastolic sound heard
in either the LV or RV when there is diastolic stiffening due to high
afterload.

MEHLMANMEDICAL.COM 6
MEHLMANMEDICAL.COM

- 99% of mitral stenoses are due to Hx of rheumatic heart disease (i.e., the
patient had rheumatic fever as a child, where at the time it was mitral
regurg, but years later it has now become mitral stenosis).
- One 2CK NBME Q mentions patient with history of rheumatic heart
disease who, years later, now has 4/6 rumbling diastolic murmur without
an opening snap; this is still mitral stenosis. Although opening snap is
buzzy for MS, just be aware it’s not mandatory and that this Q exists on
NBME.
- Other HY presentation on USMLE is pregnant women with new-onset
dyspnea in 2nd trimester and a diastolic murmur. This is because 50%
increase in plasma volume by 2nd trimester causes the underlying
subclinical MS to become symptomatic. Don’t confuse this with severe
dyspnea and peripheral edema in late third-trimester, which is instead
peripartum cardiomyopathy (antibody-mediated).
- The 1% of MS that’s not due to Hx of RF can be marantic (non-bacterial
thrombotic endocarditis; NBTE) à endocarditis seen due to
hypercoagulable state in the setting of malignancy, where the vegetations
are small and verrucous, on both sides of the valve. This is in contrast to
bacterial endocarditis, which causes large, floppy vegetations that lead to
MR, not MS.
- Libman-Sacks endocarditis seen in SLE is due to antiphospholipid
antibodies and is a type of NBTE.
- Most common murmur.
- Described as mid-systolic click.
- “Myxomatous degeneration” is buzzy term that refers to connective
tissue degeneration causing MVP in Marfan and Ehlers-Danlos.
- Almost always asymptomatic. On 2CK forms, they want you to know
about “mitral valve prolapse syndrome,” which is symptomatic MVP that
presents as repeated episodes of “fleeting chest pain” on the left side in an
otherwise healthy patient 20s-30s. They might say there is Hx of MI in the
family, but this is MVPS, not MI. Answer on surgery form is “no treatment
Mitral valve prolapse necessary."
- USMLE loves using MVP as a distractor in panic disorder questions,
particularly on the 2CK Pysch CMS forms. They will give long paragraph
about panic attack/disorder + also mention there’s a mid-systolic click;
they’ll ask for cause of patient’s presentation à answer = panic disorder,
not MVP à student is confused because they say mid-systolic click, but
the MVP isn’t the cause of the patient’s presentation; the panic disorder is;
MVP’s are usually incidental, benign, and asymptomatic.
- MVP does not progress to mitral regurg almost always. So don’t think
that MVP and MR are the same.
- Decrescendo holo-diastolic (pan-diastolic) murmur; can also be described
as “early diastolic murmur,” or “diastolic murmur loudest after S2.”
- Causes wide pulse pressure (i.e., big difference between systolic and
diastolic pressures, e.g., 160/50, or 120/40) à results in head-bobbing and
bounding pulses (don’t confuse with slow-rising pulses of aortic stenosis).
- The bounding pulses can be described on NBME as “brisk upstroke with
precipitous downstroke.” In turn, they can just simply say, “the pulses are
Aortic regurgitation brisk,” meaning the systolic component is strong.
- I would say 4/5 times bounding pulses means AR. The other 1/5 will be
PDA and AV fistulae (discussed below). Bounding pulses occur when blood
quickly leaves the arterial circulation. In AR, the blood quickly collapses out
of the aorta back into the LV. In PDA, it leaves the aorta and enters the
ductus arteriosus; in AV fistulae, it leaves for a vein.
- Highest yield cause on USMLE is aortic dissection à can retrograde
propagate toward the aortic root causing aortic root dilatation and AR.

MEHLMANMEDICAL.COM 7
MEHLMANMEDICAL.COM

- Even though MVP is most common in Marfan and Ehlers-Danlos, AR is


second most common in these patients, since if they get aortic dissection,
this can lead to AR.
- Can lead to volume overload on the LV and eccentric hypertrophy.
- Mid-systolic murmur, or just “systolic” murmur; can also be described as
“late-peaking systolic murmur with an ejection click.”
- Radiates to the carotids. This descriptor shows up quite frequently on
NBME (way more than radiation to the axilla for MR).
- Causes slow-rising pulses, aka “pulsus parvus et tardus” (don’t confuse
with bounding pulses of AR).
- SAD à Syncope, Angina, Dyspnea; classic combination seen in AS, albeit
not mandatory. If you get a question where they say systolic murmur but
you’re not sure of the diagnosis, if they say chest pain or fainting, you
Aortic stenosis
know it’s AS.
- Often caused by bicuspid aortic valve. The patient need not have Turner
syndrome and often won’t. Bicuspid valve is usually inherited as an
autosomal dominant familial condition.
- The bicuspid valve need not calcify in middle-age prior to the AS forming.
Bicuspid valve can present with AS murmur in high schooler on NBME.
- Do aortic valve replacement on 2CK if 1) cross-section of valve is <1.0
cm2, or 2) there is SAD. They ask both of these as separate Qs where they
want valve replacement.
- Will be described on USMLE as a holosystolic murmur that increases with
inspiration.
- Right-sided heart murmurs get worse with inspiration à diaphragm
moves down à decreased intra-thoracic pressure à increased right-heart
filling.
- Can cause pulsatile liver.
- Highest yield cause of TR on USMLE is pulmonary hypertension / cor
Tricuspid regurgitation pulmonale. I see this all over the NBME exams. For whatever reason, these
conditions do not cause pulmonic regurg; they cause tricuspid regurg. In
other words, if you see tricuspid regurg in a Q, your first thought should be
pulmonary hypertension or cor pulmonale (right heart failure due to a
pulmonary cause).
- IV drug user endocarditis is obvious risk factor for TR, but weirdly
nonexistent on USMLE.
- Carcinoid syndrome is theoretical cause but lower yield.
- Nonexistent murmur on USMLE. I don’t think I’ve ever seen this assessed
once on any NBME exam for Steps 1 and 2 combined.
Tricuspid stenosis - In theory, would be a rumbling diastolic murmur similar to mitral
stenosis, but would increase with inspiration since it’s on the right side of
the heart.
- Same as with tricuspid stenosis, this is a nonexistent murmur on USMLE.
I’ve never seen it assessed.
Pulmonic regurgitation
In theory it would be the same as aortic regurg but on the right (i.e., holo-
diastolic murmur), but increases with inspiration.
- Seen in tetralogy of Fallot.
- Described as mid-systolic murmur, or just regular “systolic” murmur, that
increases with inspiration, at the left sternal border, 2nd intercostal space.
Pulmonic stenosis
This is the theoretical location, whereas AS is the 2nd intercostal space on
the right, not left. But the USMLE often isn’t strict about murmur locations
this way.
- Ductus arteriosus is special vessel in fetal circulation that connects the
Patent ductus arteriosus
proximal pulmonary trunk to the descending arch of the aorta. This allows
(PDA)
for blood to bypass the high-resistance lungs in utero. After birth, this

MEHLMANMEDICAL.COM 8
MEHLMANMEDICAL.COM

vessel should close, resulting in a remnant called the ligamentum


arteriosum, but sometimes it does not close à PDA.
- If a PDA occurs, blood moves in the neonate LàR (i.e., opposite of in
utero) from the descending arch of the aorta to the pulmonary trunk.

- You can see in the above diagram, somehow the blood become more
oxygenated from the RV to the pulmonary artery, which is ordinarily
impossible. The only way this could have occurred is if oxygenated blood
came LàR from the aorta to the pulmonary artery via a PDA.
- Murmur described three ways on USMLE: 1) continuous, machinery-like
murmur; 2) pan-systolic pan-diastolic murmur (meaning it’s continuous
throughout both systolic and diastole); and 3) to-and-fro. The latter shows
up on 2CK offline NBME 6.
- Classically associated with congenital rubella (HY). They’ll give a kid born
with a PDA and then ask what the mom experienced while pregnant;
answer = arthritis and/or rash (rubella often presents as arthritis in adults).
- Indomethacin (NSAID) will close the PDA.
- Prostaglandin E1 is used to keep a PDA open (if a kid with congenital
heart malformations is born cyanotic and we need to buy time until
surgery).
- An open PDA can mask cyanosis in a newborn in a variety of conditions
(i.e.,., hypoplastic left heart syndrome or pre-ductal coarctation). If they
tell you a kid is born with normal APGAR scores but a week later becomes
cyanotic and they ask why, the answer is “closure of ductus arteriosus.”
- Systolic murmur seen in the setting of higher heart rate caused by
infection, anemia, or pregnancy. Caused by increased flow across the
pulmonic and/or aortic valves.
- Known as a functional murmur because this means it goes away once the
heart rate comes back down.
Functional (flow) murmur - Seen all over 2CK Peds forms, where they try to trick you into thinking the
kid has a valvular pathology of some kind, but there isn’t; there will merely
be an infection or simple viral infection.
- Can be seen sometimes with ASD, where the patient will have fixed
splitting of S2 “plus a systolic murmur” à merely higher right-sided
volume, so more flow across the pulmonic valve.
- On 2CK Peds form; described as a murmur in the neck that abates when
Venous hum the kid is laid supine + the neck rotated.
- Benign + don’t treat.
- Associated with cardiac tumors (i.e., myxoma in adult, or rhabdomyoma
in kids for tuberous sclerosis).
“Ball-in-valve” murmur
- Described as a diastolic rumbling murmur that abates when the patient is
re-positioned unconventionally (e.g., onto his or her right side).
- Both are diastolic sounds.
S3 versus S4 - S3 is due to high volume/preload in the left ventricle, causing a
reverberation against the wall.

MEHLMANMEDICAL.COM 9
MEHLMANMEDICAL.COM

- S3 can sometimes be physiologic (i.e., normal / no problem) in pregnancy


and high-endurance athletes. Patient will have eccentric hypertrophy
(sarcomeres laid in linear sequence). If pathologic, it is due to dilated
cardiomyopathy with reduced ejection fraction (<55%) or high-output
cardiac failure (EF >70%).
- There is one question on IM CMS form 7 where they give an S3 in
diastolic dysfunction. I’m convinced this is an erratum, but I need to
mention it because it exists on the NBME form.
- S4 is due to high pressure/afterload on the left (but sometimes right)
ventricle, causing a stiffened ventricle with diastolic dysfunction and
concentric hypertrophy (sarcomeres laid in parallel). It is always
pathologic. It is usually caused by systemic hypertension causing afterload
on the LV, or aortic stenosis.
- S4 can sometimes be right-sided on USMLE. There is a 2CK Q where they
give severe mitral stenosis and say there’s an S4, but it’s for the RV not LV.
Some weird/annoying points:
- The combo of S3 and S4, seen together in the same vignette, can be seen
in high-output cardiac failure. For example, they will say a patient as an AV
fistula/conduit, or has Paget disease, and they will say there’s S3 and S4
and ask for diagnosis à answer = high-output cardiac failure. The take-
home point is that high-output failure can present with either an isolated
S3 or the combo of S3 and S4 together, but never S4 alone on USMLE.
- One of the highest yield cardiac sounds on USMLE, almost always
overlooked by students.
- Means pulmonary hypertension or cor pulmonale on USMLE.
- The pulmonic valve slams shut due to high pressure distal to it.
- For example, they’ll give a smoker who simply has a loud P2 à this just
means patient has pulmonary hypertension. Not complicated.
Loud P2 - Also recall that I said above that highest yield cause of tricuspid regurg on
USMLE is pulmonary hypertension / cor pulmonale. So both what I want
you to remember is both TR and loud P2 for this.
- Sometimes the UMSLE will just say “loud pulmonic component of S2,” or
“loud S2,” rather than saying “loud P2.” I’ve never seen “loud A2” on
USMLE, but in theory this means systemic hypertension.
- A soft P2 refers to pulmonic stenosis, but is LY.
- Means right ventricular hypertrophy on USMLE.
- A2 and P2 are far apart.
- You don’t have to worry about the mechanism. But in short, the more
Wide splitting of S2 pressure you have in a ventricle, the more delayed the semilunar valve will
close. So if we have RVH, P2 occurs later, widening the split.
- Wide splitting of S2, right-axis deviation on ECG, and right bundle branch
block (RBBB) all = right ventricular hypertrophy on USMLE.
- Means left ventricular hypertrophy on USMLE.
- A2 occurs after P2 (normally we have A2 before P2).
- Left ventricular pressure is high and A2 delayed to the point that it
Paradoxical splitting of S2
actually occurs on the opposite side of P2.
- Paradoxical splitting of S2, left-axis deviation on ECG, and left bundle
branch block (LBBB) all = left ventricular hypertrophy on USMLE.
- Refers to narrowing of the aortic arch (this is referred to as coarctation;
do not use the word stenosis to describe this).
- Classically seen in Turner syndrome, but absolutely not mandatory.
Coarctation of the aorta Shows up idiopathically in plenty of NBME Qs. I point this out because
students often think the patient must have Turner syndrome.
- Presents as upper extremities that have higher BP, brisk pulses, and are
warmer; the lower extremities have lower BP, weak pulses, and are cooler.

MEHLMANMEDICAL.COM 10
MEHLMANMEDICAL.COM

- Sometimes the Q can just say, “the radial pulses are brisk.” à The
implication is, “Well if they’re saying specifically that the radial pulses are
brisk, that must mean the pulses in the legs aren’t.”
- Murmur sound not important for USMLE. Can sometimes be described as
a systolic murmur heart in the infrascapular region.
- Can cause LVH with left-axis deviation ECG (on Step 1 NBME).
- USMLE doesn’t give a fuck about pre- vs post-ductal. Pre-ductal in theory
will be a very sick neonate. Post-ductal will be an adult (most cases).
- Confusing condition when you’re first learning things that is low-yield on
Step 1 but high-yield on 2CK.
- The vertebral artery (goes to brain) is the first branch of the subclavian
artery (goes to arm).
- If there is a narrowing/stenosis of the proximal subclavian prior to the
branch point of the vertebral artery, this can lead to lower pressure in the
vertebral artery.
- This can cause a backflow of blood in the vertebral artery, producing
miscellaneous neuro findings such as dizziness.
Subclavian steal syndrome - Blood pressure is different between the two arms.
- USMLE will ask the Q one of two ways: 1) they’ll give you dizziness in
someone who has BP different between the arms and then ask for merely
“subclavian steal syndrome,” or “backflow in a vertebral artery” as the
answer. Or 2) they’ll give you BP in one of the arms + give you dizziness,
then the answer will be, “Check blood pressure in other arm.”
- Next best step in Dx is CT or MR angiography (asked on 2CK NBME).
- I should point out that probably 3/4 questions on USMLE where blood
pressure is different between the arms, this refers to aortic dissection. But
1/4 is subclavian steal syndrome. As per my observation.
- Presents same as subclavian steal syndrome with otherwise unexplained
dizziness, but blood pressure is not different between the arms because
the subclavian is not affected.
Vertebral artery stenosis
- Caused by atherosclerosis. CT or MR angiography can diagnose.
- “Vertebrobasilar insufficiency” is a broader term that refers to patients
who have either subclavian steal syndrome or vertebral artery stenosis.
- 2CK Neuro forms assess vertebral artery dissection, where they want you
to know a false lumen created by dissection in a vertebral artery can lead
to stasis and clot formation, which in turn can embolize to the brain and
cause stroke.
- NBME can mention recent visit to a chiropractor (neck manipulation is
Vertebral artery dissection
known cause).
- The answer on the NBME is heparin for patients who have experienced
posterior stroke due to vertebral artery dissection. Sounds weird because
it’s arterial, but it’s what USMLE wants. Take it up with them if you think
it’s weird.
- As discussed above in the aortic regurg section, USMLE loves this as most
common cause of AR due to retrograde propagation toward the aortic
root. For example, patient with Hx of HTN, cocaine use, or a connective
tissue disorder (i.e., Marfan, Ehlers-Danlos) who has a diastolic murmur,
you should be thinking immediately that this is dissection.
- “Medial necrosis” is a term that is used on NBME exams to describe
Aortic dissection changes to the aorta in dissection. In the past, “cystic medial necrosis”
used to be buzzy for dissection due to Marfan syndrome, but I haven’t
seen USMLE care about this. I have, however, seen a dissection Q on
NBME where it is due to hypertension, and simply “medial necrosis” is the
answer.
- As mentioned above, 3/4 Qs where BP is different between the arms
refers to aortic dissection. A Q on 2CK IM form 7 has “thoracic aortic

MEHLMANMEDICAL.COM 11
MEHLMANMEDICAL.COM

dissection” where not only is the BP different between the arms, but it’s
also different between the L and R legs (i.e., L-leg BP is different from R-leg
BP) à sometimes thoracic aortic dissections can anterograde propagate
all the way down to the abdominal aorta.

- You do not need to memorize these aortic aneurysm types. I’m just
showing you that if the common iliacs are involved (as with left image), BP
can differ as well between the legs.
- Caused by deceleration injury. Most common cause of death due to car
accident or fall. Exceedingly HY on 2CK.
- Will be described as patient following an MVA who has “widening of the
mediastinum.” They’ll then ask for the next best step à answer = aortic
angiography, aka aortography.
- Labetalol used first-line in patients who have aortic dissection and
Traumatic rupture
traumatic rupture of the aorta. Nitroprusside comes after.
of the aorta
- Labetalol is answer on NBME even in patient who has low BP due to
rupture or dissection due to the drug ¯ shearing forces. I’ve seen students
get this wrong saying, “But patient has low BP though.” My response is, file
a complaint with the exam not with me.
- 2CK Q gives “esmolol + nitroprusside” as answer to a traumatic rupture
Q, but almost always, they will just want “labetalol.”
- Can present as “visible pulsation” on USMLE.
- For aortic aneurysm, they can say “visible pulsation above the
manubrium,” or “pulsatile mass above the manubrium.” There can also be
a tracheal shift. I’ve seen students select pneumothorax here. But for
whatever reason you can get tracheal shift in thoracic aortic aneurysm. For
AAA, there can be “visible pulsation in the epigastrium.”
- Biggest risk factor for AAA is smoking.
- For Family Med, do a one-off abdominal ultrasound in both men and
women 65+ who are ever-smokers. This screening used to be just
performed on men, but now it includes women.
- For Surgery, AAA repair is indicated if the aneurysm is >5.5 cm or the rate
Aortic aneurysm of change of size increase is >0.5cm/month for 6 months. This is on
Surgery form, where they give a patient with a 4-cm AAA and ask why
serial ultrasounds are indicated à answer = “size of aneurysm.”
- In general, perioperative MI risk is assessed using a pre-op stress test.
2CK NBME Q has dipyridamole and thallium pharmacologic stress test as
answer in patient with 6-cm AAA prior to surgery.
- Diabetes is protective against aneurysm. Non-enzymatic glycosylation of
endothelium causes stiffening of the vascular wall.
- Don’t do AAA repair on USMLE in patient who has advanced
comorbidities or terminal disease, e.g., stage 4 lung cancer.
- Tangential: 2CK Surg loves “pulsatile hematoma” in the neck in trauma
patients, where the answer is “endotracheal intubation.” Sounds nitpicky,

MEHLMANMEDICAL.COM 12
MEHLMANMEDICAL.COM

but shows up repeatedly. I guess I just threw this random factoid here
because we’re talking about stuff that’s pulsatile LOL!
- Can be idiopathic, iatrogenic (i.e., dialysis), from injury (i.e., stab wound),
or caused by other disease (i.e., hereditary hemorrhagic telangiectasia or
Paget disease of bone).
- Similar to aortic aneurysms, AV fistulae can sometimes present with
pulsatile mass, but in a weird location, e.g., around the left ear in patient
with tinnitus (on NBME exam). Student says, “Why is it at the left ear
though?” à No fucking idea. Take it up with NBME.
- Highest yield point is they can cause high-output cardiac failure. This is
because blood quickly enters the venous circulation from the arterial
circulation à combo of ­ preload back to right heart + poorer arterial
perfusion distal to the fistula à compensatory ­ CO.
- AV fistulae can sometimes present with a continuous machinery murmur
similar to a PDA, since blood is continuously flowing through it. They
might say a continuous machinery-like murmur is auscultated in the leg at
site of prior stab wound.
- As discussed earlier, they can present with bounding pulses similar to AR.
- Student says, “Well how am I supposed to know if it’s AV fistula then if it
sounds like other conditions too?” à by paying attention to HY points like,
“Is there lone S3 or S3/4 combo or EF >70%? Is there Hx of penetrating
trauma? Or does the patient have Paget? Etc.”
- 2CK NBME Q shows you obscure angiogram of a fistula in the leg + tells
you there’s a continuous machinery murmur; they ask what most likely
determines prognosis in this patient à answer = “size of lesion.”

Arteriovenous fistula

- NBME exam shows obscure image similar to above (without the arrow) +
they tell you there’s continuous murmur à answer = “size of lesion.”
- Another NBME Q gives 45-year-old male will nosebleeds since
adolescence + S3 heart sound + dyspnea + they show you pic of tongue;
they ask for the cause of dyspnea.

MEHLMANMEDICAL.COM 13
MEHLMANMEDICAL.COM

- Answer = “Pulmonary arteriovenous fistula” (leading to high-output


failure); diagnosis is hereditary hemorrhagic telangiectasia. USMLE will
basically always show you a pic of red dots on the tongue/mouth or finger
in a patient with nosebleeds.
- Likewise, be aware intraosseous AV fistulae can occur in Paget, as
mentioned before.

HY Murmur / ECG points for USMLE


- Described as “irregularly irregular” rhythm.

- Notice how the QRS complexes are at random and irregular distances from
one another. This is the “irregularly irregular” pattern.
- AF is hugely important because it can cause turbulence/stasis within the left
atrium that leads to a LA mural thrombus formation. This thrombus can launch
off (i.e., become an embolus) and go to brain (stroke, TIA, retinal artery
occlusion), SMA/IMA (acute mesenteric ischemia), and legs (acute limb
ischemia). These vignettes are higher yield for 2CK, but the concept is important
for Step 1.
- AF HY in older patients, especially over 75. Vignette will usually be an older
patient with a stroke, TIA, or retinal artery occlusion, who has normal blood
pressure (this implies carotid stenosis is not the etiology for the embolus).
- AF usually is paroxysmal, which means it comes and goes. The vignette might
say the patient is 75 + had a TIA + BP normal + ECG shows sinus rhythm with no
abnormalities à next best step is Holter monitor (24-hour ambulatory ECG
monitor) to pick up the paroxysmal AF (e.g., when the patient goes home and
has dinner).
Atrial fibrillation (AF) - After AF is diagnosed with regular ECG or Holter, 2CK wants echocardiography
as the next best step to visualize the LA mural thrombus.
- Patient who has severe abdominal pain in setting of AF or hyperthyroidism
(which can cause AF), diagnosis is acute mesenteric ischemia; next best step is
mesenteric angiography; Tx is laparotomy if unstable (answer on NBME).
- Severe pain in a leg + absent pulses in patient with irregularly irregular rhythm
= acute limb ischemia; USMLE wants “embolectomy” as answer.
- Any structural abnormality of the heart, either due to LV hypertrophy,
ischemia, growth hormone/anabolic steroid use, prior MI, etc., can lead to AF.
- For 2CK, you need to know AF patient will get either aspirin or warfarin. This is
determined by the CHADS2 score. There are variations to the score, but the
simple CHADS2 suffices for USMLE à CHF, HTN, Age 75+, Diabetes,
Stroke/TIA/emboli. Each component is 1 point, but stroke/TIA/emboli is 2
points. If a patient has 0 or 1 points, give aspirin; if 2+ points, give warfarin. This
is important for 2CK.
- “Emboli” refers to Hx of AF leading to stroke, TIA, acute, mesenteric ischemia,
or acute limb ischemia – i.e., any Hx of embolic event. 2CK IM form 7 gives short
vignette of 67F with chronic AF + Hx of acute limb ischemia + no other info
relating to CHADS, and answer is warfarin to prevent recurrence; aspirin is
wrong.
- Some students will ask about NOACs, e.g., apixaban, etc., for non-valvular AF
à I’ve never seen NBME care about this stuff. They seem to be pretty old-
school and just have warfarin as the answer, probably because there isn’t
debate around whether it can be used; use of NOACs is less textbook.

MEHLMANMEDICAL.COM 14
MEHLMANMEDICAL.COM

- AF patient should also be on rate control or rhythm control. The USMLE


actually doesn’t give a fuck about this component of management, although in
theory metoprolol or verapamil is standard. You could be aware for Step 3 that
flecainide is first-line for rhythm control if patients fail rate-control and have a
structurally normal heart and no coronary artery disease.
- NBME 9 for 2CK has “electrical cardioversion” as the answer for patient with
AF who has hemodynamic instability (i.e., low BP). What you need to know is:
sometimes AF can trigger “rapid ventricular response,” where HR goes >150 and
low BP can occur.
- Has classic sawtooth appearance.

Atrial flutter

- Low yield for USMLE. I think it’s asked once on a 2CK NBME. But as student
you should know it exists / the basic ECG above.

- Causes wide-complex QRS complexes (>120 ms; normal is 80-120 ms).

- Exceedingly HY for 2CK that you know VT is wide-complex, whereas SVT is


narrow-complex. If you look at above ECG, even if you say, “No idea what I’m
looking at.” You can tell the complexes look wide like mountains in comparison
to a typical ECG.
- VT is treated with anti-arrhythmics, like amiodarone. If patient has coma or
Ventricular tachycardia hemodynamic instability (low BP), the NBME answer is direct current
(VT) countershock or cardioversion (same thing).
- Premature ventricular complex (PVC) is asked on 2CK.

- Note on the above strip, we have a wide complex (meaning ventricular in


origin) that occurs earlier (hence premature). What they do on the NBME is
show you this strip and ask where this abnormality originates from, then the
answer is just “ventricle.”
- Don’t treat PVCs on USMLE.
Supraventricular - Causes narrow / needle-shaped complexes. Make sure you’re able to contrast
tachycardia this with VT above, which is wide-complex.
(SVT)

MEHLMANMEDICAL.COM 15
MEHLMANMEDICAL.COM

- Notice the complexes are narrow / look like needles. This means the tachy
originates above the ventricles (hence SVT).
- Treatment of SVT exceedingly HY on 2CK.
- First step is carotid massage (aka vagal maneuvers). In pediatrics, they can do
ice pack to the face.
- If the above doesn’t work, the next step is give adenosine (not amiodarone).
- Same as with VT, if the patient has coma or low BP, shocking the patient is the
first step. In other words, for both SVT and VT, you must shock first in the
setting of coma or hemodynamic instability. It’s for stable SVT and VT that the
treatments differ on USMLE.
- Will present as ST-elevations in 3-4 contiguous leads.

Acute MI (STEMI)

- The above is an inferior MI, as evidenced by ST-elevations in leads II, III, and
aVF. The answer for the affected vessel is the posterior descending artery (PDA
supplies the diaphragmatic surface of the heart); since >85% of people have
right-dominant circulation (meaning the PDA comes of the right main coronary),
sometimes the answer for inferior MI can just be “right coronary artery.”
- If the Q says left-dominant circulation, the sequence USMLE wants is: left main
coronary à left circumflex à PDA.
- The apex of the heart is supplied by the left anterior descending artery (LAD).
If there are ST-elevations in leads V1-V3, choose LAD as the answer.
- The left-lateral heart is supplied by the left circumflex artery. If there are ST-
elevations in leads V4-V6 for lateral MI, choose left circumflex.
- Reciprocal ST-depressions in the anterior leads V1-V3 can reflex posterior wall
MI (i.e., we have “elevations” out the back of the heart, so they look like
depressions on the anterior wall leads).

MEHLMANMEDICAL.COM 16
MEHLMANMEDICAL.COM

- USMLE wants you to know gross appearance of fresh vs several-day-old


myocardial infarcts:

At 12-24 hours, fresh infarcts show dark mottling (green arrow); by 10-14 days,
an infarct becomes a yellow, softened area (pink arrow).

- Old infarcts will appear white. There is an NBME Q that shows image similar to
the following, where the answer is “congestive heart failure resulting from
repeated infarcts”:

Note the old infarcts are white in appearance (black arrows).

- Left ventricular free-wall rupture can cause tamponade (Beck triad):

MEHLMANMEDICAL.COM 17
MEHLMANMEDICAL.COM

Note the myocardial free-wall rupture at the lower left aspect.

- Ventricular septal rupture has similar image:

Note the thinned/ruptured area of the interventricular septum (white arrow).


- Post-MI papillary muscle rupture resulting in mitral regurg is exceedingly HY.
As discussed earlier, if patient has MI followed by new-onset systolic murmur
hours to days later, with or without dyspnea, that’s mitral regurg.
- Stroke-like presentation in patient who had MI weeks ago à “embolus from
ventricular septal aneurysm” (on 2CK Neuro form).
- Most common cause of death due to MI is ventricular fibrillation (VF).
- Fibrosis of myocardium in the months-years post-MI increases risk of
arrhythmias such as AF, SVT, VT, etc. There’s no specific arrhythmia you need to
memorize. Just know the risk is there in the future.
- Q waves on an ECG mean old MI / history of MI. The vignette might give you
patient who has light-headedness / fainting + they say patient has Q waves in II,
III, aVF, and the answer will be something like “paroxysmal supraventricular
tachycardia.” Student thinks this specific arrhythmia matters, but it doesn’t. The
point is that Hx of MI means patient is at risk for nearly any arrhythmia now.
- MI classically causes coagulative necrosis of the myocardium.
- With cardiogenic shock as a result of MI, the arrows USMLE wants are: ¯
cardiac output, ­ peripheral vascular resistance, ­ PCWP.
- MI can lead to acute tubular necrosis from cardiogenic shock à acute drop in
renal perfusion. This is not pre-renal. I discuss this in detail in the renal section.
- First treatment for MI is aspirin. After aspirin is given, the next drug to give is
clopidogrel (an ADP P2Y12 blocker) as dual anti-platelet therapy.
- USMLE wants you to know anyone with acute coronary syndrome (i.e., MI or
unstable angina) gets coronary catheterization. This is answer on new 2CK
NBME exam.

MEHLMANMEDICAL.COM 18
MEHLMANMEDICAL.COM

- It’s to my observation that more extensive management of MI on USMLE, such


as use of beta-blockers, nitrates, morphine, oxygen, statin, percutaneous
coronary intervention, etc., isn’t assessed in detail. I can comment, however,
that one 2CK Q wants you to know nitrates are contraindicated in right-heart
MIs, which includes inferior MI in most people due to the right coronary
supplying the PDA. This is because right-sided MIs are preload-dependent,
which means they need sufficient preload to maintain BP.
- Shows up on ECG as diffuse ST-elevations (i.e., in all leads rather than 3-4
contiguous leads as with MI). PR depressions can also be seen, but I’ve never
seen the USMLE give a fuck about the latter.
- Patient will have pain that’s worse when lying back, better when leaning
forward. In turn, the patient can present walking through the door bent over at
the waist.
- Serous pericarditis will be post-viral, secondary to autoimmune disease, or due
to cocaine use.
- Step 1 NBME Q gives pericarditis + a bunch of different organism types (i.e.,
bacterium, fungus, etc.), and answer is “virus.”
- Patient with rheumatoid arthritis or SLE notably at risk for pericarditis. In other
words, don’t get confused if they mention pericardial friction rub in vignette of
RA or SLE; this is common.
- For cocaine use, they’ll say a 22-year-old male has chest pain after a night of
Pericarditis
heavy partying + ECG shows diffuse ST-elevations à Dx = pericarditis.
- Uremic pericarditis is HY for 2CK. Q will give ultra-high creatinine and BUN and
say there’s a friction rub à treatment = hemodialysis.
- Treatment for pericarditis is same as acute gout à NSAIDs, colchicine,
steroids.
- Fibrinous pericarditis is post-MI and occurs as two types: 1) literally “post-MI
fibrinous pericarditis,” which will simply be friction rub within days of an MI; 2)
Dressler syndrome (antibody-mediated fibrinous pericarditis occurring 2-6
weeks post-MI).
- ECG is first step in Dx of pericarditis, but USMLE wants echocardiography as
next best step in order to visualize a concomitant effusion that can occur
sometimes. Vignette will give you stereotypical pericarditis + will ask for next
best step in diagnosis; ECG might not be listed and you’re like huh? à Answer is
echocardiography to look for potential effusion concomitant to the pericarditis.
- I should make note that chronic constrictive pericarditis is a separate condition
that doesn’t present with the standard pericarditis findings as described above.
- This is low-yield for USMLE, but students ask about it because it can be
confused with tamponade.
- There’s two ways this can show up:
1) Tuberculosis is a classic cause; there may or may not be calcification
around the heart on imaging. So if you get a Q where patient has TB +
some sort of heart-filling impairment à answer = chronic constrictive
pericarditis.
2) Kussmaul sign will be seen in the Q, where JVD occurs with
Chronic constrictive inspiration rather than expiration.
pericarditis - Normally, inspiration facilitates RA filling (¯ intrathoracic pressure à ­
pulmonary vascular compliance/stretching à ­ high-low pressure gradient
from right heart to the lungs à ¯ in afterload on RV from the lungs à blood
moves easier from right heart to the lungs à blood is pulled easier from
SVC/IVC to the RA).
- However, if there is ­ compressive force on the heart, the ­ in negative
intrathoracic pressure during inspiration is not transmitted to the right side of
the heart, so JVP does not ¯ (and can even paradoxically can ­).
- In tamponade, however, as discussed below, the ­ in negative intrathoracic
pressure during inspiration is able to be transmitted to the right side of the

MEHLMANMEDICAL.COM 19
MEHLMANMEDICAL.COM

heart, so Kussmaul sign does not occur. This is likely because in constrictive
pericarditis, the rigid pericardium prevents expansion of the right heart
altogether, whereas in tamponade, the pericardium isn’t rigid per se, but is just
filled with blood that can move/shift during the respiratory cycle, thereby
allowing right heart expansion during inspiration.
- Cardiac tamponade = pericardial effusion + low blood pressure.
- What determines whether we have a tamponade or not is the rate of
accumulation of the fluid, not the volume of the fluid – i.e., a stab wound or
post-MI LV free-wall rupture resulting in fast blood accumulation, even if
smaller volume, might cause tamponade, but cancer resulting in slow, but large,
accumulation might not cause tamponade.
- Tamponade presents as Beck triad: 1) hypotension, 2) JVD, 3) muffled/distant
heart sounds. The question will basically always give hypotension and JVD.
Occasionally they might not mention the heart sounds. But you need to
memorize Beck triad as HY for tamponade.
- Pulsus paradoxus (i.e., drop in systolic BP >10 mm Hg with inspiration) is
classically associated with tamponade, although not frequently mentioned in
vignettes. I’ve seen a 2CK NBME Q where they say “the pulsus paradoxus is <10
mm Hg,” which is their way of saying the Dx is not tamponade. I consider that
wording odd, but it’s what the vignette says.
- ECG will show electrical alternans / low-voltage QRS complexes.

Pericardial effusion /
Cardiac tamponade

- You can see the amplitudes (i.e., heights) of the complexes are short. This
refers to “low-voltage.” You can also see the heights every so slightly oscillate
up and down. This refers to electrical alternans. They show this ECG twice on
2CK NBMEs.
- USMLE wants ECG à echocardiography à pericardiocentesis or pericardial
window as the management sequence. Students often erroneously jump on
pericardiocentesis, but NBME wants echo first to confirm diagnosis. NBME 8
offline for 2CK has pericardial window as answer, where pericardiocentesis isn’t
listed.
- HY type of VT that has sinusoidal pattern on ECG.

Torsades de pointes
(TdP)

- USMLE wants you to know this can be caused by some anti-arrhythmic agents,
such as the sodium- and potassium-channel blockers, such as quinidine and
ibutilide, respectively. They ask this directly on the NBME exam, where Q will

MEHLMANMEDICAL.COM 20
MEHLMANMEDICAL.COM

say patient is given ibutilide + what is he now at increased risk of à answer =


torsades.
- QT prolongation is risk factor for development of TdP. Agents such as anti-
psychotics, macrolides, and metoclopramide prolong the QT.
- Tx USMLE wants is magnesium (asked directly on new 2CK form), which
stabilizes the myocardium in TdP.
- Seen in hyperkalemia.

Peaked T wave

- Asked once on one of the 2CK forms, where they show the ECG.
- Highest yield point is that if a patient has hyperkalemia and ECG changes, the
Tx USMLE wants is IV calcium gluconate or calcium chloride, which stabilizes the
myocardium. Calcium gluconate is classic, but calcium chloride shows up as an
answer on a 2CK NBME.
- Means hypokalemia.

U-wave

Shows up on NBME 12 for 2CK in anorexia patient. First time I’ve ever seen it
show up anywhere on NBME material. But Q doesn’t ride on you knowing it
means hypokalemia to get it right. It’s HY and pass-level to know that purging
(anorexia or bulimia) causes hypokalemia anyway.
- Seen in Wolff-Parkinson-White syndrome (WPW; accessory conduction
pathway in heart that bypasses the AV node, resulting in reentrant SVT).
- Classically described as a “slurred upstroke” of the QRS, where the PR interval
is shortened.

Delta wave

- Both the delta-wave and WPW have basically nonexistent yieldness on USMLE,
but I mention them here so you are minimally aware.
- They mean hypothermia. You don’t need to be able to identify on ECG. Just
J waves know they exist, as they show up in a 2CK vignette where patient has body
temperature of 89.6 F (not 98.6).

MEHLMANMEDICAL.COM 21
MEHLMANMEDICAL.COM

HY Heart block points for USMLE


- Prolonged PR interval (>200 ms). Should normally be 80-120 ms.

First degree

- Note that above on the ECG, the PR-segment in particular (just prior to
the QRS complex) is extra-long.
- Not really assessed on USMLE. Just know the definition.
- Don’t treat on USMLE.
- Gradually prolonging PR interval until QRS drops. Then cycle repeats.

Second degree Mobitz type I


(aka Wenckebach)

- Don’t treat on USMLE.


- No gradual prolongation of PR interval, followed by a random dropping
of the QRS.

- Can also sometimes occur as patterns of 2:1, 3:1, etc., where there will
be a P to QRS ratio of 2:1 or 3:1, etc.

Second degree Mobitz type II

- Regardless as to whether the dropped QRS is random or in a numerical


pattern, there is no gradual prolongation of the QRS before the dropped
complex.
- More dangerous than Mobitz I. This is because Mobitz II has higher
chance of progression into type III heart block.
- Treatment on USMLE is insertion of pacemaker. This is asked on a new
2CK NBME exam.
- Two things you want to look for on ECG:
- 1) Ultra-slow HR (i.e., 30-40). You’ll see the QRS’s are super far apart.
This is the ventricular escape rhythm.
- 2) No relationship between the P-waves and QRS complexes.

Third degree

- Treatment on USMLE is insertion of pacemaker.

MEHLMANMEDICAL.COM 22
MEHLMANMEDICAL.COM

- So what you want to remember is that Mobitz II and 3rd-degree are the
ones where we insert pacemaker; 1st-degree and Mobitz I we don’t.

HY Cardiomyopathy points for USMLE


- Can be isolated ventricular or diffuse 4-chamber dilation. Causes are
multifarious, but a key feature is systolic dysfunction, where ejection
fraction is reduced (i.e., <55%, where normal range is 55-70).
- CXR shows enlarged cardiac silhouette.

Dilated (DCM) - An S3 heart sound can sometimes be heard.


- Cardiac exam shows lateralized apex beat. Can be described as the
point of maximal impulse being in the anterior axillary line. Should be
noted that this lateralization just means an enlarged LV, so it is non-
specific, and can also be seen in LV hypertrophy from any cause. But
many vignettes will mention it.
- The arrows USMLE wants are: ¯ EF; ­ LVEDV; ­ LVEDP. I discuss this
stuff in more detail in my HY Arrows PDF.
- Causes of DCM are ABCD:
- A: Alcohol.
- B: Wet Beriberi (thiamine deficiency).
- C: Coxsackie B virus, Cocaine, Chagas disease.
- D: Drugs à doxorubicin (aka Adriamycin).
- Other notable causes are:
- Pregnancy (peripartum cardiomyopathy); hemochromatosis; rheumatic
heart disease (myocarditis leading to DCM).
- Heart failure due to systemic hypertension.
- Characterized by diastolic dysfunction (the heart can pump just fine but
cannot expand as easily).
- Can be associated with S4 heart sound.
- The arrows USMLE wants are: « EF; « LVEDV; ­ LVEDP.
- Ejection fraction is normal because the heart can pump perfectly fine.
- Students get confused about LVEDV, thinking it should be low, if the
heart cannot expand as easily. But this is not the case for USMLE. They
want you to know normal volume can be achieved; it just merely requires
Hypertrophic (HCM) more force/pressure to get there.
- As with DCM, the apex beat / point of maximal impulse can be
lateralized, which merely reflects LV hypertrophy.
- As described earlier, paradoxical splitting of S2, left-axis deviation on
ECG, and LBBB can all be seen due to LVH. When you see these findings
in vignettes, don’t get confused. They just mean LVH.
- Often associated with hypertensive retinopathy (fundoscopy shows
narrowing of retinal vessels, flame hemorrhages, and “AV-nicking”),
hypertensive nephropathy (hyperplastic arteriolosclerosis with increased
creatinine).

MEHLMANMEDICAL.COM 23
MEHLMANMEDICAL.COM

- Caused by mutations in b-myosin heavy-chain gene; autosomal


dominant; results in disordered/disarrayed myocardial fibers.
- HOCM causes asymmetric septal hypertrophy that results in the
anterior mitral valve leaflet obstructing the LV outflow tract (so it can
sound similar to aortic stenosis).
- Classically sudden death in young athlete; cause of death is ventricular
fibrillation due to acute left heart strain in the setting of fast heart rate.
- Presents with systolic murmur that worsens with Valsalva or standing
(aortic stenosis, in contrast, gets softer or experiences no change with
Valsalva) or standing.
Hypertrophic obstructive
- HOCM and MVP are the only two murmurs that get worse with less
(HOCM)
volume in the heart. All other murmurs get worse with more volume.
Valsalva increases intra-thoracic pressure and decreases venous return,
so there’s less volume in the heart. Standing simply decreases venous
return.
- NBME wants you to know that ­ HR from exercise or stress means
“diastole is shortened more than systole.”
- Beta-blockers (metoprolol or propranolol are both answers on NBME)
are given to slow heart rate, which maximizes diastolic filling and
decreasing symptoms / risk of death.

- Heart failure due to diastolic dysfunction, where HTN is not the cause.
- JVD is HY for RCM. An S4 can also be seen. The heart will not be dilated.
- HY causes are Hx of radiation (leads to fibrosis), amyloidosis, and
hemochromatosis.
- Student might say, “I thought you said hemochromatosis was DCM. So
if we have to choose on the exam, which one is it?” The answer is,
whichever the vignette gives you. If they say a large cardiac silhouette
with an S3 and lateralized apex beat, that’s DCM. If they say JVD + S4 +
nothing about a lateralized apex beat, you know it’s RCM.
- Amyloidosis is protein depositing where it shouldn’t be depositing.
Highest yield cause of amyloidosis on USMLE is multiple myeloma, which
will lead to RCM.

Restrictive (RCM)

Cardiac amyloidosis.
Myocardium is pink; amyloid is white.

- Since RCM is diastolic dysfunction, the arrows are the same as HCM,
which are: « EF; « LVEDV; ­ LVEDP.

MEHLMANMEDICAL.COM 24
MEHLMANMEDICAL.COM

Important points about atherosclerosis for USMLE


- Endothelial cell damage / dysfunction (e.g., from HTN or smoking) à monocytes leave
blood and accumulate beneath internal elastic lamina of artery à monocytes within
tunica media of artery are now called macrophages à phagocytosis of oxidized LDL
particles à macrophages are now called foam cells à accumulation of foam cells leads to
fatty streak/plaque à stimulates vascular smooth muscle cell migration/proliferation
within plaque à cap thinning + rupture à myocardial infarction, etc.

Mechanism

- Most acceleratory risk factors are diabetes mellitus (I and II), followed by smoking,
followed by HTN, in that order.
- HTN is most common risk factor, but DM and smoking are worse. I talk a lot about this
stuff in my HY Risk Factors PDF if you want extensive detail.
- HTN is most acceleratory specifically for carotid stenosis (systolic impulse pounds
carotids à endothelial damage).
- Stroke, TIA, or retinal artery occlusion in patient with high BP is due to carotid plaque
launching off to the brain/eye. If patient has normal BP, think AF instead, with left atrial
mural thrombus launching off.
- Patient over 50 with Hx of cardiovascular risk factors who now has accelerated HTN,
HY points
think renal artery stenosis (narrowing due to atherosclerosis).
- Plaques can calcify. The more calcium there is in a plaque, the more mature it is often
considered to be. Calcium scoring is routinely done in patients who have coronary artery
disease in the assessment of plaque progression.
- Statins have 2 HY MOAs on USMLE: 1) inhibit HMG-CoA reductase; 2) upregulate LDL
receptors on hepatocytes.
- Ezetimibe blocks cholesterol absorption in the small bowel.
- Bile acid sequestrants (e.g., cholestyramine) result in the liver pulling more cholesterol
out of the blood.
- Fibrates upregulate PPAR-a and lipoprotein lipase; best drugs to decrease triglycerides.

HY Angina points for USMLE


- Chest pain that occurs predictably with exercise.
- Due to atherosclerotic plaques causing >70% occlusion; can be calcific.
- Classically causes ST depressions on ECG.
- Nitrates (e.g., sublingual isosorbide dinitrate) used as Tx à nitrates
Stable angina
“donate” nitric oxide (NO) that upregulates guanylyl cyclase within venous
smooth muscle à increased cGMP à relaxation of venous smooth muscle
à increased venous pooling of blood à decreased venous return à
decreased myocardial oxygen demand à mitigation of chest pain.

MEHLMANMEDICAL.COM 25
MEHLMANMEDICAL.COM

- Nitrates are contraindicated with PDE-5 inhibitors (e.g., Viagra) due to


risk of low blood pressure.
- Sodium nitroprusside used for hypertensive emergencies dilates
arterioles in addition to the veins. If USMLE asks you where this drug acts,
choose arterioles.
- Chest pain that is unpredictable and can occur at rest.
- Due to partial rupture of atherosclerotic plaque leading to partial
occlusion.
Unstable angina
- ST depressions on ECG.
- Diltiazem is answer on new 2CK NBME for patient with unstable angina.
- Patients need cardiac catheterization.
- Vasospastic angina that occurs at rest (i.e., watching TV or while sleeping)
in younger adults; it is not caused by atherosclerosis.
- ST elevations are seen on ECG.
- You must know that Prinzmetal is also known as variant angina pectoris.
There is an NBME Q that gives vignette of Prinzmetal, but answer is
Prinzmetal angina
“variant angina pectoris.”
(variant angina pectoris)
- Treatment is nitrates (can cause coronary artery dilation unrelated to the
venous pooling effects) or dihydropyridine calcium channel blockers (e.g.,
nifedipine). Avoid a1-agonists in these patients (cause vasoconstriction),
as well as non-selective b-blockers like propranolol (can cause unopposed
a effects).

HY histopath / cardiac marker points for MI


- Coagulative necrosis occurs within the first day of the MI. USMLE wants you to
know cellular architecture is maintained in coagulative necrosis.
Histopath - Neutrophils will infiltrate at 1-3 days.
- Macrophages will infiltrate at 3-14 days.
- Scar formation occurs from ~2 weeks onward.
- Percutaneous coronary intervention used to open blocked coronary artery can
cause reperfusion injury, where fast influx of oxygen to tissue results in high density
of oxygen free radicals and peroxidation of lipid membranes à further myocardial
HY random points
necrosis. But this outcome is superior to non-intervention.
- Adenosine is molecule that causes cardiac pain in MI and angina. It is main
autoregulator of coronary blood flow. It is increased in MI due to exhaustion of ATP.
- USMLE wants you to know during acute MI, cardiac markers are not initially
elevated. We rely on the ECG to show us ST elevations.
- Troponin rises the most and stays elevated the longest.
- CK-MB is the answer if they ask about recurrent MI. This is because its shorter
duration of elevation makes it useful to discern a new cardiac event has occurred.

Cardiac markers

MEHLMANMEDICAL.COM 26
MEHLMANMEDICAL.COM

HY Endocarditis points
- Bacterial infection of valve in patient with no previous heart valve problem.
- Caused by Staph aureus on USMLE.
- Left-sided valves (i.e., aortic and mitral) most commonly affected because of
greater pressure changes (i.e., from high to low) within left heart, resulting in
Acute endocarditis
turbulence that enables seeding.
- IV drug users à venous blood inoculated with S. aureus à travels to heart and
causes vegetation of tricuspid valve.
- Staph aureus is coagulase positive.
- Bacterial infection of valve in patient with history of valve abnormality (i.e.,
congenital bicuspid aortic valve, Hx of rheumatic heart disease).
- Caused by Strep viridans on USMLE. You need to know S. viridans is can be
further broken down into: S. sanguinis, S. mutans, and S. mitis.
- Hx of dental procedure is HY precipitating event, where inoculation of blood
Subacute endocarditis
occurs via oral cavity à previously abnormal valve gets seeded.
- Carbohydrate limit dextrins produced by the Strep enable colonization of the
abnormal valve.
- S. viridans are alpha-hemolytic (i.e., demonstrate partial hemolysis, causing a
green zone of hemolysis on blood agar).
- New-onset murmur + fever = endocarditis till proven otherwise on USMLE.
- Reactive thrombocytosis (i.e., high platelets) can occur due to infection. This is
not unique to endocarditis, but it is to my observation USMLE likes endocarditis
as a notable etiology for it. In other words, if you get an endocarditis question
and you’re like, “Why the fuck are platelets 900,000?” (NR 150-450,000), don’t
be confused.
Random points
- Hematuria can occur from vegetations that launch off to the kidney.
- Endocarditis + stroke-like episode (i.e., focal neurologic signs) = septic
embolus, where a vegetation has launched off to the brain.
- Janeway lesions, Osler nodes, splinter hemorrhages, etc., are low-yield for
USMLE and mainly just school of medicine talking points.
- HACEK organisms nonexistent on USMLE.
- Blood cultures before antibiotics is important for 2CK.
- Transesophageal echocardiography (TEE) confirms diagnosis after blood
cultures. Transthoracic echocardiography (TTE) is not done for endocarditis.
- For 2CK, empiric treatment for endocarditis is vancomycin, PLUS either
gentamicin or ampicillin/sulbactam.
- Vancomycin targets gram-positives (including MRSA). Gentamicin targets
gram-negatives.
- Endocarditis prophylaxis given prior to a dental procedure is usually ampicillin
or a second-generation cephalosporin, such as cefoxitin.
Management - Indications for endocarditis prophylaxis are:
1) Hx of endocarditis (obvious);
2) If there is any prosthetic material in the heart whatsoever;
3) If there is any congenital cyanotic heart disease that has not been completely
repaired (if it’s been completely repaired with prosthetics, give prophylaxis);
4) Hx of heart transplant with valvular regurgitation of any kind.
- Highest yield point for USMLE about endocarditis prophylaxis is that mitral
valve prolapse (MVP) and valve regurgitations or stenoses are not an indication.
In other words, do not give prophylaxis if the patient has MVP, MR, AS, etc. In
addition, bicuspid aortic valve is not an indication.

Rheumatic heart disease (rheumatic fever) HY points


- Strep pyogenes (Group A Strep) oropharyngeal infection results in production of antibodies against S.
pyogenes’ M-protein that cross-react with the mitral valve (i.e., molecular mimicry; type II hypersensitivity).
- Can occur with the aortic valve in theory, but on USMLE, it is always mitral valve.

MEHLMANMEDICAL.COM 27
MEHLMANMEDICAL.COM

- Results in mitral regurgitation acutely and mitral stenosis late, as discussed earlier.
- Presents as JONES (J©NES) à Joints (polyarthritis), © Carditis, subcutaneous Nodules, Erythema
marginatum (annular, serpent-like rash), Sydenham chorea (autoimmune basal ganglia dysfunction that
results in dance-like movements of the limbs).
- Cutaneous Group A Strep infections don’t cause rheumatic fever, but can still cause PSGN.
- Treatment is penicillin.

Conditions confused for cardiac path


- Psych forms love trying to make you think this is an MI.
- They’ll give you young, healthy patient who feels doom / like he or she is
going to die.
- Sometimes they mention in stem Hx of MI in family as distraction.
Panic attack - They can say patient has mid-systolic click, as discussed earlier, and then
they ask for cause of patient’s symptoms à answer = panic disorder, not
MVP. Student gets confused, but MVP is almost always asymptomatic, where
panic attack is clearly cause of the patient effusively hyperventilating.
- Treat with benzo.
- Orthostatic hypotension is defined as intravascular fluid depletion causing a
drop of systolic BP >20 mmHg and diastolic BP >10 mmHg when going from
supine to standing.
Orthostasis
- Shows up on 2CK IM form as exactly a drop of 20 and 10, respectively, for
systolic and diastolic BPs in a patient with fainting à answer = “intravascular
fluid depletion.”
- Fainting in response to stressor (e.g., emotional trigger).
- Stress triggers an initial sympathetic response, which in turn triggers a
compensatory parasympathetic response. This latter response is excessive in
Vasovagal syncope some people, where the peripheral arterioles dilate and the heart slows too
much à decreased cerebral perfusion à lightheadedness/fainting.
- 2CK wants you to know a tilt-table test can be used to diagnose, where a
reproduction of symptoms can occur.
- USMLE likes this for both Steps 1 and 2.
- They’ll say dude was shaving then got lightheadedness or fainted.
Carotid sinus
Mechanism is ­ stretch of carotid sinus baroreceptors à ­ afferent CN IX
hypersensitivity
firing to solitary nucleus of the medulla à ­ efferent CN X parasympathetic
firing down to cardiac nodal tissue à ¯ HR à ¯ CO à ¯ cerebral perfusion.
- Not cardiac. This is MSK. But I’m putting this here real quick because
students often confuse this for cardio.
- Inflammation of cartilage at rib joints.
Costochondritis - Will present as chest pain that worsens with palpation or when patient
reaches over the head or behind the back. These two findings are clear
indicators we have an MSK condition, not cardiac.
- Can be idiopathic, caused by strain (e.g., at the gym), or even post-viral.
- MSK condition asked twice on 2CK material (once on FM form; also on Free
120) that has nothing to do with the lungs, despite the name.
- This is viral infection (Coxsackie B) causing sharp lateral chest pain due to
Pleurodynia
intercostal muscle spasm. Sometimes students choose pericarditis, etc., even
though the presentations are completely disparate.
- Creatine kinase can be elevated in stem due to ­ tone of muscle.
- I discuss this in detail in gastro section later.
Diffuse esophageal spasm
- Can cause angina-like pain in patient without cardiovascular disease.
- I discuss GERD in detail in the gastro section later.
Gastroesophageal reflux
- Can present as chest pain confused for MI. ECG will be normal, clearly.

MEHLMANMEDICAL.COM 28
MEHLMANMEDICAL.COM

Arterial vs venous disease


- Caused by atherosclerotic disease; presents as diminished peripheral pulses in
patient over 50 who has risk factors, e.g., diabetes, smoking, HTN.
- Lower legs can be shiny and glabrous (trophic changes).
- Arterial ulcers are small and punched-out; located on tops/bottoms of feet and
toes.

Arterial disease

- Ankle-brachial indices (ABIs) are first step in diagnosis (exceedingly HY on 2CK),


which compare BP in ankle to the arm; if <0.9, this reflects ¯ peripheral blood
flow due to atherosclerosis.
- If ABIs are not listed as first step in diagnosis for whatever reason, choose
Doppler ultrasound. There is one 2CK NBME Q where this is the case.
- After ABIs, next step is exercise stress test (if listed) in order to determine
exercise tolerance. If not listed, go straight to “recommend an exercise /
walking program.” Do not choose cilostazol first or arteriography as answers.
- 2CK CMS Surg form 5 has “prescription for an exercise program” as the
answer. Students say, “Why does it say ‘prescription’?” No fucking idea. Ok?
- Congestion of venous system usually from valvular incompetence; idiopathic /
familial; varicose veins are one type of venous disease and are not synonymous;
patients can have venous disease without varicosities.
- Peripheral pulses are normal (those reflect arteries, not veins).
- Lower legs demonstrate “brawny edema,” which is a brown, hemosiderin-
laden edema due to ­ pressure / micro-extravasations; hyperpigmentatory
changes resulting in brown/red skin is known as stasis dermatitis, aka post-
phlebitic syndrome; the latter is a term is asked on 2CK, so know the annoying
vocab.
- Venous ulcers are large and sloughy, and located at the malleoli.

Venous disease

- Diagnose with venous duplex ultrasonography of the legs; first treatment is


compression stockings. Never choose answers such as venous stripping or glue
agents, etc.
- Venous disease ­ risk for DVT and superficial thrombophlebitis. If patient has
active DVT or STP, answer = subcutaneous enoxaparin (heparin) over
compression stockings.

MEHLMANMEDICAL.COM 29
MEHLMANMEDICAL.COM

Cardiac stress test points (2CK only)


- Most 2CK Qs that ask about stress tests are in the context of evaluating patients for perioperative MI risk.
- It is rare the Q will force you to choose between different types of stress tests. 4/5 Qs will just list one
stress test, where it is simply assessing, “Do you know a stress test should be done, period, in this scenario.”
- Stress tests are also done for peripheral arterial disease prior to recommending an exercise/walking
program (as mentioned above).
- Most common stress test.
- The answer on USMLE for patients who have stable angina, where you’re looking for ST
depressions (i.e., evidence of ischemia) with exertion.
- Requires a patient has a normal baseline ECG in order to perform.
Exercise ECG - In other words, the Q will give you a big 15-line paragraph + mention in the last line that
the patient’s baseline ECG shows, e.g., a LBBB from a year ago that’s unchanged. This
means ECG stress test is wrong in this situation, since you need to have a normal ECG to
do it. The 1/5 Qs that force you to choose between stress tests want you to know this
detail, basically, where you just choose the non-ECG stress test instead.
- Used to look for heart failure (i.e., ¯ EF) with exertion, not overt ischemia.
- In other words, the answer on USMLE for patients who don’t get chest pain with exertion
Exercise echo (i.e., don’t have stable angina), but who get shortness of breath with exertion. This
reflects, at a minimum, left heart decompensation with possible ¯ EF.
- Also the answer for patients who have abnormal baseline ECG.
- Refers to numerous answer choices on USMLE – i.e., dobutamine-echo, dipyridamole-
thallium.
- The answer on USMLE for patients who cannot exercise, such as in the setting of angina
when merely walking up a single flight of steps, or in patients imminently undergoing
major surgery (e.g., AAA repair), where perioperative MI risk needs to be assessed. I have
seen both of these scenarios on 2CK forms.
- The USMLE will typically not force you to choose between stress tests. As I mentioned at
Pharmacologic
the top of this table, they will usually just have the pharmacologic stress test as the only
one listed.
- Dobutamine is a b1-agonist that stimulates the heart (i.e., ­ oxygen demand). Echo can
then be done to look for ¯ EF (i.e., heart failure).
- Dipyridamole is a phosphodiesterase inhibitor that dilates arterioles. HR goes up to
compensate, thereby ­ myocardial oxygen demand. Thallium is then used to look at
perfusion of the myocardium.
- “Cardiac scintigraphy” is a broad term that refers to any evaluation of the heart in which
some form of radiotracer is used (i.e., thallium, technetium, sestamibi).
- This is the same as pharmacologic stress test for all intents and purposes on USMLE, even
Cardiac
though technically it need not require myocardium is stimulated and can just be used to
scintigraphy
look at blood flow to the heart in the resting state.
- The point is: This is an answer on 2CK sometimes as just another way of them writing
“pharmacologic stress test.” Choose it if the patient cannot exercise.
- “Myocardial perfusion scan” is one type of cardiac scintigraphy that evaluates blood flow
to myocardium. It is non-invasive, whereas coronary angiography is invasive and evaluates
Myocardial
coronary blood flow via the use of a catheter.
perfusion scan
- This is interchangeable with cardiac scintigraphy and pharmacologic stress test on USMLE
for all intents and purposes.

Knee trauma causing popliteal arterial injury


- If Q gives you MVA where the knee is injured + absent pulses distally, the sequence of
Knee dislocation answers they want is knee relocation first, followed by arteriography to look for
popliteal artery injury.

MEHLMANMEDICAL.COM 30
MEHLMANMEDICAL.COM

- In one of the 2CK Qs, they already tell you the knee is relocated, then the answer is
“arteriography with runoff.” Students say, “what’s the runoff part?” No fucking idea,
it’s just what they want.
- Doesn’t sound like a big deal, but there are 2CK NBME Qs on this, where they want
that order.
- If Q gives gunshot wound to the knee + absent distal pulses, go straight to “surgical
Penetrating exploration” as the answer.
trauma - This could be thought of as the knee-equivalent of a gunshot wound to the abdomen,
where straight to laparotomy (even if patient is stable) is the answer.

HY Vasculitides (fancy word that’s pleural for vasculitis)


- Formerly known as Wegener granulomatosis.
- Answer on USMLE for adult with triad of 1) hematuria, 2) hemoptysis, and 3)
“head-itis” – i.e., any problem with the head, such as nasal septal perforation,
Granulomatosis with
mastoiditis, sinusitis, otitis.
polyangiitis
- Associated with cANCA and anti-proteinase 3 (anti-PR3) antibodies.
- Causes “necrotizing glomerulonephritis” that can lead to rapidly progressive
glomerulonephritis (RPGN).
- Formerly known as Churg-Strauss.
Eosinophilic - Presents as combo of asthma + eosinophilia +/- head-itis.
granulomatosis with - Head-itis always seen in Wegener vignettes, but maybe only ~50% of CS Qs.
polyangiitis - Renal involvement rare for CS.
- Associated with pANCA and anti-myeloperoxidase (anti-MPO) antibodies.
- Will just present as hematuria in a patient who is pANCA / anti-MPO (+).
Microscopic polyangiitis
- Similar to Wegener, can cause RPGN.
- The above three conditions can be associated with a weird neuropathy called mononeuritis multiplex,
which means neuropathy of “one large nerve in many location” – e.g., wrist drop + foot drop in same
patient.
- You don’t need to worry about the fancy term “mononeuritis multiplex,” but what I do want you to be
aware of is that neuropathy will sometimes show up in these vasculitis vignettes, so don’t be confused
about it.
- The answer on USMLE for a male 20s-40s with hematuria and hemoptysis.
Similar to Wegener and microscopic polyangiitis, can cause RPGN.
- Caused by anti-glomerular basement membrane antibodies (anti-GBM),
which are antibodies against collagen IV.
- Renal biopsy will show linear immunofluorescence. USMLE likes showing the
bright green image for this.

Goodpasture syndrome

- Do not confuse this with Alport syndrome, which is an XR condition caused


by mutations in collagen IV.
- Medium-vessel vasculitis that causes a “string of pearls” appearance of the
Polyarteritis nodosa (PAN) renal vessels.

MEHLMANMEDICAL.COM 31
MEHLMANMEDICAL.COM

- Causes fibrinoid necrosis, which means it looks like fibrin but it ain’t fibrin.
- Offline Step 1 NBME has “segmental ischemic necrosis” as the answer.
- Can be caused by hepatitis B.
- For whatever reason, USMLE wants you to know PAN spares the lungs – i.e.,
it does not affect the pulmonary vessels.
- Aka “pulseless disease.” Classically affects Asian women 40s or younger.
- Inflammation of large vessels, including the aorta.
Takayasu arteritis
- Always affects the subclavian arteries (which supply the arms), which is why
it can cause weakly, or non-palpable, pulse in the upper extremities.
- Aka giant cell arteritis.
- 9/10 Qs will be painful unilateral headache in patient over 50. I’ve seen one
Q on NBME where it’s bilateral.
- Flares can be associated with low-grade fever and high ESR.
- Patients can get proximal muscle pain and stiffness. This is polymyalgia
rheumatica (PMR). The two do not always go together, but the association is
HY. (Do not confuse PMR with polymyositis. The latter will present with ­ CK
and/or proximal muscle weakness on physical exam. PMR won’t have either of
these findings. I talk about this stuff in detail my MSK notes.)
- Patients can get pain with chewing. This is jaw claudication (pain with
chewing).
- Highest yield point is we give steroids before biopsy in order to prevent
Temporal arteritis
blindness.
- An NBME has “ischemic optic neuropathy” as the answer for what
complication we’re trying to prevent by giving steroids in temporal arteritis.
- IV methylprednisolone is typically the steroid given, since it’s faster than oral
prednisone.
- It’s to my observation many 2CK NBME Qs will give the answer as something
like, “Steroids now and then biopsy within 3 days,” or “IV methylprednisolone
and biopsy within a week.” Students ask about the time frames, but for
whatever reason USMLE will give scattered/varied answers like that.
- Another 2CK Neuro CMS Q gives easy vignette of temporal arteritis and then
asks next best step in diagnosis à answer = biopsy. Steroids aren’t part of the
answer. Makes sense, since they’re asking for a diagnostic step.
- Aka Buerger disease; technically a vasculitis.
- Dry gangrene of the fingers or toes seen generally in male over 30 who’s a
Thromboangiitis obliterans heavy smoker.
- Treatment is smoking cessation.
- Don’t confuse with Berger disease, which is IgA nephropathy.
- Tertiary syphilis can cause ascending aortitis + aortic aneurysm.
Ascending aortitis
- Causes “tree-barking” of the aorta.

MEHLMANMEDICAL.COM 32
MEHLMANMEDICAL.COM

Thrombophlebitis
- DVT will be unilateral thigh or lower leg swelling in patient with risk factors
such as: post-surgery, prolonged sedentation, OCP use, Hx of thrombotic
disorders (e.g., Factor V Leidin, prothrombin mutation).
- Virchow triad for ­ DVT risk: 1) venous stasis (e.g., post-surgery sedentation),
2) hypercoagulable state (e.g., estrogen use, underlying malignancy), 3)
endothelial damage (i.e., smoking).
- OCPs contraindicated in smokers over 35 because estrogen causes
hypercoagulable state for two reasons: 1) estrogen upregulates fibrinogen; 2)
estrogen upregulates factors Va and VIIIa.
- USMLE loves nephrotic syndrome as cause of DVT (loss of antithrombin III in
the urine à hypercoagulable state).
- Antiphospholipid syndrome à DVTs despite paradoxical ­ PTT (i.e., if PTT is
high, you’d think you have bleeding diathesis, not thromboses); may or may
not be due to SLE. Antibodies against phospholipids cause in vivo clumping of
platelets + ­ clot initiation, but disruption of in vitro PTT assay means ­ PTT.
- Major danger is DVT can embolize to lungs causing PE à acute-onset
shortness of breath and tachycardia + death if saddle embolus.
Deep vein thrombosis - Homan sign can mean DVT, which is pain in the calf with dorsiflexion of foot.
- Diagnose DVT with duplex venous ultrasound of the leg/calf.
- Treatment is heparin.
- Harder surgery stuff for 2CK is that they care about prophylactic vs
therapeutic doses of heparin. Prophylactic dose is lower-dose and is used
perioperatively in patients with venous disease/stasis or who are high risk. If a
patient has an actual full-blown DVT, however, give therapeutic dose, which is
higher-dose.
- There are two 2CK Qs on this stuff. One just mentions a guy going into
surgery who has Hx of venous stasis à answer = “prophylactic heparin dose”;
“therapeutic heparin dose” is wrong answer.
- The second question gives a guy who’s already on prophylactic heparin but
gets a DVT anyway. The answer is then “heparin.” It’s weird because students
are like, “Wait what? He’s already on heparin though.” And I’m like, yeah, but
what they mean is, we have to give therapeutic dose now for the active DVT,
which is higher dose.
- DVT can rarely cause stroke if an ASD is present (paradoxical embolus).
Dumb and low-yield, but it shows up, and students get fanatical over it.
- Thrombophlebitis means inflammation of a vein.
- Post-surgery, this is usually due to changes in hemostasis and coagulability.
Post-op migratory
- Will present as pink/red painful lesions appearing asymmetrically on the
limbs within days of surgery. You just need to be able to diagnose this.
- Migratory thrombophlebitis classically due to head of pancreas
Trousseau sign of
adenocarcinoma. But this can also be seen with adenocarcinomas in general,
malignancy
e.g., pulmonary.
- Shows up on 2CK Surg form as patient who had a catheter in and then
Catheter-associated septic develops a 4-cm indurated, painful, fluctuant cord in his arm (refers to vein).
thrombophlebitis (CAST) - Answer = “excision of vein.” Obscure question, but not my opinion. Take it
up with NBME if you think it’s weird.
- Important for 2CK Obgyn Qs.
- The answer on USMLE in a woman who has post-partum endometritis (fever
+ tender lower abdomen) with persistent fever >48 hours despite antibiotics.”
Pelvic septic - Endometritis can lead to ­ risk of local infective clots in the ovarian veins.
thrombophlebitis (PST) - If they give you a post-partum woman with sepsis (i.e., SIRS + infection), but
the vignette doesn’t fit PST as described above, the answer is “puerperal
sepsis” on the Obgyn form. The latter is a more general term and can refer to
many causes of post-partum sepsis (including PST confusingly enough).

MEHLMANMEDICAL.COM 33
MEHLMANMEDICAL.COM

- Painful palpable cord in the ankle that may or may not track up to the knee.
- Seen in patients with venous insufficiency.
- Answer is “subcutaneous enoxaparin.” Compression stockings are typically
Superficial
the answer for first step in venous insufficiency, but if you have an active ST or
thrombophlebitis
DVT, heparin must be given as first step.
- There will occasionally be some intentional redundancy on my end with
things I write in this doc if I believe they’re HY enough (as with this).

HY familial dyslipidemias for USMLE


Condition Mechanism HY point(s)
- ­ chylomicrons + TGAs.
Hyperchylomicronemia Deficiency of lipoprotein lipase - Pancreatitis (abdo, not chest, pain).
(AR) or Apolipoprotein C-II - Xanthomas.
- Plasma appears “creamy”.
- ­ LDL.
- LDL usually 3-400 in heterozygotes, with
MI/death in 30-40s (answer = deficiency
Deficiency (heterozygous) or
Hypercholesterolemia of functional LDL receptor).
absence (homozygous) of LDL
(AD) - LDL usually 700-1000 in homozygotes,
receptor or Apo B-100
with MI in teens (answer = absence of
functional LDL receptor).
- Xanthomas.
Hypertriglyceridemia - ­ TGAs.
­ Hepatic production of VLDL
(AD) - Pancreatitis.

Random HY cardio pharm points


- Non-dihydropyridine calcium channel blocker (acts on nodal calcium channels).
- Causes constipation (HY on Family Med).
Verapamil
- Used for AF for rate control sometimes in place of metoprolol (don’t worry about
the use-case; USMLE doesn’t give a fuck; just know MOA and side-effect).
- Dihydropyridine calcium channel blocker (acts on arteriolar calcium channels à
dilates arterioles à ¯ peripheral vascular resistance à ¯ BP).
- Causes peripheral edema / fluid retention (HY on Family Med).
Nifedipine
- Used for essential HTN in patients without diabetes, atherosclerotic disease, or
renal disease (if that sounds confusing, I talk about this in extensive detail in my HY
Risk Factors PDF; this is strict 2CK FM talking point).
- Can cause depression and sexual dysfunction.
- Avoid in patients with lung disease or history of severe or psychotic depression.
- Metoprolol used for rate-control in AF.
- Labetalol used first-line in patients who have aortic dissection and traumatic
rupture of the aorta. Nitroprusside comes after.
- 2CK Q gives “esmolol + nitroprusside” as answer to a traumatic rupture Q, but
almost always, they will just want “labetalol.”
- Timolol can be used topically for glaucoma.
b-blockers - Propranolol used for innumerable things à Tx of tachycardia in hyperthyroidism
(because also has additional effect of ¯ peripheral conversion of T4 à T3); akathisia
(restlessness as an extra-pyramidal side-effect of antipsychotics); essential, alcoholic,
or idiopathic tremors (have seen various tremors treated on NBME forms this way,
not just essential tremor); social phobia (if patient has asthma, 2CK Psych form wants
benzo instead); HOCM (¯ HR ­ LVEDV à ¯ murmur/obstruction).
- For propranolol, USMLE wants: ­ LVEDV; ¯ CO, ­ peripheral vascular resistance.
- For labetalol, USMLE wants: ­ LVEDV; ¯ CO, ¯ peripheral vascular resistance.

MEHLMANMEDICAL.COM 34
MEHLMANMEDICAL.COM

- Both drugs antagonize b1, which slows HR à ­ diastolic filling time à ­ LVEDV. The
reduced chronotropy and inotropy à ¯ CO.
- Propranolol is b1/2-non-selective; b2 agonism normally has dilatory effect on
peripheral arterioles, so if we antagonize à ­ peripheral vascular resistance.
- Labetalol, in contrast, has some a1 blockade effect in addition to b1, so PVR is ¯.
- Step 1 NBME wants beta-blockers as “slowing the rate of diastolic depolarization.”
In other words, if HR slows, then fractionally more time is spent in diastole, which
means the process of returning to systole is delayed/protracted.
- Potassium channel blocker.
Amiodarone - Can cause TdP, greyish skin discoloration, and thyroiditis.
- Used for VT in patients without coma or low BP.
- Sodium channel blocker.
Quinidine
- Can cause TdP and cinchonism (headache + tinnitus).
- Directly blocks myocardial Na+/K+ ATPase pump à causes indirect inactivation of
myocardial Na+/Ca2+ ATPase à more Ca2+ remains in myocardial cell à increased
contractility.
- Also has parasympathomimetic effect at nodal tissue that slows HR.
Digoxin
- In other words, digoxin both slows HR + increases contractility.
- Hypokalemia can cause toxicity. This is because digoxin binds to extracellular K+
binding site, so if less K+ is around to compete, lower dose is needed to induce effect.
- Toxicity presents classically as yellow/wavy “Vincent van Gogh” vision.
- USMLE-favorite ACE inhibitor.
- Can cause dry cough; also can ­ serum K+; avoid in hereditary angioedema.
Lisinopril
- Used for HTN in patients with pre-diabetes, diabetes, atherosclerotic disease, or
renal disease (I talk about this in HY Risk Factors PDF in more detail).
- Angiotensin II receptor blocker (ARB).
- Use-cases are identical on USMLE to ACEi (i.e., if you see both as answer choices to
Valsartan
a question, they’re usually both wrong because they’re the “same”).
- Doesn’t cause dry cough the way ACEi do.
- a1 agonists à constrict arterioles à ­ BP à HR ¯ due to baroreceptor reflex.
- Highest yield uses on USMLE are for nasal decongestion à constrict capillaries
Oxymetazoline, within nasal mucosa à ¯ inflammation à relief of congestion.
Phenylephrine - Can cause rhinitis medicamentosa, which means rebound nasal congestion upon
withdrawal if used non-stop for ~5 days. In other words, patients should use only as
needed for a maximum of about ~3-4 days while sick.
Isoproterenol - b1/2 agonist à increases HR and decreases peripheral vascular resistance.
- a2 agonists.
Methyldopa,
- Methyldopa used for HTN in pregnancy (nifedipine and labetalol also used).
Clonidine
- Clonidine used for various psych treatments (e.g., Tourette).
- a2 antagonist.
Mirtazapine
- Used to treat depression in patients who have anorexia (stimulates appetite).
Ritodrine - b2 agonist used as tocolytic (i.e., slows/delays labor).
- Dilates arterioles à ¯ BP.
Hydralazine - Used for hypertensive emergencies in pregnancy.
- Affects calcium currents (but not a calcium channel blocker).
- As discussed earlier, they liberate NO which ­ guanylyl cyclase à relaxation of
venous smooth muscle à venous dilation/pooling à ¯ preload on heart à ¯ oxygen
Nitrates
demand à relief of anginal pain.
- For sodium nitroprusside, choose arterioles as site of action.
- As mentioned earlier, statins have 2 HY MOAs on USMLE: 1) inhibit HMG-CoA
reductase; 2) upregulate LDL receptors on hepatocytes.
Statins
- Can cause myopathy and toxic hepatitis. An offline Step 1 NBME has myopathy as
correct over toxic hepatitis.
- Fibrates (e.g., fenofibrate) upregulate PPAR-a and lipoprotein lipase; best drugs to
Fibrates
decrease triglycerides.

MEHLMANMEDICAL.COM 35
MEHLMANMEDICAL.COM

- Can cause hepatotoxicity and myopathy, as well as cholesterol gall stones.


- If the USMLE asks you why statins + fibrates combined have ­ chance of myopathy,
the answer is “P-450 interaction.”
Ezetimibe - Blocks cholesterol absorption in the small bowel.
- Bile acid sequestrant. Causes reduced enterohepatic circulation of bile acids at
Cholestyramine terminal ileum à liver must now convert more cholesterol into bile acids in order to
replenish them à liver pulls cholesterol out of the blood to accomplish this.
Orlistat - Pancreatic lipase inhibitor used for obesity; can cause steatorrhea.
- Vitamin B3. Two MOAs USMLE wants you to know: 1) ¯ VLDL export by the liver; 2)
­ HDL more than any other medication.
- Deficiency à pellagra à 3Ds à dementia, dermatitis, diarrhea.
Niacin - Can present as delirium instead of overt dementia (biggest risk factor for delirium is
underlying dementia, so old patient with delirium often has underlying cognitive
decline); dermatitis will present on USMLE as either hyperpigmentation of the skin of
the forearms or Casal necklace.
- Sodium channel blocker thafft can be used in Wolff-Parkinson-White.
Procainamide
- Can cause drug-induced lupus with anti-histone antibodies.
- Sodium channel blocker used as first-line rhythm control in patients fail rate-control
for AF.
Flecainide
- Patient should have no structural or coronary artery disease. Otherwise use a
potassium channel blocker like amiodarone or dofetilide.
Dofetilide, - Potassium channel blockers.
Ibutilide - Can cause torsades (asked directly on Step 1 NBME).
Omega/fish oils - Decrease triglycerides.
- 2CK Surgery form gives patient who is in pain from surgery despite being on max
doses of morphine + they say blood pressure is elevated + ask what is next best step
à answer = “increase bolus of morphine.” Unusual, since they say max dose of
morphine already, but 1) we always full treat pain, and 2) explanation for the Q talks
about how pain can lead to high BP.
- My point here is that just be aware HTN can be caused by pain à treating pain can
bring down the BP.
Morphine
- Separately, treatment of MI, acute limb ischemia, and sickle crisis usually includes
oxygen + morphine.
- Answer on NBME exam for how a patient using a self-administering pump develops
morphine overdose = “morphine is converted into active metabolites that
accumulate.”
- Otherwise morphine is odd drug for me to put in this table; didn’t know where else
to put it though.

MEHLMANMEDICAL.COM 36
MEHLMANMEDICAL.COM

- 44M + Hx of epistaxis since adolescence + they show you pic of a tongue with a red dot on it +

dyspnea + high ejection fraction (75-80%); Dx? à answer on NBME = pulmonary arteriovenous

malformation à presentation is hereditary hemorrhagic telangiectasia (Osler-Weber-Rendu) à

USMLE will always show you a red dot on the tongue or fingernail à Q may also mention fatigue (GI

bleeding) à autosomal dominant.

- Child + idiopathic arrhythmia disorder + seizure-like episode; Dx? à Adam-Stokes attack à asked on

the peds CMS/NBME form even if you find this menacing, low-yield, or inconvenient à not a true

seizure if EEG performed à arrhythmia causes hypoxia of brainstem à seizure-like fit ensues.

- Postmenopausal woman + stress factor + bulging of LV on echo + hypercontractile LV base; Dx? à

Takotsubo cardiomyopathy à “ballooning of LV” à once again, weird Dx but USMLE likes it.

- Atherosclerosis; where does the process start? à USMLE answer = endothelial cell, not adipocyte.

- 3F + rumbling murmur auscultated in the neck that goes away when child is supine and the neck

rotated; Dx? à NBME answer = venous hum à student says wtf? à call it low-yield all you want but

it’s on the pediatrics CMS/NBME à benign peds murmur that will go away as child grows.

- Congenital heart block; Dx in the mom? à SLE à 1-5% of SLE mothers will have kid with CHB.

- Neonate with supravalvular aortic stenosis; Dx? à kid has William syndrome (chromosome 7, AD;

elfin-like facies; hypercalcemia due to increased vitamin D sensitivity; well-developed verbal skills).

- Left ventricular hypertrophy; USMLE asks arrow Q à answer = transcription factor c-Jun activity is

increased; beta-myosin heavy chain gene expression is increased; endothelin is increased.

- 65M + 2-3-day Hx of severe chest pain + dyspnea + visible pulsation above manubrium + tracheal

deviation + murmur in 2nd intercostal space on the right; Dx? à USMLE answer = aortic aneurysm.

- Dysphagia and/or hoarseness caused by dilated cardiac structure; which structure is dilated? à

answer = left atrium à the hoarseness is due to recurrent laryngeal nerve impingement by LA (Ortner

syndrome).

- Location of SA node? à answer on NBME = “junction of superior vena cava and right atrium.”

- Location of AV node? à answer on NBME = “inferior to the opening of the coronary sinus” à it is

located on the posteroinferior interatrial septum near the coronary sinus.

- Location of coronary sinus? à answer = “between the opening of the IVC and the tricuspid valve.”

MEHLMANMEDICAL.COM 37
MEHLMANMEDICAL.COM

- Neonate with truncus arteriosus; Q is which of the following populations of cells was most likely

absent during cardiac development? à USMLE answer = “ectodermal neural crest” cells.

- Fetal alcohol syndrome + heart/lung fistula; mechanism? à answer = “failure of migration of neural

crest cells.”

- Most common cause of death due to fall or MVA? à traumatic rupture of the aorta (thoracic).

- Where does rupture of the aorta occur? à where the ligamentum arteriosum wraps around the top

of the descending arch à ligament is taut but arch is more mobile à leads to shearing.

- What will the NBME/USMLE Q say for traumatic rupture à MVA or fall followed by “widening of the

mediastinum on CXR.”

- 32M + MVA + widening of mediastinum on CXR; next best step in Dx? à aortic arteriography (aka

aortography).

- Tx for traumatic rupture? à if ascending arch: labetalol + surgery; if descending arch: labetalol only.

- Traumatic rupture + low BP; next best step? à labetalol (decreases shearing forces, even with low BP

it’s the answer on the NBME).

- BP different between the arms; Dx? à aortic dissection or coarctation of aorta.

- Most likely cause of dissection? à HTN, but connective tissue disorders (e.g., Marfan, Ehlers-Danlos,

MYH11 mutations) also important.

- Tx for dissection? à if ascending aortic arch, answer = labetalol + surgery; if descending arch, answer

= labetalol only (HY, since everyone chooses surgery) à do not choose sodium nitroprusside here.

- Patient with dissection has low BP; next best step in pharm Mx? à labetalol (yes, even with low BP

need to decrease shearing forces).

- High TGAs + high LDL on lab report; Dx + mechanism? à familial hyperchylomicronemia; answer on

USMLE = “deficiency of apolipoprotein C-II” or “deficiency of lipoprotein lipase.”

- Normal TGAs + high LDL on lab report; Dx + mechanism? à familial hypercholesterolemia; answer on

USMLE = “deficiency of LDL receptor.” If total cholesterol is ~3-500s mg/dL, USMLE wants

“deficiency”; if total is ~700-1000 mg/dL, the answer = “absence of functional LDL receptors on

hepatocytes.”

- High TGAs + normal LDL on lab report; Dx + mechanism à familial hypertriglyceridemia; answer on

USMLE = “increased hepatic production of VLDL.”

MEHLMANMEDICAL.COM 38
MEHLMANMEDICAL.COM

- Confused old man + temp of 96F + CO high + PCWP low + TPR low; Dx? à septic shock à dementia

increases risk of aspiration pneumonia due to diminished gag reflex (important cause of sepsis) à by

all means the vignette might say urinary retention in BPH, or tell you there’s a catheter à also

important causes of sepsis here.

- Patient with infective endocarditis + now has limb weakness or sensory findings; Dx + Tx? à septic

embolus à answer = give immediate IV antibiotics.

- Intracranial aneurysm in someone just diagnosed with endocarditis; Dx? à mycotic aneurysm

(infected aneurysm; despite the name, almost never due to fungi).

- Patient with alternating tachycardia + bradycardia; Dx? à sick sinus syndrome à caused by damage

to SA node (i.e., due to coronary artery or valvular disease, or autoimmunity, e.g., sarcoidosis) à Tx

with atrial pacemaker.

- Diffuse ST-segment elevations on ECG; Dx? à pericarditis.

- If infective, most likely etiology of pericarditis? (answers are bacteria, parasitic, fungal, etc.); answer =

viral à viruses can cause serous pericarditis.

- 22M after night of heavy partying + central chest pain worse when leaning back + better when leaning

forward; Dx? à serous pericarditis caused by cocaine.

- 68F diabetic + high K + high BUN + high Cr + friction rub in central chest; Dx + Tx? à uremic

pericarditis à answer on USMLE = immediate hemodialysis.

- 72M + had STEMI two days ago + now has central friction rub; Dx? à post-MI fibrinous pericarditis.

- 72M + had STEMI 2-6 weeks ago + now has central friction rub; Dx? à Dressler syndrome

(autoantibodies causing fibrinous pericarditis).

- 34F + ulnar deviation + MCP/PIP pain + heart problem; Dx? à pericarditis à autoimmune diseases

like RA and SLE cause serous pericarditis.

- 34F + anti-Scl70 (topoisomerase I) Abs + heart issue; Dx? à pericardial fibrosis due to systemic

sclerosis.

- Kid + heart tumor = cardiac rhabdomyoma until proven otherwise (usually tuberous sclerosis).

- Adult + heart tumor = cardiac myxoma until proven otherwise (ball-in-valve tumor in the left atrium

à causes a diastolic rumble that abates when patient is positioned in an unusual way, e.g., on his

right side while leaning diagonally).

MEHLMANMEDICAL.COM 39
MEHLMANMEDICAL.COM

- 2-year-old boy has cardiac myxoma (correct, not rhabdomyoma) + perioral melanosis (sophisticated

way of saying hyperpigmentation around the mouth/lips) + hyperthyroidism; Dx? à answer = Carney

complex à this is asked on the USMLE à classically triad of cardiac myxoma + perioral melanosis +

endocrine hypersecretion (classically bilateral pigmented zona fasciculata hyperplasia resulting

Cushing syndrome, but can be hyperthyroidism or growth hormone).

- What does S1 heart sound mean? à closure of atrioventricular valves (mitral + tricuspid), signifying

onset of systole.

- What does S2 heart sound mean? à closure of semilunar valves (aortic + pulmonic), signifying onset

of diastole.

- What does S3 heart sound mean? à early-diastolic reverberation caused by dilated left ventricle à

often synonymous with dilated cardiomyopathy on the USMLE; yes, it can be seen sometimes

incidentally in pregnancy and young athletes (due to increased preload), but almost always it just

means dilated cardiomyopathy.

- What does S4 heart sound mean? à late-diastolic reverberation caused by stiff left ventricle à

caused by increased afterload on the LV, either due to systemic HTN or aortic stenosis (AS) or

hypertrophic obstructive cardiomyopathy (HOCM) à I have seen 2CK-level Qs where an S4 shows up

implying right ventricular hypertrophy (rare but possible) à for instance patient with mitral stenosis

+ S4. An S4 on the USMLE is often seen in hypertrophic cardiomyopathy (HCM), which just means a

stiff LV; don’t confuse HCM with HOCM; HOCM is an AD mutation in the beta-myosin heavy-chain

gene causing asymmetric septal hypertrophy; HCM is merely a hypertrophied LV due to increased LV

afterload (i.e., due to systemic HTN, AS, or HOCM); in turn HOCM can cause HCM, but they’re not the

same thing; if HOCM causes HCM, then USMLE loves to give S4 heart sound for that.

- What is a parasternal heave? à a parasternal heave means the heartbeat can be felt (or sometimes

seen) along the left sternal border, usually due to RVH (since the RV is most anterior) à RVH can be

seen in ventricular septal defect (VSD), so parasternal heave can be seen in VSD.

- What is a palpable thrill? à a palpable thrill is merely a palpable murmur; it carries no additional

diagnostic significance; a thrill is seen in grades 4-6 of the heart sounds.

- What are the 6 grades of heart sounds? (not asked on USMLE, but just for your own knowledge with

respect to this document) à 1: very faint; not heard in all positions (“cardiologist only”); 2: faint;

MEHLMANMEDICAL.COM 40
MEHLMANMEDICAL.COM

heard in all positions; 3: loud, with no thrill; 4: loud with palpable thrill; 5: loud with palpable thrill +

can be heard with stethoscope partially off chest; 6: loud with palpable thrill + can be heard with the

stethoscope completely off the chest.

- Which murmurs are holosystolic (aka pansystolic)? à mitral regurgitation (mitral insufficiency; MR) +

tricuspid regurgitation (tricuspid insufficiency; TR); ventricular septal defect (VSD).

- Which murmurs are mid-systolic (crescendo-decrescendo systolic) à aortic stenosis (AS) +

hypertrophic obstructive cardiomyopathy (HOCM) + pulmonic stenosis (PS).

- Which murmur has a diastolic opening snap? à mitral stenosis (MS) à has diastolic opening snap,

followed by a mid-late decrescendo diastolic murmur.

- Which murmur has a mid-systolic click? à mitral valve prolapse (MVP).

- Which murmur can also be described as a late-peaking systolic murmur with an ejection click? à AS.

- Which murmur is continuous machinery-like? à patent ductus arteriosus (PDA).

- Which murmur is pansystolic-pandiastolic? à PDA (same as continuous machinery-like).

- Which murmur is to-and-fro? à PDA; outrageous, but it’s on NBME 6 for 2CK and relies on you

knowing this description to get it right; every student gets this Q wrong and then says “wtf is to-and-

fro.” (my students of course will say, “got that one right because of you”).

- Which murmur is fixed splitting of S2? à atrial septal defect (ASD).

- Which murmurs are holodiastolic (pandiastolic)? à aortic regurgitation (aortic insufficiency; AR) +

pulmonic regurgitation (pulmonic insufficiency; PR).

- Which murmur is pandiastolic and loudest in early-diastole? à classically AR (decrescendo

holodiastolic murmur).

- Young child + hypocalcemia + harsh systolic murmur at left sternal border; Dx? à DiGeorge syndrome

associated with tetralogy of Fallot à on the USMLE, you should essentially think of ToF and DiGeorge

syndrome as interchangeable à you can by all means get other heart defects in DiGeorge, e.g.,

truncus arteriosus, but I can’t emphasize enough that ToF is almost always seen in DiGeorge on

USMLE.

- Important initial principle regarding heart murmurs à all will get worse / more prominent with more

volume in the heart, however MVP and HOCM are the odd ones out; they’ll get worse with less

volume in the heart.

MEHLMANMEDICAL.COM 41
MEHLMANMEDICAL.COM

- 8F + sickle cell + fever + HR of 120 + normal BP + 2/6 mid-systolic murmur at upper right sternal

border; Dx? à transient, functional high-flow murmur secondary to tachycardia à murmur will

subside once HR returns to baseline.

- 13F + Hb of 7 g/dL + HR of 120 + normal BP + + 2/6 mid-systolic murmur at upper right sternal border;

Dx? à once again, transient, functional flow murmur à I point this out because students often

erroneously think there’s some heart abnormality when they see this type of murmur.

- Aortic stenosis (AS) – what will you auscultate? à mid-systolic (crescendo-decrescendo systolic)

murmur classically at 2nd intercostal space, right sternal border, with radiation to the carotids;

however will also show up as “late-peaking systolic murmur with an ejection click.” à don’t confuse

the latter with “mid-systolic click,” which is mitral valve prolapse (MVP).

- Who gets AS? à classically bicuspid aortic valve à can be familial autosomal dominant; also seen in

Turner syndrome (45XO) à leads to early calcification of valve in the 40s onward; however a young

patient without significant calcification can easily have AS.

- What about if the patient doesn’t have bicuspid valve? à AS can still occur in the general population

with normal senile calcification seen typically age 70s-80s onward (i.e., incidental 1/6 or 2/6 mid-

systolic murmur in otherwise healthy elderly patient).

- If patient is diagnosed with bicuspid aortic valve, next best step in Mx? à annual transthoracic echos

à if valve cross-sectional area falls below 1.0 cm2 then do aortic valve replacement; there’s a surgery

NBME Q where they say cross-sectional area is 0.8 cm2 and the answer is straight-up “aortic valve

replacement.”

- How does AS classically present Sx-wise? à SAD à Syncope, Angina, Dyspnea.

- AS causes what kind of LVH? à concentric hypertrophy due to pressure overload à can also cause

hypertrophic cardiomyopathy with an S4 heart sound (stiff LV). This is in contrast to aortic

regurgitation (aortic insufficiency), which causes eccentric hypertrophy due to volume overload.

- What kind of pulse is seen in AS? à slow-rising pulse (“pulsus parvus et tardus”). Don’t confuse this

with AR, which causes bounding pulses with head-bobbing (Q will often say for AR: “pulse has brisk

upstroke with precipitous downstroke.”).

- Any weird factoid about AS? à Heyde syndrome is the combo of AS + angiodysplasia (painless rectal

bleeding in elderly due to superficial tortuous vessels on the bowel wall) à shows up on NBME.

MEHLMANMEDICAL.COM 42
MEHLMANMEDICAL.COM

- What does HOCM sound like? à same as AS (mid-systolic murmur, aka crescendo-decrescendo

systolic murmur).

- What causes HOCM? à autosomal dominant mutation in beta-myosin heavy-chain gene.

- What’s the structural change in the heart with HOCM? à asymmetric septal hypertrophy that causes

the anterior mitral valve leaflet to block off the LV outflow tract under states of lesser preload à

student says, “if the LV outflow tract is blocked off (i.e., where the aortic valve is), why is it the mitral

valve leaflet that blocks it off then?” Yeah, I know, it’s weird. But the asymmetric septal hypertrophy

causes this to happen.

- What’s the cause of death in HOCM? à ventricular fibrillation (really HY!!) à the “sudden death in

young athlete” is not due to an MI à i.e., the patient has clean coronary arteries à do not select

coronary artery occlusion as the answer.

- What about if the vignette is sudden death in middle-aged patient with heart disease? à answer =

ischemic heart disease (MI), not HOCM.

- 18M athlete + 2/6 mid-systolic murmur at right sternal border 2nd intercostal space + there is

paradoxical splitting of S2 + there is no change in the murmur with Valsalva; Dx? à ”bicuspid aortic

valve” (AS), not HOCM à students say “oh em gee young athlete! HOCM!” à the USMLE will slam

you on this and wants you to know that the key way to distinguish between AS and HOCM murmurs is

that HOCM gets worse with lower volume in the heart; AS will soften or there will be no change.

Don’t just automatically jump on HOCM because it’s a young athlete.

- How to Tx HOCM à can give propranolol to keep HR from getting too fast (the slower the HR, the

more time the heart spends in diastole à more diastolic filling à greater preload à less occlusion of

LV outflow tract) à should be noted tangentially that although beta-blockers increase preload, they

decrease chronotropy + inotropy so the net effect is still decreased myocardial oxygen demand.

- Can you explain “splitting of S2”? What does that even mean? à the aortic valve normally shuts (A2)

just before the pulmonic valve (P2), so A2 will occur slightly before P2 à when we talk about changes

in splitting of the S2 heart sound (i.e., wide splitting, paradoxical splitting, etc.), if pressure in a

ventricle is greater, the sound will occur later / is protracted. So if RV pressure becomes greater for

whatever reason à P2 occurs later à wider splitting. So pulmonary artery hypertension = wide-

splitting. If LV pressure becomes greater à A2 occurs later, and can even occur after P2 à

MEHLMANMEDICAL.COM 43
MEHLMANMEDICAL.COM

paradoxical splitting. So LVH = paradoxical splitting. When R or L ventricular pressure exceeds the

pulmonic arterial and aortic pressure, respectively, the valves open. Then the ventricle will lose

pressure as blood ejects, followed by isovolumetric relaxation marking the onset of diastole, and the

pressure within the ventricle falls below the pressure distal to the valve à valve shuts. Normally

splitting oscillates with the respiratory cycle. Inhalation causes P2 to occur later à decrease in

intrathoracic pressure à increased venous return to right atrium à more blood in right ventricle à

more preload à more pressure à time it takes for RV pressure to fall below pulmonic arterial

pressure is greater à P2 will occur slightly later with inhalation. With exhalation it’s the opposite. P2

occurs slightly sooner because increased intrathoracic pressure will attenuate venous return à less

preload in RV à less pressure in RV à time it takes for RV pressure to fall below pulmonic arterial

pressure is less à pulmonic valve closes slightly sooner à distance between A2 and P2 is less.

- What is fixed splitting of S2? à Super HY for atrial septal defect (ASD) à sometimes can be written as

“wide, fixed splitting of S2” à it’s not the “wide” that matters; you need to remember fixed splitting.

- What does “splitting of S1 mean”? à highly unlikely to show up on the USMLE, don’t worry, but for

the sake of some people who’d ask, it’s usually seen in right bundle branch block (BBB), which causes

delayed closure of the tricuspid valve.

- Maneuvers that decrease blood in the heart à Valsalva; standing up from seated position; sitting up

from supine position; administration of nitrates à any of these will cause MVP + HOCM to get worse;

all other murmurs will soften or not change.

- Maneuvers that increase blood in the heart à Lying down; leg raise while supine; squatting; hand-

grip.

- How does Valsalva decrease blood in the heart? à attempted exhalation against a closed glottis à

robust increase in intrathoracic pressure à decreased venous return à decreased cardiac preload.

- How do nitrates decrease blood in the heart? à if administered venously à increased venodilatation

+ venous pooling à decreased venous return to the heart à decreased cardiac preload. If

administered arterially à decreased afterload à easier for the LV to eject blood à decreased blood

in the LV; it should be noted that it would be incorrect to say arterial nitrates decrease preload; this is

an indirect effect in this case.

MEHLMANMEDICAL.COM 44
MEHLMANMEDICAL.COM

- How does hand-grip increase blood in the heart? à hand-grip increases afterload à LV cannot eject

blood as readily à greater volume of blood left in the LV; it should be pointed out, however, that it

would be incorrect to say hand-grip increases preload, as this effect is indirect.

- Exceptions regarding handgrip in terms of murmur intensities? à decreases intensity of aortic

stenosis (same as HOCM) + increases intensity of mitral stenosis (in an NBME vignette).

- How does respiration relate to left- vs right-sided murmurs à inspiration makes right-sided murmurs

worse; exhalation makes left-sided murmurs worse.

- Why does inspiration make right-sided murmurs worse? à inspiration à decreased intrathoracic

pressure à easier for blood to return to the RA à increased venous return à more preload in right

heart à worsening of TR, TS, PR, PS.

- Why does inspiration soften left-sided murmurs? à decreased intrathoracic pressure à increased

pulmonary vascular compliance à transient decrease in pulmonary venous return to the LA à

decreased LA preload; it should be noted that although RA preload increases, this effect does not

carry over to the LA because of pulmonary vascular pooling.

- Why does expiration soften right-sided murmurs? à expiration à increased intrathoracic pressure

à harder for blood to return to the RA à decreased venous return à less preload in right heart à

softening of TR, TS, PR, PS.

- Why does expiration intensify left-sided murmurs? à expiration à increased intrathoracic pressure

à decreased pulmonary vascular compliance à transient increase in pulmonary venous return to the

LA à increased LA preload; it should be noted that although RA preload decreases, this effect does

not carry over to the LA because of pulmonary vascular compression.

- What does VSD sound like? à USMLE will describe it two ways: 1) holosystolic murmur (aka

pansystolic) at the left sternal border (or lower left sternal border) with a parasternal heave or thrill;

2) holosystolic murmur at the left sternal border with a diastolic rumble (weird, but in NBME Qs and

possibly an effect from movement across the valve even during the diastolic filling stage).

- Most common congenital heart defect? à VSD.

- If you patch/repair a VSD, what will happen to pressure in the LV, RV, and LA? (up or down arrows) à

repairing a VSD will cause up LV, down RV, down LA à the down always confuses people à repair of

VSD means less blood entering RV à less blood going back through the lungs to the LA.

MEHLMANMEDICAL.COM 45
MEHLMANMEDICAL.COM

- Who gets AVSD (atrioventricular septal defect)? à Down syndrome (aka endocardial cushion defect).

- What does ASD sound like and why? à as discussed earlier, fixed splitting of S2 à when you’ve got

an ASD, blood is constantly moving L –> R from LA –> RA (pressure is always greater on the left side).

So the effects of inhalation/exhalation are minimized in terms of the A2-P2 split bc you’ll always have

relatively constant LA à RA flow (and resultant steady RA preload) irrespective of inspiration. The

sound can also be described as “wide, fixed splitting” bc of increased RV preload à delayed closure

of P2 relative to A2 à slight widening, but it’s still fixed for the reasons explained above.

- What is the fossa ovalis? à impression in the interatrial septum following closure of the wall during

embryological development. Failure of closure leads to patent foramen ovale, which is a type of ASD.

- Ostium primum ASD? à Down syndrome.

- Ostium secundum ASD? à most common type of ASD; 20% of patients also have MVP.

- ASD/VSD in relation to blood pO2? à USMLE loves making you guess whether you have an ASD or

VSD based on info they give you about blood pO2. If they say pO2 increases from SVC to RA, you

know the answer is ASD. If they say pO2 increases from RA to RV, you know VSD is the answer. Path

of blood through the heart/lungs (IVC/SVC à RA à RV à pulmonary arteries (deoxygenated) à

pulmonary veins (oxygenated) à LA à LV à aorta).

- What does MVP sound like? à as mentioned earlier, mid-systolic click, over 4th intercostal space, left

mid-clavicular line.

- Who gets MVP? à most common heart murmur; polygenic; usually benign + incidental; can also get

in connective tissue disorders (i.e., Marfan syndrome, Ehlers-Danlos).

- Murmurs seen in connective tissue disorders à MVP or AR.

- What does myxomatous degeneration mean? à answer = MVP à connective tissue degeneration.

- What does MR sound like? à holosystolic (pansystolic) murmur over 4th intercostal space, left mid-

clavicular line; classically radiates to the axilla but by all means doesn’t have to.

- Most common cause of MR? à ischemia; student says “what do you mean?” à atherosclerosis (i.e.,

due to diabetes, smoking, HTN, familial) à ischemia à structural heart change (LVH + LV dilatation) +

papillary muscle weakening à MVP. This is not the same thing as full-blown papillary muscle rupture

following an MI; mere ischemia in the absence of MI can cause MR.

MEHLMANMEDICAL.COM 46
MEHLMANMEDICAL.COM

- 68M + T2DM + dilated heart on CXR + S3 heart sound + 2/6 holosystolic murmur over left chest; Dx?

à MR due to ischemia from atherosclerosis.

- 68M + T2DM + crushing central chest pain + 3 days later has sudden-onset 4/6 holosystolic murmur

over left chest; Dx? à MR from papillary muscle rupture.

- 22M + obvious Marfan syndrome in vignette + stethoscope Q where it starts hovering over aortic

valve region; what do you do? à you’re listening for either AR or MVP, so if you don’t hear anything,

just move the stethoscope to the mitral area and you’ll hear the mid-systolic click.

- 32M + fleeting/stabbing chest pain along the left-lateral chest wall + has had 20-30 episodes like this

in the past + mid-systolic click; Dx? à mitral valve prolapse syndrome à do not need to treat

overwhelming majority of the time, even when the patient is symptomatic; on the 2CK NBMEs, they

give a symptomatic presentation just like this, and the answer is reassurance/observation, not

propranolol.

- What does mitral stenosis sound like? à diastolic opening snap with a decrescendo mid-late diastolic

murmur.

- Who gets MS? à 99% are due to previous rheumatic fever (exceedingly HY!!).

- What is mechanism for rheumatic fever? à type II hypersensitivity against M-protein of Group A

Strep (S. pyogenes) à immune system makes antibodies against Group A Strep M-protein that cross-

react with the mitral valve (molecular mimicry).

- How does rheumatic fever present? à JONES (J©NES) à Joints (polyarthritis) + © (myocarditis /

mitral valve disease) + Nodules (subcutaneous nodules over bony prominences) + Erythema

marginatum (appears annular [ring-like] and serpiginous [snake-like]; important vocab words actually

for medicine) + Sydenham chorea (antibody-mediated destruction of corpus striatum of basal ganglia)

à my biggest advice here is to remember “marginatum” because it’s specifically seen in RF; don’t be

that person going around saying “RF has……..erythema multiforme…..?” The latter is usually seen as

one of the types of drug rashes.

- 12F + red tongue + salmon body rash; Dx and Tx? à scarlet fever caused by Group A Strep. Must give

penicillin to prevent RF.

MEHLMANMEDICAL.COM 47
MEHLMANMEDICAL.COM

- RF caused by cutaneous Group A Strep? à No. Cutaneous infections (i.e., impetigo, erysipelas,

cellulitis) can cause post-streptococcal glomerulonephritis (PSGN), but not RF. RF is caused by

pharyngitis / scarlet fever (pharyngitis + body rash).

- 40M + Hx of rheumatic fever as a child; what murmur does he most likely have now? à MS.

- 40M + Hx of rheumatic fever as a child + rumbling diastolic murmur + S4 heart sound; Dx? à MS with

a right-ventricular S4 (unusual to be right-sided, but this is on the 2CK NBME).

- 40M + Hx of rheumatic fever as a child + rumbling diastolic murmur; which of the following is most

likely part of his Hx as a child? à mitral regurgitation. à this is exceedingly HY à RF causes MR

acutely in the child but will cause MS later in life as the valve scars over.

- 12M + fever + sore throat + painful joints + annular skin rash + heart murmur; most likely murmur

that he has? à MR, not MS.

- 12M + fever + sore throat + painful joints + annular skin rash + 2/6 holosystolic murmur over left

chest; as an adult, what might we expect in this patient? à answer = diastolic rumbling murmur with

opening snap (MS); even though right now he has MR, later in life it will become MS.

- 33F + pregnant at 20 weeks + new-onset dyspnea + crackles in both lung fields + diastolic rumbling

murmur; Dx? à mitral stenosis presenting symptomatically now that plasma volume has increased

~50% in pregnancy.

- 33F + pregnant at 38 weeks + prominent bilateral ankle pitting edema + dyspnea; Dx? à peripartum

dilated cardiomyopathy (antibody-mediated; idiopathic).

- 33F + pregnant at 32 weeks + mild ankle edema; Dx? à normal edema seen in pregnancy.

- 33F + peripartum dilated cardiomyopathy; next best step in Mx? à transthoracic echo (TTE) to check

for ejection fraction (guides management based on severity).

- Does peripartum dilated cardiomyopathy come back in susbsequent pregnancies? à Yes, and it gets

worse.

- 33F + peripartum dilated cardiomyopathy; best way to predict prognosis if she goes on to have a

future pregnancy? à TTE (ejection fraction predicts prognosis for future pregnancy).

- 33F + Hx of peripartum dilated cardiomyopathy + she gets pregnant a second time; what needs to be

done at antenatal counseling? à discuss options for termination à this sounds outrageously wrong

MEHLMANMEDICAL.COM 48
MEHLMANMEDICAL.COM

but is the correct answer in UWorld for Step 3 à basis is that there is high risk of maternal death

because the cardiomyopathy gets worse in subsequent pregnancies. This is not imposing a decision

on the patient; this is merely discussing risks and letting her know that maternal and fetal death is

important concern.

- If 99% of MS is due to Hx of rheumatic fever, what else can cause it? à Libman-Sacks endocarditis in

SLE is associated with MS. 60% of those with LS endocarditis have anti-phospholipid antibodies (lupus

anticoagulant).

- 32F + SLE + diastolic rumbling murmur; most likely characteristic of valvular lesion? à answer =

“verrucous vegetations on both sides of the mitral valve” for LS endocarditis.

- 28M IV drug user + 2/6 holosystolic murmur at left sternal border + fever; most likely characteristic of

valvular lesion? à “large, friable, floppy vegetation” à bacterial endocarditis (probably tricuspid

regurg in this case bc IV drug user).

- New-onset murmur + fever; Dx? à infective endocarditis (IE). Unlike RF, this is actual bacterial

colonization of a heart valve + production of vegetation; RF is a mere antibody response.

- What is acute endocarditis? à no Hx of valve abnormality; caused almost always by S. aureus;

classically seen in IV drug users); classic new-onset murmur + fever presentation.

- What is subacute endocarditis? à Hx of valve abnormality, i.e., congenital defect, Hx of RF; classically

occurs following dental procedures; S. viridans (same thing as S. sanguinis or S. mutans) is classic

cause; USMLE wants you to know S. viridan’s production of limit dextrins (carbohydrates) enables

binding to mitral + aortic valve.

- What is HACEK? à Gram (-) organisms that can cause endocarditis – Hemophilus species,

Aggregatibacter actinomycetemcomitans, Cardiobacterium hominis, Eikenella corrodens, and Kingella

kingae – the yield on the USMLE is extremely low so you do not need to memorize these, but I

mention them because students occasionally ask about HACEK + the USMLE likes you to know for

some magical reason that Eikenella corrodens is associated with human bites, grows white, and has a

bleach-like odor.

- For IE, blood cultures before Abx, or Abx before blood cultures? à Always blood cultures (draw three

tubes of blood) before giving Abx.

MEHLMANMEDICAL.COM 49
MEHLMANMEDICAL.COM

- Empiric Tx for endocarditis? à Gentamicin (aminoglycoside) PLUS either vancomycin or

ampicillin/sulbactam. Add rifampin if patient has prosthetic valve.

- If culture comes back positive for MSSA (not MRSA)? à switch to six weeks nafcillin (highly simplified,

but the bottom line is if MSSA is confirmed, USMLE answer is you switch to the beta-lactam).

- Why switch to beta-lactam if MSSA? Why not just stay on vanc? à Beta-lactams are way more

efficacious than vanc à vanc is actually not a very good drug, but if the organism causing IE is MRSA,

it’s first-line.

- Who gets pulmonic stenosis and what does it sound like? à sounds like aortic stenosis (midsystolic

murmur) but increases in intensity with inspiration because it’s right-sided; classically seen as part of

tetralogy of Fallot in DiGeorge syndrome; also seen classically in Noonan syndrome (USMLE will not

ask you about Noonan syndrome).

- Who gets pulmonic regurg and what does it sound like? à sounds like aortic regurg (holodiastolic)

but increases with inspiration; rare, but can be seen in endocarditis in IV drug users.

- Who gets tricuspid regurg and what does it sound like? à same as mitral regurg (holosystolic

murmur) but gets louder with inspiration; seen in IV drug user endocarditis; also seen in carcinoid

syndrome (small bowel, appendiceal, or bronchial neuroendocrine tumor that secretes serotonin,

leading to diaphoresis, tachycardia, diarrhea, and tricuspid regurg; Dx with urinary 5-hydroxyindole

acetic acid [5-HIAA]); 2CK NBMEs love pulmonary hypertension causing TR (i.e., you’ll have cor

pulmonale with TR and be like “huh? Why is there TR? What am I missing here?” But once again it can

be seen in PH).

- Who gets tricuspid stenosis and what does it sound like? à sounds like mitral stenosis presumably

(diastolic rumbling murmur, with or without opening snap); very rare; I’ve never seen this in any

USMLE question.

- How does isolated left heart failure present? à fluid in the lungs (pulmonary edema) +/- pleural

effusion; orthopnea, paroxysmal nocturnal dyspnea (PND); depending on the etiology of the heart

failure, the structure of the heart will take on different characteristics, but the important point about

LH failure is fluid in the lungs à also really important you know that pulmonary capillary wedge

pressure (PCWP) is increased in any LH pathology (even if the pressure is within the acceptable range

MEHLMANMEDICAL.COM 50
MEHLMANMEDICAL.COM

prior to full-blown LH decompensation, the PCWP is still increased relative to the patient’s original

baseline in LH pathology.

- What is PCWP? à equal to left atrial pressure; if you stick a catheter through the venous circulation

all the way back to the right heart, and then into the pulmonary circulation, and then into a distal

pulmonary capillary such that it can’t go any farther, the pressure reverberations are said to best

reflect those of the left atrium. The USMLE is obsessed with PCWP; you need to know it is increased

not just in cardiogenic shock, but also in LH pathology as I’ve stated above.

- What is orthopnea? à reflective of LV decompensation à requirement to sleep on multiple pillows

when supine to prevent fluid buildup in lungs; when supine, there’s greater venous return à greater

preload à worsening of dyspnea because the heart cannot handle the volume (i.e., decompensates)

because greater O2 demand by the LV with greater preload.

- What is PND? à like orthopnea, reflective of LV decompensation à severe dyspnea that occurs while

sleeping due to redistribution of fluid into the lungs; unlike orthopnea, does not immediately subside

when sitting up.

- How does isolated right heart failure present? à right ventricular hypertrophy (unless tricuspid

pathology), jugular venous distension (JVD), peripheral edema, hepatosplenomegaly (late).

- What’s the most common cause of right heart failure? à left heart failure.

- What is congestive heart failure (CHF)? à right heart failure + left heart failure.

- What causes left heart failure? à systemic HTN, ischemia (atherosclerosis), valvular abnormalities.

- Since left heart failure is the most common cause of right heart failure, what usually causes isolated

right heart failure? à lung pathology à when you have lung pathology causing RH failure, that’s

called cor pulmonale.

- “Wait, can you explain cor pulmonale a little more. I’ve heard that a lot but don’t really understand

it.” à when you have a lung condition like COPD, cystic fibrosis, chronic asthma, etc., that leads to RH

failure, we call that cor pulmonale. Probably the most important piece of info regarding this condition

is that PCWP is normal, which tells you the cause of the RH failure cannot be from LH origin. For

instance, if you have a guy with COPD who also has heart disease, if his PCWP is elevated, then we

cannot conclude that his right heart failure is a result of the lung disease in isolation because

increased PCWP can lead to RH failure.

MEHLMANMEDICAL.COM 51
MEHLMANMEDICAL.COM

- 45M + 70-pack-year Hx of smoking + JVD + peripheral edema; Dx? à cor pulmonale à signs of RH

failure in someone who has lung disease.

- 45M + 70-pack-year Hx of smoking + systemic HTN + JVD + peripheral edema + has crackles in lungs +

dilated heart on CXR à CHF à the dilated heart in someone with HTN suggests left heart failure.

- 25M + cystic fibrosis + JVD + peripheral edema + crackles in lungs à cor pulmonale à crackles due to

CF, not LH failure.

- If USMLE asks you for the mechanism of cor pulmonale, what’s the answer? à pulmonary

hypertension à hypoxic vasoconstriction (e.g., in chronic bronchitis, CF) or obliteration of pulmonary

parenchyma (emphysema) will cause a backup of blood and pulmonary HTN à increased afterload on

RV à starts the process of cor pulmonale (mere pulmonary HTN is not cor pulmonale; there must be

RH failure). Can also be due to fibrosis (e.g., CREST or radiation) + loss of lung parenchyma

(emphysema).

- How will USMLE describe pulmonary HTN? à increased pulmonary vascular markings; loud P2.

- 28F + non-smoker + dyspnea + JVD + increased pulmonary vascular markings; Dx? à primary

pulmonary hypertension.

- What is primary pulmonary HTN? à mutation in BMPR2 gene leading to narrowing of pulmonary

vessels + RH failure.

- Tx for pulmonary HTN à most patients will respond to dihydropyridine CCBs; if fail, can use agents

like bosentan (endothelin-1 receptor antagonist), prostaglandins (i.e., epoprostenol), or sildenafil

(yes, Viagra).

- 28F + non-smoker + dyspnea + JVD + increased pulmonary vascular markings; which of the following

might describe her condition? à USMLE answer = increased endothelin-1 expression (which makes

sense as we know bosentan is a Tx).

- What is dilated cardiomyopathy (DCM) + what are the causes? à heart failure with dilatation of the

LV cavity + systolic dysfunction with decreased ejection fraction; classic causes are systemic HTN and

ischemia (coronary atherosclerosis), but may also be due to ABCD à Alcohol (EtOH directly damages

myocardium); Beriberi (wet beriberi seen in thiamine [B1] deficiency; this is not the same as alcoholic

cardiomyopathy; it’s coincidental that this also occurs in alcoholics; alcoholics can get DCM and need

MEHLMANMEDICAL.COM 52
MEHLMANMEDICAL.COM

not be B1 deficient); Cocaine, Chagas disease (Trypanosoma cruzi), Coxsackie B virus; Doxorubicin

(Adriamycin); DCM can also be caused by pregnancy (as discussed earlier) and hemochromatosis.

- How does DCM present? à enlarged cardiac silhouette on CXR (dilated heart), lateralized apex beat

(dilated heart); sometimes S3 heart sound; fluid in the lungs (pulmonary edema) +/- pleural effusion;

can present with orthopnea, paroxysmal nocturnal dyspnea (PND).

- Is HOCM the same thing as hypertrophic cardiomyopathy (HCM)? à Once again, no. HOCM is an AD

condition (as discussed earlier). HCM is the diastolic dysfunction of the LV that ensues secondary to

increased LV afterload (i.e., from systemic HTN, AS, or HOCM) à the USMLE will often give you an S4

heart sound for HCM. so in turn, HOCM can be a cause of HCM, but don’t use the terms

interchangeably.

- What about restrictive cardiomyopathy (RCM)? à diastolic dysfunction with failure of the heart to

expand, in the absence of a thickened LV as seen in HCM; causes are fibrosis, amyloidosis, sarcoidosis,

scleroderma, prior radiation, etc.

- What are the important arrows for systolic dysfunction? à Ejection fraction – decreased; LV

pressure – increased; LV volume – increased.

- What are the important arrows for diastolic dysfunction? à Ejection fraction – normal; LV pressure –

increased; LV volume – normal.

- “Can you explain restrictive cardiomyopathy vs constrictive pericarditis?” à both are characterized

by diastolic dysfunction (decreased ability of heart to expand), but in RCM this is due to myocardial

stiffness / inelasticity, whereas in CP, the etiology is strictly pericardial, with TB being the most

common cause of chronic constrictive pericarditis; CP is associated with calcification on CXR in about

a third of patients; calcification is not seen in RCM.

- 22M + stab wound to left chest + JVD + muffled heart sounds + hypotension; Dx? à cardiac

tamponade à Beck triad always seen in USMLE Qs = JVD + muffled heart sounds + hypotension; also

associated with pulsus paradoxus.

- What is pulsus paradoxus? à drop in BP of >10 mm Hg on inspiration à reflects inability of the heart

to fill à seen in cardiac tamponade, severe lung disease (i.e., severe asthma, COPD), and sometimes

severe sleep apnea à my observation is students love to focus on miscellaneous causes of PP, but in

MEHLMANMEDICAL.COM 53
MEHLMANMEDICAL.COM

reality the USMLE only gives a fuck about cardiac tamponade; Qbank might venture down the asthma

route once in a blue moon.

- What’s the difference between pericardial effusion and cardiac tamponade? à all tamponades are

effusions, but not all effusions are tamponades à a tamponade is merely a pericardial effusion that is

associated with hemodynamic decompensation (low BP).

- What determines whether a pericardial effusion becomes a tamponade? à acuteness of fluid

accumulation à tamponades can be a small volume that accumulates quickly, e.g., from a stab

wound or post-MI LV free-wall rupture; whereas we could have, e.g., a slowly accumulating chylous or

serosanguinous accumulation in the setting of lymphoma/malignancy that is large volume but does

not lead to tamponade.

- USMLE asks where pericardial fluid is secreted into (Step 1) à between the visceral and parietal

serous layers à the pericardium = visceral + parietal serous layers + an outer fibrous layer.

- 6M + strong radial pulses + cold lower extremities; Dx? à coarctation of the aorta à this is the

presentation of one of the Step 1 NBME Qs à coarctation need not only occur in Turner syndrome; in

fact, it’s actually twice as common in males à coarctation on USMLE will be too easy if they say high

BP in upper limbs + low BP in lower limbs; simply look for description of pulses, etc.

- What is pre-ductal vs post-ductal coarctation? à if the coarctation occurs proximal to the ductus

arteriosus insertion on the descending aortic arch, it’s called pre-ductal à adequate blood flow to

lower limbs is therefore dependent on a PDA if the coarctation is severe à if the coarctation is

severe, the neonate will become cyanotic a few days to a week after birth contemporaneous to the

ductus arteriosus closure; pre-ductal is also the answer for the type seen in Turner syndrome; in

post-ductal, although blood flow to the lower limbs is impaired even if a PDA is present, it is usually

not as severe as pre-ductal and therefore yields greater time for adequate collateral circulation to

develop, leading to rib notching (dilatation of intercostal arteries) and presentation later in life,

sometimes adulthood.

- Congenital rubella syndrome + cardiovascular defect; Dx? à PDA à continuous machinery-like

murmur, pansystolic-pandiastolic, or to-and-fro; can also present with bounding pulses similar to AR.

- How to close PDA? à indomethacin (NSAID).

- How to keep PDA open? à prostaglandin.

MEHLMANMEDICAL.COM 54
MEHLMANMEDICAL.COM

- Kid is cyanotic at birth; before surgery, what should be given? à prostaglandin à open PDA can mask

and/or mitigate the effects of cyanotic congenital heart disease.

- Bile acid sequestrants; important info? à colestipol, cholestyramine, colesevelam à decrease

enterohepatic circulation of bile acids à liver needs to make more à liver pulls cholesterol out of the

blood in order to convert it to more bile acids à decreases serum LDL à may slightly increase TGAs

à just be aware of their names + mechanism for Step 1 à these drugs do not decrease mortality.

- Ezetimibe; mechanism? à inhibits absorption of cholesterol through the small bowel wall at brush

border à does not decrease mortality à just be aware of this drug name + MOA for Step 1.

- Fibrates; important info? à gemfibrozil, fenofibrate à upregulate lipoprotein lipase (LPL) à

increases TGA clearance out of blood à answer on the USMLE is TGAs >300 mg/dL; first-line agent

to treat severe hyper-TGAs à also upregulates PPAR-alpha, which increases HDL synthesis à cause

hepatotoxicity (same as statins), myositis with increased creatine kinase (CK; especially when

combined with statins), and cholesterol gallstones (due to inhibition of 7-alpha-hydroxylase, which

converts cholesterol to bile acids) à do not decrease mortality.

- Why is myositis/rhabdomyolysis more likely when fibrates are combined with statins? à USMLE

wants “inhibition of P-450 enzymes” as the answer (fibrates inhibit P-450).

- Statins; mechanism? à competitive, reversible inhibition of HMG-CoA reductase à this secondarily

causes upregulation of hepatic LDL receptors à both mechanisms decrease serum LDL à statins

decrease morality, not because of their LDL-lowering effect (because other drugs do that too), but

because they have an antioxidant effect that transcends the cholesterol-lowering effect à USMLE

wants you to know that statins increase HMG-CoA mRNA synthesis (compensatory; makes sense, but

students get the Q wrong; this is on an NBME); cause hepatotoxicity and myositis/rhabo (the latter

especially when combined with fibrates, as said above).

- Orlistat; MOA à pancreatic lipase inhibitor à sometimes used to Tx obesity à can cause fat-soluble

vitamin deficiencies + steatorrhea.

- Evolocumab + alirocumab; MOA? à PCSK9 inhibitors à Proprotein convertase subtilisin/kexin type 9

is an enzyme that breaks down LDL receptors à therefore these drugs prevent breakdown of LDL

receptors and decrease LDL cholesterol by enabling greater clearance à these are newer agents than

MEHLMANMEDICAL.COM 55
MEHLMANMEDICAL.COM

statins and are fair game on the USMLE à New England Journal of Medicine study showed they

decrease mortality.

- Digoxin; MOA? à inhibits Na/K-ATPase pump à leads to buildup of Na inside the cardiac myocyte à

leads to indirect inhibition of Na/Ca-ATPase pump bc the buildup of intracellular Na disfavors the

inward movement of Na via this latter pump à therefore Ca doesn’t move out of the cell à

increased intracellular Ca à increased inotropy/contractility à also slows HR via Vagal stimulation.

- Yellow wavy vision + drug OD? Dx? à digoxin toxicity.

- How to Tx digoxin toxicity? à normalize potassium + give anti-digoxin Fab fragments + give Mg.

- Electrolyte disturbance causing digoxin toxicity? à hypokalemia (digoxin normally binds to

extracellular K+ binding site on the cell, so low K+ means more digoxin binding).

- What are the Ia Na channel blockers? à “the Queen Proclaims Diso’s pyramid.” à Quinidine,

Procainamide, Disopyramide à you need to know quinidine causes cinchonism (think

“quinchonism”), which is tinnitus + headache à procainamide causes DILE (drug-induced lupus

erythematosus, with anti-histone antibodies; really HY!) à procainamide is the answer on the USMLE

for the drug used to Tx Wolf-Parkinson-White syndrome (delta wave on ECG) à disopyramide is

ultra-LY and unlikely to show up, but it’s mentioned in nearly every resource and completes the

mnemonic well à the Ia Na channel blockers increase the risk of torsades de pointes (TdP; a

sinusoidal ventricular arrhythmia with high chance of progression to VF and death) à they also

increase the action potential duration.

- What are the Ib Na channel blockers? à “I’d buy Liddy’s Mexican Tacos.” à Lidocaine, Mexiletine,

Tocainide à highest yield detail is that they cause “CNS stimulation/depression”; in other words,

you’ll get a Q where a patient was started on an anti-arrhythmic and gets, e.g., delirium, and then

they’ll ask you for the drug, which will be, e.g., mexiletine à these agents preferentially act on

ischemic tissue à Ibs shorten the action potential duration.

- What are the Ic Na channel blockers? à Flecainide, Encainide, Propafenone à highest yield detail is

that flecainide is the first-line anti-arrhythmic to Tx atrial fibrillation in the absence of structural or

coronary artery disease à Ics do not change action potential duration à do not use post-MI.

MEHLMANMEDICAL.COM 56
MEHLMANMEDICAL.COM

- What are the type II anti-arrhythmics? à beta-blockers à HY mnemonic for agents that antagonize

beta-1 only are “A BEAM of beta-blockers” à Atenolol, Bisoprolol, Esmolol, Acebutolol, Metoprolol

à metoprolol used largely as rate control for atrial fibrillation + flutter.

- What are the beta-blockers that also act on alpha-receptors? à carvedilol + labetalol antagonize both

alpha- and beta-receptors.

- Which beta-blockers decrease mortality in heart failure? à nebivolol, carvedilol, bisoprolol,

metoprolol XR.

- Notable uses for propranolol? à essential tremor (AD/familial, bilateral resting tremor in young

adults); migraine prophylaxis (on FM CMS/NBME, is the answer in patient with HTN + migraines);

tachycardia in hyperthyroidism (beta-blockade decreases peripheral conversion of T4 to T3);

esophageal varices prophylaxis (decreases portal blood flow); akathisia (due to anti-psychotics);

HOCM (increase end-diastolic filling à decrease murmur); social phobia; infantile hemangiomas (only

if patient with severe complications).

- Notable use of timolol? à topical solution used for glaucoma à decreases aqueous humour

production.

- Notable side-effects of beta-blockers? à patients with depression + sexual dysfunction.

- Contraindications to beta-blockers à severe lung disease (asthma w/ Hx of hospitalization or O2 use;

COPD); severe or psychotic depression; 2nd or 3rd degree heart block; symptomatic bradycardia; use

cautiously in severe diabetes due to masking of hypoglycemic events.

- Tx for beta-blocker overdose? à glucagon.

- Important point about BB in relation to pheochromocytoma? à Never give beta-blocker before

alpha-blocker; always give phenoxybenzamine (irreversible alpha-blocker) before beta-blocker in

pheo.

- What are the type III anti-arrhythmics? à potassium channel blockers à amiodarone, dronedarone,

sotalol, ibutilide, dofetilide à often used in ventricular arrhythmias and in patients being defibrillated

unsuccessfully à can be used in atrial fibrillation for rhythm control in those with structural heart

disease who cannot take flecainide.

MEHLMANMEDICAL.COM 57
MEHLMANMEDICAL.COM

- Points about amiodarone? à causes TdP, pulmonary fibrosis, drug-induced thyroid dysfunction

(~40% iodine by weight), corneal deposits, blue-grey skin discoloration à do PFTs, TFTs, LFTs before

commencing amiodarone.

- What are the type IV anti-arrhythmics? à non-dihydropyridine calcium channel blockers à

verapamil, diltiazem à verapamil is aka non-dihydropyridine CCB à acts selectively on cardiac

(nodal) tissue; in contrast, dihydropyridines like nifedipine act on vascular smooth muscle; diltiazem is

said to be a mixed agent à verapamil’s notable use is for AF rate control in patients who cannot

receive beta-blockers à verapamil causes constipation (really HY); it can also cause

hyperprolactinemia; diltiazem has occasional utility in the Tx of anal fissure and achalasia.

- What is ivabradine? à inhibits myocardial “funny” Na channels (phase IV of AP) à may be attempted

in select patients with stable angina who cannot take beta-blockers à can cause “luminous

phenomenon,” which is brightness in an area of the visual field.

- What is sacubitril? à neprolysin inhibitor (now you’re like, “wtf is neprolysin?”) à neprolysin is an

enzyme that breaks down ANP and BNP à therefore sacubitril can be used as an antihypertensive to

allow the kidney to excrete sodium and water à sacubitril usually used in combination with valsartan

as sacubitril/valsartan in the Mx of heart failure (i.e., can be used in place of ACEi or ARB

monotherapy during the initial Tx of HF).

- Tx of heart failure? à Start with ACEi, ARB, or sacubitril + ACEi/ARB (beta-blocker first is wrong

answer on USMLE) à then make sure patient is euvolemic with furosemide à then add beta-blocker

(metoprolol XR, bisoprolol, carvedilol, or nebivolol; these four decrease mortality in HF) à if EF still

low, add spironolactone à if EF still low, add COMBO of hydralazine + nitrates (combo decreases

mortality, especially in African Americans; USMLE Q will mention CHF patient on like 12 meds, and

you’ll see hydralazine is one of them, which isn’t typical, and then they’ll ask how the pharm regimen

can be modified to decrease morbidity / risk of mortality, and the answer = “add isosorbide ditrate”)

à if EF still low, add digoxin à if EF still low, use implantable defibrillator.

- Super important point about above sequence? à Digoxin and furosemide DO NOT decrease

mortality (USMLE is obsessed with that point).

- What is ranolazine? à sodium channel blocker used rarely for angina.

MEHLMANMEDICAL.COM 58
MEHLMANMEDICAL.COM

- Mechanism of nitrates? à NO activates guanylyl cyclase à increased cGMP production à increased

protein kinase G à increased dephosphorylation of myosin light-chain kinase à vascular smooth

muscle relaxation à work on veins >> arteries à venodilatation is main effect à must not use with

sildenafil (sildenafil inhibits PDE-5 à prevents breakdown of cGMP à severe hypotension with

nitrates).

- What about sodium nitroprusside? à works on both arteries and veins à used in hypertensive

emergencies à can cause cyanide toxicity à if patient with HTN emergency gets confusion after

administration of SN, the answer = cyanide toxicity; if patient has confusion before administration,

answer = hypertensive encephalopathy.

- What is fenoldopam? à dopamine 1 (D1) receptor agonist à dilates both renal afferent and efferent

arterioles à used to maintain renal perfusion in hypertensive emergency.

- 66M + back pain + hypercalcemia + renal insufficiency + Q shows you a pic of a white, fibrotic-

appearing heart; Dx? à answer = cardiac amyloidosis à seen classically in multiple myeloma (renal

insufficiency is often renal amyloidosis).

- Hypovolemic shock arrows à CO down, VR down, TPR up, PCWP down (or normal).

- Cardiogenic shock arrows à CO down, VR down, TPR up, PCWP up.

- Septic + anaphylactic shock arrows à CO up, VR up, TPR down, PCWP normal.

- Neurogenic shock + adrenal crisis arrows à CO down, VR down, TPR down, PCWP normal.

- “Can you explain ‘autoregulation’?” à Carotid sinus baroreceptors (think “sinus pressure”) are

stretch-dependent (higher BP = more stretch = more firing of CN IX; lower BP = less stretch = less

firing of IX) à so if, for instance, there is high BP, we get increased CN IX (glossopharyngeal) afferent

firing to solitary nucleus (nucleus solitarius) of the medulla à result is increased CN X

parasympathetic efferent firing + to atria (M2 receptors) to slow HR + decreased sympathetic outflow.

- “I’m still a little confused though. If patient is hypovolemic, then what happens in terms of

autoregulation?” à decreased stretch of carotid sinus baroreceptors à DECREASED afferent firing of

CN IX à decreased efferent CN X firing à decreased ACh binding to M2 receptors at atria + increased

sympathetic outflow à HR increases.

- Mediator that causes cardiac pain? à adenosine.

- Main regulators of coronary blood flow? à O2, CO2, adenosine, NO.

MEHLMANMEDICAL.COM 59
MEHLMANMEDICAL.COM

- Main regulators of skeletal muscle blood flow? à “CHALK” à CO2, H+, adenosine, lactate, K+.

- Q asks if coronary blood flow is greater or equal during systole or diastole when person is at rest vs

exercising à answer = coronary blood flow is: diastole > systole regardless if patient is at rest or

exercising.

- ST-elevations in leads II, III, aVF; which vessel is affected? à answer = posterior descending artery à

supplies inferior portion of the heart à sometimes the answer will just be straight-up “right coronary

artery” (~70-80% of the time the PDA comes off the RCA).

- ST-elevations in leads V1, V2, V3; which vessel is affected? à left anterior descending artery (LAD;

aka anterior interventricular artery) à supplies anterior heart.

- ST-depressions in leads V1, V2, V3; which vessel is affected? à posterior descending artery à

posterior infarcts can present as reciprocal ST-depression in the anterior leads.

- ST-elevations in V4, V5, V6; which vessel is affected? à left circumflex artery (LCx) à supplies left

lateral heart.

- Hypokinesis of the apex of the heart on echo; which vessel is affected? à LAD à supplies apex.

- Wide-complex tachyardia à ventricular tachycardia (VT).

- Narrow-complex tachy à SVT.

- Tx for SVT à vagal/carotid massage (“vagal maneuvers”) first; if doesn’t work, then adenosine. On

the USMLE, they will mention a carotid stretch having occurred (e.g., a wrestler has pressure applied

against his neck) followed by low HR, and the answer = “increased cardiac parasympathetic activity”

(sounds a bit misleading as the effect is due to the CN IX à CN X loop starting from the carotid sinus

baroreceptors, but I’m quoting the NBME).

- Tx of VT à anti-arrhythmics, e.g., amiodarone.

- Tx of SVT or VT in setting of coma / unconsciousness à direct-current countershock.

- Tx of first-degree heart-block or second-degree Mobitz I (Wenckebach) à observe.

- Tx of second-degree Mobitz II or third-degree heartblock à pacemaker.

- First-degree heart-block à PR-interval >200 milliseconds.

- Mobitz I à gradually prolonging PR-interval before a QRS drops.

- Mobitz II à no gradual prolongation of PR-interval; QRS randomly drops.

MEHLMANMEDICAL.COM 60
MEHLMANMEDICAL.COM

- Third-degree à HR super slow at 30-40; no relation between p-waves and QRS complexes.

- Infective causes of third-degree à Lyme disease, congenital lupus, diphtheria.

- Mechanism for Goodpasture syndrome? à antibodies against type IV collagen à “2, 3, 4… 2, 3, 4… 2,

3, 4. The Goodpasture is marching in the field, 2, 3, 4!ӈ Type 2 hypersensitivity against the alpha-3

chains of type 4 collagen.

- Hematuria + hemoptysis + “head-itis” (mastoiditis, sinusitis, otitis, nasal septal perforation) à

Wegener granulomatosis.

- Annoying new name for Wegener à granulomatosis with polyangiitis.

- Dx of Wegener à c-ANCA (anti-PR3; anti-proteinase 3).

- Asthma + eosinophilia à Churg-Strauss.

- Annoying new name for CS à eosinophilic granulomatosis with polyangiitis.

- Dx of CS à p-ANCA (anti-MPO; anti-myeloperoxidase).

- Hematuria in isolation + p-ANCA in serum à microscopic polyangiitis (MP).

- Severe renal disease in Wegener or Goodpasture or MP à rapidly progressive glomerulonephritis

(crescentic).

- 44M + hematuria + hemoptysis à Goodpasture syndrome.

- 44M + hematuria + hemopytisis + head-itis à Wegener.

- What is polyarteritis nodosa à medium-vessel vasculitis characterized by immune complex

deposition in vascular walls and fibrinoid necrosis.

- Polyarteritis nodosa is associated with what infection? à 30% of patients are HepB positive.

- What do you see on renal artery angiogram in PN à “beads on a string” (similar to fibromuscular

dysplasia, although completely unrelated diseases).

- 30M + red eyes + hearing loss / tinnitus / vertigo; Dx? à Cogan syndrome (rare vasculitis).

- Kid jumps into cold lake; what happens to (arrows) central blood volume, ADH, and ANP levels? à

answer = central blood volume increases, ADH decreases, ANP increases à cold means increased

alpha-1 agonism in arterioles to constrict distally to retain heat à increased blood volume in large

arteries à increased right atrial filling à increased ANP release; increased central blood volume also

increases baroreceptor activity at carotid sinus à not only leads to increased activity of the

MEHLMANMEDICAL.COM 61
MEHLMANMEDICAL.COM

autoregulation CN IX/X loop, but also suppresses ADH release to decrease free water reabsorption in

the medullary collecting duct of the kidney.

- Medial malleolus ulcer + hyperpigmentation of lower legs; Dx? à chronic venous insufficiency

- Punched-out ulcer on foot + intermittent claudication; Dx? à arterial insufficiency (peripheral

vascular disease)

- What causes venous insufficiency? à valvular incompetence (most commonly familial), resulting in

venous reflux + insufficiency.

- What causes arterial insufficiency à atherosclerosis (diabetes, followed by smoking, are the two

most acceleratory risk factors; hypertension is the most common risk factor)

- How do you Dx venous insufficiency? à duplex ultrasound of the calves showing stasis and/or

occlusive disease (the latter may result from venous insufficiency or cause it)

- How do you Dx arterial insufficiency? à USMLE always wants ankle-brachial indices (ABI) first à

after this is done, the answer is exercise stress test (to determine exercise tolerance), followed by

recommend an exercise/walking program. Do not choose arteriography or cilostazol immediately

after the ABI.

- Tx for venous insufficiency à compression stockings

- Tx for varicose veins à compression stockings

- Varicose veins and venous insufficiency same thing? à varicose veins are one of the mere

presentations of venous insufficiency, so yes, patients with varicose veins have venous insufficiency.

- 47F has varicose veins + painful palpable cord by the ankle (is the treatment compression stockings or

subcutaneous enoxaparin; both are listed) à answer = subcutaneous enoxaparin because this is

superficial thrombophlebitis.

- Tx for arterial insufficiency à exercise regimen first, THEN cilostazol (phosphodiesterase 3 inhibitor)

- What must you do before starting the exercise regimen in the Tx of arterial insufficiency à ECG stress

test to ascertain patient’s exercise tolerance.

- What is patient has abnormal baseline ECG (e.g., BBB) à do echo stress test instead.

- What if the patient can’t exercise à do dobutamine-echo stress test

- What if the patient gets stable angina after merely walking up a flight of stairs à skip stress test and

go straight to myocardial perfusion scan (myocardial scintigraphic assay); this is answer on the NBME.

MEHLMANMEDICAL.COM 62
MEHLMANMEDICAL.COM

- Patient has severe ischemia on stress test or myocardial perfusion scan à do coronary angiography

à then do coronary artery bypass grafting if three-vessel disease, OR two-vessel disease + diabetic,

OR single-vessel disease if it’s the left main coronary.

- Patient with CVD is on various medications + has hyperkalemia; why? à ACEi, ARB, and

spironolactone all can cause hyperkalemia.

- Patient with CVD is on various medications + hypokalemia; why? à furosemide (Loop diuretic)

- When do we start patients on furosemide? à to fluid unload (dyspnea in heart failure or peripheral

edema)

- Patient is started on furosemide + still has fluid overload; what’s the next diuretic to use à

spironolactone (this is really HY on the USMLE and is on Steps 1 and 2CK NBMEs) à essentially

furosemide causes increased K wasting, so we must give a potassium-sparing diuretic to balance the

effect (spironolactone).

- What’s the MOA of spironolactone à aldosterone receptor antagonist.

- Side-effects of spironolactone à hyperkalemia; gynecomastia.

- When do we give patients spironolactone apart from as a step-up from Loops? à added onto heart

failure management after a patient is already on ACEi (or ARB) + beta-blocker. In other words, for

heart failure: give ACEi (or ARB) first, then add beta-blocker, then add spironolactone.

- Major side-effects of beta-blockers à depression + sexual dysfunction (avoid in these patients)

- Major side-effect of naproxen à fluid retention (edema) due to increased renal retention of sodium.

- What is naproxen? à NSAID that the USMLE is obsessed with for some reason.

- Why might NSAIDs cause fluid retention / renal retention of sodium? à knocking out COX à

decreased prostaglandin synthesis à decreased renal afferent arteriolar dilatation à decreased

renal blood flow à PCT of kidney compensates for perceived low blood volume by increasing Na

reabsorption à water follows sodium à edema.

- Most common cause of carotid plaques? à HTN à the strong systolic impulse from the heart pounds

the carotids --> endothelial damage --> atherosclerosis.

- 55M + BP 150/90 + TIA; next best step in Mx? à carotid duplex USS à the first thing you want to

think about is, "does this guy have a carotid plaque that has resulted in a clot embolizing to his brain."

MEHLMANMEDICAL.COM 63
MEHLMANMEDICAL.COM

- 80M + good blood pressure (e.g., 110/70) + stroke or TIA; next best step in Mx? à ECG à you want

to think, "Does he have atrial fibrillation with a LA mural thrombus that's now embolized to the

brain."

- 80M + good blood pressure (e.g., 110/70) + stroke or TIA + ECG shows sinus rhythm with no

abnormalities; next best step in Mx? à Holter monitor à when you first see this scenario you're

probably like, "Wait, the ECG is normal, so it's not AF?" à No, it is likely AF, but AF is often

paroxysmal, so in order to detect it in this scenario, the next best step is a Holter monitor (24-hour

wearable ECG). This means that later in the day when he sits down to have dinner and then pops into

AF, the Holter monitor will pick it up.

- What % of people over age 80 have AF? à 8% of people over age 80 have AF, which is why age is a

huge risk factor. In other words, if the vignette says the guy is 58, AF is probably less likely just based

on shear probability, regardless of hypertensive status." And, once again, knowing that AF is often

paroxysmal is really important.

- Age 50s-60s + high BP + TIA/stroke/retinal artery occlusion; next best step in Dx? à answer = carotid

duplex ultrasound to look for carotid plaques.

- Age >75 + good BP + TIA/stroke/retinal artery occlusion; answer = ECG to look for AF à if normal, do

Holter monitor to pick up paroxysmal AF.

- 55M + good BP + carotid bruit heard on auscultation; next best step in Mx? à answer = carotid

duplex ultrasound to look for carotid plaques à in this case, if they are obvious and explicit about the

suspected etiology of the stroke, TIA, or retinal artery occlusion, then you can just do the carotid

duplex ultrasound.

- How to Mx carotid plaques? à first we have to ask whether the patient is symptomatic or

asymptomatic. A bruit does not count as symptoms (that's a sign). Symptomatic means stroke, TIA, or

retinal artery occlusion. According to recent guidelines: carotid occlusion >70% if symptomatic, or

>80% if asymptomatic à answer = do carotid endarterectomy. Below these thresholds à answer =

medical management = statin, PLUS clopidogrel OR dipyridamole + aspirin. The USMLE will actually

not be hyper-pedantic about the occlusion %s (that’s Qbank). They'll make it obvious for you which

answer they want. They'll say either 90% à answer certainly = carotid endarterectomy, or they'll say

MEHLMANMEDICAL.COM 64
MEHLMANMEDICAL.COM

50% à answer = medical management only. There’s one NBME Q where they say a guy has a bruit

but is asymptomatic, and has 10 and 30% occlusion in the left vs right carotids, respectively, and he’s

already on aspirin + statin, and the answer is "maintain current regimen” à if he were symptomatic,

even with low occlusion, he’d certainly need statin, PLUS clopidogrel OR dipyridamole + aspirin.

- How to Tx AF? à we have to consider both arms of management: blood thinning + treating the actual

AF. For blood thinning, CHADS2 score is standard in terms of evaluating risk (there are variants, but

the USMLE won't ever be borderline with how this plays into a question; they'll either give you a full-

blown obvious high-risk patient where all are positive, or they'll make it clear that the patient is low-

risk and merely just has AF alone).

o CHADS2 = CHF, HTN, Age 75+, Diabetes, Stroke/TIA (latter is 2 points; the rest are 1 point).

o If 0 or 1 points, give aspirin (anti-platelet therapy).

o If 2+ points, give warfarin (anti-coagulation therapy).

o If valvular AF (i.e., AF in someone with a mitral or aortic valve lesion), answer = warfarin.

o If non-valvular AF, can give other agents (e.g., dabigatran, apixaban).

- For the actual Tx of the AF, we do rate control before rhythm control (the management is actually

heavily involved, but for the USMLE know the following):

o Rate control: beta-blocker first-line (metoprolol). If beta-blocker avoided (i.e., severe or

psychotic depression, sexual dysfunction, COPD, Hx of asthma requiring oxygen or

hospitalization, 2nd/3rd-degree heart block), verapamil is the next choice. If rate control

fails, go to rhythm control.

o Rhythm control: Flecainide (type-Ic Na channel blocker) first-line in those without any

structural (i.e., LVH or valvular problems) or coronary artery disease (any symptomatology of

CVD or PVD means patient has coronary artery disease). In those who cannot receive

flecainide, other anti-arrhythmics like amiodarone, dronedarone, and dofetilide may be

used.

- 68F + diabetic + diffuse, dull abdo pain 1-2 hours after meals; Dx? à chronic mesenteric ischemia due

to atherosclerosis of SMA or IMA, not duodenal ulcer (if they want the latter, they’ll say 29M from

Indonesia) à essentially stable angina of the bowel.

MEHLMANMEDICAL.COM 65
MEHLMANMEDICAL.COM

- 68F + Hx of intermittent claudication + CABG + abdo pain 1-2 hours after eating meals; Dx? à chronic

mesenteric ischemia.

- 78M + Hx of AF + acute-onset severe abdo pain “out of proportion to physical exam”; Dx? à acute

mesenteric ischemia due to embolus.

- 16F + Hx of severe anorexia + BMI of 14 + has episode of ventricular fibrillation due to hypokalemia +

now has severe abdo pain; Dx? à acute mesenteric ischemia due to episode of decreased blood flow

(should be noted that hypokalemia causing arrhythmia is most common cause of death in anorexia).

- 68F + diabetic + Hx of diffuse, dull abdo pain 1-2 hours after meals + now has 2-day Hx of severe abdo

pain out of proportion to physical exam; Dx? à acute on chronic mesenteric ischemia due to

ruptured atherosclerotic plaque (akin to an “MI” of the bowel).

- Dx of acute + chronic mesenteric ischemia? à USMLE answer = mesenteric angiography.

- Tx of acute mesenteric ischemia? à endarterectomy might be able to restore blood flow if caught in

time, but on the USMLE, they will say “IV antibiotics are administered; what’s the next best step in

Mx?” and the answer is just “laparotomy.”

- Tx of chronic mesenteric ischemia à endarterectomy to clear vessel.

- 45M + has FHx of DCM + cirrhosis + generalized hyperpigmentation; his heart may show accumulation

of what? à answer = iron (hemochromatosis; AR; chromosome 6, HFE gene) à “bronze diabetes” à

1) increased glucose levels + 2) hyperpigmentation + 3) other miscellaneous finding like

cardiomyopathy, or pseudogout, or infertility.

- 92F dies in her sleep; heart is most likely to show what on biopsy? à lipofuscin à yellow-brown “age

pigment” that reflects autodigestion of intracellular lipid residues by lysosomal enzymes.

- Post-MI dyspnea; mechanism for fluid in lungs? à answer = “increased pulmonary capillary pressure”

à the wrong answer is “increased permeability of pulmonary capillaries.” The former refers to

transudate; the latter refers to exudate.

- Left dominant coronary circulation in someone with STEMI in leads II, III, and aVF; what is the

pathway of vessels here? à firstly, need to identify this as inferior infarct; then identify that the

posterior descending artery (PDA) supplies the inferior portion of the heart; Q gives you “LCA,” “LCx”

and “PDA” as answers in different order; answer = LCA à LCx à PDA (left-dominant means must

start with LCA; we know PDA finishes the supply, so LCx must be in the middle).

MEHLMANMEDICAL.COM 66
MEHLMANMEDICAL.COM

- Patient has end-systolic volume of 100 mL + end-diastolic volume is 190 mL; what’s the ejection

fraction in terms of %? à answer = (EDV – ESV / EDV) = (190 – 100 / 190) = 90/190 = 47.3% à USMLE

will ask you to calculate.

- What is normal ejection fraction? à 55-70%.

- When might the Q say the EF is high? à sometimes in high-output cardiac failure due to AV shunts,

e.g., in Paget disease of bone, hereditary hemorrhagic telangiectasia (pulmonary AVM), or patient

with previous dialysis + shunt.

- 29F + SOB with exertion + S3 heart sound + laterally displaced apex beat + CXR shows cardiomegaly +

Kerley B lines + TTE shows EF of 30%; Dx? à answer = CHF à you’re supposed to say, “Wait, in a

young patient without any specific disorder? And they don’t mention pregnancy either?” à but the

rest of the presentation is overwhelming, so you need to know this is ultra-classic / HY for CHF.

- NBME Q on CHF asks for (up or down) for LV stroke volume, LA pressure, and TPR à answer = LV

stroke volume down; LA pressure up; TPR up.

- When to start a statin? 2020 guidelines:

o LDL >190 mg/dL in anybody age 20-39.

o LDL >100 mg/dL in anybody age 40-75 (literature says 70-100, but USMLE wants >100).

§ Additional risk factors in this group change intensity of statin, not whether the

patient is commenced on one; so essentially the new guidelines suggest practically

all people should receive statins once they hit age 40, since it’s absurdly rare that

anyone has natural LDL <70 mg/dL.

o LDL >100 mg/dL in diabetics age 20-39.

§ In other words, non-diabetics age 20-39 don’t need a statin unless LDL >190 mg/dL.

o Patients age <20 if familial dyslipidemia with elevated LDL.

o Patients >75 engage in “discussion” with the physician regarding discontinuing their statin,

maintaining their dose, or lowering their dose.

§ Evidence of link between high cholesterol and morbidity/mortality in older

population not as clear as patients <75.

- Empiric Tx for community acquired pneumonia (CAP) (outpatient)? à macrolide (azithromycin).

MEHLMANMEDICAL.COM 67
MEHLMANMEDICAL.COM

- Empiric Tx for CAP (outpatient) if patient has been on Abx past three months? à respiratory

fluoroquinolone (e.g., levofloxacin).

- Can patient in hospital be treated for CAP, or do we treat special as hospital-acquired pneumonia

(HAP) or ventilator-acquired pneumonia (VAP)? à pneumonia is considered CAP if patient’s

presentation is <48 hours of admission to hospital.

- Empiric Tx for CAP (inpatient, non-ICU)? à fluoroquinolone, OR beta-lactam + macrolide.

- Empiric Tx for pneumonia (inpatient, ICU)? à beta-lactam, PLUS either macrolide or fluoroquinlone.

- Empiric Tx for HAP / VAP? à must cover for MRSA + Pseudomonas à vancomycin or linezolid, PLUS

anti-Pseudomonal agent(s) (pipericilin + tazobactam combo; ceftazidime; cefepime; meropenem;

amikacin; etc.).

- 56M + MI + has coronary angioplasty to restore blood flow + now has idioventricular arrhythmia;

why? à answer = “generation of reactive oxygen species.”

MEHLMANMEDICAL.COM 68
MEHLMANMEDICAL.COM

YouTube
@mehlmanmedical

Instagram
@mehlman_medical

MEHLMANMEDICAL.COM 69
MEHLMANMEDICAL.COM

MEHLMANMEDICAL
HY CARDIO

All material is copyrighted and the property of mehlmanmedical.

Copyright © mehlmanmedical

MEHLMANMEDICAL.COM 70
MEHLMANMEDICAL
HY PULMONARY
MEHLMANMEDICAL.COM

YouTube
@mehlmanmedical

Instagram
@mehlman_medical

MEHLMANMEDICAL.COM 2
MEHLMANMEDICAL.COM

HY Pulmonary – by Dr Mike Mehlman

HY Pulmonary cell types + path points


- Simple squamous; flat and long in appearance; carries out gas exchange; 95%
surface area of the alveoli. Although Step 1 is P/F now, USMLE is known to
occasionally give a fuck and show alveolar histo, so just be able to identify the
basic cell types.

Type I pneumocyte

- You say, “No idea what I’m looking at.” Notice how the type I pneumocytes
appear long and flat. The type II pneumocytes and macrophages are bulkier.
- Emphysema is destruction of alveolar surface area (i.e., mostly destruction of
type I pneumocytes) à ¯ surface area for gas exchange à chronic CO2 retention
à chronic respiratory acidosis (high bicarb to compensate).
- Two functions on USMLE:
- 1) Stem cells of the lung that proliferate in response to damage (i.e., if Q says
type I pneumocytes are damaged + ask what cell is responsible for replenishing
Type II pneumocyte
them, the answer is type II pneumocyte).
- 2) Contain specialized organelles called lamellar bodies that produce surfactant.
- In neonatal respiratory distress syndrome (NRDS), there are ¯ lamellar bodies.
- Present throughout the respiratory tree from the nasopharynx down to the
respiratory bronchioles.
Pseudostratified - USMLE wants you to know this is the primary cell that’s fucked up in smoking.
columnar epithelial - Even though function of other pulmonary cell types will also be diminished, this
cells with cilia is specifically the answer on the NBME.
- In other words, you might think that activity of this cell type might be ­ in
smoking in order to ­ particulate clearance, but activity is actually impaired.
- The answer on USMLE for the cell that initiates pulmonary fibrosis. For example,
they’ll show image of ferruginous body in asbestosis + ask which cell initiates
pulmonary fibrosis in this patient à answer = macrophage.
Alveolar macrophage - The cell that contains TB during an infection.
- Activity is ¯, not ­, in smokers. Similar to the pseudostratified columnar ciliated
cells, even though we might expect activity to ­ in order to clear out particulates,
the latter actually impair the function of alveolar macrophages.
- NBME Path questions are obsessed with you knowing integrity of basement
Basement membranes membranes must be maintained for restoration of normal pulmonary
architecture following infection or trauma.

MEHLMANMEDICAL.COM 3
MEHLMANMEDICAL.COM

- Q might say patient has pulmonary abscess treated successfully + year later
there is lesion still seen; why? à answer = “failure of maintenance of basement
membranes.”
- Or, the Q says patient has pneumonia + simply asks what enables for there to be
complete restoration of pulmonary architecture à answer = “maintenance of
basement membranes.” It’s important I’m repetitive on this point because it
shows up repeatedly on the Step 1 NBME forms.
- Aka neuroendocrine cell of the respiratory tract.
Kulchitsky cell
- This is the cell that is the origin for small cell lung cancer.

Obstructive vs restrictive lung disease


Obstructive Restrictive
- Asthma. - Pulmonary fibrosis due to any cause
- COPD (chronic bronchitis + emphysema). (i.e., idiopathic; radiation; drugs; auto
HY conditions
- Old age (i.e., healthy person age 70 vs 20) immune disease).
- Pneumoconioses.
FEV1 ¯ ¯/«
FVC ¯ ¯
FEV1/FVC ¯ (<70%; normal is 70%) ­/« (>70-80%)
TLC ­ ¯
- The above lung volumes are the HY ones you need to know for USMLE.
- FEV1 = Forced Expiratory Volume in 1 second = amount of air patient can forcefully blow out in 1
second, starting from maximal inhalation.
- FVC = Forced Vital Capacity = amount of air patient can blow out starting from maximal inspiration
all the way until maximal expiration.
- The reason FEV1/FVC is ­/« in restrictive, as compared to ¯ in obstructive, is because of radial
traction, which is a “stickiness” on the outside of the airways (such as from fibrosis) that prevents
them from closing as rapidly as obstructive.
- TLC = Total lung capacity à increased in obstructive due to air-trapping (hence “obstructive”).

The below lung volumes are lower yield for USMLE. But I include them here for greater elucidation. For the
sake of passing the Step 1, it is the arrows at the top of this chart that are most vital.

TV « «

MEHLMANMEDICAL.COM 4
MEHLMANMEDICAL.COM

ERV ¯ ¯
IRV ¯ ¯
RV ­­ ¯
FRC (ERV +
­ ¯
RV)
DLCO ¯ (but ­ in asthma on USMLE) ¯ (« if neuromuscular)
- TV = Tidal Volume; generally preserved in lung diseases.
- ERV = Expiratory Reserve Volume = additional amount that can be forcibly exhaled following the end of a
normal exhalation.
- IRV = Inspiratory Reserve Volume = additional amount that can be forcibly inhaled following the end of a
normal inhalation.
- RV = Residual Volume = amount of air remaining in the lungs after the end of maximal expiration.
- FRC = Functional Residual Capacity = total amount of air remaining in lungs after end of normal expiration.

- Don’t freak out about the above flow-loops. For USMLE (all Steps), you just need to know their general
shape. I’d say the most important point is that the expiratory component of the obstructive curve (purple)
tends to have a scooped-out, or concave, or L-shape.

- What USMLE will do is give you a real bootleg Windows 95-type spirometry curve as per above in a 34-
year-old + no family Hx + 5-year smoking Hx. You look at curve and say, “No idea what I’m looking at, but
top of curve is concave, and I know that’s obstructive. So between COPD or asthma here, which are both
concave and hence possible answers, I’ll choose asthma as most likely in this particular patient.”

MEHLMANMEDICAL.COM 5
MEHLMANMEDICAL.COM

Shunt versus dead space


- Shunt = ¯ V/Q (ventilation/perfusion) à refers to almost all lung conditions on USMLE.
- Dead space = ­ V/Q à refers to basically just pulmonary embolism on USMLE.

- We use the term shunt to refer to ¯ V/Q (i.e., reduced ventilation relative to perfusion) to the point that
the patient’s arterial oxygen becomes decompensated (i.e., a shunt is always pathologic in pulmonary
terms), but ¯ V/Q can also be physiologic (i.e., normal / not a shunt) at the lung bases, where the V/Q is
normally 0.6, since gravity pulls greater perfusion to the bases.
- Dead space means ­ V/Q (i.e., low perfusion relative to ventilation), but need not be pathologic (i.e., there
is a natural ­ V/Q mismatch of ~3.0 at the lung apices. Types of dead space (asked on Step 1 NBME):
- Alveolar dead space = natural, physiologic ­ V/Q within the alveoli, where some areas of lung
receive more ventilation than perfusion – i.e., the apices (V/Q of ~3.0) compared to the bases
(~0.6).
- Anatomic dead space refers to parts of the respiratory tree that are naturally ventilated but do not
partake in gas exchange, such as the trachea, bronchi, and terminal bronchioles. The respiratory
bronchioles and alveoli partake in gas exchange, so the anatomic V/Q starts to decrease from the
level of the respiratory bronchioles distal.
- Physiologic dead space is the sum of anatomic and alveolar dead space. This reflects the alveolar
and respiratory tree dead space seen in healthy individuals.
- Pathologic dead space is seen classically in pulmonary embolism (as well as amniotic fluid, fat, and
air emboli), where blood flow is impeded by an embolus. V/Q will increase in an area of lung simply
because perfusion is blocked in that area. As you can see, this is quite distinct from physiologic
dead space.
-
- “Yeah but I’m still confused by shunt and dead space. Could you please elaborate a little more.” à A shunt
means a right to left effect of oxygenation due to insufficient alveolar ventilation – i.e., a mixing of
deoxygenated blood (right) with oxygenated blood (left) such that a patient’s net oxygenation is less than
what it should be because some of the alveoli aren’t receiving enough oxygen. This is different from a R to L
cardiac shunt, where deoxygenated blood in the right heart is literally mixing with oxygenated blood in the
left heart; in a pulmonary R to L shunt, deoxygenated blood from insufficiently oxygenated pulmonary
venules (coming from an area of lung that’s obstructed, e.g., from a peanut or mucous plugging) mixes with
oxygenated blood from adequately oxygenated pulmonary venules; this effect of deoxygenated blood being
averaged in with oxygenated blood creates a “R to L” effect; we call this R to L movement a shunt. This is
made more confusing by the existence of L to R cardiovascular “shunts,” which refer to pathologies such as
VSD, ASD, PDA, and AV fistulae, where the patient is not cyanotic/deoxygenated and has blood moving from
the left circulation to the right. However the term “shunt,” as applied to ventilation and perfusion in
pulmonology (i.e., when we say “what is shunt vs dead space?”) refers to a R to L process. It is not sufficient
to merely say a shunt is a R to L process, period, because pulmonary embolism (i.e., dead space, not shunt)
also ultimately results in deoxygenated blood mixing with oxygenated blood.

- “I’ve heard something about oxygen not helping in a pulmonary shunt. No idea what that means though.
Can you explain.” à A classic effect of a pulmonary shunt is an inability to effectively raise arterial pO2 even
when oxygen is administered. For instance, if a patient swallows a peanut, the area of lung blocked off
could be said to have a “zero” for oxygen (i.e., no ventilation). The result is: that zero is mixed in with all of
the other areas of normal lung à this means the average of all areas of lung cannot achieve normal
oxygenation because that zero is mixed in, so even if O2 is administered and the remaining alveoli are highly
ventilated and oxygenated, the net result is still an arterial oxygenation that is insufficient. We use the
peanut as an easy example to help visualize this process, but when this is applied to, e.g., COPD or asthma,
what occurs is many tiny alveolar pockets become obstructed, with lots of mini shunts being formed; so we
gets lots of tiny zeros throughout the lungs, where even if we give oxygen and many alveoli have high pO2,
the patient’s arterial pO2 remains low because the net oxygen from all of the alveoli combined is low. These
zeros being mixed in with normal lung reflects our R to L process (i.e., the shunt).

MEHLMANMEDICAL.COM 6
MEHLMANMEDICAL.COM

Annoying Alveolar-arterial (A-a) gradient stuff


- “Can you explain A-a gradient? Shitz got me on fire bro.” à A is alveolar oxygen; a is arterial oxygen. In
settings where the patient’s arterial oxygen is low, the A-a gradient (normally 5-10 mmHg) tells us whether
there’s a lung pathology impeding gas exchange or if the patient is merely hypoventilating.
- In other words, ­ A-a gradient = the lungs are fucked up.
- Normal A-a gradient = the lungs are normal; patient is just not breathing enough.
- A high A-a gradient means the patient has low arterial oxygen despite good alveolar oxygen (i.e.,
something is impeding gas exchange); normal A-a gradient means low arterial oxygen because of low
alveolar oxygen (i.e., the patient merely isn’t breathing enough).
- Probably one of the highest yield points is knowing that opioids, benzos, and barbiturates cause a normal
A-a gradient because these agents cause respiratory depression. That is, if a patient is on fentanyl for pain
following surgery and has a low arterial O2, we know the low arterial O2 is because the patient isn’t
breathing adequately, not because there is defective gas exchange. The question might not overtly tell you
the respiratory rate is 6/minute; they’ll sometimes say it’s 12 (normal 12-16), but the answer is still “normal
A-a gradient”; adequate depth of respirations also matters.
- USMLE also wants you to know that the mechanism for the patient’s hypoxemia in pulmonary edema is
“high A-a gradient”; this makes sense, as the transudate in the alveolar spaces impedes gas exchange, but
the patient’s ventilation is otherwise fine.
- Old age in healthy persons causes: ­ A-a gradient, ¯ arterial pO2, ­ TLC. This combo is answer on NBME –
i.e., even if people are non-smokers, as we live out life inhaling particulates from car exhaust, etc., the lungs
experience obstructive changes with age (essentially “very slow COPD”).
- My biggest advice is to not memorize lung pathology in general as synonymous with “high A-a gradient.”
The USMLE will slam students on normal A-a gradient à e.g., patient being weaned from a ventilator and
has pCO2 of 70 mmHg (normal 33-44) à answer = normal A-a gradient because the patient is merely
hypoventilating (low arterial O2 because alveolar O2 is low). The mistake I see students make is they just
think “lung problem = A-a gradient must be high.”

HY pulmonary / respiratory tract cancers for USMLE


- Small basophilic (purple) cells. NBME can describe these cancer cells as “twice the
size of lymphocytes.”

Small cell
bronchogenic
carcinoma

- Occurs centrally in the lung (i.e., hilar / medially). Smoking biggest risk factor.
- Paraneoplastic syndromes exceedingly HY: 1) SIADH (ADH secretion); 2) Cushing
syndrome due to ACTH secretion; 3) Lambert-Eaton syndrome (production of
antibodies against presynaptic voltage-gated calcium channels); 4) Cerebellar
dysfunction / ataxia (anti-Hu/-Yo antibodies).
- Treatment on 2CK is “chemotherapy.” Do not do surgery. I believe this is the only
time I’ve seen straight-up “chemotherapy” as correct answer on NBME.
Squamous cell - Stains positive for keratin; may have keratin pearls on histo (pink circles).
carcinoma

MEHLMANMEDICAL.COM 7
MEHLMANMEDICAL.COM

- Similar to small cell, occurs centrally in the lung (i.e., hilar / medially). Smoking
biggest risk factor. In other words, the two cancers that start with an “ssss” sound
for “central” (i.e., Small cell and Squamous cell) are Central.
- Can cavitate à if USMLE gives you lung cancer with a cavitation (i.e., cavity/hole),
the answer is squamous cell.

- Highest yield point is that it secretes PTHrp (parathyroid hormone-related


peptide). This acts like PTH and increases calcium / decreases phosphate, but this is
not the same as PTH. Endogenous PTH is suppressed due to negative feedback from
high calcium. In other words, choose a down arrow for PTH in squamous cell
carcinoma of the lung.
- Adenocarcinoma means cancer of glands. If the Q says something about biopsy
showing glandular morphology, you know they’re talking about adenocarcinoma.
- The answer on USMLE for lung cancer in a non-smoker; classically “female non-
smokers,” but I’ve seen NBME Qs with this in men.
- Normal ground/Earth radiation due to radon is accepted cause of lung cancer in
non-smokers. There is NBME Q where they mention non-smoker living in a
basement and develops lung cancer. The correlation is probably nonsense in real
Adenocarcinoma life, but it’s in an NBME question somewhere.
of the lung - Does not occur centrally on NBME exam, unlike small cell and squamous, and
hence will be described as apical or peripheral lung lesion in non-smokers.
- Apical tumors can are known as Pancoast tumors and cause Horner syndrome
(miosis, ptosis, anhidrosis) due to impingement on C8 superior cervical ganglia
(sympathetic nerves). They can also cause SVC syndrome (flushing of the face +
congestion of neck veins) or brachiocephalic syndrome (only right side of
face/neck) due to impingement on venous return. (+) Pemberton sign is worsening
of flushing + neck vein congestion when raising the arms above the head.

MEHLMANMEDICAL.COM 8
MEHLMANMEDICAL.COM

- Can be associated with migratory thrombophlebitis (Trousseau sign of


malignancy). The latter is not limited to head of pancreas cancer. Adenocarcinomas
in general are known to be associated with hypercoagulable state due to
malignancy. USMLE won’t ask specific mechanism, but liberation of tissue factor
(factor III) by these cancers is a proposed etiology.
- Associated with hypertrophic osteoarthropathy (clubbing + hand pain due to lung
cancer); mechanism is fibrovascular proliferation. Literature says adenocarcinoma
of lung is most common cause, although the association isn’t 100% specific.
- Nonexistent lung cancer on USMLE. Bogus/garbage diagnosis. I don’t think I’ve
Large cell carcinoma
ever seen this assessed.
- Pulmonary nodule that secretes serotonin or serotonin-like derivatives, resulting
in carcinoid syndrome (i.e., tachycardia, flushing, diarrhea).
Bronchogenic - A type of neuroendocrine tumor.
carcinoid tumor - Carcinoid tumors are classically appendiceal and of the small bowel, but you
should be aware that bronchogenic carcinoids exist.
- USMLE wants urinary 5-hydroxy indole acetic acid (5-HIAA) for initial step in Dx.
- Cancer of “mesothelial cells” (answer on NBME) seen in patients with prior
occupational exposure to asbestos.
- Asbestosis occurs first, which then gives rise to mesothelioma years later.
- Shipyard and construction workers are buzzy for prior occupational exposure.
- Asbestosis will be described as pleural or supradiaphragmatic plaques (“soft tissue
plaques seen on CXR”). Pulmonary biopsy shows ferruginous bodies.
- Calretinin (+); a protein that is highly indicative of mesothelioma on staining.
- Mesothelioma appears as a whiteish cancer and is described as an “encasing rind
of pleural-based tumor” (i.e., circumferentially surrounds/wraps around the lungs).

Mesothelioma

Nasopharyngeal
- Can be caused by EBV.
carcinoma
- Squamous cell carcinoma of vocal cords.
Laryngeal cancer - Smoking is major risk factor.
- New NBME Q wants you to know this spreads to cervical lymph nodes.
Laryngeal - Pediatric condition characterized by warts of the vocal cords.
papillomatosis

MEHLMANMEDICAL.COM 9
MEHLMANMEDICAL.COM

- Lesions will have papillary structures on biopsy.

- Due to HPV 6/11 exposure from maternal vaginal canal.

Pneumoconioses for USMLE


- As already mentioned above in the mesothelioma section, this is associated with
shipyard workers, construction workers, and electricians. It can give rise to
mesothelioma later in life.

Asbestosis

- The above ferruginous body is dumbbell-shape. Choose macrophage as the answer


on USMLE for the cell that initiates pulmonary fibrosis (in response to asbestos and in
general).
- Causes restrictive lung pattern.
- Occupational exposure to beryllium in the aerospace / aeronautical industry.
Berylliosis - Causes restrictive lung pattern.
- Can cause granulomas.
- Occupational exposure to silicon (sand) in foundry or stone quarry workers.
- Can cause egg-shelf calcifications in upper lobes.
Silicosis - Increases risk of tuberculosis infections.
- Avoid anti-TNF-a agents (i.e., infliximab, adalimumab, etanercept) in these patients
due to increased TB risk (TNF-a needed to suppress/fight TB).
- Aka “coal miner’s lung.”
Anthracosis - Black discoloration of the lung.
- Can be either obstructive or restrictive.
- Rheumatoid arthritis + any pneumonociosis, presenting as pulmonary nodules.
Caplan syndrome - Clinical relevance is that patients with RA are at increased risk for developing
pneumoconioses if they have a workplace exposure.

MEHLMANMEDICAL.COM 10
MEHLMANMEDICAL.COM

HY General lung conditions for USMLE


- The answer on USMLE for a patient over the age of 50 who has 6-12+
months of unexplained dry cough. This is how it shows up 4/5 times.
- Textbook restrictive lung disease, with « or ­ FEV1/FVC. The reason
for the FEV1/FVC being greater than in obstructive lung disease is radial
traction, as mentioned earlier.
- CXR and CT scan show “reticular” or “reticulonodular” pattern. These
descriptors are exceedingly HY on USMLE, where students will overlook
them in the vignette, but they are hugely buzzy for restrictive lung
disease. They are colloquially known as “honeycombing,” but I have not
seen the USMLE give a fuck about the latter colloquialism. They
frequently just say “reticular” and “reticulonodular,” and then you know
right away, “Boom. Restrictive lung disease,” i.e., fibrosis, etc.
Tangentially, it’s to my observation that NBME will say “reticulogranular”
frequently for NRDS, but the two vignettes are clearly disparate anyway.

- After the CXR and spirometry are performed, 2CK wants “high-
Idiopathic pulmonary fibrosis resolution CT of chest” as answer for next best step.
(Usual interstitial pneumonitis)

“Honeycombing” = reticular / reticulonodular pattern.

- New 2CK NBME wants “lung biopsy” as answer to confirm diagnosis of


interstitial lung disease (i.e., idiopathic pulmonary fibrosis) after imaging.
- Vignette can also mention loud P2 (means pulmonary hypertension)
with “dry inspiratory crackles heard bilaterally.”
- 1/10 times, the Q will be patient over 50 with increasing fatigue and
shortness of breath over 6-12 months, with no mention of cough, where

MEHLMANMEDICAL.COM 11
MEHLMANMEDICAL.COM

it initially sounds like heart failure, and they’ll say CXR shows “interstitial
markings” instead of reticular/reticulonodular patterning. However, they
say patient has “­ FEV1/FVC showing restrictive pattern” in the stem,
which gives it away.
- You need to know that “usual interstitial pneumonitis” (UIP) is another
name for idiopathic pulmonary fibrosis. Yes, the name is weird, but it’s
not my opinion and it’s asked twice on the NBMEs, where instead of
writing “idiopathic pulmonary fibrosis” as the answer, they write “usual
interstitial pneumonitis.” UIP is technically a broad term that can refer to
many restrictive lung conditions, but as I said, on NBME they use this
synonymously with idiopathic pulmonary fibrosis.
- COPD = chronic bronchitis + emphysema.
- Smokers will have combination of the two. When we say a smoker has
COPD, we are saying they have chronic bronchitis + emphysema at the
same time.
- The term COPD can in theory apply to any obstructive disease of the
lung that is chronic (e.g., asthma, Kartagener, etc.) but when the term is
used without any specific condition attached, it refers to the combo of
chronic bronchitis and emphysema.

COPD

- Hyperinflated lungs in COPD (due to air trapping) can push the heart to
the midline. NBME will say there’s a “long, narrow cardiac silhouette,” or
a “point of maximal impulse palpated in the sub-xiphoid space.” In left
ventricular hypertrophy, in contrast, there will be a lateralized apex beat,
or a point of maximal impulse in the anterior axillary line.
- Home oxygen is indicated on 2CK if O2 sats are:
- <88% saturation (55 mm Hg), or
- <89% saturation (60 mm Hg) if the patient has cor pulmonale.
- First-line Tx is now considered to be a long-acting muscarinic receptor
antagonist (i.e., LAMA such as tiotropium) or a long-acting b2 agonist
(i.e., LABA such as olodaterol), either alone or in combination.
- If insufficient, add an inhaled corticosteroid (e.g., fluticasone).
- Should be noted that a question on an earlier Family Med form has
ipratropium (SAMA) as the answer for 1st-line in COPD, but there aren’t
any other anti-muscarinic or b2 agonists listed. So the point is that if you
are forced to choose a SAMA or SABA (i.e., albuterol) on USMLE, these
are OK, but newer guidelines say start with a LAMA or LABA.
- “Exacerbation of COPD” as a diagnosis on USMLE will always give a
patient with high CO2. This is really important. In other words, if they

MEHLMANMEDICAL.COM 12
MEHLMANMEDICAL.COM

give you a big paragraph and you’re not sure of the Dx, if you see CO2 is
not elevated, the answer is not COPD exacerbation.
- COPD exacerbations can be triggered by minor chest infections, so
always give antibiotics on 2CK, even though etiology is usually viral.
- Patients with COPD are chronic CO2 retainers, so they will have a
chronic respiratory acidosis (i.e., ­ CO2 and ­ bicarb; pH can either be ¯
or compensated back into normal range).
- For 2CK, lung cancer screening with annual low-dose chest CT is done in
patients who meet all of the following:
1) age 50-80;
2) have 20-pack-year Hx of smoking; and
3) smoked within the past 15 years.
- Chronic bronchitis = productive cough for at least 3 months in a year,
for 2+ years.
- Reid index >0.5 (ratio of the thickness of bronchial mucous-secreting
glands to the bronchial wall itself). <0.4 is considered normal.
- Chronic bronchitis is known as “blue bloater” because the mucous sits
in the alveolar spaces and impairs gas exchange. This results in a shitload
of hypoxic vasoconstriction à patient can become acutely blue and
hypoxic during exacerbations.
- The hypoxic vasoconstriction of pulmonary vessels causes pulmonary
hypertension (i.e., if the vessels constrict, then pressure in the more
Chronic bronchitis proximal pulmonary arterioles increases). This increased afterload on the
right heart can lead to right ventricular hypertrophy and right heart
decompensation. When right heart failure (i.e., evidence of JVD or
peripheral edema) occurs due to a pulmonary cause, we now call that
cor pulmonale, as discussed in the Cardio PDF.
- The pulmonary hypertension can cause a loud P2 and tricuspid regurg
prior to cor pulmonale occurring.
- Don’t confuse chronic bronchitis with acute bronchitis, which can
present as a worsening cough in patient (with or without COPD)
following a viral infection. This is a temporary irritation/inflammation of
the airways that is self-resolving.
- Emphysema = loss/destruction of alveolar surface area.
- When alveolar surface area is reduced, so is alveolar capillary surface
area, since the capillaries are within the alveolar walls à ¯ gas exchange.
- Known as “pink puffer,” since although gas exchange is impaired,
sudden and acute increases in hypoxic vasoconstriction, as with chronic
Emphysema bronchitis, are not a feature.
- “Bullous changes” on CXR are synonymous with emphysema on USMLE.
- Smokers can centri-acinar emphysema (proximal alveolar structure is
destroyed).
- a1-antitrypsin deficiency patients get pan-acinar emphysema (entire
alveolus destroyed).
- Codominant genetic condition resulting in emphysema and hepatic
cirrhosis.
- ZZ allele combo is worst and results in disease (asked on USMLE).
- a1-antitrypsin is an enzyme produced by the liver that travels to the
lungs and breaks down neutrophilic elastase. Elastase is an enzyme that
a1-antitrypsin deficiency normally causes damage to the alveoli. Homeostatically, elastase is
required in small amounts for normal pulmonary function and cell
turnover, but in high amounts it destroys the alveoli, resulting in
emphysema.
- Vignette will give young adult who has sibling or parent who’s had
early-onset emphysema or cirrhosis.

MEHLMANMEDICAL.COM 13
MEHLMANMEDICAL.COM

- The USMLE Q can absolutely say the patient is a smoker or drinks


alcohol. For instance, they might say the patient is 34 and has been
smoking for 5 years and has bullous changes on CXR, or that the father
died from alcoholic cirrhosis, and the student thinks, “Oh that can’t be
a1-antitrypsin deficiency though because they said smoking/alcohol.”
But the key point is that this condition increases the risk for early-onset
emphysema and cirrhosis. Normally, COPD should take 20+ years of
smoking to develop, not 5.
- Bronchospasm that occurs either idiopathically/hereditarily, or in
response to certain allergens or cold air.
- One-third of patients with asthma only present with a dry cough and no
problems breathing. This is called cough-variant asthma. It will often
present as a patient with a dry cough that’s worse in the winter.
- Can also present as part of atopy constellation – i.e., dry cough in
winter, seasonal allergies / rhinoconjunctivitis / urticaria in spring, and
eczema in summer.
- Aspirin-induced asthma mechanism HY for USMLE à inhibition of COX
by aspirin à shunting of arachidonic acid down lipoxygenase pathway à
increased leukotrienes à increased bronchoconstriction.
- Samter triad = aspirin allergy, asthma (due to aspirin), nasal polyps.
- “Increased expiratory phase” is a buzzy phrase that will be thrown into
quite a few asthma vignettes. It is not specific for asthma and can refer
to any obstructive pathology, but I just make note of it here because
you’ll see it quite a bit for asthma and say, “What’s that mean?” à in
obstructive conditions, it takes us a lot longer to exhale (FEV1 is ¯¯).
- Acutely, asthma causes ¯ CO2, ­ pH, « bicarb. This is an acute
respiratory alkalosis. Even though the patient is having difficulty
breathing, CO2 diffuses quickly, whereas O2 diffuses slowly, so insofar as
the patient’s respiratory rate is ­, CO2 will be ¯. Bicarb is unchanged
because it takes the kidney about a day to alter excretion. In contrast,
CO2 is ­ in COPD because there is large amounts of hypoxic
vasoconstriction due to excessive mucous (chronic bronchitis) or ¯
Asthma
alveolar surface area, so CO2 can’t get out, even with faster breathing. In
asthma, alveolar surface area is intact, and the degree of mucous
production and hypoxic vasoconstriction is not nearly as bad as chronic
bronchitis.
- The combination of ¯ CO2 and ¯ O2 in acute asthma attack is known as
type I respiratory failure. Eventually, the patient will begin to fatigue and
breathing rate will slow. This will be observed as CO2 and pH rebounding
to normal levels despite O2 remaining low. This means the patient is in
transition to a type II respiratory failure (i.e., hypoventilation), where we
have ­ CO2 and ¯ O2. USMLE wants intubation as the intervention.
- 2CK Q gives vignette of asthma and then asks for best initial step in
diagnosis à answer = “spirometry.” The expiratory component of the
curve, as discussed earlier, will appear concave.
- Should be aware methacholine challenge can also be done to diagnose.
This is is a muscarinic agonist that bronchoconstricts and can induce
symptoms. Never give during acute episode. This can be tried between
episodes. But if you’re forced to choose diagnostic modality, go with
simple spirometry.
- USMLE cares about both outpatient management of asthma as well as
acute attacks.
- For outpatient management:
1) First-line Tx is b2-agonist (albuterol) for acute attacks.
2) If the Q says the patient has weekly episodes, or they ask for
what could decrease risk of future episodes if patient is already

MEHLMANMEDICAL.COM 14
MEHLMANMEDICAL.COM

on albuterol, the answer is inhaled corticosteroid (ICS; i.e.,


fluticasone).
3) If the combo of albuterol + ICS is insufficient, the next step is
increasing the dose of the ICS.
4) If that doesn’t work, adding a LABA (i.e., salmeterol) is the
next step.
5) If insufficient, agents such as mast cell stabilizers, leukotriene
blockers, etc., can be tried. If patient has Hx of aspirin allergy,
the latter in particular can be effective.
6) Last resort for outpatient asthma is oral prednisone. It is most
effective at decreasing recurrence of episodes, but we want to
avoid it if at all possible because of the risk of Cushing syndrome
and ¯ linear bone growth in Peds.
- Acute management of severe asthma attack (i.e., not outpatient) is
nebulized albuterol, oxygen, and IV methylprednisolone. Inhaled
corticosteroids have no role in acute asthma management.
- NBME for 2CK wants you to know that any asthma patient who requires
hospital management for an acute episode must automatically be given
ICS (i.e., fluticasone) on discharge. In other words, if patient isn’t
currently on an ICS, it must be commenced at discharge. I’m explicit here
because this particular NBME Q doesn’t mention albuterol anywhere in
the stem, but inhaled fluticasone is the answer.
- Another 2CK NBME Q gives a patient who is currently not receiving any
asthma medications but who gets 2+ episodes weekly à answer =
“inhaled fluticasone + albuterol.” Students think this is weird because
they jump straight to dual management + are even audacious enough to
put the ICS before albuterol in the wording of the answer, but it’s what
they want. Any patient with 2+ episodes per week needs ICS in addition
to the albuterol and can be commenced right away on dual therapy.
- Dilation of the airways (ectasia means dilation) that occurs due to “loss
of musculature of the airways.”
- Transverse CT scan will show cystic dilation of the airways.

Bronchiectasis

- Most common cause worldwide is TB. Most common cause in western


countries is CF. But on USMLE, it will usually be a smoker. Asthma does
not cause bronchiectasis.
- Presents almost always as “cups and cups of foul-smelling sputum.”
- “Foul-smelling” means anaerobes, such as Bacteroides.
- One question on 2CK Peds form gives bronchiectasis as the answer in a
child who has right middle lobe syndrome, where they say there is scant
white sputum (not cups and cups) and a thin, linear opacity visualized in
the right middle lobe. If you think it’s weird, complain to NBME not me.

MEHLMANMEDICAL.COM 15
MEHLMANMEDICAL.COM

- Clubbing tends to be seen in these patients. It’s not mandatory, but it’s
to my observation USMLE likes it for bronchiectasis.
- Confusing word that refers to “lung collapse,” or “collapse of alveoli.”
- Highest yield point for USMLE is that it is the most common cause of
fever within 24 hours of post-surgery. If this is the first time you’re
reading this, that might sound weird, but this is pass-level and extremely
important for 2CK.
- There is one 2CK Q I’ve seen where they say a woman had a C-section
two days ago and the answer was still atelectasis, so even though it’s
most common <24 hours, just be aware one Q exists where, oh em gee,
it’s 2 days later.
- The mechanism is related to combo of pain meds + sedentation, where
breathing becomes slower + shallower in hospital bed, leading to mild
collapsing of some alveoli. This is why breathing exercises can be
Atelectasis important post-surgery.
- Will often present as bibasilar shadows or opacities. In other words,
patient had surgery yesterday + now has fever + CXR shows mild opacity
at the lung bases à answer = atelectasis.
- Step 1 NBME assesses obstructive (aka resorptive) atelectasis. This is
when an area of lung distal to an obstruction from, e.g., a tumor, can
cause alveoli to collapse. This then increases the chance for pneumonia
distal to the obstruction.
- 2CK IM form has “endobronchial obstruction” as answer for distal area
of lung collapse (i.e., atelectasis) in patient with lung cancer; “vascular
occlusion by tumor” is wrong answer (makes sense, as the tumor
obstructs the respiratory tree, not blood vessel, in this case, but I’ve seen
students accidentally choose the latter).
- Can be obstructive (i.e., usually from obesity) or central (i.e., brain-
related). What USMLE wants you to know:
- These patients develop chronic respiratory acidosis – i.e., ­ CO2, ­
bicarb, pH «/¯.
- Cor pulmonale can occur as a result of pulmonary hypertension from
hypoxic vasoconstriction. JVD or peripheral edema will be seen with cor
pulmonale. Descriptors such as RBBB, right-axis deviation on ECG, and
Obstructive sleep apnea (OSA)
wide splitting of S2 all mean RVH. If the patient merely has pulmonary
hypertension but not yet cor pulmonale, the vignette can say loud P2 or
tricuspid regurg (holosystolic murmur that ­ with inspiration).
- Chronic fatigue and poor oxygenation can lead to dysthymia /
depression. The answer on NBME is “mood disorder due to a medical
condition.”
- Polysomnography (sleep study) is what USMLE wants to diagnose.
- Acute dyspnea and bilateral wheezes in patient who was gardening
(stung by a bee), who ate a particular food (e.g., peanuts), or who was
commenced on a recent drug (e.g., TMP/SMX).
- Vignette often gives tachycardia and low BP.
- Mechanism is IgE crosslinking on surface of mast cells and basophils
that leads to degranulation and histamine + prostaglandin release.
Eosinophils can be recruited in response.
Anaphylaxis
- USMLE wants: ¯ vascular resistance, ­ CO, ¯/« PCWP.
- Tx = intramuscular epinephrine à the strong b2-agonistic effect opens
the airways; the strong a1-agonistic effect constricts the arterioles and
restores BP.
- NBME 9 for 2CK asks, “In addition to self-injectable epinephrine
therapy, what is most appropriate therapy to ¯ recurrences” à answer =
venom immunotherapy (VIT). What this does is desensitizes the patient

MEHLMANMEDICAL.COM 16
MEHLMANMEDICAL.COM

to the antigen by allowing him/her to develop neutralizing IgG antibodies


against it.

HY Congenital lung diseases


- Autosomal recessive; chromosome 7.
- Mutations in CFTR gene, which codes for a chloride channel.
- Most common mutation is DF508, which is deletion of a phenylalanine.
- USMLE wants “abnormal protein structure,” or “abnormal protein
folding,” a common mechanistic answer for CF.
- In the disease, the CFTR chloride channel, normally located at the cell
surface, is instead sequestered at the RER. This is a HY point, where they
want you to know patients with disease do not have the chloride channel
on the cell membrane.
- Disease results in impaired ability to secrete Cl- into alveolar and
pancreatic secretions. This causes greater (-) charge within the cell. Na+
then moves through ­ ENaC into the cell to balance charge. Water
follows Na+, resulting in dried up alveolar and pancreatic secretions.
These are known as inspissated secretions. Inspissation means
desiccated (dried up) within a lumen. The inspissated alveolar secretions
lead to mucous plugging, airway obstruction, and recurrent pneumonias.
The inspissated pancreatic secretions lead to “exocrine pancreatic
insufficiency” (HY phrase) and fat-soluble vitamin malabsorption. It is to
my observation that vitamin E deficiency is the highest yield vitamin
deficiency due to CF across NBME questions, which will present as
neuropathy in a child who has CF.
- Meconium ileus HY; refers to failure to pass stool in the first 24 hours
after birth.
- CF can sometimes cause nasal polyps.
- USMLE doesn’t obsess over pathogen causes for CF pneumonia, but
Cystic fibrosis
classically Pseudomonas is a culprit. Prior to age 10, S. aureus exceeds
Pseudomonas as most likely organism; after age 10, Pseudomonas
eclipses S. aureus. The caveat is you have to use your head though: if
they say 8-year-old with pneumonia due to gram-negative rods, you
know that’s Pseudomonas. Likewise, if they say a 17-year-old with
pneumonia due to gram-positive cocci in clusters, you know that’s Staph
aureus.
- Sweat glands reabsorb Cl- rather than secrete it. This is impaired in CF,
so we have ­ Cl- in the sweat (i.e., >60 mEq/L). The sweat chloride test is
more accurate for diagnosis than genotyping due to allelic heterogeneity
of CFTR gene, where many different mutations cause the disease and can
be difficult to detect with routine screening panels.
- A transepithelial nasal voltage test can also be done, where there is ­
nasal potential difference.
- Male patients can have congenital bilateral absence of vas deferens
(CBAVD), leading to absent sperm in a sample.
- Since this is an AR condition, there is a 2/3 chance a phenotypically
normal sibling of a CF patient is a carrier. This applies to any AR disorder,
but USMLE likes to use CF as the archetypal example. If this 2/3-point
sounds confusing, I talk about this stuff in detail in my HY Genetics PDF.
- CF medications not HY for Step, but you could be aware that -caftor
agents (e.g., ivacaftor, lumacaftor) facilitate with proper CFTR channel
localization to the cell membrane + structural folding. Dornase-alfa
(correct, not alpha) is a nucleotidase that softens mucous. Guaifenesin is
also a mucous-softening agent.

MEHLMANMEDICAL.COM 17
MEHLMANMEDICAL.COM

- Answer on USMLE for a question that sounds like CF but patient has
situs inversus or dextrocardia – i.e., patient will have recurrent
pneumonias and organs (or just heart) on opposite side of body.
- Abnormality of cilia function due to defective dynein arm. This is a HY
point, where sometimes the answer will just be “dynein.”
- A cilium on cross-section has a 9x2 microtubule configuration. Dynein is
a molecule that is necessary for cilia function.

Primary ciliary dyskinesia


(Kartagener syndrome)

- Patient will sperm in sample, but motility will be decreased. This is in


contrast to CF, where sperm are absent due to CBAVD.
- Women can get ectopic pregnancy due to abnormal Fallopian tube cilia.
- This diagnosis sounds straightforward enough, but I’ve seen plenty of
students miss these questions. For the situs inversus, they will describe
this as cardiac sounds loudest on the right, or there being a large mass
palpable beneath the left costal margin (liver on the opposite side).
- Answer on USMLE for a woman 20s-30s, non-smoker, who has
increased pulmonary vascularity/markings and either a loud P2 or
triscupid regurg (as I talked about in cardio section, these are HY findings
for pulmonary hypertension).
- “Primary” means the pulmonary hypertension inherently starts with the
lungs and is not due to a secondary (i.e., external) cause such as smoking,
CF, systemic sclerosis, etc.
- Due to mutations in BMPR2 gene.
- Do not confuse this with cor pulmonale. Recall that the latter is right
Primary pulmonary heart failure findings or RV structural changes due to a pulmonary cause.
hypertension (PPH) If the question gives evidence of RVH (i.e., RBBB, right-axis deviation on
ECG, boot-shaped heart on CXR), or has overt right heart failure findings
(i.e., JVD or peripheral edema), then we can say the patient now has cor
pulmonale due to primary pulmonary hypertension. A loud P2 and
tricuspid regurg, however, are HY findings for pulmonary hypertension
that don’t necessarily mean cor pulmonale.
- One of the highest yield points is that endothelin-1 is ­ in these
patients. This is a vasoconstrictor that is ­ in pulmonary hypertension
from any cause, but notably USMLE likes it for PPH.
- Bosentan blocks endothelin-1 receptors and is HY Tx.

HY Autoimmune-related pulmonary conditions


- Idiopathic autoimmune disease characterized by multi-organ system
fibrosis and hardening of tissues (i.e., sclerosis).
Systemic Sclerosis
- Divided into limited and diffuse subtypes.
(aka scleroderma)
- Limited scleroderma = CREST syndrome (Calcinosis, Raynaud’s,
Esophageal dysmotility, Sclerodactyly, Telangiectasias).

MEHLMANMEDICAL.COM 18
MEHLMANMEDICAL.COM

- USMLE can describe Raynaud as color change of the fingers with cold
weather. They describe sclerodactyly as tightening of the skin of the
fingers. Esophageal dysmotility presents as GERD. I haven’t seen NBMEs
give a fuck about calcinosis, which is abnormal Ca2+ deposition in tissues.
- Diffuse scleroderma = CREST syndrome + renal involvement
(presenting as ultra-HY BP, e.g., 220/120, due to renal involvement
causing a surge in RAAS. If patient doesn’t have high BP, it’s not diffuse
type on USMLE.
- Apart from just being able to diagnose these conditions, the highest
yield point on USMLE is that both types cause pulmonary fibrosis,
leading to pulmonary hypertension.
- Pericardial fibrosis can also occur.
- An offline Step 1 NBME wants ¯ LES sphincter tone and ¯ esophageal
peristalsis as an answer.
- USMLE wants “dress warmly in cold weather” as an answer for how to
¯ recurrence of Raynaud. Dihydropyridine CCBs can also be used (e.g.,
nifedipine).
- USMLE wants to know which drugs you avoid in patients with Raynaud
à answer = a1 agonists (e.g., phenylephrine, oxymetazoline), since
these constrict arterioles/capillaries.
- Idiopathic autoimmune disorder that is one of the highest yield
conditions on USMLE. You must know this condition extremely well.
- Characterized by non-caseating granulomas within lung tissue. These
consist of activated macrophages called epithelioid macrophages, aka
histiocytes.
- These histiocytes in the lung secrete 1a-hydroxylase (normally
produced in the PCT of the kidney in response to PTH), which will convert
inactive 25-OH-D3 into active 1,25-(OH)2-D3 (i.e., causing high vitamin D;
aka hypervitaminosis D), which then goes to the small bowel and ­
absorption of calcium, causing hypercalcemia.
- Even though 1,25-(OH)2-D3 also ­ small bowel absorption of phosphate
along with calcium, it’s to my observation that phosphate can be normal
in sarcoidosis NBME Qs, so do not be confused if you see phosphate in
the normal range and only see hypercalcemia.
- PTH is suppressed in sarcoidosis due to the high Ca2+ (exceedingly HY).
- In other words, on USMLE you will select ­ 1,25-(OH)2-D3 and ¯ PTH.
- Archetypal presentation is African-American woman 20s-30s with 6+
months of dry cough and red shins (erythema nodosum). Other findings
Sarcoidosis like low-grade fever with flares can occasionally be seen.
- Bihilar lymphadenopathy seen on CXR or CT. They can also describe
this as CXR or CT “shows hilar nodularity.”

MEHLMANMEDICAL.COM 19
MEHLMANMEDICAL.COM

- There is a Family Med Q that gives early-30s African-American woman


with dry cough + they say CXR shows no abnormalities à answer =
“activation of mast cells” (i.e., asthma is the answer); wrong answer is
“non-caseating granulomatous inflammation.” So your HY point here is:
if the vignette sounds like it could be sarcoidosis but they say CXR is
normal or shows “mild hyperinflation” (also buzzy for asthma), choose
asthma over sarcoidosis.
- A 2CK IM CMS form gives a sarcoidosis vignette where they mention
bihilar lymphadenopathy on a CT scan + high hepatic AST + ALP + weight
loss; they don’t mention hypercalcemia. Unusual combo of findings, but
the answer is inferable based on the CT.
- Lupus pernio is an enlarged nose due to sarcoidosis (not SLE, despite
the name).
- USMLE wants steroids (i.e., oral prednisone) as treatment.
- Rheumatoid arthritis can cause a restrictive lung disease known as
rheumatoid lung.
Rheumatoid lung - In addition, methotrexate, the first-line DMARD for RA, can cause
pulmonary fibrosis. So patients with advanced RA can have pulmonary
fibrosis often from a combo of the two.

HY Pulmonary embolic conditions


- DVT that has embolized up the venous circulation to the pulmonary
arteries.
- Presents as acute-onset shortness of breath and tachycardia in patient
with one of the following risk factors: post-surgery (including C-section);
sedentation, e.g., due to trauma such as hip injury; OCP use; and
thrombotic disorders (i.e., FVL, prothrombin mutation, MTHFR).
- S1Q3T3 is most specific finding on ECG for PE that is nonexistent on
USMLE and will get you questions wrong. What you want to remember
is: sinus tachycardia (i.e., normal ECG but HR is simply high) is the most
common ECG finding for PE.
- First step is management (2CK) is heparin.
- After heparin, do a spiral CT of the chest to diagnose.
- If the patient is already on warfarin and gets a PE, the first answer they
want is CT to diagnose, followed by IVC filter. Do not choose IVC filter
prior to confirming the PE with a CT.
- One 2CK NBME Q has tPA as the answer for PE in patient who has
Pulmonary embolism
obstructive shock (i.e., low BP due to PE).
- V/Q scan is done instead of CT first-line in pregnancy.
- One 2CK Q gives V/Q that is performed and shows “multiple sub-
segmental perfusion defects” à answer = recurrent pulmonary emboli.
- Most common cause of death from PE is ventricular fibrillation à acute
right heart strain from a saddle embolus precipitates the Vfib.
- The acid-base arrows USMLE wants for PE are: ¯ CO2, ­ pH, « bicarb.
In other words, we have an acute respiratory alkalosis from high RR.
Bicarb is unchanged because the kidney requires minimum 12 hours to
accomplish this. These arrows are the same as acute asthma attack.
- One 2CK Q gives lab values in PE of ¯ CO2 and ¯ pH. What this means
is: patient has lactic acidosis (hence ¯ bicarb) due to ischemia from poor
perfusion. It’s HY for USMLE that you know any cause of shock, whether
that be septic, hypovolemic, cardiogenic (or even obstructive from PE),
can cause lactic acidosis and low bicarb. Don’t be confused by this.
- PE is textbook example of dead space (as discussed earlier).

MEHLMANMEDICAL.COM 20
MEHLMANMEDICAL.COM

- The answer on USMLE if they say 30 seconds to 2 minutes after delivery


of the placenta, the patient gets shortness of breath and tachycardia à
sounds like PE, but answer is amniotic fluid embolism to the lungs. In
contrast, if they say a woman had a C-section 2 days ago + now gets up in
Amniotic fluid embolism hospital to use bathroom + now has acute-onset SoB and tachyardia à
answer = PE, not amniotic fluid embolism.
- Can cause disseminated intravascular coagulation (DIC). It’s one of the
weird causes for USMLE. They can say the patient has bleeding from IV
sites / catheter lines following delivery of the placenta.
- The answer on USMLE if they say long-bone fracture (i.e., usually
femur) in patient who now develops shortness of breath + petechiae on
the shoulders/chest.
- The patient will also have thrombocytopenia, but only about half of
vignettes mention this.
Fat embolism - The petechiae on the shoulders/chest are not due to the ¯ platelets,
since conditions like ITP, for instance, don’t present with this finding.
Literature suggests it might be related to microvascular plugging by fat in
the dermis.
- Question can mention pulmonary biopsy stained with Oil Red O or
Sudan black (stains for fat).
- The answer on USMLE if they say patient has sudden death following
Air embolism
insertion of a central venous line à air goes to the lungs + kills patient.
- Will present as two ways on USMLE: 1) violaceous lesions on the feet in
someone who just had a AAA repair à cholesterol plaque launches off to
feet + they show you image like this one below:

Cholesterol embolism

The second way (2CK NBME 10) has cholesterol emboli Q where they
don’t show an image but describe the violaceous foot lesions in someone
with mere Hx of AAA (rather than undergoing repair). They say “Maltese
crosses are seen.” Obscure, but apparently refers to birefringence of
cholesterol esters that can be visualized under polarized light.

HY Trauma and pleural space conditions


- Difficult diagnosis on USMLE Surg forms.
- Diagnosis of exclusion, meaning all of these 2CK Qs require eliminating
the other answer choices to get there.
- The Q will say patient was in an accident + now has some form of
Pulmonary contusion difficulty breathing and patchy infiltrates in one or more areas of lung.
They may or may not mention rib fractures just above the areas of
infiltrates. Sounds vague as I said, but that’s what USMLE will say.
- Textbook descriptors such as “white-out of the lung” are nonsense. I
don’t think I’ve ever seen this on NBME material.

MEHLMANMEDICAL.COM 21
MEHLMANMEDICAL.COM

- Contused (bruised) lung is known to be fluid-sensitive, where the Q can


say patient was given IV saline and now has worsening of O2 sats. This is
classic, but I’ve also seen this in myocardial contusion questions, making
the Dx difficult as I said. But you’ll be able to eliminate myocardial
contusion if they don’t mention severe bruising or pain over the sternum
or post-MVA arrhythmia (e.g., premature atrial contractions).
- The answer on USMLE Surg forms if they say a patient has paradoxical
Flail chest breathing (i.e., the chest wall moves outward with expiration and inward
with inspiration).
- The answer on USMLE Surg forms if they say there’s a “persistent air
Ruptured bronchus
leak despite placement of a chest tube.” This will be in a trauma patient.
- The answer on USMLE Surg forms for an MVA followed by CXR showing
“obscured left hemidiaphragm and the NG tube present in the lower left
side of the chest.”
Diaphragmatic rupture
- Don’t confuse with ruptured bronchus above, or with esophageal
rupture, which will tell you water-soluble contrast swallow is visualized
in the mediastinum.
- Air in the pleural space. The terms almost always refers to a
“spontaneous pneumothorax,” which will be acute-onset sharp chest
pain in a tall, lanky patient in teens or 20s due to “ruptured subapical
bleb.”

Pneumothorax

- Pneumothorax + low BP = tension pneumothorax.


- Mechanism for tension pneumothorax is compression of the vena cavas
leading to ¯ venous return à can cause JVD.
- Simple/spontaneous pneumothoraces cause ipsilateral tracheal shift.
- Tension pneumothoraces cause contralateral tracheal shift.
- Tension pneumothorax need not be due to overt trauma. It will literally
just be pneumothorax + low BP 9/10 times on USMLE. I’ve seen only one
Q on a 2CK Surg form where BP is lower end of normal, but the patient is
tachy around 120 bpm (implying barely holding BP in normal range).
- Patient will have hyper-resonance to percussion, ¯ breath sounds, and
¯ tactile fremitus. Breath sounds are ¯ because the air in the pleural
space masks the sounds of underlying air flow in the alveoli. Tactile
fremitus is ¯ because the vibration of air in the alveoli is masked by the
overlying air.
- Treatment for pneumothorax is “needle decompression followed by
chest tube.” That phrase is HY.
- One 2CK Q just has “tube thoracostomy” as the answer without needle
decompression being listed. So if they don’t list the needle, don’t be

MEHLMANMEDICAL.COM 22
MEHLMANMEDICAL.COM

confused, and just choose the chest tube. Thoracostomy means making a
hole in the thorax.
- Pleurodesis I’ve never seen as correct answer on NBME material but
can be listed as distractor à means putting talc into pleural space to
obliterate it in patients who have recurrent pneumothoraces.
- Should be noted that very small pneumothoraces in stable patients
with minimal symptoms can be observed. There is one Q on a 2CK NBME
form where the answer is observe. But the vignette goes out of its way
to emphasize how unremarkable the patient’s presentation is.
- There is a 2CK Q where pneumothorax is caused by barotrauma from
ascending too quickly from underwater. This is different from Caisson
disease (“the bends”), where nitrogen bubbles form in the blood.
Pulmonary barotrauma from quick ascent, resulting in pneumothorax,
can occur if some of the alveoli expand too quickly.
- Fluid in the pleural space; often refers to “hydrothorax,” which means
transudate or exudate of plasma-like fluid.

Pleural effusion - USMLE wants: dullness to percussion, ¯ breath sounds, and ¯ tactile
fremitus. Similar to pneumothorax, the latter two are ¯ because the fluid
in the overlying pleural space masks air movement in the alveoli.
- Many causes of pleural effusion on USMLE. I’ve observed left heart
failure as a notable cause (i.e., patient will have pulmonary edema +/-
pleural effusion). NBME can also give you pleural effusion with
pneumonias, tuberculosis, and aortic dissection.
- Meigs syndrome = triad of ovarian fibroma, ascites, right-sided pleural
effusion.
- USMLE wants you to know transudative versus exudative pleural
effusions à transudates are more water-like, with fewer solutes;
exudates contain more solutes. As per Light’s criteria (HY for both Steps
1 and 2CK), an exudate will have:

- Pleural fluid protein : serum protein ratio >0.5.


- Pleural fluid LDH : serum LDH ratio > 0.6.
- Pleural fluid LDH >2/3 upper limit of normal of serum LDH.

- In other words, transudate contains less protein and LDH as the two
main distinctions.
- Transudates contain fewer WBCs than exudates, but I routinely see 500
WBCs/µL in transudate Q on NBME.

MEHLMANMEDICAL.COM 23
MEHLMANMEDICAL.COM

- Highest yield cause of transudate on USMLE is left heart failure, as


discussed in the Cardio PDF.
- ARDS, pulmonary emboli, and infections can cause exudative pleural
effusions (due to inflammation).
- Progression to empyema HY (see below).
- For 2CK Surg, it is exceedingly HY you know about empyema. There’s
two ways it presents on NBME:
- 4 out of 5 questions, it will present as a sequence of pneumonia à
parapneumonic effusions (i.e., exudative pleural effusions that occur due
to pneumonia) à empyema. In other words, if a patient has a
pneumonia that then leads to an exudative pleural effusion, we simply
call that a parapneumonic effusion. This effusion can become
progressively worse and purulent, to the point that the pH of the pleural
Empyema fluid falls below ~7.1ish. We now call it empyema, which is frank pus in
the pleural space. Low pH of the pleural fluid to < ~7.1 is very HY on Surg
forms. I’ve seen Qs where they say 7.12, 7.11, and 6.99, and they want
empyema.
- 1 out of 5 Qs will not mention anything about Hx of pneumonia. They
will give patient who has persistent fever + recurrent pleural effusions
in someone who has cirrhosis (i.e., ¯ oncotic pressure à transudative
pleural effusions). Then the answer is just empyema.
- NBME wants tube thoracostomy to drain the purulence.
- Chyle (lymph fluid) in the pleural space.
- The answer when they tell you a patient has milky or white-cloudy fluid
in the pleural space where the pH is significantly above 7.1.
- For instance, they might give you 15-line paragraph where they say
Chylothorax
milky fluid + tons of lab values, but you notice the pH of the pleural fluid
is 7.40. This means it’s chylothorax and empyema is wrong.
- Can be caused iatrogenically by insertion of central line via the left
internal jugular vein, or by cancers, e.g., lymphoma.
- Blood in the pleural space.
- Apart from massive trauma, the way this shows up on USMLE is as
malignant pleural effusion.
- They’ll give you massive paragraph with tons of info + tell you patient
Hemothorax has history of breast or lung cancer + has dullness to percussion à
answer = malignant pleural effusion.
- Patient can have hypotension and muffled heart sounds, similar to
cardiac tamponade, but rather than JVD (to complete Beck triad), can
have flattened neck veins (due to loss blood from the circulation).

HY Alveolar fluid conditions


- As discussed in the Cardio PDF, this will be due to left heart failure on
USMLE.
- There is such thing as “non-cardiac pulmonary edema” (i.e., ARDS, TRALI
[discussed below in this table]), but for USMLE purposes, if you see
“pulmonary edema” as an isolated phrase on USMLE, it refers to transudation
of fluid into the alveolar spaces due to ­ pulmonary capillary hydrostatic
Pulmonary edema pressure from left heart pathology.
- PCWP and LAP are both elevated.
- USMLE can give you vignette of, e.g., MI with dyspnea, and they ask for the
mechanism of the dyspnea in the patient à answer = “increased alveolar-
arteriolar oxygen gradient” – i.e., the patient can breathe just fine so alveolar
O2 is normal, but fluid impairs the gas exchange, so we have low
arterial/arteriolar oxygen.

MEHLMANMEDICAL.COM 24
MEHLMANMEDICAL.COM

- “Cephalization of pulmonary vessels” is buzzy and synonymous with


pulmonary edema on USMLE. Shows up in some NBME vignettes.
- The answer on USMLE for bilateral exudative chest infiltrates and ¯ O2 sats
in patient following: pancreatitis; aspiration of vomitus; near-drowning
episodes (aspiration of fresh/sea water); improper insertion of NG tube into
the lungs with feeding initiated; toxic shock syndrome; or general trauma /
sepsis.
- 2CK NBME Q gives vignette of toxic shock syndrome and then asks most
likely cause of death in this patient à answer = ARDS.
- Pulmonary decompensation associated with pancreatitis is very buzzy.
- Another 2CK Q gives patient who is brought to hospital following near-
drowning episode + they ask what the patient needs to be monitored for à
Acute respiratory distress answer = ARDS.
syndrome (ARDS) - ARDS is technically defined as a PaO2/FiO2 <300, but USMLE doesn’t give a
fuck. I’ve never once seen them apply this ratio or care.
- I emphasize up above the bilateral nature of ARDS because many Qs give
ARDS as DDx for unilateral conditions and it’s wrong.
- In theory, patient can be ventilated as follows: prone positioning (patient on
stomach) + low-tidal volume setting + permissive hypercapnia. IIRC this is
asked once on a 2CK form.
- An offline Step 1 NBME Q wants “increased surfactant protein D” as the
answer in a patient who is recovering from ARDS. Apparently surfactant
protein D is a marker of lung injury and is ­ in patients with ARDS (or who are
recovering from it). Call it weird all you want but it’s on the NBME.
- The answer on USMLE if a patient has an ARDS-like presentation with
bilateral crackles and low O2 sats <6 hours following a transfusion.
Transfusion-associated - Mechanism is abnormal priming of neutrophils in the lung that react to
lung injury (TRALI) cytokines within transfused blood products (this is straight from a CMS Surg
form 8 explanation).
- This is technically a type of non-cardiogenic pulmonary edema.
- Aka transfusion-induced hypervolemia.
- Differs from TRALI in that this is a type of cardiogenic pulmonary edema (i.e.,
the left heart can’t handle the ­ hydrostatic pressure from ­ volume, so
transudation into the alveoli occurs).
- Annoying diagnosis but presents two ways:
1) If a patient with Hx of heart failure or MI develops respiratory distress
following transfusion of repeated blood products.
- A 2CK NBME Q gives a 72-yr-old with Hx of MI ten years ago who
gets shortness of breath and bilateral crackles 30 minutes after
transfusion with crystalloid solution and 4 packs of RBCs (they don’t
specify the volume of crystalloid in the Q).
Transfusion-associated 2) They don’t mention Hx of cardiovascular disease + the vignette will sound
circulatory overload exactly like TRALI > 6 hours following a transfusion.
(TACO) - Surgery form 7 for 2CK gives an elderly dude who received only 3
packs of RBCs + he develops bilateral crackles and low O2 sats, but
they say this occurs 12 hours after admission (not <6 hours as with
TRALI), where the answer is “X-ray of the chest” as the next best step
in diagnosis. The Q doesn’t rely on you discerning TACO vs TRALI to
get it right, but the explanation says it’s TACO.

In summary, if ARDS-like picture after transfusion:

If <6 hours à TRALI.


If >6 hours à TACO.
If heart disease à TACO regardless of time frame.

MEHLMANMEDICAL.COM 25
MEHLMANMEDICAL.COM

HY Infection-related stuff
- Lobar pneumonia = Strep pneumoniae on USMLE (right lower lobe
consolidation with dullness to percussion).
- Bilateral interstitial pneumonia (aka atypical pneumonia) in
immunocompetent patients = Mycoplasma on USMLE.
- Lobar pneumonia where they say “interstitial markings” and Strep
pneumo isn’t listed à answer = Mycoplasma (the word “interstitial” wins
over location).
- Bilateral interstitial / “ground-glass” pneumonia in AIDS patient à
Pneumocystis jirovecii pneumonia (PJP).
- Lobar pneumonia in AIDS patient à Strep pneumo, not PJP.
- Bacterial pneumonia specifically post-influenza infection à S. aureus.
- Bilateral pneumonia + low Hb or (+) Coombs test à Mycoplasma à can
cause cold agglutinins, which means IgM against RBCs à hemolysis).
- Pneumonia + hyponatremia and/or diarrhea à Legionella.
- Pneumonia in 3-wk-old neonate who had conjunctivitis 1-2 weeks ago
à Chlamydia trachomatis (the STI; drains through nasolacrimal duct to
lungs).
- Pneumonia in newborn first few days of life à Group B Strep (Strep
agalactiae), which is gram (+) cocci. If they say gram (+) rods, that’s
instead Listeria. If they say gram (-) rods, that’s E. coli.
- Pneumonia + rabbits à Francisella.
- Pneumonia + bird keeper à Chlamydia psittaci.
- Pneumonia + southwest US and/or earthquake dust à Coccidioides.
- Patients who have lung cancer are prone to obstructive pneumonias
(on 2CK form).
- Pneumonia in CF à Pseudomonas or S. aureus.
- Pneumonia in patient with central venous catheter + right upper lobe
Pneumonia lesion à answer = Staph epidermidis (on NBME). Cather = biofilms.
- USMLE wants bronchoalveolar lavage for diagnosis of PJP.
- USMLE wants for pneumonia: adventitious/bronchial (i.e., abnormal)
breath sounds + ­ tactile fremitus (air vibrates due to movement
through infective consolidation within alveoli).
- Community-acquired pneumonia (CAP) empiric Tx = azithromycin on
2CK (on NBME). This covers the atypicals (Mycoplasma, Legionella,
Chlamydia) as well as S. pneumo.
- If patient has been on antibiotics in the past 3 months or has severe
lung disease, levofloxacin (respiratory fluoroquinolone) can be given
first-line.
- CAP that results in sepsis or septic shock à give ceftriaxone (sometimes
cefotaxime for peds).
- Nosocomial pneumonia (i.e., hospital- or ventilator-acquired) requires
coverage for MRSA and Pseudomonas. USMLE wants vancomycin PLUS
either ceftazidime (a 3rd-gen cephalosporin) or cefepime (a 4th-gen ).
- For fungal pneumonia, Tx = fluconazole.
- For fungal pneumonia + fungemia (high fever, chills) à Amphotericin B.
- NBME 9 for 2CK wants you to know sputum culture, followed by blood
cultures are done in all patients with pneumonia who are septic. What
they do in this patient is give you patient who has sputum culture
performed, then they ask what should be done next for diagnosis? à
answer = blood culture.
- If you get Pneumocystis pneumonia, jump straight to bronchoalveolar
lavage as the answer.
- If you get a patient who has CXR or CT showing cavitary lesions in the
lungs filled with a mass (likely Aspergillus fungus ball), they want “open
lung biopsy” as the most confirmatory test.

MEHLMANMEDICAL.COM 26
MEHLMANMEDICAL.COM

- When a pneumonia fully resolves and USMLE asks why the CXR is
normal after the fact, the answer is “maintenance of integrity of
basement membranes,” as mentioned earlier.
- 2CK wants you to know that pneumonias can occur in patients distal to
areas of lung obstructed by tumors; the answer will be “endobronchial
obstruction” as the reason the patient has distal pneumonia (same
answer for why there can be distal atelectasis).
- Step 1 NBME wants you to know rituximab (monoclonal antibody
against CD20 on B cells) increases the risk of bacterial pneumonia. This
makes sense, since B cells are required for antibody production as part of
humoral immunity against bacteria.
- Answer on USMLE for a kid <18 months old who has low-grade fever
and bilateral wheezes.
Bronchiolitis - Caused by respiratory syncytial virus (RSV).
- Tx is supportive care on USMLE. Don’t choose answers like ribavirin or
palivizumab.
- Caused by paramyxovirus (aka parainfluenza virus).
- Presents as hoarse, barking, or seal-like cough in school-age kid. The Q
can say the cough gets better when his dad brings him out into the cold
air.
- Neck x-ray shows “steeple sign,” which is sub-glottic narrowing.

Laryngotracheal bronchitis
(aka croup)

- Sometimes the Q can give you easy vignette of croup, but then the
answer is just “larynx” (literally inflammation of the larynx, trachea, and
the bronchi). “Sub-glottic) means below the area of the vocal cords. The
larynx is the area encompassing the vocal cords.
- Tx is supportive. If they force you to choose an actual Tx however,
nebulized racemic epinephrine is the answer.
- Caused by Haemophilis influenzae type B.
- Seen in unvaccinated and immigrants (can be unvaccinated), as well as
patients with asplenia or sickle cell (auto-splenectomy).
- Can also cause meningitis.
- X-ray of neck shows “thumbprint sign.”

Epiglottitis

MEHLMANMEDICAL.COM 27
MEHLMANMEDICAL.COM

- Presents as child who has fever + difficulty breathing. They can say the
kid is drooling and/or in tripod positioning (facilitates use of accessory
muscles).
- USMLE wants intubation as answer. Epiglottitis is a medical emergency
that can lead to sudden occlusion of the airway.
- Tx = 3rd-gen cephalosporin (cefotaxime in peds, or ceftriaxone); give
rifampin to close contacts.
- Nonexistent diagnosis on USMLE, but theoretically caused by S. aureus
and can present as stridor (reflects upper airway inflammation).
Bacterial tracheitis - I mention it here because many Peds resources tend to list this
diagnosis alongside croup and epiglottitis as a differential, so you should
at least be aware it exists, but I haven’t seen it assessed on NBMEs.
- Classic whooping cough presents as succession of many coughs
followed by an inspiratory stridor.
- What you need to know for USMLE is that this can absolutely present in
an adult and that they can be vague about it, just describing it as a
regular cough. The way you’ll know it’s pertussis, however, is they will
say there’s either hypoglycemia or post-tussive emesis, which means
vomiting after coughing episodes.
- Pertussis can cause super-high WBC counts in the 30-50,000-range,
Pertussis where there are >80% lymphocytes. This makes it resemble ALL. So you
should know for Peds that ALL-like laboratory findings + cough =
pertussis.
- Q will ask number-one way to prevent à answer = vaccination (not
hard, but they ask it). Pertussis is part of TDaP. The pertussis component
is killed-acellular; the tetanus and diphtheria are toxoid.
- Erythromycin can be given to patients with active cough; USMLE
doesn’t give a fuck about pertussis stages.
- Close contacts should also receive erythromycin.
- As mentioned in the Cardio PDF, this is an MSK condition I’ve seen
asked twice on 2CK material that has nothing to do with the lungs,
despite the name.
Pleurodynia
- This is viral infection (Coxsackie B) causing sharp lateral chest pain due
to intercostal muscle spasm.
- Creatine kinase can be elevated in stem due to ­ tone of muscle.
- Mycobacterium tuberculosis has unique cell wall composed of mycolic
acid that is difficult to gram stain. Requires acid-fast stain.
- Produces cord factor (asked on NBME) as a virulence factor.
- Can present similar to lung cancer, where patient can have B symptoms
(i.e., fever, night sweats, weight loss) and hemoptysis.
- Living in a homeless shelter or immigrant status from endemic area is
buzzy. I’ve seen rural India and Albania as two locations in NBME Qs.
- Can cause cavitations and calcification in the lung grossly; on histo,
causes caseating granulomatous inflammation.
Tuberculosis - Ghon foci/complexes are textbook descriptors for TB lesions but not
assessed eponymously on USMLE.
- Can cause constrictive pericarditis (can also calcify).
- Disseminated TB (miliary TB) can affect multiple organ systems, leading
to psoas abscess, Pott disease (TB infection of the vertebrae), adrenal
insufficiency, meningitis, osteomyelitis, and arthritis.
- First step in diagnosis is PPD test (type IV hypersensitivity).
- If PPD test is (+), the next best step is CXR.
- If PPD is (+) but CXR (-), next best step is “treat for latent TB, “ or “give
TB prophylaxis.” This is isoniazid (INH) for 9 months + vitamin B6 (since

MEHLMANMEDICAL.COM 28
MEHLMANMEDICAL.COM

INH can cause B6 deficiency). It is exceedingly HY you know that


neuropathy in a patient being treated for TB has B6 deficiency.
- If PPD and CXR are both (+), the next best step is “treat for active TB,”
which is RIPE for 2 months + RI for 4 more months (6 months total). RIPE
= rifampin, isoniazid, pyrazinamide, ethambutol.
- BCG vaccine is live-attenuated. USMLE wants you to know Hx of BCG
vaccine does not change management based on PPD guidelines.
- If USMLE asks you how long after TB exposure will someone’s sputum
cultures be positive, the answer is 2-5 weeks.
- Interferon-gamma release assay (Quantiferon Gold) can be used in
patients who have Hx of BCG to ¯ false (+)s, but USMLE doesn’t assess it.
The reason I mention it is because they want you to know interferon-g is
required for stimulation of alveolar macrophages to control TB.
- Patients who have IFN-g deficiency or IL-12 receptor deficiency (IL-12
normally ­ IFN-g) have ­ susceptibility to TB infections. If this immuno
stuff sounds confusing, I talk about this in detail in my HY Immuno PDF.
- TNF-a is also required to suppress TB. Therefore drugs such as
infliximab, adalimumab, and etanercept ­ risk of TB, which is why they
should be avoided in silicosis patients, as mentioned earlier.
- What is considered a positive PPD test differs depending on risk factors:
- >5mm (+): Hx of close contact to someone with active TB;
immunocompromised patient (AIDS, organ transplant recipient
receiving immunosuppressants, chronic corticosteroid user);
calcification on CXR.
- >10mm (+): Health care worker or prisoner/prison worker;
immigrant from endemic area; TB laboratory personnel,
children <4.
- >15mm (+): everyone else.
- If a PPD test is (+), never repeat it. If it is negative, it must be repeated
in 1-2 weeks (i.e., sometimes false-negatives).
- Answer on a 2CK form for a guy who moves into a new apartment
Hot tub lung building with a hot tub and gets bilateral chest infection à inhalation of
fumes with Mycobacterium avium complex (MAC).
- Exceedingly HY on USMLE.
- USMLE wants “aspiration of oropharyngeal normal flora,” or “aspiration
of oropharyngeal anaerobes” as the cause.
- Q will give aspiration risk factor, such as alcoholism, dementia (can
cause loss of gag reflex), Hx of stroke (leading to dysphagia), or epilepsy.
- Q can also mention broken or missing teeth (hypodontia) as risk factor.
- Often described on NBME as pulmonary lesion with an air-fluid level.
This is buzzy, but not a mandatory descriptor. This refers to the top half
of the circle being air, and the bottom being pus, the latter settling due
to gravity.

Pulmonary abscess

MEHLMANMEDICAL.COM 29
MEHLMANMEDICAL.COM

- The stem can say the patient has “foul-smelling sputum.” This
descriptor is exceedingly HY and is synonymous with anaerobes on
USMLE.
- Oropharyngeal normal flora = Bacteroides (strictly anaerobic gram-
negative rods); as well as Peptostreptococcus and Mobiluncus. The latter
two are not HY, but Bacteroides is. I mention all three, however, because
the Q can say sputum sample shows “gram-negative rods, gram-positive
cocci, and gram-positive rods,” which refers to all three. But the bigger
picture concept is, this = mixed normal flora.
- Tx = clindamycin. USMLE loves this.
- If Q tells you patient was treated for pulmonary abscess + a year later
there’s still a lesion seen on CXR à answer = “failure of maintenance of
basement membranes.”
Acute bronchopulmonary - Presents as asthma-like presentation in patient with ­ sensitivity to
aspergillosis (ABPA) aspergillus skin antigen.
- Answer on USMLE for bilateral lung condition + fever in farmer who has
exposure to hay (on new Step 1 NBME).
- They will tell you the fever self-resolves after 2 days and he now is
Hypersensitivity pneumonitis
afebrile.
- Byssinosis (pneumoconiosis from hemp) is wrong answer, since this
won’t present with fever + classically presents in textile workers.
- Idiopathic restrictive lung disease where patient has pneumonia-like
presentation that fails to improve with antibiotics. Not actual
pneumonia.
Cryptogenic organizing - Formerly known as bronchiolitis obliterans organizing pneumonia
pneumonia (COP) (BOOP).
- Nonexistent yieldness on USMLE, but I mention it because you will
sometimes see this as a wrong answer choice, particularly on hard 2CK
Qs, and I’ve seen enough students erroneously pick it.
- Infection of portion of ear just deep to tympanic membrane.
- Most commonly Strep pnuemo.
- Will present as red, immobile tympanic membrane. Immobility of the
tympanic membrane is highly sensitive for OM, meaning that if the Q
says mobility is normal, we can rule out.
- “Ear tugging” can be a sign in children of either otitis media or externa.
Otitis media (OM)
- Tx is amoxicillin or penicillin.
- Augmentin (amoxicillin/clavulanate) is classically given for recurrent
OM. So if you are forced to choose between amoxicillin/penicillin alone
or Augmentin, go with the former.
- For 2CK Peds, a tympanostomy tube (aka grommet) is used if the kid
has >3 OM occurrences in 6 months, or >4 in a year.
- Aka otitis media with effusion.
- Presents as fluid behind the tympanic membrane in a kid weeks after
Serous otitis media resolution of 1 or 2 otitis media infections.
- Almost always benign and self-resolves in 4-8 weeks. Answer is
observation.
- “Tympanic membrane perforation” is the answer on new 2CK NBME for
2-year-old who had 3-day Hx of viral infection followed by awakening
with severe ear pain + has dried blood on ear lobe and pillow + otoscopy
Tympanic membrane
cannot visualize tympanic membrane because of seropurulent fluid
perforation
draining from the ear canal.
- Can occur due to otitis media, although vignette on NBME doesn’t
sound like classic OM and is as described above.
- Inflammation of mastoid bone caused by untreated otitis media.
Mastoiditis - The mastoid process is the posterior part of the temporal bone that is
felt just behind the ear.

MEHLMANMEDICAL.COM 30
MEHLMANMEDICAL.COM

- Can present as a painful ear pinna that is displaced (e.g., upward and
outward).
- Diagnosis is made by CT or MRI. X-ray is wrong answer.
- 2CK IM Q gives a 2-year-old with mastoiditis where the answer is “CT of
the temporal bone.” Sounds wrong, since this is ­ radiation for a kid, but
it’s what they want.
- Isolated inflammation of the tympanic membrane.
Myringitis - Can be bullous (i.e., bullous myringitis).
- Caused by Strep pneumo or Mycoplasma.
- Infection of ear superficial to tympanic membrane.
- Classically caused by Pseudomonas.
- Increased risk in swimmers and diabetics.
- An NBME form has “necrotizing otitis externa” as answer for black skin
Otitis externa (OE)
within the ear canal in a patient. This is aka “malignant otitis externa.”
- USMLE wants “acetic acid-alcohol drops” as prophylaxis in college
student who does crew + continues to have water exposure.
- Tx (not prophylaxis) = “topical ciprofloxacin-hydrocortisone” drops.
- The answer if they tell you a school-age kid has a lingering fever after
an upper respiratory tract infection (URTI) for 10-14+ days.
- Whenever a URTI lingers for more than ~10ish days, you want to think
about sinusitis as a differential.
- A 2CK vignette gives nocturnal cough (reflects aspiration; in this case,
from the sinuses) and grey membranes in the oropharynx.
- The grey oropharyngeal membranes detail sounds weird, since that is
normally buzzy for Diphtheria, but it shows up on an NBME Q where the
answer is sinusitis and Diphtheria isn’t listed.
- IgA deficiency Qs, which presents as recurrent sinopulmonary
infections, can say patient has Hx of pneumonias + presents today with
Sinusitis
sore left cheek à reflects sinusitis.
- For 2CK, CT scan is done if chronic sinusitis >12 weeks. After CT is
performed for chronic sinusitis, nasal endoscopy can be performed.
- For Step 1, they can say nasal endoscopy shows obstructed drainage
from the maxillary sinus + ask for where a drainage cannula should be
placed à answer = “at the hiatus seminlunaris in the middle meatus.” If
you think it sounds stupid or low-yield, take it up with the NBME exam.
Student will then encounter it on exam and be like oh wow lol.
- Tx is amoxicillin/clavulanate (Augmentin). This is in contrast to OM and
Strep pharyngitis, which are treated with just amoxicillin or penicillin
alone, without the clavulanate (unless recurrent).

HY Pediatric-related respiratory stuff


- HY pediatric condition in which the proximal esophagus ends in a blind
pouch + the distal esophagus connects to the trachea.

Tracheoesophageal fistula
(TEF)

MEHLMANMEDICAL.COM 31
MEHLMANMEDICAL.COM

- Will present as a neonate who coughs up milk with initial feeding.


- Highest yield point about TEF is that diagnosis is made via insertion of
nasogastric tube (which cannot be inserted fully because it hits the blind
pouch of the esophagus).
- An NBME Q wants “endoderm” as the abnormal embryology for TEF
(makes sense, since esophagus is epithelial lining of the gut à endoderm).
- Weird condition in which the nasal passages don’t develop patency, so
the neonate is an obligate mouth-breather.

Choanal atresia

- Will present as a child who becomes blue/hypoxic while breast feeding


(because he can’t breathe through the nose), then cries/becomes pink
once detaching from the breast.
- Similar to TEF, diagnosis is made via insertion of nasogastric tube.
- Part of CHARGE syndrome à Coloboma of the eye (hole in the eye),
Heart defects, Atresia of the choanae, Renal defects, Genitourinary
anomalies, Ear anomalies. If you’re studying for Step 1, you don’t need to
worry about CHARGE syndrome. For 2CK Peds, you should know it.
- One of the highest yield Peds conditions on USMLE.
- Caused by failure of formation of pleuroperitoneal membranes.
- Always occurs on the left, where bowel from the abdomen can herniate
up into the left-chest.
- Can present as ¯ bowel sounds in the abdomen + ­ bowel sounds in the
left hemithorax.

Congenital diaphragmatic
hernia

MEHLMANMEDICAL.COM 32
MEHLMANMEDICAL.COM

- Vignette might say there are cystic-appearing areas in the left hemithorax
seen on CXR (loops of bowel).
- Aka hyaline membrane disease.
- The answer on USMLE for respiratory distress in kid who is born <34
weeks’ gestation.
- Due to insufficient surfactant production by type II pneumocytes due to ¯
lamellar bodies (the specialized organelles that produce surfactant).
- These kids have ¯ lecithin/sphingomyelin ratio (i.e., <2.0). Normally it is
>2-2.4.
- Another name for lecithin is dipalmitoyl phosphatidylcholine. This is
asked on NBME.
- USMLE can give simple vignette of NRDS and then ask for various
manipulation of the ratio – i.e., “­ sphingomyelin” might be an answer
(makes sense, since this would ¯ the ratio).
Neonatal respiratory distress - ¯ surfactant production means ¯ alveolar compliance and ­ elastic
syndrome (NRDS) recoil. Surfactant is hydrophobic and normally prevents the alveoli from
collapsing, so if it’s deficient, the hydrophobic interactions of the alveolar
walls enable ­ collapsing/elastic recoil.
- CXR shows a “reticulogranular” appearance. Very buzzy and HY.
- In order to prevent NRDS, a pregnant woman giving birth <34 weeks’
gestation must be two boluses of corticosteroids within 24 hours of
parturition, which accelerate fetal lung maturity. For example, there is a
2CK Obgyn Q where they tell you a woman giving birth at 33 weeks’
gestation was given a bolus of corticosteroids 12 hours ago. They ask for
next best step à answer = “bolus of corticosteroids.” Sounds weird
because they said it was just done, but she needs two boluses.
- For management, 2CK wants tactile stimulation first, then place under
warming lights. This sequence is HY. Then give exogenous surfactant and
oxygen.
- The answer on USMLE when the vignette sounds like NRDS but the kid is
term. For instance, they’ll say neonate has difficulty breathing, but then
you’ll notice he’s >37 weeks.
Transient tachypnea of the
- Usually seen following C-section or fast vaginal delivery.
newborn (TTN)
- Mechanism is insufficient time for the pulmonary lymphatics to clear
amniotic fluid from the lungs.
- CXR shows “fluid within fissure lines.”
- Fibrotic lung disease in an infant caused by continued use of
supplemental oxygen.
- Vignette will give you a kid who was born at 26 weeks’ gestation who was
Bronchopulmonary dysplasia in ICU on oxygen + now is 4 months old and is on home oxygen. They will
say ECG shows right-axis deviation +/- CXR shows increased pulmonary
vascularity à this reflects RVH (ensuing cor pulmonale) from pulmonary
hypertension caused by the bronchopulmonary dysplasia.
- Answer on 2CK Peds for kid under 2 who was crawling around on the
floor who now has acute-onset respiratory distress. The vignette will then
add one of two features:
1) Unilateral hyperresonance in one lung (but not
Foreign body aspiration
pneumothorax);
2) Unilateral atelectasis + ipsilateral tracheal shift.
- Exam will ask for “bronchoscopy,” or “fiberoptic examination of the
airways” as the answer.
- Not limited to kids, but can be part of atopy in patients with asthma.
- “Cobblestoning” of nasal mucosa is buzzy for allergy. This same word is
Allergic rhinitis
used to describe the tarsal conjunctiva in allergic conjunctivitis on Peds.
- NBME 9 for 2CK has “use of pillow and mattress covers” as the answer.

MEHLMANMEDICAL.COM 33
MEHLMANMEDICAL.COM

- Otherwise, USMLE wants intra-nasal corticosteroids as first-line med.


- USMLE does not want anti-histamines as first-line med.
- 2nd-gen H1 blockers (i.e., loratadine) are the answer if intra-nasal
corticosteroids are not listed. Oral prednisone is a wrong answer.
- I’ve seen students choose prednisone and then get irascible and say, “But
you said steroids were first-line.” à Yeah, bro. Intra-nasal. Not fucking
oral.
- Just know that enlarged tonsils (adenoids) are a known cause of stridor in
Adenoid hypertrophy
pediatrics. Answer = tonsillectomy.
- Pus collection causing pain and swelling at posterior oropharynx. Not
dramatic, but just know the diagnosis exists because it shows up on NBME.

Peritonsillar abscess
(Quinsy)

- Most common cause of stridor in infants.


- Softening of airway cartilage.
Laryngomalacia - Vignette will give kid under the age of 1 who has noisy breathing that is
mitigated when placed prone (on stomach) or upright.
- Self-resolves almost always (i.e., don’t treat).
- Weird cause of stridor that USMLE likes to contrast with laryngomalacia.
- Vascular embryology variant can lead to airway compression.
Vascular ring
- In contrast to laryngomalacia, mitigated when neck is extended.
- Tx = surgery.
- High-pitched, hyper-nasal, or whistling speech in patients with Hx of cleft
palate due to shortened palate, with or without repair.
Velopharyngeal insufficiency
- It’s the answer on a 2CK NBME, so know that it exists. They will simply
just give you vignette of cleft palate + ask what the kid is at risk for.
- Idiopathic squamous proliferation in middle ear behind the tympanic
membrane that presents as an enlarging mass.
Cholesteatoma - Can cause obstructive hearing loss + grow into inner ear.
- Requires surgery.
- Sounds like an obscure diagnosis, but it’s not. Be aware it exists for Peds.
- Not pediatric, but I’m putting it here because I see this confused with
laryngomalacia.
- This is a cause of stridor in (usually) adults who have stridor developing
Tracheomalacia
months post-surgery.
- Caused by Hx of intubation leading to softening of airway cartilage.
- Just know this diagnosis / it exists + don’t confuse with laryngomalacia.

Differentiating viral vs bacterial URTI


- If you’re studying for Step 1, you can basically ignore this table. If you’re studying for 2CK, the CENTOR
criteria are exceedingly HY for Family Med and Peds.
- If 0 or 1 point, the URTI is unlikely to be bacterial (i.e., it’s likely to be viral). If 2-4 points, the chance is
much greater that the URTI is bacterial.

MEHLMANMEDICAL.COM 34
MEHLMANMEDICAL.COM

- 1) Absence of cough (i.e., no cough = 1 point; if patient has cough = 0 points).


- 2) Fever >38 C.
- 3) Tonsillar exudates.
- 4) Lymphadenopathy (cervical, submandibular, etc.).

- There is a version of the criteria that includes age, but on the USMLE it can cause you to get questions
wrong. So just use the simplified above four points.
- If 0-1 point, answer = “supportive care”; or “no treatment necessary”; or “warm saline gargle”
(same as supportive care); or “acetaminophen.” Latter is answer for 3M with viral URTI + fever on
Peds NBME form 2.
- If 0-2 points, next best step = “rapid Strep test.” If rapid Strep test is negative, answer = throat
culture, NOT sputum culture.
- While waiting on the throat culture results, we send the patient home with amoxicillin or penicillin
for presumptive Strep pharyngitis.
- If child is, e.g., 12 years old, and develops a rash with the beta-lactam, answer = beta-lactam
allergy.
- If the vignette is of a 16-17 year-old who has been going on dates recently (there will be no
confusion; the USMLE will make it clear) + gets a rash with the beta-lactam, the answer = EBV
mononucleosis; therefore do a heterophile antibody test (Monospot test).
- EBV is the odd virus out that usually presents with all four (+) CENTOR criteria and presents like a
bacterial infection.
- This is why it’s frequently misdiagnosed as Strep pharyngitis. It is HY to know that beta-lactams
given to patients with EBV may cause rash via a hypersensitivity response to the Abx in the setting
of antibody production to the virus. EBV, in a patient who does not receive Abx, can cause a mild
maculopapular rash. But the rash with beta-lactam + EBV causes a more intense pruritic response
generally 7-10 days following Abx administration on the extensor surfaces + pressure points.

Ventilator settings nonsense for USMLE


- Students will sometimes nag about pedantic ventilator settings info. The vast majority of it is garbage for
USMLE. What I can basically say is that increasing PEEP (positive-end expiratory pressure) is probably the
answer 4 out of 5 times for any ventilator settings-type of question. The Q might give you a big 15-line
rambling paragraph where you’re not sure what’s going on, and then the answer is just “increase PEEP.”
- PEEP keeps the alveoli open longer, thereby facilitating gas exchange.
- When the patient is on a ventilator, the general aims are:

- Keep FiO2 as low / as close to room air as possible (i.e., high O2 can cause free radical damage);
- Keep tidal volume as low/normal as possible (i.e., 400-500-ish mL); high tidal volumes can cause
ventilator-associated barotrauma.
- Increase PEEP if it means lower FiO2 and tidal volumes can be achieved.

- But the above needs to be taken with a caveat. For example, the Q might say the patient is on a ventilator
and has arterial pO2 of 40 mm Hg (normal is 80-100) + FiO2 is 100% + tidal volume is 1000 mL, and the
answer will be “increase PEEP.” The student says, “Wait but I thought you said we want to reduce FiO2 and
tidal volume.” à Sure, but we can’t do that here because pO2 is super-low, so that would only exacerbate it
further.
- As I talked about for ARDS earlier, you could be aware the triad of 1) prone positioning, 2) low-tidal
volume mechanical ventilation, and 3) permissive hypercapnia can be implemented.
- A very HY point is that ¯ CO2 causes ¯ cerebral perfusion (this is why patients faint in panic attacks, which
is asked on NBME). The first step in managing ­ intracranial pressure is “intubation + hyperventilation.”
- They also ask the inverse of this on NBME – i.e., they say an anesthesiologist wants to ­ a patient’s
cerebral perfusion, and the answer is “decrease respiratory rate” à causes ­ CO2 à ­ cerebral perfusion.
- Ventilator-acquired pneumonia (VAP) should be treated with vancomycin PLUS either ceftazidime or
cefepime. Vancomycin not only covers MRSA but also is effective against some high-resistance S. pneumo

MEHLMANMEDICAL.COM 35
MEHLMANMEDICAL.COM

strains. Ceftazidime (3rd generation ceph) and cefepime (4th gen ceph) are very effective against
Pseudomonas, which is a HY nosocomial organism for VAP.
- Patients should ideally be weaned from ventilators as quickly as possible. USMLE also likes patients who
are sick on ventilators as sometimes having euthyroid sick syndrome – i.e., they will say patient cannot be
weaned from ventilator + has ¯ T3 and normal TSH à answer = euthyroid sick syndrome. The full array of
arrows are: ¯ T3, ­ rT3, « T4, « TSH. If you have no idea what I’m talking about, then I recommend going
through my HY Arrows PDF.
- If a patient ever has a high CO2 (NR 33-44 mmHg) à answer can = “ventilatory insufficiency.” This applies
to both patients on and off ventilators. Don’t be confused if they give you ­ respiratory rate. New Surg form
gives RR of 40 with ­ CO2 as an example. Sometimes patients have shallow breathing despite ­ RR.

HY Pulmonary drug points


- Endothelin-1 receptor antagonist used for pulmonary hypertension.
- Endothelin is a vasoconstrictor ­ in pulmonary hypertension.
- There is an NBME Q where guy is shaving + cuts himself + they ask what will be
Bosentan
seen at site of injury à answer = ­ endothelin. So be aware that even though
14/15 times this relates to pulmonary vascular constriction, it is technically a
vasoconstrictive mediator not isolated to the lungs.
- Dihydrofolate reductase inhibitor.
- 1st-line DMARD for RA + 1st-line oral agent for psoriasis that fails topicals.
Methotrexate
- Causes pulmonary fibrosis (and hepatotoxicity and neutropenia).
- Toxicity can be mitigated with “leucovorin rescue,” aka folinic acid (not folic acid).
- Short-acting b2-agonist used for asthma.
Albuterol
- Can cause tremor (asked on a Step 1 NBME).
- Inhaled corticosteroids (ICS) used for asthma and COPD.
Fluticasone,
- Used for prevention / to decrease recurrence of episodes. They have no role in
Beclomethasone
acute exacerbations. For the latter, IV methylprednisolone is used.
- Most effective at ¯ recurrent asthma attacks, but used last resort because oral
Prednisone
steroids can cause Cushing syndrome and growth stunting.
- Leukotriene receptor antagonists (LTC, D, and E4).
- Leukotrienes normally cause bronchoconstriction. So these agents ¯ airway
Montelukast,
constriction.
Zafirlukast
- LTB4 is special and is a neutrophilic chemotactic molecule. For immuno, you can
memorize that LTB4, IL-8, and C5a are HY neutrophilic chemotactic molecules.
Zileuton - Lipoxygenase inhibitor. Blocks synthesis of leukotrienes.
- Mast cell stabilizers used for asthma. Prevent release of histamine.
- Step 1 form has a Q where they ask, “Which of the following agents blocks the
release of an autocoid from its cellular storage site?” à answer = cromolyn
Nedocromil,
sodium.
Cromolyn sodium
- Autocoids are very brief-acting hormones that “are made, play their role, and are
destroyed at the same site.” à Histamine, serotonin, bradykinin, and angiotensin
are examples.
Tiotropium - Long-acting muscarinic receptor antagonist (LAMA) used first-line for COPD.
- Long-acting b2-agonist (LABA) used first-line for COPD.
Olodaterol
- As discussed earlier, either a LAMA or LABA can be used first-line for COPD.
- Short-acting muscarinic receptor antagonist (SAMA) used for COPD.
Ipratropium - Not first-line anymore, but still shows up on Family Med material as an agent
that’s used. Some old-school practitioners still use it, so don’t disregard it.
Epoprostenol - PGI2 prostacyclin that can be used for pulmonary hypertension.
Sildenafil - Viagra; PDE-5 inhibitor that can also be used for pulmonary hypertension.
- Dihydropyridine calcium channel blocker that can be used for pulmonary HTN.
Nifedipine
- As discussed in the Cardio PDF, can cause peripheral edema/fluid retention.
Omalizumab - Monoclonal antibody against IgE.

MEHLMANMEDICAL.COM 36
MEHLMANMEDICAL.COM

- Can be used in theory in severe asthmatics who have high IgE levels.
Dornase alfa - Nucleotidase used to break down airway mucous in cystic fibrosis patients.
Guaifenesin - Mucolytic agent that softens mucous in cystic fibrosis.
Ivacaftor - Helps localize CFTR channel to membrane and correct its folding (in CF clearly).

- 58F + 6-month Hx of shortness of breath and non-productive cough + 2-year Hx of worsening

dysphagia + 20-yr Hx of hands turning white when exposed to cold; what lung condition is this patient

most at risk of developing? à answer = pulmonary hypertension à pulmonary fibrosis common in

CREST syndrome (scleroderma; limited systemic sclerosis) à pulmonary fibrosis seen in both diffuse

and limited types of systemic sclerosis; the latter is sans the renal phenomena) à can lead to cor

pulmonale, which is right-heart failure secondary to a pathology of lung etiology (i.e., the left heart

must not be causative).

- What will the USMLE frequently say in the Q if they’re hinting at pulmonary hypertension? à HY

vignette descriptors are loud S2 or P2 (pulmonic valve slams shut when the distal pressure is high);

dilation of proximal pulmonary arteries; increased pulmonary vascular markings (congestion +

increased hydrostatic pressure).

- When is “pulmonary hypertension” important as the answer? à notably in Eisenmenger syndrome

and cor pulmonale.

o Regarding Eisenmenger: the reversal of the L to R shunt across the VSD such that it’s R to L

requires the tunica media of the pulmonary arterioles to hypertrophy secondary to chronic

increased preload à proximal backup of hydrostatic pressure à increased afterload on the

right heart à now the right heart starts to significantly hypertrophy à shunt across the VSD

reverses R to L. This is important, as RVH is not the most upstream cause of Eisenmenger;

the pulmonary hypertension is.

o Regarding cor pulmonale: as mentioned above, cor pulmonale is right heart failure

secondary to a pathology of pulmonary etiology (e.g., COPD, cystic fibrosis [CF], fibrosis,

etc.); however it must be noted that the most common cause of right heart failure is left

heart failure; so for cor pulmonale to be the diagnosis, the left heart must not be the

etiology of the right heart failure; the lungs must be the etiology. If the cause of the right

heart failure is, e.g., COPD or CF, then it is the degree of hypoxic vasoconstriction that

MEHLMANMEDICAL.COM 37
MEHLMANMEDICAL.COM

determines the prognosis for the cor pulmonale; in the setting of inadequate alveolar

ventilation à hypoxic vasoconstriction of pulmonary vessels à increased afterload in the

proximal pulmonary arteries à dilatation and congestion of proximal pulmonary arteries à

more hydrostatic pressure in the proximal pulmonary arteries à increased afterload

experienced by the right heart à right heart hypertrophies à right heart eventually

decompensates secondary to chronically elevated pulmonary vascular pressure à classic

right heart failure signs ensue (JVD + pulmonary edema; hepatosplenomegaly may also be

seen [e.g., spleen tip and/or liver edge are palpable beneath the costal margin]). In other

words, the mere findings of pulmonary hypertension alone are not sufficient for a Dx of cor

pulmonale; right heart failure signs must also be seen in the vignette (one NBME Q makes

this distinction). The pulmonary hypertension need not be due to hypoxic vasoconstriction if

the disease process is pulmonary fibrosis (CREST syndrome, Hx of radiation to the chest). It

must also be noted that in cor pulmonale, pulmonary capillary wedge pressure (PCWP)

must be normal.

- “Please explain PCWP. I’ve seen that before but no idea what that means or how it applies to

questions.” à If you stick a catheter into the venous circulation and feed it all the way back up to the

right heart and into the pulmonary circulation, eventually it won’t be able to go and farther and will

be lodged in a distal pulmonary arteriole; the pressure reverberations sensed by the catheter must

therefore reflect the pulmonary capillary pressure (immediately distal to the catheter); but of course

the pulmonary capillary pressure must reflect the left atrial pressure because the latter is

immediately contiguous; therefore PCWP = left atrial pressure. This is a very simple rule that must be

memorized and understood for the USMLE. For the purpose of questions, not only will PCWP be

normal in the setting of cor pulmonale, but it will also be elevated if cardiogenic shock is the answer

when blood pressure is low à i.e., if a guy just had a myocardial infarction + has low BP + the PCWP is

high in the Q, the answer is “cardiogenic shock” for the type of shock you’d select.

o Hypovolemic shock arrows à CO down, VR down, TPR up, PCWP down (or low-normal).

o Cardiogenic shock arrows à CO down, VR down, TPR up, PCWP up.

o Septic + anaphylactic shock arrows à CO up, VR up, TPR down, PCWP normal.

o Neurogenic shock + adrenal crisis arrows à CO down, VR down, TPR down, PCWP normal.

MEHLMANMEDICAL.COM 38
MEHLMANMEDICAL.COM

- 23F + non-smoker + gradually worsening shortness of breath + loud P2 + CXR shows increased

pulmonary vascular markings; Dx? à answer = primary pulmonary hypertension à common cause is

mutations in BMPR2.

- Treatment of pulmonary hypertension? à USMLE will generally not make you pick and choose

between agents, however it is generally accepted in the literature that dihydropyridine calcium

channel blockers, such as nifedipine, are effective in vasoreactive pulmonary hypertension. Other

agents include prostaglandin I2 (epoprostenol), bosentan (endothelin-1 receptor antagonist), and

phosphodiesterase-5 inhibitors (sildenafil; yes, Viagra).

- “Am I supposed to know something about endothelin-1?” à Step 1 pulmonary material has an

obsession with it. Be aware that endothelin-1 is a potent vasoconstrictor à so increased endothelin-1

activity is associated with pulmonary hypertension.

- 23F with primary pulmonary hypertension; Q asks which of the following might be seen in this

patient; answer = “increased endothelin-1 activity” à essentially, if bosentan is a HY antagonist of

endothelin-1 as a Tx for pulmonary hypertension, then it makes sense that increased activity is

correlated with pulmonary hypertension in patients.

- 68M + S3 + crackles in both lungs + JVD + peripheral edema; what would be seen in this patient? à

answer = “increased endothelin-1 activity” à congestive heart failure is defined as right heart failure

caused by left heart failure; because the lungs need to accommodate more fluid secondary to the

backup from the decompensated left heart, the pulmonary vessels constrict (endothelin-1 activity) à

pulmonary hypertension à right heart experienced increased afterload and then also fails.

- Which lung cancers are apical vs central? à Central à sounds like Sentral à Squamous cell, Small

cell; peripheral à adenocarcinoma, large cell.

- 45F + non-smoker + apical lung mass; no other information; most likely cancer? à answer =

adenocarcinoma (classically female non-smokers with peripheral lesion).

- 45F + non-smoker + peripheral lung mass + proximal muscle weakness + increased serum CK + rash on

shoulders; Dx? à paraneoplastic dermatomyositis secondary to bronchogenic adenocarcinoma.

- 45F + non-smoker + lung mass + thrombosis; lung cancer Dx? à answer = adenocarcinoma à in

general, cancer is associated with hypercoagulable state, but this is particularly the case for

MEHLMANMEDICAL.COM 39
MEHLMANMEDICAL.COM

adenocarcinomas; in addition, Trousseau sign of malignancy (migratory thrombophlebitis) is

characteristic of adenocarcinomas (not limited to head of pancreas adenocarcinoma).

- 45F + long smoking Hx + shooting groin pain; Dx? à ureterolithiasis secondary to hypercalcemia from

bronchogenic squamous cell carcinoma PTHrp (PTH related peptide) secretion.

- 45F + long smoking Hx + shooting groin pain + serum studies show high calcium; what is the patient’s

PTH level? à answer = decreased (suppressed in the setting of high PTHrp).

- 45F + dies of lung cancer + Q shows you gross pathologic specimen of large, cavitating, central lesion;

Dx? à answer = squamous cell carcinoma à cavitations are common.

- 45F + lung cancer + Hx of stabbing flank pain a couple months ago + today presents with confusion;

Dx? à hypercalcemic crisis à squamous cell carcinoma with PTHrp secretion with Hx of

nephrolithiasis; high calcium can cause delirium.

- 45F + long smoking Hx + central coin lesion on CXR + violaceous abdominal striae + potassium level of

3.0 mEq/L; Dx? à answer = small cell carcinoma with ectopic ACTH secretion; student says, “Wait but

why’s the potassium low?” à effect of chronically high cortisol (can act to secrete K at distal kidney

similar to aldosterone; only seen in ongoing Cushing syndrome).

- 45F + confirmed Dx of early-stage small cell lung cancer + wobbly gait; Q asks the mechanism for her

presentation; answer = autoimmune à paraneoplastic anti-Hu and anti-Yo antibodies causing

cerebellar dysfunction; Dx is small cell cerebellar dysfunction (small cell is known to cause many types

of neuronal paraneoplastic syndromes).

- 45F + central lung lesion + confusion + increased urinary osmolality; Dx? à SIADH secondary to small

cell à low serum sodium + high urinary osmolality; should be noted that sodium derangement (high

or low), as well as high calcium, can cause CNS disturbance / delirium.

- 45F + long smoking Hx + difficulty getting up from chair but is successful after multiple attempts; what

is the location of her pathology? à answer = neuromuscular junction à small cell paraneoplastic

Lambert-Eaton syndrome à antibodies against presynaptic voltage-gated calcium channels.

- 45F non-smoker + episodic flushing, wheezing, and dyspnea + coin lesion seen on CXR; Dx? à answer

= bronchial carcinoid tumor; arises from bronchial Kulchitsky cells (neuroendocrine cells); small blue

cells on biopsy.

MEHLMANMEDICAL.COM 40
MEHLMANMEDICAL.COM

- 33F + Hx of recent molar pregnancy + very high serum beta-hCG levels + new-onset non-productive

cough; CXR shows multiple peripheral densities in right lobe; Dx? à metastatic choriocarcinoma

(loves to metastasize to the lungs; chorio can progress from hydatidiform mole).

- Do we do lung cancer screening? à Annual low-dose chest CT in patients who meet all of the

following:

o Age 55-80.

o >30-pack-year Hx of smoking.

o Have smoked in past 15 years.

- 3M + swallows a peanut; where in the lung does it go? à superior segment of right lower lobe if

patient is supine; goes to inferior segment of right lower lobe if patient is upright. This is because the

right mainstem bronchus is shorter and straighter than the left.

- “Can you explain V/Q mismatch?” à ventilation (V) relative to perfusion (Q) à physiologic and

pathologic V/Q mismatch exist; should theoretically be one to one; physiologic V/Q mismatch refers

to natural imbalance of ventilation relative to perfusion in healthy lung when the patient is upright.

Both ventilation and perfusion increase from apex to base, but because of the effects of gravity,

perfusion increases more from apex to base, so V/Q is lower at the bases (~0.6) compared to the

apices (~3.0). In terms of pathologic V/Q mismatch, the overwhelming majority of lung pathologies

have decreased V/Q, meaning the net amount of oxygenation of the alveoli is reduced compared to

the amount of perfusion reaching those areas (i.e., in COPD, CF, asthma). In other words, even with

hypoxic vasoconstriction in areas with lesser oxygenation (as an attempt to reduce perfusion to

inadequately ventilated areas and restore V/Q closer to 1/1), the net amount of alveolar ventilation is

less compared to perfusion through the alveoli. In pulmonary embolism, it’s the opposite: V/Q is high

because perfusion is reduced to areas that receive normal ventilation.

- What is the difference between shunt and dead space? à a shunt means ¯ V/Q (i.e., reduced

ventilation relative to perfusion) to the point that the patient’s arterial oxygen becomes

decompensated (i.e., a shunt is always pathologic in pulmonary terms), but ¯ V/Q can also be

physiologic (i.e., not a shunt) at the lung bases, where the V/Q is normally 0.6; dead space means ­

MEHLMANMEDICAL.COM 41
MEHLMANMEDICAL.COM

V/Q (i.e., low perfusion relative to ventilation), but need not be pathologic (i.e., there is a natural ­

V/Q mismatch of ~3.0 at the lung apices. Types of dead space (asked on Step 1 NBME):

o Alveolar dead space = natural, physiologic ­ V/Q within the alveoli, where some areas of lung

receive more ventilation than perfusion – i.e., the apices (V/Q of ~3.0) compared to the

bases (~0.6).

o Anatomic dead space refers to parts of the respiratory tree that are naturally ventilated but

do not partake in gas exchange, such as the trachea, bronchi, and terminal bronchioles. The

respiratory bronchioles and alveoli partake in gas exchange, so the anatomic V/Q starts to

decrease from the level of the respiratory bronchioles distal.

o Physiologic dead space is the sum of anatomic and alveolar dead space. This reflects the

alveolar and respiratory tree dead space seen in healthy individuals.

o Pathologic dead space is seen classically in pulmonary embolism (as well as amniotic fluid,

fat, and air emboli), where blood flow is impeded by an embolus. V/Q will increase in an area

of lung simply because perfusion is blocked in that area. As you can see, this is quite distinct

from physiologic dead space.

- “Yeah but I’m still confused by shunt and dead space. Can you please elaborate a little more.” à A

shunt means a right to left effect of oxygenation due to insufficient alveolar ventilation – i.e., a mixing

of deoxygenated blood (right) with oxygenated blood (left) such that a patient’s net oxygenation is

less than what it should be because some of the alveoli aren’t receiving enough oxygen. This is

different from a R to L cardiac shunt, where deoxygenated blood in the right heart is literally mixing

with oxygenated blood in the left heart; in a pulmonary R to L shunt, deoxygenated blood from

insufficiently oxygenated pulmonary venules (coming from an area of lung that’s obstructed, e.g.,

from a peanut or mucous plugging) mixes with oxygenated blood from adequately oxygenated

pulmonary venules; this effect of deoxygenated blood being averaged in with oxygenated blood

creates a “R to L” effect; we call this R to L movement a shunt. This is made more confusing by the

existence of L to R cardiovascular “shunts,” which refer to pathologies such as VSD, ASD, PDA, and AV

fistulae, where the patient is not cyanotic/deoxygenated and has blood moving from the left

circulation to the right. However the term “shunt,” as applied to ventilation and perfusion in

MEHLMANMEDICAL.COM 42
MEHLMANMEDICAL.COM

pulmonology (i.e., when we say “what is shunt vs dead space?”) refers to a R to L process. It is not

sufficient to merely say a shunt is a R to L process, period, because pulmonary embolism (dead space,

not shunt) also ultimately results in deoxygenated blood mixing with oxygenated blood.

- “I’ve heard something about oxygen not helping in a pulmonary shunt. No idea what that means

though. Can you explain.” à A classic effect of a pulmonary shunt is an inability to effectively raise

arterial pO2 even when oxygen is administered. For instance, if a patient swallows a peanut, the area

of lung blocked off could be said to have a “zero” for oxygen (i.e., no ventilation). The result is: that

zero is mixed in with all of the other areas of normal lung à this means the average of all areas of

lung cannot achieve normal oxygenation because that zero is mixed in, so even if O2 is administered

and the remaining alveoli are highly ventilated and oxygenated, the net result is still an arterial

oxygenation that is insufficient. We use the peanut as an easy example to help visualize this process,

but when this is applied to, e.g., COPD or asthma, what occurs is many tiny alveolar pockets become

obstructed, with lots of mini shunts being formed; so we gets lots of tiny zeros throughout the lungs,

where even if we give oxygen and many alveoli have high pO2, the patient’s arterial pO2 remains low

because the net oxygen from all of the alveoli combined is low. These zeros being mixed in with

normal lung reflects our R to L process (i.e., the shunt).

- “Can you explain A-a gradient? Shitz annoying.” à A is alveolar oxygen; a is arterial oxygen. In

settings where the patient’s arterial oxygen is low, the A-a gradient (normally 5-10 mmHg) tells us

whether there’s a lung pathology impeding gas exchange or if the patient is merely hypoventilating. In

other words, a high A-a gradient means the patient has low arterial oxygen despite good alveolar

oxygen (i.e., something is impeding gas exchange); normal A-a gradient means low arterial oxygen

because of low alveolar oxygen (i.e., the patient merely isn’t breathing enough).

- Probably one of the highest yield points is knowing that opioids, benzos, and barbiturates cause a

normal A-a gradient because these agents cause respiratory depression. That is, if a patient is on

fentanyl for pain following surgery and has a low arterial O2, we know the low arterial O2 is because

the patient isn’t breathing adequately, not because there is defective gas exchange. The question

might not overtly tell you the respiratory rate is 6/minute; they’ll sometimes say it’s 12 (normal 12-

16), but the answer is still “normal A-a gradient”; adequate depth of respirations also matters. USMLE

MEHLMANMEDICAL.COM 43
MEHLMANMEDICAL.COM

also wants you to know that the mechanism for the patient’s hypoxemia in pulmonary edema is “high

A-a gradient”; this makes sense, as the transudate in the alveolar spaces impedes gas exchange, but

the patient’s ventilation is otherwise fine.

- My biggest advice is to not memorize lung pathology in general as synonymous with “high A-a

gradient”; as I mentioned above, USMLE will slam students on normal A-a gradient à e.g., patient

being weaned from a ventilator and has pCO2 of 70 mmHg (normal 33-44) à answer = normal A-a

gradient because the patient is merely hypoventilating (low arterial O2 because alveolar O2 is low).

The mistake I see students make is they just think “lung problem = A-a gradient must be high.” à No.

“Can you explain type I vs type II respiratory failure?” à type I is when the patient has low arterial

pO2 and low arterial pCO2; type II is when pO2 is low and PCO2 is high. The patient’s arterial pCO2

level (normally 33-44 mmHg) is the biggest giveaway of his or her state of ventilation. If the Q tells

you the patient’s pO2 is 70 mmHg (normal is 80-100 mmHg) and the pCO2 is 55 mmHg, you know

right away the answer is “alveolar hypoventilation” as the cause of the hypoxemia. For the

overwhelming majority of USMLE questions, if pCO2 is low, it means the respiratory rate is high; if

pCO2 is high, it means the respiratory rate is low (i.e., the relationship is almost always inversely

related). It must also be pointed out that CO2 diffuses quickly while O2 diffuses slowly. This means

the patient must have healthy lung in order to achieve adequate oxygenation – i.e., high respiratory

rate cannot compensate for a shunt or pathologic dead space in order to adequately restore arterial

pO2 to normal because O2 diffuses too slowly; however, even in the setting of lung pathology,

because CO2 diffuses quickly, insofar as the respiratory rate is high, CO2 can still get out, which is why

it will almost always be low when respiratory rate is high. I will talk about a notable exception shortly.

- “Yeah but how does this type I vs II respiratory failure stuff apply more specifically to questions?” à

Highest yield for USMLE is acute asthma attack, pulmonary embolism, and opioids:

o For acute asthma, the patient will have a type I respiratory failure: low pO2, low pCO2,

normal bicarb, high pH à in this setting, RR is high (e.g., 28/min), so pCO2 is low, not high,

because the CO2 can diffuse out with no problem; however O2 is low because the

bronchospasm and secretions prevent the slow-diffusing O2 from sufficiently entering the

capillary beds; bicarb is unchanged because it’s the acute setting; it will not normally start to

appreciably decrease for at least 12-24 hours (just think, similarly, it takes at least a day for

MEHLMANMEDICAL.COM 44
MEHLMANMEDICAL.COM

bicarb to decrease when one goes to high altitude, resulting in altitude sickness); pH is high

because of acute respiratory alkalosis (once again, bicarb hasn’t decreased to compensate).

o Classic USMLE scenario is asthma attack after 30-45 minutes, where the patient’s values are

as follows: low pO2, normal pCO2, normal bicarb, normal pH; the Q asks what you should

do: answer = intubation à reason being: the low O2 clearly means the patient is still in

respiratory distress, however the rising of CO2 into the normal range (and restoration of pH

to normal) indicates this is only because the patient is getting tired – i.e., RR is falling out of

fatigue, not because the patient’s O2 has stabilized. In this setting, the patient is in transition

to a type II respiratory failure. In other words, if we don’t intubate, the patient’s values will

soon become: low pO2, high pCO2, normal bicarb, low pH (i.e., respiratory acidosis; type II

respiratory failure).

o For pulmonary embolism, we also have a type I respiratory failure: low pO2, low pCO2,

normal bicarb, high pH, same as acute asthma. Bicarb is once again unchanged because it’s

the acute setting; pCO2 is low because RR is high; O2 is low because of pathologic dead

space (high V/Q à despite good ventilation, reduced perfusion results in reduced arterial

oxygenation).

o For opioids, we expect hypoventilation, so we have: low pO2, high pCO2, normal bicarb, low

pH (respiratory acidosis; type II respiratory failure), secondary to the patient’s low RR.

- “You said there’s an exception for CO2 and respiratory rate?” à 9 out of 10 Qs will give you a

scenario where the relationship between CO2 and RR is an inverse one. However there is also this

type of Q à 57M + COPD + RR 28 (NR 10-14) + pO2 50 mmHg (NR 80-100) + pCO2 80 mmHg (NR 33-

44) + bicarb 32 mmHg (NR 22-28) + pH 7.27 (NR 7.35-7.45); Dx? à answer = “chronic respiratory

acidosis and acute respiratory acidosis” (acute on chronic, but this is how they word it) à first things

first: patients with COPD are chronic CO2 retainers, so if this guy were not in acute distress here, we’d

expect his baseline values to be just chronic respiratory acidosis: low O2 (slightly reduced at 70s

mmHg), high CO2 (slightly elevated at 50s mmHg), high bicarb (reflective of chronic respiratory

acidosis, where the elevation in bicarb means the respiratory acidosis is chronic; if it were acute,

bicarb would be unchanged), and pH normal (chronic respiratory acidosis with metabolic

compensation; the high bicarb balances the high CO2). So in the case of the above bold values, we

MEHLMANMEDICAL.COM 45
MEHLMANMEDICAL.COM

say, “Wait, his RR is high though. How is his CO2 also high? That’s so weird.” Yeah, it is. But apparently

in COPD exacerbations, this is common. COPD (chronic obstructive pulmonary disease) = the

combination of chronic bronchitis and emphysema; regarding the latter, destruction of the alveolar

septa results in decreased surface area for gas exchange and decreased CO2 diffusion even when the

RR is elevated; the patient is a chronic CO2 retainer because he or she has decreased surface area for

gas exchange; most COPD exacerbations are due to viral infection, resulting in inflammation of the

bronchi and further decreased gas exchange. Another HY tangential point for USMLE is that

antibiotics are always given in COPD exacerbations, even though most cases are viral and patient is

afebrile.

- What do I need to know about FEV1/FVC? à normal ratio is considered >0.7 (according to AAFP);

ratio is decreased in obstructive lung disease (<0.7; <70%); ratio is normal or increased in restrictive

lung disease (>0.7; >70%); should be noted that in both obstructive and restrictive, FEV1 and FVC, as

independent variables, both decrease; it’s only the ratio that differs.

- Why is the FEV1/FVC ratio normal or increased in restrictive? à typical student response is, “Well

FEV1 doesn’t go down as much as FVC in restrictive.” But why is this the case? à answer that pops up

all over NBME exams is radial traction à i.e., if the restrictive pathology is caused by fibrosis / scar

formation, we could colloquially say that fibrosis is “sticky,” where it sticks to the outside of the

airways and keeps them from closing à the amount of air one breathes out in one second (FEV1)

doesn’t go down as much as it does in an obstructive lung disease, where radial traction doesn’t play

a role (so FEV1 goes down even more in obstructive pathologies à FEV1/FVC is low in obstructive).

- 59F + advanced COPD and lung cancer + requires partial pneumonectomy + FEV1 of left lung is

600mL; next best step? à answer = “measurement of FEV1 of the right lung” à seemingly weird

answer but HY for pulm component of surg shelves à minimum acceptable postoperative FEV1 for

both lungs is 800mL. “Okay…” Yeah, I know. Just memorize it.

- “What is obstructive vs restrictive lung disease?” à obstructive lung disease means air-trapping

occurs because there is literally obstruction in the airways, resulting in prolonged expiratory phase;

restrictive means there is decreased ability for the lungs to expand (decreased compliance) but

expiratory component remains unaffected in pure restrictive pathology.

MEHLMANMEDICAL.COM 46
MEHLMANMEDICAL.COM

- What are some common obstructive vs restrictive conditions?

o Obstructive: emphysema + bronchitis (combo = COPD); asthma.

o Restrictive: autoimmune diseases (many conditions can result in fibrosis, such as rheumatoid

arthritis [rheumatoid lung], systemic sclerosis [both diffuse and limited {CREST}],

sarcoidosis); Hx of radiation to the chest (i.e., Hx of Hodgkin) leading to fibrosis;

pneumoconioses (asbestosis, silicosis, etc.); ankylosing spondylitis; obesity; neuromuscular

disease (i.e., myasthenia gravis, Guillain-Barre).

- What is definition of chronic bronchitis? à productive cough every day for >3 months in a year for at

least two consecutive years; most common cause is smoking.

- 57M + 4-day Hx of productive cough + COPD managed with ipratropium inhaler + afebrile + HR 110 +

RR 22 + BP 130/80 + O2 sats 90% + breath sounds decreased bilaterally + CXR shows hyperinflation of

both lungs; Dx? à answer on NBME = “bronchitis” à i.e., acute bronchitis; usually viral.

- Classic way to differentiate COPD from asthma? à in asthma, spirometry showing decreased

respiratory function is largely reversible with albuterol; this is modestly effective at best in COPD;

“flattened diaphragm” is classic CXR descriptor for hyperinflated lungs seen in COPD; this descriptor

can be seen in asthma, but is rare.

- “What do I need to know about DLCO?” à stands for diffusion capacity of the lung for carbon

monoxide (correct, not dioxide); test measures difference in partial pressures of inspired vs expired

carbon monoxide; you just need to know that DLCO is decreased for most lung pathologies, however

it is increased in asthma; how this applies to questions à if you get a hard vignette (e.g., COPD vs

asthma, etc.) and they tell you DLCO is down, you can say, “Ok cool, it’s not asthma.”

- “What do we need to know in terms of lung volumes and flow-loop stuff for obstructive vs restrictive?

à in general, lung volumes are increased in obstructive lung pathologies and decreased in restrictive

ones. The following diagrams illustrate some of the changes in obstructive vs restrictive lung disease:

MEHLMANMEDICAL.COM 47
MEHLMANMEDICAL.COM

- 72F + longitudinal study comparing her lung function to 20-yr-olds’ lungs; what changes would we

expect in her versus young individuals? à answer = residual volume ­; arterial pO2 ¯; A-a pO2

MEHLMANMEDICAL.COM 48
MEHLMANMEDICAL.COM

gradient ­; essentially, people develop obstructive lung disease changes as they age (likely due to

lifelong inhalation of particulates, akin to smoking).

- 38F smoker + study performed comparing her to healthy individuals; what changes would we expect

in her compared to non-smokers à answer = mucous production and secretion ­; activity of airway

cilia ¯; alveolar macrophage function ¯; increased mucous production and secretion reflects changes

tending toward bronchitis; activity of airway cilia decreases with exposure to particulates; this point is

insidiously HY for USMLE Step 1; alveolar macrophage function and activity are impaired by

particulates (even though increased clearance necessary).

- 59M + 70-pack-yr Hx of smoking + Q asks which cell is likely to be abnormal in this patient; answer =

pseudostratified columnar epithelial cells; wrong answers are alveolar macrophages, type I/II

pneumocytes, alveolar endothelial cells; once again, USMLE wants you to know cilia function is

decreased; pseudostratified columnar epithelial cells of the respiratory tract are ciliated.

- What is one of the most important descriptors in a Q for restrictive lung disease? à reticulonodular

or reticular pattern seen on CXR and CT à memorize these descriptors as = restrictive lung disease;

they are also known as honeycombing colloquially.

- “When is centri-acinar vs pan-acinar emphysema important?” à you just need to know that centri-

acinar is the distribution seen in smoking; pan-acinar is seen in alpha-1 anti-trypsin deficiency.

- 32M non-smoker + reduced lung function + has a 40-year-old brother with COPD + father died of

alcoholic liver disease; Dx? à answer = alpha-1 anti-trypsin deficiency à enzyme is produced in liver

and travels to the lungs; functions to breakdown elastase; deficiency results in pan-acinar

emphysema (Q will mention non-smoker with emphysema); can also result in hepatic cirrhosis

(alcohol exacerbates already poor liver prognosis); codominant inheritance.

- “When is bronchiectasis the answer?” à when the vignette tells you “cups and cups of foul-smelling

sputum”; this is how bronchiectasis presents 8 out of 10 times; there is destruction of the

musculature of the bronchioles resulting in ectatic (dilated) airways à “loss of elastic support to the

walls of the bronchioles” (NBME); Q might also show you a CT scan (1 out of 10 Qs) where it appears

as though there’s dilated spaces throughout both lung fields; most common cause worldwide is TB; in

western countries it’s CF; smoking is also ubiquitous cause; the hardest presentation is pediatrics (1

MEHLMANMEDICAL.COM 49
MEHLMANMEDICAL.COM

out of 10 Qs), where the Dx is “right middle lobe syndrome” à Q will say there’s a 6-12-month Hx of

non-productive cough + a linear/streaky opacity seen on CXR in the right middle lobe à answer =

bronchiectasis. Student will say “wtf? I thought it was supposed to be cups and cups of sputum.”

Yeah, it normally is, but now you know of another presentation.

- 34F + non-productive cough + CXR shows hilar nodularity; Dx? à answer = “non-caseating

granulomas” à sarcoidosis; Tx = steroids; CXR will show bihilar lymphadenopathy; non-caseating

granulomas produce 1-alpha hydroxylase, which converts inactive 25-OH-D3 into active 1,25-(OH)2-

D3, leading to increased absorption of Ca and PO4 in the small bowel; Q will sometimes give PO4 in

the normal range; also need to know that fecal calcium is low (asked on NBME; makes sense if you

absorb it); PTH is also suppressed.

- Any drug that can be used to help diagnose asthma? à answer = methacholine (muscarinic agonist);

will reproduce asthma Sx; never use during acute attacks.

- 34F + non-productive cough + CXR normal; Dx? à answer = “activation of mast cell” à asthma.

- Asthma Tx (outpatient)? à albuterol (short-acting beta-2 agonist; SABA) inhaler for immediate Mx à

if insufficient, start low-dose ICS (inhaled corticosteroid) preventer à if insufficient, maximize dose of

ICS preventer à if insufficient, add salmeterol inhaler (long-acting beta-2 agonist; LABA); in other

words:

o 1) SABA; then

o 2) low-dose ICS; then

o 3) maximize dose ICS; then

o 4) LABA.

o The above order is universal. After the LABA, any number of agents can be given in any order

– i.e., nedocromil, cromolyn sodium, zileuton, montelukast, zafirlukast, etc.

o Last resort is oral corticosteroids, however they are most effective. In other words:

- 12M + ongoing wheezing episodes + is on albuterol inhaler; next best step? à answer = add low-dose

ICS.

- 12M + ongoing wheezing episodes + is on albuterol inhaler; what’s most likely to decrease

recurrence? à answer = oral corticosteroids (student says “wtf? I thought you said ICS was what we

MEHLMANMEDICAL.COM 50
MEHLMANMEDICAL.COM

do next and that oral steroids are last resort” Yeah, you’re right, but they’re still most effective at

decreasing recurrence. This isn’t something I’m romanticizing; this distinction is assessed on the FM

NBME forms.

- MOA of nedocromil and cromolyn sodium? à answer = mast cell stabilizers.

- MOA of zileuton? à answer = lipoxygenase inhibitor (enzyme that makes leukotrienes from

arachidonic acid).

- MOA of the -lukasts? à answer = leukotriene LTC, D, and E4 inhibitors. LTB4 receptor agonism is

unrelated and induces neutrophilic chemotaxis (LTB4, IL-8, kallikrein, platelet-activating factor, C5a,

bacterial proteins).

- 16M goes snowboarding all day + takes pain reliever for sore muscles afterward + next day develops

wheezing out on the slopes again; what’s going on? à answer = took aspirin + this is Samter triad

(now cumbersomely known as aspirin-exacerbated respiratory disease [AERD]) à triad of aspirin-

induced asthma + aspirin hypersensitivity + nasal polyps). Just to be clear, other NSAIDs can

precipitate Samter triad, but the literature + USMLE will make it explicitly about aspirin.

- 16M takes aspirin + gets wheezing; what are we likely to see on physical exam? à answer on USMLE

= nasal polyps.

- “Wait I don’t understand. Why would aspirin cause asthma?” à arachidonic acid can be shunted

down either the cyclooxygenase or lipoxygenase pathways; if you knock out COX irreversibly by giving

aspirin (or reversibly with another NSAID), more arachidonic acid will be shunted down the

lipoxygenase pathway à more leukotrienes à more bronchoconstriction.

- Kid has Hx of AERD; physician considers agent to decrease his recurrence of Sx à zileuton, or -lukasts

(both are correct; and only one will be listed).

- 10F + diffuse wheezes heard bilaterally + sputum sample shows numerous eosinophils and Charcot-

Leyden crystals; which of the following is implicated in her disease pathogenesis? à answer =

leukotriene C4 (LTC4); should be noted that LTC4, LTD4, and LTE4 all cause bronchoconstriction; LTB4

stimulates neutrophilic chemotaxis; Charcot-Leyden crystals are seen in asthma and are composed to

eosinophil protein galectin-10.

- Any weird asthma Txs? à omalizumab à monoclonal antibody against IgE à used for intractable,

severe asthma unresponsive to oral steroids + in patients who have eosinophilia + high IgE levels (I

MEHLMANMEDICAL.COM 51
MEHLMANMEDICAL.COM

asked a pulmonologist about this drug years ago when I was in MS3 and he said he was managing

1000 patients with asthma and just three were on omalizumab).

- Acute asthma Mx (emergencies) à most important piece of info straight-up is: USMLE wants you to

know that inhaled corticosteroids (ICS) have no role in acute asthma management. First thing we do

is give oxygen (any USMLE Q that shows depressed O2 sats, answer is always O2) + nebulized

albuterol (face mask with mist); IV steroids are then administered. The Mx algorithm is more

complicated, but that is what you need for the USMLE.

- 25F + runny nose and eyes in spring + cobblestoning of nasal mucosa; Dx + Tx? à answer = allergic

rhinitis (“cobblestoning” is buzz term but used on the NBME); Tx = avoidance of precipitating allergen

(if known, e.g., pollen); if meds used, first-line = intranasal corticosteroids; second-line is oral or

intranasal antihistamines (e.g., loratadine).

- 27M + inhalation injury from housefire + Q asks “destruction to which structure is most likely to

preclude restoration of normal pulmonary architecture and lung function?” à answer = basement

membranes à USMLE wants you to know intact basement membranes necessary for complete

healing from lung injury or infection.

- 44M alcoholic + fever + air-fluid level seen on CXR; Dx + TX? à answer = pulmonary abscess; Tx =

clindamycin; air-fluid level = circle on CXR, where bottom half is pus (radiopaque; white); top half is

air (radiolucent; black); due to aspiration of “oropharyngeal anaerobes” (bacteroides, Mobiluncus,

Peptostreptococcus).

- 44M alcoholic + fever + CXR shows lobar consolidation + sputum is thick and red; Dx? à Klebsiella

pneumoniae (produces thick, mucoid colonies with “currant jelly sputum”); common cause of

aspiration pneumonia (increased risk in alcoholics).

- 44M alcoholic + fever + CXR shows lobar consolidation + extremely foul-smelling sputum; Dx? à

answer = Bacteroides pneumonia.

- 44M alcoholic + fever + air-fluid level seen on CXR + clindamycin is given and resolves his condition +

months later he has residual lesion seen on CXR; why? à answer = failure of maintenance of

basement membranes.

- Function of type I vs II pneumocytes? à type I composes 95% of alveolar surface area (simple

squamous) and is responsible for gas exchange; type II pneumocytes are interspersed and fewer; they

MEHLMANMEDICAL.COM 52
MEHLMANMEDICAL.COM

produce surfactant (contain specialized surfactant-producing organelles called lamellar bodies) and

also function as the stem cells of the lung (i.e., after lung injury, type II multiply and restore type I

cells, insofar as basement membranes remain intact).

- What are Clara cells à secrete glycosaminoglycans to protect the lining of bronchioles.

- Child born at 28 weeks gestation + dyspnea + Q asks which of the following is likely true in the patient

à answer = “deficiency of lamellar bodies” à neonatal respiratory distress syndrome (NRDS); aka

hyaline membrane disease à deficiency of surfactant à reduced lecithin (dipalmitoyl

phosphatidylcholine) to sphingomyelin ratio; ratio should be >2.

- Child born at 28 weeks gestation + dyspnea + Q asks which of the following is likely true in the patient

à answer = deficiency of surfactant protein D à need to know this is decreased in NRDS.

- Neonate born at 38 weeks gestation via C-section + RR of 70 (normal 40-60) + CXR shows bilateral

mild hyperinflation and prominent perihilar interstitial markings ; Dx? à answer = transient

tachypnea of the newborn à TTN is the answer when the vignette “sounds like NRDS but the kid is

term”; seen in C-sections and fast vaginal deliveries in term neonates; mechanism is delayed

absorption of the fetal lung fluid by pulmonary lymphatics; Tx is supportive.

- Neonate born at 26 weeks + required oxygen in ICU for several weeks + is now on home oxygen; child

is at increased risk for what? à answers = bronchopulmonary dysplasia, retinopathy of prematurity,

germinal matrix bleed à all can be seen in neonates on oxygen therapy.

- Newborn + decreased bowel sounds in abdomen + tracheal shift to the right; Dx? à answer =

congenital diaphragmatic hernia à incomplete formation of pleuroperitoneal membranes à occurs

on the left side, with bowel herniating into the left hemithorax.

- Newborn + bowel sounds heard in the left chest; mechanism? à answer = incomplete formation of

pleuroperitoneal membranes.

- 12-hour-old newborn + excessive oral secretions and coughing after first feed + Sx resolved after

suctioning + pregnancy was characterized by polyhydramnios; next best step in Mx? à answer =

insertion of a nasogastric tube à Dx = tracheoesophageal fistula.

- 12-hour-old newborn + becomes blue when breastfeeding + becomes pink again when crying; Dx? à

atresia of the choanae (CHARGE syndrome à Coloboma of the eye, Heart defects, Atresia of the

choanae, Retardation, Genitourinary abnormalities, Ear abnormalities/deafness).

MEHLMANMEDICAL.COM 53
MEHLMANMEDICAL.COM

- Can you comment on HY pneumoconioses? à lung disease secondary to occupational exposures.

o Asbestosis à workers in shipbuilding, plumbing, construction/insulation, roofing industries

à causes supradiaphragmatic and pleural plaques (described as “soft tissue density”

visualized on CXR on 2CK NBME); plaques can calcify; biopsy may show ferruginous bodies

(due to deposition of hemosiderin by alveolar macrophages attempting to digest asbestos

fibers); Step 1 NBME Q asks which cell is responsible for the pulmonary fibrosis seen in

asbestosis à answer = macrophage.

o Anthracosis à “coalminer’s lung”; carbon deposition; black appearance grossly; also what

city-dwellers may develop by old age; restrictive lung pattern on spirometry.

o Silicosis à stone quarry workers; eggshell calcifications; increased risk of TB due to

decreased alveolar macrophage function; do not give anti-TNF-alpha agents to these patients

(because they further increase risk of TB); do PPD test if new Dx of silicosis; restrictive on

spirometry.

o Berylliosis à aeronautical industry workers; granulomatous disease, so steroids might

provide benefit; steroids do not help in the other pneumoconioses.

o Byssinosis à pneumoconiosis caused by hemp / textile fibers.

o Bagassosis à pneumoconiosis caused by sugarcane (specific antigen = thermophilic

Actinomycetes).

o Caplan syndrome à pneumoconiosis + rheumatoid arthritis (restrictive).

- 34M hay farmer + progressive dyspnea over many months + clubbing; Dx? à answer =

hypersensitivity pneumonitis; can be acute, subacute, or chronic; occurs in response to various

antigens, including thermophilic Actinomycetes (hay farmers; sugarcane); non-tuberculous

mycobacterium (hot tub lung; on NBME); bird fancier’s lung, coffee worker’s lung, etc.; characterized

by multinucleated giant cells on biopsy.

- 28M + asthma + recurrent lung infections + skin allergy testing shows hypersensitivity to aspergillus

antigen; Dx? à allergic bronchopulmonary aspergillosis (ABPA); usually seen in patients who have

asthma or cystic fibrosis.

- 34M + HIV positive + nodular density seen in right upper lobe on CXR; next best step? à answer =

“biopsy of the mass” à Aspergilloma (fungus ball); increased risk with Hx of TB (can occupy cavities).

MEHLMANMEDICAL.COM 54
MEHLMANMEDICAL.COM

- 57M + Hx of dermatomyositis + few weeks ago had fever and pneumonia + presentation hasn’t

resolved with multiple antibiotics + multiple sputum cultures are negative; Dx? à answer =

cryptogenic organizing pneumonia (COP; formerly known as bronchiolitis obliterans organizing

pneumonia [BOOP]); à frequently presents as pneumonia-like presentation (i.e., fever, dyspnea) in

patients with Hx of autoimmune disease like dermatomyositis or rheumatoid arthritis; CXR resembles

atypical pneumonia; COP diagnosis suspected after failure of resolution with multiple Abx and

negative sputum cultures; CT shows reverse-halo sign in 20%; biopsy showing Masson bodies

confirms Dx; Tx = steroids. “Organizing” refers to persistence of alveolar exudates from a pneumonia

that ultimately fibrose.

- 43F + lung transplant several months ago + declining lung function + CXR shows mild flattening of the

diaphragm + CT shows air-trapping; Dx? à bronchiolitis obliterans (constrictive bronchiolitis;

“popcorn lung”); do not confuse with COP (BOOP); bronchiolitis obliterans is the answer for

obstructive lung disease that progressively manifests post-lung transplant (75%) or bone marrow

transplant; also associated with toxic fumes and E-cigarettes; biopsy confirms Dx; irreversible, but

steroids may help.

- 43F + rheumatoid arthritis + progressive shortness of breath over six months + CT of chest shows

reticulonodular pattern; Dx? à answer = “usual interstitial pneumonia” (UIP); student says wtf? à on

the NBME; this Dx refers to a patient with scarring and fibrosis of the lungs; if the cause is idiopathic,

we call the UIP “idiopathic pulmonary fibrosis,” but IPF is still UIP. Bottom line is: be aware of the

term “usual interstitial pneumonia” as synonymous with pulmonary fibrosis.

- Drugs causing pulmonary fibrosis? à methotrexate (patients with RA who have UIP à hard to know

whether it’s from the methotrexate or rheumatoid lung, or both); amiodarone, bleomycin, busulfan,

nitrofurantoin.

- 10-month-old + fever of 101F + wheezes bilaterally; DX + Tx? à RSV bronchiolitis à Tx is supportive;

answers such as ribavirin and palivizumab are almost always wrong on actual NBME assessment.

- 8M + leaning forward in tripod position + drooling; Dx + Tx? à answer = epiglottitis caused by

Haemophilus influenzae type B; Tx = immediate intubation; neck x-ray shows thumbprint sign; even if

kid is stable, still intubate (airway can inflame + obstruct at any moment); give ceftriaxone to patient;

give rifampin to close contacts; usually seen in immigrants due to lack of adequate vaccination.

MEHLMANMEDICAL.COM 55
MEHLMANMEDICAL.COM

- 8M + hoarse cough + improves when father brings him out in the cold; Dx + Tx? à answer = croup

(laryngotracheobronchitis) caused by parainfluenza virus (paramyxovirus); classically “seal-like,

barking cough”; neck x-ray shows steeple sign (subglottic narrowing); Tx = supportive; if you’re forced

to pick an actual Tx however, choose “nebulized racemic epinephrine.”

- 8M + two weeks ago recovered from influenza infection + now has fever + barking cough + inspiratory

stridor; Dx? à answer = bacterial tracheitis caused by Staph aureus. Dx with bronchoscopy; Tx with

antibiotics.

- 40M + 6-month Hx of episodic sinusitis + cough with blood-tinged sputum + no improvement of Sx

after several months of antibiotics, decongestants, and nasal corticosteroids + P/E shows two small

ulcerations on nasal mucosa; Dx? à answer = eosinophilic granulomatosis with polyangiitis (formerly

known as Churg-Strauss) à presents as asthma-like Sx and eosinophilia in a patient with positivity for

p-ANCA (anti-myeloperoxidase; anti-MPO); overlap of features with granulomatosis with polyangiitis

(formerly known as Wegener, which is c-ANCA [anti-proteinase 3; anti-PR3]).

- 44M + hemoptysis + hematuria + ANCA screen is negative; next best step? à answer = anti-

glomerular basement membrane antibody screen à Dx likely Goodpasture; if anti-GBM antibodies

positive, do renal biopsy showing linear immunofluorescence; anti-GBM Abs = antibodies against type

IV collagen (“2, 3, 4; 2, 3, 4; 2, 3, 4 – The Goodpasture is marching in the field! - 2, 3, 4; 2, 3, 4; 2, 3, 4”

à type II sensitivity against the alpha-3 chains of type IV collagen).

- How to Dx TB?

o PPD skin test is performed first diagnostically. If history of BCG vaccine, do interferon-gamma

release assay (IGRA) instead. Do not do IGRA in addition to PPD.

o If PPD is negative, repeat after one week. If negative again, no further studies indicated.

Repeats performed within 1 week may cause a false (+) secondary to a "booster reaction."

o If IGRA is negative, no further studies indicated.

o If PPD or IGRA is positive, do CXR. Do not repeat positive PPD tests.

o If CXR is negative, treat for latent TB / give TB prophylaxis. On the USMLE, "treatment for

latent TB" and "administer TB prophylaxis" mean the same thing.

o If CXR is positive, treat for active TB.

- How to manage a positive PPD test?

MEHLMANMEDICAL.COM 56
MEHLMANMEDICAL.COM

o Measure induration only. Erythema does not count.

- 5+ mm

o Recent contact with people with active TB

o HIV + status

o Organ transplant recipients

o Chronic prednisone use (>15mg/day for >1 month); anti-TNF-α agent use

o Findings consistent with TB on CXR

- 10+ mm

o Immigrant status (Western countries not included)

o IV drug users

o Healthcare workers; prison workers; homeless shelter personnel

o TB laboratory personnel

o Children under 4 years of age

- 15+ mm

o Everyone

- How to Tx latent TB?

o 9 months INH + pyridoxine (vitamin B6) - The USMLE Steps 1 and 2CK assess this as the

answer.

o 4 months rifampin

o 3 months INH + rifapentine + pyridoxine

o Vitamin B6 must be given with INH to prevent vitamin B6 deficiency.

- Tx of active TB

o Rifampin, INH, pyrazinamide, ethambutol (RIPE) for 2 months, followed by RI alone for 4

more months (6 months total)

o And of course add pyridoxine (annoying that it sounds similar to pyrazinamide)

- 32M + fever of 101F + CXR shows bilateral interstitial infiltrates; Dx? à answer = Mycoplasma

pneumoniae; most common cause of atypical pneumonia after viruses; classically bilateral.

MEHLMANMEDICAL.COM 57
MEHLMANMEDICAL.COM

- 32M + fever 101F + CXR shows lobar consolidation + dullness to percussion on P/E; Dx? à answer =

Streptococcus pneumoniae à need to know is gram-positive diplococci (don’t confuse with Neisseria

gonorrhea and meningitides, which are gram-negative diplococci).

- 32M + fever 101F + CXR shows lobar consolidation with interstitial markings; Dx? à answer =

Mycoplasma (on one of the 2CK NBMEs; Strep pneumo wasn’t listed) à likely implication is that even

though Strep pneumo is classically lobar and Mycoplasma bilateral, if the Q says the word

“interstitial,” that wins over distribution/location.

- Pneumonia + diarrhea + hyponatremia; Dx? à answer = Legionella pneumophila.

- Pneumonia in a cattle farmer; Dx? à answer = Coxiella burnetti.

- Pneumonia in someone with a pet rabbit; Dx? à answer = Francisella tularensis.

- Pneumonia in bird owner; Dx? à answer = Chlamydia psittaci.

- Pneumonia in 2-week-old neonate who had ophthalmia neonatorum treated a week ago; Dx? à

answer = Chlamydia trachomatis pneumonia à chlamydial neonatal conjunctivitis à drains through

nasolacrimal duct into nasopharynx and into the lung.

- Pneumonia in someone who recently recovered from viral infection (usually flu); Dx? à answer =

Staph aureus à similar to post-viral bacterial tracheitis caused by Staph.

- Lobar pneumonia in HIV patient; Dx? à answer = Strep pneumo, not Pneumocystis.

- HIV patient + CXR shows bilateral ground-glass pneumonia; next best step in Dx? à answer =

bronchoalveolar lavage with silver-staining of the yeast; Dx? à Pneumocystis jirovecii à Tx =

TMP/SMX.

- Empiric Tx for community-acquired pneumonia (CAP)? à answer = azithromycin (macrolide) à

covers atypicals (i.e., Mycoplasma, etc.) and S. pneumo.

- Empiric Tx for CAP when patient’s had antibiotics in the past three months? à answer = respiratory

fluoroquinolone (i.e., levofloxacin).

- How do we Tx CAP if admitting patient to hospital?

o If non-ICU: fluoroquinolone, OR beta-lactam + macrolide.

o If ICU: beta-lactam + EITHER macrolide or fluoroquinolone.

MEHLMANMEDICAL.COM 58
MEHLMANMEDICAL.COM

o If septic, ceftriaxone +/- vancomycin frequently given à ceftriaxone common singular Abx

choice in sepsis; very effective against S. pneumo in particular; ceftriaxone as monotherapy is

all over the 2CK-level NBMEs for Tx of sepsis (not just CAP); ceftriaxone + vancomycin is a

combo is the answer on one of the newer forms for septic patient with CAP (increasing

resistance of S. pneumo to ceftriaxone); if patient is child under 6, give cefotaxime instead of

ceftriaxone (less displacement of bilirubin from albumin; HY for 2CK NBMEs).

- When is pneumonia considered hospital-acquired (HAP)? à if Sx start >48 hours after admission to

hospital.

- How is HAP and ventilator-acquired pneumonia (VAP) Tx differently from CAP? à need to cover

MRSA and Pseudomonas just in case à Tx entails various broad-spectrum options; classics are

vancomycin + third- or fourth-generation cephalosporin; ceftazidime (3rd gen ceph) and cefepime

(4th gen ceph) are effective against Pseudomonas; piperacillin/tazobactam (“PipTaz”) is classic

combo; amikacin (aminoglycoside) is effective against Pseudomonas; ceftaroline and ceftobiprole are

5th gen cephs effective against MRSA; meropenem (or imipenem/cilastatin) also used for HAP/VAP.

- First-line Tx in COPD? à SABA or SAMA (short-acting muscarinic antagonist; ipratropium) à answer

on one of the 2CK NBMEs is just straight-up ipratropium (home oxygen and pulmonary rehabilitation

are wrong answers for first-line Tx).

- When to do home oxygen therapy in COPD? à when patient’s arterial pO2 <60 mmHg (or <55 mmHg

if patient has cor pulmonale).

- 55M with COPD; number-one way to decrease mortality? à answer = smoking cessation; if patient

has already stopped smoking, answer = home oxygen therapy (decreases mortality, but only indicated

as per criteria above).

- 48M + BMI 45 + snores loudly during sleep; what is most likely seen in this patient? à answer =

increased serum bicarbonate à Dx = obesity hypoventilation syndrome à blood levels of CO2 are

increased, with bicarb increased to compensate (chronic respiratory acidosis).

- 8-month-old girl + stridor that improves with neck extension; Dx? à answer = vascular ring à weird

but HY diagnosis for peds à aberrant embryologic development where the aorta and/or surrounding

vessels form a ring around the esophagus and/or trachea.

MEHLMANMEDICAL.COM 59
MEHLMANMEDICAL.COM

- 8-month-old girl + stridor that improves when prone or upright; Dx? à laryngomalacia à most

common cause of stridor in peds à soft cartilage of upper larynx collapses during inhalation.

- 3F + 2-wk Hx of cough and nasal congestion + snoring loudly past 6 months + P/E shows she breathes

predominantly through her mouth + 1/6 holosystolic murmur and loud S2 + CXR shows cardiomegaly

+ increased pulmonary vascular markings + echo shows RV hypertrophy and mild tricuspid regurg;

what is the most appropriate long-term Mx for this patient? à answer = adenoidectomy and

tonsillectomy à sounds weird, but HY for Peds shelf à can cause obstructive lung disease with cor

pulmonale à loud S2 and increased pulmonary vascular markings suggest pulmonary HTN; tricuspid

regurg can be seen sometimes in pulmonary HTN.

- 2F + stridor + laryngoscopy shows small growths of larynx; Dx? à answer = HPV 6/11 à laryngeal

papillomatosis.

- Highest yield points about cystic fibrosis? à autosomal recessive; chromosome 7; CFTR gene; codes

for chloride channel that functions to secrete chloride in the lungs and pancreas, and reabsorb

chloride in the sweat glands; DF508 (deletion of phenylalanine at position 508) is most common

mutation; sweat chloride test >60 mEq/L is most diagnostic (more than genotyping); mutated CFTR

channel is usually retained in rough endoplasmic reticulum in the cytosol (i.e., doesn’t make it to cell

surface); disease causes a negative transepithelial potential difference (TEPD) across nasal epithelium;

neonates can be screened with a positive blood immunoreactive trypsinogen; Pseudomonas eclipses

S. aureus as most common cause of pneumonia after age 10; before age 10, S. aureus eclipses

Pseudomonas; male infertility due to congenital bilateral absence of vas deferens (CBAVD); various

Txs exist, however a couple to be aware of: dornase-alfa is a deoxyribonuclease that can help reduce

the viscosity of mucous secretions; Ivacaftor is a CFTR potentiator that helps restore function of the

misfolded protein.

- 32M + Hx of recurrent lung infections + two years of inability to have children with wife + wife has

two children from prior marriage + his sperm sample shows immotile sperm; Dx? à answer =

Kartagener syndrome (primary ciliary dyskinesia) à dynein arm defect of cilia (a cilium on cross-

section has a 9x2 arrangement of microtubules); associated with dextrocardia / situs inversus; sperm

are immotile (require cilia function); this contrasts with CF, which has no sperm in the sample due to

CBAVD.

MEHLMANMEDICAL.COM 60
MEHLMANMEDICAL.COM

- What should I know about breath sounds, percussion, tracheal shift, etc.? Basically all of that

annoying physical exam stuff.”

- Pleural effusion in someone who has pneumonia; Dx? à answer = parapneumonic effusion

(exudative pleural effusion secondary to a lung infection).

- What is empyema? à pus in a preexisting cavity (i.e., refers to pus in the pleural space); in contrast,

an abscess is pus in a location where there was not a preexisting cavity (e.g., the forearm).

- Characteristic of parapneumonic effusion most predictive of progression to empyema? à answer =

decreased pH of the pleural fluid (<7.1).

- Transudative vs exudative pleural effusion? à Light’s criteria for exudative:

o Pleural fluid to serum protein ratio >0.5, OR

o Pleural fluid to serum LDH ratio >0.6, OR

o Pleural fluid LDH > 2/3 upper limit of serum LDH.

- Any other characteristics notable for transudative vs exudative? à qualitatively, in exudative, pleural

fluid cell count will be higher than in transudative; glucose can also be reduced in infective causes

(parapneumonic).

- 44M + long Hx of smoking + dullness to percussion 2/3 up the lung field; Dx? à answer = malignant

pleural effusion à malignancy, congestive heart failure, and infection are very common causes of

pleural effusion.

MEHLMANMEDICAL.COM 61
MEHLMANMEDICAL.COM

- 32F + ovarian fibroma + ascites + hydrothorax; Dx? à answer = Meig syndrome.

- Treatment for pulmonary embolism? à answer = heparin before spiral CT of the chest; if pregnant,

do V/Q scan instead of CT (sufficient for USMLE).

- Treatment for pulmonary embolism? à answer = heparin before spiral CT of the chest; if pregnant,

do V/Q scan instead of CT (sufficient for USMLE).

- When is IVC filter the answer for pulmonary embolism? à if the patient gets a PE while already on

anticoagulation (warfarin, dabigatran, etc.) and the spiral CT has already been performed. In other

words:

o 58F on warfarin + gets PE; next best step? à answer = CT.

o 58F on warfarin + gets PE + CT confirms Dx; next best step? à answer = IVC filter.

- 32M + plays basketball + few hours of shortness of breath; Dx + Tx? à spontaneous pneumothorax

caused by ruptured subapical bleb; Tx = needle decompression followed by chest tube (some small

pneumothoraces can be observed, but on USMLE always Tx).

- 32M + MVA + dyspnea + low BP + breath sounds decreased on left + tracheal shift to right; Dx + Tx?

à answer = tension pneumothorax; Tx = needle decompression followed by chest tube.

- “Cephalization of pulmonary vasculature”; Dx? à pulmonary edema à important description that

shows up in NBME vignettes à due to increased hydrostatic pressure from left heart pathology.

- 36M + long-bone fractures + petechiae on the chest; Dx? à answer = fat embolism.

- Motor vehicle accident (MVA) + paradoxical breathing (chest moves outward with exhalation; inward

with inhalation); Dx? à answer = flail chest.

- MVA + rib fractures + underlying infiltrates in lung + low O2 sats; Dx? à answer = pulmonary

contusion.

- MVA + no rib fractures + non-central chest pain + pulmonary infiltrates underlying the painful area;

Dx? à answer = pulmonary contusion (resources will say “white out of the lung” for pulmonary

contusion, but this is buzzywordy and never shows up on actual NBME material).

- MVA + pulmonary infiltrates + low O2 sats + bolus of normal saline given, resulting in worsening of O2

sats; Dx? à answer = pulmonary contusion (contused lung is very sensitive to fluid overload).

- MVA + bruising/pain over the sternum +/- rib fractures; Dx? à answer = myocardial contusion.

MEHLMANMEDICAL.COM 62
MEHLMANMEDICAL.COM

- MVA + bruising/pain over sternum + pulmonary infiltrates + O2 sats get worse when saline is given;

Dx? à answer = myocardial contusion (“Wait, but I thought you said that latter finding means

pulmonary contusion”) à it does, and it’s HY for pulmonary contusion, but “bruising/pain over the

sternum” wins if it’s listed; this is on a 2CK NBME.

- Important point about Mx of myocardial contusion? à do troponins + must monitor for arrhythmia.

- Adult male + abdo pain + Hx of alcohol use + diffuse pulmonary infiltrates + low O2 sats; Dx? à

answer = ARDS à must have pO2/FiO2 <300; if the Q asks about ventilator settings, know that low-

tidal volume mechanical ventilation is often used (prevents barotrauma) + permissive hypercapnia +

prone positioning.

- When you get a random ventilator Q and they want an answer? à “increase PEEP” almost always

right.

- Patient has improving O2 sats on ventilator; next best step? à “wean from ventilator.”

- 18M + Hx of asthma + 2-day Hx of right cheek tenderness + Hx of several pneumonias and sinusitis

occurrences; Dx? à answer = IgA deficiency à answer will often be listed as “impaired humoral

immunity” or “deficiency of mucosal immunoglobulin”; sore cheek = classic for sinusitis; presents as

recurrent sinopulmonary infections; also associated with Hx of Giardia infection, autoimmune

diseases (e.g., vitiligo), and atopy (dry cough in winter [cough-variant asthma], hay fever in spring,

eczema in summer); anaphylaxis with blood transfusion is “too easy” for most 2CK IgA deficiency Qs

but will rarely show up, yes.

- How to differentiate viral from bacterial upper respiratory tract infection (URTI)? à CENTOR criteria:

o If 0 or 1 point, the URTI is unlikely to be bacterial (i.e., it’s likely to be viral). If 2-4 points,

chance is much greater that URTI is bacterial.

o 1) Absence of cough (i.e., no cough = 1 point; if patient has cough = 0 points).

o 2) Fever.

o 3) Tonsillar exudates.

o 4) Lymphadenopathy (cervical, submandibular, etc.).

- There is a version of the criteria that includes age, but on the USMLE it can cause you to get questions

wrong. So just use the simplified above four points.

MEHLMANMEDICAL.COM 63
MEHLMANMEDICAL.COM

o If 0-1 point, answer = “supportive care”; or “no treatment necessary”; or “warm saline

gargle” (same as supportive care); or “acetaminophen.” Latter is answer for 3M with viral

URTI + fever on Peds NBME form 2.

o If 0-2 points, next best step = “rapid Strep test.” If rapid Strep test is negative, answer =

throat culture, NOT sputum culture.

o While waiting on the throat culture results, we send the patient home with amoxicillin or

penicillin for presumptive Strep pharyngitis.

o If child is, e.g., 12 years old, and develops a rash with the beta-lactam, answer = beta-lactam

allergy.

o If the vignette is of a 16-17 year-old who has been going on dates recently (there will be no

confusion; the USMLE will make it clear), the answer = EBV mononucleosis; therefore do a

heterophile antibody test (Monospot test).

o EBV is the odd virus out that usually presents with all four (+) CENTOR criteria.

o This is why it’s frequently misdiagnosed as Strep pharyngitis. It is HY to know that beta-

lactams given to patients with EBV may cause rash via a hypersensitivity response to the Abx

in the setting of antibody production to the virus. EBV, in a patient who does not receive

Abx, can cause a mild maculopapular rash. But the rash with beta-lactam + EBV causes a

more intense pruritic response generally 7-10 days following Abx administration on the

extensor surfaces + pressure points.

- Which parameters shift the Hb-O2 dissociation curve to the right? à ­ temperature; ­ 2,3-BPG; ­

CO2; ¯ pH; ­ H+ à right-shift means increased oxygen unloading at tissues; protons in the blood are

buffered by deoxygenated hemoglobin (Step 1 NBME).

- 50M + arterial pO2 normal + arterial O2 content low; why the latter? à answer = anemia; arterial O2

content = amount of O2 dissolved in blood (pO2) + amount of O2 bound to Hb (Hb saturation); if pO2

is normal, then it should be able to bind to Hb just fine; therefore likely way O2 content is still low is if

Hb levels are low (anemia).

MEHLMANMEDICAL.COM 64
MEHLMANMEDICAL.COM

YouTube
@mehlmanmedical

Instagram
@mehlman_medical

MEHLMANMEDICAL.COM 65
MEHLMANMEDICAL.COM

MEHLMANMEDICAL
HY PULMONARY

All material is copyrighted and the property of mehlmanmedical.

Copyright © mehlmanmedical

MEHLMANMEDICAL.COM 66
MEHLMANMEDICAL
HY NEURO
MEHLMANMEDICAL.COM

YouTube
@mehlmanmedical

Instagram
@mehlman_medical

MEHLMANMEDICAL.COM 2
MEHLMANMEDICAL.COM

HY Neuro

Purpose of this review is not to be an unabridged, superfluous, 500-page neuro textbook for MS1 and MS2; the purpose is
to increase your USMLE and Neuro shelf scores via concise factoid consolidation.

- 33F + 1-wk Hx of R-sided weakness + L-sided inability to feel temperature/pain + diminished vibratory

sensation over R foot + brisk R-sided reflexes + R-sided Babinski sign; Dx? à answer on NBME is

Brown-Sequard syndrome secondary to viral infection.

- 59F + metastatic cancer + in pain + crying + “wants to die”; Q asks most likely reason for wanting to

die; answer = “inadequate pain control”; “major depression” is wrong answer; must address pain

management in cancer patients.

- 42F + 3-month Hx of insomnia + discomfort while lying in bed; next best step in management? à

check serum iron and ferritin levels; student says wtf? à restless leg syndrome is most often caused

by iron deficiency.

- 42F + 3-month Hx of insomnia + discomfort while lying in bed + serum iron and ferritin are normal;

next best step in management? à D2 agonist – i.e., pramipexole or ropinirole, etc.

- Patient with restless leg syndrome is at increased risk for what disease later in life? à answer on

USMLE = Parkinson disease (if D2 agonist can Tx, then lack of dopamine transmission may be etiology

in some patients).

- 58M + loses consciousness while shaving + tilt-table test shows no abnormalities; Dx? à NBME

wants “carotid sinus hypersensitivity” as answer. If tilt-table test (+), answer = vasovagal syncope.

- 45F + fundoscopy shows hard exudates + cotton wool spots + scattered hemorrhages; Dx? à diabetic

retinopathy.

- Medication that can cause tardive dyskinesia that is not an antipsychotic? à answer =

metoclopramide (D2 antagonist); can also prolong QT interval and cause hyperprolactinemia.

- Anti-depressant med causing seizures? à bupropion.

- Frontal lobe injury in car accident; NBME asks which deficit is most likely to ensue; answer =

conceptual planning.

- 56M + alcoholism + acutely intoxicated + B1 is administered; the latter decreases what most

significantly? à Neuro shelf wants “anterograde amnesia” as the answer; mnemonic for Wernicke =

A COW à Ataxia, Confusion, Ophthalmoplegia, Wernicke.

MEHLMANMEDICAL.COM 3
MEHLMANMEDICAL.COM

- 56M + 3-day Hx of cutting from 12 beers a day down to 4; develops tremulousness; Tx? à

chlordiazepoxide (delirium tremens); classic vignette is guy has surgery + two days later has

tachycardia, tremulousness, and hallucinations (alcoholic hallucinosis).

- 50F + high ESR and creatine kinase (CK) + proximal muscle weakness +/- muscle pain; Dx? à

polymyositis.

- 50F + high ESR + no mention of high CK or weakness in the vignette + muscle pain + muscle stiffness;

Dx? à polymyalgia rheumatica (PMR).

- Main difference between PMR and polymyositis? à PMR has no proximal muscle weakness + a

normal creatine kinase; polymyositis has high CK and weakness; pain + stiffness of muscles can be

seen in both conditions, but classically PMR. For USMLE vignettes + neuro shelf, focus on whether

there’s weakness or elevated CK.

- 48F + high ESR and creatine kinase (CK) + proximal muscle weakness +/- muscle pain; next best step in

Dx? à Neuro shelf wants either “anti-Jo1 / -Mi2 antibodies” or “electromyography and nerve

conduction studies” as the answer to Dx polymyositis. After these, do muscle biopsy can be

performed for definitive Dx. In contrast, no specific Dx test is used for PMR.

- 59F + temporal headache + muscle pain and stiffness + high ESR; Dx? à temporal arteritis +

polymyalgia rheumatica à next best step? = IV methylprednisolone first, followed by temporal artery

biopsy.

- 68M + Hx of prostate cancer + now has neurologic findings; next best step in Dx? à MRI of spine to

look for mets à however if corticosteroids are listed, choose these before MRI à for spinal cord

compression on NBME (from cancer or trauma), choose steroids first if listed.

- 65F + breast cancer + neurologic findings; next best step? à answer on Neuro NBME is “intravenous

high-dose dexamethasone.”

- 72M + prostate cancer + neurologic findings; Dx? à answer = “epidural spinal cord compression” due

to spinal mets à do steroids first (if listed), then MRI.

- Patient has brain cancer; next best step? à steroids.

- How to Dx brain cancer? à answer = contrast head CT (done for cancer and abscess).

- How to Dx brain bleed? à answer = non-contrast CT (always done for intracranial bleeds).

MEHLMANMEDICAL.COM 4
MEHLMANMEDICAL.COM

- Epidural hematoma; next best step after CT confirms? à answer = intubation and hyperventilation

on Neuro NBME. After this is done, craniotomy is the answer.

- Subdural hematoma; next best step after CT? à Answer = craniotomy on NBME, not observation.

- What do you see on non-contrast CT with epidural vs subdural? à epidural hematoma = lens-shaped

bleed; subdural = crescent-shaped.

- Blood pressure of 220/120 + confusion; Dx + Tx? à Hypertensive encephalopathy; give IV sodium

nitroprusside à this question is asked on Neuro form 1.

- BP of 220/120 + sodium nitroprusside administered; now patient has confusion; Dx? à cyanide

toxicity caused by nitroprusside.

- Drug that can be given to prevent vasospasm after a subarachnoid hemorrhage (SAH)? à nimodipine

(dihydropyridine CCB).

- Severe headache + stiff neck; Dx? à SAH (can cause meningism, similar to meningitis).

- Brain bleed in patient with Alzheimer; Dx? à amyloid angiopathy (intracerebral hemorrhage).

- 87F + Alzheimer + low-grade fever + delirium; next best step? à answer = do urinalysis to look for UTI

as cause of delirium.

- Viral infection + tinnitus + vertigo +/- neurosensory hearing loss à labyrinthitis.

- Viral infection + vertigo à vestibular neuritis.

- Tx for acute flare of MS à IV steroids (oral is wrong and can make flares worse).

- Tx between flares of MS (the patient must by asymptomatic) à IFN-beta (interferon beta).

- Tx for spasticity in MS à baclofen (GABA-B receptor agonist).

- Incontinence in MS à urge (hyperactive detrusor; detrusor instability).

- Mechanism for MS? à T cell-mediated attack against oligodendrocytes.

- How to Dx MS? à MRI is gold standard; choose MRI over CSF IgG oligoclonal bands.

- 27F + intermittent headaches + blurry vision; Dx? à optic neuritis (multiple sclerosis) à student says

“why the headaches?” Yeah, I know. Weird. But it’s on the NBME. You need to know optic neuritis is

HY in MS and means inflammation of cranial nerve II à presents as blurry vision, or change in color

vision, or Marcus Gunn pupil (relative afferent pupillary defect).

MEHLMANMEDICAL.COM 5
MEHLMANMEDICAL.COM

- Most specific eye finding in MS à medial longitudinal fasciculus (MLF) syndrome à aka internuclear

ophthalmoplegia (INO) à when you abduct to one side, you activate CN VI on that side, which

requires the contralateral CN III to activate in order to adduct à the side that cannot adduct is the

side that’s fucked up; the normal side will have nystagmus.

- How to differentiate CN III lesion from INO? à INO patients can converge normally.

- 20F + stiffness of hands + frontal balding + impaired relaxation of hypothenar muscles; Dx? à

myotonic dystrophy (CTG TNR disorder).

- Cancer patient on cisplatin or vincristine + develops neuropathy; Dx? à answer = toxic neuropathy

(chemotherapy-induced neuropathy); these drugs are neurotoxic.

- Confusion and/or seizure due to electrolyte disturbance; Dx? à hypo-/hypernatremia and

hypercalcemia all cause CNS dysfunction.

- Confusion in the setting of high BUN and creatinine; Dx? à uremic encephalopathy.

- Floppy baby syndrome; Dx and location of CNS/PNS affected? à Werdnig-Hoffmann syndrome

(spinal muscular atrophy); anterior horns affected.

- Antipsychotic medication started + muscle rigidity + no fever; Dx + Tx? à acute dystonia, not

neuroleptic malignant syndrome (because no fever); Tx with benztropine (muscarinic receptor

antagonist) or diphenhydramine (1st gen H1 blocker, which has strong anti-muscarinic side-effects).

- Antipsychotic medication started + abnormal eye movements + stiff neck; Dx? à acute dystonia

(oculogyric crisis + torticollis).

- Antipsychotic medication started + muscle rigidity + fever; Dx + Tx? à neuroleptic malignant

syndrome; give dantrolene (inhibits ryanodine channel).

- Antipsychotic med + restlessness; Dx + Tx? à akathisia; Tx with propranolol.

- Antipsychotic med + bradykinesia; Dx + Tx? à drug-induced Parkinsonism; Tx with amantadine or

propranolol.

- Antipsychotic med + abnormal tongue movements; Dx + Tx? à tardive dyskinesia; stop antipsychotic

+ switch to atypical.

- Tx for diabetic neuropathic pain? à answer = TCA (i.e., amitriptyline). Second-line is gabapentin

- Tx for herpetic / post-herpetic neuralgia (i.e., from shingles)? à gabapentin

MEHLMANMEDICAL.COM 6
MEHLMANMEDICAL.COM

- 82M diabetic + neuropathic pain + already taking carbamazepine + gabapentin to no avail; next best

step? à switch the meds to nortriptyline (a TCA) à student then asks, “Wait, I thought you said TCAs

are first-line. Why does this Q have the guy on those two meds then?” à two points: 1) we don’t like

giving TCAs to elderly because of their anticholinergic and anti-alpha-1 side-effects, so this vignette

happen to try other agents first, but if you’re asked first-line, always choose TCA; and 2) if we do give

a TCA to an elderly patient, we choose nortriptyline because it carries fewer adverse effects.

- How to differentiate cluster headache from trigeminal neuralgia? à cluster will be a male 20s-40s

with 11/10 lancinating pain behind the eye waking him up at night (he may pace around the room

until it goes away); details such as lacrimation and rhinorrhea are too easy and will likely not show up

on the shelf. In contrast, trigeminal neuralgia will be 11/10 lancinating pain behind the eye (or along

the cheek / jaw if V2 or V3 branches affected; it’s when V1 is affected that this diagnoses are more

readily confused) à TN is brought on by a minor stimulus such as brushing one’s hair or teeth, or a

gust of wind.

- Tx and prophylaxis for cluster headache? à Tx = 100% oxygen; prophylaxis = verapamil.

- Tx and prophylaxis for trigeminal neuralgia? à Tx = goes away on its own because it lasts only

seconds; prophylaxis = carbamazepine.

- Tx and prophylaxis for migraine? à Tx = NSAID, followed by triptan (triptans are NOT prophylaxis;

they are for abortive therapy only after NSAIDs); prophylaxis = propranolol.

- 32M + diffuse headache relieved by acetaminophen + sleep; Dx? à answer = tension-type headache;

Tx = rest + taper caffeine (if taking it).

- Other HY uses for propranolol?

o Migraine prophylaxis (FM form gives patient with HTN + migraine; answer = propranolol)

o Akathisia (with antipsychotic use)

o Thyroid storm (decreases peripheral conversion of T4 to T3)

o Essential tremor (bilateral resting tremor in young adult; autosomal dominant; patient will

self-medicate with EtOH, which decreases tremor).

o Hypertension + idiopathic tremor (i.e., tremor need not be essential if patient has HTN à

answer on FM form is “beta-adrenergic blockade” for the HTN Tx).

o Esophageal varices prophylaxis (patients at risk of bleeds)

MEHLMANMEDICAL.COM 7
MEHLMANMEDICAL.COM

o Hypertrophic obstructive cardiomyopathy (increases preload à decreases murmur)

o Social phobia

- 47M + bitemporal hemianopsia; most likely tumor? à prolactinoma (most common pituitary tumor

in adults).

- 8M + bitemporal hemianopsia; most likely tumor? à craniopharyngioma (most common pituitary

tumor in children; some sources argue not true pituitary tumor because it’s derived from Rathke

pouch, which is the roof of the primitive pharynx).

- Most common primary brain cancer in children? à pilocytic astrocytoma à solid + cystic on MRI; has

Rosenthal fibers à not aggressive.

- 3F + morning vomiting + truncal ataxia; Dx? à medulloblastoma of cerebellar vermis.

- Most common primary brain cancer in adults? à glioblastoma multiforme (butterfly glioma) à areas

of necrosis + hemorrhage (large, irregular mass on head CT); often presents with seizure.

- 44F + SLE + irregular ring-enhancing lesion seen on head CT; Dx? à primary CNS lymphoma, not Toxo.

- Neuro shelf asks Tx for Toxo; answer? à sulfadiazine + pyrimethamine; prophylaxis is trimethoprim +

sulfamethoxazole. For PJP, TMP/SMX is Tx and prophylaxis. Yes, this is asked on Neuro shelf.

- Neonate + intracranial calcifications + chorioretinitis + hydrocephalus; Dx? à Congenital Toxo.

- Neonate + intracranial calcifications + deafness + hepatomegaly + rash; Dx? à CMV.

- Neonate with bilateral deafness due to maternal infection + no other info given; Dx? à answer =

congenital CMV.

- CNS tumors seen in neurofibromatosis? à oligodendroglioma + ependymoma + optic nerve glioma in

NF1; in NF2, meningioma. These may overlap, but be aware that these tumors can merely occur in

either NF, period.

- 50F + jaw pain + headaches + normal ESR; Dx? à temporal mandibular joint syndrome; if ESR is high,

answer is jaw claudication caused by temporal arteritis.

- 40F being treated for TB + has neurologic findings in extremities +/- seizure; Dx? à B6 (pyridoxine)

deficiency caused by isoniazid.

- 65F + metastatic breast cancer + suprapubic mass + decreased sphincter tone + pain over lumbar

vertebrae on fist percussion; Dx? à answer = metastases to cauda equina.

MEHLMANMEDICAL.COM 8
MEHLMANMEDICAL.COM

- 44M + saddle anesthesia + late urinary incontinence; Dx? à cauda equina syndrome à bundle of

spinal nerves and rootlets arising from the conus medullaris).

- 44M + perianal anesthesia + early urinary + fecal incontinence; Dx? à conus medullaris syndrome à

bulbous end of the spinal cord = conus medullaris.

- 48M + burns his thumb; pain sensed via which dermatome? à answer = C6; lateral digits primarily

C6; middle finger C7; medial digits primarily C8.

- 13F + 2-day Hx of lower limb paresthesias + CSF protein 95 mg/dL + CSF leukocyte count 4/uL; Dx? à

answer = Guillain-Barre syndrome (GBS). USMLE will often not mention preceding infection, especially

at 2CK level.

- What does CSF show in GBS? à albuminocytologic dissociation (high protein but normal cells).

- 24M + weakness proximally and distally in lower limbs + weakness distally in upper limbs; rest of the

vignette is vague; Dx? à GBS à this descriptor means paralysis/paresis has ascending in lower limbs

but not yet in upper limbs.

- Dx of Guillain-Barre syndrome? à Neuro shelf wants “electromyography and nerve conduction

studies” over CSF findings.

- Mechanism of GBS? à T cell attack against Schwann cells of PNS.

- Tx of GBS? à IVIG + plasmapheresis; steroids are the wrong answer.

- 32F + paresthesias in thenar region of hand +/- hand weakness + sensation intact over dorsum of

hand; next best step in Dx? à Neuro NBME exam wants “Electrophysiological testing.” Call it weird,

but it’s what they want. Examination findings such as Tinel sign, Phalen maneuver, Flick test are

insufficient for diagnosis.

- First Tx for carpal tunnel syndrome in patient who can’t stop offending activity (e.g., office worker) à

wrist splint first; then triamcinolone (steroid) injection into the carpal tunnel; do not select anything

surgical as it’s always wrong on the USMLE; NSAIDs are a wrong answer and not proven to help

- Tx for cubital tunnel syndrome à elbow splint.

- What is cubital tunnel syndrome à ulnar nerve entrapment at elbow à presents similarly to carpal

tunnel syndrome but just in an ulnar distribution and involves the forearm.

- What is Guyon canal syndrome à ulnar nerve entrapment at the wrist à hook of hamate fracture or

chronic handle bar impaction in avid cyclists.

MEHLMANMEDICAL.COM 9
MEHLMANMEDICAL.COM

- Midshaft fracture of humerus; which nerve is injured? à radial à wrist-drop + pronated forearm.

- Supracondylar fracture of humerus; which nerve is injured? à median.

- Medial epicondylar fracture; nerve injury? à ulnar.

- Surgical neck of humerus fracture; nerve injury? à axillary.

- Mastectomy + post-operative winged scapula; nerve injury? à long thoracic nerve.

- Which muscle is innervated by long thoracic nerve? à serratus anterior.

- 6M + ECG shows miscellaneous arrhythmia + seizure-like episode; Dx? à Adam-Stokes attack à not

true seizure disorder as per EEG; arrythmia leads to hypoxia of brainstem à seizure-like fits ensue.

- 75M + episodes of loss of consciousness (LoC) for 2 years + tonic-clonic-like episodes + becomes pale

and sweaty + Hx of MI; Dx? à answer = “syncope” on the NBME (convulsive syncope).

- Mechanism of narcolepsy? à answer = deficiency of orexin (hypocretin).

- Diagnosis of narcolepsy? à answer = polysomnography.

- What is cataplexy? à loss of muscle tone usually in response to emotional stimulus (e.g., laughter) à

seen in narcolepsy.

- Tx of nacrolepsy = stimulants (i.e., modafinil).

- What is simple vs complex seizure? à simple = no LoC; complex = LoC; patient staring off into space

not aware of surroundings = LoC.

- What is partial vs generalized seizure? à partial = affecting one part of the brain; generalized =

involves the wholes of both cerebral hemispheres.

- What is myoclonic seizure? à jerks in muscle or group of muscles; no LoC.

- Empiric Abx therapy for meningitis à ceftriaxone + vancomycin (+/- steroids).

- Lumbar puncture or Abx first in suspected meningitis? à new guidelines say LP first.

- When do you do CT head before LP in suspected meningitis?

o Confusion that interferes with neurologic exam / decreased GCS score.

o Seizure.

o Focal neurologic signs (motor or sensory).

o Papilledema or if the optic fundi cannot be visualized.

o Above reasons indicate potential mass lesion, where if you do an LP you can cause tonsillar

herniation and death; if CT negative, proceed cautiously to LP.

MEHLMANMEDICAL.COM 10
MEHLMANMEDICAL.COM

- Bacterial meningitis: low glucose, high protein, high neutrophils (polymorphonuclear cells; PMNs).

- Aseptic (viral) meningitis: normal glucose, normal (or slightly elevated) protein, high lymphocytes.

- Fungal meningitis: low glucose, high protein, high lymphocytes (similar to bacterial, but high

lymphocytes instead of neutrophils).

- Herpes encephalitis: lots of RBCs in CSF due to temporal lobe hemorrhage à CT is often negative,

but sometimes Q will mention wave slowing or temporal complexes on EEG. Tx = IV acyclovir.

- Difference between meningitis and encephalitis à meningitis is nuchal rigidity (neck stiffness) +

photophobia + ophthalmoplegia; encephalitis presents with confusion.

- Dx of Cryptococcal meningitis? à answer = latex agglutination if it’s listed over India ink;

mucicarmine staining (red stain) can also be done.

- Tx for Cryptococcal meningitis à amphotericin B + flucytosine, followed by fluconazole taper.

- Nodular density in upper lobe in immunocompromised pt à aspergilloma à next best step = open

lung biopsy (sounds radical, but it’s the answer on one of the NBME forms) à Tx with -azole à

invasive aspergillosis can be treated with caspofungin or voriconazole.

- 44F + diplopia + dysphagia + eyelid ptosis; all worsen throughout the day; Dx? à myasthenia gravis

(MG).

- 44F + proximal muscle weakness + able to perform upward gaze without a problem for 60 seconds;

Dx? à Lambert-Eaton (LE) syndrome (if cannot perform, answer = MG).

- Vignette where Dx is either MG or LE but it’s not listed; answer? à “neuromuscular junction.”

- 44F + difficulty getting up from chair but is able to after several attempts; Dx? à LE.

- Mechanism of MG vs LE? à MG = antibodies against postsynaptic acetylcholine receptors; LE =

antibodies against presynaptic voltage-gated calcium channels.

- MG can sometimes be paraneoplastic syndromes of which cancer? à MG from thymoma (do chest

imaging to check for thymoma after Dx of MG; if thymoma present + removed, this cures the MG). Up

to 80% of patients with thymoma + MG have anti-MUSK Abs (muscle-specific kinase).

- LE can sometimes be a paraneoplastic syndrome of which cancer? à small cell lung cancer.

- How to Dx MG vs LE? à If both are listed, choose antibodies over Tensilon (edrophonium) test.

- What is edrophonium à short-acting acetylcholinesterase inhibitor.

- How do MG vs LE perform with edrophonium? à MG improves drastically; LE less change.

MEHLMANMEDICAL.COM 11
MEHLMANMEDICAL.COM

- Tx of MG? à give an acetylcholinesterase inhibitor (i.e., pyridostigmine).

- 38F + papilledema on fundoscopy + flushing of face / venous distension of head + neck; Dx? à NBME

wants “obstruction of venous return to the heart” à SVC syndrome; classically has (+) Pemberton

sign; why this is on Neuro shelf I do not know, but it is. Tangentially, thymic lesions can cause SVC-like

syndrome (i.e., T cell ALL in peds)

- 6M + dancing eyes + HTN + lesion visualized in posterior mediastinum on CXR; Dx? à neuroblastoma;

students says wtf? à can occur anywhere in the median sympathetic chain, although classically intra-

abdominal. Dancing eyes = opsoclonus-myoclonus syndrome.

- Neuroblastoma; how to Dx? à answer on NBME is “urinary homovanillic acid (HVA) and

vanillylmandelic acid (VMA); mIBG scan may also be used; n-myc gene.

- 25M + impaired plantar flexion + inversion; Dx? à tibial nerve injury.

- 25M + impaired dorsiflexion + eversion; Dx? à common peroneal nerve injury.

- 22F returns from Central America + has tonic-clonic seizure + CT of head shows “swiss cheese”

appearance of brain, or cystic structures in ventricles, or white specs somewhat diffusely (I’ve seen all

three images on NBME forms); Dx? à neurocysticercosis (Taenia solium); Tx = albendazole (or

praziquantel).

- AIDS patient with CD4 count of 47/uL + cognitive degeneration + miscellaneous peripheral neurologic

findings; Dx? à progressive multifocal leukoencephalopathy (PML) caused by JC polyoma virus;

USMLE wants “reactivation of latent infection”; “acute infection in immunocompromised patient” is

the wrong answer. T2-weighted MRI shows multiple hyperintensities.

- 55F + few-month Hx of spinning sensation + no vomiting + when head hangs off examination table

she develops coarse, rotary nystagmus beating to one side; Dx? à answer on NBME is benign

paroxysmal positional vertigo (BBPV); need not be associated with vomiting; Dx with Dix-Hallpike

maneuver; Tx with Epley maneuver.

- Cause of BPPV? à posterior semicircular canal otolith.

- What is cholesteatoma? à non-cancerous squamous proliferation of middle ear.

- Most common cause of carotid plaques? à HTN à the strong systolic impulse from the heart pounds

the carotids --> endothelial damage --> atherosclerosis.

MEHLMANMEDICAL.COM 12
MEHLMANMEDICAL.COM

- 55M + BP 150/90 + TIA; next best step in Mx? à carotid duplex USS à the first thing you want to

think about is, "does this guy have a carotid plaque that has resulted in a clot embolizing to his brain."

- 80M + good blood pressure (e.g., 110/70) + stroke or TIA; next best step in Mx? à ECG à you want

to think, "Does he have atrial fibrillation with a LA mural thrombus that's now embolized to the

brain."

- 80M + good blood pressure (e.g., 110/70) + stroke or TIA + ECG shows sinus rhythm with no

abnormalities; next best step in Mx? à Holter monitor à when you first see this scenario you're

probably like, "Wait, the ECG is normal, so it's not AF?" à No, it is likely AF, but AF is often

paroxysmal, so in order to detect it in this scenario, the next best step is a Holter monitor (24-hour

wearable ECG). This means that later in the day when he sits down to have dinner and then pops into

AF, the Holter monitor will pick it up.

- What % of people over age 80 have AF? à 8% of people over age 80 have AF, which is why age is a

huge risk factor. In other words, if the vignette says the guy is 58, AF is probably less likely just based

on shear probability, regardless of hypertensive status." And, once again, knowing that AF is often

paroxysmal is really important.

- Age 50s-60s + high BP + TIA/stroke/retinal artery occlusion; next best step in Dx? à answer = carotid

duplex ultrasound to look for carotid plaques.

- Age >75 + good BP + TIA/stroke/retinal artery occlusion; answer = ECG to look for AF à if normal, do

Holter monitor to pick up paroxysmal AF.

- 55M + good BP + carotid bruit heard on auscultation; next best step in Mx? à answer = carotid

duplex ultrasound to look for carotid plaques à in this case, if they are obvious and explicit about the

suspected etiology of the stroke, TIA, or retinal artery occlusion, then you can just do the carotid

duplex ultrasound.

- How to Mx carotid plaques? à first we have to ask whether the patient is symptomatic or

asymptomatic. A bruit does not count as symptoms (that's a sign). Symptomatic means stroke, TIA, or

retinal artery occlusion. According to recent guidelines: carotid occlusion >70% if symptomatic, or

>80% if asymptomatic à answer = do carotid endarterectomy. Below these thresholds à answer =

medical management = statin, PLUS clopidogrel OR dipyridamole + aspirin. The USMLE will actually

MEHLMANMEDICAL.COM 13
MEHLMANMEDICAL.COM

not be hyper-pedantic about the occlusion %s (that’s Qbank). They'll make it obvious for you which

answer they want. They'll say either 90% à answer certainly = carotid endarterectomy, or they'll say

50% à answer = medical management only. There’s one NBME Q where they say a guy has a bruit

but is asymptomatic, and has 10 and 30% occlusion in the left vs right carotids, respectively, and he’s

already on aspirin + statin, and the answer is "maintain current regimen” à if he were symptomatic,

even with low occlusion, he’d certainly need statin, PLUS clopidogrel OR dipyridamole + aspirin.

- 6F + rheumatic fever + has Sydenham chorea; Q asks mechanism à answer = autoimmune disorder

to the basal ganglia.

- 22F + says cannot see out of left eye + examination shows no relative afferent pupillary defect;

answer = conversion disorder – i.e., if he truly had impaired vision, Marcus-Gunn pupil would be

present.

- 38F + hoarseness of voice + various stress factors; Dx? à conversion disorder; merely be aware that

hoarseness of voice is assessed for this.

- Back pain worse when standing or walking for long periods of time à lumbar spinal stenosis.

- Radiculopathy down an arm à cervical disc herniation.

- Bilateral paresthesias in the arms in rheumatoid arthritis patient à atlantoaxial subluxation.

- Bilateral paresthesias in the arms in rheumatoid arthritis patient à MRI of spine to Dx atlantoaxial

subluxation.

- Prior to surgery in rheumatoid arthritis patient à cervical CT or flexion/extension x-rays of cervical

spine to check for atlantoaxial subluxation.

- 40M + combination of LMN and UMN findings + no sensory abnormalities; Dx? à amyotrophic lateral

sclerosis; make sure you memorize that there are NO sensory findings, as this is the detail most

students forget. Sometimes the answer won’t be “ALS,” but will instead be “motor neuron” (i.e.,

they’ll ask for structural diagnosis, rather than disease name).

- 1-year-old boy + hypotonia + honey consumption; Dx? à answer = ingestion of spores (honey);

botulism (Clostridium botulinum).

- 13M + camping trip + Bell palsy + constipation + hypotonia; Dx? à ingestion of preformed toxin from

compromised canned goods. Tx = anti-toxin.

MEHLMANMEDICAL.COM 14
MEHLMANMEDICAL.COM

- 32M + waxing and waning tinnitus and hearing loss + difficulty hearing conversations at dinner and in

groups + has family Hx of similar Sx; Dx? à Meniere disease à low-frequency hearing loss.

- Mechanism for Meniere? à defective endolymphatic drainage; may attempt antihistamines and

diuretics.

- Viral infection + tinnitus +/- vertigo + no hearing loss; self-resolves over weeks to months; Dx? à

vestibular neuritis.

- Viral infection + tinnitus + hearing loss +/- vertigo self-resolves over weeks to months; Dx? à

labyrinthitis. One Q on the neuro forms has presentation similar to vestibular neuritis but answer is

labyrinthitis; vestibular neuritis isn’t listed as alternative answer.

- 3M + pinna displaced upward and outward + fever; Dx? à mastoiditis (malignant otitis externa).

- 3M + mastoiditis + next best step in Mx? à CT or MRI of the temporal bone (x-ray is wrong answer)

à sounds outrageous to do a CT in a kid, but “CT of temporal bone” is answer on one of the NBMEs;

in UWorld, MRI is answer + x-ray is wrong/insufficient à mastoiditis often associated with fluid

collection that must be visualized + drained to prevent brain abscess.

- 25M IV drug user + Tx for endocarditis + now the room is spinning; Dx? à CN VIII toxicity caused by

aminoglycoside (can be hearing loss, tinnitus, or vertigo); empiric endocarditis Tx is usually gentamicin

+ either vancomycin or ampicillin/sulbactam.

- Endocarditis patient + has stroke-like episode + fever; next best step? à IV antibiotics à septic

embolus to the brain; can lead to brain abscess.

- MCA stroke on dominant hemisphere (usually left-sided); HY findings? à contralateral limb + face

sensory and/or motor deficits; also classically associated with Broca and Wernicke aphasias.

- ACA stroke; HY findings? à contralateral lower leg sensory and/or motor dysfunction.

- PCA stroke; HY findings? à contralateral hemianopsia with macular sparing.

- Broca aphasia; HY findings? à non-fluent aphasia; telegraphic speech; comprehends normally but

cannot communicate easily, leading to frustration; repetition impaired.

- Where is Broca area? à posterior, lateral, inferior frontal lobe.

- Wernicke aphasia; HY findings? à fluent aphasia; word salad; non-sensical speech + does not

comprehend; repetition impaired.

- Where is Wernicke area? à superior temporal gyrus (temporal lobe).

MEHLMANMEDICAL.COM 15
MEHLMANMEDICAL.COM

- What is conduction aphasia? à patient has only impaired repetition (arcuate fasciculus connecting

Broca and Wernicke areas).

- What is global aphasia? à patient has presentation of Broca and Wernicke aphasias at the same

time.

- What is transcortical motor aphasia? à patient sounds like he/she has Broca aphasia but repetition is

intact.

- What is transcortical sensory aphasia? à patient sounds like he/she has Wernicke aphasia but

repetition is intact.

- What is mixed transcortical aphasia? à patient sounds like he/she has Broca and Wernicke at the

same time but repetition is intact.

- 42M + acalculia + left-right agnosia + finger agnosia + agraphia; Dx? à Gerstmann syndrome (angular

gyrus syndrome; junction of parietal and temporal lobes).

- Stroke causing Horner syndrome + dysphagia + loss of pain/temp from contralateral body and

ipsilateral face; Dx? à lateral medullary syndrome (PICA infarct) à “Pikachu” = PICA-chew =

dysphagia with PICA infarct.

- Stroke causing ipsilateral Bell palsy; Dx? à lateral pontine syndrome (AICA infarct) à FACIAL spelled

backwards has AICA in it à face is affected.

- Stroke causing ipsilateral tongue deviation + contralateral paralysis and loss of propioception; Dx? à

medial medullary syndrome (paramedian branch of anterior spinal artery).

- Lacunar infarct HY points? à caused by HTN leading to lipohyalinosis of lenticulostriate vessels à if

affecting posterior limb of internal capsule, deficits are pure motor (contralateral); if thalamic, deficits

are pure sensory (contralateral).

- Stroke causing locked-in syndrome; Dx? à Basilar artery stroke.

- Correct hyponatremia too quickly? à locked-in syndrome due to central pontine myelinolysis

(osmotic demyelination).

- Correct hypernatremia too quickly? à cerebral edema.

- Pain in contralateral limb months after stroke resolves; Dx? à thalamic pain syndrome.

- Coma or persistent vegetative state following deceleration injury (i.e., MVA or fall); Dx? à diffuse

axonal injury.

MEHLMANMEDICAL.COM 16
MEHLMANMEDICAL.COM

- 7F + facial grimaces past 5 months + no other motor findings or abnormal sounds + mental status

normal; next best step in Mx? à answer = “schedule a follow-up examination in 3 months” à Dx =

provisional tic disorder à 1/5 children experience some form of tic disorder; most common ages 7-

12; usually lasts less than a year; “watch and wait” approach recommended. Provisional tic disorder

used to be called transient tic disorder; the name was changed because a small % go on to develop

chronic tics.

- 10-month old boy + jerking movements of the limbs + EEG shows chaotic high-amplitude spikes with

no recognizable pattern; Dx? à answer = West syndrome (infantile spasms) à triad of spasms +

hypsarrhythmia on EEG (no recognizable pattern with high amplitude spikes) + developmental

regression; international definition of the diagnosis requires two out of three; starts age 3-12 months;

seen in 1-5% of Down syndrome kids; Tx = ACTH, prednisolone, or vigabatrin; ACTH is thought to act

by two mechanisms: 1) stimulates steroid (cortisol) release; 2) direct (cortisol-independent) effect on

melanocortin receptors.

- 13M + tonic-clonic seizure + 4-month Hx of hypnagogic/hypnopompic jerking of left arm + uncle has

epilepsy; Dx? à answer = juvenile myoclonic epilepsy; genetic with unclear inheritance pattern;

characterized by myoclonic jerks (usually hypnagogic and/or hypnopompic) that progress to tonic-

clonic seizures after several months; age of onset is usually 10-16, but can also start in adulthood; Tx

is valproic acid.

- 4F + few-month Hx of near-daily seizures + seizures typically occur while she’s sleeping + has started

putting objects in her mouth and making less eye contact + seizures not responding to anti-epileptic

meds; Dx? à answer = Lennox-Gastaut syndrome à severe childhood-onset epilepsy characterized

by near-daily seizures and cognitive decline (hyperoralism is a sign of cognitive regression [babies put

things in their mouths]); poor prognosis, with 5% mortality rate in childhood; 80-90% persistence of

seizures into adulthood.

Important summary of CSF findings:

- This first chart is relatively qualitative and sufficient for the USMLE. The second chart is more

quantitative/expansive in case you’re interested; all values are derived extensively from the

literature.

MEHLMANMEDICAL.COM 17
MEHLMANMEDICAL.COM

MEHLMANMEDICAL.COM 18
MEHLMANMEDICAL.COM

YouTube
@mehlmanmedical

Instagram
@mehlman_medical

MEHLMANMEDICAL.COM 19
MEHLMANMEDICAL.COM

MEHLMANMEDICAL
HY NEURO

All material is copyrighted and the property of mehlmanmedical.

Copyright © mehlmanmedical

MEHLMANMEDICAL.COM 20
MEHLMANMEDICAL
HY GASTRO
MEHLMANMEDICAL.COM

YouTube
@mehlmanmedical

Instagram
@mehlman_medical

MEHLMANMEDICAL.COM 2
MEHLMANMEDICAL.COM

HY Gastro – by Dr Mike Mehlman

Disorders affecting lips/oral cavity for USMLE


- Combination of perioral melanosis (fancy word for hyperpigmentation around
the lips/mouth) + hamartomatous colonic polyps.

Peutz-Jeghers
syndrome

- USMLE will show image of lips pretty much always, and then they’ll ask for what
kind of polyps are seen in the colon (i.e., hyperplastic, tubulovillous, etc.), and the
answer is just “hamartomatous.”
- Aka hereditary hemorrhagic telangiectasia; autosomal dominant.
- NBME loves showing a mouth or fingernail picture of telangiectasias.

Osler-Weber-Rendu

- Q will give nosebleeds + show you the above pic. There can be high-output
cardiac failure due to pulmonary AV fistulae.
- GI bleeding can occur leading to anemia.
- Triad of iron deficiency anemia + esophageal webs (dysphagia) + angular cheilitis
(cracked corners of mouth).

Plummer-Vinson
syndrome

- NBME Q can also mention pica (iron deficiency sign where patient eats clay,
starch, or ice), or they can show spoon-shaped nails (koilonychia), which are a
sign of severe iron deficiency.

MEHLMANMEDICAL.COM 3
MEHLMANMEDICAL.COM

- Just know it’s possible. USMLE can mention it on upper lip or forehead, and this
somehow confuses students, where they think it has to be on extensors only.

Lip psoriasis

- Aka canker sore.


- Usually appears as very painful lesion, self-resolving lesion on labial mucosa.
- Not infective; idiopathic; thought to have very mild autoimmune association;
sometimes associated with certain triggers like spicy food.

Aphthous ulcer

- Vasculitis that causes 5+ days of fever + injected (red) eyes and/or lips/tongue +
cervical lymphadenopathy + edema of dorsa of the hands + desquamation of
palms/soles (often mentioned as palms/soles “rash,” but not true rash).

Kawasaki disease

- Students obsess over coronary artery aneurysms as though they’re so HY.


They’re not. USMLE basically never mentions them.
- Often confused with herpes.

Perioral impetigo

MEHLMANMEDICAL.COM 4
MEHLMANMEDICAL.COM

- If they show you image of young kid in particular with lip lesions, it’s usually
impetigo caused by S. aureus or Group A Strep (S. pyogenes).
- Can lead to PSGN (if caused by Strep), as discussed in the HY Renal PDF.
- Caused by HSV1 or 2. USMLE doesn’t give a fuck about HSV1 being the lips and
HSV2 being the genitals. Bunch of nonsense perpetuated by other resources.

Herpes labialis

- Primary infection presents with fever and regional lymphadenopathy.


Recurrences aren’t as severe.
- HSV goes latent in sensory nerves.
- USMLE will show you above pic and then the answer is “DNA, enveloped,
linear.” Sounds nitpicky and low-yield, but it’s one of the viral structures they like
asking. Don’t confuse with HepB, which is “DNA, enveloped, circular.” I’ve made
lots of YouTube clips on that stuff.
- Treat with acyclovir à DNA polymerase inhibitor causing chain termination. HSV
resistance to it will be due to altered thymidine kinase.
- Caused by Group A Strep.
- Causes strawberry tongue / red lips + salmon-pink maculopapular body rash.

Scarlet fever

- Treatment is penicillin to prevent rheumatic fever (type II HS); can also lead to
PSGN (type III HS). I discussed this stuff in the HY Cardio and Renal PDFs.

- Caused by Coxsackie A (an RNA virus under picornaviridae).


- Causes benign, but contagious, lesions on, you guessed it LOL! – the hands, feet,
and mouth.
- Usually pediatric, but can present in adults (i.e., daycare workers, parents).

Hand-foot-mouth

- Don’t confuse with coxsackie B, which can cause dilated cardiomyopathy,


diabetes type I, and pleurodynia (latter I discussed in HY Pulm PDF).

MEHLMANMEDICAL.COM 5
MEHLMANMEDICAL.COM

- Also caused by coxsackie A.


- Presents as oropharyngeal vesicles or sores.

Herpangina

- Can occur with or without hand-foot-mouth.


- Caused by measles.
- White/blue-ish spots on buccal mucosa.

Koplik spots

- One of the ways to distinguish measles (rubeola) from rubella (German


measles).
- The same way you can memorize Koplik spots as unique to measles, you should
memorize sub-occipital and post-auricular lymphadenophaty as unique to rubella.
- Both measles and rubella have a head-to-toe masculopapular body rash.
- Both can present with fever, cough, coryza, conjunctivitis. These findings are
non-specific for viral infection; it’s been erroneously attributed to only measles
over the years, but I’ve it show up in numerous NBME Qs for a variety of viruses).
- Stones within the ducts of the salivary glands.
- Stensen duct is the opening of the parotid duct (from parotid gland) into the oral
cavity, which is located near the 2nd upper molar bilaterally.

Sialolithiaisis

- Sometimes a Q can say a patient has pain or inflammation on the buccal mucosa
across from the second upper molar, and sialadenitis (inflammation) or
sialolithiasis can be an answer.
- White-ish, painless, rough patch on lateral tongue.
Leukoplakia

MEHLMANMEDICAL.COM 6
MEHLMANMEDICAL.COM

- Precursor to squamous cell carcinoma of tongue.


- Biggest risk factor is smoking / chewing tobacco.
- Looks like leukoplakia, but caused by EBV.
Oral hairy leukoplakia
- Not considered premalignant (no dysplasia on biopsy).
- Painless white plaque on palate or tongue that bleeds when scraped.

Oropharyngeal
candidiasis

- Seen in immunocompromised patients, such as AIDS or organ transplant


recipients on immunosuppressant agents.
- Can be seen in patients with asthma who use inhaled corticosteroids. Patients
need to rinse their mouths with water after use to ¯ candida risk.
- Tx with nystatin mouthwash.
- Odynophagia (painful swallowing) in an immunocompromised patient is
Candidal esophagitis until proven otherwise.

Salivary gland neoplasia


- The answer on USMLE for benign salivary gland tumor that has variability in
cell size and shape.
Pleomorphic adenoma
- Most common salivary gland tumor; benign.
- Pleomorphic = cells and nuclei have variability in size and shape.
- The answer on USMLE for a salivary gland tumor that invades / has spread
/ if the patient has B-symptoms (fever, night sweats, weight loss).
Mucoepidermoid carcinoma - Most common malignant salivary gland tumor.
- Malignant means capable of invading, whereas pleomorphic adenoma will
not invade.
- The answer on USMLE for salivary gland tumor where they say anything
about it resembling lymphatic tissue.
Warthin tumor
- Salivary gland tumor that has lymphoid stroma (i.e., looks like lymphatic
tissue).

MEHLMANMEDICAL.COM 7
MEHLMANMEDICAL.COM

HY Esophageal conditions for USMLE


- Herniation of the stomach upward through the diaphgram.
- 95% of the time, hiatal hernia is known as “sliding,” where the cardia of the
stomach rises upward through the esophageal hiatus (opening in diaphragm
where esophagus passes).
- “Abnormal relation of the cardia to the lower end of the diaphragm” is
answer on NBME.
- 5% of the time, it is paraesophageal, where the fundus of the stomach
herniates upward through a hole in the diaphragm adjacent the esophagus.
- “Protrusion of the fundus into the chest above the level of T10” = answer for
paraesophageal hernia on NBME.

Hiatal hernia

- Often associated with gastroesophageal reflux disease (GERD).


- Surgery usually not indicated.
- Diagnose with upper endoscopy (asked on 2CK Surg).
- Irritation of esophageal mucosa by gastric acid; usually caused by ¯ lower
esophageal sphincter (LES) tone.
- USMLE wants you to know obesity is a risk factor for GERD, likely due to ­
stretching and pressure applied to the LES.
- As mentioned above, hiatal hernia is also a risk factor.
- Classic presentation is burning in the throat/chest after eating a meal.
- Can present with nocturnal cough or recurrent pneumonitis. These are HY
findings in patients who don’t have classic esophageal irritation symptoms.
- Can lead to Barrett esophagus (discussed below), which leads to esophageal
Gastroesophageal reflux
adenocarcinoma.
disease (GERD)
- First step in diagnosis is 2-week trial of proton pump inhibitor (PPI), such as
omeprazole, which will ¯ symptoms.
- PPIs are more efficacious than H2-blockers (e.g., cimetidine). If you see both
a PPI and H2-blocker as answers for a GERD Q, choose the PPI.
- One 2CK Surg form has “2-week trial of H2-blocker” as an answer, but PPI
isn’t listed, so just know the Q is out there.
- If trial of PPI doesn’t work, 24-hour esophageal pH monitoring is the answer.
- Nissen fundoplication is used last resort, but is asked on NBMEs.

MEHLMANMEDICAL.COM 8
MEHLMANMEDICAL.COM

- Answer can be written as “fundoplication” alone in patient with severe


GERD. This also shows up on Peds, where they say infant with severe GERD
and fundoplication is answer.
- Pediatric GERD will present as coughing up milk 2-3x daily. It will not be
described as high-energy or forceful (i.e., pyloric stenosis); it will not be bilious
(i.e., duodenal atresia; annular pancreas); it will not occur as choking or
spitting up during the first feed (i.e., tracheoesophageal fistula).
- Answer to pediatric GERD Q is “immature lower esophageal sphincter”. It’s
not complicated, but I’ve seen students say, “Wait, you can get GERD in kids?”
- Tx for Peds GERD is thickened feeds (addition of dry rice to formula) and
positional change prior to any use of PPIs.
- Metaplasia of distal esophagus characterized by conversion of non-
keratinized stratified squamous epithelium into columnar epithelium with
goblet cells (which secrete mucous).
- You’ll sometimes here the latter described as “intestinal columnar
epithelium.” This is because the stomach doesn’t have goblet cells; it has
mucous neck (foveolar) cells.
- Major risk factor for esophageal adenocarcinoma.

Barrett esophagus

MEHLMANMEDICAL.COM 9
MEHLMANMEDICAL.COM

- Can appear grossly red on upper endoscopy. Once visualized, the next best
step is biopsy to confirm the presence of Barrett metaplasia.
- Affects lower 1/3 of esophagus.
- HY pathogenesis is: GERD à Barrett esophagus à adenocarcinoma.
- Will present in patient over 50 who has Hx of GERD with either 1) new-onset
dysphagia to solids, or 2) dysphagia to solids that progresses to solids and
liquids.
- The “new-onset dysphagia” can refer to 3-6-month Hx in patient with, e.g.,
Adenocarcinoma
10-20-year Hx of GERD.
- 2CK wants immediate endoscopy in either of the above situations (i.e., don’t
choose barium first).
- Sometimes rather than making you choose endoscopy straight up, they’ll tell
you in the last line an endoscopy was performed and shows a stricture, then
they’ll ask for next best step à answer = biopsy of the stricture.
- Affects upper 2/3 of esophagus.
- Biggest risk factors for USMLE are heavy smoking/alcohol use.
- Other risk factors like burns, chemicals, achalasia, etc., are nonsense.
- Will present in patient over 50 who is heavy smoker/drink who has 1) new-
Squamous cell carcinoma
onset dysphagia to solids, or 2) dysphagia to solids that progresses to solids
and liquids.
- Same as with adenocarcinoma, USMLE wants immediate endoscopy as the
answer, followed by biopsy of a stricture or lesion if present.
- Outpouching of esophagus above the cricopharyngeus muscle (just above
the upper esophageal sphincter).

Zenker diverticulum

- Mechanisms on USMLE are numerous and include, “cricopharyngeal muscle


spasm” (Surg form 5), “uncoordinated swallowing,” “dysphagia,” and
“increased oropharyngeal pressure.”

MEHLMANMEDICAL.COM 10
MEHLMANMEDICAL.COM

- False diverticulum (contains only mucosa + submucosa; true diverticula, in


contrast, such as Meckel, include the muscular layer as well).
- Often presents as overweight male over 40-50 with gurgling sounds when
swallowing, or regurgitation of undigested food.
- Halitosis (bad breath) is buzzy but only mentioned in maybe 1/3 of Qs.
- Diagnosis is made with barium swallow.
- Treatment on 2CK NBME is “diverticulectomy.”
- Tightening (i.e., ­ tone) of the LES due to loss of NO-secreting neurons in the
Auerbach (myenteric) plexus.
- New NBME wants “enteric ganglia” as the answer for site of problem.
- Can be described on NBME as “dilated esophagus with constriction of the
gastroesophageal junction.”
- Presents usually as dysphagia to both solids and liquids from the start. This
is in contrast to cancer, which will be new-onset dysphagia to just solids, or
progression from solids only to solids + liquids. The HY point is that neurogenic
causes of dysphagia (i.e., achalasia) lead to solids + liquids dysphagia.
- Almost always idiopathic; can rarely be caused by Chagas disease.
- New 2CK NBME has “South American trypanosomiasis” as the answer for
achalasia, which is aka Chagas disease.
- First step in diagnosis is barium swallow, which shows a classic “bird’s beak”
appearance.
Achalasia

- After the barium is performed, esophageal manometry (a pressure study) is


confirmatory. Biopsy is not traditionally performed.
- Treatment is pneumatic (balloon) dilation, followed by myotomy (cutting of
muscle fibers of the LES).
- As discussed in detail in the HY Pulm PDF, this is an idiopathic autoimmune
disease characterized by multi-organ system fibrosis and hardening of tissues
(i.e., sclerosis).
Systemic Sclerosis
- Can lead to GERD. An offline Step 1 NBME wants ¯ LES sphincter tone and ¯
(aka scleroderma)
esophageal peristalsis as an answer. This refers to the esophageal dysmotility
component of CREST (Calcinosis, Raynaud’s, Esophageal dysmotility,
Sclerodactyly, Telangiectasias).
- Can either be iatrogenic (i.e., from endoscopy) or Boerhaave syndrome (from
increased pressure due to retching, vomiting, or straining).
- USMLE likes to say water-soluble contrast swallow shoes accumulation in the
mediastinum. If they don’t mention this, they’ll give either recent endoscopy
or straining + acutely low BP and high HR, where you have to eliminate the
other answers out of reasoning.
Esophageal perforation
- Water-soluble contrast (Gastrografin) is used in suspected perforation
because the contrast will leak into the mediastinum and barium can cause
mediastinitis.
- In contrast (no pun intended), if patient has aspiration risk, we use barium
and not Gastrografin because the latter can cause pneumonitis.
- “Esophageal repair” is the Tx on NBME. Pretty straightforward.

MEHLMANMEDICAL.COM 11
MEHLMANMEDICAL.COM

- Classically retching/vomiting in an alcoholic that causes stretching and


tearing of the gastroesophageal junction leading to small amounts of blood in
vomitus.
Mallory-Weiss tear - Can also occur in patients with eating disorders or hyperemesis gravidarum.
- It’s to my observation on NBME material that the highest yield point about
MW tear is that it does not cause high-volume hematemesis. If the vignette
mentions high-volume blood, that is ruptured varix instead (discussed below).
- Enlarged and tortuous superficial esophageal veins due to portal
hypertension in patient with cirrhosis or splenic vein thrombosis.

Esophageal varices

- The mechanism in the setting of cirrhosis: the left gastric vein connects the
esophageal veins to the portal vein. In the setting of ­ portal pressure, this
pressure backs up to the left gastric vein, which backs up to the esophageal
veins. Many USMLE Qs want “left gastric vein” as the answer for the vessel
responsible for the varices.
- In splenic vein thrombosis, the ­ splenic venous pressure causes formation
of collaterals to circumvent the thrombosis. This means nearby veins will form
small tributaries/branches from the splenic vein. The left gastric vein is one of
them à ­ esophageal venous pressure.
- It is to my observation on NBME Qs that 4/5 varices Qs will give a patient
with high-volume hematemesis. In contrast, MW tear is ‘just a little blood.”
- Ruptured varices are lethal ~50% of the time. The patient will vomit high
volumes of blood.
- 1/5 Qs might say “just a little blood” if the varix is friable but not overtly
ruptured, but the vignette will be obvious (i.e., cirrhosis) + they might just ask
for “left gastric vein” as the answer.
- Propranolol is prophylaxis, not for acute treatment.
- Treatment is endoscopy + banding.
- Octreotide can also be used for acute bleeding, but endoscopy + banding is
best answer on USMLE if you are forced to choose.
- There is a 2CK Q where patient has ¯¯ BP due to ruptured varix and the
answer is “IV fluids,” where endoscopy is the wrong answer. I’ve had a
student say, “Wait but I thought you said endoscopy and banding is what we

MEHLMANMEDICAL.COM 12
MEHLMANMEDICAL.COM

do first.” And my response is, yeah, but you always have to address ABCs first
on 2CK. They could by all means say patient has O2 sats of 50% and the
answer would be give oxygen before fluids.
- Idiopathic spasm of the esophagus.
- All you need to know is that this that causes pain that can mimic angina
pectoris, but patient will be negative for cardiac findings/disease.

Diffuse esophageal spasm

- Barium shows a corkscrew appearance.


- “Nutcracker esophagus” is a similar condition that is due to idiopathic
hyperperistalsis and causes pain similar to DES.
- Idiopathic allergic condition of the esophagus that presents with difficulty
swallowing.
- Biopsy shows eosinophils in epithelium (it’s like no shit).

Eosinophilic esophagitis

- Endoscopy shows “trachealization” of the esophagus with concentric rings.


- Garbage diagnosis for USMLE. Doesn’t exist. Mentioning it here as a negative
for USMLE giving a fuck cuz otherwise someone will be like, “Mike u didn’t
mention Schatzki ring.”
Schatzki ring - Technically a narrowing of the distal esophagus that could be considered as a
differential alongside achalasia. Once again, never seen this assessed as a
correct answer on any NBME material. Maybe just once as a distractor.
Garbage diagnosis.

HY GI exocrine hormones quick review


- Exocrine means secreted across epithelium into the GI tract. In contrast, endocrine means into the blood.
- Produced by “neuroendocrine cells of the small bowel.”
- Does 3 things: 1) Causes gall bladder contraction; 2) Causes relaxation of
sphincter of Oddi; 3) ­ exocrine pancreas secretion of proteases and lipases into
the duodenum.
Cholecystokinin (CCK)
- Impetus for secretion is macronutrients (i.e., fat, protein, carbs) entering the
duodenum from the pylorus, although fats are classically the major stimulator.
- Even though in theory acid can lead to ­ secretion of CCK, this is wrong on
USMLE. It is macronutrients that cause ­ CCK release on USMLE.

MEHLMANMEDICAL.COM 13
MEHLMANMEDICAL.COM

- Answer on USMLE for the hormone that is responsible for pain in biliary colic
(gallbladder pain from cholelithiasis, where a stone transiently obstructs
entrance into the cystic duct during gallbladder contraction).
- Step 1 NBME Q will give you pain from biliary colic after patient eats a fatty
meal, and then they’ll ask for where the hormone responsible is secreted à
answer = “neuroendocrine cells of small bowel.”
- Produced by S cells of the duodenum, but USMLE doesn’t give a fuck.
- What they care about is that secretin causes ­ bicarb release from the
exocrine pancreas into the duodenum, and that stomach acid entering the
Secretin duodenum from the pylorus is the impetus for secretion.
- What USMLE will do is show you a line graph representing pancreatic
secretions, where you can see there’s ­ bicarb release but no ­ in proteases
and lipases, and the answer is just “secretin.”
- Produced by enterochromaffin-like cells of the GI tract. Weird name, but just
deal with it. “Regular enterochromaffin” cells secrete serotonin in the GI tract.
- Binds to H2 receptors on parietal cells, stimulating acid secretion.
- Enterochromaffin-like cell hyperplasia occurs in chronic gastritis (on NBME);
Histamine this makes sense, since we have atrophy or destruction of parietal cells in
chronic gastritis, meaning histamine production goes ­ to compensate.
- Cimetidine is HY H2-blocker that ¯ stomach acid secretion; can cause
gynecomastia. It also inhibits P-450 (­ serum levels of other drugs). Ranitidine
does not inhibit P-450.
- Secreted by G cells.
- ­ acid via two main mechanisms: 1) directly agonizes gastrin receptors on
parietal cells; 2) stimulates enterochromaffin-like cells to secrete histamine,
Gastrin which in turn ­ stomach acid.
- Gastrin levels rise in chronic gastritis and H. pylori infections.
- Gastrinoma (aka Zollinger-Ellison syndrome) causes recurrent duodenal ulcers
and sometimes jejunal or ileal ulcers; can be part of MEN 1.
- Causes ­ water content / volume of pancreatic secretions.
- I’d say VIP is the highest yield GI hormone on 2CK Surg forms. VIPoma shows
up everywhere.
- ~2/3 of Qs will be WDHA syndrome à Watery Diarrhea, Hypokalemia,
Vasoactive intestinal Achlorhydria (­ serum pH + low serum Cl-).
peptide (VIP) - ~1/3 of Qs will be nothing about diarrhea, but will mention ­ serum pH + low
K+ + facial flushing. A 2CK NBME Q gives pH of 7.56 and potassium in the 2s +
facial flushing.
- USMLE will show graph of a hormone that causes ­ volume of pancreatic
secretions but pH and enzyme concentrations don’t ­ à answer = VIP.
- Stimulates peristalsis in the absence of food.
- Responsible for borborygmi (GI sounds).
Motilin
- Erythromycin (yes, the antibiotic) can agonize motilin receptors in patients
with gastroparesis.
- Aka gastric inhibitory peptide (GIP). USMLE loves this hormone.
- Causes ­ insulin release in response to oral macronutrients (not just carbs).
- What you need to know is that it is GIP that is responsible for oral glucse being
utilized faster than IV glucose, since we get ­ insulin spike if oral. USMLE will ask
this two ways:
Glucose-dependent
1) Two people both get 50g of glucose. One gets it orally. One gets it IV. The one
insulinotropic peptide
who gets it orally utilizes it faster. Why? à answer = “glucose-dependent
insulinotropic peptide.”
2) Two people get 50g IV glucose. One also gets a small amount of oral fatty
acids at the same time. The latter person utilizes the IV glucose faster. Why? à
answer = “glucose-dependent insulinotropic peptide.”

MEHLMANMEDICAL.COM 14
MEHLMANMEDICAL.COM

HY GI enzymes quick review


- Secreted by both salivary glands and the pancreas to digest starch into
monosaccharides.
- Pancreatic amylase is ­ in acute pancreatitis. The weird detail I can communicate is
Amylase that even though it’s not as reliable or specific as lipase for pancreatitis, various 2CK
NBME Qs will give only isolated ­ amylase in some Qs, without even listing lipase. So if
you get a Q where amylase is high and lipase isn’t listed, don’t be confused. It’s still
pancreatitis.
- Secreted by both salivary glands (lingual lipase) and pancreas to break down fats.
Lipase
- ­ in pancreatitis.
- Secreted by chief cells of gastric mucosa.
Pepsin - Secreted as pepsinogen, which is a zymogen (means pro-enzyme that is inactive).
- Converted into active pepsin by stomach acid, which then breaks down protein.
- Secreted by parietal cells (in addition to acid).
Intrinsic factor - Binds vitamin B12 and enables its absorption at the terminal ileum.
- ¯ in gastrectomy and pernicious anemia (discussed later).
- Secreted by exocrine pancreatic acinar cells as the zymogen trypsinogen.
- Converted by enterokinase at the brush border into trypsin.
- Trypsin is a protease that breaks down protein.
- Once active trypsin is formed, it activates other zymogens (e.g., chymotrypsinogen)
Trypsin
into active proteases.
- USMLE will ask about enterokinase, followed by wanting you to know that once we’ve
got active trypsin, it is the latter that now activates other things. Enterokinase merely
gets things started.
Enterokinase - Brush border enzyme that converts trypsinogen from the pancreas into trypsin.
- 90% of serotonin is made in the GI tract by enterochromaffin cells.
- Metoclopramide is an anti-emetic + prokinetic agent (means ­ peristalsis). It is a D2
antagonist but also an antagonist of serotonin 5HT3 and agonist of 5HT4 receptors. The
effects on serotonin receptors ­ gut peristalsis.
- Serotonin is also produced by carcinoid tumors, which are neuroendocrine tumors of
small bowel or appendix that are S-100 (+) and consist of small blue cells.
Serotonin
- Carcinoid syndrome will present as flushing, tachycardia, diaphoresis, and diarrhea.
Tricuspid regurg can also be seen.
- Diagnose with urinary 5-hydroxyindole acetic acid (5-HIAA).
- Don’t confuse with serotonin syndrome, which is a drug interaction (e.g., starting a
monoamine oxidase inhibitor too soon after discontinuing an SSRI, or if a patient takes St
John wort with an SSRI).
- Glucagon-like peptide 1.
GLP-1 - Stimulates insulin secretion.
- Exenatide + liraglutide are GLP-1 receptor agonists for type II diabetes.
- Dipeptidyl-peptidase 4.
- Breaks down GLP-1.
DPP-4 - Therefore, DPP-4 inhibitors ­ GLP-1 and ­ insulin release.
- NBME asks which drug listed ­ insulin release (i.e., is an insulin secretagogue); answer
= linagliptin (DPP-4 inhibitor).

HY GI endocrine hormones quick review


- Produced by b-islet cells in the tail of pancreas.
- ¯ blood glucose by two main mechanisms: 1) ­ GLUT-4 on skeletal muscle and adipose
tissue; 2) ­ glucokinase activity in the liver (hexokinase equivalent at liver), which pulls
Insulin
glucose out of the blood to be stored as glycogen.
- ­ Fatty acid and protein synthesis. Also ­ lipoprotein lipase activity (fat storage).
- Causes dephosphorylation of enzymes (I talk about this in detail in my HY Biochem pdf).

MEHLMANMEDICAL.COM 15
MEHLMANMEDICAL.COM

- Insulin secretion is HY: glucose enters b-islet cells via GLUT-2 à ­ ATP production within
b-islet cell à closure of ATP-gated K+ channel on membrane of b-islet cell à K+ builds up
inside the cell à more positive charge in cell à depolarization of cell à causes Ca2+ to
move into cell à triggers insulin vesicle efflux from cell.
- Insulin is normally produced as pro-peptide that must have C-peptide cleaved off as part
of the process. C-peptide and insulin are co-secreted, meaning their serum levels should
match one another. If patient has ­ insulin but ¯ C-peptide, answer = exogenous injection.
If C-peptide is normal/high, insulin production is endogenous. First step is checking serum
hypoglycemic levels (“serum hypoglycemic” = type II diabetes med that ¯ glucose, such as
sulfonylureas and meglitinides). If serum hypoglycemics are negative, then do CT of
abdomen to check for insulinoma.
- Insulinoma (and ­ insulin in general) cause Whipple triad: 1) hypoglycemia; 2) symptoms
of hypoglycemia (tachycardia, tremulousness); 3) improves with a meal / gets worse
between meals.
- Insulin is absent in type I diabetes and ­­ in early type-II diabetes (i.e., hyperinsulinemia).
- Insulin inhibits ketone formation, so we have ¯ ketones in type II, but ­ ketones in type I.
- Hyperinsulinemia causes anovulation / polycystic ovarian syndrome (see my Repro PDF).
- USMLE can show you pic of acanthosis nigricans, which is almost always due to insulin
resistance.
- Patients with chronic pancreatitis and pancreatectomy can have diabetes (loss of
pancreatic tail).
- Secreted by a-cells of the pancreatic tail.
- Causes ­ serum glucose and phosphorylation of enzymes.
- Glucagonoma will present as ­ serum glucose and a body rash called necrolytic migratory
erythema. Don’t confuse with the facial flushing seen with VIPoma; in addition, VIPoma
doesn’t ­ glucose levels.
- Patients who receive insulin for diabetes can sometimes have prolonged or exaggerated
hypoglycemic effects. If this occurs, an answer for why this occurs on USMLE is “lack of
Glucagon
counter-regulatory glucagon.” Sounds weird, but it shows up more than once on NBMEs.
Essentially, patients with diabetes, or chronic pancreatitis, or pancreatectomy and not just
prone to losing the b-islet cells, but they can also lose the a-cells. When glucose goes ¯,
glucagon should go ­ to compensate, but if this can’t happen, glucose stays low.
- Question on NBME with chronic pancreatitis (chronic burnout from repeated acute
pancreatitis; discussed more later) wants you to know that the arrows are: ¯ insulin
production, ¯ glucagon production, no-change peripheral response to insulin.
- Secreted by delta-cells of pancreatic tail.
- ¯ secretion of most GI hormones, as well as growth hormone.
Somatostatin - Somatostatinoma presents as steatorrhea (probably due to ¯ pancreatic lipase secretion).
- Octreotide is a somatostatin analogue that can be used in addition to endoscopic banding
for esophageal varices Tx à leads to ¯ portal blood flow/pressure.
- Produced by enteroendocrine cells of GI tract.
- All you need to know is that this hormone makes you feel hungry.
Ghrelin - Blood levels are highest just at the start of the meal. USMLE will show you a graph of
ghrelin levels, and they ask where on the graph corresponds to the start of a meal, and the
answer is at the peak. Not complicated. But I’ve seen innumerable students get this wrong.
- Produced mostly by adipocytes.
- Makes you feel full (i.e., opposite of ghrelin).
Leptin
- Also plays important role in hypothalamic/anterior pituitary secretion of gonadotropins.
- Leptin is ¯ in those with low BMI/anorexia à ¯ GnRH à ¯ LH + ¯ FSH à amenorrhea.

Peptic ulcers HY causes


- “Peptic ulcer” is an umbrella term that refers to both duodenal and gastric ulcers.
- Gastric ulcers cause pain immediately with meals (due to ­ acid secretion). Patients can sometimes lose
weight due to aversion to pain from eating.

MEHLMANMEDICAL.COM 16
MEHLMANMEDICAL.COM

- Duodenal ulcers cause pain 1-2 hours after meals (due to ­ acid entering duodenum). With meals, the
pylorus tightens, thereby relieving any residual discomfort. Patients may gain weight since eating ¯ pain.
- Responsible for almost all duodenal ulcers (>95%), whereas it causes a lower %
(only >60%) of gastric ulcers. This is merely because the latter are caused by many
other things as well, so we simply have ¯ fraction caused by H. pylori. In other
words, there’s no special tropism of H. pylori toward duodenal over gastric mucosa.
- Mechanism for ulcers that shows up on USMLE is: “secretes proteinaceous
substrates that damage mucosal lining.” This is correct over “­ gastric acid secretion”
if both are listed side-by-side, even though H. pylori does ­ gastrin levels, which ­
acid secretion.
- Produces urease, which causes ­ ammonia production around the organism,
allowing it to survive in the ¯ pH of the stomach.
Helicobacter pylori - Antral/pyloric ulcers can lead to gastric outlet obstruction. They will mention this in
a Surg Q as a “succussion splash.”
- ­ risk of MALT lymphoma, a type of B-cell lymphoma.
- Diagnose H. pylori with urease breath test or stool antigen.
- Treat H. pylori with CAP à clarithromycin, amoxicillin, PPI (e.g., omeprazole).
- USMLE really doesn’t give a fuck about the treatment, but CAP is safe to know.
Metronidazole, tetracycline, bismuth, and PPI tetrad is used if CAP fails (students ask
about those other drugs).
- Perforated duodenal ulcer will present as sudden-onset rigid abdomen (involuntary
guarding). Patient will often have SIRS, with abnormal vitals due to sympathetic
activation. USMLE wants “X-rays of the chest and abdomen” to look for air under the
diaphragm (HY finding that indicates ruptured viscus).
- Zollinger-Ellison syndrome causes recurrent duodenal ulcers and sometimes jejunal
or ileal ulcers.
- Can be part of MEN1 or idiopathic.
- H. pylori is more common than gastrinoma. As mentioned above, >95% of duodenal
ulcers are due to H. pylori. There is an NBME Q where they give older male with a
Gastrinoma
duodenal ulcer + no other information + they ask for most likely cause à answer =
testing for H. pylori; gastrinoma is wrong.
- Vignettes can be tricky with gastrinoma and tell you the patient has 8-10 watery
stools daily, where you say, “That sounds like VIPoma.” But they’ll also tell you the
patient has history of abdo pain after meals. Q on IM form 8 does this as example.
- Cause gastric ulcers. I haven’t seen these cause duodenal ulcers on USMLE.
- Prostaglandins are necessary for stimulation of gastric alkaline mucous production
and maintenance of the gastric lining. NSAIDs à ¯ prostaglandin production à
NSAIDs disruption of gastric lining. This can lead to both ulcers and irritation (gastritis).
- USMLE wants you to know that PPIs are first-line for ulcer treatment in general, but
that misoprostol is a PGE1 analogue that is used in NSAID-induced ulcers (i.e., we’re
replenishing the ¯ prostaglandin from NSAIDs).
- Rare, but just know they exist.
- Called Curling ulcers (think: curling irons are hot).
Burns
- Loss of fluid post-burns from evaporation à ¯ blood flow to stomach à ischemic
ulcers.
- Rare, but just know they exist.
- Head trauma or brain tumor à ­ intracranial pressure à ­ parasympathetic
outflow to stomach à ­ ACh binding to muscarinic receptors at parietal cells à ­
acid production.
Head trauma - Don’t forget that the 3 synergistic mechanism for acid production are 1) gastrin
binding directly to gastrin receptors on parietal cells; 2) gastrin stimulating
enterochromaffin-like cells to secrete histamine, which then binds to H2 receptors on
parietal cells; and 3) direct parasympathetic activity, where ACh binds to muscarinic
receptors on parietal cells.
- Not tested as overt causes of ulcers on USMLE. But you should know that smoking
Smoking/Alcohol
and alcohol are believed to decrease healing of pre-existing ulcers.

MEHLMANMEDICAL.COM 17
MEHLMANMEDICAL.COM

Gastritis causes + HY points


- Acute gastritis will present as irritation leading to bleeding of gastric mucosa (e.g., from NSAIDs).
- Chronic gastritis will be an atrophy or autoimmune destruction of the mucosa associated with ¯ mucosal
thickness, ¯ HCl production (achlorhydria), ­ gastrin production, and enterchromaffin-like cell hyperplasia
(­ histamine production to compensate for ¯ acid). NBME will give you this constellation of findings and
then just simply have “chronic gastritis” as the answer, with acute gastritis not even listed. It’s not hard, but
I’ve seen students miss this a lot.
- Causes what is referred to as “Type B gastritis,” which is inflammation tending to affect
the antrum of the stomach.
H. pylori
- Can lead to pyloric channel ulcers + gastric outlet obstruction. This will present on 2CK
Surg forms as a “succussion splash.”
- Causes what’s referred to as type A gastritis,” which affects mostly the fundus/body of
the stomach.
- Autoimmune antibody-mediated destruction of parietal cells.
- Sometimes antibodies can be against intrinsic factor.
- Ultra-HY cause of B12 deficiency (­ MCV + hypersegmented neutrophils +/- neuropathy)
on USMLE.
- Associated with other autoimmune diseases, e.g., vitiligo. So if they give you ­ MCV in
patient with, e.g., type I diabetes, you should think, “Autoimmune diseases go together, so
if the patient has one AA disease, he/she has ­ propensity for others.” Don’t worry about
strict HLA associations here.

Pernicious
anemia

- There is Q on an NBME exam where they ask you to identify the parietal cell. Histo
overall is LY for Step 1, but they want you to know that parietal cells are the “midway”
cells between the gastric surface superficially and the deeper, basophilic chief cells. Notice
how the parietal cells are slightly lighter/more eosinophilic (pink) in comparison to the
chief cells, which are darker/more basophilic (purple).
- Answer on USMLE for GI bleeding in someone taking, e.g., indomethacin or naproxen.
- Causes type A gastritis, affecting the fundus/body. USMLE doesn’t specifically give a fuck,
but you should basically be like, “H. pylori causes antral gastritis, whereas other causes like
NSAIDs and pernicious anemia are fundus/body of stomach.”
NSAIDs
- Prostaglandins are necessary for stimulation of gastric alkaline mucous production and
maintenance of the gastric lining. NSAIDs à ¯ prostaglandin production à disruption of
gastric lining. This can lead to both gastritis (inflammation) and ulcers.
- As mentioned above for ulcers, misoprostol can be used in these patients following PPI.

MEHLMANMEDICAL.COM 18
MEHLMANMEDICAL.COM

Gastric cancer points


- Perpetuated over the years as “intestinal vs diffuse” types. USMLE doesn’t care. What you need to know is
that histo will show signet-ring cells that contain mucin. And you don’t need to know the histo for this
either.
- Grossly, can cause a linitis plastica, which is a leather bottle appearance to the stomach.
- Gastric metastases can spread hematogenously to the ovaries. These will show up as bilateral ovarian
lesions that show, you guessed it, signet-ring cells that contain mucin.
- ­ incidence of gastric cancer in Japan due to ­ nitrosamines/smoked foods.
- MALT lymphoma is a B cell lymphoma that can be caused by H. pylori. This does not have signet ring cells.
- Acanthosis nigricans can be associated with gastric cancer (even though most of the time, it just means
insulin resistance).
- Virchow node (Troisier sign) à palpable supraclavicular lymph node that can reflect visceral malignancy,
especially gastric cancer.

Hyperbilirubinemia
- Bilirubin is produced from breakdown of heme from RBCs at the spleen. It will initially be
“unconjugated” in this form, where it merely leaves the spleen non-covalently bound to
albumin and is not water-soluble – i.e., it won’t show up in the urine. Unconjugated
bilirubin is aka indirect bilirubin.
- In the setting of hemolysis or ­ RBC turnover (i.e., sickle cell, hereditary spherocytosis,
blood given during surgery), where we have ­ RBC breakdown, we get ­ indirect bilirubin.
- Once it arrives at the liver, it is taken up by the liver and conjugated to glucuronide,
making it water-soluble. Conjugated bilirubin is aka direct bilirubin.
- If the there’s a problem with uptake at the liver (i.e., acute hepatitis), or there is deficient
Unconjugated conjugation enzyme (Gilbert syndrome, Crigler-Najjar), indirect bilirubin also goes ­.
(indirect) - Hemolysis or ­ RBC turnover can cause direct bilirubin to go ­ sometimes as well, but the
shift will be more toward indirect being ­. I point this out because students often get
confused by this, but if we have ­ indirect bilirubin going into the liver, then this means ­
direct bilirubin going out. A 2CK NBME Q gives 9 packs of RBCs given during surgery à days
later, total bilirubin is 5.0 and direct bilirubin 2.3 à answer = “overproduction of bilirubin.”
- In regard to neonatal labs (i.e., first month of life), I should note that hematocrit % can be
in the 50s (NR 45-61; in adults, NR is 40-50). 2CK, for instance, will often show Hct as 56%
in a newborn and the student is like “Wow that’s high!” where they think there’s neonatal
polycythemia or some other pathology, but it’s actually normal. Unconjugated jaundice can
occur in neonates because of ­ RBC turnover where HbF is replaced with HbA.
- Direct bilirubin, since it is water-soluble, shows up in the urine. It will also be secreted
into bile. Therefore, if we have a bile duct obstruction, we get ­ direct bilirubin. ALP also
goes ­ in bile duct obstruction because it is secreted by bile duct epithelium. This means “­
ALP + ­ direct bilirubin” is very buzzy for bile duct obstruction. If ALP is high but direct
bilirubin not ­, this can be due to things like bone fractures or Paget disease.
- GGT will also go up with bile duct obstruction, since it is also secreted by bile duct
epithelium, but USMLE actually rarely mentions this one. What they want you to know is
Conjugated GGT spikes with acute alcohol consumption / binge drinking.
(direct) - In the event of bile duct obstruction, not only will ALP and direct bilirubin go ­, but the
urine becomes darker from ­ direct bilirubin in it (and urobilin, which comes from direct
bilirubin, but USMLE doesn’t assess this). In addition, stools become lighter/pale (aka
“acholic stools”), since there is ¯ direct bilirubin making it to the intestines, which means ¯
stercobilin production (pigmentation in stool). In other words, “dark urine + pale stools” is
buzzy for bile duct obstruction the same way “­ ALP and ­ direct bilirubin” is.
- Acute hepatitis can also cause ­ direct bilirubin due to ¯ secretion into bile, in addition to
­ indirect. But this makes sense, since it is literally an intra-hepatic pathology.

MEHLMANMEDICAL.COM 19
MEHLMANMEDICAL.COM

- Highest yield cause of ­­ direct bilirubin on USMLE is biliary atresia in neonates


(discussed later). This is all over the place, whereas Crigler-Najjar (­­ indirect bilirubin in
neonates) is nonexistent.
- Dubin-Johnson and Rotor syndromes are virtually nonexistent on USMLE but cause ­
direct bilirubin in adults due to ¯ bile excretory pumps at the liver. Students get hysterical
about these because they sound weird, but they’re LY. Dubin-Johnson is asked once on a
new 2CK NBME form. But apart from that, LY.
- Cholangitis (inflammation of bile ducts), choledocholithiasis (stone in biliary tree),
choledochal cyst, head of pancreas cancer (impingement on common bile duct), and
cholangiocarcinoma (bile duct cancer) are all HY causes of bile duct obstruction. I discuss all
of these conditions in detail below.

Bilirubin uptake/secretion diseases


- Pronounced “Jeel-BEAR, not “GILL-burt.” Gastroenterologist I met once tripped out
over med students pronouncing this wrong.
- Partial deficiency of bilirubin uptake enzyme at the liver (UDP glucuronosyltransferase).
Gilbert - Presents as isolated ­ indirect bilirubin with yellow eyes in young adult with stress
factor, such as studying for exams, or recent surgery/trauma. The patient will otherwise
be completely healthy.
- No treatment necessary.
- Near-absence of UDP-glucuronosyltransferase causing ­­ indirect bilirubin in neonate.
Crigler-Najjar
- Nonexistent yieldness but students ask about this.
- ¯ excretion of bilirubin into bile at the liver due to ¯ secretory pumps.
- Causes ­ direct bilirubin in otherwise healthy adult.
Dubin-Johnson
- Can cause black liver in theory.
- Asked once on new 2CK NBME. Apart from that, nonexistent on USMLE material.
- Same as Dubin-Johnson but no black liver.
- Nonexistent diagnosis on USMLE. Students get maniacal over this supposedly not
Rotor having a black liver, whereas Dubin-Johnson has a black liver – “Rotor! That’s the one
that has no black liver but Dubin-Johnson does!” Relax. Take two steps back, chill the
fuck out for two seconds. USMLE doesn’t give a fuck.
- One of the highest yield Peds diagnoses. Answer on USMLE for ­­ direct bilirubin in a
kid under the age of 6 weeks.
Biliary atresia - Caused by lack of development of the intrahepatic bile ductules and biliary tree.
- Ultrasound will be done first, but USMLE wants liver biopsy to confirm the diagnosis.
- Kasai procedure is done to treat, followed by liver transplant if unsuccessful.

Autoimmune liver conditions


- Inflammation of bile ducts within the liver, leading to their destruction.
- Answer in a woman 20s-50s who has generalized pruritis, ­ serum cholesterol,
­ ALP, ­ direct bilirubin.
- USMLE loves to mention Hx of autoimmune disease in the patient or family
member (because autoimmune diseases go together). So they’ll say she has
type I diabetes mellitus, SLE, or vitiligo; or her brother has RA, etc.
- They can mention a stone is present in the gallbladder on ultrasound, which
Primary biliary cirrhosis gets some students real emotional / confused, but it makes sense since patient
has ­ cholesterol. It’s just a distractor point and doesn’t relate to the actual
diagnosis at hand.
- Diagnose with anti-mitochondrial antibodies as first step, followed by liver
biopsy to confirm.
- Initial Tx = ursodeoxycholic acid (ursodiol).
- New NBME material for Step 1 asks a couple Qs on fat-soluble vitamin
malabsorption for PBC (deficiency of A and D on NBME for each Q, respectively,

MEHLMANMEDICAL.COM 20
MEHLMANMEDICAL.COM

with no info supporting those presentations; they just ask for hypothetical
vitamin deficiency). In theory, would be due to biliary obstruction, where ¯ bile
entering small bowel merely means ¯ fat absorption. This is nothing special to
PBC, but I mention it because it’s asked twice.
Autoimmune hepatitis - Young adult with ­ LFTs who has (+) anti-smooth muscle antibodies.

HY Hepatobiliary conditions
- Head of pancreas cancer impinges on common bile duct, resulting in
obstructive jaundice (­ ALP + ­ direct bilirubin) in smoker with weight loss,
or in patient who had gallbladder taken out years ago (so it clearly can’t be
due to a stone in common bile duct).
- Patient will not be febrile; can have dull abdominal pain.
- Courvoisier sign is a painless, palpable gallbladder in an afebrile patient
who’s jaundiced. This is pancreatic cancer until proven otherwise and is
Pancreatic cancer
pass-level.
- Pancreatic enzymes are normal in pancreatic cancer.
- USMLE wants CT of the abdomen to diagnose.
- Whipple procedure is done to remove head of pancreas. If the cancer is
isolated to the tail, distal pancreatectomy is the answer.
- Patients with pancreatectomy need pancreatic enzyme supplementation.
The exam can write this as “pancrelipase.”
- Bile duct cancer. Answer on USMLE if the vignette sounds like pancreatic
cancer but they tell you in the last line CT is negative.
Cholangiocarcinoma - Answer is ERCP as next best step.
- Smoking is risk factor, same as pancreatic cancer.
- Can be caused by Clonorchis sinensis (trematode).
- Stones in the gallbladder.
- Presents with biliary colic, which is acute-onset waxing/waning spasm-
like pain in the epigastrium or RUQ.
- Pain is due to cholecystokinin causing gall bladder contractions, where a
stone within the gallbladder transiently obstructs flow of bile into cystic
duct. They ask this on 2CK as well, where patient will biliary colic and
answer is "obstruction of cystic duct opening by stone." Not dramatic.
- Classic demographic is 4Fs = Fat, Forties, Female, Fertile for cholesterol
stones. This is because estrogen upregulates HMG-CoA reductase, causing
­ cholesterol synthesis and secretion into bile. NBME will give you
standard vignette of 4Fs, and then the answer will just be “increased
secretion of cholesterol into bile.”
- Another NBME Q gives vignette of cholelithiasis, and then rather than
asking for the diagnosis, they ask what the patient most likely has à
Cholelithiasis answer = “lithogenic bile.” Slightly awkward, but means promoting the
formation of stones.
- ­ cholesterol stones in pregnancy not just due to estrogen effect but also
because progesterone slows biliary peristalsis (biliary sludging).
Tangentially, progesterone also slows ureteral peristalsis, which is why
there’s ­ risk of pyelonephritis (as discussed in the HY Renal PDF).
- Cholesterol stones most common, but pigment (calcium bilirubinate)
stones are exceedingly HY for hereditary spherocytosis and sickle cell, due
to ­ RBC turnover.
- USMLE wants you to know that splenectomy is Tx for hereditary
spherocytosis to ¯ incidence of cholelithiasis. Sometimes the vignette
wants “cholecystectomy + splenectomy” as combined Tx. Or if they say
this was in a parent’s Hx in patient with low Hb, you know the Dx is
hereditary spherocytosis (autosomal dominant).
- Infections can also sometimes cause pigment stones, but LY.

MEHLMANMEDICAL.COM 21
MEHLMANMEDICAL.COM

- Diagnose with abdominal ultrasound.


- Ursodeoxycholic acid (ursodiol) ¯ secretion of cholesterol into bile.
USMLE just wants you to know this MOA + that it can be used in patients
with asymptomatic cholelithiasis, those declining cholecystectomy, and in
pregnancy.
- Cholelithiasis + fever.
- Vignette will sound exactly like cholelithiasis, but if we add a fever on top
of it, we now just call it cholecystitis.
- Virtually always due to obstruction by a stone + infection as a result.
- Diagnose with abdominal ultrasound, showing the stones.
- If ultrasound is negative, do a HIDA scan.
- Tx is cholecystectomy.
- Emphysematous cholecystitis is air visualized within the biliary wall.
USMLE wants Clostridium perfringens as the organism (can cause gas
gangrene anywhere) in this case.
Cholecystitis
- Chronic cholecystitis is calcification of the gallbladder (aka chronic
calculous cholecystitis, or “porcelain gallbladder”). It is due to repeated
bouts of acute cholecystitis.
- ALP and bilirubin will not be increased 19/20 questions. This is because
inflammation of the gallbladder doesn’t mean we have any form of
common bile duct obstruction. However there is one nonsense Q on a 2CK
NBME where 4 of the answers are wildly wrong, with correct answer being
cholecystitis in setting of high ALP + direct bilirubin. Since cholecystitis is
caused by stones virtually always, the implication is patients can
occasionally have concurrent choledocholithiasis.
- Stone anywhere within the biliary tree. Don’t confuse with cholelithiasis.
- Obstructive jaundice (­ ALP + ­ direct bilirubin) in patient who has Hx of
cholelithiasis or Hx of cholecystectomy performed within the past week.
- Regarding the latter, the Q can say patient had cholecystectomy
performed a week ago + “intra-operative cholangiography was not
performed.” The implication is bile duct patency is supposed to be
visualized during cholecystectomy to ensure there isn’t a stone retained
within the biliary tree. However, the Q need not say “intra-operative
cholangiography was not performed.” This makes the Q pass-level.
Choledocholithiasis - USMLE wants abdominal ultrasound followed by ERCP as the answer.
ERCP tends to show up as what they want, but there is a Q floating around
somewhere that asks for ultrasound first, so know that sequence.
- As I talked about earlier, I’ve never seen MRCP as a correct answer.
- Being able to discern choledocholithiasis from pancreatic cancer is vital
for USMLE. If they give you obstructive jaundice but say the patient is a
heavy smoker with weight loss, or hasn’t had a gallbladder for 25 years,
you know it’s head of pancreas cancer, not choledocholithiasis. And then,
once again, if it sounds like pancreatic cancer but the CT is negative, that’s
cholangiocarcinoma and we do ERCP.
- Gallstone pancreatitis is a type of choledocholithiasis in which the stone
has descended distally enough in the common bile duct that it now
obstructs the hepatopancreatic ampulla. This results in a backflow to the
pancreas + acute pancreatitis.
- USMLE wants “hepatopancreatic ampulla” as correct over “common bile
Gallstone pancreatitis duct” if they ask you to choose location of the obstruction. Slightly odd,
since the stone is still in the common bile duct, but hepatopancreatic
ampulla blockage is why we have the pancreatitis.
- Vignette will give obstructive jaundice but also high pancreatic enzymes.
- In other words, the Q will give ­ ALP, ­ direct bilirubin, and ­ amylase /
lipase.

MEHLMANMEDICAL.COM 22
MEHLMANMEDICAL.COM

- I repeat that pancreatic cancer will never have elevated enzymes on


USMLE, so this is a pass-level means to distinguish.
- ERCP is what USMLE will almost always have as the answer (diagnostic
and therapeutic). But if they force you to choose between what you do
first, ultrasound vs ERCP, you will still technically do an ultrasound first.
- Inflammation of the bile ducts. You must know Charcot triad for
cholangitis: 1) fever, 2) jaundice, 3) abdominal pain. We classically learn
this as “RUQ/epigastric pain,” but I can tell you USMLE doesn’t give a fuck
and will just say “abdominal pain.” I’ve had students get cholangitis Qs
wrong because they’re like, “I thought it had to be RUQ pain though.” And
I’m like, yeah, I agree, but USMLE doesn’t give a fuck.
- Will present 3 ways on USMLE:
- 2/5 Qs will be ascending cholangitis, which is infection from ascension of
bowel flora (E. coli, Bacteroides) up the common bile duct.
- 2/5 Qs will be primary sclerosing cholangitis, which is inflammation of the
bile ducts in the setting of IBD (always due to ulcerative colitis on USMLE,
Cholangitis
but can rarely be due to Crohn in real life). p-ANCA can be positive, and
there can be a beaded appearance of the common bile duct on
cholangiogram. On 2CK NBME 10, there is a Q on primary sclerosing
cholangitis where the patient is afebrile (rare; i.e., we don’t have full
Charcot triad), but they say patient has UC + obstructive jaundice + ERCP
shows “narrowing of the bile ducts,” so the answer is evidently PSC.
- 1/5 Qs will just give you a patient with history of cholelithiasis, where the
implication is a stone has descended causing choledocholithiasis, which in
turn led to obstruction + inflammation of the common bile duct.
- USMLE wants antibiotics + ERCP to diagnose and treat. Biliary drainage
by ERCP ¯ morbidity and mortality.
- ­ amylase and/or lipase in patient who has abdominal pain.
- Pain need not radiate to the back.
- Alcoholism and stones are two biggest risk factors.
- Hyperlipidemia and hypercalcemia can also occasionally be causes.
- Can be caused by drugs (didanosine is HIV NRTI; exenatide is a GLP-1
analogue for diabetes).
- Scorpion sting and mumps as causes are nonexistent on USMLE.
- Can be caused by choledocholithiasis (gallstone pancreatitis; ­ pancreatic
enzymes in setting of ­ ALP and ­ direct bilirubin).
- Results in enzymatic fat necrosis.
- You don’t need to memorize Ranson criteria (elaborate criteria for Surg
to determine prognosis in pancreatitis). What you need to know are two
main points:
Acute pancreatitis
1) ¯ Serum calcium and ­ glucose are bad prognostic indicators.
In fact, I’d say low calcium is quite possibly the highest yield
variable on USMLE for pancreatitis. There are many long/vague
pancreatitis Qs out there, where they’ll mention ¯ calcium in the
stem, and you’re like “Boom pancreatitis.”
2) The degree of lipase and/or amylase elevation doesn’t
correlate with prognosis. In other words, if amylase is 1200 versus
10,000, that’s not important for patient outcome.
3) The rest of the Ranson criteria is more Surg rotation
masturbation info and otherwise nonsense for USMLE.

- It’s to my observation NBME Qs will sometimes list high amylase without


mentioning lipase for whatever reason. Lipase is more specific for
pancreatitis, but it’s what NBME sometimes does, so don’t be confused.

MEHLMANMEDICAL.COM 23
MEHLMANMEDICAL.COM

- HY initial management for 2CK is a triad: 1) NPO (nil per os; or nothing by
mouth; this can be written as simply bowel rest as NBME answer); 2) NG
tube decompression; 3) IV fluids with normal saline. This triad is not
specific for pancreatitis, and can be done for a number of GI-related
conditions, including cholangitis and stones. But it just tends to get tested
a lot for pancreatitis.
- After this initial triad, “CT scan of the abdomen” is answer on NBME as
next best step to look for any degree of fluid collection. If a pancreatic
pseudocyst is present, NBME answer is to drain this by ERCP.
- It’s to my observation USMLE doesn’t assess antibiotics as part of
pancreatitis Tx, but apparently the carbapenems (like imipenem) have
fantastic penetration of pancreatic tissue.

- A 2CK NBME Q gives pancreatitis + similar image as above, and the


answer is just ERCP (for drainage of the pseudocyst within black circle).
- A necrosectomy is removal of pancreatic tissue in the setting of high
fraction of necrosis. Patients with pancreatectomy require enzyme
replacement (pancrelipase).
- Acute pancreatitis is HY cause of ARDS.
- Steatorrhea in alcoholic or patient with recurrent acute pancreatitis
episodes à results in pancreatic burnout with ¯ production of pancreatic
enzymes à ¯ proteases/lipases à malabsorption.
- In other words, we have normal pancreatic enzymes – i.e., they are not
elevated.
- Question can say alcoholic has ­ fat and protein seen in stool.
- Diabetes mellitus is possible if tail is affected due to obliteration of islets.
- If patient with diabetes due to chronic pancreatitis is supplemented with
insulin and gets hypoglycemic episodes, the answer is “absence of
Chronic pancreatitis
counter-regulatory glucagon.” This is because patient can also lose the a-
cells in addition to the b-cells. This answer of “absence of counter-
regulatory glucagon” is also correct for hypoglycemia in patients with Hx of
pancreatectomy.
- D-xylose test is normal. This is a monosaccharide that is readily absorbed
by the small bowel insofar as the intestinal lining is intact. So if we get
steatorrhea + normal D-xylose, this can be due to pancreatic insufficiency
or lactose intolerance. If D-xylose test is abnormal, we know the
malabsorption is due to an intestinal lining issue, such as Celiac or Crohn.

MEHLMANMEDICAL.COM 24
MEHLMANMEDICAL.COM

- If the USMLE gives you a CT for chronic pancreatitis, they will show you
calcification within the pancreas.

- Tx = pancreatic enzyme supplementation (pancrelipase).


- The answer on 2CK Surg if they give you patient with cholecystectomy
within the past week who now has fever, abdominal pain, and ­ direct
bilirubin.
Bile duct leak
- During the surgical anastomoses created during cholecystectomy,
sometimes there can be a post-op bile leak, literally. Weird diagnosis but it
shows up on occasion.
- The answer on USMLE if the Q sounds like gallstone pancreatitis (i.e., ­
Sphincter of Oddi dysfunction ALP, ­ direct bilirubin, ­ pancreatic enzymes) but the patient hasn’t had a
gallbladder for many years (so it can’t be a stone causing the obstruction).
- Gallstone enters small bowel and causes an intestinal obstruction.
- The answer on USMLE if they say “air in the biliary tree” (pneumobilia).
- Can be caused by cholecystoduodenal fistula, which is when a patient
with cholelithiasis can develop a conduit between the gallbladder and
Gallstone ileus
small bowel.
- What the USMLE will do is give you a long, rambling paragraph where
they say “air in the biliary tree” in the last line. The answer will then just be
“gallstone ileus” straight up, or it will be “cholecystoduodenal fistula.”
- Asked on 2CK Surg. What they’re going to do is give you a long, nonsense
paragraph with ­ ALP and ­ direct bilirubin + they say “CT shows a cystic
Choledochal cyst structure within the biliary tree.” Answer = “simple excision of the cyst.”
Students are huh, what’s going on here? à choledochal cyst. You have to
just excise it. Bullshit/dumb diagnosis. Take it up with NBME not me.
- As mentioned earlier, this will be a woman 20s-50s who has generalized
pruritis, ­ cholesterol, ­ ALP, ­ direct bilirubin, and Hx of autoimmune
disease in her or a relative.
Primary biliary cirrhosis
- Diagnose with anti-mitochondrial antibodies as first step, followed by
liver biopsy to confirm.
- Initial Tx = ursodeoxycholic acid (ursodiol).
- One of the highest yield diagnoses on 2CK.
- Bacterial infection of peritoneal fluid by mixed enteric flora.
- The answer on USMLE when they give you diffuse abdominal pain and
fever in one of the three following scenarios: 1) cirrhosis; 2) recent
peritoneal dialysis; 3) nephrotic syndrome.
Spontaneous bacterial
- Shifting dullness” or a “fluid wave” are buzzy for ascites, but questions
peritonitis (SBP)
will often omit these descriptors from stems. What they care about is you
identifying when a patient either has a major risk factor for ascites, or are
aware of a peritoneal intervention as the cause for the presentation.
- In other words, they can tell you a patient has cirrhosis + abdo pain +
fever, where they don’t have to mention a fluid wave, but you just have to

MEHLMANMEDICAL.COM 25
MEHLMANMEDICAL.COM

infer, “Well he/she clearly has major risk factor for ascites, so this sounds
like SBP.”
- There is a 2CK Peds Q where they say kid with minimal change disease
(nephrotic syndrome) has abdo pain + fever à answer = spontaneous
bacterial peritonitis.
- Next best step in diagnosis is abdominal paracentesis. This refers to
aspiration of fluid from the peritoneal cavity. Do not confuse this with
pericardiocentesis. “Paracentesis” as an answer shows up everywhere, and
I’ve seen students avoid it because they’re like, “What? I thought that
meant pericardiocentesis.”
- After the paracentesis is done, USMLE wants a very specific order for
what to do next. Choose “white cell count and differential” first if it’s
listed, followed by “gram stain and culture of the fluid.”
- This order is important because “gram stain and culture of the fluid” is a
correct answer in one Q but wrong answer in another Q, where “white cell
count and differential” is correct to do first.
- The reason white cell count and differential is done first is because SBP is
diagnosed when paracentesis shows >250 WBCs/µL.
- There is a new 2CK Q where they give ascites in patient with cirrhosis, but
do not mention fever or abdominal pain. However, they say paracentesis
shows 900 WBC//µL à answer = “antibiotic therapy.” It should be made
clear though that ~6/7 SBP Qs will give abdo pain and fever.
- Treatment is ceftriaxone.
- Obscure diagnosis where they tell you adult has an abdominal CT for
unrelated reason and has a 1-2-cm hepatic lesion with a central scar.
There will be zero mention of trauma or infection. Every student says WTF.
Focal nodular hyperplasia - This is a lesion of hepatocellular hyperplasia that does not require
treatment. This is the answer on NBME, where they tell you CT shows
hepatic lesion with central scar. Answer = “no further diagnostic studies
indicated.”

HY cirrhosis points
- Cirrhosis is a small, shrunken, burnt out liver due to chronic disease.
- HY causes are alcoholism, HepB/C, Wilson disease, hemochromatosis, NASH, a1-antitrypsin deficiency etc.
- “Burned out” means LFTs are normal or low – i.e., there is not transaminitis as with acute hepatitis.
- USMLE likes ­ PT and ¯ clotting factor synthesis in cirrhotic patients.
- Hyperammonemia occurs due to ¯ urea cycle activity (normally occurs in liver). This can cause hepatic
encephalopathy (confusion) and asterixis (“hepatic flap” of the hands).
- USMLE likes acute exacerbations of hyperammonemia caused by GI bleeds à ­ ammonia absorption.
- Spontaneous bacterial peritonitis (SBP) is ­­ HY on 2CK, as discussed above.
- Esophageal varices from ­ portal pressure that backs up to left gastric vein (discussed earlier).
- Caput medusae are visible periumbilical veins (superior epigastric veins).

Systemic inflammatory response syndrome (SIRS)


- Knowing this short table is vital for understanding many IM and Surg Qs on 2CK. If you’re studying for Step
1, I’d say it’s still worth knowing this.
- In the setting of stress (i.e., due to trauma, surgery, autoimmune flare, infection), catecholamines and ­
sympathetic activity might shift the patient’s vitals out of the normal range.
- The reason knowing SIRS is important is because the patient can have abnormal vitals without having an
infection.
- 2 or more of the following:
SIRS - Temperature <36C or >38 (<96. 8F or >100.4).
- HR >90.

MEHLMANMEDICAL.COM 26
MEHLMANMEDICAL.COM

- RR >20.
- WBCs <4,000 or >12,000.
Sepsis - SIRS + source of infection.
Septic shock - Sepsis + low BP.
- Sometimes you will see a patient’s vitals slightly out of the normal range in the setting of trauma, surgery,
or autoimmune flares, and you have to be able to say, “There’s no infection. That’s just SIRS from
sympathetic activation.”
- Knowing if a patient is septic is important for management of patients on 2CK, where sometimes antibiotic
regimens are stepped up. For example, when treating PID, if the patient is septic, intravenous ceftriaxone
and azithromycin is correct on one of the 2CK NBMEs; IM ceftriaxone and oral azithromycin is wrong. This is
because the latter is for most patients who have PID but aren’t septic.
- Ceftriaxone is frequently an answer on 2CK for in-hospital patients who are septic from a variety of
community-acquired conditions, e.g., pneumonia, pyelonephritis, prostatitis. For instance, community-
acquired pneumonia is empirically treated with azithromycin (on 2CK NBME 8), but if patient is septic, we
can go straight to ceftriaxone (have seen this more than once on 2CK NBMEs).
- For hospital-acquired infections in which patients are septic, NBME goes hard-hitting with vancomycin
PLUS ceftazidime or cefepime. This regimen covers MRSA and Pseudomonas.

Miscellaneous 2CK Surgery intestinal diagnoses


- One of the highest yield causes of small bowel obstruction (SBO) on
USMLE.
- Occur as a result of major abdominal surgery – i.e., hysterectomy,
laparotomy, appendectomy.
- Prior surgery results in inflammation that yields fibrin deposition within
the peritoneal cavity. This causes “fibrous adhesions” over time that the
small bowel can get caught in.
- Vignette will say, e.g., 50-year-old who had laparotomy 10 years ago for
gunshot wound now has obstipation (failure to pass flatus in addition to
stool) for 2 days + SIRS. This is pass-level and answer is just “adhesions.”
- New 2CK NBME has “mechanical obstruction of small intestine” as the
answer in patient with Hx of appendectomy. Pass-level, but that’s another
way they can write the answer.
Adhesions - As mentioned earlier, for many abdo conditions – i.e., SBO, pancreatitis,
cholangitis, etc., a general “triad” of NPO, IV fluids, and NG tube is done
first. In other words, you might get a simple adhesions Q, and then the
answer is just “0.9% saline” as the first step, where bowel rest and NG
tube aren’t listed.
- If the patient is stable, do abdominal X-ray if SBO is suspected to look for
gas / obstruction. If the patient is unstable (i.e., low BP), go straight to
laparotomy. Never do a CT scan in an unstable patient on USMLE (this
includes patients with normal BP after IV fluids given to restore low BP –
i.e., the fluids are just acting as a temporizing measure and you need to
find source of problem asap).
- Crohn disease can cause intraluminal fibrotic strictures, not adhesions.
There is a 2CK Q that lists both as answer choices. Strictures due to Crohn
are internal / within the lumen; adhesions are external fibrotic bands.
- Surgery is a stressor / form of trauma. Especially post-abdominal surgery
in which the bowel has been touched/manipulated, there can be acute
diminution of peristalsis resulting in pseudo-obstruction (i.e., the patient is
obstipated but there’s no physical obstruction).
Post-op ileus - Opioids can ¯ peristalsis / cause constipation. These should be
considered as a contributing factor to ileus. An important rule for USMLE is
that we always treat pain fully, even if patient has Hx of drug abuse or
ileus. There is a 2CK Q where a patient has ileus post-major surgery;
answer is “maintain dose of opioids + add stool softeners.”

MEHLMANMEDICAL.COM 27
MEHLMANMEDICAL.COM

- If the Q gives you Bristol 7 stool (watery diarrhea; no solid pieces) with
laxatives but Bristol 1 (severe constipation; pellet-like stools) when on
nothing, answer on NBME is “add fiber,” with the aim being Bristol 3-4.
- Large bowel pseudo-obstruction.
- Classically follows hip surgery for whatever reason.
- Think of this as ileus but of the large, not small, bowel. “Ileus” sounds like
ileum, which is part of small bowel. So ileus isn’t used to refer to large
Ogilvie syndrome
bowel pseudo-obstruction.
- Abdominal x-ray is what we do first in stable patients when we are
looking for suspected obstruction. If patient is unstable, always go straight
to laparotomy.
- Shows up on 2CK NBME. You just need to know this is the answer for
someone who has ¯ hemoglobin + bilious vomiting.
Duodenal hematoma
- The ¯ Hb is because the patient is bleeding internally.
- Bilious vomiting implies obstruction (almost always duodenal).
- Ultra-HY on 2CK; caused by Hx of duodenal ulcers.
- Highest-yield point is that this is diagnosed with chest and abdominal x-
rays showing air under the diaphragm. Presents two ways:
1) Q will give patient who has Hx of abdo pain after meals (implying Hx of
duodenal ulcer) + now has SIRS + acutely worse abdo pain + rigid abdomen
Duodenal perforation à answer = “x-rays of the chest and abdomen.”
2) Q will say acute-onset epigastric pain as though the patient was kicked
in abdomen + board-like rigidity + x-rays show air under the diaphragm à
answer = “immediate surgical exploration of upper abdomen.”
- Not a hard presentation or management, but it shows up all over the
place + students get this wrong all the time. Easy points.
- Complication of splenectomy. Shows up on 2CK Surg.
Subphrenic abscess - Patient will have fever + abdo pain + leukocytosis post-splenectomy +
they’ll ask for diagnosis à answer = subphrenic abscess. Tx = drain.
- Answer on 2CK in patient who has diarrhea + hypoglycemia post-major
surgery of the stomach/small bowel.
Dumping syndrome - Post-gastrectomy, for instance, “rapid transit of hyperosmolar chyme” is
an answer on NBME for the mechanism. This triggers a ­­ spike in insulin
due to glucose-dependent insulinotropic peptide.
- Answer on 2CK in a patient who has diarrhea post-major surgery of the
stomach/small bowel without hypoglycemia. The vignettes can otherwise
sound pretty similar.
Blind loop syndrome - This is when a part of the proximal small bowel forms a crevice or nook in
which peristalsis bypasses this “blind” segment, thereby creating stasis
within it. This can lead to small intestinal bacterial overgrowth (SIBO) and
diarrhea.

HY Referred pain
Spleen - Splenic laceration à ULQ pain +/- can refer to left shoulder (Kehr sign).
- Diaphragmatic irritation can cause pain going to left shoulder (asked on NBME); spleen is
Diaphragm
wrong answer. The key here is they ask “irritation.”
Gallbladder - RUQ or epigastric pain +/- can refer to right shoulder.
- Epigastric pain initially (visceral peritoneal inflammation) that migrates to RLQ (parietal
Appendix
peritoneal inflammation).

MEHLMANMEDICAL.COM 28
MEHLMANMEDICAL.COM

GI Surgeries for 2CK


- There are countless fancy GI surgeries that could be discussed just for the sake of it, but it’s a vehement
waste of time. I’m going to comment on just stuff I think is HY for USMLE, without the nonsense.
- The Billroth II, or gastrojejunostomy, is a partial gastrectomy that shows up
sometimes on Surg forms as causing dumping or blind loop syndromes.
- In other words, they’ll say a patient has diarrhea following a Billroth II
gastrojejunostomy and then ask for diagnosis à answer = blind loop syndrome.
- Or they’ll give diarrhea + hypoglycemia (dumping syndrome), and the answer is
just “rapid transit of hyperosmolar chyme.”
Gastrectomy
- Can cause B12 deficiency due to removal of parietal cells. Can also cause B1
deficiency (asked twice on new 2CK NBMEs) à confusion/nystagmus (i.e.,
Wernicke findings; B12 deficiency doesn’t present with these); can have (+)
Romberg sign in both B1 and B12 deficiency Qs (for B1, this is part of dry beriberi;
for B12, this is part of subacute combined degeneration).
- Done for gastric cancer and perforations.
- In contrast to gastrectomy, no part of the stomach is removed here. A pouch is
made where part of the stomach is merely walled off from food passage.
- Answer on Family Med forms weirdly enough for the most effective way to
manage weight in someone who is morbidly obese, albeit last resort. Not sure
why this is on FM forms but they ask it.
Gastric bypass
- Sequelae same as for gastrectomy. B12 deficiency can occur due to disruption of
parietal cells, even though stomach is not removed.
- Another name for gastric bypass is roux-en-Y, which will show up tons on 2CK
Surg forms as a wrong/distractor answer. I’ve only seen “gastric bypass surgery”
as correct on FM forms, but never “roux-en-Y” on Surg forms.
- Aka pancreaticoduodenectomy.
- Used for head of pancreas cancer.
- Because of interconnected blood supply, removal of head of pancreas requires
removal of the gall bladder, duodenum, and proximal jejunum.

Whipple procedure

- As I said, just know that this is done for head of pancreas cancer.
Distal pancreatectomy - Answer on Surg form for tail of pancreas cancer. Not complicated, but it’s asked.
- Even though segmental colectomies (many variants exist) can be used to treat
localized cancers of the colon, it’s to my observation that basically all colon
cancer Qs on NBME have total colectomy as the answer if they force you to
Colectomy choose.
- A 2CK NBME Q gives an 18-year-old with recently diagnosed FAP, and the
answer is “total proctocolectomy.” Serial colonoscopy is wrong answer, since
chance of cancer is 100%. USMLE wants total removal once patient is 18.

MEHLMANMEDICAL.COM 29
MEHLMANMEDICAL.COM

- A difficult newer 2CK NBME Q gives patient with uncontrolled colonic bleeding
and severely low Hb despite transfusions + they say a radiouptake scan shows
localization to one part of the colon à answer = total colectomy; hemicolectomy
is wrong answer.
- Keyhole surgery used for appendectomy, cholecystectomy.
Laparoscopy
- Highest yield indication on USMLE is diagnosis/treatment of endometriosis.
- Used for unstable (low BP) patients after abdominal trauma (e.g., ruptured
spleen from MVA); also for ruptured ectopic pregnancies where patient is
unstable.
- If patient is unstable, never pick CT on USMLE. Go straight to laparotomy.
- If patient has abdominal trauma + low BP + was given fluids and now has normal
BP, do not do CT. Go straight to laparotomy. The fluids are just a temporizing
measure (buying time), but the underlying problem still needs to be fixed asap.
- Penetrating trauma to the abdomen (usually gunshot wounds) requires
Laparotomy
immediate laparotomy, even if the patient is stable. Below the level of the nipples
is considered “abdomen” in this scenario.
- USMLE loves this as major cause of post-surgical adhesions (SBO months to
years later).
- Celiotomy is another name for laparotomy that shows up on some of the 2CK
CMS Surg forms. It’s a correct answer on one of the forms, where every student
says wtf. Old-school surgeons might remark on differences between laparotomy
and celiotomy, but USMLE uses the terms interchangeably.
- Done in setting of trauma (splenic laceration), hereditary spherocytosis, or ITP.
- “Autosplenectomy” refers to loss of spleen due to repeated microinfarcts in
sickle cell.
- “Asplenia” can refer to anyone who’s had splenectomy, autosplenectomy, or
who didn’t develop a spleen from birth (Ivemark syndrome on 2CK NBME).
- The white pulp of the spleen ordinarily contains 50% of the immune system’s
reservoir of macrophages.
- Patients are susceptive to encapsulated organisms because the spleen has a
high phagocytic capacity, where opsonization with C3b and IgG, followed by
phagocytosis in the white pulp, is how they are cleared.
- Patients need vaccines against Strep pneumo, Neisseria meningitidis, and
Haemophilus influenzae type B.
Splenectomy
- Penicillin prophylaxis is given to patients with asplenia until age 5, or until 1 year
post-surgical splenectomy. If the USMLE forces you to choose which organism this
protects against the most, the answer is Strep pneumo.
- However, if USMLE vignette tells you a patient with asplenia misses a penicillin
prophylaxis dose + now has sepsis, the Tx is a 3rd generation cephalosporin (i.e.,
cefotaxime or ceftriaxone), not penicillin. The latter is only for prophylaxis.
- Howell-Jolly bodies (nuclear remnants) are seen within RBCs on a smear in
patients with asplenia. This is because senescent and abnormal RBCs are
ordinarily cleared out by the red pulp of the spleen.
- USMLE wants you to know the lifespan of a RBC is 4 months (118 days). They ask
this on NBME as how long it will take for all carbon monoxide to be cleared out of
the blood in the setting of exposure (i.e., not until all RBCs exposed are removed).

HY GI Diagnostic modalities for Surg/IM (2CK mostly)


- Same as with the GI surgeries table above, we could discuss endless indications for the below modalities,
but I’m staying concise here with just the highest yield points.
- “Insertion of NG tube” is the answer for tracheoesophageal fistula and choanal
Nasogastric tube
atresia (discussed in HY Pulm PDF).
- For suspected Zenker; shows outpouching.
Barium swallow
- First step in achalasia (before monometry); shows bird’s beak.

MEHLMANMEDICAL.COM 30
MEHLMANMEDICAL.COM

- Do not use for esophageal perforation. Can cause mediastinitis if it leaks out
through a hole in the esophagus.
- Can be used for patients with aspiration risk, since it doesn’t cause pneumonitis.
- Aka water-soluble contrast swallow.
- Used for esophageal perforations because it doesn’t cause mediastinitis.
Gastrografin swallow
- Do not use if patient has aspiration risk; causes pneumonitis. Use barium
instead.
Esophageal - Answer for achalasia after barium swallow shows bird’s beak.
manometry - A pressure study of the esophagus.
- Aka esophagogastroduodenoscopy.
- Immediate answer for any patient with new-onset dysphagia and Hx of GERD or
heavy smoking/alcohol (for esophageal cancer). Then biopsy any lesion/stricture.
Endoscopy
- Endoscopy + banding for esophageal varices emergent management.
- Diagnosis of hiatal hernia (asked on NBME).
- Done in patients over 50 who have H. pylori positivity (on newer 2CK form).
Capsule endoscopy - Always wrong fucking answer on USMLE.
- Congenital midgut volvulus in pediatrics; will show a corkscrew.
Upper GI series
- This is a contrast swallow followed by X-rays to visualize the upper GI tract.
- Cholelithiasis.
- First step to diagnose cholecystitis; if negative, do HIDA scan.
Abdominal ultrasound - First step for choledocholithiasis, then do ERCP.
- Intussusception Dx, then do enema (definitively diagnostic and therapeutic).
- Pyloric stenosis.
- Answer for confirmatory diagnosis of cholecystitis (not cholelithiasis alone) if
ultrasound is negative.
HIDA scan - Radiocontrast is injected + secreted into bile. If gallbladder lights up, there is no
obstruction of the cystic duct and it is negative; if gallbladder doesn’t light up, we
know there’s an obstruction by a stone and it confirms cholecystitis.
- USMLE won’t force you to choose, but just assume contrast CT is always used.
The only times non-contrast CT will be an answer is for urolithiasis and
intracranial bleeds.
- Diagnosis of pancreatic cancer (highest yield indication on USMLE).
CT of abdomen
- Diagnosis of liver cancer and focal nodular hyperplasia.
- Renal injury (ultra-HY; discussed in HY Renal PDF).
- Blunt force trauma to abdomen in patient who is stable.
- Diverticulitis.
- Used to look for bowel gas in suspected obstruction (e.g., sigmoid volvulus
showing coffee bean sign).
- Duodenal atresia (double-bubble sign).
- Congenital diaphragmatic hernia (bowel gas in left hemithorax).
Abdominal x-ray
- Necrotizing enterocolitis (pneumatosis intestinalis).
- Toxic megacolon if patient is stable.
- “X-rays of chest and abdomen” used for duodenal ulcer perforation to look for
air under the diaphragm.
- Endoscopic retrograde cholangiopancreatography; type of EGD that can also
enter the biliary tree, remove stones there, and inject contrast if necessary.
- Answer on USMLE for choledocholithiasis (including gallstone pancreatitis) after
ERCP
ultrasound is performed.
- Answer for diagnosis of cholangitis.
- Answer for drainage of pancreatic pseudocyst.
- Magnetic resonance cholangiopancreatography.
- Never seen this as correct answer on NBME, but I observe that students always
pick it when they don’t know what’s going on, maybe because it sounds weird
MRCP
and specific.
- Can visualize biliary tree much more safely than ERCP, but unlike ERCP, it isn’t a
form of treatment (ERCP is both diagnostic and therapeutic).

MEHLMANMEDICAL.COM 31
MEHLMANMEDICAL.COM

- Never seen this as correct answer, only wrong answer.


- Can be done to diagnose pancreatic cancer if CT is negative or to drain
Endoscopic ultrasound
pancreatic fluid collections in place of ERCP. But once again, never seen this as
correct.
- Used for spontaneous bacterial peritonitis.
Paracentesis - As mentioned earlier, choose “white cell count and differential” before “gram
stain and culture of the fluid.” Do not confuse with pericardiocentesis.
Meckel scan - Radiocontrast uptake scan that localizes to the diverticulum at terminal ileum.
- Done for diagnosis of colorectal cancer, IBD, pseudomembranous colitis.
- Do not do in diverticulitis (can cause perforation; do CT instead).
- Do if patient has anal cancer prior to excision in order to first see if there’s
greater extent of cancer (might change management). Asked on 2CK form, where
they say there is cancer at anal verge, and excision is wrong; answer is
colonoscopy.
- Commence at age 45-50 (guidelines are evolving) and then do every 10 years.
- USMLE will not force you into a position where ohemgee is it 45 or 50. If they
force you to choose 45 as a new guideline, it will be obvious the other answer
choices are wrong.
- If first-degree relative (parent or sibling) has colon cancer, commence at age 40
or 10 years prior to diagnosis in that family member, whichever is earlier. In other
words, never later than 40. For example, if dad was diagnosed at 58, commence
Colonoscopy at age 40. If dad was diagnosed at 44, commence at age 34. Then do every 5
years.
- If patient has IBD (UC or Crohn), commence 8 years after the diagnosis was
made and then do every 2-3 years. 2CK NBME is real slick about this. They give
mid-30s patient with IBD diagnosed in 20s + also has dad diagnosed with colon
cancer in 50s; wrong answer = colonoscopy at age 40; correct answer is
“colonoscopy now,” since patient has IBD and should have had one done 8 years
after his/her diagnosis.
- Patients who have history of dysplastic polyps need repeat colonoscopies every
2-5 years, depending on size/morphology of polyp(s).
- If patient has HNPCC (Lynch syndrome), start at 20-25, then do every 1-2 years.
- For FAP, start at age 10-12 and do every 1-2 years. Then do prophylactic
proctocolectomy at age 18 (on NBME).
- For Peutz-Jeghers, start at age 8, then every 1-2 years.
- “Sigmoidoscopy-guided insertion of rectal tube” is answer on NBME for
treatment of sigmoid volvulus.
Sigmoidoscopy - I’ve never seen sigmoidoscopy as a diagnostic tool. I make this point because
there are alternative theoretic regimens to colon cancer screening – i.e.,
sigmoidoscopy + barium enemas, etc., but I’ve never seen these assessed.
- Shows up on newer 2CK form as answer for diagnosis of hemorrhoids before
Anoscopy
banding is performed. This will help determine extent of hemorrhoids.

HY Polyp conditions for USMLE


- Hereditary non-polyposis colorectal cancer (HNPCC).
- Mismatch repair genes MSH2/6, MLH1, PMS2.
Lynch syndrome - Mutations cause “microsatellite instability.”
- Colonic polyps/cancer; also associated with gynecologic cancer.
- Start colonoscopy at age 20-25, then do every 1-2 years.
- Familial Adenomatous Polyposis (FAP); chromosome 5; AD.
- Hundreds to thousands of polyps on colonoscopy; 100% cancer risk.
Familial adenomatous - Start colonoscopy at age 10-12 and do every 1-2 years. Then do prophylactic
polyposis proctocolectomy at age 18 (on NBME).
- FAP + soft tissue (e.g., lipoma) or bone tumors (e.g., of the skull) = Gardner
syndrome.

MEHLMANMEDICAL.COM 32
MEHLMANMEDICAL.COM

- FAP + CNS tumors = Turcot syndrome.


- As talked about at the start of this PDF, this is combo of perioral melanosis and
Peutz-Jeghers hamartomatous colonic polyps.
- Start colonoscopy at age 8, then do every 1-2 years.
- Shows up on a 2CK NBME.
- Q will tell you there’s a teenager (i.e., juvenile, LOL!) with intermittent bleeding
per rectum + colonoscopy shows scattered polyps + biopsy shows “dilated, cystic,
Juvenile polyposis
mucus-filled glands with abundant lamina propria and inflammatory infiltrates”
à answer = juvenile polyposis.
- Only question I’ve seen on it, but it’s on new 2CK NBME so I have to mention it.
Hyperplastic polyps - All you need to know is these are not pre-cancerous / have no dysplasia.
- Dysplastic polyps that precede full-blown colorectal cancer are classically either
tubular or villous, AND either pedunculated or sessile.
- Villous is worse than tubular. Sessile is worse than pedunculated.
- This means pedunculated tubular polyps are “best” and sessile, villous polyps
are “worst.”
- Sessile means flat; pedunculated means “sticks out.”
- Polyps can have mixed chacteristics and hence be tubulovillous.
- What you need to know is: USMLE will give you a random 65-year-old with a
Tubulovillous
polyp + show you a pic + ask you what it is à answer = “tubular polyp.” Student
freaks out and says, “Wait, we need to know polyp pics??” à No. The big picture
concept is that older people who get colorectal cancer will have tubular, villous,
or tubulovillous polyps as I just said. Wrong answers would be things like
hamartomatous, juvenile, and hyperplastic. You can just eliminate to get there
without even knowing the image.
- 2CK wants you to know that patients with history of dysplastic polyps need
repeat colonoscopies every 2-5 years, depending on size/morphology of polyp(s).

Colon cancer short points


- KRAS gene is the answer on USMLE for the first gene mutated in colonic polyps, prior to progression to
overt colon cancer.
- Colon cancer often develops as a result of progressive mutations, rather than one mutation straight-up. In
other words, first KRAS, then PTEN, then DCC, then TP53.
- If they tell you a colon cancer has metastasized and force you to choose a gene that’s mutated, go with
TP53 for p53 protein.
- If they tell you a polyp is seen and there is no evidence of invasion of the stalk, choose KRAS. This reflects
earlier changes in the sequence as I just wrote above.
- Can cause colovesicular fistulas, where a passageway develops between the GI tract and bladder, leading
to UTIs and mixed enteric flora in the urine. This is also assessed for diverticulosis on 2CK and seems to be a
new diagnosis USMLE likes. This could also be due to Crohn, in theory, but I haven’t seen it on NBMEs yet.
- Can cause Strep bovis endocarditis. Obscure, but rare point assessed on USMLE.
- I already discussed colon cancer screening extensively above.

HY Peds GI diagnoses
- Forceful/projectile non-bilious vomiting in neonate days to weeks old. Obstruction is
above level of duodenum, so we won’t see bile.
- Students fixate on exact age of the kid for pyloric stenosis versus duodenal atresia.
USMLE doesn’t give a fuck and will give variable ages.
Pyloric stenosis - Hypertrophic pylorus; “olive-shape mass” in abdomen is buzzy but rarely seen in Qs.
- Ultrasound done to diagnose; myotomy to treat.
- USMLE wants you to know this is almost always a one-off / sporadic developmental
defect, where the neonate will not have a broader syndrome. In contrast, duodenal
atresia is usually Down syndrome.

MEHLMANMEDICAL.COM 33
MEHLMANMEDICAL.COM

- Bilious vomiting in neonate. Bilious = obstruction at level of duodenum or lower.


- Associated with Down syndrome, albeit not mandatory.
- Will cause double-bubble sign on abdominal x-ray.

Duodenal atresia

- Another cause of duodenal obstruction with double-bubble sign, but much more rare
than duodenal atresia.
- Step 1 is pass/fail now, but you could be aware that this is caused by “abnormal
Annular pancreas
migration of ventral pancreatic bud.” This was the type of garbage we memorized back
in the numerical Step 1 days. Mentioning it here because I think it still floats around on
the Step.
- Distinguishing between intussusception and congenital midgut volvulus is annoying as
fuck, so I’ll do my best to elucidate below based on what I’ve seen across NBMEs.

Abdominal exam:

- Intussusception can present with an abdominal mass, which can be described as


“sausage-shape.” This will usually occur in the RLQ.
- Midgut volvulus is usually first month of life + has no overt mass.
- “Dance sign” for retraction of RLQ in intussusception is floated around in the
literature quite a bit, but I haven’t seen USMLE ever give a fuck or make this a
conspicuous point. It’s the mass versus no-mass that I see as important.

Imaging:

- Midgut volvulus shows corkscrew appearance of small bowel on upper-GI series. This
Intussusception
is ultra-HY.
vs
- Intussusception shows a target sign or sausage-mass on ultrasound. However, USMLE
Midgut volvulus
doesn’t tend to mention imaging often for intussusception.
- If abdominal x-ray is performed, dilated loops of small bowel with air-fluid levels may
be seen in both. This is a non-specific finding and is buzzy for intestinal obstruction. Do
not confuse with air-fluid levels on CXR, which mean pulmonary abscess.

Age:

- Intussusception is usually 3 months – 2 years.


- Midgut volvulus is usually first month of life, with 90% under age 1.
- However, the fact that there’s overlap causes annoyance answering USMLE Qs and I
see this as a less reliable variable.
- There is a volvulus Q on Peds CMS form 6 where they give a 5-month-old (age range
more for intussusception), but they tell you overtly an upper-GI series is performed
and show you the image of the corkscrew. They also explicitly say there are no masses.
So I see this as: abdominal exam and imaging are buzzy + age not as reliable.

MEHLMANMEDICAL.COM 34
MEHLMANMEDICAL.COM

My view on how to distinguish for NBME/USMLE:

- Both intussusception and midgut volvulus can present with bilious vomiting, air-fluid
levels on abdominal x-ray, bloody stools, and intermittent crying/vomiting/squatting.
Once again, they sound very similar. So I’ve more or less been able to converge on the
combo of abdominal mass as most important, followed by buzzy imaging as second-
most important, followed by age.
- Now I will talk about each condition separately below.
- Intussusception is telescoping of the bowel into itself; 90% occurs in jejunum or
ileum of small bowel. There is an NBME Q that basically gives a vague one-liner for
intussusception where they ask the location, and the answer is just “jejunum,” since
it’s the only small bowel option listed.

Intussusception
- Almost always 3 months – 2 years of age.
- Idiopathic, but can be triggered by viral infection or rotavirus vaccination, where
mesenteric lymphadenopathy can act as a “lead point” for the intestinal telescoping.
Underlying Meckel diverticulum is also a common lead point.
- In ultra-rare scenarios, intussusception in elderly can occur due to colon cancer.
- Classic presentation is intermittent/colicky abdominal pain, where the kid will have
episodes of drawing the legs to the chest or squatting, with blood in the stool +/-
bilious vomiting.
- An abdominal mass (i.e., sausage-shaped mass) is a key feature, but not in all Qs.
- Diagnosis is first done with ultrasound, which may show a sausage mass or target sign
(I’ve never seen NBME show the image, so they don’t care). After the ultrasound,
enema is both diagnostic and therapeutic. USMLE doesn’t care what kind of enema is
used; air-contrast enema is classic, but you will see all different types of enemas as
correct answers.
- “Air enema with ultrasound” is the answer for intussusception on 2CK Surg form 7.
- Failure of rotation of proximal bowel.
- Midgut volvulus is usually first month of life, with 90% under age 1.
Midgut volvulus - There is no abdominal mass.
- Upper-GI series shows corkscrew appearance.
- Treatment is surgical.

MEHLMANMEDICAL.COM 35
MEHLMANMEDICAL.COM

- As example, USMLE will show you above image in 5-month old where you say, “Ok,
that’s age more for intussusception.” But then they’ll tell you upper-GI series is
performed + they show you the corkscrew à clearly midgut volvulus, not
intussusception. Answer will be “failure of rotation of proximal bowel,” or “gut
malrotation.”
- Outpouching near the terminal ileum.
- True diverticulum. Contains all layers of bowel (mucosa, submucosa, muscule).
- Caused by incomplete obliteration of vitelline (omphalomesenteric) duct.

Meckel
diverticulum

- Highest yield point for USMLE is that it can contain heterotopic gastric or pancreatic
mucosa, resulting in bleeding and blood in the stool.
- Another important point is that Meckel need not be pediatrics. This notion of rule of
2s (i.e., 2 feet from terminal ileum, 2 types of heterotopic tissue, age 2) is garbage with
respect to age for USMLE. There are Qs where it shows up in adults and students get
them wrong saying, “Wait, I thought it was supposed to be kids?!” à No. USMLE will
happily give you a 19-year-old with Meckel.
- Diagnosed with Meckel (Tc99) scan, which localizes to terminal ileum.
- Only treated surgically if symptomatic with bleeding. Otherwise can be left alone.
- Necrotic bowel occurring in premature neonates born <32 weeks’ gestation.
Necrotizing
- Presents with pneumatosis intestinalis (air in bowel wall); resembles bubbles.
enterocolitis

MEHLMANMEDICAL.COM 36
MEHLMANMEDICAL.COM

- USMLE will show you above image in kid born, e.g., at 26 weeks’ gestation, and then
answer is just necrotizing enterocolitis. Not hard.
- It’s to my observation necrotizing enterocolitis usually shows up as a wrong answer
choice on NBMEs, where I see students pick it in, e.g., an adult, and I’m like, “Dude you
see that in fucking hyper-preemies.” And they’re like Oh. So you need to remember
this as specifically a premature neonate condition.
- Failure of migration of neural crest cells distally within the colon.
- Causes an aganglionic distal segment that results in ­ anal sphincter tone.

Hirschsprung

- Often associated with Down syndrome.


- Presents usually as failure to pass stool at birth, as chronic constipation in an infant,
depending on the severity. But do not confuse with “meconium ileus,” which is a term
that refers to failure to pass stool at birth specifically due to cystic fibrosis.
- Diagnose with anal manometry showing ­ anal sphincter tone, followed by biopsy.
- Discussed in extensive detail in the HY Pulm PDF.
- Just a reiteration that meconium ileus + exocrine pancreatic insufficiency are
exceedingly HY.
Cystic fibrosis - Secretions within the pancreatic ducts are inspissated (meaning desiccated / dried up
within a lumen), making them sticky. This means the enzymes can’t make their way to
the duodenum à fat-soluble vitamin malabsorption à NBME exams love vitamin E
deficiency in CF in particular (presents as neuropathy).

MEHLMANMEDICAL.COM 37
MEHLMANMEDICAL.COM

- D-xylose test is normal, since the intestinal lining/architecture is intact. Any


malabsorption that occurs is due to mere paucity of enzymes in the small bowel. Recall
that abnormal D-xylose test would be Celiac and Crohn on USMLE.
- As discussed earlier, ­­ direct bilirubin in a kid under the age of 6 weeks.
- Caused by lack of development of the intrahepatic bile ductules and biliary tree.
Biliary atresia
- Ultrasound will be done first, but USMLE wants liver biopsy to confirm the diagnosis.
- Kasai procedure is done to treat, followed by liver transplant if unsuccessful.

Jaundice in Peds (2CK only)


- Normal bilirubin should be ~1.0 mg/dL.
- Direct bilirubin should be ~0.1 mg/dL.
- Pathologic jaundice in peds = if any of the following is/are present:

1) Any jaundice on the first day of life (first 24 hours of life), period = pathologic.
2) Jaundice present after one week if term, or after two weeks if preterm = pathologic.
3) Total bilirubin >15 mg/dL.
4) Direct bilirubin >10% of total bilirubin, even if total bilirubin is <15 mg/dL.
5) Rate of change of increase in bilirubin >0.5 mg/dL/hour.

- If none of the above 5 is met but the neonate has jaundice, the diagnosis is physiologic jaundice.
- If the kid has pathologic jaundice, we then want to explore the causes (all discussed below).
- Tx for pathologic jaundice = phototherapy first, followed by exchange transfusion; some literature makes
a case for IVIG after phototherapy, but USMLE has exchange transfusion as correct, without listing IVIG.
- If ­ bilirubin in the neonate accumulates in the CNS grey matter, this is called kernicterus and can cause
irreversible neurologic damage. In the gross specimen below, bilirubin has deposited in the basal ganglia
(pink arrows pointing to yellow areas). USMLE will show similar image, where the answer is just “putamen.”

- Due to b-glucuronidase in breast milk, which leads to de-conjugation of


intestinal bilirubin + increased enterohepatic circulation.
- Jaundice starts on day 3-5 and peaks at 2-3 weeks.
Breastmilk jaundice
- Tx = stop breastfeeding for ~48 hours (and do bottle feeding), which leads
to a rapid ¯ in bilirubin; once breastfeeding is resumed, bilirubin might rise,
but not back to pathologic levels.
- Insufficient feeding (e.g., failure of suckling, etc.) + decreased milk intake
leads to reduced intestinal clearance of bilirubin + increased enterohepatic
circulation.
Breastfeeding jaundice - Jaundice peaks at 3-5 days.
- Tx = formula feeding (fluid + caloric supplementation).
- What USMLE can do is tell you neonate has difficulty attaching to the
breast during feeds + has jaundice, and you’re like “Omg breastfeeding

MEHLMANMEDICAL.COM 38
MEHLMANMEDICAL.COM

jaundice!” But then you’ll see that 0/5 criteria are met for pathologic
jaundice, and the answer to the Q is just “physiologic jaundice.”
- Rh(-) mom experiences mixing of blood from Rh(+) fetus during first
pregnancy (i.e., from traumatic birth, abortion, or instrumentation like
amniocentesis). This causes mom to develop antibodies against Rh antigen.
- In a subsequent pregnancy, these antibodies, which have now evolved to
be IgG, will cross placenta and attack Rh(+) fetal RBCs that have Rh on their
surface à hemolysis.
- Can vary in severity between death in utero versus mild neonatal jaundice.
- There is 2CK NBME Q where they say woman in 3rd trimester of first
Hemolytic disease of the pregnancy has positive titers for anti-D (which means anti-Rh) + had Hx of
newborn (Rh type) bleeding in 1st trimester; they ask diagnosis à answer = Rh isoimmunization.
In other words, there was mixing of blood during early pregnancy, and now
she’s developed antibodies against Rh. However, this won’t cause Rh-type
hemolytic disease of newborn in first pregnancy; the actual hemolytic
disease will occur from 2nd pregnancy onward.
- Another 2CK Q gives ­ fetal HR in 3rd trimester in woman who’s in 2nd
pregnancy who has (+) “anti-D titers.” They ask reason why fetal HR is ­
answer = “Rh isoimmuniziation” à destruction of fetal RBCs à ¯ oxygen
delivery within fetal circulation à ­ HR to compensate.
- Mothers with O blood type will have fractional IgG (instead of IgM) against
A and B antigens à cross placenta à fetal hemolysis.
Hemolytic disease of the - USMLE will give this to you has showing up in a first pregnancy in a mother
newborn (ABO type) who is O+. Can this occur in O(-) mothers in 2nd pregnancies onward? Yes.
But USMLE does this so you can’t accidentally get lucky with the Dx if you
only know about Rh-type hemolytic disease of the newborn.

Autoimmune intestinal absorptive disorders


- Intolerance to gluten (i.e., to gliadin proteins found in wheat, oats, rye, and
barley; but not rice).
- Causes type-IV hypersensitivity response where T cells attack the small intestinal
villi, resulting in flattening of the villi and malabsorption. The biopsy/image is
pass-level. Below, we have normal villi on the left, and flattening of villi in Celiac
on the right.

Celiac disease

- Even though the disorder is T-cell-mediated destruction of the villi, patients still
develop antibodies which are HY: anti-endomysial (aka anti-gliadin) and anti-
tissue transglutaminase IgA.
- Patients with concurrent IgA deficiency will have false (-) results for tissue
transglutaminase IgA antibody. Since “autoimmune diseases go together,” and

MEHLMANMEDICAL.COM 39
MEHLMANMEDICAL.COM

“autoimmune and immunodeficiency syndromes go together” patients with IgA


deficiency have 15x greater likelihood of developing Celiac.
- Celiac presents as vague bloating/diarrhea in patients who might not be able to
pinpoint what they’re eating to cause the symptoms.
- I’d say one of the highest yield points on USMLE regarding Celiac is that it can
cause iron deficiency anemia. This is because iron absorption in the duodenum is
impaired from the flattened villi.
- For example, you might get a difficult/vague vignette where you’re not sure if
the diagnosis is Celiac or lactase deficiency, but you see that hemoglobin is low,
and you’re like “Boom. Celiac.”
- Associated with dermatitis herpetiformis, which presents as itchy, vesicular
lesions on extensor areas. This causes “IgA deposition at dermal papillae.”

- D-xylose test is abnormal because the intestinal lining/architecture is abnormal


(i.e., we have flattening of villi).
- Increased risk of gastrointestinal T-cell lymphoma in Celiac. This is called
enteropathy-associated T-Cell lymphoma (EATL).
- Diagnosis is made via antibody screening, followed by duodenal biopsy as
confirmatory.
- Treatment of Celiac is simply with gluten-free diet.
- Deficiency of brush border disaccharidase (i.e., aka lactase deficiency).
- Brush border is located at “tips of the villi.” This is in contrast to stem cells of the
GI tract, which are located at the “base of the crypts.” USMLE asks the locations
of these things.
- Diarrhea/bloating in response to dairy.
- In contrast to Celiac disease, there is no iron deficiency anemia.
- ­ incidence in Asians, but can also develop idiopathically starting in teenage
years-onward. When a young adult has a new-onset diarrhea-related condition,
it’s usually lactose intolerance, as lactase production can ¯ with age. But once
Lactose intolerance again, USMLE can trick you, so I’ll be an asshole and reiterate that if you see ¯ Hb,
choose Celiac instead.
- Secondary lactose intolerance can occur following viral gastroenteritis (e.g.,
rotavirus or Norwalk virus) à sloughing of the tips of villi à vignette will say
new-onset diarrhea/bloating following gastro illness 1-2 weeks ago. This is only
transient and patient will recover. This is HY cause of lactose intolerance.
- Biopsy is normal. This is in contrast to the flattened villi in Celiac.
- Diagnosis is made using hydrogen breath test or detecting ¯ stool pH.
- D-xylose test is normal, since the intestinal lining/architecture is intact.
- Treatment is with avoidance of lactose-containing products, or with lactase pills.
- HY for 2CK Peds but nonexistent on Step 1.
Milk protein allergy
- Biggest risk factor is not being exclusively breastfed for the first 6 months of life.

MEHLMANMEDICAL.COM 40
MEHLMANMEDICAL.COM

- Will present as blood in the stool in a child who they say is, e.g., 4 months old,
who was started on formula 3 weeks ago.
- Treatment is switching to hydrolyzed casein formula. Switching to soy-based
formula is wrong fucking answer. There is up to 50% crossover of allergy cases
with kids who have milk- and soy-protein allergy.
- Vignette can say kid was started on either a cow-milk or soy formula when
symptoms started. It doesn’t matter. Just choose hydrolyzed casein as answer.

Inflammatory bowel disease (IBD)


- Refers to both ulcerative colitis (UC) and Crohn.
- Both can present with bloody, mucoid stools.
- Both are associated with HLA-B27 à PAIR à Psoriasis, Ankylosing spondylitis, IBD, Reactive arthritis.
- For example, if a patient has psoriasis + bloody stools, you say, “The bloody stools are probably IBD.” Or
likewise, if patient with known IBD has lower back pain that’s worse in the morning, you say, “that’s
probably ankylosing spondylitis.”
- Both UC and Crohn can be associated with other autoimmune diseases unrelated to HLA-B27, like vitiligo.
- Both can cause anterior uveitis (non-specific finding seen in many autoimmune diseases).
- Both have ­ risk of colon cancer if the colon is involved. But if you’re forced to choose, UC has > risk than
Crohn because the colon is always involved in UC but not always in Crohn.
- Both are treated with 5-ASA NSAID compounds (mesalamine / sulfasalazine) or steroids. If they ask first
Tx, go with mesalamine or sulfasalazine, whichever they list (they won’t list both) over steroids. Q on 2CK
IM form 8 has “prednisone therapy” as answer for Crohn, but a 5-ASA isn’t listed.
- USMLE wants you to know anti-TNF-a agents (i.e., infliximab, adalimumab, etanercept) are used in IBD in
patients who fail initial Tx with 5-ASAs and steroids.
- If they ask you for which cytokine can be anti-inflammatory for IBD, the answer is IL-10. Sounds weird, but
just know IL-10 and TGF-b are mostly anti-inflammatory mediators. Don’t worry about Th1 vs Th2 nonsense.
- Rectum-ascending.
- Not transmural – i.e., only affects mucosa and submucosa. This means fistulae
are not seen. If they say fistulae to the skin overlying the anus or to any organ,
this means transmural involvement (Crohn).
- Colonoscopy will show pseudopolyps and crypt abscesses. You don’t need to
know what these look like. You just need to know they = UC.
- Barium enema shows “lead-pipe appearance” due to loss of haustra. This detail
is very important for UC.

Ulcerative colitis

Descending colon resembles a “lead pipe”

- Can cause primary sclerosing cholangitis, as discussed earlier. This can be


associated with pANCA antibodies.
- Pyoderma gangrenosum is a necrotic skin lesion seen rarely in patients with UC.
There is an NBME Q that describes this as an “ulcer with necrotic debris.”

MEHLMANMEDICAL.COM 41
MEHLMANMEDICAL.COM

- Can cause toxic megacolon, which presents as SIRS and sometimes low BP in a
patient with UC. They might say abdominal x-ray shows a 12-cm cecum (NR 3-8).
- If the patient has normal BP, NBME for 2CK wants steroids first for toxic
megacolon. If the patient is unstable, go straight to laparotomy.
- Colectomy is sometimes performed in patients with severe UC, but very rarely in
Crohn.
- “Mouth to anus” – i.e., can occur anywhere in GI tract. USMLE loves giving
mouth ulcers in Crohn. But terminal ileum is highest yield location.
- Transmural – i.e., can cause anal fistulae + to other organs.
- Colonoscopy shows skip lesions, where there is alternating diseased vs normal
bowel segments, with cobblestone ulcers.

Skip lesion (left part normal + right part inflamed and cobblestoned)
Crohn disease
- “Creeping fat” is buzzy term that can be seen on NBME, which refers to
intestinal fat migration that wraps around the bowel.

- Barium enema shows “string sign,” where inflamed segments are narrowed in
comparison to normal bowel.

MEHLMANMEDICAL.COM 42
MEHLMANMEDICAL.COM

- Biopsy shows non-caseating granulomas. Very HY for USMLE you know that
Crohn + sarcoidosis both have non-caseating granulomas.
- Sometimes associated with erythema nodosum. Not specific for Crohn in any
regard, but tends to have ­ association, whereas UC is pyoderma gangrenosum.
- Can cause anti-saccharomyces cerevisiae (yeast) antibodies. This is on a 2CK
NBME, where they say (-) for these Abs, but (+) for pANCA, where answer is UC.
- Intestinal malabsorption can occur, resulting in B12 deficiency most commonly
due to terminal ileum being classic inflammatory location.
- Impaired fat absorption can result in ­ calcium oxalate urolithiasis, as discussed
in the HY Renal PDF.

Irritable bowel syndrome (IBS)


- Don’t confuse with IBD. IBS is psychosomatic (i.e., psych-related) condition.
- Classic vignette will be a woman 20s-40s with stress factors who usually has alternating diarrhea /
constipation; can also present as bloating or cramping.
- Key detail is that symptoms are relieved with bowel motions.
- NBME assesses “smooth muscle hypersensitivity” as the answer for the mechanism for IBS.
- Treatment for diarrhea-predominant IBS is loperamide, which is an opioid that causes constipation. USMLE
will give vignette of IBS with diarrhea, and then the answer is just “mu-opioid receptor agonist.”

Other HY GI motility scenarios


- Diabetic gastroparesis = ¯ peristalsis due to neuropathy of the GI tract.
- Presents as GERD-like presentation in someone with Hx of advanced diabetes.
- USMLE wants endoscopy first to rule-out physical obstruction. If endoscopy is
negative, do gastric-emptying scintigraphy (aka gastric emptying scintigraphic assay) to
confirm delayed gastric emptying.
- Metoclopramide is used 1st for Tx. It is a prokinetic agent (i.e., ­ peristalsis) and anti-
Diabetes
emetic. It is a D2 antagonist but also an antagonist of serotonin 5HT3 and agonist of
5HT4 receptors. The effects on serotonin receptors ­ gut peristalsis.
- Erythromycin can also be used to agonize motilin receptors.
- “2-week trial of PPI” is wrong answer. This is the answer for “regular GERD.” If the
patient has GERD-like presentation with advanced diabetes, however, the Dx is
diabetic gastroparesis, not GERD.

MEHLMANMEDICAL.COM 43
MEHLMANMEDICAL.COM

- Neuropathy to the hypogastric nerves (sympathetic) causes severe diarrhea. This is


because the sympathetic nerves are “anti-peristalsis,” so if we knock them out, we get
too much peristalsis.
- Neuropathy to the pelvic splanchnic nerves (parasympathetic) causes severe
constipation. This is because the parasympathetic nerves are pro-peristalsis, so if we
knock them out we get unopposed hypogastric.
- In other words, diabetes + diarrhea à answer = hypogastric nerves are fucked up.
- Diabetes + constipation à answer = pelvic splanchnic nerves are fucked up.
- Hypothyroidism can cause constipation.
- Hyperthyroidism can cause diarrhea.
Thyroid
- USMLE will give vignette of a thyroid condition with GI disturbance, and then the
answer will just be “motility disorder.” This is same answer for diabetes.
- Iron and Aluminum can cause constipation (“Aluminimum amount of feces.”)
Supplements
- Magnesium can cause diarrhea.
- Verapamil causes constipation.
Drugs
- Macrolides, orlistat, and a-glucosidase inhibitors (e.g., acarbose) cause diarrhea.

Viral hepatitis
Two foundational points you need to know:

1) Hep A and E cause acute hepatitis only. Hep B, C, and D can cause chronic hepatitis.
2) HY point is that hepatocellular damage from hepatitis is due to T-cell-mediated apoptosis, not direct
viral cytopathicity. Same goes for general hepatic inflammation. Choose T cell response, not direct viral
effect.

- Hepatitis in general classically has ALT > AST, where #s can be in the hundreds to thousands, but I’ve seen
plenty of variability on NBME Qs, which is why I don’t consider this a foundational point.
- The answer for acute hepatitis in the United States most of the time. The Q might say the
patient had recent travel to Mexico.
- Fecal-oral; only causes acute hepatitis.
- RNA virus; Picornaviridae (shared with polio, coxsackie, and echoviruses). Step 1 is P/F now, but
Hep A that is some very mild-level stuff you could be aware of. If that makes you trip out, just ignore.
- IgM against HepA means acute infection.
- IgG against HepA means patient has cleared infection (because there is no chronic HepA).
- USMLE wants you to know HepA vaccine is indicated for IV drug users and MSM. There’s a 2CK
NBME Q where they just mention otherwise healthy MSM, and answer is Hep A vaccination.
- Mandatory stuff for USMLE is the serology (I discuss below).
- Most common hepatitis infection in the worldwide. USMLE likes China for hepatitis B. Just a
pattern I’ve noticed. Due to ­­ unvaccinated. In the USA, HepC is most common.
- Parenteral; can be acute or chronic.
- Transmitted vertically from mother to neonate, sex, via IV drugs, or blood exposure.
- Present in all body fluids, including breast milk.
- Virus is DNA, enveloped, circular (asked on USMLE, even though we have pass/fail exam). In
contrast, Herpesviridae are DNA, enveloped, linear. I’ve discussed in my YouTube clips how
memorizing viral structures is mostly a waste of time now that we have a P/F exam, but that this
Hep B particular distinction between HepB = circular, and Herpesviridae = linear, is assessed.
- HepB produces a DNA-dependent DNA polymerase.
- Serology very HY:

- (+) Surface antigen = patient currently has HepB.


- (+) Surface antibody = patient is immune to HepB.
- (+) Core antibody = patient has HepB now or did in the past.
- (+) Core antibody IgM = patient has acute infection.
- (+) Core antibody IgG = patient has chronic HepB, OR cleared HepB.
- (+) Surface antibody / (-) Core antibody = Immune à Vaccinated against HepB.

MEHLMANMEDICAL.COM 44
MEHLMANMEDICAL.COM

- (+) Surface antibody / (+) Core antibody = Immune à History of HepB / cleared it.
- (-) Surface antibody / (-) Core antibody = Not immune / Susceptible à need to vaccinate.
- (-) Surface antigen / (-) Surface antibody / (+) Core antibody IgM à window period (below).

- Vaccination against HepB is at birth, 2 months, and 6 months (no longer at 4 months).
- Only give HepB IVIG to neonate if mom is confirmed (+). A 2CK NBME Q gives mother’s status as
unknown when child is born à answer = “Give HepB vaccine now + only give IVIG if mother is
positive.”
- If patient has Hx of completed HepB vaccination but has titers that show susceptibility, the
answer is just “give more vaccine.” Sometimes people’s immunity wanes.

- Once a susceptible patient is exposed to HepB and the immune system attempts to clear it,
sometimes Surface antigen will decline to the point that it is no longer detectible. But at the same
time, the Surface antibody might not be high enough / at detectable levels yet. This is called the
“window period,” where both Surface antigen and antibody are negative, so it can appear as
though the patient doesn’t have an infection. However, Core antibody IgM will be (+). So the key
point is that 1) you know the double-negative Surface antibody/antigen combo is seen in the
window period, and 2) that Core antibody IgM is most reliable during the window period.

- HepB causes a “ground glass” hepatocyte appearance. You don’t have to obsess over the image.
Just memorize “ground glass” for HepB.

- USMLE really doesn’t give a fuck about HepB pharm (i.e., entecavir, tenofovir). Waste of time.
- You could be aware that interferon-a can be used for HepB.
- Hepatocellular damage is due to T cells / death is due to T-cell-mediated apoptosis, not direct
viral cytopathicity. I already mentioned this at top of chart, and this is applies to the other Heps
as well, but I reinforce this as what I’d still say is the highest yield point for HepC on USMLE.
- Parenteral; can be acute or chronic.
- Transmitted almost exclusively from IV drugs/blood exposure. Not present in breastmilk and
non-sanguineous body fluids (in contrast to HepB).
- In contrast, to HepB, HepC is not considered sexually transmitted. Large longitudinal study of
couples with one HepC(+) partner showed sexual transmission almost nil (possibly due to menses
exposure). If you’re forced to choose for FM / behavioral science Qs, however, still inform that
Hep C
abstinence or barrier contraception minimizes risk.
- RNA virus (Flaviviridae).
- No vaccine due to ­­ antigenic variation (i.e., >7 genotypes and 80 subtypes of HepC exist).
- IgM against HepC means acute infection.
- IgG against HepC means usually means chronic HepC.
- Histo shows “lobular necrosis,” in contrast to the ground-glass appearance of HepB. USMLE
doesn’t give a fuck about you knowing the image.
- Many drugs can be used to treat. USMLE doesn’t care. What you could be aware of is pegylated
interferon-a.
- Requires hepatitis B in order to infect, which can be due to co-infection (happening at the same
Hep D
time) or superinfection (occurs later in someone who already has HepB).

MEHLMANMEDICAL.COM 45
MEHLMANMEDICAL.COM

- If USMLE asks how to prevent HepD infection, answer = vaccination against hepatitis B. There is
no vaccine against HepD.
- Apparently HepB antigen forms the envelope for HepD (i.e., forms a circle around HepD).
- HepD is DNA virus; aka delta virus.
- Causes fulminant hepatitis / ­­ risk of death in pregnant women.
- Same as with HepA, only causes acute hepatitis.
Hep E
- Seen more in Asia, e.g., Tibet. But if USMLE says Mexico + pregnant woman + fast death from
hepatitis, you still have to use your head and know that’s HepE over HepA.
- Garbage diagnosis I don’t think I’ve ever seen on USMLE material. Including it here as important
negative. In theory:
Yellow - Yellow fever is in Flaviviridae family (same as Zika, Japanese encephalitis, Dengue, West Nile).
fever - Causes hepatitis and jaundice.
- Can cause “councilman bodies” on biopsy with “mid-zone necrosis.” These latter details date
back decades in USMLE resources as “Oh em gee know this.” Absolute nonsense. Waste of time.

Other HY hepatitis causes


- Answer on USMLE for liver condition in an overweight patient with normal labs.
- Non-alcoholic steatohepatitis.
- Underrated diagnosis in that students often have no idea about it but it shows up on
both Steps (higher yield on 2CK IM).
- Patient will have metabolic syndrome (i.e., ­ BMI + ­ lipids) and either mild
transaminitis or completely normal labs. I’d say ~50% of Qs will give ALT and AST a very
tiny bit elevated; the other ~50% of Qs will give you completely normal labs where you
say WTF.
NASH
- For example, they’ll give you 40-year-old patient with BMI of 35 who comes in for
routine health maintenance exam who has no complaints + lab studies are all completely
normal; then the answer is just “fatty liver” or “non-alcoholic steatohepatitis.” Student
says, “Wait, but the labs are completely normal though.” à Exactly. There’s nothing
wrong. But it’s assumed most overweight people have some degree of fatty liver.
- The condition starts out with completely normal labs, then progresses to very mild
transaminitis. Over time, LFTs can worsen. Rarely it can cause cirrhosis and carcinoma.
- Theoretical Tx is just hit the treadmill more and stop eating your dumb fast food.
- Classically AST > ALT.
- The ratio need not be 2:1. I’ve seen variations on NBME forms where there can be very
mild transaminitis (i.e., both AST and ALT are <100, with AST only slightly higher than
ALT). If the vignette is hyper-easy, they’ll say something like AST is 400 and ALT is 200.
Alcoholic
Both should normally be <50 U/L.
- Causes steatosis (fatty liver).
- USMLE wants you to know mallory hyaline is seen on biopsy. You don’t need to know
the image, just this factoid.
- Acetaminophen overdose most important for USMLE. N-acetylcysteine is antidote.
- Breakdown of acetaminophen produces metabolite called NAPQI that depletes reduced
glutathione. The latter normally mops up free radicals / prevents oxidation.
- Patient will OD on acetaminophen and be asymptomatic for 24-28 hours, then will
proceed to get ALT and AST in the tens of thousands + require liver transplant or death.
Acetaminophen - “N-acetylcysteine regenerates reduced glutathione.” Memorize that.
- Oxidized glutathione had disulfide bond (-S—S-); reduced glutathione has thiol (-SH).
- Activated charcoal should apparently be administered to patients who present very
acutely after massive ingestion, as this can ¯ acetaminophen absorption. But I’ve never
seen this on NBME. It tends to be more Qbank that’s gotchya-style this way. I’ve only
ever seen N-acetylcysteine assessed across NBMEs.
- I’d say some important drugs that can cause transaminitis are:
Other drugs
- Statins and fibrates, methotrexate, lithium, valproic acid, propylthiouracil.

MEHLMANMEDICAL.COM 46
MEHLMANMEDICAL.COM

- Mild transaminitis is normal and expected with these agents; the answer is you do not
need to decrease dose.
- For statins, myopathy is more common than toxic hepatitis (offline Step 1 NBME 23).

Obgyn-related hepatic issues


- HELLP syndrome = Hemolysis, Elevated Liver enzymes, Low Platelet count.
- Can occur as part of severe preclampsia (HTN + proteinuria after 20 weeks of
pregnancy).
- Vignette will be pregnant woman >20 weeks of pregnancy where you see the
platelets and Hb are low and LFTs are up. Not complicated.
HELLP syndrome
- Highest yield point for USMLE is you get schistocytes on a blood smear. USMLE
is obsessed with this.
- Causes of schistocytes on USMLE = HELLP syndrome, HUS, TTP, DIC, and
mechanical heart valves. If you’re about to trip out, don’t worry, I talk about
these conditions in the HY Heme PDF.
- Answer on USMLE for woman in 3rd trimester who gets itchy palms and soles
Intrahepatic cholestasis and ­ serum bile acids.
of pregnancy - Cause of 3rd trimester miscarriage.
- Treat with ursodeoxycholic acid (ursodiol).
- Pelvic inflammatory disease (chlamydia or gonorrhea) that has extended to
the liver capsule.
Fitz-Hugh-Curtis
- USMLE wants you to know this causes fibrin deposition on the liver due to the
Syndrome
inflammatory process. This is similar to post-surgical adhesions, where
inflammation leads to fibrin deposition.

Heavy metal liver diseases


- Autosomal recessive; HFE gene; chromosome 6.
- Iron overload.
- Mechanism USMLE wants is “increased intestinal iron absorption.”
- Body has poor ability to excrete iron; occurs naturally via menses in women;
otherwise there are minor losses via skin shedding.
- Main iron regulation is via shutting off intestinal absorption; this is impaired in
hemochromatosis.
- Usually presents in adulthood in males first (because of menses in women).
- Can present as “bronze diabetes” à hyperpigmentation due to hemosiderin
deposition in skin + ­ fasting sugars (iron deposition in tail of pancreas).
- Miscellaneous other findings can be seen like infertility (iron deposition in
hypothalamus, anterior pituitary, or gonads), cardiomyopathy, or arthritis
Hereditary (pseudogout).
hemochromatosis - Hereditary hemochromatosis, primary hyperparathyroidism, and hypothyroidism
are 3 most important causes of pseudogout. I used to only discuss the former two
with students over the years, but the latter shows up on a new 2CK NBME exam
where they mention chondrocalcinosis (calcium deposition in cartilage).
- USMLE wants you to know there is ­ risk of hepatocellular carcinoma. There is
easy NBME Q of hemochromatosis where they ask what patient is at increased risk
for, and the answer is simply “hepatocellular carcinoma.”
- Diagnose with ferritin >300 mg/dL. Transferrin saturation will also clearly be ­.
- It is exceedingly rare that ferritin is >300 in other conditions, however this can
occur in lymphoma and leukemia, where ­ ferritin is a poor prognostic marker in
non-Hodgkin lympoma. There is one NBME Q on 2CK where ferritin is 300 where
it’s not hemochromatosis, but USMLE won’t play gotchya.
- Treat with serial phlebotomy, not chelators.

MEHLMANMEDICAL.COM 47
MEHLMANMEDICAL.COM

- Chelators such as deferoxamine or deferasirox are for secondary


hemochromatosis due to transfusional siderosis (i.e., repeated blood transfusions
that contain iron, for e.g., b-thalassemia major).
- Autosomal recessive.
- Copper overload.
- Inability to secrete copper into bile from the liver. Copper is normally excreted by
the body via secretion into bile.
- ­ urinary copper + ¯ serum ceruloplasmin (copper-binding protein in the blood; in
the case of copper overload, body tries to minimize amount carried in blood).
- Buzzy / pass-level detail is Keiser-Fleischer rings, which is copper deposited in the
Wilson disease cornea of the eye. Vignette can give you what sounds like Wilson disease, and then
the answer is “slit-lamp exam.”
- Can cause ­ LFTs with cirrhosis, hemolytic anemia, and Parkinsonism.
- Copper deposits in basal ganglia, especially the putamen.
- Parkinsonism in a young patient = Wilson until proven otherwise.
- In old patient, Parkinsonism = Parkinson disease, normal pressure hydrocephalus,
Lewy-body dementia, or progressive supranuclear palsy.
- Treat with the copper chelator penicillamine.

Primary hepatocellular carcinoma (HCC) vs Metastases


Two main points:
1) Primary HCC is going to be one large lesion, sometimes with tiny satellite lesions. In contrast, metastases
to the liver will present as multiple lesions everywhere.
1) USMLE wants colon cancer as the most common type that goes to liver.
- USMLE loves gross images of these conditions.
- HCC will present as one large solitary lesion, sometimes with mini surrounding satellite lesions.

Notice how the HCC is one solitary lesion.

MEHLMANMEDICAL.COM 48
MEHLMANMEDICAL.COM

HCC can also grow as a “confluent multinodular” type.


But note how the cancer is still all relatively confined to one area.

MRI shows satellite nodule of HCC that grows next to the larger, initial growth.

In contrast, these are hepatic metastases. Notice how the lesions are everywhere.
USMLE will show image like this, and then the answer will just be “colorectal carcinoma.”

Another image of metastases. Not as obvious as the first image, but still you can see there are multiple
lesions not confined to one location.

Liver in relation to heart/kidney


- Right heart failure (either due to cor pulmonale or CHF) can result in venous
congestion within the liver. This can cause a characteristic mottled appearance on
Heart failure
gross specimen known as “nutmeg liver.”

MEHLMANMEDICAL.COM 49
MEHLMANMEDICAL.COM

- Just know this is an examination finding seen in tricuscpid regurg.


Pulsatile liver - As I talked about in the HY Cardio PDF, the highest yield cause of tricuspid
regurg on USMLE is pulmonary hypertension / cor pulmonale.
- Just know that in chronic liver disease, sometimes the kidneys can also fail.
- This is supposedly due to ¯ blood flow in patients with cirrhosis and ascites.
- Biopsy shows no abnormalities. That is probably the only notable point.
Hepatorenal syndrome
- Never seen this as correct answer on NBME. Only reason I’m including it is
because I see it as a distractor on some 2CK Qs in particular, so you should at
least know it exists and is likely always wrong.

Bowel ischemia
- Patient with cardiovascular disease (CVD) + blood in the stool.
- Caused by bleeding at ischemic ulcers at watershed areas in the colon
Ischemic colitis (i.e., splenic flexure and rectosigmoid junction).
- Can occur randomly or due to inciting event like recent AAA surgery.
- Diagnose with colonoscopy to visualize the ischemic ulcers.
- Patient with CVD + abdominal pain 1-2 hours after eating meals.
- Caused by atherosclerosis of the SMA or IMA à consuming food ­
oxygen demand of bowel à angina of bowel.
- The timing of the pain in relation to meals sounds like duodenal ulcers,
Chronic mesenteric ischemia
however instead of the vignette being a 29-year-old dude from Indonesia
with H. pylori, it will be a 69-year-old dude with Hx of coronary artery
bypass grafting, intermittent claudication, HTN, and diabetes.
- Next best step is “mesenteric angiography” on USMLE.
- Presents 3 different ways on USMLE:
1) Severe abdo pain in patient with AF à LA mural thrombus launches off
to SMA or IMA.
2) Severe abdo pain in patient just cardioverted/defibrillated à can launch
LA thrombus off to SMA or IMA. There’s a Q on 2CK form where this is the
case, where they don’t mention AF in the stem.
Acute mesenteric ischemia
3) Severe abdo pain in patient with Hx of chronic mesenteric ischemia (i.e.,
acute on chronic) à atheroma within SMA or IMA ruptures, effectively
causing an “MI of the bowel.”
- USMLE wants “mesenteric angiography” as next best step.
- Laparotomy is also answer on NBME form.
- I’ve never seen medications or endarterectomy as answers.

MEHLMANMEDICAL.COM 50
MEHLMANMEDICAL.COM

GI thromboses
- The splenic vein and superior mesenteric vein (SMV) merge to form the
portal. The inferior mesenteric vein (IMV) goes to the SMV.
- Cirrhosis à ­ portal venous pressure à ­ SMV + IMA pressure à stasis.
- In addition, any malignancy a patient might have à hypercoagulable state.
Mesenteric vein thrombosis - NBME Q gives patient with cirrhosis and lung cancer who has abdominal
pain + dark, mottled small bowel on surgical examination à answer =
“mesenteric venous thrombosis.” Students ask why. I say, “Well there’s ­
mesenteric venous pressure from the cirrhosis + hypercoagulable state from
the malignancy.”
- Risk factors are same as above.
- As I discussed earlier, USMLE wants you to know splenic vein thrombosis is
a cause of esophageal varices due to collateral development with the left
gastric vein (which drains the esophageal veins, so if ­ left gastric venous
Splenic vein thrombosis
pressure, then ­ esophageal venous pressure).
- USMLE wants you to know the short gastric veins feed into the splenic vein.
So if we have ­ splenic veinous or portal veinous pressure, then the short
gastric veins also have ­ pressure.
- Hepatic vein thrombosis. The hepatic vein drains the liver.
Budd-Chiari syndrome - Caused by polycythemia vera and pregnancy.
- Triad = abdominal pain, ascites, and hepatomegaly.
Portal vein thrombosis - Caused by portal venous stasis from cirrhosis (portal vein enters the liver).

Mandatory GI anatomy
Spinal
Structure HY points
level
- Supplies foregut (oral cavity until 1st part of duodenum).
- Its 3 branches are: splenic artery, common hepatic artery, left gastric artery.
- For whatever reason, the USMLE wants you to know the spleen is supplied by
Celiac trunk T12 an artery of the foregut but is not itself derived from the foregut (i.e., it is
derived from midgut).
- The short gastric arteries come off the splenic artery and supply the superior
greater curvature of the stomach.
- Supplies the midgut (2nd part of duodenum until 2/3 distal transverse colon).
- What the USMLE will do is give vignette of SMA thrombosis in peripheral
SMA L1 vascular disease, causing acute mesenteric ischemia, and then ask which
structure is involved à answer is anything that is midgut (e.g., jejunum,
ascending colon, etc.)
- Supplies the hindgut (distal 1/3 of transverse colon until the rectum).
- Same as for SMA, they’ll say there’s acute mesenteric ischemia of IMA and
then ask for involved structure (e.g., descending colon).
IMA L3 - USMLE will also ask cancer lymphatic/venous drainage, which follows the
same distribution as arterial supply. For instance, they’ll say there’s a cancer of
the descending colon, and the answer is “inferior mesenteric lymph nodes” or
“inferior mesenteric vein.” Not complicated.
Renal/gonadal
L2 - Gonadal arteries = testicular arteries in males and ovarian arteries in females.
arteries
- Bifurcation of abdominal aorta into the common iliacs occurs as L4.
- What USMLE will do is tell you a AAA repair is performed where a graft needs
Aortic to be inserted between the aortic bifurcation and the renal arteries. Then they’ll
L4
bifurcation ask blood supply to which structure will get compromised. So you need to say,
“Well the bifurcation is at L4, and the renal arteries are at L2, so anything that’s
IMA would get fucked up since IMA is L3.” à answer = “descending colon.”

MEHLMANMEDICAL.COM 51
MEHLMANMEDICAL.COM

Appendicitis
- Triad of RLQ pain, fever, and vomiting (Murphy’s triad).
- Pain starts at epigastrium (visceral pain) and migrates toward RLQ (parietal pain). USMLE will ask why
there is movement of the pain à answer = “inflammation of the parietal peritoneum.”
- Pain at McBurney’s point (1/3 of the way from the anterior superior iliac spine to the umbilicus).
- (+) Rovsing sign (pain felt in RLQ upon palpation of LLQ).
- USMLE wants you to know appendicitis pain can be RUQ in pregnancy due to displacement of bowel and a
shift in appendiceal location. What they will do is tell you pregnant woman in 3rd trimester has RUQ pain,
fever, and vomiting + negative abdo ultrasound (meaning not cholecystitis) à answer = “appendicitis.”
- “Inflammatory mass and fecalith in the cecum” is phrase used on 2CK NBME form to describe appendicitis.
- Diagnosis is done via ultrasound first, and sometimes CT. However, I have not seen USMLE assess either of
these, likely because their utility is debated. What I have seen is laparoscopic removal in the stable patient.
- USMLE will give ethics/consent Q, where they say patient has operation performed for ovarian procedure
but it is seen that she has an acutely inflamed appendix à answer = “remove due to necessity of medical
emergency.”
- However, if the scenario is reversed and, while performing an appendectomy, the surgeon notices a
suspicious lesion on, e.g., one of the ovaries, the answer is “do not biopsy.” Consent must first be obtained
for all non-emergencies.

HY colorectal / proctocolonic diagnoses


- Tortuous, superficial vessels in the colonic wall that cause painless bleeding per
rectum in elderly.
- Classically associated with aortic stenosis (Heyde syndrome; possibly related to
Angiodysplasia pressure backup to the colon).
- NBME Q gives older guy arguing with his wife + gets chest pain and bleeding per
rectum. Even though colonoscopy is used for diagnosis, they say in this Q that
colonoscopy is negative, likely to tell you it’s not cancer causing the bleeding.
- Mere presence of diverticula in the colon; >50% of people over age 60 in the US.
- ­ straining throughout life leads to herniation of mucosa + submucosa through
the muscularis propria of the colonic wall.
- Usually asymptomatic, but can bleed. Diverticular bleed is most common cause of
painless bleeding per rectum in elderly, followed by colorectal cancer, followed by
Diverticulosis
angiodysplasia.
- Can cause colovesical fistula (on new 2CK NBME), where a passageway between
the colon and bladder forms, leading to UTI and mixed flora in the urine.
- Do not confuse with diverticulitis, which is when a diverticulum becomes
inflamed.
- LLQ pain + fever in patient over 60. One of the highest yield diagnoses on USMLE.
- Inflamed diverticulum, usually in the sigmoid colon.
- CT of the abdomen with contrast is how to diagnose.
Diverticulitis - Do not scope acutely, as this can cause perforation.
- After patient is treated with antibiotics, a colonoscopy should be scheduled weeks
to months later to rule out malignancy. But once again, never scope acutely.
- Perforated diverticulitis can require colectomy.
- Patient over 75 + 2-3 days of constipation + abdo pain.
- Rotation around its mesentery causes “dilation of sigmoid colon” (answer on
NBME as what is most likely to be seen in patient).
Sigmoid volvulus
- Abdominal x-ray is used to diagnose, which shows a coffee bean sign, which is
one of the highest yield radiographic images on USMLE.

MEHLMANMEDICAL.COM 52
MEHLMANMEDICAL.COM

- Tx on NBME is “sigmoidoscopy-guided insertion of rectal tube.”


- Patient over 70 + chronic constipation + “hard stool palpated in the rectal vault.”
- Can sometimes cause fecal incontinence and paradoxical overflow diarrhea
Fecal impaction leading to encopresis (i.e., word for shitting yo pants).
- Idiopathic, but exacerbated by opioids.
- Treatment is with enema and laxatives.
- Bleeding per rectum that will be described as “blood on the toilet paper,” or
“blood that drips into the toilet bowl.”
- Hemorrhoids are technically normal vascular structures within the anal canal that
facilitate/cushion the passage of stool.
- Bleeding from internal hemorrhoids is painless. These are above the pectinate line
in the anal canal.
Hemorrhoids
- Bleeding from external hemorrhoids is painful. These are below the pectinate line.
- Pregnancy and cirrhosis are risk factors.
- 2CK NBME wants rectal exam followed by anoscopy for diagnosis.
- Hemorrhoids often self-resolve and can be managed conservatively with sitz bath,
NSAIDs, and ­ dietary fiber. However, if USMLE forces you to choose surgical
management for more severe cases, the answer is rubber band ligation.
- Young adult who has painful bowel movements +/- blood in the stool.
- The key detail is they refuse the rectal exam because the pain is so bad.
- For whatever reason, they can also say there’s an associated skin tag. I’ve seen
this more than once, where the student says, “What’s with the skin tag?” No
fucking idea.
Anal fissure
- If Step 1 forces you to choose a location for anal fissure, the answer is “posterior
in the midline.” Stupid, but it’s asked.
- Mechanism can be “increased anal sphincter tone.”
- Tx on 2CK Surg is sitz bath.
- If the USMLE forces you to choose a med, topical nitrates or diltiazem is used.
- Answer on USMLE for cystic mass at the superior aspect of the gluteal cleft.
Pilonidal cyst/abscess - Contains hair; often caused by ingrown hairs.
- Tx on 2CK Surg = incision and drainage.
- Answer on USMLE for painful, erythematous mass near anal verge.
Perianal abscess - Increased risk in Crohn and diabetes.
- Tx on 2CK Surg = incision and drainage.
- 2CK Surg Q gives cancer at anal verge + asks for next best step in management à
Anal malignancy answer = colonoscopy; excision is wrong answer. Presumably the scoping is done to
first investigate the extent of the disease, as that might alter management.

MEHLMANMEDICAL.COM 53
MEHLMANMEDICAL.COM

HY GI bacterial infections
Gram-negative rods
- Enterotoxigenic E. coli causes traveler’s diarrhea.
- Will present as brown/green diarrhea in person who’s gone to Mexico or Middle
ETEC East classically.
- Heat-labile toxin ADP ribosylates adenylyl cyclase à ­ cAMP.
- Heat-stable toxin ADP ribosylates guanylyl cyclase à ­ cGMP.
- Enterohemorrhagic E. coli causes bloody diarrhea 1-3 days after consumption of
beef.
EHEC
- Produces shiga-like toxin, which can cause hemolytic uremic syndrome (HUS;
triad of renal dysfunction, schistocytosis, and thrombocytopenia).
- Bloody diarrhea 1-3 days after consumption of beef.
- Also can cause HUS via shiga toxin.
- Requires very few organisms to cause infection.
- Main virulence is via its ability to invade, not the toxin itself.
Shigella
- Both shiga toxin of Shigella and shiga-like toxin of EHEC inhibit protein synthesis
by cleaving the eukaryotic 60S ribosomal subunit. Step 1 is P/F now, although
that’s known to be asked, so you can decide yourself whether you want to
memorize the nonsense.
- Food poisoning is caused by Salmonella typhimurium and Salmonella enteritidis.
The nomenclature has changed with time, but those names are acceptable.
- Bloody diarrhea 1-3 days after infection from consuming poultry, or following
exposure to eggs or reptiles (i.e., turtles, lizards, etc.). New NBME Q mentions
bloody diarrhea in someone with a pet lizard.
Salmonella
- Requires more organisms than Shigella to cause infection.
- Salmonella typhi causes typhoid fever, which will be rose spots on the abdomen
in a patient who’s “prostrated” (i.e., lying down in pain); can be either diarrhea or
constipation. The reservoir for S. typhi is humans, not chickens/reptiles.
- Salmonella can go latent in the gall bladder apparently.
- Vibrio cholerae (cholera) presents as “liters and liters” of rice-water stool in
someone who went traveling to, e.g., Mexico. The way you can differentiate this
from ETEC traveler’s diarrhea is that cholera is notably profusely high-volume.
- Acquired fecal-oral (i.e., fecal-contaminated food/water).
- Both ETEC and cholera vignettes can tell you the patient has 8-12 stools daily, so
it’s not the # of stools that matters; it’s the emphasis on volume. Cholera causes
death via severe dehydration and electrolyte disturbance.
- Vibrio toxin has same MOA as ETEC heat-labile toxin (i.e., ­ cAMP).
Vibrio
- Tx is oral rehydration on USMLE; if patient has low BP or altered mental status
(i.e., confusion/coma), IV hydration is done.
- Vibrio parahemolyticus doesn’t cause profuse, watery diarrhea the same way
cholera does. I’ve seen this organism asked once in an NBME vignette where the
patient ate sushi (can be acquired from sushi and shellfish).
- Vibrio vulnificus causes severe sepsis in half of patients. This is asked on offline
NBME 19 where a dude went running on a beach and got sepsis, with no mention
of consumption of food. But it’s apparently acquired from shellfish.
- Causes bloody diarrhea + either appendicitis-like (i.e., RLQ) pain or arthritis.
Yersinia enterocolitica - The RLQ pain is from mesenteric adenitis or terminal ileitis.
- Toxin has same MOA as ETEC heat-stabile toxin (i.e., ­ cGMP).
- Bloody diarrhea 1-3 days after consumption of poultry.
- Can cause Guillain-Barre syndrome (ascending paralysis + ¯ tendon reflexes +
Campylobacter jejuni
albuminocytologic dissociation in the CSF à Tx with IVIG + plasmapheresis).
- Grows best at high temperatures (42 degrees).
Gram-positive rods
- Watery diarrhea and/or vomiting in patient who’s consumed reheated or fried
Bacillus cereus
rice. The process of heating/re-heating causes germination of spores.

MEHLMANMEDICAL.COM 54
MEHLMANMEDICAL.COM

- Can also cause eye infections (weird, but on USMLE). They’ll say patient who’s
had eye surgery + now has infection caused by gram(+) rod à will be only
gram(+) rod listed.
- Watery/secretory diarrhea following consumption of poultry.
- Causes gas gangrene (CO2 gas) due to production of a-toxin/phospholipase;
Clostridium perfringens
presents as black skin / crepitus.
- Can also cause emphysematous cholecystitis (air in gall bladder wall).
- Diarrhea (pseudomembranous colitis) ~7-10 days after commencing oral
antibiotics.
- Antibiotics kill off normal bowel flora, allowing C. difficile to overgrow.
- C. difficile is not normal flora, however. It is acquired via consumption of spores.
- Can be watery or bloody diarrhea on NBME. Can also cause LLQ cramping (not
RLQ as with Yersinia).
- There is NBME Q where they say 28-year-old with LLQ cramping and bloody
diarrhea 7 days after starting oral antibiotics à answer = C. diff; wrong answer is
Clostridium difficile Yersinia.
- USMLE doesn’t care which antibiotics cause it; can be any.
- Diagnose with stool AB toxin test; stool culture is wrong answer.
- Treat with oral vancomycin.
- Vancomycin has poor oral bioavailability, so is usually given IV for things like
endocarditis and meningitis. But for C. diff infection, that’s a good thing because
we want it to stay within the GI tract.
- Other fancy Abx like fidaxomicin, rifaximin, etc., I’ve never seen on NBME. More
just masturbation around dumb factoids.
Gram-positive cocci
- S. aureus pre-formed heat-stable toxin is acquired from two main sources on
NBME: 1) various meats sitting under a heat lamp / out for long periods of time,
e.g., at a buffet; 2) dairy products like creams, custards, potato salad.
Staph aureus
- Notably causes vomiting 1-6 hours after consumption. This is notable, as the
symptoms occur rather quickly. Diarrhea, both watery and bloody, can occur, but
is not mandatory. The main crux is the vomiting.
Other GI bacteria
- Causes Whipple disease, a GI malabsorptive syndrome where the patient can
also get arthritis and renal and cardiac disease.
- 100% of Qs will say “PAS-positive macrophages in the lamina propria.” You don’t
Tropheryma whipplei need to know the image for this. But for whatever magical reason, the USMLE
gives a fuck about this detail. Old-school pathology docs who reminisce maybe.
- New highly pedantic Q on 2CK NBME 10 wants “ceftriaxone + daily TMP/SMX for
one year” as the treatment. Organism isn’t even HY. No idea why they care.
- Causes flattening of villi similar to Celiac, but considered infective/bacterial in
origin. Literature says exact etiology not certain.
Tropical sprue - Answer on USMLE for patient living in tropical area with unknown malabsorptive
disease, where you can easily eliminate the other answers.
- Tx = tetracycline.

HY GI viral infections
- Most common cause of watery diarrhea in unvaccinated children < 5 years.
- Vaccine normally given orally at 2, 4, and 6 months of age.
Rotavirus
- Double-stranded, segmented RNA (NBME asks it).
- Wheel-shaped (also on NBME).
- Most common cause of watery diarrhea in adults and rotavirus-vaccinated children.
Norwalk virus - Cruise ships and business conferences are buzzy places to acquire (fecal-oral); basically
any place with high density of people.
Herpes - HSV1/2 causes herpes esophagitis à odynophagia + punched-out ulcers in esophagus.
CMV - CMV esophagitis àodynophagia + linear (confluent) ulcers in esophagus.

MEHLMANMEDICAL.COM 55
MEHLMANMEDICAL.COM

- CMV colitis à bleeding per rectum in AIDS patient with CD4 count <50.
- USMLE likes “intranuclear inclusions” or “intranuclear inclusion bodies” for CMV. This
refers to the “owl eyes” that can be seen on histo.

HY GI protozoal infections
- A protozoan is a unicellular eukaryote.
- ECG à Entamoeba histolytica, Cryptosporidium parvum, and Giardia lamblia are all GI protozoa that are
acquired via cysts in water (i.e., they are water-borne). If “water-borne” and “fecal-oral” are both listed as
answers, USMLE wants “water-borne” as means of acquisition.
- Bloody diarrhea in person who went to Mexico.
- Can cause “flask-shaped ulcers” in the small bowel and liver abscess.
Entamoeba histolytica
- Demonstrates “erythrophagocytosis,” where RBCs can be seen within it on LM.

MEHLMANMEDICAL.COM 56
MEHLMANMEDICAL.COM

- Treat with metronidazole.


- Iodoquinol kills intraluminal parasite.
- Watery diarrhea in person who went to Mexico.
- Appears as acid-fast cysts (same stain as TB).

Cryptosporidium parvum

- Self-limiting in immunocompetent persons à Tx = supportive care.


- Chronic diarrhea in HIV à Tx = nitazoxanide.
- Steatorrhea in person who went to Mexico.
- Steatorrhea = bloating + extremely foul-smelling stool that floats.
- The steatorrhea is due to Giardia causing malabsorptive diarrhea.
- There is one NBME Q for Giardia where they say “foul-smelling watery
diarrhea with bloating,” which is audacious, since 9 times out of 10 it is not
described as watery. But the “bloating” and “foul-smelling” are consistent with
steatorrhea in this context. Just letting you know it exists.
Giardia lamblia - Acquired via fresh water lakes / scuba diving. There is a Giardia NBME Q where
they say a woman goes scuba diving in Mexico and then gets foul-smelling
stools with bloating. Student says, “Wait but I thought Giardia was fresh water.
If she went scuba diving in the ocean then how is that Giardia?” à People can
go scuba diving in fresh water lakes too bro. Don’t know what to tell you.
- Flagellated protozoan. USMLE wants you to know the images for both the cyst
as well as the flagellated trophozoite.

MEHLMANMEDICAL.COM 57
MEHLMANMEDICAL.COM

Cyst form of Giardia. Not dramatic.

Trophozoite form of Giardia.

- Tx = metronidazole.

HY GI parasitic (helminth) infections


- USMLE wants you to know routes of acquisition for the nematodes (i.e., ingested or through feet, etc.).
- You do not need to obsess over exactly which anti-helminthic agent treats which organism. Waste of time.
What USMLE wants you to know is the big-picture concept that -bendazoles (i.e., mebendazole) and
pyrantel pamoate are the two agents that treat nematodes primarily. They inhibit microtubules (tubulin).
- Praziquantel is used for cestodes and trematodes. Albendazole is the odd one out that is used for
neurocysticercosis (Taenia solium; cestode).
- USMLE doesn’t give a fuck about diethylcarbamazine.
Nematodes (roundworms)
Ascaris lumbricoides - Ingested. Giant roundworm; causes intestinal obstruction. (Ascariasis)
Enterobius vermicularis - Ingested. Causes perianal itching / (+) tape test in children. (Enterobiasis)
Angiostrongylus - Ingested by eating raw slugs/snails. Causes eosinophilic meningitis.
- Ingested by eating pork/bear meat. Causes triad of fever, periorbital edema,
Trichinella spiralis
and myalgias. (Trichinosis)
- Acquired through soil/one’s feet. Causes intestinal obstruction + pulmonary
Strongyloides stercoralis symptoms (apparently travels to lungs via blood, then travels up respiratory
tree into esophagus, where it is swallowed).
- Acquired through soil/one’s feet. ”Hookworms” refers to Ancylostoma
Hookworms duodenale and Necator americanus.
- Cause microcytic anemia / ¯ hemoglobin due to sucking blood in GI tract.
Cestodes (tapeworms)
Diphyllobothrium latum - Fish tapeworm. Nothing else you need to know.
- Pork tapeworm. Causes cysticercosis (muscle pain/cysts) + neurocysticercosis
Taenia solium
(“Swiss cheese” appearance of brain or soap bubbles in ventricles).

MEHLMANMEDICAL.COM 58
MEHLMANMEDICAL.COM

- Aka hydatid worm, or dog tapeworm; causes hydatid cyst disease.


- Acquired from dogs (clearly).
Echinococcus granulosus
- Causes liver cysts. Only point you need to know is that you do not biopsy
because this can cause anaphylaxis. Tx = surgically remove.
Trematodes (flukes)
- Penetrates skin when swimming; snail is host.
- Schistosoma mansoni/japonicum can cause portal hypertension. Not HY but
Schistosoma
shows up once on NBME. Not to be confused with Schistoma hematobium,
which causes squamous cell carcinoma of the bladder.
- Ingested from crab meat or crayfish.
Paragonimus westermani
- Aka lung fluke. Most common cause of hemoptysis in the world.
Clonorchis sinensis - Ingested from fish. Causes cholangiocarcinoma.

HY GI pharm
- Omeprazole.
- Shut of proton pumps on parietal cells à ¯ acid secretion.
- Irreversible and non-competitive; more efficacious than H2 blockers, which are
PPIs reversible and competitive.
- Choose PPIs over H2 blockers for Tx of most things, like GERD, ulcers, and H. pylori.
- Diagnosis of GERD is 2-week trial of PPIs.
- PPIs cannot be given with sucralfate or -azole antifungals (on NBME).
- Cimetidine, ranitidine.
- Not used often. PPIs more efficacious.
- One 2CK Surg form has “2-week trial of H2 blocker” as correct for diagnosis of
GERD, but PPIs aren’t listed. As I said above though, if you’re forced to choose
H2-blockers between a PPI and H2 blocker, the PPI is correct basically always.
- Cimetidine can cause gynecomastia (HY on USMLE) and inhibits P-450.
- Ranitidine doesn’t inhibit P-450. Sounds pedantic, but there’s an NBME Q where
they mention coma in someone taking diazepam + 2nd drug à answer = cimetidine as
2nd drug; ranitidine also listed but wrong (doesn’t inhibit P-450).
- As discussed earlier, anti-emetic + prokinetic agent (means ­ peristalsis).
- D2 antagonist but also an antagonist of serotonin 5HT3 and agonist of 5HT4
receptors. The effects on serotonin receptors ­ gut peristalsis.
- HY use on USMLE is diabetic gastroparesis (i.e., sounds like GERD but patient has
severe diabetes, where answer is metoclopramide, not PPI).
Metoclopramide - Because it’s a D2-antagonist, adverse effects are similar to the anti-psychotics – i.e.,
prolong QT interval, hyperprolactinemia, anti-pyramidal effects (acute dystonia,
parkinsonism, akathisia, tardive dyskinesia).
- 2CK Psych Qs like metoclopramide as cause of parkinsonism, where the answer is
“discontinue metoclopramide” as next best step before simply giving the patient
Parkinson disease meds.
- Powerful anti-emetic classically used for nausea/vomiting from chemoradiotherapy.
Ondansetron
- 5HT3 antagonist.
- Motilin receptor agonist that can be used for diabetic gastroparesis.
Erythromycin
- Also a 50S ribosomal subunit inhibitor (antibiotic).
- PGE1 analogue. Used for NSAID-induced ulcers after PPIs.
- NSAIDs ¯ prostaglandins. Therefore misoprostol is the replenished prostaglandin.
Misoprostol
- Prostaglandins ¯ acid production, ­ mucous and bicarb production, and ­ gastric
mucosal blood flow.
Magnesium - Antacid. Causes diarrhea.
Aluminum. - Antacid. Causes constipation (“Aluminimum amount of feces.”)
Calcium - Antacid. Causes milk-alkali syndrome (­ Ca2+ + ­ HCO3-+ renal dysfunction).

MEHLMANMEDICAL.COM 59
MEHLMANMEDICAL.COM

- Bisphosphonates (e.g., alendronate) and tetracyclines (e.g., doxycycline) cannot be


taken with calcium or iron supplements (i.e., divalent cations), since the latter
chelate the drugs and ¯ absorption / oral bioavailability. USMLE loves this.
- USMLE will give you, e.g., prostatitis treated with ciprofloxacin, + list all sorts of
other meds the patient is on, as well as a calcium supplement, and then tell you the
patient’s condition isn’t improving + ask why à answer = calcium carbonate.
Bismuth - Can be used as part of H. pylori regimens (but not first-line).
- Can be used to coat ulcers / form a barrier to protect against acid and ¯ pain.
Sucralfate
- Cannot take with PPIs. The latter ¯ ability of sucralfate to crosslink/work correctly.
5-ASA compounds - Mesalamine, sulfasalazine; NSAIDs used for IBD, as discussed earlier.
- Infliximab, adalimumab, and etanercept.
- USMLE likes these for IBD after 5-ASA compounds and steroids are attempted.
TNF-a blockers
- Infliximab and adalimumab are monoclonal antibodies against TNF-a.
- Etanercept is a recombinant receptor that mops up soluble TNF-a.
- Somatostatin analogue used for esophageal varices Tx after banding.
Octreotide
- Acts by ¯ portal blood flow.
Propranolol - Beta-blocker that can be used for esophageal varices prophylaxis (not Tx).
- Used to Tx hyperammonemia in cirrhosis.
- Carbohydrate that is metabolized by gut bacteria into acidic substrates that trap
Lactulose
ammonia as ammonium. The latter is then excreted (ions aren’t absorbed as easily).
- The answer on USMLE is “acidification, NH4+” for what lactulose causes in the gut.
- Used to Tx hyperammonemia in cirrhosis.
Neomycin
- Antibiotic that kills ammonia-producing bacteria.
- Mu-opioid receptor agonist used to treat diarrhea-predominant IBS.
Loperamide
- Opioids cause constipation, so “this is a good thing” in this case.
Ezetimibe - Blocks cholesterol absorption in the small bowel.
- Bile acid sequestrant. Causes reduced enterohepatic circulation of bile acids at
Cholestyramine terminal ileum à liver must now convert more cholesterol into bile acids in order to
replenish them à liver pulls cholesterol out of the blood to accomplish this.
Orlistat - Pancreatic lipase inhibitor used for obesity; can cause steatorrhea.
- Mouth wash used for oropharyngeal candidiasis.
Nystatin
- Forms pores in fungal ergosterol membrane.
- Used for candidal esophagitis.
Fluconazole - Inhibits ergosterol synthesis (i.e., lanosterol à ergosterol) by inhibiting 14a-
demethylase. USMLE can write this as “P-450-mediated demethylation reaction.”
- HSV esophagitis; HSV1/2 causes “punched out” ulcers.
Acyclovir - DNA polymerase inhibitor; causes chain termination; resistance = altered viral
thymidine kinase.
Ganciclovir - CMV esophagitis and colitis; causes linear/confluent ulcers. Same MOA as acyclovir.
- Clarithromycin, Amoxicillin, PPI.
- First-line Tx for H. pylori.
CAP
- If patient still has (+) urease breath test after 4 weeks, assume antibiotics resistance
and switch out the CA and add tetracycline, metronidazole, and bismuth (keep PPI).
Vancomycin - Given orally to treat C. difficile.
- “Anaerobes below the diaphragm.”
Metronidazole - Used for diverticulitis in combo with a fluoroquinolone.
- Tx for Giardia + Entamoeba.
Imipenem - Carbapenem antibiotic that has fantastic penetration for pancreatitis.
- Used for most nematodes (i.e., Ascariasis, Enterobiasis, hookworms, etc.).
Mebendazole
- Inhibits microtubules. An NBME Q has “tubulin” as the answer for protein it inhibits.
- Same as mebendazole, but for whatever reason it’s known as a preferred agent for
Albendazole
neurocysticercosis, which is caused by a cestode (Taenia solium).
- Agent used against nematodes. Shows up on an offline NBME as answer.
Pyrantel pamoate
- Causes paralysis.
Praziquantel - Used for cestodes and trematodes.

MEHLMANMEDICAL.COM 60
MEHLMANMEDICAL.COM

- Causes paralysis.

- 44M + fasting glucose of 112 mg/dL + dark skin on forearms + arthritis; Dx? à hereditary

hemochromatosis à AR, chromosome 6, HFE gene, C282Y or H63D missense mutations account for

90% à “Bronze diabetes” à hyperpigmentation (from hemosiderin deposition) + diabetes due to

iron deposition in tail of pancreas (normal fasting glucose is 72-99 mg/dL; impaired fasting glucose

[pre-diabetic] is 100-125 mg/dL; diabetic is two fasting glucoses 126 or greater, or a single HbA1c

>6.5%, or any random glucose >200 mg/dL) + third finding such as arthritis, cardiomyopathy, or

infertility.

- 44M + fasting glucose of 130 mg/dL + hands are sore + x-ray of hands shows DIP involvement; what’s

the Dx for the type of arthritis? à answer = pseudogout, not osteoarthritis. Student says wtf? The

two most common etiologies for pseudogout are hemochromatosis and primary hyperparathyroidism

(pseudogout is calcium pyrophosphate deposition disease, and will present as either a monoarthritis

of a large joint such as the knee, or as an osteoarthritis-like presentation of the hands.

- Tx of hereditary hemochromatosis à serial phlebotomy, not chelation therapy.

- 44M patient above + USMLE asks what’s the mechanism for his disease à answer = “increased

intestinal absorption of iron.”

- 44M above + next best step in Dx? à check serum ferritin (>300 ug/L in men + post-menopausal

women; or >200 in premenopausal women; USMLE will always say >300 so don’t worry).

- Why do men get hemochromatosis younger + with worse Sx than women? à menstruation slows

progression of disease.

- What is secondary hemochromatosis? à aka transfusional siderosis (amazing to remember if you

want to sound sophisticated) à due to chronic blood transfusions à each transfusion of RBCs

contains iron à seen classically in beta-thalassemia major or any other patients receiving ongoing

transfusions.

- Tx for secondary hemochromatosis (transfusional siderosis) à chelation therapy (e.g., deferoxamine),

not serial phlebotomy.

- Three main conditions causing hepatocellular carcinoma? à HepB, HepC, hemochromatosis.

MEHLMANMEDICAL.COM 61
MEHLMANMEDICAL.COM

- Tumor marker of HCC? à AFP (same as yolk sac tumor, aka endodermal sinus tumor).

- Parkinsonism in young patient; Dx? à Wilson disease (AR, chromosome 13)

- Parkinsonism in older patient; Dx? à Parkinson disease

- Parkinsonism + axial dystonia; Dx? à progressive supranuclear palsy (this is on the USMLE!!)

- Parkinsonism + visual hallucinations + cognitive decline; Dx? à Lewy body dementia

- Parkinsonism + urinary incontinence + gait instability + cognitive dysfunction; Dx? à normal pressure

hydrocephalus (NPH) = “wet, wobbly, wacky” + Parkinsonism.

- 23F + unilateral resting tremor + increased LFTs + hemolytic anemia; Dx? à Wilson disease

- 23F + unilateral resting tremor + increased LFTs + hemolytic anemia; next best step? à answer = do a

slit-lamp exam.

- How is most copper normally excreted by the body? à through bile (hepatocyte transport pump).

- Important serum/blood findings in Wilson? à decreased ceruloplasmin; increased urinary copper.

- How do we normally excrete iron? à humans have poor elimination mechanism; losses are natural

through skin (or menstruation in women).

- Where do we absorb iron and copper? à duodenum mainly.

- Where do we absorb bile acids and B12? à terminal ileum.

- Tx for Wilson disease? à penicillamine (copper chelator).

- “Opposite” of Wilson disease? à Menkes disease (can’t absorb intestinal copper).

- What vitamin helps absorb iron? à vitamin C ferrireductase converts small bowel Fe3+ to Fe2+; only

Fe2+ can be absorbed out of the lumen.

- What does cholecystokinin (CCK) do? à increases contraction of gall bladder, relaxes sphincter of

Oddi, and increases exocrine pancreas secretion of lipases, proteases, and amylase.

- Which cells make CCK à answer = “enteroendocrine cells of the small intestine” à HY.

- Macronutrients entering the duodenum (i.e., fats, proteins, carbs); what hormone is notably secreted

in response? à USMLE wants CCK for this one.

- Acid entering the duodenum; what hormone is notably secreted in response? à USMLE wants

secretin à causes bicarb secretion from pancreas.

- What do gastric chief cells do? à secrete pepsinogen (inactive zymogen) à acid activates to pepsin.

- What do parietal cells secrete à acid + intrinsic factor.

MEHLMANMEDICAL.COM 62
MEHLMANMEDICAL.COM

- 22M + vitiligo + macrocytic anemia; Dx? à pernicious anemia causing B12 deficiency à

“autoimmune diseases go together.”

- Mechanism for pernicious anemia? à autoantibodies against parietal cells or intrinsic factor.

- Blood smear in pernicious anemia? à hypersegmented neutrophils + high MCV RBCs.

- Pt has B12 deficiency + atrophic gastritis; what is most likely to be increased in this pt? à answer =

gastrin à need to assume pernicious anemia à atrophy of parietal cells due to Abs à decreased

acid production à increased gastrin à USMLE likes this question!

- Which nerve must be severed to remove cancer at gastroesophageal junction à answer = vagus (just

memorize it; it’s on a retired NBME).

- HY structures passing through diaphragm? à “I Ate 10 Eggs At 12.” à IVC T8; T10 Esophagus +

thoracic duct; Aortic hiatus (aorta, azygous vein, thoracic duct) at T12.

- Drug-/pill-induced esophagitis à USMLE wants bisphosphonates (i.e., alendronate) + tetracyclines +

clindamycin as causes.

- What part of the brain is damaged in Wilson? à USMLE wants putamen (they will show you a

transverse head CT and expect you to pick out the letter labeling the putamen).

- Most active part of the bowel in terms of cell division? à answer = “base of the crypt.” Memorize it.

“Top of the villi” is the wrong answer.

- Where are Brunner glands à duodenum, proximal to the sphincter of Oddi.

- What do Brunner glands secrete? à alkaline mucous.

- What do Paneth cells secrete? à Defensins in the small intestine.

- Person who’s vomiting; what’s the biochemical disturbance? à hypokalemic hypochloremic

metabolic alkalosis à low K, low Cl, high pH, high bicarb, low H, anion gap normal (even though it’s

alkalosis, not acidosis, the USMLE will still ask an arrow for the anion gap here).

- 2-week-old male + forceful non-bilious vomiting; Dx? à hypertrophic pyloric stenosis.

- Dx of pyloric stenosis? à abdominal ultrasound to show olive-shaped hypertrophied pylorus.

- Tx of pyloric stenosis? à myomectomy

- Who gets pyloric stenosis? à first-born males (weird, but it’s on an old NBME) + neonates taking oral

erythromycin for chlamydial ophthalmia neonatorum (erythromycin is a motilin-receptor agonist).

MEHLMANMEDICAL.COM 63
MEHLMANMEDICAL.COM

- 2-week-old male + bilious vomiting; Dx? à duodenal atresia, annular pancreas, congenital midgut

volvulus, or Hirschsprung (correct, Hirschsprung can present with bilious vomiting).

- 2-week-old + Down syndrome + bilious vomiting + passed meconium ok; Dx? à duodenal atresia

- 2-week-old + Down syndrome + bilious vomiting + slow to pass meconium; Dx? à Hirschsprung

- How do you Dx duodenal atresia? à abdominal x-ray (AXR) showing double-bubble sign (very HY).

- How do you Dx Hirschsprung? à rectal manometry, followed by confirmatory rectal biopsy showing

absence of ganglion cells.

- Mechanism for Hirschsprung? à failure of migration of neural crest cells distally to the rectum.

- Failure to pass meconium at birth. Most likely cause overall? à cystic fibrosis.

- 18-month-old + intermittent abdominal pain + crying + blood in stool; Dx? à intussusception.

- 18-month-old + intermittent squatting + crying + FOBT positive; Dx? à intussusception.

- 18-month-old + occasionally brings legs to chest + vomits + FOBT positive; Dx? à intussusception.

- 18-month-old + occasionally brings legs to chest + vomits + FOBT negative; Dx? à volvulus à this is

congenital midgut volvulus.

- 18-month-old + occasionally brings legs to chest + vomits + FOBT negative

- Presentation sounds like intussusception but no blood per rectum à answer = congenital midgut

volvulus.

- Cause of intussusception? à >99% are in kids under age 2; caused by lymphoid hyperplasia due to

viral infection (e.g., rotavirus) or recent vaccination; if in adult (usually elderly), it is caused by

colorectal cancer.

- Dx and Tx of intussusception? à USMLE wants enema as the answer. Even though ultrasound can be

done which shows a target sign, the USMLE always wants enema. And it can be any type. I’ve seen

“air contrast enema”, “air enema,” “contrast enema” all as answers. I also had a student simply get

“water-soluble contrast enema” on the exam, which means gastrografin. Barium would refer to

regular contrast.

- For contrast swallows, when to do barium vs water-soluble (gastrografin)? à barium most of the

time; if at risk of aspiration, must do barium because aspiration of gastrografin will cause

pneumonitis. If patient has suspected esophageal perforation, do not do barium, as that will cause

mediastinitis; must do gastrografin in this case.

MEHLMANMEDICAL.COM 64
MEHLMANMEDICAL.COM

- Level of celiac trunk and main branches + what’s it supply? à T12; splenic artery, common hepatic

artery, left gastric artery; supplies foregut (mouth to duodenum at ampulla of Vater).

- Level of SMA + what’s it supply? à L1; supplies midgut (duodenum at ampulla of Vater until 2/3 distal

transverse colon); so for instance, the right colic and middle colic arteries come off SMA.

- Level of IMA + what’s it supply? à L3; supplies hindgut (2/3 distal transverse colon until the pectinate

line 2/3 distal on the anal canal); left colic artery comes off IMA.

- Renal + gonadal (testicular in men; ovarian in women) arteries come off of L2 most often.

- Abdominal aorta bifurcates into common iliacs at L4.

- Abdominal aortic aneurysm occurs in males over 55 who are ever-smokers à a one-off abdo

ultrasound is done in this patient group.

- Most common locations for atherosclerosis (in descending order) à abdominal aorta, coronary

arteries, popliteal arteries, carotid arteries.

- USMLE favorite question à “Which of the following is supplied by an artery of the foregut but is not

itself derived from the foregut?” à answer = spleen.

- Short gastric arteries come off which artery? à splenic.

- What’s the main arterial supply to the pancreas? à Arteria pancreatica magna (greater pancreatic

artery) à a branch of the splenic.

- 79M + Hx of atrial fibrillation + severe, acute, diffuse abdo pain; Dx? à acute mesenteric ischemia

caused by mural thrombus embolizing to SMA or IMA.

- Above 79M; next best step in Mx? à mesenteric arteriography.

- Above 79M; Tx? à antibiotics (for necrotic bowel) then laparotomy (to remove necrotic bowel) à

they will tell you in last line of vignette that IV Abx are administered and then ask for the next step,

which is just laparotomy. It should be noted that the literature mentions various Txs like

embolectomy, but the USMLE wants resection of nonviable bowel as the answer.

- 52F + short episode of ventricular fibrillation + defibrillated + now has severe abdo pain; Dx? à acute

mesenteric ischemia due to ischemia caused by VF, not an embolus à antibiotics; CT if stable; if

unstable go straight to laparotomy.

MEHLMANMEDICAL.COM 65
MEHLMANMEDICAL.COM

- 55F diabetic + Hx of intermittent claudication + Hx of abdo pain 1-2 hours after eating meals; Dx? à

chronic mesenteric ischemia (CMI) caused by severe atherosclerosis of SMA or IMA (essentially

angina of the bowel).

- 55F diabetic + Hx of CABG + Hx of abdo pain 1-2 hours after eating meals; next best step in Dx? à

mesenteric arteriography (CMI).

- 55F diabetic + Hx renal artery stenosis + Hx of abdo pain 1-2 hours after eating meals; Tx? à

angioplasty + stenting (CMI) to restore blood flow.

- Patient with CMI who has a 2-day Hx of severe abdo pain + fever; Dx? à acute mesenteric ischemia

(acute on chronic due to a thrombosis; essentially akin to an “MI” of the bowel) à do mesenteric

arteriography to Dx; Tx with Abx + laparotomy to remove necrotic bowel.

- What is pectinate line? à separates upper 2/3 of the anal canal (part of hindgut; endoderm-derived)

from the lower 1/3 of anal canal (aka proctodeum, which is ectodermal).

- Lymphatic drainage above/below pectinate line? à above: internal iliac; below: superficial inguinal.

- Arterial supply above/below pectinate line? à above: superior rectal artery; below: middle/inferior

rectal arteries.

- Venous drainage above/below pectinate line? à above: superior rectal vein; below: middle/inferior

rectal veins.

- How does pectinate line relate to hemorrhoids? à above: internal hemorrhoids (painless); below:

external hemorrhoids (painful).

- Tx for hemorrhoids? à conservative first, i.e., fiber + exercise; if they want intervention, banding

(rubber band ligation) is the answer.

- How do you Dx congenital midgut volvulus? à upper-GI series (AXR + contrast follow-through of

esophagus, stomach, and duodenum with barium or gastrografin).

- What does upper vs lower GI mean? à upper is above ligament of Treitz (suspensory ligament of

duodenum; separates duodenum from jejunum; this ligament connects end of duodenum to the

diaphragm) à below this ligament (jejunum and lower) is lower GI.

- Any significance to upper vs lower GI meaning? à upper-GI bleeds tend to cause melena (black, tar-

like stools caused by blood exposed to acid); lower-GI bleeds tend to cause hematochezia (frank

blood); large upper-GI bleeds can sometimes cause hematochezia.

MEHLMANMEDICAL.COM 66
MEHLMANMEDICAL.COM

- What are the mucous-producing cells in the stomach? à foveolar cells (aka surface mucous cells)

secrete alkaline mucous; these are distinct from mucous neck cells, which secrete an acidic fluid

containing mucin proteins.

- What are the mucous-producing cells in the intestines? à goblet cells

- “Wait, so there aren’t goblet cells in the stomach?” à correct.

- Barrett esophagus; what are the changes in mucosa (from what to what)? à metaplasia of non-

keratinized stratified squamous à intestinal simple columnar epithelium (intestinal means “has

goblet cells”).

- What does Barrett look like on endoscopy? à bright red mucosa (UWorld has a Q where they show

the endoscopy).

- Barrett will go to what kind of cancer? à adenocarcinoma of distal third of esophagus

- Biggest risk factor for Barrett? à GERD (often in obese patients due to lower LES tone)

- Tx of Barrett? à PPIs are standard to decrease GERD; they’re more efficacious than H2-blockers.

- Acute gastritis (inflammation of lining) main causes? à NSAIDs, glucocorticoids, H. pylori.

- Leading cause of chronic gastritis? à H. pylori.

- What is atrophic gastritis? à a type of chronic gastritis in which ongoing inflammation of glandular

cells causes them to be replaced with fibrotic tissue.

- What is type A vs B atrophic gastritis? à Type A = non-antral (mainly fundus); caused by autoimmune

attack against parietal cells (pernicious anemia), resulting in B12 deficiency due to insufficient intrinsic

factor production. Type B = antral; caused by H. pylori.

- How do you Dx H. pylori? à urea breath test or stool antigen test.

- How do you Tx H. pylori? à CAP = Clarithromycin, Amoxicillin, Proton pump inhibitor. If patient has

positive urea breath test four weeks after Tx, assume resistance of Abx, so switch out the

clarithromycin and amoxicillin and give metronidazole + tetracycline + bismuth instead (with the PPI).

- Urease, oxidase, catalase; H. pylori is positive for which ones? à all three (asked in a USMLE Q, where

they had different + and – combos, and the answer was all three +).

- What is peptic ulcer? à gastric ulcer and/or duodenal ulcer.

- Most common cause of gastric ulcers à H. pylori, then NSAIDs, then smoking.

- What about alcohol? à EtOH doesn’t cause ulcers; it just prevents their healing.

MEHLMANMEDICAL.COM 67
MEHLMANMEDICAL.COM

- Weird causes of gastric ulcers? à Cushing ulcers (head trauma à increased ACh outflow à increased

M3 receptor agonism on parietal cells à increased acid secretion); Curling ulcers (sloughing of

intestinal mucosa due to acute fluid losses typically seen with burns; think “Curling irons are hot.”)

- Most common cause of duodenal ulcers à caused almost exclusively by H. pylori.

- What does USMLE care about relating to sucralfate? à can coat the base of ulcers + protect them;

activated by acidic pH, so can be inactivated in patients on antacids.

- Mechanism via which H. pylori causes ulcers? à secretion of proteinaceous products that damage

intestinal mucosa (on USMLE Free 120 I believe).

- Gastrin levels in H. pylori? à elevated; H. pylori causes increased acid secretion

- Cancer caused by H. pylori? à MALT lymphoma (mucosa-associated lymphoid tissue lymphoma); H.

pylori is considered a pre-MALT lymphoma condition, where eradication causes remission of 80% of

low-grade stage 1E MALT lymphomas (confined to mucosa + submucosa).

- Key points about Whipple disease? à caused by bacterium Tropheryma Whipplei à causes PAS-

positive macrophages in the lamina propria of the small bowel (USMLE is obsessed with this detail);

can cause arthritis and diarrhea. Tx = Abx (e.g., doxycycline).

- What is Tropical sprue? à malabsorptive disease characterized by flattening of intestinal villi (similar

to Celiac histo); etiology obscure/manifold but bacterial infection is accepted as one cause; Tx = Abx

(e.g., doxycycline).

- Celiac disease important points? à gluten intolerance; fluten found in wheat, oats, rye, and barley, so

therefore will get Sx after eating, e.g., pasta (too easy for the vignette to mention though); causes

flattening of intestinal villi on biopsy (image HY); patients often present with iron deficiency anemia

(HY way to differentiate from lactose intolerance); Dx with Abs: anti-endomysial IgA (anti-gliadin

IgA), anti-tissue transglutaminase IgA à after you get positive Abs in Dx of Celiac, USMLE wants

duodenal biopsy to confirm (“no further studies indicated” is the wrong answer) à Tx = dietary

changes to avoid gluten.

- “I’ve heard IgA deficiency relates somehow to Celiac. Can you explain.” à Remember that

“autoimmune diseases go together,” so increased risk of one means increased risk of another; the

HLA associations are not super-strict à if patient has Celiac, he or she is 10-15x more likely to have

MEHLMANMEDICAL.COM 68
MEHLMANMEDICAL.COM

IgA deficiency (which one comes first is up for debate) à because these patients are IgA deficient,

they will have false-negative results on antibody screening (since Abs are IgA).

- Weird factoid about Celiac? à increased risk of T cell lymphoma; HLA-DQ2/8 positive.

- What immunoglobulin is produced at Peyer patches (GALT; gut-associated lymphoid tissue)? à IgA.

Peyer patches contain large number of IgA-secreting B cells. IgA is a dimer connected by a J-chain.

- USMLE wants you to know colipase deficiency is a reason why a patient with chronic pancreatitis

might not be able to digest triglycerides à yes, weird and random, but I don’t know what to tell you.

It’s on the NBME.

- What’s Zollinger-Ellison syndrome? à gastrin-secreting tumor à USMLE likes “recurrent” duodenal

ulcers, or the presence of any single jejunal or ileal ulcer à frequently seen as part of MEN1

(pancreas, pituitary, parathyroid).

- Dx of ZES? à answer = check serum gastrin levels; if USMLE mentions secretin-stimulation test, it’s

only because they’ll say in the vignette that “gastrin is not suppressed with secretin stimulation” as a

way to tell you the Dx is ZES à secretin should normally lower gastrin levels, but they remain

elevated in ZES.

- Tx of ZES? à PPIs, but definitive is surgical resection of tumor.

- What is Menetrier disease? à don’t confuse with Meniere disease; Menetrier is atrophy of parietal

cells (causing achlorhydria) and hypertrophy of foveolar cells (surface mucous cells) to the extent that

the inner lining of stomach resembles brain gyri; can be caused by CMV infection; Meniere disease, in

contrast, is a tinnitus/vertigo syndrome caused by defective endolymphatic drainage from the inner

ear.

- Types of stomach cancer (apart from MALT lymphoma) à intestinal vs diffuse type; USMLE doesn’t

ask about intestinal (it’s characterized by irregular tubular histology); diffuse = linitis plastica, which is

“leather bottle” appearance of the stomach à cells contain mucin and are called signet ring cells à

often associated with Virchow node (pronounced ver-cough), which is a palpable left supraclavicular

lymph node à this positive node is called Troisier sign of malignancy à if gastric cancer metastasizes

hematogenously to ovaries, the mets are called Krukenberg tumors à biopsy shows signet ring cells

containing mucin; you’ll know it’s not mucinous cystadenocarcinoma (MC) of ovary because 1, MC

MEHLMANMEDICAL.COM 69
MEHLMANMEDICAL.COM

isn’t the bilateral type (serous cystadenocarcinoma is), 2, MC is associated with pseudomyxoma

peritonei (peritoneal infiltration by mucous from tumor), and 3, MC has a “locular,” or “loculated”

appearance.

- High ALP + high direct bilirubin + high amylase or lipase à gallstone pancreatitis =

choledocholithiasis.

- High ALP + high direct bilirubin + high amylase or lipase + remote Hx of cholecystectomy à sphincter

of Oddi dysfunction (can’t be a stone cuz the gallbladder was removed ages ago).

- High ALP + high direct bilirubin + normal amylase or lipase in someone with recent cholecystectomy

à choledocholithiasis (retained stone in cystic duct that descended, but not distal to pancreatic duct

entry point).

- Dx and Tx of choledocholithiasis à ERCP.

- High ALP + high direct bilirubin + normal amylase or lipase in someone with remote cholecystectomy

à pancreatic cancer.

- Dx of pancreatic cancer à CT abdo with contrast.

- High ALP + high direct bilirubin + normal amylase or lipase in someone with remote cholecystectomy

+ CT is negative à cholangiocarcinoma.

- High ALP + high direct bilirubin + normal amylase or lipase + diffuse pruritis + high cholesterol à

primary biliary cirrhosis (PBC).

- High ALP + high direct bilirubin + normal amylase or lipase + autoimmune disease (in pt or family) à

PBC.

- Dx of PBC à anti-mitochondrial Abx next best step; liver biopsy is confirmatory.

- Recent cholecystectomy + fever + abdo pain à post-op bile leak.

- High ALP + high direct bilirubin + normal amylase or lipase + CT shows cystic lesion in bile duct à

choledochal cyst à do simple excision of cyst (cholangiocarcinoma not cystic + CT can be negative).

- Imaging to view liver or pancreas à CT with contrast.

- Imaging to view gallbladder à abdominal ultrasound.

- Imaging to view gallbladder in suspected cholecystitis only if USS negative à HIDA scan.

MEHLMANMEDICAL.COM 70
MEHLMANMEDICAL.COM

- Imaging to view bile ducts à ERCP or MRCP (choose ERCP > MRCP if both listed).

- 22M + stressed studying for exams + yellow eyes + has had a few similar episodes in the past + is

otherwise healthy; Dx? à answer = Gilbert syndrome (proncouned jeel-BEAR).

- Mechanism for Gilbert? à decreased expression of glucuronosyltransferase (bilirubin conjugating

enzyme) in the liver à decreased ability to take up unconjugated bilirubin at the liver à jaundice.

- Tx for Gilbert? à reassurance; benign condition.

- Criteria for pathologic jaundice? à student says “I really need to know that?” Absolutely. HY on peds

shelves and 2CK. If any one or more of the following is positive, the etiology of the kid’s jaundice is

considered pathologic:

o Any jaundice present in first 24 hours of life, period.

o Any jaundice present after one week if term or two weeks if preterm.

o Total bilirubin >15 mg/dL.

o Direct bilirubin >10% of total bilirubin, even if total is under 15 mg/dL.

o (The one everyone forgets) Rate of change of increase of total bilirubin >0.5 mg/dL/hour.

- How do those pathologic jaundice guidelines relate to actual USMLE Qs though? à if pathologic,

USMLE wants phototherapy as the Tx; if that’s insufficient, do exchange transfusion à in addition,

even in adults, if you see a Q where someone’s direct bilirubin is >10% of total, that’ll be a huge clue

to you that an obstructive pathology is present.

- What are normal bilirubin levels? 0.1 mg/dL direct; 1.0 mg/dL total (yes, the lab values will be there

for you on the exam, but do you want to wear training wheels forever? You must know these for 2CK

and Step 3).

- 8-day-old neonate + jaundice + direct bilirubin 14 mg/dL + total bilirubin 15 mg/dL; Dx? à answer =

biliary atresia à super HY Dx.

- 8-day-old neonate + jaundice + direct bilirubin 14 mg/dL + total bilirubin 15 mg/dL; next best step in

diagnosis? à answer = straight-up liver biopsy à if positive, must do liver transplant.

- 8-day old neonate + jaundice + direct bilirubin 1 mg/dL + total bilirubin 20 mg/dL; Dx? à Crigler-

Najjar syndrome.

- Mechanism of Crigler-Najjar? à absence (type I) or deficiency (type II) of glucuronyosltransferase.

MEHLMANMEDICAL.COM 71
MEHLMANMEDICAL.COM

- Tx of Crigler-Najjar? à phenobarbital is helpful in type II (why the USMLE occasionally asks this I don’t

know why); Type I does not respond to phenobarbital; plasmapheresis + phototherapy are

temporizing measures; liver transplant is curative.

- What are Dubin-Johnson vs Rotor syndrome? à never fucking asked on the USMLE but I mention

them here otherwise some students would probably spasm out à high direct bilirubin due to

decreased ability to secrete it into bile à DJ has black liver; Rotor does not.

- Over-arching HY point about hepatitis infections for USMLE à they want you to know that

hepatocellular damage is due to “cytotoxic T lymphocytes,” not “viral cytopathic effects.”

- Hepatitis A HY points à fecal-oral; acute; shortest incubation period (2-6 weeks); vignette will

mention person getting acute hepatitis in US or Mexico à can be asymptomatic, but jaundice, fever,

anorexia common; self-limiting à there’s a Q on one of the 2CK NBMEs where a patient gets HepA

followed by all cell lines (RBCs, WBCs, platelets) down, and the diagnosis was simply viral-induced

aplastic anemia, but this was slightly unusual as we classically associate Parvo B19 with viral-induced

aplastic anemia.

- Hepatitis B HY points à parenteral transmission; in all body fluids and can be transmitted through

breastmilk, sex, and IV drug use à most common transmission is vertical at birth (through birth

canal); vignettes associate HepB with China; 30% of patients with polyarteritis nodosa are HepB

seropositive; HepB can also cause membranous glomerulonephritis; at birth, give HepB vaccine,

followed by a second dose at 2 months, and a third dose at 6 months (apparently some vaccine

schedules are saying it’s no longer given at 4 months); if mom is positive for HepB, give neonate

immunoglobulin + vaccine; if mom’s status is unknown, give neonate vaccine + only give

immunoglobulin if mom’s results come back positive; liver shows ground-glass appearance on biopsy

à I believe a UWSA2 question for Step 1 gave an IV drug user + they showed a liver with a ground-

glass appearance, and the answer was HepB; everyone selects HepC because they say, “oh wow IV

drug user,” but it was HepB; HepC has a nodular appearance of the liver; both HepB and C and cause

hepatocellular carcinoma; USMLE is obsessed with HepB serology:

MEHLMANMEDICAL.COM 72
MEHLMANMEDICAL.COM

- USMLE wants you to know HepB is “DNA, enveloped, circular” and has a polymerase enzyme with

dual function: DNA-dependent DNA polymerase + RNA-dependent DNA polymerase (reverse

transcriptase).

- Tx for HepB? à a variety of meds; the USMLE isn’t really fussed and won’t ask you; this is more

Qbank where they may show up; but some drugs are interferon-alpha, entecavir, tenofovir,

telbivudine, and lamivudine.

MEHLMANMEDICAL.COM 73
MEHLMANMEDICAL.COM

- If you get a Q where they tell you someone was vaccinated for HepB but their surface Ab is still

negative, the next best step in Mx = give HepB vaccination; some patients don’t seroconvert

immediately, or their surface antibody titer wanes.

- HepC important info à longest incubation period (2-26 weeks); can become chronic; parenteral

transmission; transmitted by blood only; not sexually transmitted (if you spend an hour and do a

comprehensive literature review yourself, you’ll learn that the transmission rate among heterosexual

couples where one partner is infected is exceedingly low, i.e., on the order of 1 in 190,000 sexual

contacts); if transmission occurs sexually, it is due to blood exposure; HepB, for instance, is in sexual

secretions; HepC is not.

- Appearance of liver in HepC? à nodular; in contrast, HepB à ground-glass.

- Dx of HepC à ELISA + serum HCV RNA levels.

- Tx of HepC à USMLE likely won’t ask you; but pegylated-interferon-alpha has the greatest chance of

showing up on the exam; other drugs are daclatasvir and sofosbuvir.

- HepD important points à called a “subviral satellite” because it depends on HepB to cause infection

(i.e., without HepB, HepD exposure won’t cause infection) à what’s the best way to prevent it? à

answer = simply vaccinate against HepB à three types of HepB proteins form an envelope around the

HepD ssRNA à infection with HepD can occur in someone with preexisting HepB infection

(superinfection) or at the same time as HepB infection (coinfection); coinfection with HepB+D carries

the highest mortality rate of any of the hepatitis viruses (~20%).

- HepE important points à fecal-oral transmission (enteral); acute disease only; classically when

someone travels to India or Tibet à high mortality in pregnant women is highest yield point.

- Patient takes Abx for several days + has watery diarrhea; Dx? à C. difficile (pseudomembranous

colitis).

- Patient takes Abx for several days + has crampy LLQ pain + bloody diarrhea; Dx? à C. difficile à this

is on a 2CK NBME à Yersinia enterocolitica was also listed and was wrong; this is a good distractor

because Y. enterocolitica causes pseudoappendicitis due to ileitis / mesenteric adenitis, but is RLQ

pain, not LLQ.

- Which Abx cause C. diff overgrowth? à clindamycin, cephalosporins, ampicillin are highest yield.

MEHLMANMEDICAL.COM 74
MEHLMANMEDICAL.COM

- Mechanism for C. diff infection à “Ingestion of spores” is correct over “bacterial overgrowth” on the

USMLE à yes, disruption of normal flora leads to C. diff overgrowth, but ingestion of spores is correct

answer on NBME exam.

- Dx of C. difficile? à answer = stool AB toxin test, not stool culture (exceedingly HY).

- Tx of C. difficile? à guidelines as of Feb 2018 say oral vancomycin first-line, not metronidazole à

apparently UW is updated on this too now à note that vanc is given orally à apart from C. diff, it’s

always given IV because it has terrible oral bioavailability, but in the case of C. diff, where we want

the drug confined to the lumen of the colon, that makes sense.

- Mechanism of colonic necrosis in C. diff colonic necrosis? à answer = “cytoskeletal disruption.”

- Patient is treated with vanc for C. diff but gets recurrence weeks later; why? à answer =

“regermination of spores.”

- C. diff + fever of 104F + tachy + diffuse abdominal pain; next best step in Mx? à AXR à look for toxic

megacolon à Tx w/ NPO (nothing by mouth), NG decompression + rectal tube (decompression) + Abx

(vancomycin or fidaxomicin) + steroids (if UC) + correct any electrolyte imbalances (sometimes low K)

à if patient doesn’t improve with conservative therapy, must do surgery (subtotal colectomy +

ileostomy); do not do a colonoscopy on a patient with toxic megacolon as this will cause perforation.

- Damage to which nerves can cause constipation? à answer = pelvic splanchnic (because these are

parasympathetic, which drive bowel function) à in contrast, hypogastric nerves = sympathetic

(internal anal sphincter); pudendal nerve = somatic (external anal sphincter).

- Hepatocellular carcinoma + peanut farmer from China; cause? à aflatoxin à you don’t have to like

this factoid, but it’s on the NBME.

- Vinyl chloride exposure + liver pathology; what’s the Dx? à answer = hepatic angiosarcoma.

- 17M + fever + tonsillar exudates + cervical lymphadenopathy + cough + hepatomegaly; Dx? à EBV

mononucleosis.

- Alcoholic + liver biopsy shows what? à answer = Mallory hyaline à damaged intermediate filaments.

- 45M + pan-acinar emphysema + increased LFTs; Dx? à alpha-1-antitrypsin deficiency à inability to

break down elastase in the lungs, but enzyme is synthesized in liver; also causes cirrhosis.

- 45M + cirrhosis + fluid wave + fever + abdo pain; Dx? à spontaneous bacterial peritonitis (SBP)

- 69F diabetic + undergoing peritoneal dialysis + fever + abdo pain; Dx? à SBP

MEHLMANMEDICAL.COM 75
MEHLMANMEDICAL.COM

- 8M + viral infection + pedal/periorbital edema + fluid wave + fever + abdo pain; Dx? à SBP à

minimal change disease (nephrotic syndrome) causing ascites.

- SBP Dx à paracentesis (peritoneal aspiration; don’t confuse with periocardiocentesis) + do gram-

stain + look for >250 WBCs per HPF.

- Tx for SBP à ceftriaxone.

- Cause of spider angiomata, palmar erythema, and gynecomastia? à answer = failure of the liver to

process estrogen à answer also = hyperestrogenism.

- Clubbing causes? à pulmonary disease like CF and COBD; cardiac RàL shunts; GI disease; familial,

etc. Bottom line is à just be aware GI disease can cause clubbing (i.e., IBD, Celiac, primary biliary

cirrhosis).

- Woman 20s-50s + high cholesterol + diffuse pruritis + sister has rheumatoid arthritis; Dx? à primary

biliary cirrhosis à USMLE likes “autoimmune diseases go together” in patient (or family).

- How to Dx PBC? à anti-mitochondrial Abs first; if positive, liver biopsy is confirmatory.

- Neonate born 32 weeks gestation + fever + abdominal distension + feeding intolerance +

hematochezia; Dx? à necrotizing enterocolitis à bowel infection in premature neonates usually <32

weeks gestation.

- Dx of NE? à abdominal x-ray (AXR) visualizing pneumatosis intestinalis (air in the bowel wall), air in

the portal vein, or free air under the diaphragm.

- Tx of NE? à NPO (nil per os; nothing by mouth), NG decompression, broad-spectrum Abx; if necrotic

bowel, must do surgery.

- 49M + Hx of abdo pain after meal; now presents with sepsis + diffuse, acute abdo pain; Dx? à

ruptured viscus (duodenum).

- Above 49M; next best step in Dx? à answer = “x-ray of chest + abdomen” to look for air under the

diaphragm (confirms diagnosis); USMLE will never give you choice A) CXR; B) AXR, etc.; they’ll either

give you CXR alone, AXR alone, or both; one of the 2CK surgery NBMEs has both as the answer.

- 28F + pregnant + severe, acute abdo pain + jaundice + hepatomegaly + ascites + encephalopathy; Dx?

à Budd-Chiari syndrome à hepatic vein thrombosis à rare, but associated with pregnancy and

malignancy.

MEHLMANMEDICAL.COM 76
MEHLMANMEDICAL.COM

- Bad type of colonic polyp/adenoma? à villous + sessile characteristics are more sinister than tubular

+ pedunculated.

- Colon cancer progression? à step-wise à APC à KRAS à PTEN à p53. That is highly simplified, but

the point is that USMLE wants you to be aware CRC occurs as a result of many mutations in sequence.

- 18F + CRC in family + has hundreds or thousands of polyps on colonoscopy; Dx? à FAP (familial

adenomatous polyposis) à AD, chromosome 5, APC gene.

- FAP + soft tissue or bone tumors; Dx? à Gardner syndrome

- FAP + CNS tumors; Dx? à Turcot syndrome

- 18F + FAP; Tx? à answer = total proctocolectomy (answer on 2CK NBME; sounds overkill right off the

bat, but it’s the answer) à 100% chance of colon cancer.

- 20M + FAP + skull tumor; Dx? à Gardner, not Turcot à skull is bone, not CNS. Oh wow, craziness.

- 20M + FHx of CRC + has ten polyps seen on colonoscopy; Dx? à HNPCC (Lynch syndrome), not FAP à

FAP has hundreds or thousands of polyps on colonoscopy; HNPCC has “some polyps.”

- Mechanism of HNPCC? à answer = “microsatellite instability,” or “defect in mismatch repair genes”

à mutations in MLH1, MSH2, PMS1.

- 22F + has 30 polyps on colonoscopy + mom died of endometrial cancer; Dx? à HNPCC à Lynch

syndrome associated with gyn tumors such as ovarian + endometrial, as well as other organ system

tumors such as pancreas, stomach, and small bowel. Testicular + prostate very rare, but gyn common.

- 26F + they show you pic of spoon-shaped nail + tell you her lips have been cracked + they ask you

what other symptom she might have; answer = dysphagia. Dx = Plummer-Vinson syndrome à triad

of iron deficiency anemia (causes koilonychia; spoon-shaped nails) + angular cheilosis (cracked

corners of mouth) + esophageal webs (dysphagia).

- USMLE wants the arrow combination (up, down, unchanged) for LES tone and peristalsis in CREST

syndrome; answer = down arrow for both (this is on retired NBME 13 I think for Step 1).

- What does CREST stand for? à Calcinosis, Raynaud phenomenon, Esophageal dysmotility,

Sclerodactyly, Telangiectasias

- 42M + dysphagia to solids + liquids + no other Hx; Dx? à achalasia à inability to swallow solids +

liquids together implies neurogenic etiology.

MEHLMANMEDICAL.COM 77
MEHLMANMEDICAL.COM

- 42M + EtOH Hx + dysphagia to solids that progresses to include liquids; Dx? à esophageal cancer

(SCC) à dysphagia to solids that progresses to solids + liquids = cancer until proven otherwise.

- HY points about esophageal cancer? à adenocarcinoma is distal 1/3 and is caused by GERD (obesity

à low LES tone à GERD à Barrett à adenocarcinoma); SCC is upper 2/3 of esophagus and is caused

by smoking + alcohol; can also be caused by webs, burns, chemicals, and achalasia (difficult, because

achalasia is LES so your thought is, “how can that cause SCC of upper 2/3?” à probably the dysphagia

causes increased esophageal irritation, which then becomes the risk factor for SCC).

- Mechanism for achalasia? à loss of NO-secreting neurons in myenteric plexus of LES à increased LES

tone à bird’s beak appearance on contrast swallow + increased tone on esophageal manometry;

cause is often idiopathic, but Chagas disease (Trypansoma cruzi) is a known infective cause (rare).

- How do you Dx achalasia? à USMLE wants barium (or gastrografin) swallow, then manometry, then

confirmatory biopsy, in that order. There is a Q on an NBME for 2CK where both barium and

manometry were listed, and the answer was barium swallow, not manometry.

- So when is manometry the answer for achalasia? à when they show you a pic of the bird’s beak from

the barium swallow already performed, so clearly the next best step is manometry. The USMLE will

sometimes show a graph of a manometry that’s been performed, and you’ll simply see that the

pressure is high at the LES à hence Dx = achalasia. The confirmatory / most accurate test is biopsy of

LES showing loss of neurons.

- Tx for achalasia à surgery first-line (endoscopic myotomy is superior to pneumatic dilatation); if

patient has high surgical risk, can use botulinum toxin as first-line therapy à if fails, dCCB or nitrates.

- 42M + overweight + halitosis + gurgling sound when drinking fluids + occasionally regurgitates

undigested food; next best step in Mx? à barium (or gastrografin) swallow; Dx = Zenker.

- Above 42M + Hx of GERD à go straight to endoscopy as the answer (cancer, not Zenker).

- Location + mechanism of Zenker? à false diverticulum just superior to the cricopharyngeus on the

posterior pharyngeal wall à USMLE answers are “increased oropharyngeal pressure” and

“cricopharyngeal muscle spasm.” They want you to know it is not a congenital weakness. Dysphagia is

a risk factor because this increases oropharyngeal pressure. I’ve noticed Zenker vignettes often

mention the patient is overweight, although the association is non-specific.

MEHLMANMEDICAL.COM 78
MEHLMANMEDICAL.COM

- What is Whipple triad? à means insulinoma à 1) hypoglycemia, 2) Sx of hypoglycemia (diaphoresis,

tachycardia, tremulousness), 3) relief of Sx with meals (or they’ll say it gets worse between meals).

- If patient has Whipple triad, what’s next best step in Mx? à check serum C-peptide levels à now this

is where I get you a point: if C-peptide is high, the wrong answer is CT abdo to look for insulinoma

because the Dx is not automatically insulinoma. Now you’re probably like, “Really? Wait, why? I’m

not following.” à if C-peptide high, answer = first check serum hypoglycemic levels à meaning,

some patients can surreptitiously take sulfonylureas (i.e., glyburide, etc.), which are insulin

secretagogues, so they’re C-peptide levels will be high. Only after the serum hypoglycemic screen is

negative do you consider CT abdo.

- 32M + high glucose levels + body rash; Dx? à glucagonoma à rash is called necrolytic migratory

erythema and is seen in glucagonoma.

- 32M + watery diarrhea + hypokalemia + achlorhydria; Dx? à VIPoma, aka WDHA syndrome (Watery

Diarrhea, Achlorhydria, Hypokalemia).

- 32M + increased bowel motions + facial flushing; Dx? à VIPoma (another presentation I saw that was

harder to Dx).

- 32M + steatorrhea + pancreatic tumor; Dx? à somatostatinoma.

- Biggest risk factor pancreatic cancer? à smoking

- Biggest risk factor for gall bladder cancer à smoking

- 52F + 2 kids + BMI 28 + recurrent colicky epigastric pain; next best step in Dx? à ultrasound

(cholelithiasis).

- Above 52F + USS shows calcification in the gall bladder wall; next best step in Mx? à

cholecystectomy à porcelain gallbladder carries 1/3 risk of cancer à must do surgical removal.

- Why increased risk of cholesterol stones in pregnancy? à estrogen upregulates HMG-CoA reductase

+ progesterone slows biliary peristalsis à both of these hormonal effects are exceedingly HY.

- Brown pigment stones à bacterial infection à bacteria deconjugate bilirubin, making it less water

soluble à stone precipitation.

- Black pigment stones à hemolytic anemia / increased RBC turnover syndromes (e.g., sickle cell).

- What are pigment stones composed of? à calcium bilirubinate.

MEHLMANMEDICAL.COM 79
MEHLMANMEDICAL.COM

- Random vignette of sickle cell anemia; Q asks you which of the following is the patient at increased

risk of + all answers seem obscure à answer = cholelithiasis à black pigment stones.

- USMLE Q will ask you whether high vs low cholesterol, bile acids, and phosphatidylcholine = good or

bad for cholesterol stone formation? à high cholesterol = bad; high bile acids = good; high

phosphatidylcholine = good.

- 45F + recurrent duodenal ulcers + Hx of renal calculi + serum gastrin levels elevated + she is started

on PPI; next best step in Mx? à check serum calcium levels à MEN1 (pancreatic, parathyroid,

pituitary).

- Travel + watery (or brown-green) diarrhea; Dx? à Travelers diarrhea à ETEC HL/HS toxins.

- MOA of ETEC HL toxin? à HL toxin ADP ribosylates adenylyl cyclase à increases cAMP à increases

Cl secretion into small bowel lumen à Na follows Cl à water follows Na à secretory diarrhea.

- Which organism has same MOA as ETEC HL toxin? à Cholera toxin à difference is cholera is

described as “liters and liters” of high-volume stool (“rice-water stool” is buzzwordy and rarely seen);

if vignette wants to describe rice-water, they’ll say “specks of mucous” in high-volume watery stool.

- MOA of ETEC HS toxin? à ADP ribosylates guanylyl cyclase à increases cGMP à decreases Cl

reabsorption from lumen à more Na stays in lumen à more water stays with Na à watery diarrhea.

- Which organism has same MOA as ETEC HS toxin? à Yersinia enterocolitica toxin à difference is Y.

enterocolitica causes bloody, not watery, diarrhea, and Y. enterocolitica also causes

pseudoappendicitis in children (due to terminal ileitis / mesenteric adenitis) and arthritis in adults.

- Reheated fried rice + watery diarrhea; organism + mechanism? à Bacillus cereus à activation of

spores.

- Kid + bloody diarrhea + petechiae + red urine; Dx? à hemolytic uremic syndrome (HUS) caused by

EHEC O157:H7 or Shigella à HUS = triad of thrombocytopenia + schistocytosis (microangiopathic

hemolytic anemia) + renal insufficiency; toxin will inhibit ADAMTS13 in afferent arterioles + cause

endothelial damage à platelet consumption + clumping (thrombocytopenia) à RBCs shear as they

pass by the platelet clumps (schistocytosis).

- How does HUS contrast with TTP? à TTP is caused by a mutation that results in defective ADAMTS13,

or antibodies against ADAMTS13, resulting in the inability to cleave vWF multimers à platelet

MEHLMANMEDICAL.COM 80
MEHLMANMEDICAL.COM

clumping à similar progression as HUS. One of the points of contrast is that TTP is not toxin-induced,

and TTP also tends to be a pentad of the HUS findings + fever + neurologic signs.

- Bloody diarrhea + chicken / turtles à Salmonella enteritidis / typhimurium.

- Chicken + bloody diarrhea + most common cause overall in the US à Campylobacter jejuni

- Bloody diarrhea + ascending paralysis à Guillain-Barre syndrome caused by Campylobacter jejuni

(many vignettes for GBS will not mention recent infection; GBS can also be caused by Shigella,

Salmonella, and Yersinia).

- Homeless shelter + diarrhea or constipation + rose spots on abdomen + prostrated (lying down in

pain); Dx? à typhoid fever à Salmonella typhi à don’t confuse with food poisoning Salmonella

strains (enteritidis + typhimurium) à the reservoir for typhoid is humans; it is not spread by chickens

or turtles.

- Shigella vs Salmonella points à Salmonella produces H2S gas, is motile, and requires many organisms

to cause infection; Shigella does not produce H2S gas, is non-motile, and very few organisms cause

infection; they are both oxidase-negative.

- Incubation period for infective diarrhea? à gram-negative rods are 1-3 days (E.coli, Salmonella,

Shigella, Yersinia).

- Tx for food poisoning diarrhea à don’t treat majority of time à answer = “Abx increase duration of

fecal excretion of organism.”

- Creams + custards + mayo + potato salad + vomiting 1-6 hours after meal; Dx? à S. aureus heat-

stable preformed toxin.

- Immunocompromised + watery diarrhea; Dx? à Cryptosporidum parvum.

- Tx for C. parvum? à supportive is correct over nitazoxanide.

- Fresh water lake / scuba diving + floaty stools; Dx? à Giardia causing steatorrhea.

- Who notably gets Giardia on USMLE? à IgA deficiency.

- Tx for Giardia à metronidazole.

- Travel + bloody diarrhea + epigastric/RUQ pain; Dx? à Entamoeba histolytica + liver abscess.

- Tx for the liver abscess à drain.

- Intestinal obstruction + rough-surfaced eggs in stool; Dx? à Ascariasis (Ascaris lumbricoides)

- Tx for most nematodes (roundworms) à mebendazole or pyrantel pamoate.

MEHLMANMEDICAL.COM 81
MEHLMANMEDICAL.COM

- Kid + perianal itching; Dx? à Enterobius vermicularis (pinworm).

- Walking on soil / beach + microcytic anemia à hookworm (Ancylostoma duodenale / Necator

Americanus) à USMLE will ask how you acquired hookworm, and the answer is “through your feet.”

- Strongyloides stercoralis; how do you acquire? à through skin (“through your feet”).

- Pregnant woman + ring-enhancing lesions on CT + cats is not an answer; how did she acquire? à

answer = pork à can also get Toxo from pork.

- Travel to Mexico + cystic lesion seen in lateral ventricle on CT and/or “swiss cheese” appearance of

brain; Dx? à cysticercosis Taenia solium à pork tapeworm (cestode).

- Travel to Mexico + muscle cysts à taenia solium.

- Pork or bear meat + fever + myalgia + periorbital edema; Dx? à Trichinella spiralis (trichinosis) à

pork nematode (in contrast pork cestode [tapeworm] is Taenia solium).

- Fish + high MCV; Dx? à Diphyllobothrium latum (fish tapeworm) causing B12 deficiency.

- Beef + tapeworm; Dx? à Taenia saginata.

- Crabmeat + hemoptysis; Dx? à Paragonimus westermani.

- Worm + portal hypertension; Dx? à Schistosoma mansoni / japonicum

- Worm + cholangiocarcinoma (bile duct cancer); Dx? à Clonorchis sinensis.

- Dogs or travel + liver cysts + diarrhea; Dx? à Hydatid cyst disease à Echinococcus granulosis.

- Neonate + regurgitating milk while feeding; next best step in Mx? à answer = insertion of NG tube à

Dx = tracheoesophageal fistula à USMLE wants “endoderm” as the answer if they ask embryo à

most common variant is proximal esophagus ends in blind pouch + distal esophagus connects to

trachea.

- Rupture of gastric ulcer à left shoulder pain; which structure is irritated? à answer = diaphragm.

- Alcoholic liver disease à AST/ALT classically ~2/1 and in the low-hundreds (e.g., 250/125), but I’ve

seen plenty of Qs where this is not the case (e.g., ALT is normal and AST slightly elevated); in chronic

disease, enzymes can be completely normal; in contrast, if you see ALT and AST in the thousands, e.g.,

both are 1200 and ALT is equal to or greater than AST, think viral hepatitis.

- Acetaminophen toxicity à metabolite called NAPQI causes necrosis; give activated charcoal acutely

(apparently there’s a UW Q where this is the answer); give N-acetylcysteine otherwise to “regenerate

reduced glutathione” à reduced-G has a thiol group (-SH) group.

MEHLMANMEDICAL.COM 82
MEHLMANMEDICAL.COM

- Other notable hepatotoxic drugs? à RIP from the TB RIPE drugs (i.e., rifampin, INH, and pyrazinamide

are hepatotoxic); statins + fibrates; antifungal -azoles + terbinafine; methotrexate; halothane;

acetaminophen; those are highest yield probably.

- 45F + BMI 29 + BP 140/90 + high TGAs + low HDL + elevated fasting glucose + slightly elevated AST

and ALT; Dx? à NASH (non-alcoholic steatohepatitis) à caused by metabolic syndrome à Tx is

lifestyle modification to eradicate the metabolic syndrome.

- Above 45F + completely normal liver enzymes; Dx? à NASH à I’ve seen plenty of Qs where all labs

values they show you are completely normal + the only thing you’re left with is, “well she’s fat / has

metabolic syndrome,” and you eliminate the others to just say, “well, this is NASH.”

- Histo of liver showing you large-ish circular lesion + tiny circles immediately next to it; Dx? à primary

hepatocellular carcinoma à smaller lesions are satellite lesions, not mets.

- Histo of liver showing you cancer everywhere; Dx? à colon cancer; bc cancer is everywhere it’s mets.

- Grehlin stimulates appetite (greatest when haven’t eaten for a while).

- Leptin causes satiety (feeling of fullness); greatest just after a meal has started à sounds obvious,

but they show this in graph form, and the wrong answer is immediately when you start eating.

- Liver disease + ascites; why the ascites? à increased hydrostatic pressure (portal HTN).

- Liver disease + peripheral edema; why the edema? à decreased oncotic pressure (decreased

albumin production by the liver).

- Tx for chronic pancreatitis à give patient pancreatic enzymes à “pancrelipase” is an answer on

another NBME Q, which is a mixture of pancreatic lipases, proteases, and amylases.

- Cause of chronic pancreatitis à multiple bouts of acute pancreatitis.

- Major causes of pancreatitis à alcohol + gallstones are major causes; hypertriglyceridemia +

hypercalcemia are other known causes; may also be caused by ERCP (endoscopic retrograde

cholangiopancreatography), mumps, drugs (e.g., sulfa). Absolutely do not say “scorpion sting”

without saying the other causes first or you’ll get upbraided on your surg rotation.

- Gallstone pancreatitis = choledocholithiasis = stone in ampulla of Vater = high ALP + high direct

bilirubin + high lipase or amylase à Dx + Tx = ERCP.

- First Tx for pancreatitis in general à USMLE wants triad of NPO (nil per os; nothing by mouth) + fluids

+ NG tube à if USMLE asks for imaging, do CT with contrast to look for fluid collections + degree of

MEHLMANMEDICAL.COM 83
MEHLMANMEDICAL.COM

necrosis à drain fluid collections percutaneously; if pseudoabscess forms and doesn’t regress,

answer = internal drainage via ERCP or EUS (endoscopic ultrasound); if frank pus (fat enzymatic

necrotic pancreas) >30%, must do necrosectomy (excision of necrotic pancreas) or aggressive

percutaneous drainage.

- Acute pancreatitis + dyspnea + low O2 sats + bilateral infiltrates = ARDS.

- 72M + fatigue + low Hct à answer = do colonoscopy; most common cause of per rectum blood loss in

elderly à diverticular bleed + colorectal cancer + angiodysplasia

- What is angiodysplasia à tortuous, superficial vessels on colonic wall that rupture + bleed à painless

bleeding in elderly.

- Question on 2CK NBME mentions elderly guy with 2/6 mid-systolic who gets per rectum bleeding

after argument with wife à answer = angiodysplasia à Heyde syndrome = aortic stenosis +

angiodysplasia à mechanism might be due to increased pressure within colonic vasculature.

- Difference between diverticulosis vs diverticulitis à diverticulosis is simply the presence of diverticula

à 50% of the US population over age 60 has them à most commonly in sigmoid colon due to law of

Laplace (decreased diameter of sigmoid means greater pressure on the wall à greater propensity for

outpouching); diverticular bleed is most common cause of per rectum blood loss in elderly à they

need not be inflamed; in contrast, diverticulitis is when there is inflammation of diverticula.

- 69M + LLQ pain + fever = diverticulitis à Dx with CT with contrast of abdomen à Tx w/ Abx

(metronidazole, PLUS fluoroquinolone or Augmentin; USMLE won’t ask you the exact Abx, but you

should be aware that metro covers anaerobes below the diaphragm) à never do a colonoscopy on

someone with suspected diverticulitis, as you may cause perforation. However, after the diverticulitis

is fully treated + cleared, patient will need a follow-up colonoscopy to rule out malignancy.

- 12M + pic showing you perioral melanosis (sophisticated way of saying hyperpigmentation around the

lips); Dx? à Peutz-Jeghers syndrome à they’ll sometimes just show you the pic and then ask what

kind of colonic polyps you’ll see à answer = hamartomatous polyps

- They show you pic of PJS hamartomatous polyp; high cancer risk from this lesion? à answer = non-

cancerous.

MEHLMANMEDICAL.COM 84
MEHLMANMEDICAL.COM

- Patient with PJS; need special CRC screening? à yes, not bc of the hamartomatous lesions, but

patients with PJS have increased risk of many types of cancer; for 2CK: do colonoscopy at age 8; if

polyps present, repeat every three years; if not present, repeat at age 18 and then every three years.

- What is IBS? à Irritable bowel syndrome à classically constipation +/- diarrhea +/- other GI Sx like

cramping pain or GERD-like Sx that are relieved with defecation à there are many ways to Tx IBS,

such as starting with psych screen, but if the USMLE asks about meds, they like lubiprostone, which is

used for constipation-predominant IBS (PGE1 analogue that causes increased Cl secretion in bowel à

Na follows Cl à water follows Na à softens stool).

- What is IBD? à IBD = inflammatory bowel disease = Crohn and ulcerative colitis (UC) collectively.

- HLA association with IBD? à HLA-B27 à “PAIR” à Psoriasis, Ankylosing spondylitis, IBD, Reactive

arthritis.

- Crohn GI findings? à mouth to anus; classically terminal ileum; frequently intermittent bloody

diarrhea if colonic involvement; skip lesions causing “string sign” on contrast studies; “cobblestone

mucosa”; transmural inflammation with non-caseating granulomas; perianal fistulae; B12 + fat-

soluble vitamin malabsorption.

- 25M + oral ulcer + bloody diarrhea; Dx? à Crohn

- Extra-intestinal manifestations of Crohn? à classically erythema nodosum (red shins; not a rash; this

is panniculitis, which is inflammation of subcutaneous fat); anterior uveitis (red eyes); oxalate nephro-

/ ureterolithiasis (decreased fat absorption à more binding of fat to calcium [saponification] à

therefore less calcium binds to oxalate à more oxalate absorbed à oxalate stones).

- Any weird factoid about Crohn? à sometimes patients (+) for anti-Saccharomyces cerevisiae Abs

- How to Dx Crohn? à USMLE wants colonoscopy

- Tx for Crohn? à USMLE wants NSAIDs (either sulfasalazine or mesalamine [5-ASA] will be listed)

before steroids à use immunosuppressants late (i.e., infliximab, azathioprine, etc.).

- Does Crohn share anything with UC? à Yes, bear in mind in real life, there is overlap between the

two diseases, so don’t pigeonhole things; think of these disease-associations as propensities rather

than as intransigent rules.

- 28M + lower back pain worse in morning and gets better throughout the day + mouth ulcer; Dx? à

Crohn disease (oral involvement only) + sacroiliitis (back pain Sx of ankylosing spondylitis).

MEHLMANMEDICAL.COM 85
MEHLMANMEDICAL.COM

- 20F + bloody diarrhea + sore joints + eczematoid plaque on forehead à IBD + psoriatic arthritis

- UC GI findings? à rectum-ascending (meaning, starts at rectum and ascends; does not involve anus;

Crohn of course is mouth to anus); bloody diarrhea; not transmural (mucosa + submucosa involved

only; unlike Crohn); no granulomas (unlike Crohn); lead pipe appearance of colon on contrast studies

(unlike “string sign” of Crohn); crypt abscesses (just memorize) à lead pipe means loss of haustra (so

the colon looks smooth from the outside; this is really HY!) à USMLE might also there are

“microabscesses within the crypts” on colonoscopy (meaning crypt abscesses in UC).

- Extra-intestinal manifestations of UC? à primary sclerosing cholangitis (beaded appearance of

common bile duct; can be p-ANCA positive); pyoderma gangrenosum (crater on the forearm with

necrotic debris); like Crohn, is associated with anterior uveitis + HLA-B27 associations.

- How to Dx UC? à USMLE wants colonoscopy.

- Tx for UC? à same as Crohn for USMLE purposes, but just be aware in severe cases colectomy is

performed.

- 65M + intermittent bloody diarrhea + now has fever of 104F + abdominal pain + high leukocytes; Dx?

à answer = toxic megacolon à Dx with AXR, not colonoscopy! à if you scope, patient will perforate

and die à AXR will show dilated bowel (e.g., one NBME Q says “12-cm cecum”); in general, know that

AXR is done when you’re looking for gas.

- Where do most colonic ischemic ulcers occur? à watershed areas à splenic flexure (watershed of

SMA and IMA) + sigmoidal-rectal junction (watershed of IMA and hypogastric artery).

- Ondansetron à 5HT3 (serotonin) receptor antagonist à anti-emetic classically for those with

malignancy / undergoing chemotherapy à for Step 1, USMLE is content with you knowing MOA for

ondansetron + that it acts at the chemoreceptor trigger zone (CTZ) of the caudal medulla à you need

to be able to identify this on sagittal MRI (i.e., they’ll show you letters at different locations and you

need to choose caudal medulla for where ondansetron acts; I’d post an image here but I have zero

interest in copyright infringement à Googling “ctz medulla mri” is more than sufficient; for 2CK be

aware that ondansetron is used for hyperemesis gravidarum during pregnancy (metoclopramide also

used, however ondansetron decreases vomiting significantly more than metoclopramide; nausea

reduction is same).

MEHLMANMEDICAL.COM 86
MEHLMANMEDICAL.COM

- Metoclopramide à D2 receptor antagonist used as a prokinetic agent / anti-emetic à don’t fuck up

this MOA; students all the time remember that it acts at D2 receptors, but not whether it’s an

antagonist or agonist à easy way to remember is based on knowing its side-effects (similar to

antipsychotics!) à hyperprolactinemia + tardive dyskinesia. USMLE Step 1 also wants you to know

that it prolongs the QT interval on ECG. I had a student come out of the exam saying they were asked

which drug doesn’t prolong QT, and they had listed agents such as metoclopramide, azithromycin,

risperidone, amitriptyline, which all CAN prolong QT.

- Really HY point is that metoclopramide is first-line pharmacologic agent in those with diabetic

gastroparesis. USMLE will slam people on how this contrasts with GERD:

- 55M + BMI of 33 + vignette doesn’t mention diabetes + 3 months burning in throat à Dx = GERD à

Tx? = trial of PPIs (i.e., trial of omeprazole) for two weeks à relief of Sx is consistent with GERD as

the correct Dx.

- 55M + BMI of 33 + poorly controlled diabetes (type I or II) + 3 months of burning in throat à Dx =

diabetic gastroparesis, NOT GERD (woahhh crazy) à first pharm Tx = metoclopramide, not PPIs. If

metoclopramide not listed, choose erythromycin (motilin receptor agonist).

- Regarding gastroparesis, the USMLE vignette will make an explicit point about bad diabetic disease,

i.e., peripheral edema (renal insufficiency due to decreased oncotic pressure from albuminuria) +/-

cataracts (osmotic damage from intracellular sorbitol) +/- urinary retention (neurogenic bladder due

to osmotic denervation leading to hypocontractile detrusor) +/- they simply say HbA1C of 12%

(diabetes is 6.5% or greater; prediabetic is 6-6.49). There will be no question as to whether they want

gastroparesis vs simple GERD.

- If the vignette (more 2CK here) doesn’t ask straight-up which drug you choose and they ask for next

best step in Mx for suspected gastroparesis à first do endoscopy to rule out physical obstruction à if

negative, then do gastric-emptying scintigraphy (scintigraphic gastric-emptying assay) à if delayed

gastric emptying, first Tx = smaller meals; if insufficient, then do metoclopramide, then add

erythromycin.

- Ursodeoxycholic acid (Ursodiol) à naturally occurring bile acid given to patients with cholesterol

cholelithiasis as an alternative to cholecystectomy. On USMLE, Dx of cholelithiasis = abdominal

MEHLMANMEDICAL.COM 87
MEHLMANMEDICAL.COM

ultrasound, followed by Tx = cholecystectomy. Ursodiol can be given to select patients but is not

considered the universal next best step.

- USMLE also wants you to know ursodiol is given to pregnant women with intrahepatic cholestasis of

pregnancy (ICP) à classically itchiness of palms + soles in 3rd trimester of primigravid women à

answer = yes, there is increased risk of fetal demise à Dx by checking serum bile acids (high in ICP) à

mechanism = estrogen + progesterone may impair bile secretory transporters, resulting in release

into blood à mechanism of ursodiol in the Tx is unclear, but its use is first-line and HY for 2CK + Step

3 obgyn. I mention it here because it’s otherwise a HY drug for gastro as per above.

- Octreotide à somatostatin analogue à in general, think of somatostain as a hormone that shuts of

secretion of other hormones (e.g., GH, VIP); it also decreases portal blood flow à used for Tx of

esophageal varices AFTER banding (endoscopic ligation) is performed; in other words, on the USMLE,

choose banding for varices before octreotide; propranolol is mere prophylaxis (also decreases portal

blood flow).

- Octreotide also can be used for carcinoid tumors by causing decreased serotonin release from tumor

à carcinoid tumors are usually small-bowel or appendiceal (can also be bronchial) à small blue cells

+ S-100 positive + neuroendocrine origin à Dx with urinary 5-HIAA (5-hydroxyindole acetic acid) à

symptomatic only if metastasizes to liver à diaphoresis + flushing + tachycardia + tricuspid regurg +

clonus (considered UMN finding).

- Cyproheptadine (serotonin receptor antagonist) can also be used for carcinoid, but is classically used

for serotonin syndrome instead à classically drug interactions like switching to an MOAi from an SSRI

without not enough time passing; can also be caused by taking St John Wort if on SSRI, or by tramadol

alone à serotonin syndrome does not cause tricuspid valve lesions because it’s too acute.

- Octreotide + somatostatinomas cause steatorrhea due to inhibition of pancreatic lipase secretion.

- Aluminum (antacid) à causes constipation à “Aluminimum amount of feces.”

- Magnesium (antacid) à causes diarrhea à this actually showed up as a case on 2CS (correct, CS).

- Calcium carbonate (antacid) à can cause rebound gastric acid hypersecretion + milk alkali syndrome

à hypercalcemia + metabolic alkalosis.

MEHLMANMEDICAL.COM 88
MEHLMANMEDICAL.COM

- Orlistat à pancreatic lipase inhibitor used in some patients for weight loss à could theoretically

cause fat-soluble vitamin malabsorption due to decreased intestinal fat absorption à for USMLE

simply know the MOA and that the drug exists.

- Loperamide à mu-opiod receptor agonist used in the Tx of diarrhea à NBME exam asks this drug as

an arrow question à addictive potential LOW (DOWN arrow) à because it can be used in the Tx of

diarrhea, it can also therefore theoretically cause constipation (not rocket science).

- Lactulose à used to Tx hepatic encephalopathy by decreasing serum NH3 à undigestible

carbohydrate that gut bacteria convert to acidic end-product à intraluminal NH3 (absorbable)

produced by bacteria is converted to NH4+ (not absorbable) à USMLE Q will ask you whether the

drug makes gut conditions more or less acidic, as well as whether it’s NH3 or NH4+ that’s not

absorbed (they give you different combos) à answer = “acidic; decreased NH4+ absorption.”

- Neomycin à used to Tx hepatic encephalopathy by killing NH3-producing bacteria in the gut; USMLE

will give you a big, rambling paragraph on hepatic encephalopathy and simply tell you this drug is

given then ask for MOA à answer = “kills intraluminal gut bacteria.”

- Proton pump inhibitors (e.g., omeprazole) are more efficacious than H2-blockers. PPIs are irreversible

and non-competiive; H2-blockers are reversible and competitive.

- Three mechanisms for stomach acid secretion are 1, ACh binding directly to M3 receptors on parietal

cells (Vagus activity), 2, gastrin binding directly to gastrin receptors on parietal cells, and 3, gastrin

causes enterochromaffin-like cells secrete histamine, which then binds to H2 receptors on parietal

cells à these three effects are synergistic à USMLE, in contrast, wants “permissive” for the effects

of cortisol on catecholamines (cortisol upregulates alpha-1 receptors so NE + E can bind and do their

job), and “additive” for the effects of anti-platelet agents used together.

- What is Dumping syndrome? à caused by gastric bypass surgery, diabetes, or malfunctioning pyloric

sphincter, in which stomach contents following a meal enter the duodenum too quickly; there are

two types: early vs late à both show up in vignettes (without people even realizing they’re seeing a

Dumping syndrome Q).

- Early Dumping syndrome à 10-30 minutes after a meal à rapid entry of hyperosmolar gastric

contents into duodenum à osmotic expansion of small bowel lumen à diarrhea + bloating à on

USMLE, answer = “rapid emptying of hyperosomlar chyme into small bowel.”

MEHLMANMEDICAL.COM 89
MEHLMANMEDICAL.COM

- Late Dumping syndrome à 1-2 hours after meal à rapid absorption of carbohydrates through small

bowel wall à hyperglycemia à pancreas secretes lots of insulin à rebound hypoglycemia à USMLE

merely wants you to identify this in a vignette as Dumping syndrome. They might say Hx of gastric

bypass + now there’s a meal + patient gets diarrhea +/- hypoglycemia à Dx simply = Dumping

syndrome.

- What is Blind loop syndrome? à disturbance of normal floral balance in the small bowel due to

disruption of peristalsis (i.e., surgery / post-surgical ileus), but may also be caused by conditions like

IBD and scleroderma à leads to steatorrhea + B12 def + fat-soluble vitamin deficiencies à USMLE

merely wants you to be able to make the diagnosis from a vignette à Tx is with antibiotics

(doxycycline or fidaxomicin).

- Important points about intestinal transporters? à apical = side of intestinal lumen; basolateral = side

of blood; SGLT-1 are GLUT5 are apical transporters that take in monosaccharides from small bowel

lumen; GLUT5 takes in fructose; SGLT1 takes in glucose + galactose (think 5 for fructose being a

pentose, so SGLT1 is for the hexoses, glucose + galactose). Once the monosaccharides are in the

enterocyte (small bowel cell), GLUT2 on the basolateral membrane takes them into the blood.

- 32M + exquisitely painful anal verge + refuses rectal exam; Dx? à anal fissure.

- Where do anal fissures occurs? à posterior in the midline, below the pectinate line.

- Tx for anal fissure? à Sitz bath

- Acanthocytes on blood smear; Dx? à abetalipoproteinemia or liver disease à USMLE loves heat

stroke as cause of acanthocytes à 82F found unconscious on summer day + body temperature of

107F + blood smear shows acanthocytes; Dx = liver failure (heat stroke) à heat stroke = end-organ

damage due to hyperthermia; heat exhaustion is hyperthermia + mental status change + fatigue + no

end-organ signs.

- 8M + bloody stool + perfectly healthy otherwise; Dx? à Meckel diverticulum; student says, “huh, I

thought that was age 2.” I agree with you. But there’s an NBME Q where the kid was 8, and the

answer was Meckel à bleeding due to “heterotopic gastric / pancreatic tissue.”

- How to Dx Meckel diverticulum? à Meckel scan (Tc99 uptake scan that localizes to diverticulum).

- Tx for Meckel à if asymptomatic, can leave alone; if symptomatic, surgical removal.

MEHLMANMEDICAL.COM 90
MEHLMANMEDICAL.COM

- Meckel diverticulum (true or false diverticulum?) à true à contains all layers of bowel à mucosa +

submucosa + muscularis propria + adventitia; in contrast, false (Zenker) is just mucosa + submucosa.

- 16F + fever + high leukocytes + RLQ pain that migrated from epigastrium; Dx? à appendicitis (easy,

but so HY how can I not at least mention it classically) à USMLE wants you to know that migration is

because, initially, epigastric pain = visceral pain; RLQ pain = inflammation of parietal peritoneum.

Must do a pregnancy test if female + adnexal ultrasound to look for gyn causes, i.e., ruptured cyst,

etc. If male, go straight to laparoscopy. If rule out gyn cause in female, do laparoscopic removal.

Ultrasound + CT can be done, but false-negatives have led to rupture + death, so they don’t change

management if clinical suspicion is high, which is why pt goes straight to laparoscopy if under high

suspicion for appendicitis à if during surgery the appendix is normal, answer = still remove it.

- Mallory-Weiss tear vs esophageal varices HY point à MWT usually caused by vomiting/retching in

alcoholic + presents with a little bit of blood in the vomitus; varices present with LOTS of blood à

about half of patients with ruptured varix die.

- Mallory-Weiss tear vs Boerhaave à MWT is not transmural; Boerhaave is transmural à causes

subcutaneous emphysema (crepitus due to air under the skin).

- Tx of varix? à banding + octreotide.

- Prophylaxis for varix? à propranolol.

MEHLMANMEDICAL.COM 91
MEHLMANMEDICAL.COM

YouTube
@mehlmanmedical

Instagram
@mehlman_medical

MEHLMANMEDICAL.COM 92
MEHLMANMEDICAL.COM

MEHLMANMEDICAL
HY GASTRO

All material is copyrighted and the property of mehlmanmedical.

Copyright © mehlmanmedical

MEHLMANMEDICAL.COM 93
MEHLMANMEDICAL
HY ENDOCRINE
MEHLMANMEDICAL.COM

YouTube
@mehlmanmedical

Instagram
@mehlman_medical

MEHLMANMEDICAL.COM 2
MEHLMANMEDICAL.COM

HY Endocrine

- Na/K role of aldosterone à upregulates Na/K-ATPase pump on basolateral membrane of cortical

collecting duct + late-DCT à reabsorbs 3Na into blood for secretion of every 2K into tubular cell

(goes to urine)

- Acid-base role of aldosterone à causes direct proton secretion at apical membrane of cortical

collecting duct

- How does aldosterone relate to ENac? à increased reabsorption of Na at basolateral membrane

means decreased intracellular Na à favorable high-low gradient of Na from urine into tubular cell à

ENac upregulates on apical membrane to increase movement of Na into tubular cell

- Acid-base / biochemical disturbance in Addison à low Na, high K, low bicarb, low pH (metabolic

acidosis)

- Fatigued patient + hyperpigmentation + low BP à Addison disease (primary hypoadrenalism; low

aldosterone + low cortisol)

- Infective cause of Addison à Waterhouse-Friderichsen syndrome (WFS)

- What is WFS à hemorrhagic (not ischemic) necrosis of adrenal glands

- Cause of WFS à meningococcal septicemia (non-blanching rash)

- Low BP in WFS + fluids are given; next best step? à give dexamethasone to compensate for low

cortisol

- If WFS is hemorrhagic necrosis, what HY scenario contrasts à Sheehan syndrome is ischemic necrosis

of anterior pituitary following labor

- Cause of Sheehan à anterior pituitary doubles in size during pregnancy to increase prolactin

secretion; blood loss during labor à ischemic infarction

- USMLE arrow Q for Sheehan à down ACTH, down TSH, down prolactin, up aldosterone (NBME exam)

- How do you Dx Addison disease à ACTH stimulation test (if cortisol doesn’t go up appreciably, Dx

confirmed)

- Weird hematologic finding in Addison à eosinophilia (don’t go chasing ova, stool, parasites)

- Why fatigue in Addison à low cortisol causes chronic fatigue syndrome

MEHLMANMEDICAL.COM 3
MEHLMANMEDICAL.COM

- Why hyperpigmentation in Addison à low cortisol, so less negative feedback at hypothalamus +

anterior pituitary à increase in CRH à increase in POMC (precursor to ACTH and alpha-MSH)

- Tx for Addison à fludrocortisone

- Important point about fludrocortisone à corticosteroid with high mineralocorticoid effect (acts like

aldosterone) à can be used to overcome low aldosterone + low cortisol caused by Addison

- Important point about prednisone + hydrocortisone + dexamethasone à glucocorticoids with very

low mineralocorticoid effect (do not act like aldosterone) à hydrocortisone classically used to treat

secondary hypoadrenalism (only decreased cortisol)

- Cause of Addison à autoimmune, typically part of polyglandular syndrome

- Cause of secondary hypoadrenalism à decreased ACTH production à usually due to impingement

by pituitary tumor (e.g, prolactinoma); can also be caused by Sheehan syndrome

- ACTH levels in Addison vs secondary hypoadrenalism à high in Addison; low in secondary

- Renin levels in Addison? à high

- Potassium levels in secondary hypoadrenalism? à normal because aldosterone intact through RAAS;

cause is merely decreased ACTH

- Primary hyperaldosteronism à Conn syndrome (tumor) or diffuse hyperplasia (autoimmune)

- Renin level in Conn syndrome à low

- High BP + high renin/aldosterone ratio in young woman à fibromuscular dysplasia (FMD)

- Dx of fibromuscular dysplasia? à MR angiography of renal arteries

- High BP + high renin/aldosterone ratio in older patient with cardiovascular disease à renal artery

stenosis

- Cause of renal artery stenosis à atherosclerosis

- Patient with high BP + given ACEi + now creatinine increases; Dx? à renal artery stenosis or FMD

- Acid-base / biochemical disturbance in RAS or FMD à high Na, low K, high bicarb, high pH (metabolic

alkalosis)

MEHLMANMEDICAL.COM 4
MEHLMANMEDICAL.COM

- How does RAAS work? à Low blood volume à JGC secrete renin à renin cleaves angiotensinogen

(produced by the liver) in the plasma into angiotensin I à AT-I goes to lungs and is cleaved by ACE

into AT-II à AT-II goes to zona glomerulosa of adrenal cortex à upregulates aldosterone synthase

à converts 11-deoxycorticosterone into aldosterone

- Important functions of AT-II unrelated to aldosterone? à potent vasoconstrictor on systemic

arterioles (increases afterload) + renal efferent arterioles (increases filtration fraction [GFR/RPF] in

setting of low blood volume, meaning that GFR is maintained despite low renal plasma flow) +

increases PCT reabsorption of Na

- Why is AT-II-mediated PCT reabsorption of Na important? à water follows Na; mechanism to

increase fluid retention in setting of low blood volume

- Effect of giving ACEi on efferent arteriolar diameter à increases diameter

- ACEi (e.g., enalapril) effect on RAAS à increases renin, increases AT-I, decreases AT-II, decreases

aldosterone

- ARB (e.g., valsartan) effect on RAAS à increases renin, increases AT-I, increases AT-II, decreases

aldosterone

- Spironolactone effect on RAAS à increases renin, increases AT-I, increases AT-II, increases

aldosterone

- Eplerenone vs spironolactone à eplerenone way less likely to cause gynecomastia

- If suspected Dx of Conn syndrome à CT abdomen

- If CT abdomen negative (hyperplasia not tumor) à adrenal venous sampling to confirm

- What is pheochromocytoma à catecholamine-secreting tumor of adrenal medulla

- Paroxysmal headaches/palpitations (high BP) + normal electrolytes à pheochromocytoma (PCC)

- Paroxysmal headaches/palpitations (high BP) + high glucose à PCC (catecholamines cause liver to

make glucose)

- Tx for PCC à phenoxybenzamine first (irreversible alpha-1 blocker); never beta-blocker first

- Why phenoxybenzamine first to treat PCC à if you give beta-blocker first, you get “unopposed

alpha,” meaning all of the NE + E (catecholamines) floating around bind to alpha-1, causing massive

arteriolar vasoconstriction, shooting BP through the roof

MEHLMANMEDICAL.COM 5
MEHLMANMEDICAL.COM

- Dx of PCC à urinary or serum metanephrines

- Who gets PCC à NF1, MEN2A/2B (ret gene)

- PCC adults or kids? à adults

- PCC-like presentation in kids? à neuroblastoma (n-myc gene)

- Dancing eyes + high BP + midline mass in kid à neuroblastoma

- Dancing eyes? Wtf? à opsoclonus-myoclonus syndrome (seen in neuroblastoma; yes, weird)

- Dx of neuroblastoma à abdominal ultrasound

- Kid with painless flank mass that doesn’t cross midline à Wilms tumor

- Dx of Wilms tumor à renal ultrasound

- Neonatal boy with midline suprapubic mass à posterior urethral valves

- Dx of posterior urethral valves à voiding cystourethrogram

- Cushing syndrome vs Cushing disease à syndrome = what you look like + refers to any cause of

Cushingoid appearance; Cushing disease only = anterior pituitary ACTH-secreting tumor; in other

words, Cushing disease is a cause of Cushing syndrome

- Most common cause of Cushing syndrome à exogenous glucocorticoids (e.g., prednisone)

- Pt not on exogenous steroids + Cushingoid; most common cause? à Cushing disease most common

cause of endogenous Cushing syndrome

- Main causes of Cushing syndrome à exogenous steroids (most common overall), Cushing disease

(most common endogenous), small cell bronchogenic carcinoma (ectopic ACTH), cortisol-secreting

tumor (or diffuse hyperplasia) of zona fasciculata of adrenal cortex; CRH tumor rare as fuck

- Patient with chronic disease (i.e., IBD, SLE, RA) + Cushingoid; what are the ACTH + cortisol levels à

need to know this means patient is taking prednisone à low ACTH + low cortisol (prednisone is NOT

the same thing as cortisol) à prednisone suppresses CRH and ACTH secretion at hypothalamus and

anterior pituitary à decreased endogenous cortisol production

- Patient with Cushing disease; ACTH + cortisol levels? à high ACTH + high cortisol

- Smoker + Cushingoid à small cell bronchogenic carcinoma

- Smoker + Cushingoid; ACTH + cortisol levels? à high ACTH (ectopic) + high cortisol

- When to do dexamethasone suppression test à Cushingoid patient not on exogenous glucocorticoids

and has high serum cortisol

MEHLMANMEDICAL.COM 6
MEHLMANMEDICAL.COM

- Low-dose dexamethasone suppression test à tells us yes or no, patient has pathologic cause of

Cushing syndrome (i.e., Cushing disease, or SCC of lung, or cortisol-secreting tumor), but we can’t

establish the causation from this; if cortisol doesn’t suppress à yes, patient has true Cushing

syndrome (proceed to high-dose test); if cortisol suppresses à no, patient does not have Cushing

syndrome (do not proceed to high-dose test)

- High-dose dex à only cause of Cushing syndrome that will suppress in response is Cushing disease

- Pt has no suppression to low-dose dex + suppresses to high-dose à Dx = Cushing disease (ACTH

secreting tumor of anterior pituitary)

- Pt has no suppression to low- or high-dose dex à ACTH high? à Yes, answer = SCC of lung; No à

answer = cortisol-secreting tumor (or diffuse hyperplasia) of adrenal cortex

- Cushingoid + low ACTH + low cortisol à exogenous steroids

- Cushingoid + low ACTH + high cortisol à cortisol-secreting tumor (or diffuse hyperplasia) of adrenal

cortex

- Cushingoid + high ACTH + high cortisol à Cushing disease

- Most accurate test for Dx Cushing syndrome à 24-hour urine cortisol

- Why dex test not most accurate? à false-positives in e.g., depression, alcoholism

- Acanthosis nigricans + low K + hyperpigmentation à Cushing syndrome

- Why acanthosis nigricans à caused by insulin resistance (unrelated: also can be caused by visceral

malignancies)

- Why low K in Cushing syndrome à chronic elevation of glucocorticoid effect at kidney can push out

potassium similar to aldosterone

- Why hyperpigmentation à high ACTH secretion means POMC is high à high alpha-MSH as well

- Purple striae in obese patient à Cushing syndrome

- Why purple striae à glucocorticoids weaken collagen à micro bleeding into skin

- Why hypertension in Cushing syndrome à cortisol upregulates alpha-1 receptors on arterioles,

thereby allowing NE and E to do their job

- Graph shows you two scenarios: 1) NE given alone, then BP increases a little; 2) NE + cortisol given

together, then BP increases a lot; why the difference? à cortisol is permissive of the effects of

MEHLMANMEDICAL.COM 7
MEHLMANMEDICAL.COM

catecholamines (don’t choose synergistic or additive); once again, cortisol merely allows NE and E to

do their job; cortisol isn’t directly increasing BP

- Why normally ratio of E to NE in the blood is 80/20? à NE draining venously out of the adrenal

medulla passes through the adrenal cortex à cortisol upregulates PNMT (converts NE to E)

- What does low cortisol cause? à chronic fatigue syndrome (as mentioned earlier but super

important)

- Function of PTH à increases blood Ca + decreases blood PO4

- Function of PTH at bone à increases bone resorption

- Cell PTH binds to at bone à osteoblast, not -clast

- If PTH causes bone resorption, why the fuck would it bind to osteoblasts (which build bone) à

because PTH causes osteoblasts to express RANK-Ligand on their cell surface à binds to RANK

receptor on osteoclasts à bone resorption

- How do osteoclasts resorb bone à intracellular carbonic anhydrase II (CAH-II) à creates H2CO3 from

H2O and CO2; then the H2CO3 à bicarb + proton à protons accumulate at bone-osteoclast interface

à surface for resorption

- What is teriparatide à N-terminus PTH analogue that binds to osteoblasts, and then rather than

causing bone resorption, actually stimulates bone growth (difference in mechanism not well

elucidated) à used in advanced osteoporosis

- Avoid teriparatide in whom? à Paget disease of bone patients; Hx of osteosarcoma

- Three effects of PTH at the kidney? à 1) it upregulates 1-alpha hydroxylase activity in the PCT

(converts inactive 25-OH-D3 into active 1,25-(OH)2-D3; 2) decreases PO4 reabsorption in PCT by

downregulating three types of apical membrane brush-border sodium-phosphate symporters; 3)

increases Ca reabsorption in the late-DCT by upregulating the apical TRPV5 transporter à reabsorbs

calcium.

- What does secretion in kidney terms mean à excretion through the tubular wall (excretion is

umbrella term à filtration = excretion through Bowman capsule; secretion = through tubular wall)

- Primary hyperparathyroidism biochemical disturbance? à high Ca, low PO4, high ALP, high PTH

- Why is ALP high à ALP reflects osteoblast activity; if PTH high, then ALP also high (but annoyingly, if

PTH is low, ALP will be normal, not low)

MEHLMANMEDICAL.COM 8
MEHLMANMEDICAL.COM

- Who gets primary hyperPTH à usually parathyroid adenoma (e.g., 22 yr old girl with nodule)

- Who else gets primaryPTH à MEN1 + MEN2A (can be diffuse 4-gland hyperplasia in MEN patients)

- Fuck, what are the MEN syndromes again? à Relax. MEN1 = parathyroid, pituitary, pancreas; MEN2A

= parathyroid, PCC, medullary thyroid carcinoma; MEN2B = PCC, medullary thyroid carcinoma,

mucosal neuromas, and Marfanoid body habitus (“oid” means looks like but ain’t)

- Question says girl has high Ca + low PO4 + nodule of left, superior parathyroid gland; what’s the

embryo derivative? à 4th pharyngeal pouch

- 3rd pouch = thymus + two inferior parathyroids

- 4th pouch = two superior parathyroids

- USMLE classically likes DiGeorge syndrome for agenesis of 3rd + 4th pouches, but will see if you can

distinguish by asking parathyroid adenoma Qs

- Other weird info I need to know for primaryPTH? à Yeah, firstly, urinary Ca is high, not low.

- Wtf, how is that possible if PTH reabsorbs Ca from urine à because serum Ca is high, so the net

amount in the urine is still high (this is a HY arrow Q that everyone gets wrong); in other words, on

the USMLE, in primaryPTH à serum Ca up; serum PO4 down; serum ALP up; urinary Ca up (oh wow)

- Second weird factoid about primaryPTH à urinary cAMP is elevated (USMLE likes this for some

magical reason)

- What is the biochemical disturbance in secondary hyperparathyroidism? à Ca and PO4 opposite of

primary à low Ca; high PO4, high ALP + high PTH

- Who gets secondaryPTH à chronic renal failure

- Why does chronic renal failure cause secondaryPTH à inability to activate vitamin D3 in PCT à

decreased Ca absorption through small bowel à low serum Ca à stimulates PTH release (this

mechanism is on 2CK NBME interestingly); kidney also simply cannot reabsorb Ca as well, further

decreasing serum Ca and stimulating PTH release

- What about low Ca in acute renal failure; doesn’t it take a while for vitamin D effects to occur à if

low Ca in acute, due to mere inability to reabsorb from urine

- Why high PO4 in renal disease à kidney can’t filter it out; even though there’s less activated D3 and

PO4 absorption through small bowel is also decreased, the inability of the kidney to excrete it “wins”

MEHLMANMEDICAL.COM 9
MEHLMANMEDICAL.COM

- Biochemical disturbance of vitamin D deficiency à low Ca, low PO4, normal/high PTH, normal/high

ALP

- Function of vitamin D à increases Ca + PO4 absorption in the bowel (don’t worry about minimal role

in bone)

- I don’t get it though; if in chronic renal failure there’s vitamin D deficiency due to less activation, why

is there low Ca and high PO4? Isn’t vitamin D deficiency always low Ca and low PO4? à chronic renal

failure “wins” in terms of phosphate always à so although low vitamin D, there’s still high PO4.

- Any changes to the bones in a patient with chronic renal disease à renal osteodystrophy

- Hip fracture in renal failure + x-ray shows “pseudofractures” à vitamin D deficiency

- Osteitis fibrosa cystica à high PTH can cause “brown tumors” of bone + cholesterol accumulation in

bone

- Kid with bowing of tibias + chostochondritis + craniotabes à rickets (vitamin D deficiency)

- Adult with pseudofractures à osteomalacia (vitamin D deficiency)

- Where does vitamin D deficiency start à stratum basale of skin (asked on USMLE)

- What is the sequence for production? à 7-dehydrocholsterol in skin goes to cholecalciferol via UV-B

à then cholecalciferol goes to liver where it takes on a hydroxylation to make 25-hydroxy-D3

(calcidiol) à then this goes to the kidney where, via PTH activing 1-alpha-hydroxylase, it gets

converted to active 1,25-(OH)2-D3 à then this goes to small bowel to cause Ca + PO4 absorption

- If person doesn’t get sunlight, what can he/she not make à cholecalciferol (7-dehydro is wrong

answer; this is on NBME)

- What about 7-dehydrocholesterol in relation to sunlight à we make this on our own, then UV-B

converts it into cholecalciferol

- Alcoholic who eats plenty of dairy + gets sunlight + has vitamin D deficiency à answer = decreased

hepatic hydroxylation

- Inject person with Ca; what happens to their vitamin D notably à answer = increased 24,25-(OH)2-

D3.

- Wtf is 24,25? à 25-OH-D3 is immediately converted to 24,25 as a storage form (think of them as the

same)

MEHLMANMEDICAL.COM 10
MEHLMANMEDICAL.COM

- Inject with Ca à Ca binds to Ca-sensing receptor at parathyroid gland à negative feedback à causes

reduction in PTH release à decreased 1-alpha-hydroxylase activity at kidney à buildup of 25 form à

shunted to 24,25 form.

- Low calcium or potassium in a patient + he/she not responsive to supplementation à answer =

hypomagnesemia à Mg needed for basal levels of PTH release (so low PTH à low Ca) + renal

retention of K

- Alcoholic + low Ca or low K not responsive to supplementation à check serum magnesium

- Why low Mg in alcoholic à dietary deficiency

- Who gets high vitamin D (hypervitaminosis D) à granulomatous disease (sarcoidosis) à don’t be the

fool who says “girl tried to commit suicide by ODing on vitamin D pills?”

- Why high vitamin D in sarcoidosis à epithelioid (activated) macrophages secrete 1-alpha hydroxylase

à activate vitamin D à increased small bowel absorption

- Ca and PTH levels in sarcoidosis à high Ca, low PTH (negative feedback)

- Sarcoidosis, any weird fact they ask about Ca? à Answer = “decreased Ca in feces” (makes sense)

- Biochemical disturbance in high vit D à high Ca, normal or high PO4, normal or low PTH, normal or

low ALP (USMLE will only give you one correct answer, don’t worry, but I write the possibilities here

so you don’t get a Q where you see normal PTH or PO4 and are wondering wtf, but Ca always up)

- Sarcoidosis other weird info à increased serum ACE (correct, angiotensin-converting enzyme; weird)

- Sarcoidosis, what happens first, high urinary Ca or high serum Ca à high urinary Ca (kidney will

compensate before serum levels go up)

- 20s-30s African American woman with dry cough; CXR shows nodularity; Dx? à sarcoidosis;

nodularity just means “bihilar lymphadenopathy”

- 20s-30s African American woman with dry cough; CXR shows nodularity; Dx? à “noncaseating

granulomas” (sarcoidosis)

- 20s-30s African American woman with dry cough; CXR is normal Dx? à asthma (1/3 of asthma

patients only have dry cough; this is called cough-variant asthma)

- 20s-30s African American woman with dry cough; CXR is normal Dx? à “increased activation of mast

cells” (asthma)

MEHLMANMEDICAL.COM 11
MEHLMANMEDICAL.COM

- Mutation in calcium-sensing receptor on parathyroid gland à loss of negative feedback à PTH goes

up, serum Ca up

- Mutation in calcium-sensing receptor on parathyroid gland à cause of familial hypocalciuric

hypercalcemia (FHH)

- High PTH + high serum Ca + low urinary Ca à FHH

- High PTH + high serum Ca + high urinary Ca à primary PTH

- Mechanism for low urinary Ca in FHH? à not fully elucidated, but literature suggests increased

tubular reabsorption, possibly paracellular, as serum Mg in FHH elevated relative to primaryPTH

- What is cinacalcet à calcimimetic à activates Ca-sensing receptor at PTH gland

- When to use cinacalcet à tertiary hyperPTH

- What is tertiary hyperparathyroidism? à classically seen in renal transplant patients; autonomous

four-gland hypersecretion of PTH following long-term secondary hyperparathyroidism and

hypocalcemia à parathyroid chief cell hyperplasia that does not resolve with renal transplant à

result is high Ca, variable PO4 (high if still renal impaired), high ALP, high PTH

- What is pseudohypoparathyroidism? à insensitivity to PTH à high PTH but low Ca + high PO4

- Anything special about pseudohypoparathyroidism? à Yes. This is one of the highest yield yet

underemphasized conditions on the USMLE Step 1. In other words, my students get these Qs on the

exam regularly but the resources don’t emphasize the different types of this condition

- What is presentation of pseudohypoparathyroidism? à need to know Types 1a, 1b, 2

- Type 1a = Albright Hereditary Osteodystrophy = is simply the name of the phenotype à AHO has

shortened 4th + 5th metacarpals, short stature + intellectual disability + urinary cAMP does not

increase with exogenous PTH administration (normally should go up in response to PTH)

- Type 1b à high PTH + low Ca + high PO4 + no AHO phenotype (just biochemical disturbance)

- Type 2 à same as Type 1b but urinary cAMP increases in response to exogenous PTH

- Mike, this sounds pedantic though. The USMLE really asks about pseudohypoparathyroidism like

that? à Yeah. They ask it as arrow questions. And I personally had two pseudohypoPTH Qs on my

Step 1 + my students do.

- What is pseudopseudohypoparathyroidism? à AHO phenotype but no biochemical disturbance

- Graves disease parameters à low TSH, high T3, high T4, increased iodine uptake

MEHLMANMEDICAL.COM 12
MEHLMANMEDICAL.COM

- Mechanism of Graves à activating TSH-receptor antibody, called TSI – thyroid stimulating

immunoglobulin)

- Tx for Graves à propylthiouracil (PTU) or methimazole to address thyroid derangement

- PTU and methimazole MOAs à both inhibit thyroperoxidase, but PTU also inhibits peripheral

conversion by inhibiting 5-deiodinase à propylthiouracil is a longer word than methimazole, so it’s

the one that does both

- Tx for tachycardia in Graves à propranolol (because beta-blockade also decreases peripheral

conversion of T4 to T3)

- Important point about Graves? à proptosis/exophthalmos + pretibial myxedema are specific; in

other words, there are numerous causes of hyperthyroidism (e.g., toxic multinodular goiter, toxic

adenoma, etc.), but only Graves will cause the eye findings

- Tx for ophthalmopathy in Graves à steroids

- Why do the eye findings occur in Graves? à glycosaminoglycan deposition in/around extra-ocular

muscles

- High BP + fever + increased CK à thyroid storm (Tx with PTU + propranolol)

- What is the role of potassium iodide (KI) in hyperthyroid Tx? à shuts off gland production (Wolff-

Chaikoff effect) à answer in person exposed to nuclear fallout or radioiodine vapors in laboratory

- Hashimoto parameters à high TSH, low T3, low T4, decreased iodine uptake

- Mechanism for Hashimoto à antibodies against thyroperoxidase + thyroglobulin; anti-microsomal

- Histo of Hashimoto à lymphocytic infiltrate (easy to remember bc the non-eponymous name for

Hashimoto is chronic lymphocytic thyroiditis)

- Tx for Hashimoto = levothyroxine (synthetic T4)

- 50M + low mood + BMI 26 à Hashimoto

- Proximal muscle weakness in Hashimoto + increased serum CK à hypothyroid myopathy

- 45M + decreased ability to get up from chair unassisted + HR of 60 à Hashimoto

- 45M + high cholesterol + high hepatic AST + HR of 55 à Hashimoto (hypothyroidism can cause

bradycardia, high cholesterol, and high AST [the latter is weird, correct])

- Thyroid cancer in Hashimoto à thyroid lymphoma (autoimmune diseases à increased risk of

lymphoma)

MEHLMANMEDICAL.COM 13
MEHLMANMEDICAL.COM

- 22M + viral infection + very tender thyroid à subacute granulomatous thyroiditis (de Quervain)

- De Quervain parameters à triphasic à causes hyper-, then hypo-, then rebounds to euthyroid state

à most important point is that iodine uptake is DECREASED in de Quervain + drug-induced

thyroiditis, even if patient is in a hyper-phase.

- Tx for de Quervain thyroiditis? à answer = NSAIDs

- 22M + very tender thyroid + HR of 88 + tremulousness + heat intolerance à low TSH, high T3, high

T4, decreased iodine uptake (in contrast to Graves, which is painless and uptake is high)

- 27F + gave birth to healthy boy 6 months ago following uncomplicated labor + no weight change or

mood disturbance + on no meds + vitals WNL + dry skin + thyroid gland enlarged and non-tender +

TSH high + T4 low; most likely explanation for these findings? à answer = “thyroiditis” à Dx =

postpartum thyroiditis (a type of silent thyroiditis) à characterized by thyrotoxicosis followed by

hypothyroidism (1/3 of women experience both phases; 1/3 experience just hyperthyroid phase; 1/3

only hypothyroid phase); affects 5-10% of women postpartum; hyperthyroid phase usually occurs 1-4

months postpartum; hypothyroid phase occurs about 4-8 months postpartum; thought to be caused

by postpartum immunologic rebound (immune system normally suppressed during pregnancy); Dx w/

Hx + ordering serum TSH; increased risk of progression to Hashimoto; Tx w/ short course of

propranolol if hyperthyroid; give short course of levothyroxine if hypothyroid

- Tx for subacute thyroiditis à aspirin first, not steroids; steroids may be used later

- Drugs causing thyroiditis à lithium + amiodarone

- Surreptitious thyrotoxicosis àself- injection of thyroxine à low TSH, high T3, high T4, small thyroid

gland with decreased uptake

- Injection of triiodothyroinine (T3) à TSH will go down, T3 goes up (clearly), T4 does not go up

because T3 isn’t converted to T4; only T4 is converted to T3

- Injection of thyroxine à TSH will go down (negative feedback), T4 goes up (clearly), T3 goes up (due

to peripheral conversion), reverse T3 also goes up

- What is reverse T3? à an inactive form of T3; T4 is converted peripherally into T3 (active) and reverse

T3 (inactive)

- Anything else I need to know about reverse T3? à it’s increased in euthyroid sick syndrome à times

of stress/surgery/illness à cortisol increases à cortisol decreases conversion of T4 to active T3, so

MEHLMANMEDICAL.COM 14
MEHLMANMEDICAL.COM

more T4 is converted to reverse T3 à parameters in euthyroid sick syndrome: normal TSH, normal

T4, low T3, high reverse T3

- What is sublinical hypothyroidism à high TSH but normal T3 + T4 (don’t confuse with ESS)

- Subclinical hypothyroidism Tx à don’t treat unless TSH >10 (normal is 0.5-5), Hashimoto Abs are

present, or patient is pregnant à they ask this info on 2CK + Step 3

- Want to check thyroid function, what’s the first thing to order à TSH

- Want to check thyroid function in pregnancy, what’s the first thing to order à free T4

- What is free T4 à most thyroid hormone is protein-bound and inactive; free T4 tells you definitively

whether the patient has thyroid derangement or not

- Pregnancy and thyroid à estrogen causes increased thyroid-binding globulin production by the liver

à mops of T4 à less free T4 à less negative feedback at hypothalamus + anterior pituitary à TSH

goes up transiently to compensate à more T4 made à free T4 rebounds to normal but now total T4

is high à parameters you need to know for pregnancy: normal TSH + high total T4 + normal free T4 +

normal free T3 + high thyroid-binding globulin

- Hyperthyroidism in pregnancy à LH, FSH, TSH, hCG all share same alpha-subunit; their beta-subunits

differ; some women have increases sensitivity of TSH receptor to alpha-subunit, so high hCG in early

pregnancy can stimulate thyroid gland and cause transient hyperthyroidism

- Hashimoto in pregnancy à increase pregnant woman’s dose of levothyroxine by 50%

- Graves in pregnancy à avoid methimazole in first trimester (teratogenic; causes aplasia cutis

congenita) à give PTU in first-trimester à in second + third trimesters, switch from PTU to

methimazole (methimazole no longer teratogenic later in pregnancy + PTU is heavily hepatotoxic)

- Pt being treated for Graves + mouth ulcers à agranulocytosis (neutropenia) caused by methimazole

or PTU.

- Young child with normal free T4 and low total T4 à thyroid-binding globulin deficiency (opposite of

pregnancy)

- Young child + large belly + large tongue + hypotonia à cretinism (congenital hypothyroidism)

- Most common cause of cretinism à iodine deficiency

- Most common cause of hyperthyroidism in elderly à toxic multinodular goiter

- Evaluation of thyroid cancer, first step? à palpation of thyroid gland (on FM 2CK form as answer)

MEHLMANMEDICAL.COM 15
MEHLMANMEDICAL.COM

- If thyroid nodule present, then check TSH; if TSH normal or high à answer = FNA, not USS; if TSH low,

do radioiodine uptake scan; thyroid cancer is cold, not hot, which is why no FNA with low TSH

- Struma ovarii à thyroid hormone secreting ovarian germ cell tumor

- Diabetic ketoacidosis parameters à low serum Na, high serum K (hyperkalemia), low total body K,

low bicarb, low pH, low CO2 (all exceedingly HY)

- Why low serum Na in DKA à osmotic effect of high glucose in blood à dilutional hyponatremia; in

addition, there’s believed to be a contributory renal mechanism related to increased absorption of

water in the PCT due to increased SGLT2 reabsorption of glucose in PCT

- Why high serum K in DKA à three main reasons: 1) insulin normally drives K into cells, so if insulin

isn’t there, K is higher in blood; 2) less glucose driven into cells by GLUT4 (bc normally upregulated by

insulin) means less ATP production à normally 1 ATP drives 2K into cell and 3Na out; so if less ATP-

ase activation, less K enters cell à higher in blood; 3) potassium-proton exchange; if acidosis ensues,

more H driven into cells means K moves out to balance charge à hyperkalemia

- Then what does low total body potassium mean à just to be clear, the patient is hyperkalemic (high

K in the blood) yet has low K overall in the body à kidney senses high K in urine and therefore

increases excretion of it (kaliuresis) à body is now losing K à but three above mechanisms leading to

hyperkalemia continue unabated, so K stays high in serum even though body is now urinating it out.

- Why does the potassium stuff matter so much with DKA à because when you Tx DKA and start giving

insulin (fluids first btw; giving insulin immediately is the wrong answer; give insulin after first

administering a bolus of normal saline), K will now be driven into the cells, which will bring K down to

normal in the serum, but bear in mind it was low in the cells à so now risk of normal in cells but low

in blood à need to supplement K to patient when K falls below 5.2 (normal is 3.5-5 mEq/L). Stop all

insulin if K falls below 3.3.

- Overweight, middle-age patient with polydipsia + polyuria à type II DM

- High in serum in type II DM à insulin is high initially; ketones absent (only ketones in DKA; DKA is

type I only)

- Type II diabetic crisis? à hyperosmolar hyperglycemic non-ketotic syndrome (HHNS) à still give

fluids first

- Glucose numbers in DKA vs HHNS à low-hundreds for DKA (i.e., 2-300s); can be 600-1000 for HHNS

MEHLMANMEDICAL.COM 16
MEHLMANMEDICAL.COM

- Acid-base disturbance in aspirin toxicity first 20 mins à resp. alkalosis (low CO2, high pH, normal O2,

normal bicarb [too acute to change])

- Acid-base disturbance in aspirin toxicity after 20 mins à mixed metabolic acidosis-respiratory

alkalosis (low CO2, low pH, normal O2, low bicarb)

- Tx for aspirin toxicity à bicarb (increased excretion through urinary alkalinization)

- Acid-base disturbance in PE à resp. alkalosis (low CO2, high pH, low O2, normal bicarb [too acute to

change])

- Acid-base disturbance in asthma à resp. alkalosis (low CO2, high pH, low O2, normal bicarb [too

acute to change])

- Electrolyte abnormality in anorexia à hypokalemia

- Most common cause of death in anorexia à arrhythmia from hypokalemia

- Refeeding syndrome (even if normal BMI) à hypophosphatemia

- Amenorrhea in anorexia à low FSH + low estrogen (hypogonadotropic)

- Premature ovarian failure + Turner syndrome + menopause à high FSH (low inhibin) + low estrogen

- High BMI female + irregular menstrual cycles à anovulation

- Anovulation + hirsutism à PCOS

- Anovulation. Cause USMLE wants? à insulin resistance à causes abnormal GnRH pulsation

- Why hirsutism in anovulation à abnormal GnRH pulsation causes high LH/FSH ratio

- Why high LH/FSH ratio important in anovulation/PCOS à ovulation stimulated when follicle not

ready à no ovulation (anovulation) à follicle retained as cyst

- What’s LH do? à Stimulates theca interna cells (females) and Leydig cells (males) to make androgens

- What’s FSH do? à Stimulates granulosa cells (females) and Sertoli cells (males) to make aromatase;

also primes follicles

- Tx for PCOS à if high BMI, weight loss first always on USMLE

- Tx for PCOS if they ask for meds and/or weight loss already tried à OCPs (if not wanting pregnancy);

clomiphene (if wanting pregnancy)

- PCOS increases risk of what à endometrial cancer (unopposed estrogen)

MEHLMANMEDICAL.COM 17
MEHLMANMEDICAL.COM

- Where does ADH (vasopressin) act à medullary collecting duct à causes aquaporin insertion on

apical membrane à enables free H2O reabsorption

- Where is ADH made? à supraoptic nucleus of hypothalamus à merely stored in posterior pituitary

- Where is oxytocin made à paraventricular nucleus of hypothalamus à merely stored in p. pituitary.

- Neurophysins I’ve heard about. What are those à Showed up in UWorld à carrier proteins needed

to transport oxytocin + ADH from hypothalamus to posterior pituitary

- When does ADH go up à when serum sodium too high à brings sodium back down; ADH will also be

secreted in response to lower blood volume, although aldosterone is major volume regulator; ADH is

main tonicity regulator

- When considering SIADH vs diabetes insipidus (DI) vs psychogenic polydipsia (PP) à what’s the next

best step in evaluation/management à fluid restriction

- SIADH important causes à small cell bronchogenic carcinoma ectopic ACTH, or head trauma (can

actually cause SIADH or DI)

- SIADH parameters à high urine osmolality (concentrated) + low serum sodium (normal is 135-145)

- Tx for SIADH à if small cell lung cancer, chemotherapy (HY to know you can’t do surgery for small

cell); if insufficient, give -vaptans (conivaptan, tolvaptan), which are ADH receptor antagonists, or

demeclocycline (a tetracycline antibiotic that causes nephrogenic DI, but is a Tx for SIADH).

- Central diabetes insipidus à lack of production by hypothalamus or storage problem of posterior

pituitary à low ADH + low urine osmolality (dilute urine) + high serum sodium (concentrated serum)

- Cause of CDI à idiopathic, local malignancy, or head trauma

- Nephrogenic DI à lack of sensitivity of kidney V2 receptors to ADH à high ADH, low urine osmolality

(dilute urine) + high serum sodium (concentrated serum)

- Cause of NDI à lithium, demeclocycline

- Patient with bipolar disorder + tremors + polyuria + polydipsia à answer = NDI

- Diabetes insipidus urine parameters relative to serum: PCT is isotonic (same; always unchanged),

medullary collecting duct is hypotonic (dilute compared to serum), juxtaglomerular apparatus (JGA) is

hypotonic (always hypotonic no matter what the patient’s condition bc value measured at the top of

thick ascending loop of Henle after ions have been absorbed out of urine)

MEHLMANMEDICAL.COM 18
MEHLMANMEDICAL.COM

- SIADH urine parameters relative to serum: PCT is isotonic (same; always unchanged), medullary

collecting duct is hypertonic (reabsorbing lots of free H2O), JGA is hypotonic (as discussed above)

- Dude jumps into cold lake; what happens to central blood volume + atrial natriuretic peptide (ANP) +

ADH levels? à CBV up (cold à sympathetic activation à alpha-1 agonism peripherally to decrease

surface area of blood vessels to conserve heat à blood forced to core) + ANP up (if CBV up, then right

atrial stretch up; ANP is body’s natural diuretic à causes PCT to decrease Na reabsorption) + ADH

down (baroreceptor at carotid sinus senses greater stretch à has a role not just on HR but also ADH

release)

- Psychogenic polydipsia (PP)? à person drinks too much à low serum sodium + low urine osmolality

- Prolactin does what à milk production à acts through JAK/STAT tyrosine kinase

- Oxytocin does what à milk letdown/secretion

- Growth hormone in young à giantism; in adults à acromegaly

- Important points about acromegaly à causes diabetes mellitus (GH causes insulin resistance),

hypertension, carpal tunnel syndrome, arthritis, cardiomyopathy; and yes, prognathism (lantern jaw)

- Growth hormone acts directly at tissues? à USMLE wants you to know it causes liver to increase

insulin-like growth factor 1 (IGF-1), which acts peripherally

- Which hormone counteracts GH à somatostatin à generally acts to shut off other hormone

secretions, and antagonizes the effects of GH (can be Tx for acromegaly)

- What is pegvisomant? à GH receptor antagonist

- Congenital adrenal hyperplasia (CAH) à caused by 21, 11, or 17 hydroxylase deficiency in adrenal

cortex (21 is 90-95% of cases) à autosomal recessive conditions

- 21 hydroxylase deficiency à severe lack of mineralocorticoid and glucocorticoid production by

adrenal à low BP + high K + high DHEA-S (adrenal androgen)

- 11 hydroxylase deficiency à adrenal can still make 11-deoxycorticosterone in zona glomerulosa + 11-

deoxycortisol in zona fasciculata à BP not low (sometimes high) + K not high; DHEA-S still high

- 17 hydroxylase deficiency à can’t convert pregnenolone + progesterone to 17-OH forms à adrenal

gland makes aldosterone in excess but not cortisol or DHEA-S

- What does ACTH do at adrenal gland à upregulates desmolase, which converts cholesterol into

pregnenolone to start adrenal hormone synthesis

MEHLMANMEDICAL.COM 19
MEHLMANMEDICAL.COM

- What does angiotensin II do at the adrenal gland à upregulates aldosterone synthase, converting 11-

deoxycorticosterone into aldosterone

- What is metyrapone testing? à 11-beta hydroxylase inhibitor à can be used in the diagnosis of

adrenal insufficiency or Cushing à re the former, if you give metyrapone, cortisol should go down

normally and ACTH + 11-deoxycorticosterone should go up; if ACTH goes up but 11-

deoxycorticosterone doesn’t à adrenal dysfunction (Addison); if ACTH doesn’t go up, then it’s

secondary hypoadrenalism)

MEHLMANMEDICAL.COM 20
MEHLMANMEDICAL.COM

YouTube
@mehlmanmedical

Instagram
@mehlman_medical

MEHLMANMEDICAL.COM 21
MEHLMANMEDICAL.COM

MEHLMANMEDICAL
HY ENDOCRINE

All material is copyrighted and the property of mehlmanmedical.

Copyright © mehlmanmedical

MEHLMANMEDICAL.COM 22
MEHLMANMEDICAL
HY OBGYN/REPRO
MEHLMANMEDICAL.COM

YouTube
@mehlmanmedical

Instagram
@mehlman_medical

MEHLMANMEDICAL.COM 2
MEHLMANMEDICAL.COM

HY Obgyn/Repro

Purpose of this review is not to be a 600-page obgyn textbook with every detail catered to; the purpose is to increase your

USMLE and Obgyn shelf scores via concise factoid consolidation. If you’re studying for Step 1 and want just pure “repro”

without a lot of the clinical stuff, you can skip down to middle of page 30. But I recommend this whole PDF regardless.

- 32F + not breastfeeding + upper-outer quadrant warm, tender, red non-fluctuant mass +/- fever; Dx?

à answer on Obgyn NBME = mastitis, not breast abscess; the key here is non-fluctuant mass;

abscess is identical presentation but fluctuant. For mastitis, the easier, Step 1 presentation is the

standard red, cracked, fissured nipple in a breastfeeding woman à S. aureus à Tx = continue

breastfeeding through the affected breast; can give oral dicloxacillin (answer on newer Obgyn form)

or cephalexin for mastitis; for abscess, answer = always drain before Abx.

- 32F + recently stopped breastfeeding + temp 99.5F + tender, fluctuant mass in lateral breast + not

warm + not erythematous; Dx? à answer on Obgyn NBME = galactocele (milk retention cyst);

classically subareolar or in lateral breast; Tx on Obgyn NBME is warm compresses (“application of

heat to the area”).

- 31F + gave birth two days ago + exclusively bottle-feeding neonate + breasts are engorged and tender

+ fever of 101F + Sx of dysuria + suprapubic tenderness + urinalysis normal; Dx? à answer on Obgyn

NBME = breast engorgement à every student gets this wrong because it sounds like obvious

infection; learning point is: can present with fever; occasional Sx of dysuria + normal U/A are not

atypical in women.

- 24F + amenorrhea since D&C 13 months ago for postpartum hemorrhage + progestin withdrawal test

shows no withdrawal bleeding; Dx? à answer = Asherman syndrome or “uterine synechiae” on

Obgyn shelf.

- 27F + spontaneous abortion at 10 weeks gestation complicated by postpartum endometritis + sharp

D/C to remove infected material; patient is subsequently at increased risk for what? = answer =

amenorrhea (Asherman syndrome).

- What does progestin withdrawal test mean? à if progestin is given then withdrawn, bleeding should

occur (hormonal stabilization of lining followed by allowing it to slough, akin to forcing a

menstruation); if bleeding occurs, estrogen is not deficient and the Dx is anovulation (PCOS is just

anovulation leading to 11+ cysts bilaterally + hirsutism; anovulation as independent term is same

MEHLMANMEDICAL.COM 3
MEHLMANMEDICAL.COM

mechanism as full-blown PCOS) à if anovulation occurs, there’s no corpus luteum and therefore no

progesterone released à cannot establish endogenous rise + fall of progestin, therefore no

sloughing/menstruation; in contrast, if bleeding does not occur with progestin withdrawal test, either

estrogen is deficient (primary ovarian failure or hypogonadotropic disorder) or the uterus is scarred

(Asherman).

- 18F + no bleeding after progestin withdrawal test; Q asks, if not Tx over ten years, what is patient at

risk for? à answer = osteoporosis (progestin withdrawal result means low estrogen).

- Question shows you a graph where basal body temperature increases ~0.5F mid-cycle and stays at

this higher temp; why? à answer = progesterone (ovulation).

- 45F + she asks about best way to decrease risk of osteoporosis; answer = weight-bearing exercise, not

calcium + vitamin D.

- 72F + already has osteoporosis + Q asks best way to most greatly decrease fracture risk; answer =

going on long walks; wrong answer is swimming / pool exercises (weight-bearing component makes

sense, but actually tricky considering elderly have high falls risk).

- 69F + Caucasian + nulliparous + on beta-blocker + drinks daily + compression fracture of vertebra;

what is strongest predisposing risk factor (family Hx not discussed or listed)? à answer = race; white

race confers higher risk of osteoporosis; wrong answers are alcohol use, beta-blocker, nulliparity,

HTN.

- 42F + 8-month Hx of severe pelvic pain and heavy bleeding during menses + regular periods + two

kids + does not want more kids + husband to get vasectomy soon + no other abnormalities; next best

step? à answer = endometrial ablation.

- 11F + Tanner stage 3 breast and pubic hair; these findings are most predictive of what? à answer =

“menarche is imminent.” USMLE wants you to know that menarche is imminent once girl is Tanner

stage 3. Normal sequence is adrenarche à thelarche à pubarche à menarche.

- 13F + Tanner stage 2 + never had menstruation + brought in by mom concerned about lack of

menstruation; answer = follow-up in 6 months (Tanner stage 2 so menarche is not yet imminent).

- 14F + 4x6cm mass in left breast + slightly tender + vitals normal + aunt died of breast cancer; next

best step? à follow-up in 6 months à virginal breast hypertrophy is normal response to increased

estrogens in adolescence (also seen in males; asked on peds and FM shelves).

MEHLMANMEDICAL.COM 4
MEHLMANMEDICAL.COM

- 23F + 10 weeks gestation + nausea and vomiting for 4 weeks + lost 1.8kg; what is the most likely

adverse effect on the fetus? à answer = “no significant adverse effect.”

- How to Dx hyperemesis gravidarum (HG)? à answer = urinary ketones.

- When does HG present + what’s the mechanism? à 8-10 weeks gestation; an effect of beta-hCG

(levels are highest at 8-10 weeks).

- Biochemical disturbance in HG? à hypokalemic, hypochloremic, metabolic alkalosis (low K, low Cl,

high bicarb); yes, they ask this on Obgyn shelf.

- Tx for HG? à answer = admit to hospital and give parenteral anti-emetic therapy.

- Important drug causing hyperprolactinemia apart from antipsychotics? à metoclopramide à D2

antagonist.

- Amenorrhea in patient with anorexia; why? à decreased GnRH pulsation (hypogonadotropic) à

decreased LH + FSH; Q wants “¯ FHS, ¯ estrogen” as the answer; in contrast, premature ovarian

failure, Turner syndrome, and menopause have “­ FHS, ¯ estrogen” as the answer.

- 28F + tight-fitting sports bra and/or breast trauma; Dx? à fat necrosis (can calcify).

- 36F + rubbery, mobile, painless mass in breast; Dx? à fibroadenoma à first Dx with USS only if age

<30; do USS +/- mammogram if age >30; do FNA next; if confirmed, Tx = surgical excision; should be

noted that guidelines vary (i.e., observe for change, etc.), but excision is definitive. Obgyn shelf will

only ask you for Dx based on presentation.

- Mammogram guidelines? à start age 50 + every two years until age 75.

- 44F + painless unilateral cyst in breast that drains brown serous fluid; Dx? à answer on Surg form 6 =

fibrocystic change; everyone says wtf because, yes, classic presentation is bilateral breast tenderness

in woman 20s-40s that waxes and wanes with menstrual cycle; Tx is supportive (Evening Primrose oil

/ warm bath); histological descriptors can be: sclerosing adenosis; blue dome cysts; apocrine

metaplasia.

- 25F + sharp pain in outer quadrant of right breast + exam shows 2cm tender area in right breast but

no mass found; Dx? à answer = fibrocystic change.

- 47F + breast lump self-palpated + breast USS shows 3cm complex cyst + FNA performed of the cyst

revealing straw-colored fluid + mass still present after aspiration; next best step? à answer = biopsy

of the mass.

MEHLMANMEDICAL.COM 5
MEHLMANMEDICAL.COM

- 45F + unilateral rusty nipple discharge; Dx? à intraductal papilloma until proven otherwise.

- 45F + unilateral rusty nipple discharge + biopsy shows stellate morphology; Dx? à answer = invasive

ductal carcinoma, not intraductal papilloma.

- 45F + mammography shows cluster of microcalcifications in upper-outer quadrant; next best step? à

answer = needle-guided open biopsy (FNA wrong answer) à microcalcifications are ductal carcinoma

in situ (DCIS) until proven otherwise.

- 45F + inverted nipple + greenish discharge; Dx? à mammary ductal ectasia (widening of lactiferous

duct).

- 42F + recurrent miscarriage + SLE; Dx? à antiphospholipid syndrome (lupus anticoagulant) à Obgyn

shelf will ask for “uteroplacental insufficiency” as the answer à Tx with aspirin or heparin; warfarin is

contraindicated in pregnancy (bone abnormalities + bleeding in fetus).

- 45F + SLE + commencing third course of corticosteroids during past 18 months; Q asks what else she

should be given; answer = “alendronate now” à give bisphosphonate to patients commencing

steroids indefinitely, or to patients receiving steroids frequently.

- Intrauterine growth restriction (IUGR) of the fetus; which lifestyle factor most contributory; answer =

smoking, not alcohol à causes decreased placental blood flow à answer = “Doppler ultrasonography

of the umbilical artery.”

- Which fetal parameter most reflective of IUGR? à abdominal circumference; sounds wrong, as you’d

expect perhaps femur length, or biparietal diameter, etc., but answer is abdominal circumference.

- 23F + 33 weeks gestation + FVL mutation + intrauterine female demise; Q asks which vessel the

thrombosis most likely occurred in; answer = uteroplacental artery.

- Female at 24 weeks gestation + HTN + proteinuria; most likely cause for her findings? à answer =

“uteroplacental insufficiency” or “placental dysfunction”; this is the cause of preeclampsia.

- Female at 16 weeks gestation + HTN + proteinuria + fundal height measured at the umbilicus; Dx? à

answer = hydatidiform mole, not preeclampsia; preeclampsia will occur after 20 weeks gestation;

molar pregnancy presents large for gestational age à fundal height at umbilicus is normally reflective

of 20 weeks gestation.

- Uteroplacental insufficiency can cause what issue on the fetal heart tracing? à answer = late

decelerations (fetal hypoxia).

MEHLMANMEDICAL.COM 6
MEHLMANMEDICAL.COM

- What do early, variable, and late decels mean? à early = fetal head compression; variable = cord

compression; late = fetal hypoxia.

- Fetus has HR at 120bpm (NR 110-160), however there’s zero variability; Dx? à answer on Obgyn

NBME = fetal sleep state.

- Fetus has HR at 180bpm, however there’s zero variability; Dx? à answer on Obgyn NBME = maternal

fever.

- What are accelerations? à fetal well-being à rise of ~20bpm lasting ~20 seconds; 2-3 occurences

every 20 minutes.

- What is a biophysical profile? à assesses fetal wellbeing; often done when non-stress test (checking

for accelerations) is non-reactive; five components of biophysical profile (you do not need to have

these memorized for the USMLE; more just be aware that if the vignette mentions qualitative non-

reassurance of any aspect of the biophysical profile, then there is possibly fetal/maternal pathology):

o Non-stress test shows at least two accelerations in 20 minutes.

o Rhythmic breathing episode of >30 seconds in 20 minutes.

o Fetal movements (at least 2 or 3 of the limbs).

o Fetal muscle tone (at least one episode of flexion/extension of the trunk + limbs together).

o Amniotic fluid volume (at least 2cm in vertical axis, or fluid index >5cm).

- 21F + 41 weeks’ gestation + 4cm dilated + variable decels; next best step? à answer on Obygn NBME

= amnioinfusion (wrong answers were external cephalic version, forceps delivery, amniocentesis,

cordocentesis) à can’t attempt delivery if not 10cm dilated + forceps not tried first anyway because

it can cause nerve damage or sternocleidomastoid trauma (vacuum extraction / suction cup delivery

first).

- What is external cephalic version? à transabdominal manipulation of a breech fetus into cephalic

engagement; only performed after 36 weeks, as the fetus can spontaneously engage cephalically

prior.

- What is internal podalic version? à reorienting fetus within the womb during a breech delivery; may

be attempted for transverse and oblique lies when C-section not performed; also used for delivery of

second twins. I’ve never seen this as correct answer on NBME assessment; it just shows up a lot as an

incorrect answer choice, so I’m mentioning it here because students always ask, “what’s that?”

MEHLMANMEDICAL.COM 7
MEHLMANMEDICAL.COM

- 2-day-old neonate + purplish fluctuant mound on scalp + crosses suture lines; Dx? à caput

succedaneum

- Difference between caput succedaneum and cephalohematoma?

o Caput succedaneum is poorly defined soft tissue edema on the scalp; caused by pressure of

fetal scalp against cervix during parturition, leading to transient decreased blood flow and

reactive edema; crosses suture lines; can be purplish in color similar to cephalohematoma

(i.e., don’t use color to distinguish); complications rare; disappears in hours to few days.

o Cephalohematoma is well-defined, localized, fluctuant swelling; caused by subperiosteal

hemorrhage; does not cross suture lines; may be associated with underlying skull fracture,

clotting disorders, jaundice; disappears in weeks to months.

- 32F + G1P0 + third trimester + itchy hives-like eruptions within abdominal striae; Dx + Tx? à answer =

pruritic urticarial papules and plaques of pregnancy (PUPPP); occurs in ~1/200 pregnancies (usually

primigravid); cause is unknown, presents as pruritic hives-like eruption within striae; Tx is with topical

emollients; for severe cases, topical steroids can be given; resolves spontaneously within a week of

delivery.

- 25F + G1P0 + third trimester + itchy palms + soles; Dx + Tx? à answer = intrahepatic cholestasis of

pregnancy (ICP); usually occurs third trimester; pruritis, particularly of palms + soles; diagnosis is

achieved by ordering serum bile acids (elevated); Tx = ursodeoxycholic acid (ursodiol); important to

note that ICP is associated with increased risk of third-trimester spontaneous abortion – i.e., it is

not benign; delivery at 35-37 weeks may be considered; if bile acid levels normal, new literature

suggests waiting until 39 weeks is acceptable.

- 32F + 30 weeks gestation + 10-day Hx of nausea and generalized itching + bilirubin 2.1 mg/dL +

ALT/AST/ALP all normal; Dx? à Obgyn shelf answer = intrahepatic cholestasis of pregnancy; no

mention of palms + soles itching in vignette.

- 36F + G1P0 + 36 weeks gestation + nausea/vomiting + jaundice + high bilirubin + high ALT and AST +

no mention of pruritis of palms/soles; Dx? à answer = acute fatty liver of pregnancy; caused by

deficiency of long-chain 3-hydroxyacyl-CoA dehydrogenase (sounds absurdly pedantic but asked on

Obgyn shelf); often fatal; Tx is IV hydration + hepatology/high-risk obgyn consults + delivery.

MEHLMANMEDICAL.COM 8
MEHLMANMEDICAL.COM

- 29F + G1P0 + 2nd or 3rd trimester + intensily itchy eruption around umbilicus that spreads outward; Dx

+ Tx? à answer = herpes gestationis (gestational pemphigoid); not HSV, but instead an idiopathic

autoimmune phenomenon; Tx = topical steroids.

- 13F + never had menstrual period + morning nausea/vomiting + suprapubic fullness; next best step?

à answer = beta-hCG à can get pregnant before first menstruation; Q also on peds NBME.

- Tx for preeclampsia? à HTN Mx (labetalol, methyldopa, etc.); definitive Tx is delivery.

- Tx for eclampsia? à Mg for seizures; definitive is delivery.

- Tx for HTN emergencies in pregnancy? à just know hydralazine can be used for this purpose.

- Female at 8 weeks gestation + cysts visualized bilaterally on pelvic USS; Dx? à theca-lutein cysts à

benign finding in pregnancy + will almost always naturally regress à increased occurrence in high

beta-hCG states like multiple gestation pregnancy, moles, choriocarcinoma.

- Complete vs partial mole? à complete mole = karyotype of 46; empty egg fertilized by a sperm that

duplicates; bunches of grapes / snowstorm appearance on USS; chance of progression to

choriocarcinoma higher than partial; partial mole = karyotype of 69; fetal parts visible on USS; lesser

chance of progression to choriocarcinoma.

- Anovulation + hirsutism + BMI 27; Dx? à PCOS.

- Anovulation; mechanism USMLE wants? à insulin resistance à causes abnormal GnRH pulsation à

high LH/FSH à LH high enough to precipitate ovulation but follicle not yet adequately primed à no

ovulation (anovulation) à follicle retained as cyst.

- Why hirsutism in anovulation à higher relative LH à more androgen production by theca interna

cells.

- What’s LH do? à Stimulates theca interna cells (females) and Leydig cells (males) to make androgens.

- What’s FSH do? à Stimulates granulosa cells (females) and Sertoli cells (males) to make aromatase;

also primes follicles.

- Best Tx for PCOS? à if high BMI, weight loss first always on USMLE; if they ask for meds and/or

weight loss already tried? à OCPs (if not wanting pregnancy); clomiphene (if wanting pregnancy;

estrogen receptor partial agonist à leads to increased GnRH outflow).

- PCOS increases risk of what à endometrial cancer (unopposed estrogen); insulin resistance also

greater risk of T2DM.

MEHLMANMEDICAL.COM 9
MEHLMANMEDICAL.COM

- 32F + unable to conceive for 3 years + BMI 30 + acanthosis nigricans; Dx? à answer = T2DM (PCOS or

anovulation not listed as answers; wrong answer is “hypercortisolism”) à Q doesn’t mention any

characteristic features such as purple striae, muscle wasting, or central obesity.

- 40F + vasomotor Sx; which hormone to confirm Dx? à answer = high FSH for premature ovarian

failure.

- 28F + Hashimoto thyroiditis + hot flashes for 6 months + high FSH; Dx? à answer = “autoimmune

ovarian failure”; this is a cause of premature ovarian failure (autoimmune diseases go together).

- Thyroid and pregnancy? à TSH normal, T3 normal, free T4 normal, total T4 elevated à due to

increased thyroid-binding globulin due to higher estrogen.

- What do we order to evaluate thyroid function in pregnancy? à always choose free T4 if you are

asked. TSH is for screening in non-pregnant persons. Free T4 can be an answer in non-pregnant

persons if they ask for most definitive marker for thyroid function.

- Levothyroxine dose in pregnancy for those with Hashimoto? à may need to be increased up to 50%.

- Thionamides in pregnancy? à methimazole is teratogenic in first trimester (causes aplasia cutis

congenita); give PTU in first trimester; 2nd trimester onward switch to methimazole (PTU significantly

hepatotoxic + methimazole only teratogenic early in pregnancy).

- 27F + 34 weeks gestation + thyroid storm; Tx? à Obgyn NBME answer = PTU.

- 27F + gave birth to healthy boy 6 months ago following uncomplicated labor + no weight change or

mood disturbance + on no meds + vitals WNL + dry skin + thyroid gland enlarged and non-tender +

TSH high + T4 low; most likely explanation for these findings? à answer = “thyroiditis” à Dx =

postpartum thyroiditis (a type of silent thyroiditis) à characterized by thyrotoxicosis followed by

hypothyroidism (1/3 of women experience both phases; 1/3 experience just hyperthyroid phase; 1/3

only hypothyroid phase); affects 5-10% of women postpartum; hyperthyroid phase usually occurs 1-4

months postpartum; hypothyroid phase occurs about 4-8 months postpartum; thought to be caused

by postpartum immunologic rebound (immune system normally suppressed during pregnancy); Dx w/

Hx + ordering serum TSH; increased risk of progression to Hashimoto; Tx w/ short course of

propranolol if hyperthyroid; give short course of levothyroxine if hypothyroid.

- Neonate born with cretinism; what could have prevented this? à answer = “routine newborn

screening”; yes, on obgyn shelf.

MEHLMANMEDICAL.COM 10
MEHLMANMEDICAL.COM

- 16F + anterior vaginal wall pain and dysuria for 6 months + U/A normal + vitals normal; Dx? à chronic

interstitial cystitis à Tx is supportive; do not choose steroids.

- Important factoids about acute appendicitis in pregnancy? à can be upper right quadrant; if

appendicitis, yes, perform laparascopic appendectomy.

- Beta-hCG in mole vs ectopic? à super-high in mole; low in ectopic (and slow rate of increase).

- 32F + presentation similar to stroke + beta-hCG hundreds of thousands; Dx? à choriocarcinoma

(brain mets); chorio loves to metastasize to lungs.

- 24F + pregnancy visualized in the corneum of the uterus; Dx? à answer = ectopic pregnancy.

- 27F + pregnancy visualized in the parametrium of the uterus; Dx? à answer = ectopic pregnancy.

- Most common location for ectopic? à ampulla of fallopian tubes.

- Most common etiology for ectopic? à Hx of PID à scarring of fallopian tubes.

- Tx for ectopic pregnancy? à laparoscopic removal (salpingostomy / salpingectomy).

- When to give methotrexate to Tx ectopic? à all must be fulfilled: beta-hCG <6,000; < 3 cm in size;

fetal HR not detectable; no evidence of fluid leakage in the cul de sac; mom stable vitals.

- Organisms causing PID + Tx? à chlamydia and/or gonorrhea; Tx = IM ceftriaxone, PLUS either oral

azithromycin or oral doxycycline. If patient is septic (2+ SIRS), answer = admit to hospital and give IV

antibiotic therapy (they make this distinction on Obgyn shelf).

- PID + fever does not improve after several days on Abx; next best step? à adnexal USS to look for

tubo-ovarian abscess à must drain if present.

- Difference between inevitable and threatened abortions? à inevitable = bleeding + open cervix;

threatened = bleeding + closed cervix; Tx for inevitable = vacuum aspiration; Tx for threatened = bed

rest.

- 32F + 9 weeks gestation + bleeding and passage of clots per vaginum + intrauterine pregnancy seen

on USS; Dx? à answer = incomplete abortion (passage of clots means it’s already underway).

- Difference between complete and missed abortions? à Complete = no products of conception seen

on USS (abortion is literally over/complete); missed = fetal demise without passage of products of

conception.

- 35F + vaginal bleeding at 6 weeks gestation and beta-hCG 450 mIU/mL + USS shows thickened

endometrial stripe and no fetal pole + one week later beta-hCG is 90 mIU/mL; next best step? à

MEHLMANMEDICAL.COM 11
MEHLMANMEDICAL.COM

answer = “third measurement of beta-hCG within one week” à Dx here is spontaneous abortion;

must measure beta-hCG weekly until negative; same for gestational trophoblastic disease (moles).

- 43F + bleeding per vaginum + uterus is large and smooth; Q asks for which type of uterine fibroid;

answer = submucosal leiomyomata.

- 43F + no bleeding per vaginum + uterus is globular; which type of fibroid? à answer = subserosal.

- 43F + beefy red mass protruding from the vagina; Dx? à answer = pedunculated submucosal

leiomyomata uteri, not cervical cancer à the latter will often be described as an ulcerated, exophytic

mass.

- 42F + comes in for routine exam + no complaints + large uterus on exam + USS shows various

leiomyomata; next best step? à answer = observation (because asymptomatic); otherwise Tx =

NSAIDs, OCPs.

- 44F + dysmenorrhea + menorrhagia + USS shows large, smooth uterus with no overt masses; Dx? à

answer = adenomyosis (endometrium growing within myometrium); may present similar to

submucosal fibroids, with vaginal bleeding, however uterus is diffusely enlarged and no masses seen

on USS; Tx with NSAIDs, OCPs; leuprolide; definitive is hysterectomy.

- 27F + 30 weeks gestation + weakness of thumb abduction bilaterally; Dx? à carpal tunnel syndrome

(normal in pregnancy).

- 23F + unintended pregnancy + fever of 104F + vaginal discharge + abdo pain + laceration visualized on

cervix; Dx? à septic abortion à she tried to self-abort using, e.g., a hanger.

- 32F + rupture of membranes (ROM) >18 hours + abdo pain + fever; Dx + Tx? à chorioamnionitis; Tx =

ampicillin + gentamicin + clindamycin (amp + gent alone seen as answer on one Obgyn shelf Q).

- 32F + C-section 12 hours ago + abdo pain + fever; Dx + Tx? à postpartum endometritis; Tx =

ampicillin + gentamicin + clindamycin.

- Organism(s) causing chorioamnionitis + endometritis? à answer = polymicrobial.

- 25F + postpartum endometritis + low BP; Dx? à answer = puerperal sepsis; gynecologic infection

starting 1-10 days after parturition leading to sepsis.

- Lump seen at 4 or 8 o’clock position on vulva; Dx + Tx? à Bartholin gland cyst/abscess; Tx = warm

compresses for cyst; drain if abscess.

- Organism(s) causing Bartholin gland abscess? à answer = polymicrobial.

MEHLMANMEDICAL.COM 12
MEHLMANMEDICAL.COM

- 37F + Bartholin gland abscess + Q asks “most serious complication of this condition?” à answer =

necrotizing fasciitis; wrong answer = “gram positive sepsis” (polymicrobial; need not be gram +).

- Grey/whitish patchy/rough area on the vulva or perineum; Dx + Tx? à lichen sclerosus à must do

punch biopsy first to rule out SCC; if confirmed LS, do topical steroids; if SCC, surgically excise.

- SCC of perineum in diabetic; biggest risk factor in this patient? à answer = HPV, not dysglycemia.

- 24F + sharp adnexal pain + no adnexal mass mentioned in vignette + 10-15 mL of serosanguinous fluid

aspirated from the cul de sac; Dx? à ruptured cyst (usually corpus luteal); Tx = supportive.

- 24F + Hx of ovarian cyst + colicky pelvic pain past few weeks + pain has become constant past couple

days + 6x8cm palpable adnexal mass; Dx? à ovarian/adnexal torsion (cyst is a risk factor).

- 24F + Hx of ovarian cyst + intermittent pelvic pain for four hours that has become constant past two

hours + 8x10cm palpable adnexal mass; Dx? à ovarian/adnexal torsion (pain may be weeks or hours).

- 24F + increasingly severe pelvic pain the past couple days + 6x8cm mass palpable in the adnexa; Dx?

à torsion.

- 25F + normal periods + LMP 20 days ago + 5cm mobile mass in right adnexa on examination + slightly

tender to palpation; Dx? à answer = hemorrhagic corpus luteum cyst; wrong answer is

endometrioma (chocolate cyst seen in endometriosis).

- 18F + tampon use + diffuse rash + BP 90/60; Dx? à toxic shock syndrome (S.aureus).

- 24F + 30 weeks gestation + spotting on underwear 12 hours after sexual intercourse + bleeding

gradually increasing since + USS normal; Dx? à answer = cervical trauma.

- 36F + 26 weeks gestation + severe flank pain + feels faint when attempting to urinate; Dx? à

urolithiasis (progesterone slows ureteral peristalsis).

- Mechanism for increased cholesterol gallstones in pregnancy? à progesterone slows biliary

peristalsis + estrogen increased activity of HMG-CoA reductase (compensatory for lowering serum

levels of cholesterol).

- 26F + three first-trimester miscarriages + has single kidney; Q asks most likely reason for recurrent

miscarriage; answer = congenital uterine abnormalities.

- Tx for torsion? à laparoscopic detorsion.

MEHLMANMEDICAL.COM 13
MEHLMANMEDICAL.COM

- 32F + dull right-sided pelvic pain + beta-hCG negative + USS shows simple 5cm cyst; Tx? à answer =

“oral contraceptive therapy and a second pelvic examination in 6 weeks”; the wrong answer is

“reassurance and schedule follow-up examination in 1 year.”

- 23F + extremely painful periods + needs to miss grad school classes sometimes because of the pain +

examination shows no abnormalities; Dx? à answer = primary dysmenorrhea = “prostaglandin

production” = PGF2alpha hypersecretion.

- Above 23F; next best step in Mx? à answer = NSAIDs; pregnancy test is wrong answer.

- 23F + extremely painful periods + needs to miss grad school classes sometimes because of the pain +

examination shows nodularity of the uterosacral ligaments; Dx? à answer = endometriosis. Obgyn

shelf will often omit details such as pain with defecation or dyspareunia because they’re too easy.

- How to Dx endometriosis? à answer = diagnostic laparoscopy.

- 26F + dull pelvic pain + USS shows cystic mass with calcification; Dx? à answer = dermoid cyst

(mature cystic teratoma); details such as “hair, skin, teeth” are too easy for Obgyn shelf.

- 65F + multiple masses “caked” on the omentum; Dx? à ovarian cancer.

- 31F with epilepsy + 10 weeks gestation + has seizure + phenytoin serum level below therapeutic

range; next best step? à answer = increase dose of phenytoin (yes, during pregnancy) à seizure

leads to fetal hypoxia, which is worse case scenario, so must prevent at all costs.

- 31F on valproic acid wanting to get pregnant; what do we do? à stop valproic acid (contraindicated

in pregnancy due to high chance of neural tube defects) à can use other anti-epileptics during

pregnancy instead.

- Hx of many pregnancies + downward movement of vesicourethral junction à stress incontinence à

answer on one Obgyn NBME Q is “decreased external urethral tone.”

- Tx of stress incontinence à pelvic floor exercises (Kegel); if insufficient à mid-urethral sling.

- Hyperactive detrusor or detrusor instability à urge incontinence.

- Need to run to bathroom when sticking key in a door à urge incontinence.

- Incontinence in multiple sclerosis patient or perimenopausal à urge incontinence.

- 52F + hot flashes + urge incontinence; Q asks mechanism; answer = “estrogen deficiency.”

- Tx of urge incontinence à oxybutynin (muscarinic cholinergic antagonist) or mirabegron (beta-3

agonist).

MEHLMANMEDICAL.COM 14
MEHLMANMEDICAL.COM

- Incontinence + high post-void volume (usually 3-400 in question; normal is <50 mL) à overflow

incontinence.

- Incontinence in diabetes à overflow incontinence due to neurogenic bladder.

- Tx for overflow incontinence in diabetes à bethanechol (muscarinic cholinergic agonist).

- Incontinence in BPH à overflow incontinence due to outlet obstruction à eventual neurogenic

bladder.

- What is the only approved indication for hormone-replacement therapy (HRT)? à severe vasomotor

Sx (hot flushes, urge incontinence); HRT is not used for preserving bone density; increases risk of

thromboembolic and cerebrovascular events; estrogen increases fibrinogen and factor VIII levels.

- 57F + blood stains on underwear for 6 months + painful sexual intercourse + atrophic, friable vaginal

mucosa on exam + cervix and bimanual exams normal; Dx + Tx? à atrophic vaginitis à answer =

“hypoestrogenic state” à Tx = lubricants; if insufficient, topical estrogen may be used.

- 25F + currently breastfeeding + menstruation not yet resumed + dyspareunia + erythematous vagina

with no discharge; next best step in Mx? à answer = “recommendation for use of a lubricant” à high

prolactin levels during breastfeeding à hypoestrogenic state à Sx similar to atrophic vaginitis in

menopause.

- HRT increases the risk of what kind of cancer? à answer= breast, not endometrial; greater absolute

amount of estrogen over female’s life increases breast cancer risk; HRT does not increase endometrial

cancer risk; latter is unopposed estrogen as risk factor, which is why HRT is estrogen + progesterone;

only time HRT is given as estrogen only is for women with Hx of hysterectomy.

- 53F + taking HRT past six months + stopped taking progesterone component because she didn’t like

how it affected her moods + vaginal bleeding; next best step? à answer on Obgyn shelf =

endometrial biopsy.

- 53F + started HRT three months ago + normal mammogram when started HRT + now has cyst seen on

ultrasound after self-palpation; next best step? à answer = FNA biopsy of the cyst.

- How do combined oral contraceptive pills affect cancer risk: ¯¯ ovarian (~50% ¯ risk), ¯ endometrial,

« breast; ­ cervical (from decreased barrier protection à ­ HPV infections; not from pill itself).

Some studies have suggested possible increased risk for breast, but no significance.

MEHLMANMEDICAL.COM 15
MEHLMANMEDICAL.COM

- 16F + aunt died of ovarian cancer + asks GP how to screen for ovarian cancer; what is your response?

à answer = no screening, but offer her information about oral contraceptive pills.

- 25F + BRCA mutation confirmed + three first-degree family members with gynecologic cancers; next

best step? à answer = total abdominal hysterectomy and bilateral salpingo-oophorectomy.

- 47F + total abdominal hysterectomy and bilateral salpingo-oophorectomy performed for

leiomyomata uteri; Q asks what we do re Pap smears; answer = “no longer indicated.”

- 22F + T1DM + 33 weeks gestation + fundal height 38cm; Dx? à polyhydramnios (fundal height in cm

should approximately = # of weeks pregnant).

- Neonatal girl with karyotype 46XX + has phallus and scrotum; Q asks mechanism; answer = “ACTH

hypersecretion” à in congenital adrenal hyperplasia caused by 21- and 11-hydroxylase deficiency,

cortisol is low, so ACTH goes up to compensate, leading to cortical hyperplasia; in addition, precursors

are shunted to DHEA-S and androstenedione, leading virilization of newborn.

- 33F + prenatal USS shows two fetuses with thick dividing membrane; what kind of twin pregnancy is

this? à answer = dichorionic diamniotic; thick dividing membrane = two chorions; # of placentae = #

of chorions.

- 33F + prenatal USS shows one fetus much larger than the other; what kind of twin pregnancy is this?

à most likely to be monochorionic monoamnionic in the setting of twin-twin transfusion syndrome,

where one fetus “steals”/siphons nutrients and blood flow from his or her twin.

- 43F + receiving beta-hCG as part of IVF protocol + develops severe abdo pain + ascites; Dx? à answer

= ovarian hyperstimulation syndrome à almost always due to iatrogenic beta-hCG administration;

causes vascular hyperpermeability.

- 21F + requests OCPs + Pap smear is normal; Q asks what else needs to be done; answer = check for

chlamydia à should be noted that whilst Pap smears always start at 21, STI checks are done from age

of sexual onset.

- 33F + regular periods + Hx of multiple sexual partners + unable to conceive with husband for 3 years +

husband has normal semen sample; next best step? à answer = hysterosalpingogram (assess tubal

patency and uterine architecture; possible Hx of PID leading to tubal occlusion (despite no Hx of

ectopic in the patient).

MEHLMANMEDICAL.COM 16
MEHLMANMEDICAL.COM

- 35F + hysterosalpingogram shows spillage of dye into the peritoneal cavity; Dx? à normal finding

(fallopian tubes are normally open at both ends).

- What is uterine didelphys? à uterus develops as paired organ (double uterus) + double cervix +/-

double vagina.

- 52F + presents for routine screening for first time in 4 years; Q asks “in addition to cholesterol

screening, Pap smear, and mammography; what does she need? à answer = colonoscopy. Similar

answers might be influenza vaccine if fall/winter (every year).

- How often are Pap smears indicated, and when are they started and stopped? à commenced at age

21, then every 3 years; starting age 30, can become every 5 years if co-test for HPV; performed until

age 65 (past ten years must be normal findings + no Hx of moderate or severe dysplasia).

- Pap smears in HIV? à at time of diagnosis, then every year.

- Mx of Pap smear result: atypical squamous cells of undetermined significance (ASC-US) à repeat

cytology in a year, OR test for HPV; if positive, do colposcopy + biopsy; if negative, repeat co-testing in

three years.

- Mx of LSIL on Pap smear? à if negative HPV testing, repeat co-testing in one year; if (+) HPV testing

or no testing, do colposcopy + biopsy.

- Mx of high-grade squamous intraepithelial neoplasia (HSIL) on Pap smear? à regardless of HPV

status: immediate loop electrosurgical excision procedure (LEEP), OR colposcopy + biopsy.

- Mx of cervical intraepithelial neoplasia (CIN) I seen on biopsy à immediate LEEP, OR colposcopy +

cytology every 6 months.

- Mx of CIN II/III seen on biopsy à immediate LEEP demonstrating clear margins, then do Pap + HPV

contesting 1 and 2 years postoperatively.

- 57F + vaginal hysterectomy performed for CIN III; next best step? à Obgyn shelf answer = “Pap smear

annually.”

- 32F + colposcopy is performed for LSIL + entire squamocolumnar junction cannot be visualized; next

best step? à answer on Obgyn NBME = cone biopsy.

- 47F + Pap smear shows atypical glandular cells + colposcopy normal + endocervical curettage shows

benign cells; next best step? à Obgyn NBME answer = endometrial biopsy.

MEHLMANMEDICAL.COM 17
MEHLMANMEDICAL.COM

- 35F + two minutes after separation of placenta has shortness of breath + tachycardia + bleeding from

venipuncture sites; Dx? à amniotic fluid embolism; can cause DIC; supportive care.

- 35F + two days after C-section + gets up to go to the bathroom + SoB + tachycardia; Dx? à pulmonary

embolism à heparin followed by spiral CT (if not pregnant) or V/Q scan (if pregnant).

- 39F + pregnant + Sx of pulmonary embolism + V/Q scan performed showing segmental defects; next

best step in Dx? à answer = spiral CT; student says “wait but I thought we don’t do CT in pregnancy.”

Right, we don’t. But if they ask for next best step after V/Q scan, that’s still the answer they want.

- 27F + two days after C-section + temp 100.8F + breath sounds decreased at both lung bases + urinary

catheter specimen is negative + remainder of exam unremarkable; Dx? à answer = atelectasis (most

common cause of fever within 24 hours of surgery (but shelf has two days after C-section for one Q).

- 27F + triad of third-trimester painless bleeding + ROM + fetal bradycardia; Dx? à answer = vasa

previa (fetal vessels overlying the internal cervical os); associated with velamentous cord insertion

(vessels not protected by Wharton jelly).

- 22F + uncomplicated delivery of newborn + heavy vaginal bleeding + placenta shows large, non-

tapering vessel extending to margin of membranes; Dx? à answer = succenturiate placental lobe;

students says wtf? à just need to know sometimes placenta can have auxiliary lobe with connecting

vessels; this is a cause of vasa previa, in addition to velamentous cord insertion.

- 35F + C-section 6 weeks ago + required 3 units of transfused RBCs + 9kg weight loss + has cold

intolerance + could not breastfeed; Dx? à Sheehan syndrome (arrow Q on shelf; answer is ¯ for

prolactin, ACTH, GH, FSH, TSH); should be noted tangentially that on newer NBME for Step 1, Q with

Sheehan syndrome has ­ for aldosterone (not hyperaldosteronism, but higher baseline to

compensate for lower cortisol).

- 15F + never had menstrual period + one-wk Hx of constant, severe pelvic pain + 10-month Hx of

intermittent pelvic pain + BP of 90/50 + bluish bulge in upper vagina; Dx? à hematometra à

imperforate hymen with blood collection in the uterus à vagal response causes low BP à Tx =

cruciate incision of the hymen.

- 15F + never had menstrual period + one-wk Hx of constant, severe pelvic pain + 10-month Hx of

intermittent pelvic pain + BP normal + bluish bulge in upper vagina; Dx? à hematocolpos à blood

MEHLMANMEDICAL.COM 18
MEHLMANMEDICAL.COM

collection in the vaginal canal, but not backed up to the uterus like hematometra à Tx = cruciate

incision of the hymen.

- 27F + delivered newborn 5 days ago + pain in calf with dorsiflexion of foot; next best step in Dx? à

answer = duplex ultrasonography of the calf; positive Homan sign for DVT in hypercoagulable state.

- Down syndrome important testing?

o First trimester screen (11-13 weeks): ¯ pregnancy-associated plasma protein A (PAPPA), ­

beta-hCG, ­ nuchal translucency, hypoplastic nasal bone.

o Second trimester screen (16-18 weeks): ¯ AFP, ­ beta-hCG, ¯ estriol, ­ inhibin-A; in Edward

syndrome, all decreased; Patau is variable.

o Cell-free DNA (as early as 10 weeks).

- Most common cause of abnormal AFP measurement? à answer = dating error.

- 32F + AFP measurement comes back 2.6x upper limit of normal; next best step? à answer = re-

ultrasound; wrong answer = perform AFP measurement again à need to simply do ultrasound to

reapproximate dates.

- Important locations for the “celes”:

o Cystocele: anterior superior vaginal wall.

o Urethrocele: anterior inferior vaginal wall.

o Enterocele: posterior superior vaginal wall (Q on shelf says “high on posterior vaginal wall;

another Q says the patient can feel movement within her vagina à weird, but presumably

gut peristalsis).

o Rectocele: posterior inferior vaginal wall.

- 32F + protrusion of distal urethra through urethral meatus; Dx? à urethral prolapse; sounds

reasonable, but don’t confuse with stress incontinence; the latter will sometimes be described as

“downward mobility of vesicourethral junction with Valsalva” (not urethral prolapse).

- 22F + 24 weeks gestation + fundal height 20cm + no cervix palpated + examination shows fetus in

breech position in vagina; Dx? à cervical incompetence; Tx w/ cervical cerclage; notable risk factor is

prior conization.

- 30F + 37 weeks gestation + fetus in breech position; during labor, risk of which complication is

greatest? à answer = cord prolapse.

MEHLMANMEDICAL.COM 19
MEHLMANMEDICAL.COM

- 32F + 14 weeks gestation + Hx of two LEEP + cervix flush against upper vagina and measures 2cm in

diameter + pelvic USS shows funneled lower uterine segment; Dx? à cervical incompetence à

“funnel” means cervical incompetence (“cervical funneling” / “funneled lower uterine segment”).

- 87F + partial prolapse of uterine cervix through the introitus + uterus can easily be pushed back into

the uterus; next best step? à answer = vaginal pessary.

- Stages of labor:

o Stage 1 latent: 0-6cm cervical dilation (old guidelines: 0-4cm)

o Stage 1 active: 6-10cm (complete) cervical dilation. (old guidelines: 4-10cm)

o Stage 2: 10cm (complete) cervical dilation to delivery of fetus.

o Stage 3: delivery of fetus to delivery of placenta.

o Obgyn NBME has Q where 32F has been at 5cm dilation for past 4 hours; answer = “arrest of

active phase”; the wrong answer is “protracted latent phase.”

- What is definition of protracted latent phase? à dilating <1-2cm per hour, which reflects the 95%tile

in contemporary women. Women <6cm are in latent phase; regardless of parity, may take 6-7 hours

to progress from 4-5cm, and 3-4 hours to progress from 5-6cm.

- What does “arrest of active phase” mean? à no cervical change in >4 hours despite adequate

contractions (>200 Montevideo unites [MVU]), or >6 hours if contractions inadequate.

- 28F + 38 weeks gestation + cervix completely dilated + strong contractions + fetal station remains

unchanged over next hour; Dx? à answer = cephalopelvic disproportion (baby too big for pelvis).

- 5F + foul-smelling yellow vaginal discharge + blood spotting on underpants + no dysuria + mild vulvar

erythema seen on exam; Dx? à answer = vaginal foreign body, not sexual abuse; presumably sexual

abuse there would be lacerations or trauma seen on physical exam.

- 82F + Alzheimer + brought in by daughter for blood on underwear + 3cm vaginal laceration +

erythematous, edematous perineal body; Dx? à answer = sexual assault.

- 23F + dysuria + bacteriuria + pyuria; Q asks how to decrease future episodes; answer = “voiding

immediately after coitus.”

- 23F + three UTIs over past year + Hx of UTIs being Tx successfully with TMP-SMX; Q asks for most

appropriate med for daily UTI prophylaxis; answer = TMP-SMX; slightly unusual question, but it’s on

the Obygn NBME.

MEHLMANMEDICAL.COM 20
MEHLMANMEDICAL.COM

- 37F + dysuria + urinalysis shows 20-50 WBCs/hpf + one week of TMP-SMX does not improve Sx; next

best step? à answer = urethral culture for chlamydia à if patient doesn’t improve with Tx of UTI,

check for STIs.

- 20F + 40 weeks gestation + epidural catheter placed + lidocaine and epinephrine injected + develops

metallic taste in mouth; Dx? à answer = “intravascular injection of anesthetic.”

- 25F + 5 weeks post-delivery + insomnia + irritable + finds baby’s cry annoying and leaves him in crib

crying for long periods of time; next best step? à answer = “arrange for immediate psychiatric

evaluation” à post-partum depression; Tx = sertraline (SSRI) and CBT; if mania, delusions, or

hallucinations à post-partum psychosis; if more mild + within 7-10 days of delivery à post-partum

blues.

- 25F + 42 weeks gestation + oligohydramnios + cervix long, closed, and posterior; next best step? à

answer = “administer a prostaglandin”; wrong answer is amnioinfusion (do for variable decelerations

with ROM).

- 34F + pregnant + low serum iron and ferritin + microcytic anemia + proceeds to take iron for three

weeks + three weeks later, iron and ferritin are normal but still has microcytic anemia; next best step

in Mx? à answer = “hemoglobin electrophoresis”; Dx is thalassemia (alpha trait usually, as this is

asymptomatic + picked up in pregnancy) à microcytic anemia non-responsive to iron

supplementation; Hb electrophoresis will show presence of HbA2.

- 28F + 7 weeks gestation + started taking prenatal vitamin 3 weeks ago + microcytic anemia; next best

step? à answer = hemoglobin electrophoresis; same as above, the implication is that the

supplement contains iron + she is possibly non-responsive to it à thalassemia.

- 28F + African American + 7 weeks gestation + microcytic anemia + Hb electrophoresis shows 95%

HbA1; Dx? à answer on Obgyn shelf = iron deficiency anemia; thalassemia would show HbA2.

- 28F + pregnant + MCV 87 + Hb 10.5 g/dL; Dx? à answer = physiologic dilution of pregnancy à Hb

drop to 10.5 g/dL is normal finding.

- “What about platelets in pregnancy?” à reduction normal; gestational thrombocytopenia is the Dx

when level drops to <150,000 per uL.

MEHLMANMEDICAL.COM 21
MEHLMANMEDICAL.COM

- 24F + immune thrombocytopenic purpura (ITP); Q asks the potential effect on the fetus à answer =

“fetal platelet destruction”; maternal IgG against her own platelet GpIIb/IIIa can cross placenta,

attacking the fetal platelets. This is on new Obgyn form.

- 20F + 42 weeks gestation + shoulder dystocia + neonate born with arm pronated, adducted, and

internally rotated; Dx? à “injury to the 5th and 6th cervical nerve roots” (Erb-Duchenne palsy).

- Most common cause of postpartum bleeding? à uterine atony (hypocontractile uterus).

- Tx for uterine atony? à uterine massage first, followed by oxytocin, then ergonovine.

- 33F + postpartum bleeding despite uterine massage and oxytocin; next best step? à answer =

ergonovine therapy (do not give in HTN).

- Diabetic mom giving birth + shoulder dystocia + McRoberts maneuver implemented; what is notable

risk to the fetus here? à answer = clavicular fracture (anterior shoulder caught behind pubic

symphysis à McRoberts maneuver is flexing mom’s hips + applying suprapubic pressure à clavicular

fracture not uncommon).

- Diabetic mom giving birth + shoulder dystocia + McRoberts maneuver implemented + postpartum

bleeding + uterus is firm on palpation; most likely cause of bleeding? à answer on Obgyn shelf =

vaginal laceration, not uterine atony.

- 34F + delivers term neonate + placenta delivers after gentle cord traction + now has moderate vaginal

bleeding + HR 60 + BP 60/40 + IV saline doesn’t help + uterus cannot be palpated on physical exam;

Dx? à answer = uterine inversion.

- Episiotomy performed posterior in the midline; what does the obstetrician cut into if he cuts too far?

à answer = external anal sphincter.

- 37F + 40 weeks gestation + Hx of C-section + constant, sharp abdominal pain + maternal vitals all

normal + fetal late decels + “Leopold maneuvers show fetal small parts above the fundus”; Dx? à

answer = uterine rupture.

- 37F + 40 weeks gestation + oxytocin administered + robust contractions occurring every two minutes

+ abdo pain + hypotension + fetal head palpated in RUQ; Dx? à uterine rupture.

- What are tachysystole and uterine hypertonus? à tachysystole is >5 contractions every ten minutes;

uterine hypertonus is a sustained contraction >2 minutes.

MEHLMANMEDICAL.COM 22
MEHLMANMEDICAL.COM

- What are Leopold maneuvers? à abdominal palpatory maneuvers used to determine the position

and lie of the fetus.

- 62F + ovarian mass + bleeding per vaginum + endometrial biopsy shows atypical complex hyperplasia;

Q asks for which ovarian cancer is the Dx? à answer = granulosa cell tumor à unopposed estrogen

à endometrial hyperplasia à endometrial cancer risk.

- 47F + 9-month Hx of irregular periods where they occur at 2-3-month intervals + endometrial biopsy

shows proliferative endometrium; next best step? à answer on shelf = “cyclic progestin therapy” à

control irregular menses and prevent endometrial hyperplasia.

- 32F + menometrorrhagia + LMP 2 weeks ago + periods 28-30-day intervals + just started taking OCPs

for Tx; what is the most likely explanation for improvement in patient’s bleeding? à answer =

“synchronization of endometrium.”

- 27F + G3P2 + Rh negative + received RhoGAM both prior pregnancies + arrives now at first prenatal

visit for third pregnancy; next best step? à Obgyn shelf answer = “indirect antiglobulin (Coombs)

test” à must see if she’s developed antibodies to Rh antigen.

- 29F + G1P0 + O+ blood type + fetus is A or B blood + goes on to develops pathologic jaundice

postpartum; Dx? à hemolytic disease of the newborn (ABO type) à mothers with O blood type will

have fractional IgG (instead of IgM) against A and B antigens à cross placenta à fetal hemolysis à

severity highly variable; Obgyn shelf will always give first pregnancy and an O+ mom so that student

can’t accidentally get lucky with the Dx if he/she only knows about Rh type hemolytic disease of the

newborn.

- 29F + G2P1 + Rh negative + fetus experiences hydrops; Dx? à hemolytic disease of the newborn (Rh

type) à presumably mother made antibodies against fetal Rh antigen from prior pregnancy following

mixing of circulations.

- When to give RhoGAM? à normally at 28 weeks gestation + again at parturition; also give for

spontaneous or instrumental abortions + procedures (e.g., amniocentesis) + trauma/insults (e.g.,

abruptio placentae).

- 34F + G3P2 + Rh negative + all pregnancies with same male partner + indirect Coombs test positive for

anti-Kell antigens at titer of 1:256; next best step? à answer = “Kell typing of the father’s blood”;

implication is mom is Kell negative but prior fetus(es) Kell positive; fetal blood must have entered

MEHLMANMEDICAL.COM 23
MEHLMANMEDICAL.COM

maternal blood during prior pregnancy, however mom has no titers against Rh, just Kell, because

RhoGAM was presumably given.

- Painful third-trimester bleeding following MVA or cocaine use; Dx? à abruptio placentae.

- Painless third-trimester bleeding; Dx until proven otherwise? à placenta previa à placental

implantation site can spontaneously move off the internal os before 36 weeks, so don’t plan for

Caesar before then.

- Postpartum hemorrhage due to placental issue; Dx? à placenta accreta/increta/percreta.

- 21F + recently took Abx + red vaginal introitus and itching + cervical and vaginal discharge are normal

+ KOH prep and wet mount show no abnormalities; Dx? à answer on Obgyn NBME = vaginal

candidiasis (thick white discharge is otherwise classic). Tx = topical nystatin or oral fluconazole.

- 67F + T2DM + vaginal candidiasis Tx with topical miconazole + doesn’t respond to Tx; Q asks why;

answer = T2DM.

- 21F + mucopurulent discharge + no organisms grow; Dx? à chlamydia à oral azithromycin or

doxycycline. Azithromycin is ideal because it’s one-off stat oral dose; doxy is BID for a week.

- 21F + mucopurulent discharge + gram negative diplococci; Dx? à gonorrhea à cotreat for chlamydia

à IM ceftriaxone + oral azithro, OR IM cefixime + oral azithro.

- 21F + erythematous cervix + yellow/green discharge + wet mount confirms Dx; Dx? à trichomoniasis

(flagellated protozoa) à Tx = topical metronidazole for patient and partner.

- 21F + erythematous vaginal canal + thin, watery discharge + wet mount confirms Dx; Dx? à bacterial

vaginosis (Gardnerella vaginalis) à met mount shows clue cells (squamous cells covered in bacteria)

à Tx = topical metronidazole.

- 21F + thin, grey discharge + KOH prep Whiff test is performed yielding fishy odor; Dx? à bacterial

vaginosis.

- 21F + VDRL positive at titer of 1:4 + physical exam shows no abnormalities + complains of no Sx +

chlamydia and gonorrhea testing negative; next best step? à answer = Obgyn shelf answer =

fluorescent treponema antibody (syphilis).

- 19F + painless vulvar ulcer + rapid plasmin reagin negative + all other tests negative; next best step?

à Obgyn NBME answer = repeat rapid plasma reagin (slightly unusual answer, but can sometimes be

negative early in primary syphilis).

MEHLMANMEDICAL.COM 24
MEHLMANMEDICAL.COM

- 21F + one-week Hx of 0.25-cm crusty, painless papule on the posterior fourchette; Dx? à

condylomata acuminata à HPV6+11.

- 22F + soft pink papillary lesions on labia minora and posterior fourchette; Tx? à answer on obgyn

NBME = podophyllum resin; student says wtf? à used to treat warts.

- Gardasil HPV vaccine protects against which types? à 6, 11, 16, 18 (6+11 warts; 16+18 SCC).

- 24F + recently went backpacking in Asia + painful vulvar crater + gram (-) rods cultured; Dx + Tx? à

answer = chancroid (haemophilus ducreyi); Tx with azithromycin.

- 35F + G1P0 + exposed to child with chickenpox + never been vaccinated against VZV; next best step?

à administer VZV IVIG within 96 hours (to be most effective, but still advised up to 10 days post-

exposure).

- When is VZV IVIG advised for neonates? à maternal active lesions between 5 days prior to and 2

days post-delivery.

- Neonate born with patent ductus arteriosus; what Sx did the mom have while pregnant? à answer =

arthritis, not rash; Dx is congenital rubella syndrome in the neonate (causes PDA).

- 25F + 22 weeks gestation + develops low-grade fever and rash + fetus develops hydrops; Dx? à

maternal infection with parovirus B19.

- 21F + painful vesicles on vulva; do we give oral or topical acyclovir? à answer = HSV à always oral if

asked.

- Herpes and pregnancy? à acyclovir indicated to reduce chance of active lesions at time of labor; if

active lesions or prodromal Sx present at parturition, C-section is indicated; acyclovir is safe during

pregnancy.

- HIV and pregnancy? à most important USMLE point is HAART therapy during pregnancy is more

important than not breastfeeding in terms of decreasing vertical transmission; sounds strange, as the

virus is literally in breastmilk, but the answer is HAART therapy to decrease viral load is most

important to prevent vertical transmission; in addition, administer zidovudine to mom prior to C-

section, then zidovudine within 12 hours to neonate post-delivery (latter Q on peds NBME).

MEHLMANMEDICAL.COM 25
MEHLMANMEDICAL.COM

- Hepatitis B and pregnancy? à if mom HepB +, give both HBIG + vaccine within 12 hours of birth; if

mom HepB negative, give just vaccine within 12 hours of birth; if mom status unknown, give vaccine

within 12 hours of birth, and give HBIG within 7 days if mom’s test comes back + or remains unknown.

- 27F + 14 weeks gestation + not immune to HepB; next best step? à answer = vaccinate to HepB now.

- Influenza and pregnancy? à safe to give IM killed vaccine during pregnancy (in fall or winter).

- MMR vaccine and pregnancy? à vaccinate before pregnancy; do not give during pregnancy.

- TB and pregnancy? à Tx for latent and active TB, yes; for active, Tx with RIPE for 2 months, followed

by RI for 7 more months (9 months total); if not pregnant, RI is only given for 4 more months.

- Breastfeeding and OCPs? à Obgyn shelf wants you to know that estrogen-containing contraception

decreases protein content of breastmilk; also linked to lower milk supply + shorter duration of

breastfeeding; contraindicated < 6 weeks postpartum; if hormonal contraception used, progestin-only

recommended.

- How to differentiate between androgen insensitivity syndrome and Mullerian (paramesonephric duct)

agenesis? à both phenotypically female teenagers with normal Tanner stage development; both

have vagina that ends in blind pouch; the clinical difference is that in androgen insensitivity

syndrome, they will say absent or sparse pubic and axillary hair; in Mullerian agenesis, the hair

pattern will be normal, or they’ll even explicitly say “coarse” pubic and axillary hair. If androgen

insensitivity syndrome suspected, next best step = karyotyping (46XY); Mullerian agenesis is 46XX.

- 16F + never had menstrual period + 5’9” + sparse pubic and axillary hair; Dx? à AIS à pointing out

that the Q will say “a 16-year-old girl comes in,” but karyotypically the patient is still a male.

- 12F + 1-year Hx of progressive hair growth and acne + 2-cm vaginal canal + significant clitoromegaly +

posterior labioscrotal fusion + no cervix or palpable uterus; Dx? à 5-alpha-reductase deficiency à

“phallus at age 12” (i.e., penis at age 12, since surge of testosterone at puberty yields significant DHT

production despite deficient enzyme); Obgyn shelf will merely ask for the karyotype here; answer =

46XY (i.e., male, even though stem will say “12-year-old girl”).

- 17F + never had menstrual period + high FSH + absent breast development + scant pubic hair; next

best step? à answer = karyotyping (Turner syndrome).

- 15F + Tanner stage 2 + 4’11” + bone age is equal to chronologic age; answer = karyotyping (Turner).

MEHLMANMEDICAL.COM 26
MEHLMANMEDICAL.COM

- 37F + C-section two days ago + incision site erythematous + abdomen tender + vitals normal + two

palpable lymph nodes in groin; Dx? à answer = “normal postoperative course.”

- 37F + vaginal bleeding + hydroureter; Q asks for what kind of cancer; answer = cervical SCC

(impingement on the ureter).

- When are OCPs contraindicated? à smokers over 35; migraine with aura; HTN (>160/100); current

or past venous thromboemboli; thrombotic disorder (i.e., prothrombin mutation, FVL);

cerebrovascular event; ischemic heart disease; current breast cancer; liver tumor; among others;

Obgyn shelf will ask which is contraindicated, and the answer is “triphasic oral contraceptives” (same

thing as OCP).

- 18F + menstrual cycles with 14-40-day intervals + beta-hCG negative; next best step? à answer =

“cyclic progesterone therapy” à means OCPs, but this is shelf wording.

- What is most effective form of emergency contraception? à answer = copper IUD; second-best is

ulipristal (selective progesterone-receptor modulator; SPRM).

- 31F + copper IUD in place + pelvic exam shows enlarged uterus + USS shows 4cm fibroid; next best

step? à answer = “leave the IUD in place but inform the patient that the leiomyoma may cause

heavier menses.”

- Important points about Depo vs Implanon? à Depo is progestin injection that is effective for three

months; it can cause decreased bone density; Implanon is a progestin implant contraceptive that is

effective for three years; it is associated with erratic periods.

- Type of cancer patient is at increased risk for if commencing Depo? à answer on Obgyn shelf =

breast.

- Important contraindication to IUD? à active STI/PID or Hx of infection within past 3 months; current

pregnancy (obvious); Hx of gynecologic malignancy.

- 42F + HTN managed with meds + often forgets to take meds + wants contraception; what is most

appropriate recommendation? à answer = levonorgestrel IUD (for patients with poor pharmacologic

adherence).

- 27F + Hx of difficulty remembering to take daily meds + wants contraception + Tx for chlamydia three

months ago; Q asks most appropriate form of contraception; answer = “Depo medroxyprogesterone”;

IUD not ideal because of Hx of infection past three months.

MEHLMANMEDICAL.COM 27
MEHLMANMEDICAL.COM

- 68F + Hx of breast cancer + paresthesias bilaterally in legs; next best step? à steroids first for

possible spinal mets (decrease inflammation); then do MRI of spine.

- 28F + G2P1 + 10 weeks gestation + prior pregnancy resulted in neonate of 4540 grams; Q asks what

she’s at increased risk for during current pregnancy; answer = gestational diabetes.

- When to screen for gestational diabetes (GD) for normal risk women? à 24-28 weeks gestation.

- How is most screening for GD carried out?

o First do 50-gram oral glucose tolerance test (OGTT); if serum glucose >140mg/dL at 1 hour,

proceed to 75- or 100-gram diagnostic OGTT.

o For 75- and 100-gram OGTT, GD is diagnosed if 2 or more of the following are met:

§ >95 mg/dL fasting

§ >180 mg/dL at one hour

§ >155 mg/dL at two hours

§ >140 mg/dL at three hours (only applies to 100-gram test)

- How to manage gestational diabetes? à manage with insulin (easier to adjust at labor).

- 28F diabetic + 37-weeks gestation + delivers neonate with neonatal respiratory distress syndrome

(NRDS) + macrosomia (>4000 grams); Q asks which hormone in the serum of the fetus is responsible;

answer = insulin à inhibits surfactant production; should be noted that insulin does not cross the

placenta; fetus produces more endogenous insulin with maternal diabetes.

- Mechanism for NRDS? à decreased surfactant production à decreased lecithin/sphingomyelin ratio;

lecithin is aka dipalmitoyl phosphatidylcholine.

- 37F + 33 weeks gestation + C-section scheduled in 12 hours + bolus of steroids given 12 hours ago;

next best step? à answer = give bolus of steroids; two boluses of steroids must be given within 24

hours of delivery <34 weeks.

- When to give steroids and magnesium prior to delivery? à steroids before 34 weeks (two boluses); if

34 0/7 – 36 6/7 weeks, give one bolus of steroids; add magnesium if before 32 weeks.

- When are tocolytics used? à <34 weeks gestation if delivery would result in premature birth (i.e., do

not use after 34 weeks); only able to delay birth up to a few days; terbutaline (beta-1/-2 agonist),

ritodrine (beta-2 agonist), and nifedipine frequently used; notably effective in helping expectant

MEHLMANMEDICAL.COM 28
MEHLMANMEDICAL.COM

mother to receive two boluses of corticosteroids in the 24-hour period prior to <34-week delivery;

various contraindications, including infection, IUGR, and cervical dilation >4cm.

- What are Braxton-Hicks contractions à irregular, spontaneous contractions sometimes felt in third

trimester; they are normal and benign; in contrast, labor presents are regular and increasingly

sustained contractions.

- When to give GBS prophylaxis?

o Hx of prior pregnancy with early-onset GBS disease in neonate (i.e., pneumonia, meningitis,

sepsis); do not give if prior pregnancy demonstrated mere colonization of GBS.

o GBS bacteriuria at any point during current pregnancy (e.g., first trimester), even if treated

successfully.

o Positive rectovaginal swab at 36 weeks.

o If maternal status is unknown, give if one or more of the following:

§ Maternal fever >38C.

§ ROM >18 hours.

§ Preterm delivery (<37 weeks).

o Successful GBS prophylaxis is IV penicillin or ampicillin within 4 hours of delivery of fetus;

oral amoxicillin/clavulanate (Augmentin) is the wrong answer.

- “Can you explain that annoying Bishop score stuff real quick?”

o 5 criteria summing to 13 points; higher is better; >8 indicates likely successful vaginal

delivery; <6 suggests cervical ripening may be required.

o USMLE will not make you calculate, don’t worry. But students sometimes ask about this.

o Cervical position: Posterior – 0 points; Middle – 1 point; Anterior – 2 points.

§ Becomes more anterior as labor nears.

o Cervical consistency: Firm – 0 points; Medium – 1 point; Soft – 2 points.

§ More rigid and resistant to stretch in primigravid women.

o Cervical effacement: 0-30% – 0 points; 30-50% – 1 point; 50-70% – 2 points; >70% – 3 points.

§ How “thin” the cervix is; normally cervix is 3cm long; becomes “paper-thin” when

fully effaced.

o Cervical dilation: Closed 0 points; 1-2cm – 1 point; 2-4cm – 2 points; >4cm – 3 points.

MEHLMANMEDICAL.COM 29
MEHLMANMEDICAL.COM

§ Most important indicator of progression through first stage of labor.

o Fetal station: -3 – 0 points; -2 – 1 point; -1, 0 – 2 points; +1, +2 – 3 points.

§ Fetal head position relative to ischial spines (usually 3-4cm intravaginal and non-

palpable); - numbers mean the fetal head is above the ischial spines; + numbers

mean head has descended below the ischial spines for impending delivery.

- “Oh yeah can you quickly explain the fetal fibronectin test?” à fetal fibronectin (fFN) is the “glue”

found between the chorion and decidua; if a woman is 22-35 weeks gestation and having symptoms

of preterm labor, fFN test predicts whether preterm labor is likely; if negative, <5% chance of delivery

within next two weeks; if positive, preterm labor likely.

- 28F + 33 weeks’ gestation + clear fluid leaking from vagina past two days + no contractions or

bleeding; next best step? à answer = sterile speculum exam; likely preterm premature rupture of

membranes (PPROM); wrong answers are fetal fibronectin test (only if premature labor /

contractions).

For those of you studying for Step 1 (although you should read above parts of PDF regardless):

- “What do I need to know about embryologic development (i.e., # of weeks certain things develop,

etc.)?”

o At 2 weeks’ gestation, the embryo has 2 layers (epiblast + hypoblast).

o At 3 weeks, neural tube development has commenced. This is completed by week 4.

o Between 3-8 weeks, most organogenesis is occurring. Fetal heart beat doesn’t commence

until week 4.

o What this means for USMLE: the range of 3-4ish weeks is when the fetus is most susceptible

to neural tube defects (i.e., spina bifida) if there is folate deficiency, or exposure to drugs

such as valproic acid or other anti-epileptics (which cause folate malabsorption). In addition,

if they Q asks you when the fetus is most susceptible to teratogens in general, select the

answer that is 3-4 weeks as priority; if that tight range isn’t listed, select the broader one

that encompasses it, e.g., 3-8 weeks. This is all over NBME exams.

- “What do I need to know about which bodily structures/organs originating from certain germ layers,

i.e., ectoderm, etc.?”

o Most embryologic derivative memorization is nonsense, especially now that Step 1 is P/F.

MEHLMANMEDICAL.COM 30
MEHLMANMEDICAL.COM

o A good rule of thumb is: if you literally have no idea on a USMLE question what the answer

is, neural crest is usually correct. I’d say this is the case in at least 3/4 of questions.

o “Failure of neural crest migration” is answer for heart defects due to DiGeorge syndrome and

fetal alcohol syndrome. It’s also answer for Hirschsprung disease.

o Additionally, you should be aware that pheochromocytoma, melanoma, and

neurofibromatosis I/II are neural crest.

o Craniopharyngioma = derived from Rathke pouch, which is the “roof of the primitive

pharynx”; considered ectoderm.

o Thyroglossal duct cyst = derived from “endoderm of foramen cecum”; the latter is the base

of the tongue; in other words, the thyroid gland starts off embryologically at base of tongue

and descends.

o CAP = Clefts, Arches, Pouches; clefts (aka grooves) become ectoderm; arches become

mesoderm; pouches become endoderm.

o Ectoderm à highest-yield structures are: skin + anal canal below pectinate line.

o Mesoderm à highest-yield structures are: muscle + connective tissue; spleen + kidneys.

o Endoderm à esophagus + lining of GI tract until the pectinate line; parathyroids + thymus.

o 3rd + 4th pharyngeal pouches are highest yield of the CAP on USMLE:

§ 3rd pouch = the two inferior parathyroids + thymus (they form a triangle; so 3).

§ 4th pouch = the two superior parathyroids.

§ Agenesis in DiGeorge syndrome. USMLE can also ask about, e.g., a missing

parathyroid gland, or a parathyroid adenoma, and you need to know whether it’s

the 3rd or 4th. It’s not hard, but you need to know these structures.

o Pharyngeal arch #1 à things innervated by CN V (muscles of mastication).

o Pharyngeal arch #2 à things innervated by CN VII (muscles of facial expression, stapedius,

platysma).

o Pharyngeal arch #3 à things innervated by CN IX (stylopharyngeus).

o Pharyngeal arch #4 à things innervated by superior laryngeal nerve branch of CN X

(cricothyroid).

MEHLMANMEDICAL.COM 31
MEHLMANMEDICAL.COM

o Pharyngeal arch #6 à things innervated by recurrent laryngeal nerve branch of CN X

(laryngeal muscles, but not cricothyroid). 5th arch has no major contributions.

o Back in the numerical Step 1 days, memorizing every structure had utility when our aim was

to get a 280+. But now that the exam is Pass/Fail, the above is literally enough to get the vast

majority of embryo. Remember, the point here is yieldness, not superfluousness.

o HY endoderm stuff regarding foregut, midgut, hindgut, I discuss in the Gastro PDF, but this

stuff is so HY that I will again quickly mention HY points here:

§ Foregut à supplied by Celiac trunk (T12); spans esophagus to 1st part of duodenum.

§ Midgut à supplied by SMA (L1); spans from 2nd part of duodenum to distal 2/3 of

transverse colon.

§ Hindgut à supplied by IMA (L3); spans from last third of transverse colon to the

anal canal at the pectinate line.

§ L2 (between the SMA and IMA, clearly) à renal arteries and gonadal arteries

(testicular arteries in males; ovarian arteries in females) come off here.

§ Weird factoid USMLE likes: “Which organ is supplied by an artery of the foregut but

is not itself derived from the foregut” à answer = spleen; supplied by Celiac trunk

but derived from midgut).

- “What do I need to know about terms like malformation vs deformation, etc.?”

o Malformation = developmental defect as a result of genetics or teratogens.

§ Example is cleft lip (polygenic) or heart defect (polygenic or teratogen).

o Deformation = developmental defect as a result of extrinsic/mechanical force.

§ Example is clubbed feet in Potter sequence (due to oligohydramnios).

o Disruption = distortion of previously normal tissue (i.e., originally formed OK).

§ Example is amniotic band syndrome (fibrous bands in amniotic sack compress limbs

of the fetus).

- “What are specific HY teratogens I need to know for USMLE?”

o Lithium à Ebstein anomaly (“atrialization of right ventricle” à the right ventricle is tiny and

right atrium massive).

MEHLMANMEDICAL.COM 32
MEHLMANMEDICAL.COM

o Anti-epileptics à valproic acid, phenytoin, and carbamazepine are all known to cause neural

tube defects, but especially valproic acid.

o Isotretinoin à high-dose vitamin A used for acne that can cause cleft lip/palate in neonate;

USMLE cares less about “what” isotretinoin causes, and more just that you know b-hCG

needs to be done in any female commencing it.

o Warfarin à bone anomalies or bleeding in fetus.

o Fluoroquinolones (e.g., ciprofloxacin) à cartilage abnormalities.

o ACE inhibitors à renal defects.

o Aminoglycosides à congenital hearing loss.

o Diethylstilbestrol (DES) à vaginal clear cell carcinoma.

o Alcohol à fetal alcohol syndrome; most common cause of mental retardation; philtrum

changes are highest yield (i.e., long, smooth philtrum); hypertelorism; heart/lung defects.

o Cocaine and smoking à intrauterine growth restriction (IUGR) due to reduced blood flow.

- “What do I need to know about placental anatomy?”

o Two components: fetal (trophoblast) and maternal (decidua).

§ Trophoblast = syncytiotrophoblast (deep layer) + cytotrophoblast (superficial layer).

§ Syncytiotrophoblast secretes hCG, which functions to maintain the corpus luteum

(ruptured follicle). The corpus luteum secretes progesterone in order to maintain

the endometrial lining / pregnancy. At 8-10 weeks, hCG peaks. This is because after

this point, the placenta takes over production of progesterone, so we no longer

need hCG to maintain the corpus luteum.

§ Fetal and maternal circulations do not mix and merely exchange gas and nutrients

across placenta. Fetal hemoglobin (alpha-2 gamma-2) has stronger affinity for

oxygen and can pull it off of the maternal hemoglobin (alpha-2 beta-2) despite

membrane separation.

§ IgG from the mom can cross placenta; IgA is passed through breast milk.

- “What do I need to know about poly- vs oligohydramnios?”

o We want to think of fetal versus maternal causes.

MEHLMANMEDICAL.COM 33
MEHLMANMEDICAL.COM

o Polyhydramnios à maternal diabetes (insulin does not cross placenta; high glucose crosses

placenta, leading to polyuria in fetus); fetal anencephaly or tracheoesophageal fistula.

o Oligohydramnios à maternal smoking / cocaine use; maternal SLE (antiphospholipid

syndrome); these all result in “uteroplacental insufficiency” as the HY cause of the

oligohydramnios; fetal Potter sequence; fetal posterior urethral valves; these both cause

decreased urination.

- “What do I need to know about twinning?”

o Number of placentas = number of chorions; number of amnions = number of yolk sacs.

o The yolk sac comes from hypoblast; the amnion comes from epiblast.

o Splitting of embryo days 0-4 post-fertilizationà dichorionic-diamniotic; two placentas with

two yolk sacs, showing thick, dividing membrane (chorion) on ultrasound.

o Splitting at days 5-8 à monochorionic-diamniotic; fetuses share one placenta; there is

absence of thick, dividing membrane on ultrasound, but two distinct amniotic sacs, and

hence two yolk sacs.

o Splitting at days 9-12 à monochorionic-monoamniotic; the fetuses share single placenta and

yolk sac; risk of twin-twin transfusion syndrome is greatest.

o Splitting after day 12 à conjoined twins.

- “What do I need to know about umbilical cord anatomy?”

o Contains one umbilical vein (oxygenated), two umbilical arteries (deoxygenated), and the

allantois (tube for fetal urine to go back to mom); these are surrounded internally within the

cord by Wharton jelly.

o The deoxygenated umbilical arteries are derived from the fetal internal iliac arteries (not

veins).

- “What are the allantois, urachus, etc.?”

o Allantois = tube that carries urine from fetal bladder back to placenta; it runs from the fetal

bladder, through the umbilical cord, and all the way to the placenta.

o Urachus = thicker, fibrous part of the allantois that runs from the fetal bladder to the

umbilicus (fetal belly button); in other words, urachus just = the name of the part of the

allantois still inside the fetal body.

MEHLMANMEDICAL.COM 34
MEHLMANMEDICAL.COM

o Post-birth, the urachus closes and is known as the median umbilical ligament.

o If the urachus remains patent or partially open, it can be known as a urachal diverticulum, or

urachal cyst, or just patent urachus. The latter, for instance, could present as the neonate’s

urine coming out through the umbilicus.

- “What is the vitelline duct (omphalomesenteric duct)?”

o Connects the fetal midgut lumen to the yolk sac.

o What you need to know: failure to fully involute/obliterate causes Meckel diverticulum.

- “What do I need to know about cleft lip/palate?”

o Answer on USMLE is “polygenic” or “multifactorial.”

o If couple has child with cleft lip/palate, chance of having another child with it is 3-4% (this

statistic is similar for things like ASD/VSD and pyloric stenosis).

o Cleft lip embryo = “failure of fusion of maxillary and medial nasal processes” on NBME.

o Cleft palate embryo = “failure of fusion of lateral palatine shelves.”

- “What do I need to know about Mullerian vs Wolffian duct stuff, etc.?”

o Mullerian (paramesonephric) duct à becomes female internal structures à Fallopian tubes,

uterus, upper portion of vagina.

o Wolffian (mesonephric) duct à becomes male internal structures à SEED à Seminiferous

tubules, Epididymis, Ejaculatory duct, Ductus deferens.

o SRY gene on Y chromosome produces testis-determining factor à causes testes

development à testes are composed 90% of seminiferous tubules (coiled tubes for sperm

production) à Leydig cells produce testosterone (necessary for Wolffian development /

internal male structures) à converted to DHT via 5a-reductase (necessary for prostate +

external male structures – i.e., penis).

o Sertoli cells produce Mullerian inhibitory factor (MIF) à shuts off development of female

structures. Sertoli cells also produce androgen-binding protein (keeps local testosterone

concentration high for sperm production), inhibin B (induces negative feedback at

hypothalamus for GnRH), and aromatase (converts androgens to estrogens).

- “What do I need to know about LH and FSH for basic repro physiology?”

MEHLMANMEDICAL.COM 35
MEHLMANMEDICAL.COM

o LH stimulates the Leydig cells (in males) and theca interna cells (in females) to make

androgens (androstenedione and testosterone).

o FSH stimulates the Sertoli cells (in males) and granulosa cells (in females) to make

aromatase.

o The androgens from the Leydig cells / theca interna cells are then converted to estrogens via

the aromatase produced by the Sertoli cells / granulosa cells.

o Both androgens and inhibin B can shut off GnRH production at the hypothalamus, but

androgens have a stronger effect shutting off LH; inhibin B has a stronger effect shutting off

FSH.

o Low estrogen production by the ovaries in Turner syndrome, premature ovarian failure, and

menopause leads to high LH in the female due to lack of negative feedback; low inhibin B

results in high FSH. Specifically, high FSH is exceedingly HY as a marker of menopause.

- “What do I need to know about hysterosalpingograms?”

o USMLE loves hysterosalpingograms (dye injected into uterus via the cervix + visualization by

x-ray). By far the highest yield point you need to know is that since the Fallopian tubes are

normally open on both ends, spillage of dye into the peritoneal cavity is normal. Do not

select answers such as “rupture of Fallopian tubes,” etc. When the ovum is released from the

ovary, it will be drawn into the Fallopian tube, which is open at its lateral end.

o If USMLE shows you a hysterosalpingogram where dye does not spill into/enter the

peritoneal cavity, this can be reflective of Hx of pelvic inflammatory disease, where there is

scarring of the Fallopian tube.

o If USMLE shows you image of a uterus with a septum running down the middle of it, this is

called a bicornuate uterus à causes increased risk of premature delivery + miscarriage. The

mechanism of this on NBME = “failure of paramesonephric ducts to fuse.”

o An actual complete double uterus is called uterine didelphys.

- “What do I need to know about male vs female embryologic equivalent structures?”

o Genital tubercle becomes:

§ Glans penis (males) = glans clitoris (females).

§ Corpus cavernosum/spongiosum (males) = vestibular bulbs (females).

MEHLMANMEDICAL.COM 36
MEHLMANMEDICAL.COM

o Urogenital sinus becomes:

§ Prostate gland (males) = urethral/paraurethral glands of Skene (females).

§ Bulbourethral (Cowper) glands (males) = greater vestibular (Bartholin) glands

(females).

o Urogenital folds become:

§ Ventral shaft (underside) of penis (males) = labia minora (females).

o Labioscrotal swelling becomes:

§ Scrotum (males) = labia majora (females).

- “What do I need to know about abnormal urethral/penile development?”

o Hypospadias = urethral meatus opens on the ventral shaft of penis (pointing downward).

o Epispadias = urethral meatus opens on dorsal shaft (top) of penis (pointing upward).

o Posterior urethral valves = most common genitourinary (GU) abnormality in fetal/neonatal

males, where valves within the urethra that normally prevent backflow of urine are pointing

the opposite direction, therefore preventing the excretion of urine. Severity can vary, where

some cases result in oligohydramnios; other cases present as a newborn male who hasn’t

urinated (suprapublic mass = full bladder), or as infant male who has recurrent UTIs or

pyelonephritis. Diagnosis is with ultrasound first, followed by voiding cystourethrogram.

Don’t confuse with retrograde urethrogram (used to Dx urethral injury) or retrograde

cystourethrogram (used to Dx bladder injury). Treatment is often surgical.

o Peyronie disease = fibrosis of tunica albuginea; results in abnormal curvature/bent penis;

surgery not typically done for cosmetic purposes; reserved for functional impairment.

- “What are phimosis and paraphimosis?”

o Phimosis = inability to retract the foreskin (prepus).

o Paraphimosis = inability to reduce (put back) the foreskin. Paraphimosis is considered an

emergency since ischemia can result.

- “What do I need to know about hydrocele / varicocele?”

o Hydrocele = failure of closure of processus vaginalis à leads to fluid buildup within testis

that transilluminates; treatment = observe under the age of 1.

MEHLMANMEDICAL.COM 37
MEHLMANMEDICAL.COM

o Varicocele = congestion of the pampiniform plexus (venous plexus) draining the testes; can

cause ¯ sperm production due to ­ scrotal temperature; does not transilluminate; Tx is

usually elective surgical repair.

o Mechanism for varicocele is high-yield. It almost always occurs on the left because of the

venous drainage. The left testicular vein enters left renal vein at 90 degrees. This creates

pressure and congestion on the left side. The left renal vein will then go to the IVC. In

contrast, the right testicular vein goes “right to the IVC,” where there is no pressure effect.

o There is Q on 2CK Peds CMS form where bilateral varicocele is the answer, where you have

to eliminate to get there. In other words, just know that it is technically possible / is asked.

- “Do I need to know anything about lymphatic drainage or arterial supply?”

o Lymphatic drainage of testes and ovaries is to para-aortic lymph nodes.

o The scrotum is drained by the superficial inguinal nodes, not the para-aortic.

o The testicular and ovarian arteries come directly off the abdominal aorta at L2.

o The “gonadal arteries/veins” is a generic term that means testicular arteries/veins in males

and ovarian arteries/veins in females.

- “What is cryptorchidism?”

o Undescended testis. Tx = observe within the first 6 months of life; most will spontaneously

descend; after 6 months, orchidopexy can be performed (surgery to move the testicle down

into the scrotum). USMLE wants you to know that any Hx of cryptorchidism means the

patient has an increased risk of testicular cancer (usually seminoma) in the future.

- “What do I need to know about epididymitis vs testicular torsion?”

o Epididymitis will have intact cremasteric reflex; it is absent in torsion. This reflect is

retraction of scrotal skin with direct palpation or palpation of medial thigh; this is mediated

by the genitofemoral nerve.

o Epididymitis has a positive Prehn sign; it is negative in torsion. This sign is relief of pain upon

elevation of the testis.

o Epididymitis is usually chlamydia or gonorrhea in younger males; males who are 40s and

older, E. coli should be considered. This also applies to organisms causing prostatitis, where

on NBME, E. coli is correct over chlamydia in a 45-year-old male.

MEHLMANMEDICAL.COM 38
MEHLMANMEDICAL.COM

- “What is torsion of appendix testis?”

o This is not the same as torsion of testis. There is a structure called the appendix testis that

can also torse. This is asked on one of the 2CK pediatrics forms, but you could be aware of it

for Step 1 for the sake of it.

o The question will tell you a kid has acutely painful testis, where the superior pole is blue;

they will say cremasteric reflex is normal/intact; answer = torsion of appendix testis.

- “What about orchitis?”

o Inflammation of testis; exceedingly rare on USMLE. In theory, classically caused by mumps.

- “Do I need to know about all of the ligaments relating to the uterus/ovaries, etc.?”

o Unfortunately, USMLE cares. But I’ll tell you exactly the HY points:

o Suspensory ligament of ovary = connects lateral ovary to abdominal wall.

§ Contains ovarian vessels.

§ Ligation of ovarian vessels during oophorectomy risks injury to the ureter.

o Ovarian ligament = connects medial ovary to uterus.

o Round ligament of uterus = connects uterine horns to labia majora.

§ Uterine horns are the superolateral parts of the uterus that connect to the Fallopian

tubes.

o Cardinal ligament = connects the lateral cervix to pelvic wall.

§ Contains uterine vessels.

§ Ligation of uterine vessels during hysterectomy risks injury to ureter.

o Broad ligament = large ligament that connects uterus, Fallopian tubes, and ovaries to pelvic

wall.

§ Contains the ovaries, Fallopian tubes, and round ligaments of uterus.

o Parametrium of uterus = fat and connective tissue surrounding the uterus.

§ 2CK Obgyn form mentions embryo developing within parametrium of the uterus;

answer = ectopic pregnancy. So just be aware of this term.

- “Do I need to know vagina/perineal anatomy?”

o The answer is not really. But there are a couple HY points you could be aware of.

MEHLMANMEDICAL.COM 39
MEHLMANMEDICAL.COM

o If an episiotomy is performed posteriorly in the midline, if the obstetrician cuts too far,

USMLE wants you to know that you cut into the external anal sphincter.

o For Kegel (pelvic floor) exercises, the USMLE wants you to know that the internal anal and

urethral sphincters are not strengthened. This might sound a bit unusual, as you could say,

“Well there are tons of muscles not strengthened, e.g., the deltoids.” But the point here is

that internal sphincters are under sympathetic control (i.e., they’re not voluntary/somatic),

so clearly they can’t be strengthened by a voluntary exercise. A key muscle that is

strengthened would be levator ani.

- “Do I need to know penile anatomy?”

o Unfortunately yes. You need to know the 2D-cross-section of the penis, where you have to

identify the erectile muscle (i.e., they ask you where sildenafil would help, and you would

just choose whichever muscular compartment is labeled with a letter).

- “What do I need to know about nerves/innervation about erection, etc.?”

o Erection = parasympathetic = S2-4 (“S2, 3, 4 keeps the penis off the floor.”) = pelvic

splanchnic nerves.

o Emission = sympathetic = hypogastric nerves = T11-L2.

o Ejaculation = somatic (i.e., voluntary) = pudendal nerve.

- “Do I need to know anything about reproductive histology?”

o USMLE wants you to know that the endometrium during the proliferative/follicular phase of

the menstrual cycle will have straight/tubular glands.

o The secretory/luteal phase will have coiled glands.

§ Early-luteal phase will have coiled glands + no secretions within lumens.

MEHLMANMEDICAL.COM 40
MEHLMANMEDICAL.COM

§ Late-luteal phase will have coiled glands + secretions with lumens.

o Vagina and ectocervix are stratified squamous epithelium.

o Uterus and Fallopian tubes are simple columnar epithelium. Fallopian tubes are ciliated.

o The transformation zone of the cervix = squamocolumnar junction between the stratified

squamous of the ectocervix and simple columnar of the endocervix.

o Ovaries are simple cuboidal.

o The Sertoli cells in males are more linear and form the blood-testes barrier.

o Leydig cells (aka interstitial cells) are more randomly distributed (i.e., the cells that are not

linear).

- “What do I need to know about spermatogenesis, oogenesis, and gametes, etc.?”

o Spermatogonia = stem cells that will differentiate into sperm.

o Oogonia = stem cells that will differentiate into ova.

o Process of spermatogenesis is low-yield. End-result is spermatids that are haploid (contain

half the number of chromosomes of normal cell – i.e., 23 instead of 46).

o Sperm require cilia for motility; motility is impaired in Kartagener syndrome (primary ciliary

dyskinesia).

o Sperm are absent in cystic fibrosis (CBAVD; congenital bilateral absence of vas deferens).

o Ooogonia (stem cells) mature into primary oocytes that are locked in prophase I until

ovulation.

o At ovulation, the released ova are known as secondary oocytes and are locked in metaphase

II until fertilization.

o Once fertilization has occurred, they complete meiosis.

- “What do I need to know about hydatidiform moles?”

o A mole is a nonviable conceptus due to abnormality with chromosomes/fertilization.

o Complete mole = empty egg fertilized by two sperm, or when ovum is fertilized by a single

sperm that then duplicates; all genetic material is paternal; chromosome number = 46; no

fetal parts are presents; ultrasound shows a “snowstorm” or “bunches of grapes”

appearance; high risk of becoming choriocarcinoma (cancer of

placental/syncytiotrophoblastic tissue).

MEHLMANMEDICAL.COM 41
MEHLMANMEDICAL.COM

o Incomplete/partial mole = normal ovum fertilized by two sperm; chromosome number is 69;

fetal parts are present; can lead to choriocarcinoma, but not as high-risk as complete mole.

o b-hCG will be abnormally high in both types of moles (i.e., hundreds of thousands).

o Women present large for gestational age – e.g., Q will say fundal height is measured at level

of umbilicus when woman is only 16 weeks’ gestation (this is normally level of fundus at 20

weeks).

o Can present similarly to preeclampsia (i.e., HTN + proteinuria), but before 20 weeks’

gestation; after 20 weeks’ preeclampsia is most likely diagnosis.

o Tx for both is dilation and curettage (D&C), or suction curettage.

- “What do I need to know about gynecologic tumors/cancers?”

o It is in my view that resources vastly overemphasize certain details regarding this stuff. I’ll tell

you exactly what you need to know for the USMLE.

o Choriocarcinoma = cancer of placental/trophoblastic tissue; Q will give very high b-hCG; likes

to metastasize to the lungs (nodules on CXR) or brain (presents like stroke); appears grossly

like “bloody mess.”

o Serous cystadenoma of ovary = benign; contains Fallopian tube-like epithelium.

o Serous cystadenocarcinoma of ovary = malignant variant; has psammoma bodies (calcium

rings).

§ Both serous cystadenoma and adenocarcinoma can be bilateral.

o Mucinous cystadenoma of ovary = benign; contains mucous-like material; “loculated” (i.e.,

honeycomb-like).

o Mucinous cystadenocarcinoma of ovary = malignant variant; can cause pseudomyxoma

peritonei (inflammation of peritoneal cavity due to mucous production/spillage onto

adjacent structures).

o Dermoid cyst (aka mature cystic teratoma) = classically the “skin, hair, teeth tumor,” since it

is derived from all three germ layers; can calcify (an NBME Q mentions this as only finding);

almost always benign (unless they specifically say “immature” histo).

o Dysgerminoma = tumor of ovary; can present with high LDH and pulling sensation in groin.

o Yolk sac tumor = pediatric ovarian tumor; secretes AFP.

MEHLMANMEDICAL.COM 42
MEHLMANMEDICAL.COM

o Leydig-Sertoli cell tumor (yes, in a female) = secretes androgens à virilization.

o Granulosa cell tumor = secretes estrogens à can cause endometrial hyperplasia.

o Struma ovarii = ovarian germ cell tumor that secretes thyroid hormone.

o Krukenberg tumors = bilateral gastric cancer metastases to ovaries; have signet ring cells on

biopsy; these cells contain mucin.

o Brenner tumor = ovarian tumor containing bladder (transitional) epithelium.

o Endometrial adenocarcinoma = biggest risk factor is unopposed estrogen in setting of patient

with prior anovulation (ovulation normally leads to corpus luteum that secretes

progesterone, which limits overgrowth of endometrium); Hx of high BMI or abnormal

periods can imply endometrial hyperplasia and risk of endometrial cancer; can present as

midcycle bleeding in perimenopausal woman, or any bleeding in postmenopausal woman;

must do endometrial biopsy.

o Uterine leiomyoma (aka fibroid) = most common tumor in women; benign; stains positive for

muscle markers; can be described as white/whorled appearance grossly; highest yield point

on USMLE is that these are almost always just simply observed – i.e., don’t do myomectomy

etc., even if the Q tells you many are present and she’s going to get pregnant; if they bleed,

patient can be given OCPs and/or NSAIDs initially.

o Leiomyosarcoma = malignant variant; only point you need to know is that this is not derived

from leiomyoma; presumably this point is important because it justifies why we almost

always just observe leiomyomas (i.e., they won’t become malignant).

o Cervical cancer = squamous cell carcinoma; HY causes are HPV 16+18; Pap smear discussion,

etc., is earlier in this document.

o Clear cell vaginal carcinoma = history of maternal use of diethylstilbestrol (DES).

o Sarcoma botryroides = rare rhabdomyosarcoma seen in pediatrics.

o Gynecologic cancers in general demonstrate increased risk in BRCA1/2 and HNPCC patients.

- “What is lichen sclerosus?”

o Described as white/grey parchment-like, rough area of vulva in woman over 50; next best

step is biopsy to rule out squamous cell carcinoma; if histo confirms lichen sclerosus, Tx is

topical steroids; if SCC, must excise.

MEHLMANMEDICAL.COM 43
MEHLMANMEDICAL.COM

- “What is Bartholin gland cyst/abscess?”

o Presents as tender/painful bump at the 4 or 8-o’clock position on the labia majora; can treat

with warm compresses or Sitz bath; if lesion is warm, erythematous, and tender, can be

drained immediately; USMLE wants you to know this is polymicrobial.

- “What do I need to know about testicular cancers?

o Seminoma = most common; ages 15-35 classically; can present as hard nodule or mass that

does not transilluminate; can be discovered incidentally after trauma (in an NBME question);

increased risk in cryptorchidism or Klinefelter; histo can show large, clear cells; highly

radiosensitive (i.e., responds well to radiotherapy, even If it’s metastasized); can produce

placental alkaline phosphatase (placental ALP) as tumor marker, but not mandatory.

o Yolk sac tumor = pediatric testicular tumor; secretes AFP.

o Choriocarcinoma = secretes b-hCG; same as described prior.

o Leydig-Sertoli cell tumor = can present with gynecomastia in males – i.e., the androgens can

be aromatized into estrogens.

- “What do I need to know about breast cancers?”

o Fibroadenoma = benign; most common; rubbery, mobile, painless mass in woman 40s or

younger generally; do FNA to diagnose; if diagnosed, surgically remove, even though benign.

o Ductal carcinoma in situ (DCIS) = has malignant potential, but hasn’t yet crossed basement

membranes; presents with clusters of microcalcification; management step is “needle-

guided open biopsy” (on NBME); FNA is wrong answer for that same question; Paget disease

of breast often presents with underlying DCIS (i.e., eczematoid nipple in woman over 50 with

underlying nipple mass = underlying DCIS).

o Intraductal papilloma = unilateral bloody nipple discharge; don’t confuse with DCIS.

o Invasive ductal = same as DCIS but has already crossed basement membranes; can be

described as having “stellate morphology” on histo.

o Lobular carcinoma in situ = malignant, but hasn’t crossed basement membranes; can be

described as linear, or “Indian rows” of cells.

o Invasive lobular carcinoma = same as LCIS, but has crossed basement membranes. Both

lobular carcinoma types can occur bilaterally.

MEHLMANMEDICAL.COM 44
MEHLMANMEDICAL.COM

o Cystosarcoma phyllodes = “leaf-like” tumor that is fast-growing.

o Inflammatory carcinoma = cancer of breast that can appear red/inflamed and with pain;

classically associated with peau d'orange, or mottling of skin due to tethering of edematous

skin by Cooper ligaments of breast

o Comedocarcinoma = “cheese-like” breast cancer.

- “What do I need to know about fibrocystic change?

o Benign; can be unilateral or bilateral; classic textbook description is “bilateral breast

pain/tenderness that waxes/wanes with menstrual cycle,” but Obgyn forms can have it

presenting as unilateral pain, or as a unilateral painless cyst that drains dark fluid; no

treatment is necessary most of the time; if patient has a singularly enlarged cyst that appears

after starting hormone-replacement therapy, FNA is the answer.

- “What do I need to know about menstrual cycle / pregnancy hormones?”

o First half of menstrual cycle = proliferative/follicular phase; second half = luteal/secretory

phase. The luteal phase is always 14 days; if menstrual cycle changes length, it’s because of

variation in the follicular phase.

o Estrogen gradually increases throughout the follicular phase and is highest just prior to

ovulation, then it declines after. The high estrogen causes an LH spike that triggers ovulation.

o The corpus luteum is the follicular remnant and produces progesterone that maintains the

endometrial lining during the luteal phase.

o If pregnancy occurs, b-hCG will maintain the corpus luteum, which will enable continued

progesterone production so the pregnancy can be maintained. If pregnancy does not occur

and b-hCG is not present, the corpus luteum degrades, progesterone production ceases, and

sloughing of the endometrium occurs (menstruation).

o As discussed earlier, b-hCG peaks at 8-10 weeks of pregnancy. After this point, the placenta

takes over production of progesterone, so we no longer need hCG to maintain the corpus

luteum.

o Human placental lactogen (hPL) is a hormone that increases during third trimester of

pregnancy and causes insulin resistance in the mother. This ensures that glucose levels are

MEHLMANMEDICAL.COM 45
MEHLMANMEDICAL.COM

high enough so that brain development in the fetus occurs properly. The tradeoff is that this

is the hormone that increases risk of gestational diabetes.

o Prolactin produced by the anterior pituitary stimulates milk production.

o Oxytocin produced by the supraoptic nucleus of the hypothalamus (and stored in the

posterior pituitary) causes milk letdown (release). It also stimulates uterine contractions.

- “Do I need to know anything about Tanner stages?”

o Tanner stages 1-5 are a system for genital/breast development. You don’t need to know the

strict Tanner stages, just a few key factoids.

o For whatever reason, it’s exceedingly HY on 2CK Obgyn forms that you know once a female

hits Tanner stage 3, menarche is imminent (meaning, will occur very soon); they ask this

directly in one Q; they also incorporate it into other Qs. For instance, they’ll say a 14-year-old

girl who’s never had a menstrual period is brought in by her mom + she is Tanner stage 3 +

they ask for next step in management à answer = follow-up in 6 months (since she’s Tanner

stage 3, we know menarche is imminent, so we’ll just wait it out).

o Low Tanner stage (i.e., 1 or 2) can be the USMLE’s way of telling you a boy or girl has

constitutional short stature (i.e., will achieve normal height, but has growth curve that is

delayed / shifted to the right). For instance, they can say a boy is shortest in his class

freshman year of high school + is Tanner stage 1 à answer = constitutional short stature.

This diagnosis is also made where bone age is less than chronologic age. If bone age =

chronologic age, then the short stature is genuine.

o Turner syndrome classically has Tanner stage 1-2 breasts (i.e., “shield chest”), but it is not

mandatory the Tanner stage is low on Obgyn material.

- “What do I need to know about precocious puberty?”

o Emergence of secondary sex characteristics in females age 8 or younger, or males 9 or

younger.

o Childhood height will be increased; adult height will be decreased.

o Question might ask how we know if the cause of the precocious puberty is due to the

hypothalamus/pituitary axis or not à answer = if LH is high (if LH low, we know cause is

primary testicular production of testosterone).

MEHLMANMEDICAL.COM 46
MEHLMANMEDICAL.COM

o If DHEA-S is abnormally high, we know the adrenal gland is the cause (the zona reticularis of

the adrenal cortex secretes DHEA-S and androstenedione).

- “What do I need to know about imperforate hymen?”

o Will present as bluish bulge behind hymen in female who’s never had a menstrual period;

they can describe Hx of cyclical pain (due to menses with blood backup behind the hymen).

o Hematocolpos = blood backed up in the vaginal canal behind the hymen.

o Hematometra = blood backed up all the way to the uterine cavity, precipitating and vagal

response and low blood pressure.

- “What is placenta previa?”

o Placental implantation site over the internal cervical os.

o Causes third-trimester painless bleeding.

o Can spontaneously move off the os prior the 36 weeks’ gestation; after this point, C-section

must be done, otherwise patient may experience hemorrhagic shock during parturition.

o USMLE wants you to know that prior C-section is a risk factor for placenta previa (i.e., if the

endometrial lining has been disturbed in the past in any way, then that simply increases the

risk of an abnormal implantation).

- “What is abruptio placentae?”

o Abruption (separation) of the placenta in utero.

o Causes third-trimester painful bleeding, or painful cramping.

o Deceleration injury (i.e., car accident, fall) and cocaine use are known risk factors.

- “What is placenta accrete/increta/percreta?”

o Presents as postpartum bleeding/hemorrhage.

o Accreta = placenta attaches to surface of myometrium. A = attaches.

o Increta = placenta inserts into/inside myometrium. I = inserts inside.

o Percreta = placenta perforates through myometrium and attaches onto external structures,

such as the bladder. P = perforates.

- “What is most common cause of postpartum bleeding?”

MEHLMANMEDICAL.COM 47
MEHLMANMEDICAL.COM

o Uterine atony à presents as boggy uterus postpartum; Tx with uterine massage, followed by

intra-myometrial oxytocin injection, followed by ergotamine injection (avoid the latter if HTN

or migraine Hx).

o Less common causes are retained placental parts (if they tell you all lobes of placenta are not

present), vaginal lacerations (e.g., from macrosomia in maternal diabetes, where the fetus

experienced shoulder dystocia), or thrombotic disorders (e.g., von Willebrand).

- “What is vasa previa?”

o When the fetal vessels overly the internal cervical os. Normally, the vessels are protected by

Wharton jelly within the umbilical cord, but sometimes the vessels can be abnormally

exposed in the setting of velamentous cord insertion, or if there is the presence of a

succenturiate lobe of the placenta.

o Presents as triad of 1) rupture of membranes; 2) vaginal bleeding; 3) fetal bradycardia.

- “What do I need to know about ectopic pregnancy?”

o Can present as LLQ or LRQ pain in female who has a missed menstrual period.

o b-hCG will be positive, but the numerical value will be described as a lot lower than

expected. This is in contrast to hydatidiform mole or choriocarcinoma, where the b-hCG is

much higher than expected.

o Methotrexate can be given for small, stable ectopics. Otherwise, laparoscopic salpingostomy

is performed. If the patient is unstable (i.e., low BP in ruptured ectopic), laparotomy is the

answer.

- “What is preclampsia, eclampsia, and HELLP syndrome?”

o Preeclampsia = HTN and proteinuria after 20 weeks’ gestation. That is the most simplified

definition and sufficient for USMLE.

o Eclampsia = preeclampsia + seizure.

§ Tx = magnesium as Tx (and prophylaxis in severe preeclampsia).

o HELLP syndrome = Hemolysis, Elevated Liver enzymes, Low Platelet count.

§ Severe manifestation of preeclampsia. Schistocytes are seen on a blood smear.

- “What is supine hypotensive syndrome?”

o Low blood pressure in woman >20 weeks’ gestation due to compression of IVC.

MEHLMANMEDICAL.COM 48
MEHLMANMEDICAL.COM

- “What do I need to know about endometriosis?”

o Endometrial tissue growing outside the uterus, usually on the ovary; can cause severely

painful periods; descriptors such as pain with defecation or dyspareunia are often too buzzy

and omitted from questions. Physical examination will be abnormal (e.g., nodularity of

uterosacral ligaments); patient can get hemorrhagic (“chocolate”) cysts; diagnosis is done via

exploratory laparoscopy. Treatment is with OCPs +/- NSAIDs. Definitive is laparoscopic

removal of lesions.

- “What do I need to know about primary dysmenorrhea?”

o This is “normal period pain” due to prostaglandin secretion; physical examination is normal,

in contrast to endometriosis. Treatment is with NSAIDs. Don’t confuse with mittelschmerz,

which is ovulatory pain mid-cycle.

- “What is adenomyosis?”

o Diffusely enlarged uterus in woman generally 30s-40s, often with vaginal bleeding. They can

say a woman had a tubule ligation 2 years ago, but now has vaginal bleeding with a uterus

that is 8 weeks’ gestation in size. Treatment is with OCPs + NSAIDs.

- “What do I need to know about BPH?”

o Prostatic enlargement occurs as a result of DHT stimulation.

o USMLE loves post-renal obstruction due to BPH causing “increased tubular hydrostatic

pressure,” or “increased Bowman capsule hydrostatic pressure” in the setting of high

creatinine.

- “What do I need to know about HY Repro pharm?”

o Tamoxifen + raloxifene are selective estrogen receptor modulators (SERMs). They can be

used in ER(+) breast cancer. They are antagonists at breast + agonists at bone. Highest yield

point is that tamoxifen is partial agonist at endometrium à increased risk of endometrial

cancer. Never give tamoxifen to woman who has a uterus. Give raloxifene instead.

o Anastrozole + exemestane are aromatase inhibitors. These can be used in breast cancer.

o Trastuzumab (Herceptin) targets HER2/neu(+) breast cancer. It is cardiotoxic.

MEHLMANMEDICAL.COM 49
MEHLMANMEDICAL.COM

o Clomiphene is partial agonist at the hypothalamus (the effect is as though it’s an antagonist).

This stimulates GnRH secretion à promotes ovulation. It is used in women who have

abnormal periods, where ovulation is difficult to predict. Or it can be used in IVF.

o Mifepristone is a progesterone receptor antagonist used as an abortifacient.

o Misoprostol is a prostaglandin E1 analogue used with mifepristone as an abortifacient.

o Ulipristal is a progesterone receptor modulator used as emergency contraception.

o Dinoprostone is a prostaglandin E2 analogue used to soften the cervix.

o Ritodrine is a beta-2 agonist used as a tocolytic.

o Danazol is an androgen receptor partial agonist used as a tertiary agent in endometriosis.

However it is one of the first-line agents for hereditary angioedema (causes liver to produce

more C1 esterase inhibitor).

o Combined estrogen oral contraceptive pills à contraindicated in women who are smokers

over 35, have migraine with aura, active breast cancer, or Hx of thrombotic disorders / DVT.

o Finasteride is a 5a-reductase inhibitor that prevents the conversion of testosterone to DHT

in the treatment of BPH.

o Tamsulosin and terasozin are a1-antagonists used in the treatment of BPH. They relieve

constriction of the internal urethral sphincter.

o Leuprolide is a GnRH receptor agonist that, when given continuously, causes desensitization

of the GnRH receptor, thereby effectively acting as an antagonist. This causes a reduction in

LH and FSH. It is used for prostate cancer. It can also be used for adenomyosis and fibroids,

albeit not first line.

o Flutamide is an androgen receptor antagonist used in the treatment of prostate cancer. This

is given prior to leuprolide, since the latter will cause a transient increase in LH and FSH prior

to desensitization of the GnRH receptor. The transient increase in LH can theoretically cause

a transient increase in testosterone, so flutamide must be given first to block receptors.

o Oxybutynin is a muscarinic receptor antagonist used in the treatment of urge incontinence.

o Bethanechol is a muscarinic receptor agonist used in the treatment of overflow incontinence

due to diabetes (neurogenic / hypotonic bladder). If the cause of the overflow incontinence

is BPH, however, the BPH itself must be treated first as per above.

MEHLMANMEDICAL.COM 50
MEHLMANMEDICAL.COM

YouTube
@mehlmanmedical

Instagram
@mehlman_medical

MEHLMANMEDICAL.COM 51
MEHLMANMEDICAL.COM

MEHLMANMEDICAL
HY OBGYN/REPRO

All material is copyrighted and the property of mehlmanmedical.

Copyright © mehlmanmedical

MEHLMANMEDICAL.COM 52
MEHLMANMEDICAL
HY ANATOMY/MSK/RHEUM
MEHLMANMEDICAL.COM

YouTube
@mehlmanmedical

Instagram
@mehlman_medical

MEHLMANMEDICAL.COM 2
MEHLMANMEDICAL.COM

By Dr Michael D Mehlman

The purpose of this document is to focus on the highest yield anatomy/MSK and rheumatology for USMLE
without all of the superfluous nonsense.

Some students romanticize the discussion of every muscle insertion/origin and physical examination
maneuver, as well as go through loads of CT scans and MRIs of muscles, etc. Waste of time. The focus here is
not to prep you for some ultra-pedantic school of medicine anatomy exam. The purpose is to drive your
performance up on the USMLE. Especially now that Step 1 is pass/fail, it’s an absolute waste of time for you to
be off studying/memorizing nitpicky anatomy.

If you are studying for 2CK, you can ignore the overly anatomy-oriented points in this doc and focus on the HY
presentations. Much of this document absolutely helps with 2CK as well.

MEHLMANMEDICAL.COM 3
MEHLMANMEDICAL.COM

HY Anatomy/MSK/Rheum

- Student Q showed electron micrograph (EM) pic of sarcomere + they asked what does not change

length during muscle contraction + had letters at different locations. Answer = A-band.

o Before you instantly freak out, relax. First look at above EM of sarcomere. Then compare

with bottom images here:

o All you need to know is: as the myosin and actin overlap during muscle contraction, the H-

zone, I-band, and sarcomere (Z-Z) shorten. The A-band (myosin; neon-green bar above) does

not change in length.

MEHLMANMEDICAL.COM 4
MEHLMANMEDICAL.COM

o Tropomyosin is a protein on actin that covers up myosin binding sites. During contraction,

calcium is released from the sarcoplasmic reticulum and binds to troponin, causing a

conformational change that releases tropomyosin from actin, thereby allowing myosin to

bind.

o ATP is required to relax muscle (i.e., rigor mortis in the deceased due to ¯ ATP).

- 24M + partakes in research study of muscle contraction; Q asks, on the following electron micrograph

of a sarcomere, which segment will not change length? (choose the number):

o Answer = #2 (A-band). In contrast, the I-band (#6), H-zone (#7), and length of sarcomere (#1;

Z-Z) all shorten during contraction.

Muscle fiber type Slow-twitch (type I) Fast-twitch (type II)


Predominance in whom Long-distance runners Powerlifters; sprinters
Oxygen usage Aerobic Anaerobic
Glycogen content Low High
Fatiguability Slow to fatigue Quick to fatigue
Mitochondria Many Few
Myoglobin content (red in color) High (more red) Low (more white)

- 28M + femoral nerve injury resulting in denervation of rectus femoris; Q asks what is most likely to be

seen during nerve recovery; answer = “fiber type grouping of type I and II muscle cells”; you need to

know that reinnervation of muscle results in aberrant reorganization of type I and II muscle units. This

aberrant reorganization is called fiber type grouping. This is answer on an NBME.

MEHLMANMEDICAL.COM 5
MEHLMANMEDICAL.COM

- 24M + partakes in powerlifting routine; Q asks ­ or « for changes in muscle cell number, muscle cell

size, and mitochondria; answer = « muscle cell number, ­ muscle cell size, and « mitochondria.

Skeletal muscle response to activity is hypertrophy, not hyperplasia. Powerlifting is anaerobic and

does not increase mitochondria.

- 20F + paraplegic following accident; Q asks what is most likely to be seen in this patient; answer =

“polyubiquitination”; proteins are tagged with ubiquitin in order to be sent to the proteasome for

breakdown; atrophy in the setting of inactivity = ­ ubiquitination.

- 31M + in wilderness for 3 weeks without food; Q asks what allows him to maintain normal serum

glucose levels during this time; answer = “skeletal muscle protein” à you must know that skeletal

muscle does not contain glucose-6-phosphatase and therefore does not directly carry out

gluconeogenesis; glucogenic amino acids can be liberated in the fasting state from skeletal muscle,

with the liver carrying out the gluconeogenesis. The kidney can also carry out gluconeogenesis.

- 16F + receives insulin injection + serum glucose lowers; Q asks why; answer = “increased glucose

uptake by skeletal muscle”; both skeletal muscle and adipose tissue take up glucose via GLUT4 in

response to insulin.

- Q on offline Step 1 NBME form asks why ATP does not fall appreciably during a series of muscle

twitches à answer = “ATP is quickly regenerating from creatine phosphate.”

- “What do I need to know about shoulder anatomy for USMLE?” à USMLE is known to occasionally

give images of shoulder, clavicular, and humeral fractures. Spending a few moments to gain an idea of

normal shoulder anatomy is not “nitpicky.”

MEHLMANMEDICAL.COM 6
MEHLMANMEDICAL.COM

- Now identify:

- “What do I need to know about the rotator cuff? It’s high-yield right?” à Yes. Same as with the above

x-rays, you need to know the rotator cuff muscles. Just deal with it.

MEHLMANMEDICAL.COM 7
MEHLMANMEDICAL.COM

- Now identify:

Rotator cuff Innervation


Function HY Points
muscles (SITS) (mostly C5-6)
- Answer if patient has difficulty abducting arm
first 15 degrees.
Abduction of - Empty-can (thumb-down) / full-can (thumb-up)
Supraspinatus arm (first 15 Suprascapular nerve tests for diagnosis à patient abducts arm to 90
degrees) degrees with thumb up or down à downward
pressure applied to arm à if elicits pain, answer =
supraspinatus injury.

MEHLMANMEDICAL.COM 8
MEHLMANMEDICAL.COM

- There is Q on new 2CK CMS IM form 7 where


answer is supraspinatus tendonitis, and they say
patient has reduced ability to abduct the first 60
degrees.
- Notion of “pitcher injury” = infraspinatus does
more harm than good for USMLE; vignette can by
all means give a pitcher with (+) full-can test
Lateral above and answer is supra-, not infra-, spinatus
Infraspinatus (external) Suprascapular nerve injury.
rotation - Q might say patient simply cannot externally
rotate arm, nothing more, where teres minor isn’t
listed as another answer, so infraspinatus is only
one that could be right.
- Same as infraspinatus, just know it externally
rotates the arm.
Adduction; - Also adducts arm, so if patient has issues with
Teres minor lateral Axillary nerve both lateral rotation and adduction, answer is
rotation teres minor over infraspinatus.
- I’ve never seen NBME material assess the
diagnostic tests for infraspinatus or teres minor.
- Can medially rotate and adduct the arm.
- I’ve had students get asked Gerber lift-off test on
both Step 1 as well as 2CK Family Med shelf,
where answer = subscapularis.
Adduction;
- Gerber lift-off test = patient places dorsal aspect
medial Subscapular nerves
Subscapularis of hand on lower back, with palm facing
(internal) (upper and lower)
posteriorly à examiner applies pressure into the
rotation
patient’s palm against his/her lower back à
patient is then asked to move hand away under
the pressure à if elicits pain / difficult to do,
answer = subscapularis injury.

- “Are there other shoulder conditions sometimes confused with rotator cuff injury?” à Yes. USMLE

likes subacromial bursitis and biceps tendonitis as well.

Frequently confused shoulder conditions for USMLE


- In NBME vignettes, I’ve seen both can give Hx of patient doing frequent
overhead movement (i.e., painting a fence) + pain with palpation + pain that’s
worse when lying on one’s shoulder in bed at night, making differentiating these
Subacromial bursitis vs difficult.
rotator cuff tendonitis - Subacromial bursitis will only present with above findings, which collectively are
known as impingement syndrome.
- Rotator cuff tendonitis will present with weakness when performing exam
maneuvers (as described in prior table).
- Presents as anterior shoulder pain with focal tenderness over the biceps tendon
Biceps tendonitis
(i.e., when pressing on anterior shoulder).
- Aka “frozen shoulder,” or arthrofibrosis.
- Decreased passive and active motion of shoulder in all directions.
Adhesive capsulitis - Idiopathic, but increased risk in diabetes.
- Tx = range of motion exercises / physiotherapy.
- This is a Dx that is LY on Step 1, but for whatever reason, becomes HY on 2CK.

MEHLMANMEDICAL.COM 9
MEHLMANMEDICAL.COM

- “I know upper limb nerves are HY. What do I need to know for USMLE without all of the bullshit.” à

You need to know the injuries associated with the nerves, and then just basic motor/sensory issues.

Upper limb nerve HY Points for USMLE


- Main innervation of the deltoid, allowing for abduction of arm 15-90 degrees.
USMLE wants you to know deltoid inserts onto the clavicle and axillary nerve
innervating it is at C5/C6.
Axillary
- Palsy caused by surgical neck of humerus fracture.
- USMLE vignette will often say “flattened deltoid” or “loss of sensation over lateral
upper arm / deltoid.”
- Main innervation is lateral “3 and a half” fingers / thenar pad, and lateral forearm.
- Does thumb abduction. (In contrast, ulnar nerve does thumb adduction)
- NBME wants “palmar cutaneous branch of median nerve” as answer for sensation
over thenar region.
- Palsy caused by supracondylar fracture of humerus, or “distal shaft fracture.”
Former is buzzy; latter sounds non-specific, but I’ve seen it this way on NBME.
- Entrapment of median nerve causes carpal tunnel syndrome; will present as
paresthesia/numbness of lateral hand / thenar region; can be caused by
Median hypothyroidism (GAG deposition), acromegaly (growth of tendons), and pregnancy
(edema); can occur bilaterally in construction workers using jackhammer.
- Tx for carpal tunnel ultra-HY on 2CK FM forms. “Use of wrist pad when using
computer” is answer on new 2CK NBME. If not listed, “wrist splint” is answer on FM
form. If vignette says wrist splint fails, NSAIDs are wrong answer and not proven.
USMLE wants “triamcinolone injection into carpal tunnel” (not IV steroids) as next
answer. Surgery is always wrong answer for carpal tunnel on USMLE.
- 2CK wants “electrophysiological testing” and “electromyography and nerve
conduction studies” as next best step in diagnosis for carpal tunnel.
- Main innervation of medial “1 and a half” fingers, and medial forearm.
- Ulnar nerve also does finger abduction and adduction (i.e., interosseous muscles).
- USMLE loves Froment sign for ulnar nerve injury, which is inability to pinch a piece
of paper between the thumb and index finger (ulnar nerve needed for thumb
adduction against index finger, despite thumb being most lateral digit).
- Distal compression (i.e., of wrist and hand only, not forearm) is aka Guyon canal
syndrome and is caused by hook of hamate fracture; this can sometimes be seen
in cyclists due to handlebar compression; presents as paresthesias / numbness of
Ulnar 4th and 5th fingers + hypothenar eminence.
- Proximal compression (i.e., medial forearm + wrist/hand) is aka cubital tunnel
syndrome and is one of the most underrated diagnoses on USMLE, since its
yieldness, especially on 2CK, is comparable to carpal tunnel syndrome, but students
often haven’t heard of it. Essentially, patient will get paresthesias of medial
forearm + hand, where it “sounds like carpal tunnel but on the ulnar side instead”
à answer = cubital tunnel syndrome.
- Tx for cubital tunnel syndrome is “overnight elbow splint.” Surgery is wrong
answer on USMLE.
- Main innervation for finger, wrist, and elbow extension.
- Innervates BEST à Brachioradialis, Extensors, Supinator, Triceps.
- Palsy occurs as a result of midshaft fracture of the humerus, or as a result of
fracture at the radial groove (latter is obvious).
Radial
- Retired Step 1 NBME Q says construction worker sustains “comminuted spiral
fracture of humerus” (unusual, since spiral fracture classically = child abuse), and
they ask for the resulting defect à answer = “loss of radial nerve function.”
- Highest yield point is that injury results in pronated forearm + wrist drop.
- Main innervation of the biceps.
Musculocutaneous - Just need to know injury results in loss of sensation over lateral forearm +
decreased biceps function.

MEHLMANMEDICAL.COM 10
MEHLMANMEDICAL.COM

- USMLE doesn’t give a fuck about what kind of injury causes palsy.

- 45M + construction worker + sensation to pinprick is reduced on the thumb and a portion of the

forearm; physician suspects compression of C5/C6; Q asks, which movement is most likely fucked up

in this patient à answer = “abduction of upper extremity”; deltoid is innervated by axillary nerve

from C5/C6.

- 32F + office worker + paresthesias of lateral three fingers; wrist pad when using computer and wrist

splint have not worked; next best step? à answer = triamcinolone injection into carpal tunnel;

NSAIDs are wrong answer.

- 28M + lifts weights + paresthesias down medial forearm and 4th/5th fingers past week; Q asks next

best step in management à answer = overnight elbow splint; diagnosis is cubital tunnel syndrome.

- 42M + avid cyclist + difficulty keeping piece of paper between thumb and index finger on physical

exam; Q asks location of injury; answer = ulnar nerve; patient has (+) Froment sign; ulnar nerve

needed for thumb adduction, even though it’s lateral digit.

- 49F + skiing accident + decreased sensation over lateral forearm + weakened flexion at elbow; Q asks

nerve that’s fucked up à answer = musculocutaneous.

- 35M + in car accident and wasn’t wearing seatbelt + x-ray of arm is shown below; Q wants to know

the most likely deficit in this patient:

MEHLMANMEDICAL.COM 11
MEHLMANMEDICAL.COM

o Answer = inability to extend the wrist; answer can also be inability to supinate the forearm;

x-ray shows midshaft fracture of humerus à radial nerve injury resulting in pronated arm

with wrist drop.

- 29F + motorcycle accident + has x-ray of arm shown below; Q asks most likely deficit to be seen in this

patient:

o Answer = impaired abduction of arm, or loss of sensation over deltoid; x-ray shows surgical

neck of humerus fracture à axillary nerve injury.

- 45M + has arm slashed in street fight + now is unable to supinate and extend wrist; which letter on

the following brachial plexus image corresponds to this patient’s deficit?

MEHLMANMEDICAL.COM 12
MEHLMANMEDICAL.COM

o Answer = radial nerve (choice C). A = musculocutaneous nerve; B = axillary nerve; D = median

nerve; E = ulnar nerve.

o USMLE will ask you relatively easy presentation for a neurologic deficit but require you to

identify where on the brachial plexus is affected.

- 24M + in motorcycle accident + has loss of sensation over deltoid + diminished ability to abduct

shoulder; question asked which nerve on the following diagram is fucked up.

o Answer = E (axillary nerve); A = musculocutaneous nerve; B = median nerve; C = ulnar nerve;

D = radial nerve; F = long thoracic nerve; the black lines on the left side of the above image,

from top to bottom, are the C5-T1 nerve roots.

- “Does USMLE care about upper limb reflexes?” à Not really. But you could be aware that if biceps

reflex is weakened, they want C5 as the nerve root that’s fucked up. And if triceps reflex is weakened,

it’s C7 that’s fucked up.

- 14-month-old boy + arm has been adducted, pronated, and wrist flexed since birth but has gradually

been improving with physiotherapy; Q asks location of injury à answer = upper brachial plexus (C5-

C6); Erb-Duchenne palsy presents with “waiter’s tip” deformity; most commonly seen in neonates

during traumatic birth, but can also occur in older patients due to trauma.

MEHLMANMEDICAL.COM 13
MEHLMANMEDICAL.COM

- 22M + grabbed onto tree branch while falling from tree + presents with claw-like appearance of hand;

Q wants to know location of injury à answer = lower brachial plexus (C8-T1); you just need to know

Klumpke palsy = lower brachial plexus injury and presents with claw-hand.

- 49F + underwent mastectomy one year ago + physical examination demonstrates winged scapula; Q

asks nerve injury; answer = long thoracic nerve à innervates serratus anterior; nerve can be

damaged during mastectomy.

- 21F + paresthesias of the hand and forearm + sometimes exacerbated by wearing backpack for long

periods; Q wants diagnosis à answer = thoracic outlet syndrome; common cause is “first cervical

rib.” USMLE just wants you to know there’s some strange anatomic variant of the cervical ribs where

MEHLMANMEDICAL.COM 14
MEHLMANMEDICAL.COM

a “first cervical rib” can cause miscellaneous paresthesias and/or muscle wasting of upper limb. This is

called thoracic outlet syndrome. Nothing else you need to know.

- 46M + diabetic + difficulty moving shoulder passively and actively in all directions; Q just simply wants

the diagnosis? à answer = adhesive capsulitis (aka arthrofibrosis, or “frozen shoulder”); idiopathic

condition with decreased movement of shoulder in all directions +/- pain; increased risk in diabetics;

Tx = range of motion exercises. This condition is HY on 2CK in particular.

- 31F + was painting fence + pain in anterior/lateral shoulder that is worse when lying on side in bed +

no weakness in shoulder; Q asks diagnosis à answer = subacromial bursitis (impingement syndrome).

- 31F + was painting fence + pain in anterior/lateral shoulder that is worse when lying on side in bed +

weakness on external rotation of shoulder; Q asks diagnosis à answer = rotator cuff tendonitis.

- 26M + pain in anterior shoulder + palpation of anterior shoulder elicits pain; Q wants diagnosis à

answer = biceps tendonitis.

- “What do I need to know about bones of hand / wrist?” à USMLE likes carpal bones.

o The two rows of carpal bones (i.e., proximal and distal), from lateral to medial, are: “She

Looks Too Pretty. Try To Catch Her.”

§ First row: Scaphoid, Lunate, Triquetrum, Pisiform;

§ Second row: Trapezium, Trapezoid, Capitate, Hamate.

- 16M + skateboarding + falls on outstretched hand + no pain over anatomic snuffbox + pain in central

palm; Q asks which bone is fractured (choose a letter):

MEHLMANMEDICAL.COM 15
MEHLMANMEDICAL.COM

o Answer = D (lunate); this is on Step 1 NBME; as described, they’ll give pain in central palm +

no pain over anatomic snuffbox.

- 17M + skateboarding + falls on outstretched hand + pain over anatomic snuffbox + x-ray shows no

abnormalities; Q asks which bone is fractured:

MEHLMANMEDICAL.COM 16
MEHLMANMEDICAL.COM

o Answer = C (scaphoid); next best step is thumb-spica cast; x-ray is often negative initially;

patient needs thumb-spica cast to prevent avascular necrosis, followed by re-x-ray in 2-3

weeks.

- 14M + falls skateboarding + has pain over anatomic snuffbox; Q asks strain of which of the following

tendons might also be seen in this patient: answer = extensor pollicis longus (only one listed that is

one of the three that form the border); borders of anatomic snuffbox are: abductor pollicis longus,

extensor pollicis brevis, extensor pollicis longus.

MEHLMANMEDICAL.COM 17
MEHLMANMEDICAL.COM

- 17F + injured index finger at softball game + can’t flex distal interphalangeal joint of right index finger;

Q wants to know which tendon is most likely damaged à answer = flexor digitorum profundus;

USMLE wants you to know that flexor digitorum superficialis does not flex the DIPs, but flexor

digitorum profundus does.

o “Wait, are you saying we need to know all about hand/forearm muscles and tendons, etc.

then?” à No. USMLE really doesn’t give a fuck about hyper-nitpicky details, but I’ve seen

this one asked on an offline Step 1 NBME.

- 24M + hunting accident in which he sustains gunshot wound and severs flexor digitorum profundus

tendon; Q asks which forearm tendon can be used as a graft to repair the damaged tendon à answer

= palmaris longus; classically used as graft tendon; not present in about 1/7 people in the population.

- 59M + alcoholic + smoker + worked in construction + image of hand shown below; Q wants to know

biggest risk factor in this patient:

MEHLMANMEDICAL.COM 18
MEHLMANMEDICAL.COM

o Answer = alcoholism; diagnosis is Dupuytren contracture, which is abnormal proliferation of

palmar connective tissue and fascia; risk factors are Norwegian descent, alcoholism,

diabetes, and epilepsy. Constellation of risk factors makes no sense, but just memorize it.

- 33F + breastfeeding + severe pain in lateral wrist; pain is worsened with maneuver shown below; Q

asks for the next step in management

o Answer = steroid injection; diagnosis is deQuervain tenosynovitis; tenosynovitis means

inflammation of tendon sheaths; deQuervain is classic in breastfeeding women and is

worsened with Finkelstein test (shown above) à 1) thumb is placed in palm; 2) 2nd-5th

fingers are wrapped over the palm; 3) patient ulnar deviates the wrist à this causes pain.

Patient should avoid offending activity, but since this is often breastfeeding, steroid injection

can provide immediate relief.

- 35F + painless 2-cm bump on dorsal aspect of hand/wrist + image shown below; Q wants diagnosis:

MEHLMANMEDICAL.COM 19
MEHLMANMEDICAL.COM

o Answer = ganglion cyst, which is gelatinous collection of joint fluid; Tx is needle drainage;

recurrence common; can occur on ankles and flexor areas as well, but classic location is

dorsum of hand/wrist.

o Offline NBME 23 has Q where they ask for most likely outcome of this pathology if

untreated, and answer = “spontaneous regression.”

- 16F + history of easy bruising + hyperextensible skin + image shown below; Q wants to know

mechanism:

o Answer = “defect in synthesis of fibrillar collagen”; diagnosis is Ehlers-Danlos syndrome

(usually caused by collagen III defect); wrong answer = “abnormal synthesis of extracellular

glycoprotein” (refers to fibrillin, which is a glycoprotein that forms a sheath around/stabilizes

elastin); do not confuse fibrillar collagen (Ehlers-Danlos) with fibrillin (Marfan syndrome).

Marfan syndrome has nothing to do with collagen.

- 54F + smoker past 40 years + bilateral hand pain with clubbing; Q wants next best step in diagnosis à

answer = chest x-ray; Dx is hypertrophic osteoarthropathy (osteoarthropathia hypertrophicans);

presents as arthritis + clubbing as a result of lung cancer.

- “What do I need to know for lower limb nerves for USMLE?”

MEHLMANMEDICAL.COM 20
MEHLMANMEDICAL.COM

- Now identify:

MEHLMANMEDICAL.COM 21
MEHLMANMEDICAL.COM

Lower limb nerve HY Points for USMLE


- The answer on USMLE if patient loses both eversion and dorsiflexion of the foot.
Common peroneal
- Sensation to upper third of lateral leg (around and below lateral knee).
(fibular) nerve
- Splits into superficial and deep peroneal (fibular) nerves.
- The answer on USMLE if patient loses only eversion of the foot, but dorsiflexion
Superficial peroneal
stays intact.
nerve
- Sensation to lower lateral leg and dorsum of foot.
- The answer on USMLE if patient only loses dorsiflexion of the foot, but eversion
stays intact.
- Deep for Dorsiflexion, which means superficial is the one that does eversion
instead.
Deep peroneal nerve - Loss of dorsiflexion causes a high-steppage gait (patient has to lift foot high into
the air with each step).
- Also does sensation to webbing between 1st and 2nd toes. I’ve never seen NBME
Qs ask or give a fuck about this sensation detail, but students get fanatical about
it as if it’s supposed to be high-yield.
- The answer on USMLE if patient loses plantarflexion of the foot (can’t stand on
Tibial nerve tippytoes).
- Sensation to bottom of foot / heel.
- The answer on USMLE if patient has motor dysfunction of tibial and common
peroneal nerves at the same time, or has sciatica (shooting pain down leg).
- Splits into the common peroneal nerve and tibial nerve.
- Does not supply sensation to thigh; sensation encompasses that supplied by the
combination of the common peroneal nerve and tibial nerves.
Sciatic nerve - Supplies some motor function to muscles of thigh but USMLE doesn’t care.
- Sciatica = shooting pain from the lower back down the leg usually as the result
of disc herniation; 2CK Neuro forms simply want NSAIDs as treatment; straight-
leg test is classically used in part to diagnose, but I’ve seen this test show up on
NBME material for simple lumbosacral strain (i.e., the test is non-specific and not
reliable).
- The answer on USMLE if patient has inability to adduct the hip with loss of
Obturator nerve
sensation to medial thigh.
- The answer on USMLE if patient cannot extend knee and/or has buckling at the
knee.
Femoral nerve
- Also does sensation to anterior thigh + medial leg (not thigh), although I haven’t
seen sensation specifically asked for femoral nerve.
- The answer on USMLE if patient loses sensation to medial leg.
Saphenous nerve
- Pure sensory branch of the femoral nerve.
- The answer on USMLE if patient loses sensation to lower lateral leg. In contrast,
if sensation loss is upper lateral leg, that’s common peroneal nerve instead.
Sural nerve
- Often confused with saphenous. Good way to remember is: suraL is Lateral,
therefore saphenous must be the one that’s medial.
- The answer on USMLE if patient has Trendelenburg gait à opposite side of
pelvis will fall while walking, so patient will tilt trunk toward side of lesion while
Superior gluteal nerve
walking to maintain level pelvis.
- Innervates gluteus medius and minimus.
- The answer on USMLE if patient cannot squat, stand up from a chair, or go
Inferior gluteal nerve up/down stairs.
- Innervates gluteus maximus.

- “Do I need to know hip dislocation stuff? I’ve seen an occasional question like that but don’t really get

it?” à Not super-high-yield, but you just need to know that posterior hip dislocation causes a

MEHLMANMEDICAL.COM 22
MEHLMANMEDICAL.COM

shortened and internally rotated leg; anterior hip dislocation causes a shortened and externally

rotated leg.

- “What are some HY things I need to know about MSK spinal stuff?” à I talk about neuro-related stuff a lot

more in my HY Neuroanatomy PDF, but below is a table of some HY MSK spinal stuff.

HY MSK spinal conditions for USMLE


- The answer on USMLE if they say patient over 50 has neck pain + MRI shows
degenerative changes of cervical spine.
- Can occur in lumbar spine, but USMLE likes cervical spine for this.
- Technically defined as degeneration of pars interarticularis component of
vertebral body.

Cervical spondylosis

- Increased mobility between the first (atlas) and second (axis) vertebrae.
- Really HY on 2CK Surg and Neuro forms in patients who have rheumatoid
arthritis.
Atlantoaxial - Must do CT or flexion/extension x-rays of cervical spine prior to surgery when a
subluxation patient will be intubated; I’ve seen both of these as answers for different Surg Qs.
- Q on one of the Neuro CMS forms gives patient with RA not undergoing surgery
who has paresthesias of upper limbs à answer is just MRI of cervical spine
(implying atlantoaxial subluxation has already occurred).

MEHLMANMEDICAL.COM 23
MEHLMANMEDICAL.COM

- Narrowing of the spinal canal.


- The answer on USMLE if they mention a patient over 50 who has lower back
pain that’s worse when walking down a hill (i.e., relieved when leaning forward),
or when standing/walking for extended periods of time.
- Can cause “neurogenic claudication,” where the vignette sounds like the patient
has intermittent claudication, but they’ll make it clear the peripheral pulses are
normal and that the patient doesn’t have cardiovascular disease; this shows up in
particular on 2CK Neuro CMS forms.
- Technically an osteoarthritic change of the spine; therefore increased risk in
obesity (but not mandatory for questions).

Lumbar spinal stenosis

- The answer on USMLE if they say old woman has difficulty fastening buttons +
weakness of hand muscles + loss of sensation of little finger à answer = “C7-T1
foraminal stenosis” (offline NBME 20).
- Not stenosis of cervical spinal canal, but stenosis of foramen where nerve exits.

Cervical foraminal
stenosis

- The answer on USMLE if they a “step-off” between infra-/suprajacent vertebrae.


In other words, they’ll say one vertebra “juts out” or has a “step-off” compared to
Spondylolisthesis those above/below it.
- Can be due to trauma or idiopathic development.

MEHLMANMEDICAL.COM 24
MEHLMANMEDICAL.COM

- Herniation of nucleus pulposus through a tear in annulus fibrosis.

Disc herniation - The answer on USMLE if they mention radiculopathy (i.e., shooting pain down a
leg) after lifting a heavy weight or bending over (e.g., while gardening). They can
write the answer as “herniated nucleus pulposus.”
- As I mentioned with the radiculopathies above, be aware of the L4, L5, and S1
differences.
- Be aware that cervical disc herniation “is a thing,” meaning it’s possible and also
assessed on USMLE. 2CK neuro forms ask this a couple times, where patient has
shooting pain down an arm, and answer is “C8 disc herniation.”
- If suspected, newest NBMEs want “no diagnostic studies indicated.” X-ray and
MRI are not indicated unless there is motor/sensory abnormality (i.e., weakness
or numbness). But for mere radiculopathy (i.e., radiating pain), no imaging
necessary on new NBME content.
- Straight-leg raise test is not reliable. Mere pain alone is a negative test. The test
is only positive when they say it reproduces radiculopathy/radiating pain. There
is a 2CK Q where they say straight-leg test causes pain (i.e., negative test) and
answer is “no further management indicated” (i.e., Dx is only lumbosacral strain).
- Tx is NSAIDs + light exercise as tolerated. Bed rest is wrong answer on USMLE.
- The answer on USMLE if they say patient has paraspinal muscle spasm following
Lumbosacral strain lifting of heavy box without radiculopathy. If they say radiculopathy, the answer
is disc herniation instead.

MEHLMANMEDICAL.COM 25
MEHLMANMEDICAL.COM

- Straight-leg test can cause pain (i.e., negative test). The test is only positive if
they it reproduces radiating pain.
- Do not x-ray. This is really HY for 2CK. Apparently lumbar spinal x-rays are one of
the most frivolously ordered tests, and USMLE wants you to know that you do not
order one for simple lumbosacral strain. In contrast, if there is radiculopathy, do
x-ray followed by MRI, as discussed above.
- Tx is NSAIDs + light exercise as tolerated. Bed rest is wrong answer on USMLE.
- 90% of the time is due to disc herniation.

Sciatica

- Straight-leg test classically (+) – i.e., reproduces radiating pain.


- Tx = Light exercise as tolerated + NSAIDs. Bed rest is wrong answer on USMLE.
- On one of the 2CK CMS forms, ibuprofen straight-up is listed as the answer.
- The answer on USMLE if they say patient has pain or paresthesias running down
the lateral thigh.
Meralgia paresthetica - Due to entrapment of lateral femoral cutaneous nerve.
- Often seen as incorrect answer choice on Step, so at least be aware of it.

MEHLMANMEDICAL.COM 26
MEHLMANMEDICAL.COM

- Sideways curvature of spine, creating an S- or C-shaped curve.


- Usually idiopathic; affects 3% of population; girls 4:1.

Scoliosis

- Can be associated with Marfan syndrome, Freidreich ataxia, NF1.


- Adams forward bend test used to diagnose.
- USMLE wants you to know most children do not need treatment, but that
curvatures will remain throughout life.
- Answer = bracing if curvature is >25 degrees and child is still growing.
- I’ve never seen surgery as answer for scoliosis on NBME; literature says
recommended only when curvature >40 degrees.
- Abnormal convex curvature of thoracic spine.
- Usually idiopathic due to old age; can be due to degenerative disc disease and
Kyphosis compression fractures (osteoporosis).
- If severe, can in theory cause restrictive lung disease due to impaired chest wall
expansion.

MEHLMANMEDICAL.COM 27
MEHLMANMEDICAL.COM

- “What do I need to know for cauda equina syndrome versus conus medullaris syndrome?”

o The spinal cord ends at L1-L2. The terminal cone of the spinal cord is the conus medullaris.

The nerve fibers that branch from the spinal cord and run inferior to it are the cauda equina.

o Resources make these conditions way more complicated than they need to be. For USMLE,

all they really care about is: both conditions can cause urinary retention and pain in the legs;

conus medullaris syndrome is sudden-onset (think cone is “sharp”) and causes perianal

anesthesia; cauda equina syndrome is gradual-onset and causes saddle anesthesia. If you

look these conditions up online, you’ll see numerous differences listed side by side. Waste of

time for USMLE.

MEHLMANMEDICAL.COM 28
MEHLMANMEDICAL.COM

o USMLE likes breast, prostate, and lung metastases to the spine as important cause of cauda

equina syndrome.

- 65M + painful neck + no neurologic findings + elective MRI shows degenerative changes of cervical

spine; Q asks most likely diagnosis à answer = cervical spondylosis; this is merely osteoarthritic

degenerative changes to the cervical vertebrae; it is idiopathic, but often has familial association.

- 63M + pain in buttocks and thighs when walking + no cardiovascular disease + peripheral pulses

normal; Q wants to know diagnosis à answer = lumbar spinal stenosis; this presentation is

“neurogenic claudication,” where the vignette sounds like intermittent claudication due to aortoiliac

atherosclerosis, but the patient will have no cardiovascular disease history and they’ll say peripheral

pulses are normal; this is due to osteoarthritic changes in the lumbar spine.

- 67M + lower back pain worse when standing or walking for 30 minutes + relieved when leaning

forward; Q wants diagnosis à answer = lumbar spinal stenosis; classically pain in lower back that is

worse when standing or walking for extended periods of time; it is worse when leaning back + better

when leaning forward.

- 79F + difficulty fastening buttons + weakness of hand muscles + numbness in 5th finger; Q wants

diagnosis à answer = “C7-T1 foraminal stenosis” (cervical spinal stenosis).

- 61F + vignette is big rambling paragraph of nonsense + they tell you there’s a “step-off” of one

vertebra relative to another on palpation; Q wants to know diagnosis à answer = spondylolisthesis.

- 24M + lifted heavy box + severe paraspinal muscle spasm on left + no radiating pain + straight-leg test

elicits pain; Q wants next best step in management à answer = “no diagnostic studies indicated”; Dx

is lumbosacral strain; negative radiculopathy makes disc herniation less likely; straight-leg test is

negative if pain alone is elicited; it is only positive if radiating pain is elicited.

- 48F + gardening + sudden-onset severe pain down right leg; Q wants next best step in management

à answer = x-ray; diagnosis is disc herniation. X-ray will not visualize herniation, but it is done prior

to MRI to rule out other DDx, such as infections, tumors, and spinal misalignments.

- 48F + gardening + sudden-onset severe pain down right leg + straight-leg test elicits radiating pain; Q

asks what is most likely to confirm diagnosis à answer = MRI.

MEHLMANMEDICAL.COM 29
MEHLMANMEDICAL.COM

- 48F + gardening + sudden-onset severe pain down right leg + straight-leg test elicits radiating pain; Q

asks most appropriate treatment à answer = “analgesics + exercise as tolerated”; analgesics =

NSAIDs; bed-rest is wrong answer on USMLE for both lumbosacral strain and disc herniations.

- 50M + high BMI + 24-hour history of pain starting in lower back and shooting down left leg; Q wants

treatment à answer = NSAIDs; Dx is sciatica; 90% of the time due to disc herniation; diagnosis is

made by x-ray (negative) and then MRI (showing disc herniation).

- 55F + pain + paresthesias in lateral thigh + no history of trauma; Q wants diagnosis à answer =

meralgia paresthetica.

- 74M + lifted heavy suitcases around the house + sudden-onset pain and tingling in lower back +

suprapubic mass + has not urinated for past 24 hours + perianal anesthesia; Q asks diagnosis à

answer = conus medullaris syndrome; findings tend to be sudden-onset (cone is “sharp”/sudden);

perianal anesthesia is characteristic; both conus medullaris and cauda equina syndromes can present

with urinary retention and pain in the lower back / legs.

- 74M + one-month of gradually increasing lower back and leg pain + physical examination shows

saddle anesthesia + post-void volume is 400 mL + history of lung cancer; Q wants diagnosis à answer

= metastases to cauda equina; 2CK NBMEs love mets to the spine in breast, prostate, and lung cancer,

with impingement on the cauda equina.

- “Do I need to know dermatomes for USMLE?” à USMLE doesn’t crazy-obsess, but you need to have

an idea of the basics à C6 is thumb; C7 is middle finger; C8 is pinky; nipples are T4; umbilicus is T10;

groin and lower back are L1; anteromedial leg is L4; sole of foot is S1.

- 57F + burning/itching pain along lower back close to the hip on the left; Q asks which dermatome this

represents (answers are T10, L1, S1) à answer = L1; diagnosis is shingles (herpes zoster; VZV).

MEHLMANMEDICAL.COM 30
MEHLMANMEDICAL.COM

- “What do I need to know about lower limb reflexes / radiculopathies?”

Lower limb reflexes / radiculopathies


- The answer on USMLE if patient loses knee (patellar) reflex + has weakened knee
extension.
L4 radiculopathy - Pain / paresthesias / numbness in L4 distribution (anterior thigh + medial leg).
- Disc herniation of L3-4.
- Just remember that L4 is the one where the knee reflex is fucked up.
- The answer on USMLE if patient loses dorsiflexion.
L5 radiculopathy - Pain / paresthesias / numbness in L5 distribution (lateral + anterior leg).
- Disc herniation of L4-L5.
- The answer on USMLE if patient loses ankle (Achilles) reflex + has weakened plantar
flexion.
- Pain / paresthesias / numbness in S1 distribution (sole of foot + lower leg).
S1 radiculopathy
- Disc herniation of L5-S1.
- Just remember that S1 is the one where the ankle reflex is fucked up.
- SALT à S1, Achilles, Lateral leg dermatome, Tibial motor issue (plantar flexion).

- 41F + pain radiating down distal anterior thigh, knee, medial leg, and medial foot; compression of nerve

root in which intervertebral foramina is most likely the cause of her symptoms? à answer = L3-L4.

- 41F + random leg sensory issues + knee jerk is impaired; Q asks nerve root à answer = L4.

- 41F + random leg sensory issues + ankle jerk is impaired; Q asks nerve root à answer = S1.

MEHLMANMEDICAL.COM 31
MEHLMANMEDICAL.COM

- 41F + big paragraph of nonsense + loss of sensation of sole of foot; Q asks nerve root à answer = S1.

- 65F + gradual-onset inability to stand on the tippytoes + diminished sensation on sole of the foot + MRI is

shown below; Q wants most likely diagnosis?

o Answer = Disc herniation of L5-S1; USMLE doesn’t expect you to be a radiology expert; the

diagnosis is inferable based on the vignette alone; MRI shows herniation of nucleus pulposus

of L5-S1 disc with impingement on the spinal cord.

- “Do I need to know ankle sprain stuff for USMLE?” à For both Steps 1 and 2CK Family Medicine, yes.

o Anterior talofibular ligament is on the lateral side of the ankle and will be injured if the foot

inverts (rolls inward). This is the most commonly injured ankle ligament.

o The deltoid ligament is stronger and on the medial side of the ankle. This injury is more rare

and occurs if the ankle forcibly everts (rolls outward).

o There is a Q on one of the Step 1 NBME exams where they show some arcane x-ray of the

ankle + tell you the patient had forcible eversion of the ankle + they ask what’s injured à

answer = deltoid ligament.

MEHLMANMEDICAL.COM 32
MEHLMANMEDICAL.COM

- 23M + playing basketball + lands on right ankle where it forcibly everts; x-ray is shown below; left side

of x-ray shows prior to ligamental repair; right side is after repair; Q asks which ligament is injured:

o Answer = deltoid ligament; notice the large joint space on the medial aspect of the ankle in

the left x-ray.

- 25M + twisted ankle yesterday + moderate edema of lateral side of ankle with ecchymoses +

tenderness to palpation lateral and anterior to lateral malleolus + patient can weight-bear; Q asks, in

addition to 2-day ice pack application, what is next best step in management? à answer on family

med form = “use a soft protective brace and early range of motion exercises”; wrong answer = “x-ray

of the ankle to rule out fracture.” For 2CK Family Medicine, you need to know Ottawa criteria for

suspected ankle fractures. Before development of this criteria, x-rays for the ankle used to be ordered

frivolously, with most showing no fracture. Only order an x-ray for the ankle if:

MEHLMANMEDICAL.COM 33
MEHLMANMEDICAL.COM

o Pain in the malleolar zone, AND any of the following:

o Tenderness posterior to the lateral or medial malleolus; OR

o Tenderness on the tip of the lateral or medial malleolus; OR

o Patient cannot bear weight when walking four steps.

§ The above might seem nitpicky and pedantic, but this is HY for 2CK FM as I said.

- 40M + playing basketball + rolls ankle + pain anterior to lateral malleolus + swelling of ankle + no pain

posterior to lateral malleolus + patient can bear weight; Q wants next best step in management à

answer = rest, ice, compression, elevation (RICE); wrong answer is x-ray; the patient doesn’t fulfill the

Ottawa criteria for x-raying the ankle; he has pain in the malleolar zone but does not have pain

posterior to the malleolus or on the tip of the malleolus, and he can bear weight.

- 26F + went running and rolled her ankle + pain in lateral ankle + tenderness posterior to malleolus +

can bear weight; Q wants next best step à answer = x-ray of ankle; patient fulfills Ottawa criteria for

x-ray à she has pain in malleolar region + tenderness posterior to the malleolus; although she can

bear weight, the former two findings satisfy the Ottawa criteria.

- “Does USMLE care about knowing all of the muscles and their insertions/origins, etc.?” à USMLE

does have random Qs where they show MRI cross-sections of the thigh:

MEHLMANMEDICAL.COM 34
MEHLMANMEDICAL.COM

o 1 = rectus femoris; 2 = vastus lateralis; 3 = vastus intermedius; 4 = vastus medialis; 5 =

sartorius; 6 = biceps femoris short head; 7 = biceps femoris long head; 8 =

semimembranosus; 9 = semitendinosis; 10 = adductor magnus; 11 = adductor longus; 12 =

adductor brevis; 13 = gracilis.

- 34M + plays soccer + pain in the leg + MRI cross-section shows muscle that’s injured; Q asks which

muscle it is:

o Answer = rectus femoris.

- “What do I need to know about the arthritis conditions for USMLE?”

HY Arthritides / joint pathologies for USMLE


- OA is non-inflammatory and also known as degenerative joint disease.
- Since it is non-inflammatory, WBCs in joint aspirates will be low (i.e., <10k/µL). In
addition, patient will not have elevated ESR.
- Hip, knee, and hand findings are usually asymmetric (e.g., “asymmetric joint space
narrowing”). This is in contrast to RA, which is usually symmetric. This pattern isn’t
mandatory, but you should still commit the general association to memory.
- “Eburnation” is a term that refers to the ivory-like appearance of bone in OA at
sites of cartilage erosion.
- HY causes are obesity, being big and tall, or lifting heavy weight on the legs over
many years. Patient is usually, but not always, over 40-50. In contrast, RA starts
younger (20s-30s).
Osteoarthritis
- Highest yield point to know is that weight loss is the number-one way to prevent
and treat OA.
- Apart from weight loss, acetaminophen is correct before NSAIDs, since OA is non-
inflammatory, so NSAIDs won’t do any better + NSAIDs kill the kidneys.
- Steroids (both oral and intra-articular) are wrong fucking answer for OA. OA, once
again is non-inflammatory. Steroids have no role.
- Injection of glycosaminoglycans and use of capsaicin cream are also wrong
answers on USMLE.
- I’d guess 80% of NBME vignettes for OA are a patient over age 40-50 with very
high BMI (>40). There is Q on 2CK NBME form where Dx is OA in young woman (32;
unusually young), but she is 6’2” and BMI 30 (big and tall).

MEHLMANMEDICAL.COM 35
MEHLMANMEDICAL.COM

- OA of the hands affects the DIPs (Heberden nodes) in addition to the PIPs
(Bouchard nodes).

- RA does not affect the DIPs (MCPs + PIPs only). So if you ever get a hand x-ray and
instantly say “No idea what I’m looking at” à Relax. Just say “Do I see anything
fucked up with the DIPs” (i.e., osteophyte-looking swellings, etc.). Yes? Cool, we
know immediately it’s not RA (and likely OA).
- In contrast to OA, RA is inflammatory, where joint aspirate will show generally
>20-50k WBCs/µL. USMLE doesn’t give a fuck about exact numbers (i.e., “what
about 10-20k?”). You should just be aware, more or less, that <10k/µL is non-
inflammatory for OA, but >20k is where we say, “Ok, there’s something
inflammatory here.” ESR will also be high if they list it.
- In contrast to OA, RA is symmetric. As I mentioned up above, the association of
symmetry for RA versus asymmetry for OA isn’t a 100% mandatory finding, but you
should be aware of that general distinction.
- “Pannus” is a term that refers to growth of new bone within joints in RA.
- Most RA vignettes will be a woman 20s-40s. NBME exams like to mention other
autoimmune diseases in either the patient or family member (i.e., brother has IBD,
or sister has SLE, or patient has T1DM), since “autoimmune diseases go together.”
The HLA associations are not strict. Do not pigeon-hole that stuff.
- If USMLE vignette sounds like RA, they want “arthrocentesis” as first step in
diagnosis. They won’t force you to choose between this and antibodies.
- For antibodies, anti-cyclic citrullinated peptide (anti-CCP) is more specific than
Rheumatoid arthritis rheumatoid factor. Both should be ordered for RA.
- Tx for RA is HY. Two arms of management: 1) symptoms; 2) disease progression.
For symptoms, give NSAIDs first, followed by steroids. These do not slow disease
progression. NSAIDs and steroids are for symptoms only. For disease progression,
we use disease-modifying anti-rheumatic drugs (DMARDs), which slow disease
progression. Methotrexate is given first, followed by adding an anti-TNF-a agent
(i.e., infliximab, adalimumab, or etanercept).
- Rheumatoid arthritis can cause pulmonary fibrosis / restrictive lung disease. This
is called rheumatoid lung. At the same time, methotrexate can also cause
pulmonary fibrosis. So restrictive lung disease in RA patients is often a mix of
rheumatoid lung and methotrexate-induced pulmonary fibrosis.
- RA can cause serous pericarditis (friction rub), as can other autoimmune diseases,
such as SLE.
- Low hemoglobin in RA = anemia of chronic disease; obviously this could be for any
autoimmune disease, but this shows up a lot in RA NBME vignettes.
- Swan neck = extension of PIP and flexion of DIP; Boutonniere = flexion of PIP and
extension of DIP.

MEHLMANMEDICAL.COM 36
MEHLMANMEDICAL.COM

- Rheumatoid arthritis + neutropenia + splenomegaly = Felty syndrome.


- Rheumatoid arthritis + pneumoconiosis = Caplan syndrome.
- As I mentioned earlier, atlantoaxial subluxation is really HY on 2CK Surg and
Neuro forms.
- 2CK Peds forms love JRA. It will sound like regular RA but just in a kid.
- USMLE will structure these Qs where they want you to pick between JRA and
septic arthritis (SA) as answer choices. This can be confusing since SA can occur in
patients with JRA. They might say a kid has a low-grade fever and a warm, red,
painful knee (sounds like SA), but then they say he’s had similar episodes in the
past (i.e., they want JRA over SA). This is because SA is usually a one-off event; for
Juvenile rheumatoid JRA, however, the vignette will say “intermittent” or “episodic.” Low-grade fever
arthritis can occur in autoimmune flares (not limited to JRA; HY for sarcoidosis as well).
- “Salmon-pink” maculopapular rash only in ~50% of JRA Qs. Often described as a
buzzy finding, but I’d say about half of JRA vignettes don’t even mention rash.
- 2CK forms are obsessed with anemia of chronic disease in JRA. HY point is that
MCV can absolutely be low. Resources push normal MCV for AoCD. This is absolute
nonsense. Plenty of 2CK NBME Qs give MCV as 70s in AoCD.
- Diagnosis and treatment are same as RA.
- 15% of patients with psoriasis will get arthritis before any skin findings.
- Can cause “pencil-in-cup” deformity of the DIPs.

Psoriasis

- Plaques are described as silvery and scaly and over extensor regions (elbows);
plaques can also show up on the face (i.e., forehead and lip).
- Auspitz sign is bleeding of the scales if removed.
- Munro microabscesses = pathognomonic for psoriasis = collections of neutrophils
in stratum corneum of epidermis.

MEHLMANMEDICAL.COM 37
MEHLMANMEDICAL.COM

- HY point for USMLE is that psoriasis is part of the HLA-B27 constellation (PAIR à
Psoriasis, Ankylosing spondylitis, IBD, Reactive arthritis).
- For example, if 22M has silvery plaque on elbow and forehead + bloody stool à
the latter is most likely IBD due to HLA-B27 association.
- Don’t confuse psoriasis + IBD combo with dermatitis herpetiformis + Celiac
disease combo.
- Treatment for plaque psoriasis is topicals first à USMLE wants calcipotriene
(vitamin D derivative), triamcinolone or hydrocortisone (both corticosteroids), and
coal tar. Choose in that order if you are forced. Chronic application of topical
steroids causes dermal collagen thinning, so they are not preferred prior to topical
vitamin D.
- Oral meds are given if patient fails topicals, OR if patient has systemic psoriasis
(i.e., arthritis). Oral methotrexate is HY on new NBME material as the first-line oral
agent used.
- An old Step 1 form has oral acitretin (a vitamin A derivative) as an answer, where
methotrexate is not listed.
- Sacroiliitis is broad term that refers to arthritis of lower back; ankylosing
spondylitis (AS) is most severe form.
- Vignette will almost always be male 20s-40s who has lower back pain worse in the
morning that improves throughout the day.
- There is Q on 2CK Peds CMS form where AS is diagnosis in an 8-year-old, but this
is not typical demographic.
- Lower back pain in patient with IBD, psoriasis, or reactive arthritis points toward
sacroiliitis (HLA-B27 PAIR).
- High ESR and anemia of chronic disease high-yield.
- Diagnose with x-rays of the lumbosacral spine and sacroiliac joints.
- “Bamboo spine” (vertebral body fusion) is buzzy x-ray finding in AS.

Sacroiliitis

- USMLE wants “slit-lamp examination” to look for anterior uveitis in ankylosing


spondylitis. Any autoimmune disease can theoretically increase risk of this eye
finding, but for whatever reason USMLE likes it for AS.
- Severe AS can lead to restrictive lung disease (normal or ­ FEV1/FVC).
- For whatever magical reason, up to 20% of AS patients can present with aortic
regurgitation due to inflammation of the aorta (bounding pulses).
- Treat same as RA.
- Arthritis is most common presenting feature of lupus (90%). USMLE vignette will
Lupus
pretty much always give arthritis in lupus Qs.

MEHLMANMEDICAL.COM 38
MEHLMANMEDICAL.COM

- Anti-double-stranded-DNA (dsDNA) antibodies go up with acute flares and are


most closely related to renal prognosis for lupus nephritis.
- Anti-Smith (ribonucleoprotein) antibodies are most specific for SLE (more than
dsDNA).
- Anti-histone antibodies occur in drug-induced lupus (usually arthritis and
mediastinitis following procainamide administration).
- Anti-phospholipid syndrome due to lupus anticoagulant (antibodies against b2-
microglobulin or cardiolipin in patient with SLE) à patient presents with recurrent
miscarriages, as well as thromboses in spite of elevated aPTT test.
- Anti-phospholipid syndrome can cause false-positive syphilis VDRL test (HY).
- Malar rash is type III hypersensitivity. Harder Qs won’t mention this finding
because it’s too buzzy. You need to know thrombocytopenia is frequently seen in
lupus due to anti-hematologic cell line antibodies. Antibodies can also target WBC
and RBC à looks like aplastic anemia, but it’s not à answer = “increased
peripheral destruction,” not “decreased bone marrow production.”
- Mouth ulcers, discoid lupus (skin lesions), and primary CNS lymphoma are HY for
SLE. The latter can occur in any autoimmune disease, but HY for SLE.
- Similar to RA, lupus can cause pericarditis.
- Flares cause decreased serum complement protein C3.
- Congenital complement protein C1q deficiency causes ­ risk of developing SLE;
sounds nitpicky but it’s on new Step 1 NBME exam.
- For lupus nephritis, biopsy as the first step in management; steroids first is wrong
answer; biopsy first sounds wrong but it guides management.
- Lupus nephritis = diffuse proliferative glomerulonephritis (DPGN) on USMLE.
- Treat flares of lupus with steroids. USMLE doesn’t care about other Txs.

- Can cause arthritis of knees and hands, as well as gouty tophi (monosodium urate
crystal deposition).
- Classic patient is middle-age guy who drinks alcohol, eats a lot of meat, and drinks
bags of nucleic acids.
- Patient need not have hyperuricemia to have gout; likewise, patients with
hyperuricemia can be asymptomatic and not develop gout.
- Monosodium urate (uric acid) crystals are needle-shaped and yellow (negatively
birefringent) under polarized light.
- Crystals causing urolithiasis are radiolucent on x-ray.
- Tx for acute gout is indomethacin (NSAID), then oral corticosteroids, then
colchicine, in that order. If patient has renal insufficiency, use corticosteroids. If
Gout patient has allergy to NSAIDs, colchicine is answer on NBME exam (steroids not
listed for that same Q).
- Tx for chronic gout (i.e., Tx between attacks to decrease recurrence) is xanthine
oxidase inhibitors first (i.e., allopurinol or febuxostat). Do not give xanthine oxidase
inhibitors for acute gout, as they can worsen flares (sounds paradoxical, but it’s
what happens).
- I have not seen probenecid asked as an actual Tx for gout on NBME. They want
you to know it inhibits organic anion transporter (OAT) in the kidney, which both
inhibits reabsorption of uric acid and secretion of beta-lactams à therefore can be
used to maintain serum levels of beta-lactams.
- Do not give probenecid to patients with Hx or urolithiasis (due to the drug causing
increased excretion of uric acid within the renal tubules).
- Aka calcium pyrophosphate deposition disease; causes rhomboid-shape blue
(positively birefringent) crystals under polarized light.
- Biggest risk factors are hemochromatosis and primary hyperparathyroidism.
Pseudogout - Will present one of two ways on USMLE: 1) as a monoarthritis of the knee, or 2) as
an OA-like presentation of the hands in someone with hemochromatosis or primary
hyperparathyroidism.
- Tx acute pseudogout same as “regular” gout.

MEHLMANMEDICAL.COM 39
MEHLMANMEDICAL.COM

- Tx of chronic pseudogout = treat the underlying condition.


- Basic calcium phosphate deposition disease presents as “Milwaukee shoulder,”
BCP disease which is a cold (non-inflammatory) effusion of the shoulder.
- “Lower yield,” but good for students who want 280+ on 2CK.
- Biggest risk factor is abnormal joint architecture à will present in four main
groups on USMLE: 1) patients with prosthetic joints (you can’t have more abnormal
architecture than an artificial joint); 2) RA/JRA/OA patients; 3) young healthy
patients with recent trauma (e.g., car accident) or high-intensity exercise causing
microtrauma (e.g., kickboxing/soccer tournament, long hike); 4) sickle cell disease.
- USMLE will give hot, red, painful joint in one of the above patient groups, almost
always with a fever.
- First step in management is arthrocentesis. A high-yield point is that joint aspirate
Septic arthritis
can be negative for organisms. Do not rule-out SA if NBME says no organisms.
- Staph aureus is most common pathogen for the first three groups above.
- Gonococcus is the answer for sexually active patients; presents two ways on
USMLE: 1) monoarthritis of the knee in sexually active young patient, with no other
information provided; or 2) as a triad: monoarthritis or polyarthritis; cutaneous
papules; tenosynovitis (inflammation of tendon sheaths).
- Treatment is surgical drainage of the joint + antibiotics. USMLE doesn’t care about
the exact antibiotics.
- Classically presents as triad of 1) urethritis or abdominal infection, 2) polyarthritis,
and 3) “eye-itis” (i.e., conjunctivitis, episcleritis, or anterior uveitis).
- Chlamydia is the classic organism that causes reactive arthritis. Gonococcus does
Reactive arthritis
not cause reactive arthritis on USMLE.
- Rubella, Hep B+C, and Yersinia can also cause reactive arthritis.
- Part of HLA-B27 constellation (PAIR), as mentioned above.
- Aka transient synovitis. 2CK pediatrics forms are obsessed with this.
- Presents as hip inflammation/pain in child after a viral infection.
- Toxic synovitis is diagnosis of exclusion, meaning the vignette gives you various
findings that make septic arthritis less likely. Leukocytosis and inability to bear
Toxic synovitis
weight make septic arthritis the likely answer over toxic synovitis. Hard questions
will not mention warmth or redness as ways to differentiate. Fever can present in
both.
- Treatment is NSAIDs (ibuprofen) à asked on 2CK Peds.
- Pain in joint that can present soon, or many years after, injury, where other DDx
are ruled out.
- Shows up on 2CK form where it first sounds like patient has patellar tendonitis
(i.e., 2 years of knee pain first worsened with basketball but then progresses to
Post-traumatic
more constant pain), but then they go on to say that patient had fracture to
arthritis
proximal tibia 30 years ago and has varus deformity of the knee; answer =
traumatic arthritis, not patella tendonitis. The implication is that the etiology for
the patient’s arthritis is ultimately linked to the varus deformity from the prior
injury.

- 49F + left knee pain + BMI 40 + Q asks what would most likely have prevented this patient’s condition

à answer = weight loss; diagnosis is osteoarthritis (OA). As easy as it gets, but this Q is rampant on

USMLE.

- 32F + 6’2” + right knee pain + BMI 30; Q asks, in addition to weight loss, what is the most appropriate

treatment for this patient à answer = acetaminophen; diagnosis is OA; patient here is young, but big

MEHLMANMEDICAL.COM 40
MEHLMANMEDICAL.COM

and tall; NSAID as first answer is wrong. As I mentioned above, OA is non-inflammatory, so NSAIDs

don’t help more than acetaminophen + they kill the kidney.

- 60F + taking high-dose naproxen for the past 6 weeks for treatment of her OA + has peripheral

edema; Q asks the most likely reason for this patient’s edema à answer = “increased renal retention

of sodium” or “decreased renal excretion of sodium”; patient is inappropriately self-medicating with

high-dose NSAID for her OA, resulting in analgesic nephropathy; HY for 2CK Family Med forms.

- 28F + painful wrists and knee + history of thyroiditis + Q asks for next step in management à answer

= arthrocentesis to diagnose RA; if Q asks what you’re looking for on arthrocentesis, answer =

leukocytes. Student asks, “But what about antibodies though?” I agree with you, but 2CK forms are

obsessed with “arthrocentesis” or “aspiration of the knee joint” as the answer for next best step in

diagnosis/management for these questions; they will not force you into a box where you need to

choice antibodies versus arthrocentesis.

- 29F + history of intermittent wrist and knee pain + presents today with fever 101 F and a hot, red,

painful knee; Q wants next best step in diagnosis? à answer = arthrocentesis; diagnosis is septic

arthritis in patient with underlying RA.

- 59M + pain in his hands + x-ray is shown below; Q asks the mechanism for this patient’s condition:

o Answer = “degenerative joint disease”; diagnosis is OA; image is showing Heberden nodes. If

you look at x-ray, the DIPs “look a little swollen”; if you get a hand x-ray like this on USMLE

MEHLMANMEDICAL.COM 41
MEHLMANMEDICAL.COM

and have no idea what you’re looking at, just ask yourself, “Do the DIPs looked fucked up in

any way?” à if yes, answer is definitely not RA, and is probably OA.

- 34M + walks in bent over at the waist with chest pain + painful wrists and knee + hemoglobin of 10

g/dL; Q asks how to treat the patient’s hematologic findings à answer = “no specific therapy

indicated”; patient has RA; autoimmune diseases can present with serous pericarditis (patient bends /

leans forward to relieve chest pain) and anemia of chronic disease; even though we treat the

underlying condition (i.e., the RA in this case), I’ve seen NBME write “no specific therapy indicated”

for anemia of chronic disease in this situation; do not give EPO for AoCD unless the etiology is renal

failure. EPO is wrong answer otherwise.

- 49F + advanced rheumatoid arthritis + undergoing cholecystectomy; Q asks what needs to be done

prior to surgery à answer = “flexion and extension x-rays of cervical spine”; must look for atlantoaxial

subluxation in RA patients prior to intubation; answer can also be “CT of cervical spine”; I’ve seen

both on 2CK CMS forms.

- 51M + advanced rheumatoid arthritis + paresthesias of upper limbs; Q asks next step in diagnosis à

answer = MRI of cervical spine; Q has nothing to do with surgery, but upper limb neurologic findings

in RA patient suggest atlantoaxial subluxation.

- 42F + sore wrists and knee + diffuse pruritis + high serum cholesterol, direct bilirubin, and ALP; Q asks

for treatment of jaundice à answer = ursodeoxycholic acid (ursodiol); patient has rheumatoid

MEHLMANMEDICAL.COM 42
MEHLMANMEDICAL.COM

arthritis and primary biliary cirrhosis; remember that “autoimmune diseases go together”; USMLE

also wants you to know MOA of ursodiol is “decreased secretion of cholesterol into bile).

- 41M + sore knees + high ESR + low neutrophils + spleen palpable; Q just asks diagnosis à answer =

Felty syndrome; diagnosis is just asked straight-up like this on one of the 2CK forms.

- 22M + low back pain worse in the morning and gets better throughout the day + occasional hand

pain; Q wants to know next step in diagnosis à answer = x-ray of sacroiliac joints; diagnosis is

ankylosing spondylitis and psoriatic arthritis; both are linked via HLA-B27 (PAIR à Psoriasis,

Ankylosing spondylitis, IBD, Reactive arthritis).

- 29M + back pain worse in morning and gets better throughout the day + x-ray of spine shown below;

Q wants to know next step in management?

o Answer = slit-lamp examination; diagnosis is ankylosing spondylitis with x-ray showing

bamboo spine; USMLE wants you to know annual slit-lamp examination for anterior uveitis is

part of workup.

- 28M + silvery, scaly plaque on elbow + scraping of the plaque causes bleeding; Q asks for the most

appropriate treatment à answer = topical calcipotriene (vitamin D derivative); Tx of psoriasis is

topicals first à topical calcipotriene, then topical steroids (triamcinolone or hydrocortisone), then

coal tar.

- 44M + long history of plaque psoriasis that responded well to topicals + has increasing hand pain over

past year + x-ray shown below; Q wants to know the most appropriate treatment:

MEHLMANMEDICAL.COM 43
MEHLMANMEDICAL.COM

o Answer = methotrexate; x-ray shows pencil-in-cup deformity of DIP of 2nd digit;

methotrexate is first-line oral agent for systemic psoriasis (psoriatic arthritis), or if patient

has unremitting plaque psoriasis that fails topicals.

- 39M + long history of psoriasis that does not respond to topicals; Q asks for oral treatment +

methotrexate is not listed à answer = acitretin (oral vitamin A derivative used for psoriasis).

- 9F + painful knee + has had a few episodes similar to this before + hemoglobin 10 g/dL + MCV 72; Q

asks most likely hematologic diagnosis à answer = anemia of chronic disease due to JRA; you need to

know that MCV can absolutely be low in AoCD; this is exceedingly HY on 2CK forms; I see students all

of the time erroneously eliminate AoCD because of low MCV; absolute nonsense.

- 6M + runny nose for 3 days + now has left hip pain + temperature is 100.5 F + hip is not erythematous

or warm; patient can bear weight on the hip; Q asks for next step in management à answer =

ibuprofen; diagnosis is toxic (transient) synovitis; this is viral infection that, for whatever magical

reason, can cause inflammation of the hip that can be misdiagnosed as septic arthritis; recognize that

this is a common condition and if septic arthritis is less likely (i.e., kid can bear weight; there is no

leukocytosis; hip is not red and hot), do not draconianly stab the kid in the hip for a joint aspirate.

- 10M + painful left hip for 2 days + fever of 101 F + leukocytes elevated + cannot bear weight on the

hip + question says nothing about warmth or redness of hip; Q asks next best step à answer =

arthrocentesis; septic arthritis must be ruled out here; for these types of Qs, you have to use your

intuition; remember that for toxic synovitis, they’ll usually say viral infection precedes the hip pain +

no leukocytosis + patient can bear weight.

MEHLMANMEDICAL.COM 44
MEHLMANMEDICAL.COM

- 8F + intermittent knee pain and salmon-pink body rash + presents today with hot, red, painful knee

with limited mobility; Q wants to know most likely diagnosis (JRA not listed) à answer = septic

arthritis; as discussed earlier, JRA is a risk factor for septic arthritis, so although patient has JRA above,

the acute diagnosis is SA; USMLE will often give this type of Q where they have “toxic synovitis” as a

wrong answer choice (easy, since we discussed above toxic synovitis is hip pain after viral infection;

toxic synovitis is never the knee); salmon-pink body rash is buzzy for JRA, but I’d say it only presents in

half of NBME vignettes tops.

- 30F + arthritis of hands + low platelets + no other information; Q asks most likely diagnosis à answer

= SLE; you need to know lupus can present with low platelets due to antibodies causing peripheral

destruction; arthritis is most common presenting feature of lupus.

- 31F + arthritis of hands + low platelets + low WBCs + low hemoglobin; Q asks mechanism for this

patient’s hematologic findings à answer = “increased peripheral destruction”; wrong answer =

“defective bone marrow production”; in SLE, patients can have antibodies against their hematologic

cell lines; isolated thrombocytopenia is most common, but antibodies resulting in leukopenia and

erythropenia also can occur; this looks like aplastic anemia (e.g., from Parvo), but it’s not.

- 34M + sore hands and knee + malar rash + question asks what’s most likely to be deficient in this

patient à answer = C1q; for whatever reason, complement protein C1q deficiency is a cause of lupus.

Also remember that in acute flares, C3 decreases and dsDNA Abs increase.

- 36F + arthritis + thrombocytopenia + positive syphilis test; Q asks what’s most likely to be seen in this

patient à answer = antibodies against phospholipid; patients with SLE who have anti-phospholipid

syndrome can have false-positive VDRL syphilis screening.

- 45M + sore knee + drinks alcohol + takes daily aspirin for ischemic heart disease; Q wants to know

treatment for this patient’s condition à answer = indomethacin; steroids are wrong answer;

diagnosis is acute gout; student says, “But why do we give NSAID as Tx if patient is taking daily aspirin,

which is an NSAID?” à it’s what the NBME exam wants; daily low-dose aspirin is actually a risk factor

for acute gout.

- 45M + sore knee + joint aspirate shows needle-shaped crystals + patient has history of NSAID allergy;

Q asks most appropriate treatment (steroids not listed) à answer = colchicine; used for acute gout in

patients who can’t take NSAIDs (e.g., peptic ulcers or allergy).

MEHLMANMEDICAL.COM 45
MEHLMANMEDICAL.COM

- 45M + renal insufficiency + sore hands + tophi visible; Q asks most appropriate treatment for acute

presentation; answer = steroids; in renal insufficiency, avoid NSAIDs and colchicine.

- 45M + presents with swollen and painful big toe; Q asks which of the following will decrease

recurrence of this patient’s condition à answer = allopurinol; xanthine oxidase inhibitors (i.e.,

allopurinol or febuxostat) are first-line for chronic gout to decrease recurrence.

- 45M + being treated in hospital with IV nafcillin for MSSA endocarditis + is administered probenecid;

Q asks reason for this management à answer = decreases renal clearance of beta-lactam; probenecid

inhibits organic anion transporter (OAT), which both prevents reabsorption of uric acid and secretion

of beta-lactams; uricosurics not used first-line for chronic gout because they increase risk of uric acid

stones.

- 39M + left knee pain + hyperpigmentation of forearms + increased serum glucose; Q asks what is

most likely to be seen on joint aspirate à answer = calcium pyrophosphate; diagnosis is pseudogout;

biggest risk factors are hereditary hemochromatosis and primary hyperparathyroidism; patient here

has “bronze diabetes,” which is buzzy presentation for hemochromatosis; hyperpigmentation is

hemosiderin deposition; high glucose is from iron deposition in tail of pancreas.

- 50F + painful hands with soreness of DIPs + overweight + serum calcium 11 mg/dL (NR 8.4-10.2); Q

asks most likely diagnosis à answer = pseudogout, not OA; pseudogout can present as an OA-like

presentation in patients with primary hyperparathyroidism or hereditary hemochromatosis.

- 40M + painful knee + joint aspirate shows rhomboid positively birefringent crystals; Q asks most

appropriate treatment à answer = indomethacin; treat acute pseudogout exactly the same as

“regular” gout. However, for chronic pseudogout, treat the underlying condition.

- 40M + sore shoulder + ESR not elevated; Q asks what’s most likely to be seen on joint aspiration à

answer = basic calcium phosphate à BCP crystals cause “Milwaukee shoulder,” which is a cold (non-

inflammatory) shoulder effusion.

- 28M + painful red knee + fever 102 F + went hiking for 12 hours two days ago; Q asks for the most

likely organism à answer = Staph aureus; USMLE wants you to know that otherwise young, healthy

patients who’ve had recent microtrauma (e.g., hiking, soccer/kickboxing tournament) or overt trauma

(i.e., car accident where knee hit the dashboard) can get septic arthritis.

MEHLMANMEDICAL.COM 46
MEHLMANMEDICAL.COM

- 24M + med student + knocked up however many chicks the past year + presents with sore knee +

joint aspiration shows no organisms; Q asks the most likely diagnosis à answer = Gonococcal

arthritis; if Q is overt about sexual activity in younger patient, they want Gonococcus.

- 26F + history of rheumatoid arthritis + presents with hot, red, painful knee and fever of 102 F; Q asks

most likely organism à answer = Staph aureus; if Q doesn’t emphasize unprotected sexual activity +

patient has clear risk factor for SA, then S. aureus is more likely than Gonococcus.

- 14F + sickle cell + painful, red, hot knee; Q asks most likely organism à answer = Staph aureus;

Salmonella, although a common cause of osteomyelitis in sickle cell, there is not increased risk of it

causing septic arthritis.

- 28M + sexually active + painful acromioclavicular joint + gram-stain of joint aspirate shows gram-

negative diplococci; Q asks the next best step in management à answer = “culture of joint aspirate”;

weird question and answer, but this is on 2CK NBME; disseminated Gonococcus can cause septic

arthritis of a variety of joints; after gram stain is performed, you still want to do a culture of the

aspirate to check for antibiotic sensitivities.

- 27M + painful knee and wrist + cutaneous papules along the wrist; Q asks for the most appropriate

treatment à answer = ceftriaxone + azithromycin; diagnosis is Gonococcal arthritis; HY to know it can

present with cutaneous papules; must co-treat for chlamydia with azithromycin or doxycycline.

- 23M + sore hands and knees + urethral discharge + red eyes; Q asks most likely organism à answer =

Chlamydia; wrong answer is Gonorrhea; reactive arthritis is classically Chlamydia; Gonococcus does

not cause reactive arthritis.

- 17M + pain, redness, and swelling of shaft of left forearm + fever of 103 F + WBCs 14,000/µL; Q wants

most likely organism à answer = Staph aureus; Dx is osteomyelitis; S. aureus is most common

organism overall, especially when etiology is idiopathic.

- 8M + sickle cell disease + pain in foot for 3 weeks + fever 102 F + WBCs 14,000/µL; Q wants most

likely organism à answer = Salmonella spp.; if Q gives you sickle cell with osteomyelitis, the organism

they want is Salmonella. You still need to use your head though. If they say “gram-positive coccus” in

patient with sickle cell, that would be Staph aureus, not Salmonella. The latter is a gram-negative rod.

- 42F + severe diabetes + gangrenous foot ulcer + sterile probe placed to base of ulcer and cultured; Q

wants to know most likely organism à answer = polymicrobial on new 2CK NBME. Both Staph aureus

MEHLMANMEDICAL.COM 47
MEHLMANMEDICAL.COM

and Pseudomonas are wrong answers, which settles the long debate for what USMLE wants for this

answer.

- 47M + immigrant to USA from Albania + history of tuberculosis + pain over T9/T10 vertebrae; Q wants

diagnosis à answer = vertebral osteomyelitis (Pott disease).

- 28M + migrant worker + rigid abdomen + fever + holds right lower extremity in extension + PPD

shows 16 mm induration; Q asks which muscle is most likely to be affected in this patient à answer =

psoas major; you don’t have to worry about the anatomy here; just know that TB can cause psoas

abscess.

- 56M + IV drug user + diabetes mellitus type II + left-sided back pain radiating to flank + fever 102 F +

pyelonephritis not listed as answer; Q wants diagnosis à answer = psoas abscess; just be aware of it

as a DDx; most likely organism is Staph aureus; Tx = surgical drainage + IV antibiotics.

Other random joint/MSK disorders


- Tennis elbow.
- Lateral elbow pain worsened when patient extends wrist against resistance.
Lateral epicondylitis
- NBME asks “extensor carpi radialis brevis” as answer for site of inflammation.
- Tx = “forearm strap” on 2CK FM forms.
- Golfer elbow.
- Medial elbow pain worsened when patient flexes wrist against resistance.
Medial epicondylitis
- Inflammation at flexor carpi radialis and pronator teres.
- Tx = forearm strap.
- “Nursemaid elbow”; HY for 2CK Peds.
- Child stops using arm + arm extended + pronated.
Radial head - Hx of child having arm pulled/yanked, or child holding hands and running with
subluxation older sibling + the child falls, resulting in elbow pull.
- Tx = hyper-pronation, OR supination when arm partially flexed. Either is correct.
Both will not be listed at the same time as answers.
- Elbow pain, usually following contact injury.
Olecranon bursitis
- Tx = compression bandage + NSAIDs. Steroid injection is wrong answer on USMLE.
- Vignette will be lateral hip pain that is worsened with palpation + lying on one’s
Trochanteric bursitis side in bed.
- Tx = NSAIDs.
- USMLE will give inflammation of knee joint that sounds like septic arthritis, but
they will say there’s no joint effusion. This is how it presents in a 2CK NBME
Septic bursitis vignette, where students constantly ask “why not septic arthritis?” And they say in
the vignette there’s no joint effusion.
- Tx = antibiotics.
- Aka housemaid’s knee; presents as anterior knee pain in people who are
Prepatellar bursitis frequently on their knees (painters, plumbers, etc.).
- Tx = NSAIDs.
- Inflammation of patellar tendon.
Patellar tendonitis - The answer on USMLE if they describe anterior knee pain that initially occurs only
(“Jumper’s knee”) after finishing sports (e.g., basketball game), then progresses to more chronic pain.
- Tx on NBME = “quadricep strengthening exercises.”
Patellofemoral - Presentation is annoyingly similar to patellar tendonitis, but do not confuse.
instability

MEHLMANMEDICAL.COM 48
MEHLMANMEDICAL.COM

- Patellofemoral instability is misalignment of the patella at the trochlear groove of


the femur.
- Q can mention crepitus.
- Shows up on 2CK Peds CMS form 6 as teenage girl who has knee pain worse after
jumping or running + has crepitus à answer = “patellofemoral instability” (patellar
tendonitis / “jumper’s knee” not listed as answer).
- Aka chondromalacia patellae; name implies softening of cartilage in the knee.
- The answer on NBME if they say pain that worsens when sitting for long periods of
Patellofemoral pain
time, or when going up or down stairs.
syndrome
- Classic in obesity or those who squat heavy weight (knees think you’re obese).
- Tx = quadriceps strengthening exercises.
- ACL is answer if (+) anterior drawer test or Lachman test à excessive anterior
Anterior cruciate displacement of tibia relative to femur.
ligament injury - Classically injured when knee is hyper-extended, or with a rotational force on a
fixed, planted knee.
- PCL is answer if (+) posterior drawer test à excessive posterior displacement of
Posterior cruciate
tibia relative to femur.
ligament injury
- Classically injured when knee hits the dashboard in car accident.
- LCL is the answer if varus test induces excessive lateral motion of the knee
Lateral collateral compared to the unaffected side.
ligament injury - Varus test = hand placed on medial knee and pushing outward + other hand
placed on lateral ankle and pushing inward.
- MCL is the answer if valgus test induces excessive medial motion of the knee
Medial collateral compared to the unaffected side.
ligament injury - Valgus test = hand placed on lateral knee and pushing inward + other hand placed
on medial ankle and pulling outward.
- Lateral knee pain where patient experiences “locking” or “catching” of the knee in
partial flexion.
Lateral meniscal tear
- Diagnosed with McMurray test à internal rotation of leg with concurrent knee
extension causes lateral knee pain / “catching.”
- Medial knee pain where patient experiences “locking” or “catching” of the knee in
partial flexion.
Medial meniscal tear
- Diagnosed with McMurray test à external rotation of leg with concurrent knee
extension causes medial knee pain / “catching.”
- Refers to a trio injury of the ACL, medial collateral ligament, and either the medial
or lateral meniscus.
“Unhappy triad” - Students are sometimes fanatical about this triad as though it has yieldness.
USMLE doesn’t give a fuck. I cannot recall a single NBME question that has ever
assessed this. This Dx primarily resides within the domain of Qbank, not the NBME.
Pes anserine bursitis - The answer on USMLE if patient has inferomedial knee pain.
- The answer on USMLE if they say lateral knee pain, usually in a runner.
Iliotibial band - Iliotibial band runs from the hip to the knee. Pain may occur anywhere along the
syndrome hip, lateral thigh, and lateral knee, but is worst in the latter.
Tx = conservative with physiotherapy; NSAIDs for pain.
- Buzzy vignette is knee pain in fast-growing teenage male who plays soccer. Don’t
Osgood-Schlatter pigeon-hole things, but that’s classic vignette.
disease - Inflammation of the patellar ligament at the tibial tuberosity.
- Mechanism is repeated stress on the growth plate of the superior tibia.
- The answer on USMLE if they give severe heel pain that is worst when first getting
Plantar fasciitis
out of bed in the morning.
- The answer on USMLE if patient has pain + abnormal growth occurring between
the 2nd and 3rd metatarsals, usually worsened with high-heel shoes.
- Benign growth/tumor of intertarsal nerve; cause is idiopathic.
Morton neuroma
- First step in diagnosis is x-ray to rule out arthritis + fractures. Ultrasound is then
done to confirm Dx, which will show thickening of interdigital/intertarsal nerve.
- Tx = orthotics (comfortable shoes) + steroid injection.

MEHLMANMEDICAL.COM 49
MEHLMANMEDICAL.COM

- Aka Charcot joint, where patient injures joint due to lack of joint sensation from
peripheral neuropathy.
Neurogenic joint - Usually seen in diabetes; can also be seen in neurosyphilis.
- The answer on USMLE when they say diabetic patient has “disorganization of the
tarsometatarsal joints” on foot x-ray.
- No, this is not a joke. This is the answer straight-up on a 2CK NBME form.
- Vignette is healthy child age 3-12 who awakens from sleep with throbbing pain in
Growing pains
the legs.
- No treatment necessary. You just need to know this Dx is exists and isn’t a troll.
- Aka Blount disease.

Tibia vara

- Bowing of the tibias after the age of 2 years in a patient whom rickets has been
ruled out.
- Can be unilateral or bilateral.
- Bowing of one or both tibias is sometimes normal until age 2 years.
- Treatment is surgery (osteotomy).
- Aka clubbed foot.

Talipes equinovarus

- USMLE just wants you to know that this is treated initially with serial casting.
- Usually idiopathic; can be seen in Potter sequence.
- Not the same as rocker-bottom foot (aka congenital vertical talus), seen in Edward
syndrome.
- You just need to know this is fancy name for a child born with multiple joint
contractures.
Arthrogryposis
- If they give you a child with not just a clubbed foot, but also knee and/or elbow
contractures, etc., the answer is arthrogryposis.

MEHLMANMEDICAL.COM 50
MEHLMANMEDICAL.COM

- 34M + left knee pain that is worse when sitting for long periods of time, or when going up or down

stairs; Q asks for treatment à answer = “quadriceps strengthening exercises”; diagnosis is

chrondromalacia patellae (aka patellofemoral pain syndrome).

- 50M + started playing recreational basketball two years ago + knee pain initially worse with activity

but now is present even when not playing; Q wants diagnosis à answer = patella tendonitis

(“Jumper’s knee”).

- 16F + plays volleyball + pain in knee past 6 weeks worse when running or jumping + exam shows

crepitus; Q wants diagnosis à answer = “patellofemoral instability” (patellar tendonitis / “jumper’s

knee” not listed as answer); shows up on 2CK CMS Peds form 6.

- 65M + Q gives big paragraph of unnecessary info + they tell you guy has lateral hip pain that is worse

when palpated and while lying on his side in bed; Q just asks diagnosis à answer = trochanteric

bursitis.

- 50M + started playing recreational basketball two years ago + knee pain worse with activity but now

is present even when not playing; patient had fracture to proximal tibia 30 years ago that was treated

with open reduction and internal fixation; x-ray shows varus deformity of knee; Q wants diagnosis à

answer = post-traumatic arthritis, not patella tendonitis.

- 28M + playing football + injures knee + increases anterior displacement of tibia on examination; Q

wants to know what is injured on the following MRI:

MEHLMANMEDICAL.COM 51
MEHLMANMEDICAL.COM

o Answer = anterior cruciate ligament (choice B). A = femur; C = PCL; D = tibia.

- 39M + right knee pain + started after working on his knees while fixing kitchen sink; Q asks next best

step in management à answer = ibuprofen; diagnosis is prepatellar bursitis (“housemaid’s knee”).

- 34F + painful, red, warm knee + joint aspiration shows no effusion; Q asks most likely diagnosis à

answer = septic bursitis; vignette will sound like septic arthritis, but there will be no effusion.

- 15M + plays a lot of soccer + 5’11” + painful knee; Q wants mechanism of pathology à answer =

inflammation of patellar ligament at tibial tuberosity; diagnosis is Osgood-Schlatter.

- 22M + plays basketball + right heel pain for 5-10 minutes after getting out of bed in the morning; Q

asks mechanism of pathology à answer = inflammation of planta fascia.

- 16F + in car accident + hits her left knee on the dashboard + physical exam shows excessive posterior

displacement of the tibia; Q asks for diagnosis à answer = PCL injury (dashboard injury).

- 19M + hurt his knee wakeboarding + physical exam shows excessive motion of lateral aspect of knee

when pressure applied to medial knee; Q asks diagnosis à answer = lateral collateral ligament injury;

test described is the varus test.

- 24M + hurt his knee wakeboarding + medial knee pain + vignette mentions “catching” or “locking” of

the knee; Q wants diagnosis à answer = medial meniscal tear.

- 6F + awakens from sleep with terrible throbbing in her legs + 60th percentile for height and weight; Q

asks diagnosis (and they list like 15); answer = growing pains; no treatment necessary.

- 4M + serum vitamin D is normal + x-ray is shown below; Q wants diagnosis:

MEHLMANMEDICAL.COM 52
MEHLMANMEDICAL.COM

o Answer = tibia vara (Blount disease); bowing of the tibias can be normal up until age 2; after

age 2 it is called tibia vara, if rickets has been excluded. 2CK Peds forms will write the answer

as “tibia vara,” not Blount disease.

- 65M + BMI 30 + pain on the inner side of the left knee a few centimeters below the kneecap; Q wants

diagnosis à answer = pes anserine bursitis; the answer on USMLE if they give inferomedial knee pain.

- Neonate born at term + perfectly healthy + feet bent inward; Q wants to know treatment à answer =

serial casting; observation is wrong answer; diagnosis is talipes equinovarus (clubbed feet).

- Neonate born at term + clubbed feet + multiple joint contractures; Q asks for diagnosis à answer =

arthrogryposis.

- 44F + wears high-heels + has severe pain under foot along the 2nd and 3rd metatarsals; Q asks for

treatment à answer = orthotics first, then steroid injection second; Dx is Morton neuroma.

- 70M + long-standing diabetes + pain and ulceration of foot + x-ray shows disorganization of the

tarsometatarsal joints; Q wants most likely explanation for these findings; answer = “absence of

normal joint sensation”; Dx is neurogenic joint (aka Charcot joint).

HY Pediatric hip disorders for USMLE


- Aka developmental dysplasia of the hip.
- Mechanism is “poorly developed acetabulum.”
- Initial diagnosis is with Ortolani and Barlow maneuvers, where a “clicking
and clunking” is elicited on physical exam.
- After the O&A maneuvers, next best step is ortho referral. Sounds wrong,
Primary hip dysplasia
but if it’s listed, it’s the answer before going to imaging.
- Definitively diagnose with hip ultrasound if <6 months of age; hip x-ray if
>6 months of age.
- Treatment is “abduction harness,” aka Pavlik harness, which positions the
child’s legs in a frog-leg-appearing manner.
- Aka idiopathic avascular necrosis of the femoral head.
- If the etiology for the avascular necrosis is known (i.e., Gaucher, sickle
cell, corticosteroids), then the diagnosis is just “avascular necrosis,” not
LCP.
- Vignette will be child 5-8 years old with hip pain.
Legg-Calve-Perthes
- First step in diagnosis is hip x-ray, which will show a “contracted” or
flattened femoral head. The word “contracted” is HY and synonymous with
avascular necrosis.
- If x-ray is negative, diagnose with bone scan or MRI (on 2CK Peds form).
- Tx = hip replacement.
- Classic vignette is a 10-13-year-old (pre-adolescent) overweight boy with
a painful limp.
- NBME will write answer / mechanism as “displacement of the epiphysis
Slipped capital femoral
of the femoral head.”
epiphysis (SCFE)
- Resources tend to emphasize obesity as the main risk factor, but maybe
only ~1/2 of NBME Qs for SCFE give the child as overweight. This causes
problems for students, where they rely on seeing high BMI to think SCFE.

MEHLMANMEDICAL.COM 53
MEHLMANMEDICAL.COM

- This has led me to conclude that the age matters the most, since they will
always give a kid who’s about 10-13-ish. If they give you a kid who’s
younger, think LCP instead.
- 2CK NBME Q gives 13M with painful gait + no mention of weight à
answer is SCFE.
- Another 2CK Q outright says BMI 20 in a 13-year-old who’s 6 feet tall, and
answer is SCFE.
- X-ray shows “ice cream falling off the cone.”
- Offline 2CK gives Q where they say x-ray in 5-year-old shows “contracted
capital femoral epiphysis” à answer is LCP, not SCFE. As I said above,
“contracted” is HY for LCP. In this case, the younger age + the word
“contracted” win over the words “femoral capital epiphysis.”
Tx = surgical pinning.

- 4-month-old boy + clicking and clunking on physical examination; Q wants next best step in imaging

à answer = ultrasound; diagnosis is primary hip dysplasia; do ultrasound under 6 months; x-ray over

6 months.

- 4-month-old boy + clicking and clunking on physical examination; Q wants next best step in

management à answer = ortho referral; ultrasound is wrong answer. Referrals in general sound

wrong for USMLE, but you need to know this exception in the case of primary hip dysplasia.

- 7-month-old boy + clicking and clunking on physical examination; Q asks, in addition to ortho referral,

what is next best step in management à answer = x-ray; ultrasound is wrong answer.

- 4-month-old girl + clicking and clunking on physical exam; Q asks for mechanism of patient’s condition

à answer = “poorly developed acetabulum.”

- 4-month-old girl + clicking and clunking on physical exam + ultrasound of hip shows poorly developed

acetabulum; Q asks for next best step in management à answer = abduction harness; students tend

to memorize “Pavlik harness,” but you need to know USMLE often writes this as “abduction harness.”

- 5M + right hip pain + walks with limp + x-ray shows contracted capital femoral epiphysis; Q asks

straight-up for diagnosis à answer = Legg-Calve-Perthes, not SCFE. Once again, it’s on the NBME

exam, so if you think it’s weird or disagree, don’t take it up with me; take it up with NBME.

- 8M + sickle cell disease + painful hip pain and limp; Q asks diagnosis à answer = “avascular necrosis”;

Legg-Calve-Perthes is wrong answer; remember that LCP is idiopathic avascular necrosis; if the cause

is known (i.e., sickle cell, Gaucher, corticosteroids), LCP is wrong.

MEHLMANMEDICAL.COM 54
MEHLMANMEDICAL.COM

- 8M + hepatomegaly + hip pain + macrophages with wrinkled tissue appearance; Q asks mechanism

for disorder à answer = deficiency of glucocerebrosidase; diagnosis is Gaucher disease; for Gaucher,

you just need to know it’s the answer if you get a glycogen storage disease + a bone problem.

- 9F + painful left hip + x-ray shows no abnormalities + diagnosis is made using bone-scan; Q just asks

the most likely diagnosis à answer = Legg-Calve-Perthes; x-ray can be negative initially; in this case,

the next best step is bone-scan or MRI.

- 7F + hip pain + x-ray is shown below; Q asks for diagnosis:

o Answer = Legg-Calve-Perthes; student says, “No idea what I’m looking at.” Relax. If we try to

imagine, the right femoral head (left side of image) looks “contracted” or “flattened.” Once

again, this is HY and buzzy for avascular necrosis.

- 59F + advanced rheumatoid arthritis managed with multiple medications + MRI of pelvis is shown; Q

wants diagnosis:

MEHLMANMEDICAL.COM 55
MEHLMANMEDICAL.COM

o Answer = avascular necrosis; etiology is chronic corticosteroids in the setting of advanced

autoimmune disease; the right femoral head (left side of image) appears hypointense (more

black). The necrotic / lack of bone in the black superior portion of the femoral head means

the remaining white part of the femoral head is “flattened” or “contracted.” The left femoral

head (right side of image) shows a small area of necrosis as well (black medial portion).

- 13M + left hip pain + walks with antalgic gait + 95%tile for weight; Q asks mechanism for patient’s

condition à answer = “displacement of the epiphysis of the femoral head”; diagnosis is SCFE; classic

vignette is overweight pre-adolescent boy with a painful limp; “antalgic gait” just means walking with

a painful limp; it is non-specific and I’ve seen this descriptor used in LCP vignettes as well.

- 12M + left hip pain + BMI 20 + 6’0” + x-ray is shown below; Q asks diagnosis:

o Answer = slipped capital femoral epiphysis; x-ray shows the “ice cream slipping off its cone”;

student is confused because the Q says BMI is normal; as I said earlier, although the kid

being overweight is HY and buzzy, plenty of NBME Qs either don’t mention the BMI or will

say it’s normal; it’s the age that matters (i.e., 10-13).

- “Do I need to know the mechanism of PTH on bone?” à Yes, it’s HY.

o PTH binds to osteoblasts, not -clasts. This sounds paradoxical, since osteoblasts build bone

and PTH has the role of causing bone resorption (in order to increase serum calcium).

o After PTH binds to the osteoblast, it will express RANK-ligand (RANK-L) on its cell surface.

o RANK-L then binds to RANK receptor on osteoclasts, which in turn resorb (break down) the

bone.

MEHLMANMEDICAL.COM 56
MEHLMANMEDICAL.COM

- “What do I need to know about OPG in relation to RANK-L. Can you quickly clarify.”

o Osteoprotegerin (OPG) is protective of bone loss. It is a RANK receptor decoy (i.e., it looks

like RANK receptor), and therefore acts as an endogenous competitive inhibitor of RANK-L.

o In other words, if more OPG is present, then RANK-L will bind to OPG instead of RANK

receptor, thereby resulting in less osteoclast activation and less bone resorption.

o OPG is suppressed by cytokines (namely IL-1). Therefore, too much cytokine activity can

cause osteoporosis. In fact, IL-1 is also known as osteoclast-activating factor.

o Estrogen suppresses cytokine activity (IL-1) on bone, resulting in an increased OPG/RANK-L

ratio and lower risk of bone resorption. In post-menopausal state, ¯ estrogen à ¯

suppression of IL-1 on bone à ¯ OPG/RANK-L ratio à ­ bone resorption à osteoporosis.

- “What do I need to know about alkaline phosphatase (ALP) and bone?”

o USMLE really wants you to know that increased osteoblast activity causes high ALP. This is

why ALP levels go up when PTH is high, or why ALP is elevated in Paget disease of bone.

o There is Q on one of the Step 1 NBMEs where they give weird craniofacial syndrome + serum

ALP is low; they ask what cell is defective à answer = osteoblast; students have freaked out

to me over this Q (“We need to know this crazy/weird syndrome??”) à No. The syndrome is

irrelevant. The Q says low ALP, so they just want the osteoblast as the cell that’s fucked up.

HY Bone density/architectural conditions


- Bone density >2.5 SD below the mean as compared with young adult woman. Osteopenia
is 1.5-2.5 SD below the mean.
- Bone densitometry done at age 65 (2CK Family medicine).
- If Q forces you to choose between female gender and age as most important risk factor,
choose female gender.
- If Q gives you a female and forces you to choose between family history and age, choose
family history.
- Males are unlikely to develop osteoporosis, even with family history of females with the
disorder.
- If Q gives two women without family history and asks what is most protective against
Osteoporosis osteoporosis, answer is ethnicity. Black race is protective against osteoporosis.
- If Q gives old woman who has femoral fracture + no mention of osteoporosis in the
question, answer = “activity level before fracture” as most important predictor of success
in the rehabilitation of the patient à weight-bearing exercise during life is protective
against osteoporosis later.
- USMLE loves corticosteroids and Cushing as causes of osteoporosis.
- Compression fractures = osteoporosis on USMLE; e.g., patient with RA on steroids who
has compression fracture à easy Dx of osteoporosis.
- As discussed above, USMLE loves cytokine effect on bone causing osteoporosis; they will
show a highly trabecular/hollowed out vertebral body, then ask what caused this à
answer = “IL-1” straight up.

MEHLMANMEDICAL.COM 57
MEHLMANMEDICAL.COM

- Low/low-normal BMI causing osteoporosis is HY; 2CK NBMEs have a couple of nonsense
Qs where they give BMI of 19 and 20 in young woman, where they ask what patient is at
increased risk of; answer = osteoporosis. Student says, “Wait, but isn’t low BMI under
18.5?” I agree. But it’s on NBME.
- Metatarsal stress fracture HY in low-BMI young female runners who have low bone
density.
- USMLE also is known to assess low vitamin D in the setting of intestinal malabsorption
(i.e., CF, Crohn) as cause of osteoporosis, even though that makes no sense, since low Vit
D causes osteomalacia.
-Serum calcium, phosphate, PTH, and ALP are all normal in osteoporosis.
- Tx = weight-bearing exercise first (NBME has “go for a long walk outside daily” as correct;
wrong answer = “increase participation in swimming pool-based exercise classes to at
least three times weekly”).
- Calcium and vitamin D are the first medical / pharmacologic treatment.
- Bisphosphonates can be used after Ca2+/VitD.
- Teriparatide is an N-terminus PTH analogue that stimulates bone development.
- Denosumab is a RANK-L monoclonal antibody.
- Osteomalacia = vitamin D deficiency in adults; rickets = vitamin D deficiency in children.
- Rickets = craniotabes (soft skull), rachitic rosary (bony knobs at costochondral junctions),
genu varum (bowing of tibias).
- Osteomalacia means softening of bone; activated vitamin D (1,25) is necessary to convert
unmineralized osteoid into mineralized hydroxyapatite, therefore hardening bones.
- In both rickets and osteomalacia, patient will have “increased unmineralized osteoid,” or
“deficient mineralization of osteoid.”
Osteomalacia/ - Important cause of Vit D deficiency on USMLE is intestinal malabsorption (i.e., CF, Crohn).
Rickets Chronic pancreatitis (pancreatic burnout leading to steatorrhea) also HY.
- Renal failure leading to osteomalacia (due to ¯ synthesis of 1,25) is called renal
osteodystrophy; if they ask the bone condition patient has in renal failure, choose
osteomalacia.
- “Pseudofracture” on x-ray is buzzy finding in osteomalacia/rickets.
- Patients have ¯ Ca2+, ¯ PO43-, ­ PTH (due to ¯ negative feedback at parathyroid glands).
- I discuss vitamin D metabolism and PTH/endocrine relationships in high detail in the HY
Arrows PDF.
- Osteoclast dysfunction resulting in recurrent fractures in children due to bone density
being too high. Sounds weird, but bone strength is based on balanced internal
architecture of canals and networks, not just density alone.
- HY DDx against osteogenesis imperfecta and child abuse.
- Osteoclast dysfunction is due to deficiency of carbonic anhydrase II. This enzyme inside
Osteopetrosis
osteoclasts normally allows osteoclasts to form H+ to resorb bone.
- Granulocyte-macrophage colony-stimulating factor (GM-CSF) causes the differentiation
of osteoclast precursors into osteoclasts. This is asked on one of the Step 1 NBME forms,
where they give osteopetrosis and ask something along the lines of, “The cell lineage
that’s fucked up in this condition requires which of the following” à answer = GM-CSF.
- Collagen I defect that results in recurrent fractures in a child; important DDx are child
abuse and osteopetrosis.
Osteogenesis - Blue sclerae too buzzy and often not mentioned.
imperfecta - Conductive hearing loss due to defective ossicles (middle ear bones).
- Many different types of OI, some resulting in miscarriage. Harder vignette can mention
child with multiple fractures, where the mom has Hx of recurrent miscarriage.
- Idiopathic disorder of increased bone turnover. Bone is described as having mixed
osteoblastic and -clastic phases, where bone appears heterogenous on x-ray.
- Buzzy vignette is male over 50 who’s hat doesn’t fit him anymore + has tinnitus
Paget disease (narrowing of acoustic foramina).
- 19/20 questions will give isolated increase in serum ALP. You need to know Ca2+, PO43-,
and PTH are all normal in Paget disease. There is one Q on a 2CK CMS form where ALP is
given as not elevated, but it’s a one-off Q and rare.

MEHLMANMEDICAL.COM 58
MEHLMANMEDICAL.COM

- High-output cardiac failure can occur due to intraosseous AV-fistulae, where patient has
an S3 heart sound with high, rather than low, ejection fraction.

- 62F + shows graph with lines of bone density for two different women without family history of

osteoporosis; Q asks what is the reason why one woman has higher bone density than the other à

answer = ethnicity; black race is protective against osteoporosis.

- 64F + image of vertebral column shown below; Q asks what is responsible for this patient’s condition:

o Answer = Interleukin-1; IL-1 is aka osteoclast-activating factor; post-menopausal state à ¯

estrogen à ¯ suppression of cytokines (namely IL-1) on bone à ¯ OPG/RANK-L ratio à ­

bone resorption à osteoporosis.

- 22F + runs a lot + BMI 20 + no other issues; Q asks what patient is at increased risk for à answer =

osteoporosis; makes no sense, as BMI isn’t low, but this is asked on 2CK NBME form.

- 34M + bitemporal hemianopsia + 20-lb weight gain past 6 months + compression fracture; Q asks

what the tumor is secreting à answer = ACTH; patient has Cushing disease; cortisol can cause

osteoporosis; compression fracture = osteoporosis on USMLE; student asks, “But why can’t TSH be

the answer? Can’t hyperthyroidism cause osteoporosis?” à Two points: 1) Patient here has weight

gain, not weight loss. If patient has hyperthyroidism, we wouldn’t have weight gain; 2) Despite this

notion that hyperthyroidism could in theory cause osteoporosis, I’ve never seen that assessed on

NBME material.

- 72F + fractures femur + no mention of osteoporosis; Q asks, during the rehabilitation process, what is

the most important predictor of success in this patient à answer = “activity level before the fracture”

à protective of bone density / osteoporosis.

MEHLMANMEDICAL.COM 59
MEHLMANMEDICAL.COM

- 4M + came to USA from Ireland with parents one year ago + has bowed tibias; Q asks serum calcium,

phosphate, and PTH à answer = low calcium, low phosphate, high PTH; diagnosis is rickets with genu

varum.

- 70F + type II diabetes + renal failure + left hip pain; Q asks most likely MSK diagnosis à answer =

osteomalacia; renal failure causes low vitamin D, resulting in osteomalacia; renal osteodystrophy =

osteomalacia cause by renal failure.

- 70F + renal failure + left hip pain + x-ray shows pseudofracture; Q asks what’s most likely responsible

for this patient’s condition à answer = “decreased intestinal absorption of calcium”; we have

osteomalacia due to low 1,25-D3, causing reduced intestinal absorption. “Pseudofracture” is buzzy for

osteomalacia.

- 70F + left hip pain + x-ray shows pseudofracture; Q asks arrows for calcium, phosphate, PTH, and

calcitriol à answer = low calcium, high phosphate, high PTH, low calcitriol (aka 1,25-D3); even though

vitamin D is low, phosphate is always high in renal failure; so even though, yes, vitamin D deficiency

causes low calcium and low phosphate in the setting of rickets/osteomalacia, if the patient has renal

failure, the renal failure “wins” when it comes to phosphate. Exceedingly HY.

- 3M + recurrent fractures + increased bone density; Q asks which type of molecular signaling is fucked

up à answer = GM-CSF; diagnosis is osteopetrosis, which is caused by either deficiency of carbonic

anhydrase II, or a problem with GM-CSF signaling; osteoclasts are defective.

- 6M + recurrent fractures + engages well with the parents + mom has history of recurrent

miscarriages; Q asks mechanism for patient’s condition à answer = “defect of procollagen synthesis”;

diagnosis is osteogenesis imperfecta (collagen I gene mutation); USMLE doesn’t actually care about

the exact collagen step that is defective; the answer will be the only one that refers to collagen.

- 56M + tinnitus in left ear + S3 heart sound + ejection fraction 70% (NR 55-70); Q asks next best step in

management à answer = “check serum ALP levels”; diagnosis is Paget disease of bone; almost all Qs

will give isolated increase in serum ALP (calcium, phosphate, and PTH are normal); high-output

cardiac failure can occur from intraosseous AV fistulae.

- 16M + falls off bike while BMX dirt jumping + broken radius + cast applied + now has acutely

increased severe pain in arm; Q asks for most likely diagnosis à answer = compartment syndrome;

although circumferential burns are fixated on as hyper-HY for compartment syndrome on USMLE, it’s

MEHLMANMEDICAL.COM 60
MEHLMANMEDICAL.COM

to my observation NBME cares less about this and more about acute casting as the highest yield

cause. If a limb is placed in a cast before swelling has maximized, compartment syndrome can result,

where the patient experiences increasingly severe pain and paresthesias of the limb

- 20M + housefire + 3rd-degree burns all over legs + severe pain in left leg; Q asks for next best step à

answer = “measure compartment pressures”; treatment is then “fasciotomy.”

- 67F + ruptured diverticulitis + undergoes laparotomy; 12 hours after procedure, there is oozing from

suture site + partial dehiscence; Q just asks diagnosis à answer = “abdominal compartment

syndrome” à occurs when intra-abdominal pressure rises, causing multi-organ dysfunction à can be

due to peritonitis with edema, as well as massive volume resuscitation. Essentially, just know

compartment syndrome isn’t limited to the limbs – i.e., abdominal compartment syndrome “is a

thing” / exists.

- 34F + central chest pain + worsens when reaches behind her back and over her head + pain on

palpation; Q wants diagnosis à answer = costochondritis; any chest pain on USMLE that worsens

with change in positioning (except pericarditis, which worsens when leaning back), think MSK etiology

– i.e., costochondritis.

- 34F + pain in chest + worse when reaching behind her back and over her head + there is no pain on

palpation; Q wants treatment à answer = NSAIDs; diagnosis is costochondritis. I’ve seen NBME Qs

where they mention pain that changes with position and/or with palpation – i.e., it need not be both.

- 25M + recently recovered from viral infection + severe right lateral chest pain + no friction rub +

increased serum creatine kinase; Q wants diagnosis à answer = pleurodynia; this is asked on 2CK FM

forms. Despite the name, this has nothing to do with the lungs/pleura. It is intercostal muscle spasm

following viral infection (i.e., viral myalgia) that presents as lateral chest pain; CK can be elevated due

to increased muscle tone.

- 47M + alcoholic + found on bench in park + urine dipstick shows 2+ blood + urine RBCs negative; Q

wants most likely diagnosis à answer = rhabdomyolysis; alcoholic is huge risk factor; urinalysis

classically shows false-positive blood on dipstick, where the dipstick can’t differentiate between free

myoglobin and RBC hemoglobin, but light microscopy will be negative.

- 47M + alcoholic + found on bench in park + serum potassium elevated + urine shows brown

pigmented casts; Q wants most likely diagnosis à answer = rhabdomyolysis; urine can be either false-

MEHLMANMEDICAL.COM 61
MEHLMANMEDICAL.COM

positive blood on dipstick or characteristic of acute tubular necrosis (i.e., brown pigmented casts, or

dirty/muddy brown granular casts).

- 82F + found on floor in house by the staircase + 1+ blood on urine dipstick + urine light microscopy

shows 3-4 RBCs/hpf; Q wants diagnosis à answer = rhabdo. You need to know 3-4 RBCs/hpf is still

considered negative. This is on 2CK CMS forms and students get it wrong thinking that 3-4/hpf isn’t

considered negative, but it is.

- “What do I need to know about bone tumors?”

HY bone tumor points for USMLE


- Highest yield bone tumor for USMLE.
- Can occur in any long bone, as well as the pelvis/hip.
- USMLE describes it as “glistening” or “shiny” in appearance.
- The histo image below is highest yield bone tumor image on USMLE. I’ve seen this
described in NBME answer choices as “neoplastic chondrocytes embedded in lacunae,”
or “neoplastic chondrocytes filling lacunae.”

Chondrosarcoma

- This image shows up across numerous NBME and COMP exams.


- I’ve seen chondrosarcoma on Step 1 NBME written as “neoplastic chondrocytes filling
lacunae.”
- Most common primary bone cancer; usually in patients age 10-30.
- Rb mutations (congenital retinoblastoma) are associated with osteosarcoma (HY) –
i.e., 1-year-old boy has enucleation of eye for retinoblastoma; what is he at risk of
developing later in life? à answer = osteosarcoma.
- Can also occur in Paget disease of bone patients (older age).
Osteosarcoma
- Buzzy findings are “Codman triangle” and “Sunburst appearance.”
- Codman triangle = periosteal reaction with lifting of periosteum off the bone.
- Sunburst appearance = periosteal reaction described on NBME as “spiculated new
born formation.”

MEHLMANMEDICAL.COM 62
MEHLMANMEDICAL.COM

- The white arrow is the Codman triangle; the white star is the sunburst appearance.
- I’ve seen osteosarcoma on Step 1 NBME written as “pleomorphic neoplastic cells
producing new woven bone” as correct answer choice.
- The answer on USMLE if they give bone tumor in a child that presents similarly to
osteomyelitis (i.e., fever and bone pain in a kid).
- If bone scan is performed, it is most likely to show uptake in the diaphysis; in
contrast, osteomyelitis will show uptake in the metaphysis.
- Histo will show “small blue cells of neuroendocrine origin” and/or “onion-skinning.”
- I’ve seen an NBME answer choice say “closely packed, small, round, uniform
neoplastic cells.”

Ewing sarcoma

- Associated with t(11;22) translocation; don’t confuse this with the 22q11 deletion in
DiGeorge syndrome.
- The answer on USMLE if they give a bone tumor that presents with pain relieved with
Osteoid osteoma
NSAIDs.
- The answer on USMLE if they give a bone tumor in Gardner syndrome (Familial
Osteoma adenomatous polyposis + soft tissue tumors [usually osteomas or lipomas]).
- Benign bone tumors that usually grow from the skull.
- Aka osteoclastoma.
Giant cell tumor - Age of onset usually 20-40.
- Has a “soap bubble” appearance.

MEHLMANMEDICAL.COM 63
MEHLMANMEDICAL.COM

- Underrated diagnosis for USMLE. Asked on 2CK exam.


- Benign bone tumor that looks similar to osteoclastoma but age of onset usually birth
to age 20, rather than 20-40.

Unicameral bone
cyst

- Unicameral means “one chamber”; it is fluid-filled.

- USMLE loves cancer mets to the vertebrae, particularly breast, prostate, and lung
mets.
- The exam will not show images of spinal cancer mets, but they will give vignette of
Metastases either lytic lesions of vertebrae in patient with background of cancer, or will give
neurologic syndrome (i.e., of cauda equina).
- Diffuse bone pain in patient with background of cancer = mets.

MEHLMANMEDICAL.COM 64
MEHLMANMEDICAL.COM

- Above image shows Technetium-99 bone scan of cancer mets. Similar imaging is on
2CK IM forms for prostate mets.

- 1-year-old boy + leukocoria (white pupil) + Q wants to know what he is most likely to develop later in

life à answer = osteosarcoma; diagnosis is congenital retinoblastoma; Rb mutations cause both

retinoblastoma as well as osteosarcoma.

- 20M + pain in left distal femur + x-ray shows elevation of periosteum and spiculated new bone

formation; Q wants diagnosis à answer = osteosarcoma; elevation of periosteum = Codman triangle;

spiculated new bone formation = sunburst appearance.

- 61M + pain in left hip + histo image below; Q just asks for diagnosis straight-up:

MEHLMANMEDICAL.COM 65
MEHLMANMEDICAL.COM

o Answer = chondrosarcoma; easy if you know this HY image.

- 54M + pain in right hip + x-ray shows tumor + biopsy shows glistening / shiny lesion; Q wants to know

what is most likely to be characteristic of this lesion à answer = “chondrocytes embedded in

lacunae”; diagnosis is chondrosacoma.

- 25F + pain in proximal tibia + x-ray shows lesion + pain is relieved with ibuprofen; Q wants diagnosis

à answer = osteoid osteoma; this is the answer on USMLE if they give a bone tumor where the pain

is relieved with NSAIDs.

- 18F + 1-cm hard mass growing from skull + father died of colon cancer at age 32; Q wants diagnosis à

answer = Gardner syndrome presenting with osteoma. Gardner syndrome = familial adenomatous

polyposis + either osteoma or lipoma.

- 12M + fever 101 F + pain in left forearm + bone scan shows uptake in the diaphysis; Q asks diagnosis

à answer = Ewing sarcoma; can present similarly to osteomyelitis, except bone scan is more likely to

show uptake in the diaphysis; in osteomyelitis, the uptake is in the metaphysis.

- 12M + fever 101 F + pain in left forearm + biopy with histo image of lesion shown below; Q asks for

the genetics:

o Answer = t(11;22) translocation; diagnosis is Ewing sarcoma; image shows the small blue

cells of neuroendocrine origin.

- 17M + pain in upper left humerus + x-ray shown below; Q wants diagnosis:

MEHLMANMEDICAL.COM 66
MEHLMANMEDICAL.COM

o Answer = unicameral bone cyst. This is an underrated diagnosis for USMLE but shows up on

the real deal.

- 37M + pain in right elbow + x-ray is shown below; Q asks diagnosis (answers are all bone tumors):

o Answer = giant cell tumor of bone (osteoclastoma); x-ray shows soap-bubble appearance. In

contrast to unicameral bone cyst, which occurs usually from birth to age 20, osteoclastoma

usually occurs age 20-40.

- 55M + diffuse bone pain + high serum calcium; bone scan is shown below; Q wants diagnosis:

MEHLMANMEDICAL.COM 67
MEHLMANMEDICAL.COM

o Answer = metastatic malignancy (most likely prostate cancer if no past medical history);

wrong answer is primary hyperparathyroidism; metastases are important cause of high Ca2+.

- 32M + large, yellow, soft infiltrating mass in gluteus maximus + biopsy shows irregular vacuolated

cells + clear cells with many mitoses; Q asks diagnosis à answer = liposarcoma (offline NBME 20);

lipoma is wrong answer (presumably would sound less sinister / not invasive, clearly); lipoma = benign

tumor of fat (Gardner syndrome); liposarcoma = malignant tumor of fat; rhabdomyoma = benign

tumor of striated (skeletal or cardiac) muscle (usually tuberous sclerosis with cardiac rhabdomyoma);

rhabdomyosarcoma = malignant tumor of striated muscle (usually sarcoma botryoides in pediatrics).

- “What do I need to know about fracture types for USMLE?” à I’ll give summary here:

HY bone fracture points for USMLE


- Pathognomonic for child abuse.
Spiral fracture - Caused by rotational/twisting force applied to a limb.
- USMLE doesn’t expect you to diagnose based on imaging.
- Aka closed fracture – i.e., the skin is not broken and the underlying bone does not
Simple fracture
pierce the skin.
- Aka open fracture – i.e., the skin is broken and the underlying bone pierces the
Compound fracture
skin.
Comminuted fracture - Fracture where the bone is broken in at least two pieces.
Greenstick fracture - Bone bends and cracks instead of breaking completely into two pieces.
- Most common type of skull fracture, where there is a break in the skull but the
bone has not moved.
Linear skull fracture
- USMLE wants you to know this is classically associated with epidural hematoma
(asked on 2CK CMS form).
- Presents with tetrad of Battle sign (bruising over mastoid process), raccoon eyes
Base of skull fracture
(bruising around the eyes), rhinorrhea, and otorrhea.
- The answer on USMLE for pain in the metatarsal area of the foot in long-distance
Metatarsal stress runners with low BMI.
fracture - Rather than asking diagnosis directly, USMLE will often give a vignette where a
female long-distance runner with low BMI already has a metatarsal stress fracture,

MEHLMANMEDICAL.COM 68
MEHLMANMEDICAL.COM

and then they’ll ask what she’s at greatest risk of developing à answer =
osteoporosis.
- Occurs with fall on outstretched hand (FOSH), or occasionally as a result of
handlebar injury / impaction, with force transferred up to clavicle.
Clavicular fracture
- Most common site of break is the middle-third of the clavicle.
- Tx = Figure-of-8 sling.
- As discussed earlier, will present as pain over anatomic snuffbox in patient with
FOSH.
Scaphoid fracture
- X-ray will usually be negative acutely. Must do thumb-spica cast to prevent
avascular necrosis of scaphoid, followed by repeat x-ray in 2-3 weeks.
- The answer on USMLE if FOSH with pain in central palm + no pain over anatomic
Lunate fracture
snuffbox.
Hook of hamate - Cause of distal ulnar nerve injury / Guyon canal syndrome.
fracture - Often from handlebar injury / impaction.
Surgical neck of - Causes axillary nerve injury à loss of deltoid function + sensation over deltoid.
humerus fracture
Midshaft fracture of - Causes radial nerve injury à wrist-drop + pronated arm.
humerus
- Aka “distal shaft fracture.”
Supracondylar
- Causes median nerve injury à motor/sensory dysfunction of forearm muscles,
fracture of humerus
first three fingers and thenar region.
- Synonymous with osteoporosis on USMLE (i.e., post-menopausal, corticosteroid-
Vertebral
use, Cushing syndrome).
compression fracture
- Will often give point tenderness over a vertebra.
- Band of low-density bone that looks like fracture on x-ray but not actual fracture.
- Synonymous with vitamin D deficiency (osteomalacia/rickets) on USMLE.
Pseudofracture
- Can be seen in renal failure, since 1,25-D3 is low. Osteomalacia due to renal
failure is called renal osteodystrophy.
- USMLE wants you to know this can cause entrapment of inferior rectus and
inferior oblique muscles.
Orbital floor fracture - Vignette will say guy got hit in eye by baseball + has impaired upward gaze.
- I talk about extraocular muscles and lesions in my neuroanatomy document, but
this is one notable point you should be aware of here.

- 24M + car accident + linear skull fracture; Q asks what type of bleed is most likely in this patient? à

answer = epidural hematoma.

- 4M + x-ray shows spiral fracture of left femur; Q asks next best step in management à answer =

contact child protective services; spiral fractures are pathognomonic for child abuse.

- 20M + car accident + high blood pressure + bradycardia + bruising around the eyes and behind the

ears + low GCS score; Q asks for most likely reason for the patient’s high blood pressure à answer =

increased intracranial pressure; Cushing reflex = triad of HTN + bradycardia + bradypnea due to

increased ICP; patient here has base of skull fracture (Battle sign + raccoon eyes); Q need not mention

all four findings (i.e., rhinorrhea + otorrhea as well).

- 36M + avid cyclist + pain in clavicle + x-ray shows fracture of middle-third of clavicle; Q asks next best

step in management à answer = figure-of-8 sling.

MEHLMANMEDICAL.COM 69
MEHLMANMEDICAL.COM

- 37M + cyclist + mild weakness of finger abduction/adduction + has positive Froment sign; Q asks

which structure is most likely compressed à answer = “deep branch of ulnar nerve at the hook of

hamate.”

- “What do I need to know about myopathy / muscular dystrophies for USMLE?”

HY myopathies / muscular dystrophies for USMLE


- XR disorder caused by mutation in dystrophin (DMD) gene.
- Mutation results in disruption of a-/b-dystroglycan, which is required for
proper internal cytoskeletal anchoring of the muscle cell to the extracellular
matrix.
- Presents with pseudohypertrophy, where muscles appear large but are
Duchenne / Becker replaced with fibroadipose tissue (connective tissue stromal cells).
muscular dystrophy - Duchenne presents in a young boy who implements Gower maneuver to stand
up (uses arms to walk up off the floor because leg muscles are weak).
- Becker presents in adolescence or young adulthood (less severe form of
Duchenne).
- Duchenne is classically frameshift mutation; Becker is classically not
frameshift.
- Usually patient over 50 with proximal muscle pain and stiffness.
- No weakness on physical exam + creatine kinase (CK) levels are normal. If one
or both of these findings is present, the answer is polymyositis, not PMR.
- Can present with high ESR and low-grade fever (any autoimmune disease flare
can present with low-grade fever).
- PMR can present with or without temporal (giant cell) arteritis. Temporal
Polymyalgia rheumatica
arteritis can present bilaterally on NBME exams; do steroids first to prevent
(PMR)
blindness, followed by biopsy second.
- Temporal arteritis can cause jaw claudication (pain in the jaw during episodes).
In contrast to temporomandibular joint dysfunction (a separate diagnosis), jaw
claudication will not be precipitated by eating.
- No specific diagnostic test; diagnosis is made clinically.
- Tx = steroids. NSAIDs are wrong answer and are not proven to be effective.
- Usually patient over 50 with proximal muscle pain and stiffness. These findings
are not unique to PMR. The USMLE will happily give pain and stiffness in
polymyositis vignettes.
- Key distinction between polymyositis and PMR is that polymyositis will present
with 1) muscle weakness on physical exam, and/or 2) increased serum CK.
- The muscle weakness *must be on physical exam.* If the patient complains of
“weakness” but there is no physical exam findings mentioned in vignette or
physical exam shows 5/5 strength, there’s no weakness. Patients will sometimes
mention “weakness,” even though they really just have pain and/or stiffness.
- If polymyositis presents with skin findings, it is called dermatomyositis – i.e.,
Polymyositis /
Gottron papules (violaceous papules on the knuckles), mechanics’ hands (rough-
Dermatomyositis
surfaced hands), shawl rash (body rash), heliotrope rash (violaceous eyelids /
periorbital rash; don’t confuse with malar rash of SLE).
- Patients often positive for anti-Jo1 antibodies.
- USMLE wants “electromyography and nerve conduction studies” as first step
in management for polymyositis/dermatomyositis. This is what they ask on 2CK
NBME forms. I have not seen them ask anti-Jo1 antibodies vs EMG+NCS as two
separate answer choices. Usually anti-Jo1 antibodies are mentioned in the
vignette rather than as the test you need to order.
- Muscle biopsy is confirmatory, showing CD8+ T cell infiltration. The histo can
be described as “CD8 + T cells and macrophages surrounding muscle fibers.”

MEHLMANMEDICAL.COM 70
MEHLMANMEDICAL.COM

- For whatever reason, dermatomyositis can be a paraneoplastic syndrome of


ovarian cancer (shows up on Step 1 NBME).
Tx = steroids.
- This is a psych condition, not an actual muscle disorder, but is often confused
with polymyositis and PMR.
- Labs will be normal. ESR will not be elevated. Patient will not have fever.
- Will be described as woman 20s-50s with multiple (and often symmetric)
Fibromyalgia
muscle tenderness points.
- Treatment is SSRIs. USMLE can write this as “anti-depressant therapy.” This
confuses students (“But she doesn’t have depression though.”) à Right. But
SSRIs are still anti-depressant medication.
- The answer on USMLE if they give jaw pain that is precipitated by eating.
Temporomandibular
- Often confused with jaw claudication seen in temporal arteritis. In the latter,
joint dysfunction
however, the pain is not precipitated by eating.
- Autosomal dominant, CTG trinucleotide repeat expansion disease.
- Myotonia is inability to relax muscles.
Myotonic dystrophy - The answer on USMLE if they say patient cannot relax grip on doorknob /
handshake, or cannot let go of golf club.
- Sometimes associated with early / frontal balding.
- Myopathy can occur in both hypo- and hyperthyroidism, yes, but this is
especially HY for hypothyroidism on USMLE.
Hypothyroid myopathy
- They will often sneak this in as proximal muscle weakness or increased CK in
patient who has ongoing fatigue, dysthymia, menstrual irregularities, etc.
- Classically seen when statins and fibrates are combined, but both drugs can
cause myopathy independently.
- Mild CK elevations are normal and expected in patients when commencing
Drug-induced myopathy
these agents. Dose does not need to be decreased for mild CK elevations.
- USMLE wants “P450-mediated interaction” as the cause of the myopathy
when statins and fibrates are combined.
- Broad term that can refer to numerous mitochondrial diseases.
- USMLE wants you to know the patient has a mitochondrial disorder when
he/she presents with hypotonia, ear/eye problems, and lactic acidosis. You
want to memorize this tetrad as synonymous with mitochondrial disorders.
Mitochondrial myopathy - “Ragged red fibers” can be a buzzy descriptor in mitochondrial myopathy Qs.
- Mitochondrial disorders are maternally inherited only.
- Heteroplasmy refers to offspring having varying disease severity based on
variation in allocation of diseased mitochondrial genes (I talk more about this
stuff in my HY biochemistry PDF).
- The answer on USMLE if they tell you patient 50 or older has months to years
Inclusion body myositis of progressive muscle weakness + biopsy of muscle shows basophilic rimmed
vacuoles.
- Answer on USMLE if they give you a patient who has anti-U1-
Mixed connective tissue ribonucleoprotein (U1-RNP) antibodies.
disease - Patient presents as having combined features/symptoms from three different
disorders à LPS à Lupus, Polymyositis, Scleroderma.

- 52F + 6-month history of pain and stiffness of shoulders + elevated ESR; Q wants diagnosis à answer

= polymyalgia rheumatica (PMR); presents as pain and stiffness of proximal muscles, usually in patient

over 50; Q will not mention anything about elevated CK or weakness on physical exam.

- 52F + 6-month history of pain and stiffness of shoulders + elevated ESR and CK; Q wants diagnosis à

answer = polymyositis, not PMR, because Q mentions CK elevation. Pain and stiffness are not unique

MEHLMANMEDICAL.COM 71
MEHLMANMEDICAL.COM

to PMR. They can absolutely be seen in polymyositis. The notion that they are unique to PMR is

complete nonsense for USMLE purposes.

- 52F + 6-month history of pain and stiffness of shoulders + elevated ESR + normal CK + physical exam

shows 3/5 strength in shoulders; Q wants diagnosis à answer = polymyositis; even though CK is

normal, patient has weakness on physical exam.

- 60M + 6-month history of weakness, pain, and stiffness of shoulders + elevated ESR + no other info

given; Q wants most likely diagnosis à answer = PMR, not polymyositis, because not only is CK not

mentioned/elevated, but weakness is not on physical exam; patients will often report “weakness,”

even when they don’t have true weakness and instead just have pain/stiffness; there is a 2CK CMS Q

that is similar to this scenario.

- 64F + 6-month history of pain and stiffness of shoulders + elevated ESR + weakness on physical exam;

Q wants next best step in diagnosis? à answer = “electromyography and nerve conduction studies”;

diagnosis is polymyositis; Q will not force you to choose between anti-Jo1 antibodies and EMG/NCS,

but the latter is correct on NBME forms; biopsy is confirmatory.

- 64F + 6-month history of pain and stiffness of shoulders + physical exam shows weakness + anti-Jo1

antibodies are positive; Q asks the most likely pathophysiology for this patient’s condition? à answer

= “CD8 + T cells and macrophages surrounding muscle fibers.”

- 64F + bilateral temporal headache + slightly blurry vision on left + high ESR + bilateral hip pain; Q

wants next best step à answer = “IV methylprednisolone”; temporal (giant cell) arteritis can show up

bilaterally on NBME forms; do steroids first to prevent blindness, then do biopsy of temporal artery;

the patient’s hip pain is polymyalgia rheumatica, which is linked with temporal arteritis.

- 64F + right-sided temporal pain + high ESR + low-grade fever + pain in the jaw + low hemoglobin; Q

wants diagnosis à answer = giant cell arteritis; can present with jaw claudication; low hemoglobin is

due to anemia of chronic disease.

- 64F + right-sided temporal and facial pain + worsens with eating + proximal muscles a little stiff; Q

wants diagnosis à answer = temporomandibular joint dysfunction, not temporal arteritis; if it

worsens with eating, then they want TMJ dysfunction, not jaw claudication.

- 45F + 6-month history of muscle pain; physical exam shows 12 symmetric tender points; ESR not

elevated; Q wants treatment à answer = “anti-depressant therapy”; diagnosis is fibromyalgia; this is

MEHLMANMEDICAL.COM 72
MEHLMANMEDICAL.COM

a psych condition treated with SSRIs; as mentioned earlier, students get confused here because the

patient is not depressed, but SSRIs are still anti-depressant medication.

- 61F + 20-pound weight loss past one month despite no change in diet + fatigue + anemia + proximal

muscle weakness + increased serum CK + pulling sensation in groin + Q shows image below; Q wants

to know diagnosis (all cancers listed):

o Answer = ovarian adenocarcinoma; for whatever magical reason, dermatomyositis can be a

paraneoplastic syndrome of ovarian cancer. à Google/literature says around 10-20% of

women with dermatomyositis have ovarian cancer.

- 18M + frontal balding + difficulty letting go of handshake; Q wants diagnosis; answer = myotonic

dystrophy; autosomal dominant; CTG trinucleotide repeat expansion.

- 44F + 6-month history of low energy and fatigue + gradually lengthening periods + HR 60 + hepatic

AST, total cholesterol, and serum creatine kinase are elevated; Q asks most likely diagnosis à answer

= hypothyroidism (Hashimoto); elevated CK is due to hypothyroid myopathy, which may or may not

present with proximal muscle weakness in the vignette; transaminases can be elevated in thyroid

dysfunction; USMLE likes menstrual irregularities, bradycardia, high cholesterol, dysthymia, doughy

skin, and carpal tunnel syndrome in hypothyroidism. Details such as cold intolerance, weight gain,

brittle hair, and constipation are often omitted because they’re too easy/buzzy.

- 66M + recently started taking atorvastatin + mild increase in serum creatine kinase; Q wants to know

next best step in management à answer = no change in dose of statin; Dx is statin-induced

myopathy; USMLE wants you to know that mild elevation in CK is normal and expected in patients

MEHLMANMEDICAL.COM 73
MEHLMANMEDICAL.COM

who commence statins (and fibrates). The literature says the drug does not need to be discontinued

unless CK elevations >10x the upper limit of normal are observed.

- 66M + started taking a statin and fibrate simultaneously + develops myopathy; Q wants mechanism

à answer = “P-450-mediated interaction.”

- 3M + hypotonia + weakness + lactic acidosis + poor hearing and vision; 7-year-old sister has normal

vision and hearing, only mild weakness, no hypotonia, and only mildly elevated serum lactic acid; Q

wants reason for discrepancy; answer = heteroplasmy; wrong answer = X-linked recessive disorder;

the tetrad of hypotonia, lactic acidosis, and eye/ear problems is HY for USMLE.

- 3M + hypotonia + lactic acidosis + no mention of eye/ear problems + biopsy of muscle shows ragged

red fibers; Q wants most likely inheritance à answer = mitochondrial.

- 3M + large calves + uses arms to walk up off the floor; Q wants inheritance pattern; answer = X-linked

recessive; Dx is Duchenne muscular dystrophy.

- 3M + large calves + uses arms to walk up off the floor; Q wants organelle / cellular structure that is

disrupted in patient’s condition à answer = cytoskeleton; a-/b-dystroglycan are encoded by the

DMD (dystrophin) gene and are required for proper anchoring of the muscle cell cytoskeleton to the

extracellular matrix.

- 3M + large calves + uses arms to walk up off the floor + Q wants to know what will be seen on muscle

biopsy à answer = “connective tissue stromal cells” or “fibroadipose tissue”; patient will have

pseudohypertrophy, which is replacement of muscle cells with connective tissue.

- 3M + large calves + uses arms to walk up off the floor; Q wants molecular mechanism for patient’s

condition à answer = frameshift mutation; this results in truncated, non-functional protein due to

early formation of stop codon.

- 18M + increasing muscle weakness and muscular enlargement + maternal uncle died of

cardiomyopathy in his 20s; Q wants diagnosis à answer = Becker muscular dystrophy; less severe

form of Duchenne; usually due to non-frameshift mutations in the DMD gene. Becker usually presents

in adolescence or young adulthood; Duchenne will be a kid.

- 4M + short arms and legs; trunk and head are normal size; parents are normal; Q wants mechanism

for patient’s condition à answer = gonadal (germline) mosaicism; diagnosis is achondroplasia;

disease is constitutive activation of FGFR3 gene; results in failed cartilage conversion into bone;

MEHLMANMEDICAL.COM 74
MEHLMANMEDICAL.COM

presents as shortened limbs but normal head size and postural height (trunk size); condition is

autosomal dominant, but both parents will usually be of normal height, which means the original

mutation occurred in a gonadal cell (usually a spermatogonia of the father), where the child has 100%

of cells affected as a result, despite the father being unaffected. I also talk about this stuff in more

detail in my HY Biochem PDF.

- “How HY is hernia stuff for USMLE?” à Unfortunately decently HY. But good news: I’ll tell you exactly

what you need to know for them for USMLE without all of the absolute nonsense.

HY MSK hernia points for USMLE


- Small bowel herniates medial to inferior epigastric vessels.
- Occurs usually in older men.
- Occurs due to weakness of abdominal wall musculature / transversalis fascia.

Direct inguinal
hernia

- The coverings of direct inguinal hernia = skin, superficial fascia, external spermatic
fascia, cremasteric fascia, extraperitoneal tissue.
- Hernia passes through Hesselbach triangle (inguinal triangle), which is an area of
weakness in abdominal wall through which a direct inguinal hernia can protrude.
- Boundaries of Hesselbach triangle: medial border = the lateral border of rectus
abdominus muscle, aka linea semilunaris; lateral border = inferior epigastric vessels;
inferior border = inguinal ligament.

MEHLMANMEDICAL.COM 75
MEHLMANMEDICAL.COM

- NBME wants “medial to inferior epigastric artery; superior to inguinal ligament” as the
answer for direct inguinal hernia.
- Q might say older patient has palpable mass in groin that reduces when he lies down,
or worsens when he coughs.
Tx = “operative management” or “elective hernia repair,” since it is not an overt
emergency but closure should be performed prior to any type of incarceration and
strangulation (ischemia leading to pain, fever, and necrosis).
- Small bowel herniates lateral to inferior epigastric vessels, through deep inguinal ring.
- Seen classically in male infants, but can occur any age.
- Mechanism is patent processus vaginalis. This is also the mechanism for hydrocele
(asked on NBME).
- NBME wants “lateral to inferior epigastric artery; superior to inguinal ligament” as the
answer for indirect inguinal hernia.

Indirect inguinal
hernia

- Since it passes through deep inguinal ring, it can be reduced with pressure / a finger
placed over deep inguinal ring. Direct inguinal hernia, since it does not pass through
deep inguinal ring, will not reduce with pressure applied here.

MEHLMANMEDICAL.COM 76
MEHLMANMEDICAL.COM

- Tx = elective hernia repair.


- For hydroceles, observation is the answer under the age of 1 (HY on Peds forms).
- Small bowel protrudes through femoral canal, inferior to inguinal ligament.
- Account for 5% of hernias and more common in women (3:1) because of the wider
anatomy of the pelvis in women.

Femoral hernia

- Boundaries of femoral canal (and hernia) are: lateral border = femoral vein; medial
border = lacunar ligament; posterior border = pectineal ligament; anterosuperior border
= inguinal ligament.
- Tx = surgery.
- Rare abdominal hernia (<2%) that occurs lateral to the linea semilunaris of the rectus
abdominis, but Step 1 NBME asks it. You should also know this for 2CK Surg.
Spigelian hernia
- Usually in older adults.

MEHLMANMEDICAL.COM 77
MEHLMANMEDICAL.COM

- Highest yield point is that the medial border of the hernia is the rectus abdominis.
- Small bowel herniation through the umbilicus.
- Can occur at any age, but I’ve seen this show up on NBME in congenital
hypothyroidism (i.e., cretinism).
- Not to be confused with omphalocele in neonates. An umbilical hernia is completely
covered by skin. In contrast, an omphalocele is a herniation merely covered by a thin,
translucent layer of peritoneal membrane. It can be idiopathic, but is often associated
with the Trisomy 13 and 18, and Beckwith-Wiedemann syndrome.

Umbilical hernia

- Small bowel herniation through the linea alba (tendinous, fibrous raphe that runs
vertically down the abdomen).
- Not to be confused with gastroschisis in neonates. A paraumblical hernia is completely
Paraumbilical
covered by skin and protrudes usually superior to the umbilicus. Gastroschisis is not
hernia
covered by anything (not even a layer of peritoneal membrane as with omphalocele) and
protrudes to the right of the umbilucus; gastroschisis is seen sometimes in Trisomy 13
and 18.
- Small bowel protrusion in epigastrium through the linea alba, similar to paraumbilical
Epigastric hernia.
hernia - The difference is that epigastric hernia is further up in the abdomen in the epigastrium,
whereas paraumbilical hernia is literally adjacent the umbilicus.
Incisional hernia - Can occur at the site of any abdominal incision. Just know it’s possible.

MEHLMANMEDICAL.COM 78
MEHLMANMEDICAL.COM

- Rare hernia of the pelvic floor. Small bowel contents protrude through obturator
foramen.

Obturator
hernia

- Despite being rare, obturator hernia is HY for 2CK Surg and shows up on 2CK exam.
- You need to know that the Howship-Romberg sign is used to diagnosis obturator
hernia. In this test, thigh extension, medial rotation, and abduction cause lancinating
pain in the medial thigh / obturator distribution due to compression of obturator nerve.
- Rare hernia of the pelvis with protrusion of small bowel through either the greater or
lesser sciatic foramen.
Sciatic hernia - Can mimic sciatica due to compression of sciatic nerve.
- Just know this type of hernia exists. As I mentioned above, obturator hernia is HY for
2CK. They will usually list sciatic hernia as incorrect answer choice alongside it.
Lumbar hernia - Rare hernia that presents with pain and herniation in the lower back.
Perineal hernia - Rare hernia of pelvic floor. Can occur due to atrophy of the levator ani muscle.

- 1-year-old boy + enlarged testis + scrotal mass that is reducible with pressure over deep inguinal ring;

Q asks mechanism for patient’s condition à answer = patent processus vaginalis; diagnosis is indirect

inguinal hernia; mechanism is same as formation of hydrocele; since the hernia passes through deep

inguinal ring (which is lateral to inferior epigastric artery), pressure applied to the ring can reduce the

hernia. With direct inguinal hernia, in contrast, since it does not pass through the deep inguinal ring,

this examination maneuver wil not reduce the hernia.

- 12-year-old boy + testicular mass that enlarges with cough and reduces when lying down; pressure

applied over deep inguinal ring hides hernia; Q wants to know whether the hernia is lateral vs medial

to inferior epigastric artery, and whether it’s inferior or superior to inguinal ligament à answer =

lateral to inferior epigastric artery + superior to inguinal ligament; diagnosis is indirect inguinal hernia.

MEHLMANMEDICAL.COM 79
MEHLMANMEDICAL.COM

- 64M + bulging mass in groin + not painful; worsens with cough and improves when supine; Q wants

location of hernia relative to inferior epigastric artery à answer = medial to inferior epigastric artery;

diagnosis is direct inguinal hernia; usually occurs in older men.

- 40F + pain and palpable mass in groin inferior to inguinal ligament; Q wants to know which structure

is lateral to the mass à answer = femoral vein; diagnosis is femoral hernia.

- 60M + Q shows CT scan of an abnormality with arrow pointing to it; asks for what the medial border

of the hernia is (in other words, what is medial to the arrow)?

o Answer = lateral border of rectus abdominus; diagnosis is Spigelian hernia; hernia protrudes

just lateral to the linea semilunaris.

o Students get hysterical and worried when they see CT scans like this. Recognize that the

USMLE doesn’t care that you even know what a Spigelian hernia is in order to get this

question right. If you chill out for a second and try to imagine what you’re looking at in the

CT, you can tell the rectus abdominis is at the superior part of the image, which means the

hernia is just lateral to it.

- 55M + pelvic hernia + physical exam causes worsening of pain when ipsilateral leg is extended,

medially rotated, and abducted; Q wants diagnosis à answer = obturator hernia; the Howship-

Romberg sign is used to diagnose. The USMLE can be vague with this question, where they just tell

MEHLMANMEDICAL.COM 80
MEHLMANMEDICAL.COM

you “patient has hernia that’s worsened with this exam maneuver” and you have to know it’s

obturator hernia.

- 8-month-old boy + large tongue + hypotonia + umbilical hernia; Q asks what would have most likely

prevented this patient’s condition à answer = routine newborn screening; diagnosis is congenital

hypothyroidism (cretinism); heel-prick test at birth tests for hypothyroidism, PKU, and galactose

disorders, among others.

- Neonatal male + enlarged occiput + clenched fingers + rocker-bottom feet + protrusion of abdominal

contents through umbilicus that are covered in thin membrane; Q wants diagnosis à answer =

trisomy 18 (Edward syndrome); omphalocele is described; in contrast, gastroschisis protrudes to the

right of the umbilicus and is not covered in a layer of peritoneal membrane.

- Identify the hernias:

- “Is there any chest wall MSK I should know?”

o For a thoracentesis (removing fluid from the pleural space when treating a pleural effusion)

or needle decompression (for pneumothorax), the needle should be inserted just above the

rib. This is in order to avoid injury to the neurovascular bundle that sits at the inferior margin

of each rib.

MEHLMANMEDICAL.COM 81
MEHLMANMEDICAL.COM

o For an intercostal nerve block, however, we want to anesthetize the nerve, so the needle

should be inserted just below each rib (i.e., at the inferior margin).

o 40M + car accident + severe, left-sided chest pain; Q asks, the most likely pathway of this

patient’s chest pain is carried by which nerve à answer = intercostal.

o The numerical rib location for needle insertion for a thoracentesis varies depending on the

source (i.e., literature says 8-10th ribs), but offline NBME 21 has Q where answer is

midaxillary line above the 9th rib.

o When performing a needle decompression followed by chest tube for pneumothorax, the

typical location for insertion is the second intercostal space (just superior to the third rib).

- “Is there any high-yield neck anatomy I should know?”

HY neck MSK masses for USMLE


- The answer on USMLE if they say there’s a painless, midline neck lump in a child
that moves upward with swallowing or protrusion of the tongue. This buzzy
description is seen for maybe only about half of Qs.
Thyroglossal duct cyst
- Can also be described as painless mass inferior to the hyoid bone that
demonstrates uptake with a Technetium-99 scan.
- USMLE wants “endoderm of foramen cecum” as the embryology.
Sternocleidomastoid - The answer on USMLE if they say nodular mass in the lateral neck in an infant
injury who had been born via forceps deliver (risk factor for damage to the muscle).
- The answer on USMLE if they give idiopathic lateral neck mass in infant that may
Branchial cleft cyst
or may not have an opening to the skin.

- “Do I need to know nasal sinus/concha anatomy for USMLE? à There are rare Qs that show up asking

sinus anatomy, but now that Step 1 is P/F, I’d say studying this is largely venturing into left-field

territory. There is a question on one of the offline Step 1 NBMEs, however, where they say there is a

MEHLMANMEDICAL.COM 82
MEHLMANMEDICAL.COM

sinus infection with purulent discharge coming from the sphenoethmoid recess, and they ask for the

location of this structure à answer = “superior to the superior concha.” You can memorize this one

point, but as I said, I don’t think you need to be sitting in your room obsessing over nasal anatomy.

- “Are there any random HY points for pediatrics I should know?”

o USMLE likes bone age on 2CK Peds forms. If an x-ray is done of the hand, the approximate

age of the child can be ascertained; if the child is short and “bone age equals chronologic

age,” that is genuine short stature (e.g., familial short stature; I’ve also seen this in Turner

syndrome Qs); if the child is short and “bone age is less than chronologic age,” then the

answer is “constitutional short stature,” or “constitutional growth delay,” which means the

child’s growth curve is merely shifted to the right but he/she will catch up (i.e., boy is 4’11” in

9th grade but will eventually become average height); the Q will often say the parents are

average height; if they don’t mention bone age for constitutional short stature Qs, they can

say the child is Tanner stage 1 or 2 (prepubescent).

o If child abuse is suspected, sometimes the answer can be “obtain skeletal survey” in order to

evaluate for radiographic evidence of fractures at different stages of healing.

- “Is there any HY Pharm I should know in relation to MSK?”

Highest yield “MSK pharm” for USMLE


- Agonizes GABAB.
Baclofen - Used for spasticity, classically in multiple sclerosis, but I’ve seen one NBME Q
where it’s used for random spasticity in an older dude.

MEHLMANMEDICAL.COM 83
MEHLMANMEDICAL.COM

- Students frequently remember GABAB for this drug, but often say “antagonist”
when I probe them further. So remember: it’s an agonist, not an antagonist, at
GABAB.
- Muscarinic (cholinergic) receptor antagonist.
- Used to treat acute dystonia due to anti-psychotics.
Benztropine
- If patient starts anti-psychotic and then gets stiff neck, oculogyric crisis (abnormal
eye movements), or muscle rigidity without fever, the answer = benztropine.
- First-generation histamine-1 (H1) antagonists.
- Diphenhydramine is quite possibly the highest-yield drug on USMLE.
- Used to treat acute dystonia, similar to benztropine, as well as motion sickness.
- H1 blockers can treat allergies in theory, but they have nasty anti-cholinergic
(anti-muscarinic) side-effects.
- The anti-cholinergic side-effects are interestingly a good thing, however, when we
want to treat acute dystonia. Psych Qs will either list benztropine or
diphenhydramine (or chlorpheniramine) as the answer, but not both at the same
time.
Diphenhydramine / - For whatever reason, anti-cholinergic effects treat motion sickness. Scopolamine
Chlorpheniramine is an anti-cholinergic used to treat motion sickness classically. But I’ve seen NBME
ask diphenhydramine straight-up for this as well – i.e., the nasty anti-cholinergic
side-effects are, once again, a good thing if the aim is Tx of motion sickness.
- 1st-gen H1 blockers can cause cognitive dysfunction (delirium, as well as
worsening of dementia) and drowsiness. Therefore avoid in elderly and
locomotive/machine operators if at all possible.
- 1st-gen H1 blockers can also cause anti-a1-adrenergic effects (orthostatic
hypotension).
- I talk about all of the pharm-related stuff in a lot more detail in my free pharm
modules on the website.
- Blocks ryanodine Ca2+ channel.
- Tx for neuroleptic malignant syndrome (NMS) and malignant hyperthermia (MH).
- If patient gets muscle rigidity and fever following commencement of anti-
psychotic, or following administration of succinylcholine during surgery, answer =
dantrolene. (Bromocriptine for NMS is low-yield and rarely seen on NBME).
- NBME will sometimes give vignette of NMS or MH, followed by fever and rigidity,
Dantrolene
and then the answer for Tx is “decreases sarcoplasmic calcium release.”
- In NMS and MH, the ryanodine channel, which allows calcium to move from the
sarcoplasmic reticulum into the cytosol, gets stuck open, so high amounts of
calcium moves into the cytoplasm. The cell then needs to use a lot of ATP to pump
the calcium back into the sarcoplasmic reticulum. This generates heat à fever.
Dantrolene closes this channel.
- Nicotinic receptor antagonist / depolarizing neuromuscular junction blocker.
- Used to paralyze muscles during general anesthesia.
- Q on offline Step 1 NBME says patient has “prolonged apnea following
Succinylcholine anesthesia” and asks which drug caused it à answer = succinylcholine.
- The depolarizing aspect means that it can cause transient twitching / increased
neurotransmission prior to the antagonistic effects ensue. USMLE might rarely give
you a graph-type Q where you have to infer this effect refers to succinylcholine.
- Nicotinic receptor antagonist / non-depolarizing neuromuscular junction blocker.
Vecuronium /
- Used to paralyze muscles during general anesthesia.
Rocuronium
- NBME will give vignette saying MOA and then just ask for drug name straight-up.
- Bisphosphonate; inhibits osteoclasts. This MOA is HY.
- Used for osteoporosis after Ca2+/VitD.
- I’ve seen pamidronate (not alendronate) show up on 2CK forms for Tx of
Alendronate
hypercalcemia (after normal saline is given).
- Students get fanatical about bisphosphonates causing osteonecrosis of the jaw.
The yieldness of this adverse effect is basically non-existent on NBME exams.

MEHLMANMEDICAL.COM 84
MEHLMANMEDICAL.COM

USMLE wants you to know bisphosphonates cause pill-induced esophagitis. This is


very HY for 2CK FM forms in particular (K+ supplements also cause esophagitis).
- N-terminus PTH analogue that can induce bone formation. Even though PTH
causes bone resorption, this agent stimulates osteoblast-mediated bone formation
Teriparatide
more than it induces RANK-L-mediated activation of osteoclasts.
- Can be used for severe/advanced osteoporosis.
- Monoclonal antibody against RANK-L.
Denosumab
- Can be used for severe/advanced osteoporosis.

o 32F + history of multiple sclerosis with muscular spasticity; Q asks the mechanism of action

of the drug used to treat her spasticity à answer = GABAB receptor agonist; drug is baclofen.

o 45M + started on aripiprazole for schizophrenia + develops stiff neck; Q wants mechanism of

action of treatment à answer = anti-muscarinic (anti-cholinergic); diagnosis is acute

dystonia due to anti-psychotic use; treatment for acute dystonia is either benztropine

(muscarinic receptor antagonist), OR diphenhydramine or chlorpheniramine (both first-

generation H1 receptor antagonists); any of these three drugs can show up as the answer on

USMLE. The 1st-gen H1 blockers have nasty anti-cholinergic side-effects, but that’s actually a

good thing when we want to treat acute dystonia.

o 45M + started on antipsychotic + develop muscle rigidity + fever of 103 F; Q wants

mechanism of action of treatment à answer = “inhibition of intracellular calcium release”;

diagnosis is neuroleptic malignant syndrome (NMS); treatment is dantrolene.

o 45M + started on antipsychotic + develop muscle rigidity + temperature is 98.6 F; Q wants

mechanism of action of treatment à answer = anti-muscarinic (anti-cholinergic); drug is

either benztropine (muscarinic receptor antagonist) or a 1st-gen H1 blocker (anti-cholinergic

side-effects that are actually good for Tx); diagnosis is acute dystonia, not neuroleptic

malignant syndrome (NMS); if the Q wants NMS, they’ll give muscle rigidity + fever; in

contrast, muscle rigidity + no fever = acute dystonia.

o 52F + undergoes surgery + develops high fever and muscle rigidity; Q asks what drug most

likely caused this patient’s condition à answer = succinylcholine, a nicotinic receptor

antagonist that enables paralysis during surgery. It is known as a depolarizing neuromuscular

blocker. Diagnosis is malignant hyperthermia, which has same mechanism as NMS, except it

is classically caused by succinylcholine, not antipsychotics. Malignant hyperthermia is treated

with dantrolene, same as with NMS.


MEHLMANMEDICAL.COM 85
MEHLMANMEDICAL.COM

o 52F + undergoes surgery with general anesthesia + receives vecuronium; Q just wants

mechanism of action à answer = nicotinic receptor antagonist; drugs such as vecuronium

and rocuronium are non-depolarizing neuromuscular junction blockers. In contrast,

succinylcholine is a depolarizing blocker.

- “Anything I should know about random infections and muscle?”

o USMLE likes myalgias = influenza virus when patient has head cold. Vignette will be big

paragraph + they mention muscle pain in there + answers are all different viruses; answer is

simply influenza.

o Taenia solium (pork tapeworm; pork cestode) can cause cysts in muscle (myalgia).

o Trichinella spiralis (pork roundworm; pork nematode) can cause triad of fever + periorbital

edema + myalgias in patient who recently ate pork or bear meat.

- “Anything I should know about random immuno and MSK?”

o Muscle pain at the site of injection of a drug or vaccine 3-7 days post-injection = Arthus

reaction = type III hypersensitivity (immune complexes); was seen with Moderna Covid

vaccine (“Moderna arm”).

o Polyarthritis 3-7 days following injection of a drug = serum sickness = type III hypersensitivity

(immune complexes).

o USMLE will give you image of malar rash of lupus (type III hypersensitivity) and then ask

which condition is most similar à answer = Arthus reaction, or serum sickness, or PSGN.

- “Anything random I should know about genetics and MSK?”

o USMLE wants you to know the Hox genes, or homeobox genes, are necessary for proper

body/limb patterning (i.e., body parts developing at correct locations).

o They will say if Hox genes 9-12 are turned on, phalanges are produced; when only 9-11 are

turned on, carpal bones are produced. Q asks, “why did carpal bones develop instead of

phalanges?” à answer = 12 not turned on. Not hard, but I’ve seen this confuse students.

o Newborn dies shortly after birth + autopsy shows transformation of lumbar vertebrae into

thoracic vertebrae; Q wants mechanism à answer = “inappropriate expression of Hox genes

normally transcribed cranially.” Cranially = toward the head; caudally = toward the “tail.”

MEHLMANMEDICAL.COM 86
MEHLMANMEDICAL.COM

YouTube
@mehlmanmedical

Instagram
@mehlman_medical

MEHLMANMEDICAL.COM 87
MEHLMANMEDICAL.COM

MEHLMANMEDICAL
HY ANATOMY/MSK/RHEUM

All material is copyrighted and the property of mehlmanmedical.

Copyright © mehlmanmedical

MEHLMANMEDICAL.COM 88
MEHLMANMEDICAL
HY RENAL
MEHLMANMEDICAL.COM

YouTube
@mehlmanmedical

Instagram
@mehlman_medical

MEHLMANMEDICAL.COM 2
MEHLMANMEDICAL.COM

HY Renal – by Dr Mike Mehlman

Basic HY Renal lab values you must know for USMLE


- Should be <20 mg/dL.
- High urea nitrogen in the blood is called azotemia.
Blood urea nitrogen
- “Uremia” technically means “urea in the blood,” but is used to refer to patients
(BUN)
who have symptomatic renal failure with poor lab values. This has absolutely zero
to do with hyperuricemia (gout) and is sometimes confused by students.
- 0.7-1.2 mg/dL
Creatinine (Cr)
- Once you’ve hit a creatinine of 2, you’ve lost ~90% of your renal function.
- 3.5-5.0 mEq/L.
- Will always be elevated in renal failure on USMLE.
Potassium (K+) - This is because the kidney normally secretes (i.e., excretes through tubular walls
into the urine) K+ in the cortical collecting duct under the action of aldosterone. So
if the kidney is fucked up, we can’t do that, so serum K+ rises.
- 135-145 mEq/L.
- Can be variable in renal failure as per my observation on NBMEs.
- Aldosterone normally reabsorbs sodium in the cortical collecting duct in exchange
Sodium (Na+)
for potassium being secreted, so one might think it should automatically be low if
the kidney can’t reabsorb it. But the kidney has a multitude of sodium regulation
mechanisms (i.e., vasopressin), so Na+ can be normal.
- Always ¯ in renal failure. This is due to 2 reasons:
1) Ca2+ is normally reabsorbed in the late-distal convoluted tubule (DCT),
so if we can’t do that, serum Ca2+ falls.
2) Proximal convoluted tubule (PCT) of the kidney is normally where 1a-
hydroxylase, under the action of PTH, activates inactive 25-OH-D3 into
Calcium (Ca2+) active 1,25(OH)2-D3. The latter then goes to the small bowel, where it ­
absorption of Ca2+. So if we can’t do that, serum Ca2+ falls.
- Low serum Ca2+ means ¯ negative feedback at the Ca2+-sensing receptors at the
parathyroid glands à PTH goes up. We call this secondary hyperparathyroidism,
which means PTH is high due to a cause external to the parathyroid glands
themselves. You need to know secondary hyperthyroidism is due to renal failure.
- Always ­ in renal failure.
- This is for 2 reasons:
1) The kidney cannot filter it out sufficiently. Lots of phosphate is normally
filtered by healthy kidneys.
Phosphate (PO43-) 2) PTH normally acts to ¯ phosphate reabsorption in the PCT via
downregulation of 3 different types of phosphate pumps. Since the
function of PTH is impaired, the kidney reabsorbs too much phosphate, so
serum levels rise.
- Combo of ¯ Ca2+ and ­ PO43- in renal failure = secondary hyperparathyroidism.
- Always ¯ in renal failure.
- This is for 2 reasons:
1) The kidney cannot reabsorb bicarb in the PCT as well, so ­ bicarb in the
urine à ¯ bicarb in serum à metabolic acidosis.
Bicarbonate (HCO3-)
2) The kidney cannot secrete protons (H+) in the cortical collecting duct as
readily à retention of H+ mops up HCO3- à ¯ serum bicarb.
- Renal failure = Uremia = the U in MUDPILES for high anion-gap metabolic
acidoses. (If you need help on the acid-base stuff, do my HY Arrows PDF).

MEHLMANMEDICAL.COM 3
MEHLMANMEDICAL.COM

Afferent vs efferent regulation


- Paramount for understanding some renal path, so I’ll quickly mention it.
- Go to the kidney.
- Patency maintained by vasodilating prostaglandins.
- NSAIDs inhibit prostaglandin synthesis à ¯ dilation of afferent arterioles à ¯ renal
Afferent arterioles
blood flow.
- NSAIDs don’t actively constrict the afferent arterioles. This is incorrect wording.
They just merely ¯ vasodilation / ¯ diameter of the afferent arterioles.
- Leave the kidney.
- Angiotensin II normally constricts these.
- ACEi/ARBs therefore ­ diameter of the efferent arterioles. They are not dilating
them; they are merely ¯ constriction of them. As mentioned above, prostaglandins
will actively dilate the afferent arterioles.
- Some students might say, “Well doesn’t AT II act on the afferent arterioles as well?”
Efferent arterioles In theory, a very minor amount, but on USMLE, this will get you questions wrong. You
want to think:

AT II = constricts efferent;
ACEi/ARBs = ¯ constriction of efferent (­ diameter).
Prostaglandins = dilate afferent;
NSAIDs = ¯ dilation of afferent (¯ diameter).

Pre-, intra-, and post-renal azotemias


- BUN/Cr = Blood urea nitrogen to creatinine ratio.
- FENa = fractional excretion of sodium (i.e., amount of sodium in the urine).
- There’s no “normal” range you should be looking for here for either of these.
- What matters is that when there is an actual kidney problem, we look at the values to help
us figure out what the diagnosis is. But we never actually say, “Oh the BUN/Cr or FENa is out
of the normal range.”
BUN/Cr,
- BUN/Cr >20 and FENa <1% mean pre-renal azotemia.
FENa
- BUN/Cr <20 and FENa >1% just simply mean “not pre-renal.”

- The notion of intra- vs post-renal having slightly different ranges is nonsense for USMLE.
- Also, what I’ve come to learn from NBME Qs is that only ~9/10 times the BUN/Cr will be
what we expect. There are a couple 2CK NBME Qs that give BUN/Cr <20 for pre-renal
azotemia and >20 for acute tubular necrosis. If the NBME ever “breaks the rules” this way,
the vignette will be overwhelmingly obvious what the diagnosis is anyway.
- Pre-renal azotemia means there is renal pathology because of reduced renal blood flow
over the sub-acute to chronic time frame. The kidney will do everything it can to ­ retention
of fluid (because it thinks blood volume is low). The way it accomplishes this is by ­
reabsorption of urea and sodium, since water will follow. This is why blood urea nitrogen is
high (i.e., BUN/Cr >20) and sodium in the urine is low (FENa <1%).
- Notice above I stress sub-acute to chronic time frame (i.e., days-months), since one of the
highest yield points you need to know for USMLE is that acute ¯ perfusion (i.e., seconds to
Pre-renal
minutes) to the kidney from blood loss, acute heart failure exacerbation, or arrhythmia (i.e.,
VFib for 30 seconds before resusc) causes acute tubular necrosis, not pre-renal.
- This is all over the NBME exams. For example, if they say a guy loses lots of blood during
surgery and receives 20 packs of RBCs, then two days later while recovering in hospital he
gets oliguria + deterioration of renal function, the answer is acute tubular necrosis, not pre-
renal. Student says, “Wait, but there was ¯ renal perfusion though, so how does that make
any sense. Isn’t pre-renal caused by ¯ perfusion?” à The PCT of the kidney is the most

MEHLMANMEDICAL.COM 4
MEHLMANMEDICAL.COM

susceptible to anoxic/hypoxic injury due to the ­ concentration of ATPase transporters. So


acute ¯ drop in blood flow à acute hypoxia à PCT sheds.
- There is a 2CK NBME Q where they mention a guy post-op who had not lost any blood
during surgery + had not experienced any episodes of low BP + was started on ketorolac (an
NSAID) for pain post-op + now has oliguria and ­ creatinine à answer = “hypoperfusion” as
cause; this is example of where NSAIDs, not ATN, can cause post-op oliguria from pre-renal.
- HY causes of pre-renal for USMLE are NSAID or diuretic use, chronic left heart failure, or
dehydration from days of vomiting/diarrhea.
- USMLE will give patient who’s been on an NSAID (e.g., naproxen) for several weeks, or who
was commenced on furosemide (loop diuretic) a few days ago.
- Vignette can give patient who is on an NSAID + now has peripheral edema + they ask why
there’s edema à answer = “¯ renal excretion of sodium.” This is because the PCT ­
reabsorption of sodium in pre-renal (FENa <1%). They will also sometimes give NSAID +
edema + ask what you do, and the answer is just “discontinuation of ibuprofen.”
- Essentially synonymous with acute tubular necrosis on USMLE.
- The kidney can’t reabsorb urea and sodium as easily, so BUN is lower (i.e., BUN/Cr is <20)
and sodium is higher in the urine (i.e., FENa is >1%).
Intra-renal - I discuss acute tubular necrosis in more detail in its own table below, but as I mentioned
above in red font, the most important point for USMLE is that you remember acute drop in
perfusion to the kidney causes acute tubular necrosis, not pre-renal. I need to be an asshole
and inculcate that.
- Almost always due to BPH on USMLE, but can also be due to ovarian or cervical cancer
impingement on the ureter(s).
- What you need to know is: old dude + high creatinine = BPH till proven otherwise.
- The answer on USMLE is often just “increased Bowman capsule hydrostatic pressure.”
- Next best step is “insertion of catheter” to relieve the urinary retention, even if the patient
has bacteriuria (i.e., choose catheter insertion over antibiotics).
- Another 2CK Q has “check post-void volume” as answer in elderly male with high Cr.
Post-renal USMLE won’t force you to choose between catheter insertion of checking post-void volume.
- Normal post-void volume is <50ish mL. If the USMLE wants overflow incontinence, they’ll
give post-void volume ~300-400+ mL.
- Diabetic neurogenic bladder (i.e., hypotonic bladder from neuropathy to the detrusor
muscle) can also lead to overflow incontinence and post-renal azotemia, but as I said, most
Qs on USMLE will focus on the old dude with BPH.
- Hydronephrosis can be seen as large, dilated kidneys in patients with obstruction, but this
is rare diagnosis.

Nephritic vs Nephrotic syndromes


- Nephritic conditions have blood in the urine; nephrotic syndromes don’t.
- You need to memorize for USMLE which HY conditions are nephritic vs nephrotic. Don’t worry, I keep
things real clean and concise in the tables below without the bullshit nonsense of other resources.
- There are four main points you should memorize as being part of each syndrome type.

Nephritic syndromes:
1) Hematuria (blood in the urine).
2) Oliguria (¯ urinary output; the definition is <400 mL/day, but USMLE won’t assess that number).
3) Azotemia (­ blood urea nitrogen; this is because urinary output is ¯).
4) Hypertension (­ RAAS due to inflammation of renal microvasculature).

Nephrotic syndromes:
1) Proteinuria (nephrotic level by definition is >3.5g/day, but USMLE won’t assess that number).
2) Hypoalbuminemia (due to the proteinuria; will be <3.5 g/dL, but USMLE doesn’t care about the #).
3) Peripheral edema (due to the hypoalbuminemia à ¯ serum oncotic pressure à transudation of
fluid into interstitial spaces; severe can present with ascites; if the stem says “pre-sacral edema,”
this is nephrotic syndrome till proven otherwise on USMLE).

MEHLMANMEDICAL.COM 5
MEHLMANMEDICAL.COM

4) Hyperlipidemia (liver pumps out apolipoproteins in an attempt to preserve serum osmolality /


oncotic pressure).

- Should be noted that nephritic syndromes often have proteinuria, just usually not at nephrotic levels.
Students sometimes erroneously think nephritic syndromes don’t have proteinuria because it’s not part of
the 4-point categorization above.
- You don’t have to worry about the notion of which conditions are “both nephritic and nephrotic.” What is
most important is: simply know which conditions have hematuria and which ones don’t (via tables below).
- Nephrotic syndromes can ­ risk of DVT, renal vein thrombosis, and varicocele due to loss of antithrombin
III in the urine à hypercoagulable state. This is because nephrotic syndrome usually entails non-specific
massive protein loss due to loss of size and charge barrier, and antithrombin is a protein. The step 1 NBME
wants you to know left renal vein thrombus accretion from hypercoagulable state can cause varicocele.

HY Nephritic syndromes
- Aka “proliferative glomerulonephritis” (asked on NBME, where they give
vignette of PSGN, and answer is just “proliferative glomerulonephritis”).
- Answer can sometimes be written by NBME as just “acute glomerulonephritis.”
- The answer on USMLE for red urine 1-3 weeks following a sore throat caused by
Group A Strep (Strep pyogenes). This is in contrast to IgA nephropathy (which I
discuss in detail below), which is red urine 1-3 days following a sore throat.
- PSGN can be caused by skin infections. This can be impetigo (school sores),
cellulitis, or erysipelas (I discuss these in detail in my HY Derm PDF). For example,
the Q might say a 10-year-old has yellow crusties on his arm for the past 7 days
(impetigo) + now has red urine à answer = “acute glomerulonephritis.”
- Type III hypersensitivity (antigen-antibody complexes that form in blood and
Post-streptococcal deposit in the kidney). Don’t confuse with rheumatic heart disease, which is a
glomerulonephritis type II HS.
(PSGN) - Serum complement protein C3 can be ¯ (can also be ¯ in SLE, but unrelated).
- Streptolysin O or A titers will be ­.
- PANDAS is tested on Psych forms for 2CK à Pediatric Autoimmune
Neuropsychiatric Disorder Associated with Streptococci à presents as new-onset
tic/Tourette, OCD, or ADHD within weeks of Group A Strep infection.
- Descriptors such as subepithelial deposits or “lumpy bumpy” appearance on
electron microscopy (EM), etc., have been parroted across resources over the
years but are essentially garbage for USMLE.
- PSGN usually self-resolves in kids without sequela. But USMLE wants you to
know that ­ age is means worse prognosis, where chance of renal failure is ­ it if
occurs in adults. This is probably related to the more robust immune response
resulting in more advanced renal damage.
- Aka Berger disease; IgA deposition in renal mesangium.
- Red urine 1-3 days after a sore throat. This is in contrast to PSGN, which is red
urine 1-3 weeks after a sore throat. I just mentioned it above obviously. But
students fuck this up despite the inculcation so I’m reiterating it like an asshole.
- Caused by viral infection, not Group A Strep.
- Can sometimes be caused by GI infections, but USMLE usually avoids this
etiology unless including it in the Henoch-Schönlein purpura constellation.
IgA nephropathy
- Henoch-Schönlein purpura is LY for Step 1 but HY for 2CK Peds. It’s a tetrad:
1) Palpable purpura (usually on buttocks/thighs).
2) IgA nephropathy (red urine).
3) Arthralgias.
4) Abdominal pain.
- All 4 need not be present for HSP, but the abdo pain component here is
presumably viral gastroenteritis leading to IgA nephropathy.
Alport syndrome - X-linked disease; mutation in collagen IV gene.

MEHLMANMEDICAL.COM 6
MEHLMANMEDICAL.COM

- Do not confuse with Goodpasture syndrome, which is antibodies against type IV


collagen.
- The answer on USMLE for a male who has red urine + an eye or ear problem
(collagen IV is present in basement membranes in the kidney, ear, and eyes).
- The “eye problem” can be blurry vision/cataracts; the “ear problem” will be
neurosensory hearing loss (due to organ of Corti dysfunction).
- Some students have asked if it’s XR or XD. Literature appears to be varied, but
USMLE doesn’t care. On offline NBME 18, however, it’s mentioned in a Q as XR.
- You can memorize “splitting of the lamina densa” as associated with Alport.
- The answer on USMLE for red urine in a patient with hepatitis C, heroin use, or
malignancy.
- Descriptors such as “dense deposits,” “C3 nephritic factor” and “duplication of
Membranoproliferative
basement membranes” are all basically nonsense for USMLE.
glomerulonephritis
- Answer is “renal biopsy” as the next best step. This guides our management.
(MPGN)
- A 2CK IM form has “heroin” as the answer (heroin-induced nephropathy) for
patient with protein and blood in the urine. In contrast, FSGS (discussed below)
from heroin has no blood in the urine.
- The answer on USMLE for red urine in a patient with SLE.
Diffuse proliferative
- Can sometimes be associated with “wire-looping of capillary walls.”
glomerulonephritis
- Same as with MPGN, USMLE wants “renal biopsy” as the next best step, since
(DPGN)
this guides our management.

HY Nephrotic syndromes
- The answer on USMLE for otherwise unexplained edema in a kid who doesn’t
have blood in the urine.
- Can present as peripheral edema, periorbital edema, and/or ascites.
- Almost always this is pediatric. Very rarely it can be due to Hodgkin in adult.
- Classically post-viral (i.e., URTI), but ~50% of vignettes won’t mention that.
- In other words, textbook vignette is an 8-year-old who has the sniffles for 4
days, followed by peripheral and periorbital edema a week later, without blood in
Minimal change the urine. Once again though, the stem need not mention the viral infection.
disease - Called minimal change disease because light microscopy (LM) shows no
abnormalities. EM, however, shows effacement of the podocytes.
- Mechanism for nephrotic syndrome is “loss of size and charge barrier.”
- Corticosteroids are the treatment and are highly effective.
- A 2CK Peds Q gives minimal change disease as etiology for spontaneous
bacterial peritonitis (i.e., any cause of ascites can cause SBP; I discuss this stuff in
the GIT PDF in extensive detail).
- MCD is aka lipoid nephrosis (lipid droplets can be seen in urine on LM).
- The answer on USMLE for nephrotic syndrome in patient who has:
- Exposure to drugs: dapsone, gold salts, sulfonamides.
- Infection: hepatitis B (Hep C can cause it too, but rare).
Membranous
- Visceral cancers, e.g., breast, pancreatic.
glomerulonephritis /
- Autoimmune (primary): antibodies against phospholipase A2 receptor.
nephropathy
- The LM image is important and shows highly eosinophilic (pink) and inflamed /
thickened capillary walls.

MEHLMANMEDICAL.COM 7
MEHLMANMEDICAL.COM

- Can cause “spike and dome” appearance of subepithelial deposits on EM.


- The answer on USMLE for nephrotic syndrome in multiple myeloma.
- Amyloidosis means protein depositing where it shouldn’t be depositing.
- Multiple myeloma = cancer of plasma cells that produce too much
immunoglobulins à high serum IgG kappa/lambda light chains. Immunoglobulins
are proteins à these fly through the kidney and cause Bence-Jones proteinuria;
they also deposit in the renal parenchyma, causing renal amyloidosis.
- Biopsy shows classic apple-green birefringence with Congo red stain.

Renal amyloidosis

- For some magical reason, USMLE cares you know that the amyloid appears this
way because it is b-pleated sheets (i.e., answer on NBME; a-helices is wrong);
makes sense since the former are “flat” and can reflect light at different angles.
- An NBME Q has “renal parenchymal disease” as answer for kidney issue due to
multiple myeloma.
- Diabetes is the most common cause of chronic renal failure.
Diabetic - Renal failure simply means ¯ glomerular filtration rate (GFR).
glomerulosclerosis - USMLE wants the first two changes that occur in the kidney due to diabetes as
follows:

MEHLMANMEDICAL.COM 8
MEHLMANMEDICAL.COM

1) Hyperfiltration (­ glucose crossing the glomerular basement pulls


water with it à polyuria). Sometimes the Q can just simply give you
diabetes + they ask what’s most likely to be seen initially à answer = “­
glomerular filtration.” Over time, the kidney will fail and GFR will ¯.
2) Thickening of the glomerular basement membrane (non-enzymatic
glycosylation of the membrane à loss of size and charge barrier).
- Kimmelstiel-Wilson nodules are composed of hyaline and look like pink circles
on light microscopy. This image is very HY.

- Hyaline arteriolosclerosis is deposition of hyaline (protein) in the walls of the


renal vasculature. Although this can be due to many things, on USMLE, they want
you to know this is caused by diabetes.
- ACEi (e.g., lisinopril) or ARBs (e.g., valsartan) are the first drugs used in diabetes
if there is HTN (>130/80 in diabetes, not 140/90), proteinuria, or ­ creatinine or
renin.
- AT-II normally constricts efferent arterioles leaving the kidney, causing a backup
of pressure at the glomerulus. This serves the purpose of maintaining GFR (i.e.,
filtration through Bowman capsule) when renal blood flow has ¯.
- In other words, when renal perfusion ¯, RAAS goes ­, AT-II goes ­, and we get
constriction of efferent flow leaving the kidney à ­ filtration fraction (GFR/renal
plasma flow), where we have GFR staying the same despite ¯ perfusion.
- ACEi/ARBs ¯ constriction of efferent arterioles (they don’t “dilate”; this refers to
the action of prostaglandin on the afferent arterioles going to the kidney),
thereby ¯ filtration fraction and reducing GFR. This might sound like a bad thing –
i.e., “Why would we want to ¯ GFR in diabetes if it’s the most common cause of
chronic renal failure?” à The answer is because the slightly reduced GFR and
filtration fraction early on ¯ the rate at which excess glucose is filtered across the
glomerulus. Over time, it is inevitable that the glomerular basement membrane
will non-enzymatically glycosylate and thicken, so ACEi/ARBs slow this process
(i.e., slightly reduce GFR early in order to prevent a massive decrease later).
- The answer on USMLE for nephrotic syndrome in sickle cell.
Focal segmental - Can also be caused by heroin and HIV.
glomerulosclerosis - Not as responsive to steroids as MCD.
(FSGS) - FSGS literally means “some parts of some nephrons are affected.”
- Focal = some glomeruli.

MEHLMANMEDICAL.COM 9
MEHLMANMEDICAL.COM

- Diffuse = all glomeruli.


- Segmental = part of nephron.
- Global = all of nephron.

Acute tubular necrosis


- Sloughing of PCT tubular epithelial cells due to chemical or ischemic insult.
- Only ~50% of Qs will mention “muddy brown granular casts,” or “dirty brown granular casts,” or “brown,
pigmented casts.”
- The other ~50% of Qs will just mention oliguria or acutely worsening renal function in patient with classic
ATN etiologies.
Next best step in diagnosis (answer on Surg form 8) is “measurement of urine sodium and creatinine
concentrations.” We expect FENa >1% and BUN/Cr <20 almost always for ATN.
Cause HY points
- Can be caused by drugs: aminoglycosides (gentamicin, tobramycin, amikacin);
cisplatin (a chemo agent), or IV contrast.
- Any of the above drugs + new-onset oliguria or ­ creatinine (there need not be
mention of casts) = ATN.
- For instance, new Step 1 NBME simply tells you patient was given many drugs
and now has creatinine of 2 (NR 0.1-1.2) + no mention of casts + they ask for what
was given à answer = gentamicin. Not hard.
Drugs - Patient who’s recently been treated for endocarditis + now has new-onset
oliguria = ATN due to gentamicin (empiric Tx for endocarditis = gentamicin +
vancomycin [new literature suggests vancomycin is not reliably nephrotoxic]).
- Contrast nephropathy is exceedingly HY on 2CK, where they want you to know
that adequate hydration prior to IV contrast is the number-one way to prevent it.
For instance, they’ll say patient got a CT + now has ­ creatinine + they’ll ask how
it could have been prevented à answer = “0.9% saline,” or just “IV hydration
therapy.”
- Myoglobin is nephrotoxic. Big risk factors are alcoholism, falls, intense training
(e.g., marathon), McCardle syndrome, statins/fibrates.
- Rhabdo causes a false (+) blood on urine dipstick. They will say urinalysis shows
2+ blood but only 1-2 RBCs/hpf. This is because the dipstick can’t differentiate
between free myoglobin and hemoglobin on RBCs.
- One Surgery Q gives 3-4 RBCs/hpf as negative. I’ve seen this fool some students
who were already aware of the false (+) blood point about rhabdo. If the Q wants
(+) RBCs, they’ll say like 10-20+/hpf.
- The other half of rhabdo vignettes won’t mention the false (+) urinalysis and will
Rhabdomyolysis instead just say oliguria. You need to know that refers to ATN.
- For example, an NBME-favorite vignette is patient being found at bottom of
stairs in his/her house + has false (+) blood on urine dipstick and/or oliguria à
answer = rhabdomyolysis.
- They also like ­ K+ as part of rhabdo vignettes (makes sense if we have renal
failure from ATN) and ­ serum creatine kinase (CK).
- Q can say 44-year-old guy found on bench in park has ­ K+, ­ CK, and oliguria à
diagnosis = rhabdo. Or they’ll say just ­ CK + oliguria, and they ask what is most
likely to be seen in this guy à answer = ­ K+.
- It’s to my observation acute ischemia is the highest yield cause of ATN on NBME
exams.
- Even though the medulla of the kidney technically receives less blood flow
compared to the cortex, as I mentioned earlier, the USMLE is obsessed with the
Acute ischemia PCT of the kidney as most susceptible to anoxic / hypoxic injury. This is due to the
­ concentration of PCT ATPase transporters, so acute ¯ drop in blood flow à
acute hypoxia à PCT sheds.
- Step 1 Qs love cellular swelling as part of this mechanism – i.e., acute ischemia
à ¯ activity of PCT ATPases à buildup of intracellular Na+ à water stays with

MEHLMANMEDICAL.COM 10
MEHLMANMEDICAL.COM

sodium à tubular cell swelling. Answer on NBME for why there’s swelling simply
= ¯ ATPase activity (this is similar mechanism for why we get swelling + hemolysis
in pyruvate kinase deficiency).
- Acute ischemia to the kidney can be due to loss of blood from trauma/surgery,
acute arrhythmia (e.g., 30 seconds of Vfib), MI, or acute exacerbation of heart
failure (on 2CK Free 120). These conditions cause ATN, not pre-renal. Once again,
¯ flow to the kidney causing pre-renal azotemia will be more subacute/chronic,
such as due to NSAID use, recent initiation of diuretic, or dehydration. There are
rare Qs that are exceptions, but that is the general principle.
- As mentioned earlier, USMLE will give a one-liner where they say, “Dude had
surgery where he lost a lot of blood + received many packs of RBCs. 2 days later,
he’s now recovering in hospital + gets oliguria” (they don’t say anything about
brown casts) à answer = ATN, not pre-renal.
- Likewise, they’ll say woman had surgery + had 30-second episode intra-
operatively where BP fell to 80/40 à answer = ATN, not pre-renal.
- Guy has an MI (cardiogenic shock) + new-onset oliguria à ATN, not pre-renal.
- Guy has burns covering 50% of his body + develops oliguria à ATN due to
excessive fluid loss à acute ¯ perfusion to kidney.
- Acute exacerbation of heart failure causing ATN is a difficult one since heart
failure is classically associated with pre-renal from chronic ¯ blood flow to the
kidney. But in acute exacerbation of heart failure, we have acute ¯¯ blood flow,
leading to ATN (as I said, it’s on Free 120).

Other renal conditions associated with ischemia


- Diffuse cortical necrosis is a term thrown around sometimes and causes
confusion for students. I don’t think I’ve ever seen USMLE assess this. I’ve only
seen it show up rarely as an incorrect answer choice.
Diffuse cortical necrosis
- DCN is simply considered an extension of ischemic ATN, where DCN can occur
in severe cases of ischemia (e.g., obstetric hemorrhages).
- You can be aware it exists, but don’t worry about it for USMLE.
- Renal papillae = parts of kidney where the collecting ducts meet the ureters.
- The answer on USMLE for red/dark urine in a patient with sickle cell.
- In contrast, if a sickle cell patient has a renal condition + no blood in the urine,
the answer = FSGS, as mentioned earlier.
- RPN is loss of the renal medulla from ischemia, but unlike ATN, the etiology is
usually more subacute or chronic. NSAIDs are a notable cause in this regard,
Renal papillary necrosis where ¯ afferent blood flow due to ¯ vasodilating prostaglandins over weeks-
(RPN) months causes the medullar blood flow to be slowly choked off.
- RPN can occur more acutely in sickle crises, where sickling within
microvasculature feeding the medulla results in focal pockets of ischemia.
- Pyelonephritis (discussed later) can also cause RPN due to inflammatory
compression of the medullar microvasculature. There’s one NBME Q I can recall
where renal papillary necrosis is the answer due to infection. But I’d say, overall,
the highest yield point is just remembering red/dark urine in sickle cell = RPN.

Interstitial nephritis
- Exceedingly HY renal condition often confused with ATN.
- Aka interstitial nephropathy, or tubulointerstitial nephritis/-nephropathy.
- Think of this as “an allergic reaction of the kidney.”
- 4/5 Qs will be an NSAID, b-lactam, or cephalosporin, followed by getting a maculopapular rash and WBCs
(eosinophils) in the urine. This presentation is textbook/pass-level.
- Only ~50% of vignettes will mention the maculopapular rash.

MEHLMANMEDICAL.COM 11
MEHLMANMEDICAL.COM

- The stem need not say “eosinophils” either. They can just say patient is on b-lactam + now has WBCs in
the urine + no mention of rash à answer = interstitial nephritis. If they say the rash, it’s even easier.
- 1/5 Qs will just say a patient was on an NSAID, b-lactam, or cephalosporin and now has mild proteinuria
and hematuria. They don’t mention a rash or WBCs in the urine.
- There is also a singular NBME Q where they just say a patient has simple peripheral edema due to an
NSAID, with no mention of anything else, and the answer is interstitial nephropathy. This is more unusual,
since “NSAIDs + edema” classically = pre-renal, but I’ve seen the NBME assess interstitial nephropathy for
this as well.
- NBME also can give you simple vignette of interstitial nephritis, and then ask for the location in the kidney
that’s affected (e.g., efferent arterioles, etc.) à answer = “renal tubule.”
- For example, 40M + treated with nafcillin for 6 weeks for MSSA endocarditis + now has maculopapular
rash and eosinophils in the urine; diagnosis? à interstitial nephritis.
- 60F + using naproxen (an NSAID) for 6 weeks for her osteoarthritis + has peripheral edema à answer =
interstitial nephropathy.
- 35F + just recently finished 10-day course of cephalexin + has mild proteinuria and hematuria à answer =
interstitial nephropathy.
- 35F + taking ibuprofen + has maculopapular rash and eosinophils in the urine; Q asks where in the kidney
is fucked up à answer = “renal tubule.” Student says, “But don’t NSAIDs affect the afferent arteriole?” à
Yes, but this particular presentation is clearly tubulointerstitial nephropathy. I haven’t seen them be
ambiguous here with “NSAID + peripheral edema alone,” where you have to debate whether it’s interstitial
nephropathy or pre-renal. They’ll either ask one or the other.

Renal tubular acidosis


Quick notes before discussing RTA:

MUDPILES = mnemonic for high anion-gap metabolic acidoses = Methanol, Uremia (renal failure), DKA,
Phenformin (a drug you don’t have to worry about), Iron/Isoniazid, Lactic acidosis, Ethylene glycol,
Salicylates (aspirin).

Anion-gap is calculated as Na+ - (Cl- + HCO3-). Normal range is 8-12. High anion-gap = 13 or greater.
- USMLE really doesn’t give a fuck that you know the specifics of types I, II, and IV.
- The way they assess this is by you knowing this is a type of normal anion-gap metabolic acidosis (i.e., it is
not part of MUDPILES).
- For example, you’ll get a 15-line massive paragraph + tons of lab values + have no idea what’s going on,
but then you calculate the anion gap as 12 (NR 8-12), so you can eliminate all of the MUDPILES answer
choices, such as lactic acidosis, DKA, and ethylene glycol poisoning, and you’re left with, e.g., Crohn disease
or renal tubular acidosis. Then you just say, “Well this clearly ain’t Crohn, so it must be RTA.” That is how I
would say 4/5 RTA Qs show up.

- RTA type I = ¯ ability to secrete H+ in the cortical collecting duct.


- RTA type II = ¯ ability to reabsorb HCO3- in the PCT.
- RTA type IV = renal resistance to aldosterone, or hyporeninemic hypoaldosteronism.

- Type IV presents with hyperkalemia. The others do not. Type III apparently is hyper-rare.
- Type IV will be a patient who has Addisonian-like picture (i.e., ¯ Na+, ­ K+, ¯ HCO3-), but the vignette will
present in a patient who has chronic renal failure.
- Type II can be caused by Fanconi syndrome (discussed more in table below).
- Type I can apparently have renal parenchymal stones (i.e., stones in the actual tissue of the kidney, rather
than within the tubular lumina as with traditional nephrolithiasis).
- As I said, the key point is you just know RTA is normal anion-gap / not part of MUDPILES. That’ll cover you
like 4/5 times.

MEHLMANMEDICAL.COM 12
MEHLMANMEDICAL.COM

Other random renal conditions


- Lysis of tumor cells (especially leukemic) by chemoradiotherapy leads to release
of uric acid and phosphate that can overwhelm the kidneys and cause crystals,
namely from uric acid.
- USMLE loves asking what can be given to prevent renal failure in patient
Tumor lysis syndrome undergoing chemo à answer = “allopurinol.”
- Or they’ll ask for MOA of drug that can prevent renal failure à answer =
“xanthine oxidase.”
- The arrows USMLE wants for what will be seen in the blood in tumor lysis
syndrome is same as renal failure except we also have ­ uric acid.
- Inability of the PCT to reabsorb “lots of stuff.”
- Shows up on NBME as an arrow Q, where they ask about the reabsorption of
amino acids, glucose, bicarbonate, and phosphate, and the answer is a ¯ for all of
them.
- Since bicarb reabsorption is impaired, this can lead to RTA type II, but USMLE
doesn’t really give a fuck. The key point is the ¯ for the reabsorption of the above
Fanconi syndrome
substrates.
- For whatever magical reason, an important cause of Fanconi syndrome is
consumption of expired tetracyclines.
- Don’t confuse Fanconi syndrome with Fanconi anemia. The latter is an
autosomal recessive aplastic anemia with hypoplastic thumbs/radii (I’ve made
YouTube clips discussing this).
- Inability of the PCT to reabsorb tryptophan.
- Can cause niacin (B3) deficiency in theory, since tryptophan is a precursor in
niacin synthesis.
Hartnup disease
- This is more just textbook masturbation to be honest that’s been parroted over
the years. Can’t say I’ve seen this ever assessed once. Just mentioning it here
otherwise I’ll get some umbrageous student popping into my DMs about it.
- Cystine stones (hexagonal) can occur in young adult due to an inability to
reabsorb the COLA (dibasic) amino acids à Cysteine, Ornithine, Lysine, Arginine.
Cystinuria
- Cystine is two cysteines put together.
- Cystinuria can be diagnosed with cyanide-nitroprusside test.
- Nonsense diagnosis on USMLE that will sometimes show up as an incorrect /
distractor answer choice.
Hepatorenal syndrome - Apparently liver failure can sometimes cause renal failure via obscure
mechanisms.
- Renal biopsy will be normal almost always.
Thin glomerular - Benign familial condition where patient can have mild hematuria.
basement membrane - Thin basement membranes allow for passage of small amounts of RBCs.
disease - No treatment necessary.

Renovascular hypertension
- Narrowing of one or both renal arteries due to atherosclerosis that causes ­
renin-angiotensin-aldosterone system (RAAS) and ­ BP.
- Q will be patient over the age of 50 with cardiovascular disease risk factors,
such as diabetes, HTN, and/or smoking.
- Patients who have pre-existing HTN causing atherosclerosis leading to RAS will
often have 10-20 years of background HTN that then becomes accelerated over
Renal artery stenosis
a few-month to 2-year period. What this means is: the slowly developing
(RAS)
atherosclerosis in the renal arteries finally reaches a point at which the kidney is
unable to maintain autoregulation, and the RAS is now clinical (i.e., accelerated
HTN of ­­ BP within, e.g., 3 months).
- Another way USMLE will give RAS is by giving ­ BP in patient with significant
evidence of atherosclerotic disease (i.e., Hx of coronary artery bypass grafting,
intermittent claudication), and then ask for the most likely cause à RAS. You

MEHLMANMEDICAL.COM 13
MEHLMANMEDICAL.COM

have to say, “Well he clearly has atherosclerosis in his coronaries and aortoiliac
vessels, so that means he’ll have it in the renal arteries too.”
- Q can say older patient with carotid bruit has recent ­­ in BP and then ask for
diagnosis à answer = RAS. Similar to above, if the patient has atherosclerosis in
one location (i.e., the carotids), then he/she will have it elsewhere too.
- HY factoid about RAS is that ACEi or ARBs will cause renin and/or creatinine
to go up. This is a HY point that is often overlooked and is asked on NBMEs. I
have not seen NBME care whether it’s uni- or bilateral in this case. à Kidney
can autoregulate across flux in perfusion. Patients with already-compromised
renal blood flow are more sensitive to the subtle ¯ in filtration fraction that
occurs secondary to ACEi/ARB use, so renin/creatinine ­.
- If USMLE gives you unilateral RAS, renin is only ­ from that kidney. The other
kidney will not produce ­ renin.
- After renin and aldosterone levels are obtained, MR angiography of the renal
vessels is what USMLE wants for the next best step in diagnosis.
- The answer on USMLE for narrowing of the renal arteries in a woman 20s-40s.
- Not the same as renal artery stenosis, and not caused by atherosclerosis.
- If you broadly say “renal artery stenosis,” that specifically refers to
atherosclerosis of the renal arteries in patient >50 with CVD.
- FMD is tunica media hyperplasia (not dysplasia, despite the name) that results
in a “string of beads” appearance on renal angiogram.

Fibromuscular dysplasia
(FMD)

- MR angiography is answer on NBME for diagnostic modality.


- Can affect the carotid vessels. A 2CK Surg Q gives FMD vignette and also says
there is 25% occlusion of one of the carotids.
- As discussed in the cardio PDF, this is a medium-vessel vasculitis that causes a
“string of pearls” appearance of the renal vessels. This can be confused with
FMD, but note the difference in the imaging. FMD shows more beading along
the larger/proximal arterial sections, whereas PAN shows pearls more at the
vascular termini, with the more proximal parts more likely to be spared.

Polyarteritis nodosa
(PAN)

MEHLMANMEDICAL.COM 14
MEHLMANMEDICAL.COM

Angiogram of PAN shows pearls that are more distal in the renal vasculature;
FMD, in contrast, shows more proximal beading, with the termini not as
conspicuously involved.

- There is an NBME Q where they list both PAN and FMD as answer choices and
it relies on you knowing the angiogram to get it right. They don’t mention
hepatitis B, but answer is PAN. That’s why I’m harping on this here.
- Causes fibrinoid necrosis.
- Can be caused by hepatitis B.
- Spares the pulmonary vasculature.
- The answer on USMLE for hypertension in a neonate following umbilical artery
catheterization. USMLE simply wants “­ renin, ­ aldosterone” as the answer.
- Sounds weird, but you need to know umbilical artery catheterization is a major
risk factor for renal artery thrombosis in neonates.
Renal artery thrombosis - 2CK NBME Q gives brief umbilical artery catheter insertion in kid born 26
weeks’ gestation in order to monitor blood pressure for a pneumonia. 3 weeks
later, he has ­ BP (128/86) à answer = “­ renin, ­ aldosterone” as answer.
- BP in term neonates should be ~60/40. In a preemie 29 weeks’ gestation, it
should be ~50/30 according to Google.

HY Uremia points
- As mentioned earlier, “uremia” means “urea in the blood,” but is used to refer to patients who have
symptomatic renal failure with poor lab values.
- On USMLE, uremic patients will have: ­ K+, ¯ HCO3-, ¯ Ca2+, ­ PO43-. Na+ is variable.
- Friction rub in patient with ­ BUN and Cr.
Uremic pericarditis
- Answer = hemodialysis.
- Mental status change in patient with ­ BUN and Cr.
Uremic encephalopathy
- Answer = hemodialysis.
- Nosebleeds and/or petechiae in patient with renal failure.
- Qualitative, not quantitative, platelet problem. In other words, bleeding
time is ­, but platelet count is normal.
- Mechanism is ­ BUN causing impairment of platelet function.
Uremic platelet dysfunction - Answer on USMLE can be written as “acquired platelet dysfunction.”
- Treatment = hemodialysis.
- On 2CK NBME 10, Surg Q wants “initiation of dialysis” as answer in uremic
patient pre-op. Improving RFTs will ¯ peri-op morbidity/mortality + ¯ risk of
intra-op bleeding due to uremic platelet dysfunction.
- Yellow skin sometimes seen in severe renal failure due to ¯ ability to
Uremic frost excrete bilirubin. Can show up as mixed white deposits as well (frost).
- Can be associated with itchy skin (uremic pruritis).

Urolithiasis
- Urolithiasis is broad, umbrella term that refers to both nephrolithiasis and ureterolithiasis.
Stone type HY points
- Most common type of stone. Can be calcium oxalate or calcium phosphate,
although I’ve never seen USMLE once assess or give a fuck about phosphate stones.
- Most young adults with idiopathic kidney stones will have “normocalcemia and
hypercalciuria” – i.e., normal serum calcium but elevated urinary calcium.
Calcium
- Crohn disease and disorders causing fat malabsorption increase the risk for
calcium oxalate stones (­ fat retained in GI tract à ­ chelation with calcium in GI
tract à ¯ calcium available to bind oxalate à ­ oxalate absorption by GI tract à ­
urinary oxalate).

MEHLMANMEDICAL.COM 15
MEHLMANMEDICAL.COM

- Hypercalcemia (e.g., due to primary hyperparathyroidism or malignancy) are HY


etiologies.
- Ethylene glycol (Anti-Freeze) and hypervitaminosis C can cause oxalate stones.
- First step in prevention and treatment of stones is “adequate hydration.”
- Thiazides can be used to prevent recurrent stones by ¯ urinary calcium. NBME has
as an Q that asks why à answer = ­ reabsorption of calcium.
- In theory, alkalinization of the urine can help treat oxalate stones, and
acidification of the urine can help treat phosphate stones.
- Composed of ammonium magnesium phosphate.
- Formed in ­ pH in the presence of urease (+) bacteria.
- Klebsiella, Serratia, and Proteus are HY causes.
- These stones are large, ram-horn like. The image is HY.

Struvite

- Apart from adequate hydration, Tx is acidification of urine with NH4Cl + surgery.


- Seen in patients who have gout. Not complicated.
- First-line Tx for chronic gout is xanthine oxidase inhibitors (i.e., allopurinol or
febuxostat). They are preferred over uricosurics (meaning ­ urinary excretion of
uric acid) like probenecid and sulfinpyrazone because the latter agents ­ risk of uric
acid stones.
- Probenecid and sulfinpyrazone inhibit organic anion transporter (OAT), which
Uric acid
normally functions to ­ absorption of uric acid.
- Weird factoid about probenecid is that it can help maintain serum b-lactam levels,
since the latter are excreted by OAT. There’s an NBME Q around somewhere that
asks about this. Sometimes old-school docs will put a patient on a b-lactam and
probenecid. Don’t be confused by this. The patient doesn’t have gout. It’s just a
way of ¯ renal excretion of the b-lactam.
- As mentioned earlier, cystine is two cysteines put together.
- Cystine stones (hexagonal) can occur in young adult due to an inability to reabsorb
Cystine
the COLA (dibasic) amino acids à Cysteine, Ornithine, Lysine, Arginine.
- Cystinuria can be diagnosed with cyanide-nitroprusside test.
- Acyclovir (for HSV and shingles) and indinavir (for HIV) can cause crystal
Miscellaneous
nephropathy. Not particularly HY, but worthy of 1-2 lines here.

Cystic kidneys
- Autosomal dominant polycystic kidney disease; chromosome 16.
- The answer on USMLE if disease starts as an adult (i.e., 30s-40s).
ADPKD
- Cysts are technically present early in life, but only become clinical as adult (i.e., ­ BP
and ­ RFTs).

MEHLMANMEDICAL.COM 16
MEHLMANMEDICAL.COM

- These patients have ­ BP due to ­ RAAS (compression of microvasculature of kidney


due to enlarging cysts).
- Can cause saccular (berry) aneurysms of the circle of Willis à risk for subarachnoid
hemorrhage.
- Highest yield point is that serial blood pressure checks are correct over circle of Willis
MR angiogram screening. Latter is wrong answer on USMLE. MR angiogram screening
of circle of Willis is only done when there is (+) family Hx of SAH or saccular aneurysms.
- Step 1 NBME gives easy vignette of ADPKD, and then the answer is just “polycystin”
as the protein that’s fucked up. Sounds obvious, but it’s asked so I’m mentioning it.
- Cystic kidneys are part of “ciliopathies,” which is obscure term that refers to
conditions where cilia are abnormal. Polycystin is a protein required for cilia function
on renal epithelium.
- Most common extra-renal location for cysts is the liver (85% by age 30).
- Autosomal recessive polycystic kidney disease; chromosome 6.
ARPKD - The answer on USMLE for cystic kidneys in pediatrics.
- Can be associated with hepatic fibrosis.
- Just be aware renal cysts can occasionally develop in other conditions, such as chronic
Miscellaneous dialysis patients.
- Conditions such as medullary sponge kidney are garbage for USMLE. Waste of time.

HY anatomic abnormalities
- Exceedingly HY for Peds for 2CK; also shows up on Step 1 NBME.
- Most common genitourinary abnormality in neonatal males.
- Urethra normally has valves that prevent backflow of urine toward the
bladder; in PUV, the valves face the wrong way, preventing urine exit.
- USMLE will give Qs of varying severity.
- Most severe is in utero oligohydramnios.
- Can present as 12-hour-old neonate who hasn’t yet urinated + has
Posterior urethral valves
suprapubic mass (i.e., full bladder).
(PUV)
- Can also present as 6-week-old boy who has full bladder (i.e., possibly slower
accumulation due to only partially obstructed outflow).
- Obstructed outflow ­ risk for UTIs, cystitis, and pyelonephritis.
- Apart from knowing this diagnosis, highest yield point is that we do
ultrasound followed by voiding cystourethrogram to diagnose.
- NBME Qs might already mention negative ultrasound in the vignette, or
might omit it altogether, where you just select “voiding cystourethrogram.”
- Congenital abnormality in which urine from the bladder can go back up into
the ureter toward the kidney.
Vesicoureteral reflux
- This ­­ risk for recurrent acute pyelonephritis à chronic pyelonephritis (as I
discuss below).

MEHLMANMEDICAL.COM 17
MEHLMANMEDICAL.COM

- Can also occur in pregnancy due to 2 reasons: 1) larger uterus in 3rd trimester
can compress the ureters; 2) progesterone ¯ ureteral peristalsis.
- This is why pregnant women can get pyelonephritis, and also why we always
treat asymptomatic bacteriuria in pregnancy, whereas we don’t treat it if
woman is not pregnant.
- Bit of a weird one, but not me being fancy. It’s on NBME.
- “Failure of canalization of proximal ureter” will be the answer if they tell you
the renal collecting duct system is dilated, but the ureters are not dilated.
- Sounds obvious, but I see students get this wrong a lot.
Ureteral atresia - You need to simply know: kidneys à ureters à bladder à urethra, and if we
have a congenital obstruction at any point, that could be referred to as
“failure of canalization.”
- “Congenital ureteral obstruction” is an answer on one of the NBMEs for
chronic pyelonephritis causing tubular atrophy (once again, discussed below).

Genitourinary infection-related stuff


- For all of the STD/STI-related stuff, I cover that in the Repro/Obyn PDF. I’m keeping this small table more
limited to kidney/bladder stuff.
- Infection of the kidney. “Pyelo” means kidney.
- 9/10 questions will mention fever + costovertebral angle (CVA) tenderness,
which is pain with percussion of the flank.
- “Granular casts” can be seen. Do not confuse this with acute tubular necrosis.
“General granular casts” can sometimes be seen in pyelonephritis and
dehydration. It’s on one of the 2CK IM forms, where students say, “Wait, but they
say granular casts; how is this not ATN?”
- Major risk factors are vesicoureteral reflux (especially in pregnancy) and
posterior urethral valves
- Most common organism is E. coli. Other organisms, i.e., Klebsiella, Serratia, and
Proteus cause struvite stones.
- Neutrophilic infiltration will show up as lots of blue/purple (basophilic) cells on
histo. USMLE likes this image.

Acute pyelonephritis

- For above image, you say, “Mike I feel weird. Idk what I’m looking at.” The blue
cells are neutrophils infiltrating the kidney in acute pyelo. The USMLE will show
images basically identical to this for a variety of infections, e.g., prostatitis, where
the bigger picture concept is, “Oh that’s acute inflammation. Those purple cells
are neutrophils.” That’s what USMLE wants you to know. For instance, a nearly
identical image of prostate histo in old dude with prostate pain and fever à
you’d know immediately it’s prostatitis.

MEHLMANMEDICAL.COM 18
MEHLMANMEDICAL.COM

- It’s to my observation bacteria can be few in the urine in acute pyelo. This
confuses some students. But it’s typically what I see on NBME forms. If the
infection is further down, i.e., UTI in the urethra, then bacteria are more copious.
- Treatment for pyelo is ciprofloxacin or ceftriaxone. USMLE is known to ask
these.
- For example, old dude + high Cr (caused by post-renal from BPH) + treated for
pyelo à now gets sore ankle à was treated with cipro (causes tendonopathy).
- 2CK form has ceftriaxone as an answer in pyelo Q where cipro isn’t listed. But
cipro is classic Tx. It’s to my observation that ceftriaxone is HY drug on 2CK for
community-acquired “general sepsis” or “general complicated/severe infections”
– i.e., it is hard-hitting and covers wide array of community organisms.
- There is one 2CK Q where they mention pyelo is treated with amp + gent in the
stem, but I’ve never seen this assessed as an answer you need to choose. I’ve only
ever seen ciprofloxacin or ceftriaxone as actual NBME answers for pyelo.
- Due to recurrent acute pyelonephritis.
- Almost always pediatric, where they give a 4-year-old who has a small, shrunken
kidney with tubular atrophy and blunting and scarring of the renal calyces.

Chronic pyelonephritis

Kidney appears to show loss of architecture + scarring.

- USMLE will show you kidney that looks similar to the above in a child with
recurrent UTIs, and then the answer is just “tubular atrophy” for what is most
likely to be seen on microscopic examination.
- Another Q shows the same image and the answer is “congenital ureteral
obstruction” as the cause.
- “Thyroidization of the kidney” is a buzzy phrase that has been applied to chronic
pyelo over the years, but USMLE doesn’t so much care about this. What they
really like is the tubular atrophy and scarring / loss of architecture of the calyces.
- Chronic pyelo doesn’t present with cellular infiltration the way acute pyelo does.
It refers to the kidney being destroyed/shrunken from repeated prior infections.
- Suprapubic tenderness in female. Patient need not have fever.
- E. coli most common cause. HY for USMLE you know fimbriae and pilus proteins
facilitate E. coli’s attachment to urothelium.
- Will have bacteria and WBCs in the urine (i.e., bacteriuria + pyuria).
Cystitis - Urinary nitrites and leukocyte esterase can be positive. These just reflect
bacterial infections. Some students get pedantic about which organisms result in
which combo of (+) or (-) findings here, but that’s low-yield for USMLE.
- Can be caused by suprapubic catheters (on 2CK IM and Surg).
- Nitrofurantoin is HY drug to treat cystitis on USMLE.
- Not an infection.
Chronic interstitial - This is >6 weeks of suprapubic tenderness + dysuria (pain with urination) that is
cystitis unexplained, where laboratory and urinary findings are negative.
- They can mention anterior vaginal wall pain (bladder is anterior to vagina).

MEHLMANMEDICAL.COM 19
MEHLMANMEDICAL.COM

- USMLE wants you to know you don’t treat. Steroids are wrong answer.
“Treatment” is standard placating placebo nonsense such as “education,” “self-
care” and “physiotherapy.”
- Classically E. coli infection of the urethra.
- Common in women due to shorter urethras.
- As mentioned above for cystitis, fimbriae and pilus proteins facilitate E. coli’s
attachment to urothelium.
- Inoculation of the urethra following sexual activity is most common mechanism.
- Can be caused by catheters. You need to know dysuria + Hx of catheter = UTI.
- Tangentially, this is also HY for general sepsis, where if they mention Hx of IV
line/catheter, USMLE wants you to be able to say, “Got it. That’s the cause.”
- Trimethoprim, trimethoprim/sulfamethoxazole (TMP/SMX), or nitrofurantoin
are standard treatments for UTI.
General UTI - For prevention, post-coital voiding confers ¯ risk of recurrence.
- If post-coital voiding doesn’t work, “post-coital nitrofurantoin therapy” is
answer on 2CK NBME form.
- If post-coital nitrofurantoin therapy doesn’t work, “daily TMP/SMX prophylaxis”
is the answer. Sounds absurdly wrong / like a bad idea, but it’s an answer a
couple times on Obgyn assessments.
- There’s also a Q where they say a girl was treated successfully in the past with
TMP/SMX for a UTI + ask how to prevent recurrent UTIs now, and they just jump
straight to “daily TMP/SMX prophylaxis.”
- If patient is treated for a UTI and dysuria persists, the next best step is testing
for Chlamydia and Gonorrhea.

HY Bladder incontinences
- The answer for loss of urine with ­ intra-abdominal pressure from laughing, sneezing,
coughing.
- Stereotypical risk factor is grand multiparity (i.e., Hx of many childbirths) leading to
weakened pelvic floor muscles.
- I’d say only ~50% of Qs will mention Hx of pregnancy. The other ~50% are idiopathic.
- Buzzy vignette descriptors and answer choices are: “downward mobility of the
vesicourethral junction,” “urethral hypermobility,” and “urethral atrophy with loss of
urethrovesical angle.”
- Treatment is pelvic floor (Kegel) exercises. These notably strengthen levator ani,
pubococcygeus, and the external urethral sphincter.
Stress
- USMLE is known to ask which muscle is not strengthened by Kegel exercises, which
sounds obscure, since any muscle could theoretically be the answer (“Well the deltoid
isn’t strengthened.”). But a favorite answer here is internal urethral sphincter. The
way you know this is the answer is because internal sphincters are under sympathetic
(i.e., involuntary; autonomic) control, which means it’s impossible to strengthen it via a
voluntary (i.e., somatic) exercise. USMLE doesn’t expect you to be an obstetrician. The
bigger picture concept is simply knowing internal sphincter control is involuntary. It is
external sphincter control that is voluntary (somatic).
- Do not give medications for stress incontinence on USMLE.
- If Kegel exercises fail, patients can get a mid-urethral sling (LY; asked once).
- The answer on USMLE for patient who has an “urge” (NBME will literally say that
word and I’ve seen students get the Q wrong) to void 6-12+ times daily unrelated to
sneezing, coughing, laughing, etc. (otherwise stress incontinence).
- Ultra-HY for multiple sclerosis. I’ve had students ask whether MS is urge or overflow.
Urge
It shows up repeatedly on the NBMEs as urge; I’ve never seen it associated with
overflow. I’d say ~1/3 of urge incontinence vignettes on NBME forms are MS.
- Other vignettes will be peri-menopausal women, or idiopathic in old women.
- Mechanism is “detrusor hyperactivity,” or “detrusor instability.”

MEHLMANMEDICAL.COM 20
MEHLMANMEDICAL.COM

- Vignette can mention woman has urge to void when stepping out of her car, or when
sticking her key in the car/front door of her house. Sounds weird, but these are
important Qs to ask when attempting to diagnose urge incontinence.
- UTIs can present similarly to urge incontinence. Some students have asked, “Well
isn’t that because UTIs are a cause of urge incontinence?” Not really. It just happens to
be that UTIs can sometimes cause transient urinary urgency. For example, if they give a
Q where they say patient had Hx of urinary catheter + now has dysuria and urinary
urgency, answer = “urinary tract infection” on NBME; “detrusor hyperactivity” is wrong
answer.
- Treatment is oxybutynin (muscarinic receptor antagonist); this ¯ activity of the
detrusor muscle of the bladder.
- Some students get hysterical about mirabegron (b3-agonist), but I’ve never seen
NBME forms assess this.
- Will be due to either BPH or diabetes on USMLE.
- Will have ­ post-void volume. Normal is < ~50 mL. On USMLE for overflow, they’ll
give you 300-400 mL as post-void volume.
- As I talked about earlier for BPH, they will give old dude + high creatinine (post-renal
azotemia). Next best step is “insertion of catheter” to relieve the obstruction. If they
don’t have this listed, “measurement of post-void volume” can be an answer. We then
treat the BPH with finasteride (5a-reductase inhibitor) or an a1-blocker (tamsulosin,
terazosin).
Overflow
- For diabetes, the mechanism is neuropathy to the bladder causing “neurogenic
bladder,” or “hypotonic bladder,” or “hypocontractile bladder/detrusor muscle.”
- For neurogenic bladder causing overflow incontinence +­ post-void volume,
remember that USMLE is first obsessed with “measure post-void volume” and
“insertion of catheter” if they are listed. They will not force you to choose between the
two. But they like these answers prior to giving medications.
- Give bethanechol (muscarinic receptor agonist); this stimulates the detrusor muscle.
- Making sure you don’t confuse oxybutynin and bethanechol is pass-level for USMLE.
- Patient who has “wet, wobbly, wacky” presentation (i.e., urinary incontinence, ataxia,
cognitive changes) +/- Parkinsonism (e.g., short-steppage gait).
Normal pressure
- Caused by failure of reabsorption of CSF by the arachnoid granulations, resulting in
hydrocephalus
impingement on the zona radiata and “failure to inhibit the voiding reflex” (answer on
(NPH)
NBME).
- Enlargement of the lateral ventricles on head CT.
AIDS complex - Presents similar to NPH – i.e., “wet, wobbly, wacky” in AIDS patient.
dementia

Urinary retention diagnoses


- Lack of coordination between bladder sphincters and detrusor muscle.
- HY cause is spinal cord injury, which can cause either inability to void or
urinary incontinence.
Bladder neck dyssynergia
- MS is another theoretical cause, which is where “overflow incontinence”
as a result of MS has been asked by students, but once again, I’ve only
ever seen urge incontinence for MS on NBME exams. Qbank ¹ NBMEs.
- Acute urethral obstruction due to catheter removal is HY on Surg forms.
- They’ll say patient has anuria (i.e., no urinary output at all) following
removal of a catheter.
Bladder outflow obstruction
- “Acute urethral obstruction” or “bladder outflow obstruction” is the
answer.
- Treatment on NBME is “re-insertion of urinary catheter.”
- Will present as leg pain + urinary retention on USMLE.
- Can be idiopathic, from disc herniation, or other trauma.
Cauda equina syndrome
- “Saddle anesthesia” is an overrated detail and doesn’t show up in the
vast majority of NBME Qs, as per my observation.

MEHLMANMEDICAL.COM 21
MEHLMANMEDICAL.COM

- Cauda equina is the collection of nerves at the end of the spinal cord.
Impingement on these nerves is what causes the syndrome. I cover this
stuff in more detail in my HY Neuroanatomy PDF.

HY Genitourinary injuries (2CK)


- Always presents with blood at the urethral meatus.
- 3 ways this presents on USMLE:
1) Blood at urethral meatus + saddle injury (dude falls on balance beam).
Urethral injury
2) Blood at urethral meatus + pelvic fracture/instability.
3) Blood at urethral meatus + “prostate not palpable.”
- Next best step is retrograde urethrogram.
- Answer on USMLE for blunt force trauma to abdomen + exquisite suprapubic
tenderness + catheter insertion yields 10-30 mL of dark bloody fluid.
Bladder injury
- Next best step is retrograde cystourethrogram. Do not confuse with voiding
cystourethrogram, which is for posterior urethral valves.
- Just be aware this is USMLE-favorite injury in Obgyn for any surgery done in the
gynecologic area.
Ureteral injury
- Diagnosis is with retrograde pyelogram or CT urogram, although I haven’t seen
the USMLE give a fuck. Any ureter injury Q I’ve seen just has “ureter” as answer.
- Exceedingly HY on both IM and Surg forms on 2CK.
- Will present as bruising or pain over a flank following accident.
- Blood in the urine is highly sensitive, meaning if it’s not there, we can rule-out.
- First step in diagnosis: “Is there gross blood in the urine?” If yes à do CT of the
abdomen with contrast to diagnose (sounds wrong, since we’re worried about
Renal injury
kidney injury, so student says, “Why would we use contrast?” But it’s what
USMLE wants. Ultrasound is wrong answer).
- If patient does not have gross blood in the urine, next best step is urinalysis
looking for microscopic blood. If present à answer = CT of the abdomen with
contrast. If negative à “no further diagnostic studies indicated.”

Urothelial malignancies
- Aka “clear cell carcinoma”; this is most common variant of RCC.
- Classic Q is a smoker over 50 with red urine and a painful flank mass.
- Q need not mention smoking, but it is biggest risk factor.
- USMLE likes the histo for this, which will show you large clear cells.

Renal cell carcinoma


(RCC)

- Can cause polycythemia due to ­ EPO and hypercalcemia due to PTHrp


secretion (correct, same as squamous cell carcinoma of the lung).
- For example, they say 55-year-old male + red urine + polycythemia +
hypercalcemia + show you above image; they ask for the source of the
malignancy à answer “kidney,” not lung. Squamous cell carcinoma of the
lung won’t cause hematuria, nor does it metastasize to the kidney.

MEHLMANMEDICAL.COM 22
MEHLMANMEDICAL.COM

- The answer for painless flank mass in a kid age 2-4 years.
- Does not present in newborns. If they give you a 12-hour-old male, for
instance, with a midline mass, that is posterior urethral valves causing a full
bladder (let alone the fact they say “midline,” not flank). I’ve seen tons of
students select Wilms for this, and I’m like, is the kidney in your midline?
- If they give you a kid 2-4 years with a midline mass, that is neuroblastoma.
- For whatever magical reason, you need to know Wilms tumor is sometimes
caused by mutations on chromosome 11. They ask this factoid on 2CK as well.
There are various Wilms tumor syndromes you need to be aware of:

- Beckwith-Wiedemann syndrome
- Wilms tumor + macrosomia (big baby >4,000g) + hemi-hypertrophy
(half of the body is bigger than the other) + macroglossia +
Wilms tumor omphalocele + hypoglycemia.
(Nephroblastoma) - Hard 2CK Peds Q gives newborn with macrosomia + hemi-
hypertrophy + macroglossia + omphalocele; they don’t mention a
Wilms tumor (makes sense, since we said we won’t see it in
newborns); they ask for what else could be seen in this patient à
answer = “hypoglycemia.” I say hard Q because Wilms plays no role
in the vignette.
- WAGR syndrome
- Wilms tumor, Aniridia (iris abnormalities), Genitourinary anomalies,
Retardation.
- Usually an easy Q, where they mention “aniridia” straight up in the
vignette and then the answer is just “Wilms tumor” for what the kid
can go on to develop.
- Denys-Drash syndrome
- Ambiguous genitalia + Wilms tumor.
- Rare as fuck. 0-3% of renal malignancies. But shows up on Step 1 NBME.

Collecting duct carcinoma

- Offline NBME Q shows some pic of a kidney split open like this and then the
answer is just “naphthylamine” (moth balls) as the causative agent.
- Most common bladder cancer.

Transitional cell carcinoma


of the bladder

Can have papillary structure on histo.

MEHLMANMEDICAL.COM 23
MEHLMANMEDICAL.COM

- Classic vignette is hematuria in smoker without a painful flank mass or


polycythemia/hypercalcemia (otherwise RCC).
- USMLE wants you to know smoking is most common risk factor, but aniline
dyes (industrial clothing dyes) and naphthylamine are important causes.
- 2CK Surg Q gives Stage 0 bladder cancer vignette; answer is just “endoscopic
resection.”
- I’ve never seen BCG vaccine as correct answer on NBME. I’ve only seen it as a
distractor. For whatever reason, some studies in the past have suggested
intra-cystic instillation of the TB vaccine can help with bladder cancer. But this
is wrong on USMLE.
- Can be caused by Schistosoma hematobium (a trematode or fluke, which is a
type of helminth).
- S. hematobium lays its eggs in the bladder wall and cystic veins draining the
Squamous cell carcinoma
bladder.
of the bladder
- Vignette will be young guy who has red urine months after returning from
Africa where he went swimming in a lake à answer is just Schistosomiasis. Or
they ask what he’s at ­ risk of à answer = squamous cell carcinoma.

SIADH vs DI vs PP
- Syndrome of Inappropriate Anti-Diuretic Hormone secretion à means too much ADH
(vasopressin secretion).
- ADH is produced by supraoptic nucleus of hypothalamus à stored in posterior pituitary.
- ADH ­ free water reabsorption by the medullary collecting duct (MCD) of the kidney by
causing aquaporin insertion.
- Central SIADH à follows head trauma, meningitis, brain cancer, and pain (latter on 2CK Surg).
- Ectopic SIADH à small cell lung cancer secreting ADH.
- Drug-induced ADH à ultra-rare on USMLE, but carbamazepine can do it.

- Patient will have dilute serum and concentrated urine:


- ¯ Serum sodium, ¯ serum osmolality, ¯ serum specific gravity.
- ­ urinary osmolality, ­ serum specific gravity.
- You must know serum sodium is normally 135-145 mEq/L. So in SIADH, it’s <135.
- Serum vs urinary osmolalities will be all over the place and you do not need to
memorize values. They might say serum osmolality is 250 and urinary is 750, and then
you say, “Well I can tell serum is dilute compared to urine, which sounds like SIADH.”
SIADH - Specific gravity will be 1.000-1.030 on USMLE. Sounds obscure, but it shows up quite
a bit, particularly on 2CK Qs. It’s to my observation that values 1.000-1.006ish are
“dilute”; 1.024-1.030 are “concentrated.” For 9/10 Qs, values will be what you expect.

- As I talk about in detail in my HY Arrows PDF, only the medullary collecting duct osmolality will
change in response to ADH. The USMLE will ask you for the osmolality of the urine at different
nephron locations in comparison to serum, and the combo is: PCT isotonic; juxtaglomerular
apparatus (JGA) hypotonic; MCD hypertonic. à The PCT is always isotonic no matter what; the
JGA (at top of thick ascending limb of loop of Henle) is always hypotonic no matter what; the
MCD is clearly hypertonic in SIADH since we’re pulling free water out of the urine.
- “Fluid restriction” is first answer in diagnosis on 2CK. We want to see how serum/urinary
values change in response.
- Demeclocycline is answer on 2CK offline NBME 8 for treatment of SIADH. Demeclocycine is
technically a tetracycline antibiotic but isn’t used because it can cause insensitivity to ADH at
the kidney (i.e., nephrogenic diabetes insipidus). So we essentially cause a 2nd problem that
cancels out the 1st problem.
- Conivaptan and tolvaptan are ADH receptor antagonists that can be used for SIADH.
- Central diabetes insipidus à means not enough ADH secretion by hypothalamus, or the
DI
posterior pituitary is unable to release it properly.

MEHLMANMEDICAL.COM 24
MEHLMANMEDICAL.COM

- Nephrogenic DI à insensitivity to ADH at the kidney (serum ADH is ­).


- Similar to central SIADH, central DI can be caused by head trauma, meningitis, and cancer.
- Nephrogenic DI is caused by lithium, demeclocycline, hypercalcemia, and NSAIDs.
- There is an NBME Q that asks about a patient’s response to ADH who is on chronic NSAIDs,
and the answer is “« Response to ADH” and “« urinary osmolality” (meaning no change).
- 2CK Q gives patient with primary hyperparathyroidism and ­­ serum calcium + nephrogenic
DI; Q asks cause of the DI à answer = hypercalcemia. High calcium can cause renal insensitivity
to vasopressin.

- Serum vs urinary values are the opposite of SIADH:


- ­ Serum sodium (>145 mEq/L), ­ serum osmolality, ­ serum specific gravity.
- ¯ urinary osmolality, ¯ serum specific gravity.

- PCT isotonic; JGA hypotonic; MCD hypotonic. à The PCT is always isotonic no matter what;
the JGA is always hypotonic no matter what; the MCD is clearly hypotonic in DI since we’re not
pulling free water out of the urine.
- When we are trying to first diagnose DI, the first thing we do is fluid restriction, same as with
SIADH. We want to see how serum/urinary values change first.
- After we determine that the urine is staying dilute + the serum is staying concentrated, the
next best step is desmopressin (analogue of vasopressin). If the urine gets more concentrated,
(i.e., if the drug works), we know central DI is the diagnosis and we’re merely deficient in ADH.
- If desmopressin doesn’t work, we know we have nephrogenic DI. I should point out that even
in nephrogenic DI, desmopressin might work but only very little, whereas with central DI,
administration will ­­ urinary osmolality robustly. It will be obvious on USMLE. But my point is,
don’t say, “Oh well desmopressin worked like 5% so we can’t have nephrogenic DI here.”
- Treatment for central DI is therefore desmopressin.
- Treatment for nephrogenic DI is NSAID + a thiazide. Sounds weird, but ¯ Na+ reabsorption
induced by thiazides in the early-DCT promote compensatory ­ Na+ reabsorption in the PCT,
where water follows Na+ and our net loss of fluid is less than without the thiazide. In healthy
individuals, however, they will lose more net fluid with the thiazide. The NSAID presumably ¯
renal blood flow, which will ¯ GFR and ¯ net fluid loss.
- There is difficult 2CK NBME Q where they say patient is on lithium + has ­ urinary output, and
fluid restriction is wrong answer to this question (I say hard because 9/10 times, fluid restriction
is correct when it’s listed); answer = NSAID + thiazide diuretic. The implication is, if it’s obvious
what the patient’s diagnosis is already nephrogenic DI, going straight to Tx is acceptable.
- Psychogenic polydipsia means the patient is simply drinking too much.
- Both the urine and serum will be dilute.
- Serum vs urinary values are the opposite of SIADH:
- ¯ Serum sodium (<135 mEq/L), ¯ serum osmolality, ¯ serum specific gravity.
- ¯ urinary osmolality, ¯ serum specific gravity.
PP
- I’d say 3/4 Qs on USMLE are obvious and will say some psych patient is drinking lots to “clear
himself from evil spirits,” etc.
- Probably 1/4 Qs won’t be an obvious psych vignette, but will just show you the lab values
where you have to say, “The urine and serum are both dilute, so this is psychogenic polydipsia.”
- First step in diagnosis is fluid restriction in order to see how urinary/serum values change.

Obscure miscellaneous conditions


- Disease that presents as though patient is on loop diuretic.
Bartter syndrome
- For whatever magical reason, is associated with juxtaglomerular cell hyperplasia.
Gitelman syndrome - Disease that presents s though patient is on thiazide diuretic.

MEHLMANMEDICAL.COM 25
MEHLMANMEDICAL.COM

Hyper-terse summary of drugs causing HY renal conditions


- Discussed all this stuff earlier. But students confuse this stuff a lot. So the summary is warranted.
- Aminoglycosides (gentamicin, tobramycin, amikacin).
Acute tubular necrosis
- Cisplatin.
Interstitial nephritis - NSAIDs, b-lactams, cephalosporins.
Membranous glomerulonephropathy - Dapsone, gold salts, sulfonamides.

Diuretics / Renal-related drugs

- Osmotic diuretic that ­ osmolality of urine, thereby retaining free water within
the PCT and descending limb of loop of Henle.
- Can be used as Tx for ­ intracranial pressure after patient has been intubated +
hyperventilated (¯ CO2 à ¯ cerebral perfusion).
- Avoid in heart failure (transient ­ in serum osmolality prior to renal excretion à ­
retention of free water within vascular space à transient ­ preload on heart).
- USMLE asks about mannitol’s effect on serum osmolality and ADH. This is a Q
repeated twice on the NBMEs. They will show a graph same as below and ask what
change will be expected with mannitol administration:

Mannitol

Answer = C; serum osmolality goes ­, which then causes ADH to go ­ (this is


because ADH normally ­ free water reabsorption in setting of ­ serum osmolality).
- Carbonic anhydrase inhibitor (enzyme that enables H2CO3 « HCO3- + H+) that acts
within the tubular cells of the PCT.
- Classic use is altitude sickness à causes ¯ reabsorption of HCO3- in the PCT à
Acetazolamide results in ­ urinary pH and volume + ¯ serum pH and volume. In altitude sickness,
patients hyperventilate (¯ CO2 à ­ pH) to take in more O2, but it takes a while a
minimum of 12-24 hours for the kidney to ¯ HCO3- reabsorption, so acetazolamide
forces the ¯ reabsorption by causing metabolic acidosis.
- Can cause Fanconi syndrome (¯ ability of PCT to reabsorb amino acids, glucose,
Expired tetracyclines
bicarb, and phosphate).

MEHLMANMEDICAL.COM 26
MEHLMANMEDICAL.COM

- Classic loop diuretic that inhibits the apical Na+/K+/2Cl- ATPase symporter on the
thick ascending limb. Apical means side of urine. Basolateral means side of blood.
- Most efficacious diuretic at enabling fluid unloading for pulmonary and peripheral
edema.
- Answer on USMLE for patient with heart failure who has ¯ O2 sats who “refuses
to lie supine/back on the gurney” because he/she can’t breathe. Q might say CXR or
exam shows fluid 2/3 up the lung fields.
- Also answer for first drug to ¯ peripheral edema.
- Application of loops can be confused by students with ACEi/ARB. For instance, the
latter are used first-line to ­ EF in heart failure even if O2 sats are low (¯ AT-II
constrictive effect on arterioles à ¯ afterload à heart pumps easier), but loops
are first-line if the vignette specifically emphasizes the dyspnea and low O2 sats of
Furosemide
the patient as the focus.
- Loops do not decrease mortality in cardiac patients. They are simply used for fluid
unloading for pulmonary and peripheral edema.
- Can cause ototoxicity (tinnitus, vertigo).
- HY point is that they can cause hypokalemia. What USMLE loves to do is tell you a
patient is initiated on furosemide but it is insufficient + now needs a second
diuretic à answer = anything that is potassium-sparing (i.e., ENaC inhibitor such as
amiloride or triamterene; or aldosterone receptor antagonist such as
spironolactone or eplerenone).
- Loops ­ urinary calcium. This is in contrast to thiazides, which ¯ it.
- Ethacrynic acid is another loop you can be aware of. It is used in patients with
sulfa allergies, since furosemide is a sulfa drug. It is also ototoxic.
- Hydrochlorothiazide (HCTZ) is HY example; chlorthalidone is “thiazide-like.”
- Inhibit the Na+/Cl- symporter on the apical membrane of the early-DCT.
- Thiazides and dihydropyridine calcium channel blockers are used first-line for HTN
in patients without any renal or cardiovascular issues. If patient has proteinuria, ­
in creatinine or renin, or pre-diabetes or diabetes, ACEi or ARBs are used first.
- Used to ¯ recurrence of calcium stones by promoting ­ reabsorption of urinary
calcium. In turn, they can sometimes cause hypercalcemia.
Thiazides - Can be used in heart failure in patients with diabetic nephropathy who are already
on ACEi and b-blocker. Sounds specific, but there’s a new 2CK Q that has thiazide as
correct over spironolactone in a diabetic. Spironolactone can cause hyperkalemia in
patients who have worrisome kidney function.
- Thiazides can cause gout (i.e., contraindicated in gout).
- Offline NBME 20 wants you to know thiazides can cause galactorrhea (milky
discharge from the nipples) via some obscure mechanism. Literature search shows
it accounts for ~0.2% of adverse effects of thiazides, but count on NBME to ask it.
- ENaC inhibitors à block apical sodium channel in cortical collecting duct à ¯ Na+
reabsorption à ¯ water reabsorption.
- These are potassium-sparing, which means they do not ¯ serum K+. This is
because by inhibiting the apical ENaC channel, they indirectly inhibit the basolateral
Na+/K+ ATPase à ¯ Na+ reabsorption + ¯ K+ secretion.
Amiloride, - The answer on USMLE for a second diuretic given in a patient already on
Triamterene furosemide who needs additional fluid unloading. Be careful however. I’ve seen
one Q where the patient is on furosemide, but the point of the Q is he/she needs
HTN control, and the answer is a thiazide, not the ENaC inhibitor. You want to
select an ENaC inhibitor specifically if the Q says, “We have a patient who has
peripheral/pulmonary edema due to X cause + is already on furosemide; what do
we do now?” à answer = amiloride or triamterene.
- Aldosterone receptor antagonists.
Spironolactone, - By blocking aldosterone receptor, it ¯ activity of the basolateral Na+/K+ ATPase à
Eplerenone leads to indirect ¯ activity of apical ENaC à ¯ sodium reabsorption à ¯ water
reabsorption.

MEHLMANMEDICAL.COM 27
MEHLMANMEDICAL.COM

- Potassium-sparing diuretic. Same as with ENaC inhibitors, it is used for fluid


unloading in patient who is already on furosemide in whom we worry about
dropping their K+ too much.
- Used in heart failure up the hierarchy of meds – i.e., ACEi/ARB first-line, followed
by adding b-blocker, followed by spironolactone (but if patient is diabetic with poor
renal function, don’t add spironolactone here; give thiazide instead as per new 2CK
NBME).
- Can be for aldosteronoma (Conn syndrome) prior to surgery.
- Can cause gynecomastia (anti-androgenic effect by blocking androgen receptors).
- Eplerenone has ¯ risk of gynecomastia compared to spironolactone, but USMLE
doesn’t give a fuck. Resource masturbation garbage that’s been regurgitated at
times over the years.
- ADH receptor antagonists at V2 receptor in the medullary collecting duct.
Tolvaptan,
- Can be used for SIADH.
Conivaptan
- Remember, however, that demeclocycline is also answer on 2CK NBME for SIADH.

- 27M + dilated renal pelvis and collecting system + ureters not dilated; Dx? à USMLE wants

“incomplete recanalization of proximal ureter.”

- Student says: “I can never remember which conditions are subepithelial deposits vs subendothelial.”

à Cool. Well you ready for something epic? à If the renal condition has “proliferative” in the name,

it has subendothelial deposits. If it doesn’t have “proliferative” in the name, it must not be

subendothelial. So for instance, membranoproliferative glomerulonephritis (MPGN) and diffuse

proliferative glomerulonephritis (DPGN); what would you see on biopsy? à USMLE answer =

subendothelial deposits. Yeah, I know. As I said, epic.

- Four classic findings you see in nephrotic syndrome?

o Proteinuria (usually high, i.e., >3g/24 hours)

o Hypoalbuminemia (because of the proteinuria)

o Peripheral edema (due to decreased intravascular oncotic pressure because of the

hypoalbuminemia)

o Hyperlipidemia (liver exports more apolipoproteins in an attempt to compensate for the low

serum albumin)

- Four classic findings you see in nephritic syndrome?

o Hematuria (due to glomerular inflammation allowing RBCs to escape into the urine)

o Oliguria (<400mL/24 hours in adults; due to defective glomerular filtration with diminished

GFR)

o Azotemia (means high nitrogen compounds in the blood à blood urea nitrogen [BUN])

MEHLMANMEDICAL.COM 28
MEHLMANMEDICAL.COM

o Hypertension (renal inflammation leads to microvascular disruption + upregulation of RAAS

à high angiotensin-II probably more responsible for HTN than aldosterone, as AT-II is potent

systemic vasoconstrictor, and patient need not have full-blown hyper-aldosteronism to have

HTN in nephritic syndrome, in addition to HTN being able to occur in acute renal failure).

- Is there proteinuria in nephritic syndrome? à almost always yes, but just typically not as much as in

nephrotic syndrome. The key differentiator between nephritic and nephrotic is the former has blood

in the urine.

- Important nephrotic syndromes for USMLE? à membranous glomerulonephritis/nephropathy;

minimal change disease (lipoid nephrosis); focal segmental glomerulosclerosis (FSGS); renal

amyloidosis; diabetic glomerulonephropathy.

- Important nephritic syndromes for USMLE? à IgA nephropathy (Berger disease); post-streptococcal

glomerulonephritis (PSGN);

- 12M + sore throat + blood in the urine 1-3 days later; Dx? à answer = IgA nephropathy, not post-

streptococcal glomerulonephritis (PSGN).

- 12M + sore throat + blood in the urine 1-2 weeks later; Dx? à PSGN. This distinction between IgA

nephropathy and PSGN is mega-HY.

- 12M + viral infection + red urine 2 days later; renal biopsy finding? à IgA deposition in the

mesangium (IgA nephropathy).

- 12M + GI infection (diarrhea) + red urine 2 days later; Dx? à IgA nephropathy.

- 3F + violaceous lesions on her buttocks/thighs + arthralgias + abdominal pain + red urine; renal Dx? à

IgA nephropathy. The tetrad of findings here is classic for Henoch-Schonlein purpura (HSP). The renal

component is IgA nephropathy.

- “Wait I’m confused. How does HSP relate to IgA nephropathy?” à don’t think of IgA nephropathy and

HSP as distinct / unrelated conditions; they’re on the same disease spectrum. While some patients

might just get IgA nephropathy from a viral or GI infection, others might get the immune complex

deposits in the skin (purpura), immune complex deposits in joints (reactive arthritis; type III

hypersensitivity), and mesenteric adenitis (abdo pain), leading to presentation of HSP. HSP is seen in

pediatrics, whereas IgA nephropathy can easily occur in adults.

MEHLMANMEDICAL.COM 29
MEHLMANMEDICAL.COM

- 12F + crusty yellow lesions around the mouth for past week + today has red urine; Dx? à PSGN

secondary to impetigo à give topical mupirocin to Tx the impetigo.

- Can you get PSGN from skin infections? à as per above, clearly à impetigo, cellulitis, and erysipelas,

if caused by Group A Strep, can cause PSGN, yes. This is HY for the USMLEs. However it should be

noted that rheumatic heart disease does not occur from cutaneous Strep infections; it occurs from

Strep pharyngitis only.

- 28M + blurry vision and/or hearing issue + red urine; Dx? à Alport syndrome.

- Inheritance pattern of Alport? à X-linked recessive (according to retired NBME 19 for Step 1; I point

this out because some sources, such as FA2020, say X-linked dominant, but the NBME says XR; the

NBME always wins).

- Mechanism of Alport? à mutation in type IV collagen (in contrast to Goodpasture, which is

antibodies against type IV collagen) à important in the lens of the eye + stereocilia within the

cochlea + basement membranes within the kidney à classically “ear and/or eye problem + red urine”

in a male. Question on retired NBME 19 for Step 1 gave the above 28M presentation, and the answer

to the Q was “X-linked recessive.”

- 13M + upper URTI + red urine after one week + decreased complement C3; Dx? à “acute

glomerulonephritis” à yes, PSGN – same thing – but on one of the 2CK NBME’s, they list the answer

as just simply “acute glomerulonephritis,” which for some reason throws people off.

- 13M + PSGN; renal biopsy shows what? à USMLE wants subepithelial deposits. “Lumpy bumpy

appearance” is perpetuated over the years in FA but never actually shows up in NBME material.

- When is membranoproliferative glomerulonephritis (MPGN) the answer? à if they say

subendothelial deposits (DPGN also subendothelial but is instead the answer in lupus), or dense

deposits (one type of MPGN is aka “dense deposit disease”), or you see “C3 nephritic factor”

mentioned, or “duplication of glomerular basement membrane.” Also the answer if the Q describes a

nephritic syndrome in someone with HepC.

- “Splitting of glomerular basement membrane”; Dx? à MPGN. “Tram-track appearance.”

- When is diffuse proliferative glomerulonephritis (DPGN) the answer? à SLE. They’ll also mention

“capillary looping.”

MEHLMANMEDICAL.COM 30
MEHLMANMEDICAL.COM

- When is rapidly progressive glomerulonephritis (RPGN) the answer? à acute deterioration in renal

function (high Cr, high BUN, oliguria) in someone who has a vasculitis (granulomatosis with

polyangiitis [formerly Wegener], eosinophilic granulomatosis with polyangiitis [formerly Churg-

Stauss], or microscopic polyangiitis) or Goodpasture syndrome.

- Biopsy shows what in RPGN? à fibrin crescents; parietal cell hyperplasia; leukocytic infiltrate à

should be noted that the crescents on biopsy are extremely HY on Steps 1+2CK; on Step 1, knowing

their composed of fibrin is high-yield.

- If the question is about Wegener, what will the renal biopsy often show? à “necrotizing”

glomerulonephritis.

- Antibodies in Wegener vs CS vs MP? à Wegener = c-ANCA; CS + MP = p-ANCA. It should be noted

that in real life, there is often overlap between the ANCAs, as well as times when patients may be

ANCA-negative, but on the USMLE, the classic associations stand correct.

- Which antibodies are present in the ANCAs? à c-ANCA = anti-proteinase 3 (anti-PR3); p-ANCA = anti-

myeloperoxidase (anti-MPO).

- Mechanism for Goodpasture? à antibodies against the alpha-3 chains of type IV collagen (anti-

glomerular basement membrane antibodies; anti-GBM) à an NBME Q asks for mechanism, and

answer is à “autoantibodies against host cell basement membranes.”

- Diagnosis of Goodpasture? à biopsy shows linear immunofluorescence.

- How does Goodpasture present? à 40s male with hemoptysis + hematuria.

- How does Wegener present? à hematuria + hemoptysis + “head”-itis (i.e., any type of head finding,

e.g., nasal septal perforation, mastoiditis, otitis, sinusitis).

- How does Churg-Strauss present? à asthma + eosinophilia; head findings rare.

- How does microscopic polyangiitis present? à just hematuria almost always.

- Wrist- or foot-drop in someone with W, C-S, or MP; Dx? à mononeuritis multiplex (palsy of one large

nerve in multiple locations à yes, weird description à means you can have a palsy of usually the

radial or common peroneal nerve à they are large nerves at different locations.

- 38F + hematuria + hemoptysis + anti-PR3; Dx? à Wegener.

- 38F + hematuria + nasal septal perforation + c-ANCA; Dx? à Wegener.

- 38F + hematuria + p-ANCA; Dx? à MP.

MEHLMANMEDICAL.COM 31
MEHLMANMEDICAL.COM

- 38F + hematuria + anti-MPO; Dx? à MP.

- 38F + asthma + eosinophilia + nasal septal perforation; Dx? à Churg-Strauss (yes, “head”-itis is more

rare, but the asthma + eosinophilia are overwhelmingly CS over Wegener; saw one question like this).

- 44M + hematuria + hemoptysis + no other findings; Dx? à Goodpasture most likely.

- 44M + hematuria + hemoptysis + renal biopsy shows necrotizing glomerulonephritis; Dx? à Wegener.

- 44M + hematuria + hemoptysis + linear immunofluorescence on renal biopsy; Dx? à Goodpasture.

- Alport syndrome vs Goodpasture key difference? à Alport = mutation in type IV collagen gene

(COL4A3/4/5); Goodpasture = antibodies against type IV collagen.

- 12M + periorbital edema + red urine + low C3; Dx? à PSGN.

- 12M + periorbital edema + no blood in the urine + no other information; Dx? à minimal change

disease (MCD; lipoid nephrosis).

- 12M + recent viral infection + periorbital/pedal edema + ascites; Dx? à MCD.

- 40M + Hodgkin lymphoma + renal condition (no blood in urine); Dx? à MCD à student says wtf? à

MCD is almost always children following viral infection, but it’s also seen in adults with Hodgkin. First

time you hear this it’s super-weird, but then you integrate the factoid and realize it’s not a big deal.

- Tx for MCD? à steroids (highly effective)

- Sickle-cell + nephrotic syndrome; Dx? à FSGS.

- HIV / IV drug use + nephrotic syndrome; Dx? à FSGS.

- Interferon use + nephrotic syndrome; Dx? à FSGS.

- What does FSGS mean? à focal vs global; segmental vs diffuse; focal means a part of the kidney;

diffuse means the whole kidney; segmental means part of a single nephron; global means an entire

single nephron. So in FSGS, parts of some nephrons are involved. This makes sense for sickle-cell, for

instance, where sickling will cause irregular microvascular pathology leading to non-uniform renal

involvement.

- Tx for FSGS? à steroids (not as effective as compared to MCD).

- Sulfa drug + kidney issue + no blood in urine; Dx? à membranous glomerulonephropathy (MG).

- Biopsy finding in MG? à subepithelial deposits; “spike and dome” appearance is prevalent in

resources / Qbank but buzzy and not on the NBMEs.

MEHLMANMEDICAL.COM 32
MEHLMANMEDICAL.COM

- HepB or C + nephrotic syndrome; Dx? à MG à usually due to HepB.

- Autoantibodies in membranous glomerulonephropathy? à Sometimes patients are positive for anti-

phospholipase A2 receptor antibodies.

- HepC + nephritic syndrome; Dx? à MPGN.

- Multiple myeloma + renal diagnosis? à renal amyloidosis à immunoglobulin light chains in high

levels moving through the kidney (Bence Jones proteinuria) leads to deposition in the renal

parenchyma. You should memorize the sentence: “Multiple myeloma is the most common cause of

renal amyloidosis.”

- First change in the kidney with diabetes? à hyperfiltration à increased filtered glucose pulls water

with it.

- First histologic change in the kidney with diabetes? à thickening of the glomerular basement

membrane à due to non-enzymatic glycosylation of basement membrane. If the question asks you

for the first renal change seen overall in diabetes, select hyperfiltration.

- USMLE Q mentions guy with diabetes + polyuria; asks why he has increased urinary output à answer

= “increased glomerular filtration rate.”

- Amount of glucose reabsorbed in PCT? à 100% physiologically; glycosuria not seen until serum levels

exceed around 180 mg/dL.

- Late finding seen histologically in diabetic nephropathy? à Kimmelstiel-Wilson nodules.

- What are KW nodules composed of? à hyaline à HY on Step 1 for some reason; don’t confuse this

fibrin, which is the answer for the crescents in RPGN.

- Example of type II diabetes drug that acts in the kidney? à Dapagliflozin (SGLT2 inhibitor in PCT;

prevents reabsorption of glucose).

- What is hyaline arteriolosclerosis? à hyaline deposition in the arterioles of the kidney usually seen in

diabetes à non-enzymatic glycosylation of vascular endothelium leads to leakage of plasma proteins

into vessel walls.

- First drug given to diabetics with HTN or proteinuria? à ACEi (e.g., enalapril) or ARB (e.g., valsartan).

- Most common cause of chronic renal failure? à diabetes mellitus.

- Comment on which endogenous mediators affect diameter of the renal afferent vs efferent arterioles

à prostaglandins dilate the afferent; angiotensin-II constricts the efferent.

MEHLMANMEDICAL.COM 33
MEHLMANMEDICAL.COM

- What do NSAIDs do to renal vasculature? à decrease dilation of the afferent arterioles (decreased

prostaglandin synthesis doesn’t cause constriction; we simply see attenuated degree of dilation).

- What do ACEi/ARBs do to renal vasculature? à decrease constriction of the efferent arterioles

(decreased AT-II synthesis/binding doesn’t cause dilation; we simply see attenuated degree of

constriction). USMLE will show you pic of glomerulus + ask where NSAID or ACEi acts, so just simply

be able to identify that.

- Student asks, “Why does AT-II act on the efferent arteriole? What’s the physiologic reason for that?”

à physiologically, AT-II increases in the setting of low blood volume à constriction of efferent

arteriole à increased filtration fraction (GFR/RBF) à GFR can be maintained in the setting of reduced

renal blood flow

- What do ACEi/ARB do to filtration fraction? à decrease it.

- Student asks, “If diabetes is most common cause of chronic renal failure, and renal failure is defined

as low GFR, why would we give them ACEi or ARB then first-line if those drugs decrease filtration

fraction?” à It’s paradoxical, yes. Although you’d be decreasing filtration fraction with one of these

drugs, you’d also be decreasing the rate at which glucose is being filtered across the basement

membrane, thereby slowing the non-enzymatic glycosylation of the BM + progression of disease.

Patients with end-stage renal disease are generally taken off ACEi/ARB, however early on these drugs

are a mainstay of treatment.

- BUN/Cr ratio in pre-, intra-, vs post-renal failure? à >20 in pre-; <20 if not pre- à FA for Step 1 had

stratified this out as <15 for intra- and 15-20 for post-, but I’ve seen at least three 2CK NBME/CMS Qs

where the diagnosis was acute tubular necrosis and the BUN/Cr was in the 16s or 17s. So I’ve learned

to just tell students: >20 = pre-; if <20, you simply know it’s not pre-.

- Fractional excretion of sodium (FeNa) in pre-/intra-/post-renal failure? à need to know it’s <1% in

pre-renal; >1% in intra-/post-renal à the low FeNa in pre-renal is due to the PCT’s attempt to

reabsorb Na (water follows sodium) in low-volume state (or in FMD, RAS).

- Urine osmolality in pre-/intra-/post-renal failure? à concentrated (high; >500 mOsm) in pre-; dilute

(low; <350 mOsm) in intra-/-post.

- When is pre-renal failure the answer apart from the BUN/Cr >20? à CHF classically (decreased renal

perfusion); can also be hypovolemia generally not in the acute setting; it’s to my observation that if

MEHLMANMEDICAL.COM 34
MEHLMANMEDICAL.COM

the NBME/CMS Q mentions acute hypovolemia + no other information (i.e., does not mention BUN or

Cr), the answer is acute tubular necrosis (intra-renal), not pre-renal. Pre-renal can also be the answer

for contrast-nephropathy; regarding this point, contrast agents can cause either pre-renal (due to

afferent arteriolar spasm) or intra-renal (direct nephrotoxicity); always give fluids to prevent (HY). The

USMLE Q will sometimes give a presentation of pre-renal + ask the etiology, and the answer is

“decreased glomerular filtration” (NBME).

- Guy has MI + has low BP; NBME Q asks what is most likely to be seen (ask a bunch of reabsorption /

secretion answers); answer = “increased potassium secretion” à low-volume status leads to RAAS

upregulation and distal renal K wasting.

- When is intra-renal failure the answer? à classically acute tubular necrosis (ATN) secondary to

episodes of hypoxia at the kidney, usually due to blood loss (i.e, intra-operative requiring lots of

blood, or traumatic exsanguination), or arrhythmia (i.e., patient had episode of VF and was

resuscitated) à vignette will mention one of the above scenarios and then tell you patient has acute

oliguria +/- dark urine. They do not have to mention BUN, Cr, or muddy brown granular casts for ATN.

- Other causes of intra-renal failure? à drugs (i.e., gentamicin); rhabdomyolysis (myoglobin is

nephrotoxic); contrast nephropathy.

- Why does acute hypoxia cause acute tubular necrosis? à proximal convoluted tubules have high

concentrations of transporters (namely Na-K-ATPases) with high oxygen demand; also explains why

diffuse cortical necrosis is classically associated with obstetric catastrophes.

- Lab animal is given 100% nitrogen in dumb experiment; most likely part of the kidney to experience

anoxic injury (they list everything) à answer = “proximal tubule.”

- Classic finding in urine with ATN? à muddy brown granular casts; it should be noted that general

“granular casts” are not specific to ATN. There’s an IM CMS Q with an elderly woman who has CVA

tenderness + granular casts on U/A; answer is pyelo not ATN.

- USMLE Q gives guy who has MI requiring resuscitation + subsequent oliguria; then they ask what you

see on microscopic examination of the kidney; answer = “degenerating epithelial cells and dirty

brown granular casts” or “necrosis of epithelial cells in proximal convoluted tubules.”

- Electrolyte disturbance in ATN? à first week is oliguric phase (hyperkalemia due to decreased

filtration); weeks 2-3 are polyuric phase (hypokalemia due to increased kaliuresis from PCT cells not

MEHLMANMEDICAL.COM 35
MEHLMANMEDICAL.COM

being able to reabsorb K yet) à btw, kaliuresis means urination of K (great word if you want to feel

sophisticated).

- When is post-renal the answer? à classically BPH or distal obstruction secondary to strictures or

malignancy.

- How does USMLE like to assess post-renal? à classically will give you BPH + show you a pic of

hydronephrosis (massively dilated kidney), then they’ll ask the most likely cause of this patient’s

condition; answer = “increased Bowman capsule hydrostatic pressure,” or “increased tubular

hydrostatic pressure.” This answer is on several NBMEs.

- BPH Tx? à alpha-1 blocker (e.g., tamsulosin, terazosin) and/or 5-alpha-reductase inhibitor (i.e.,

finasteride); patients may receive mono- or dual therapy.

- Tx for prostatic adenocarcinoma? à flutamide + leuprolide administered together (if the USMLE

forces you to choose a sequence, pick flutamide then leuprolide, but in practice their administered

together) à flutamide blocks androgen receptors; leuprolide is a GnRH receptor agonist (given

continuously it shuts off LH/FSH secretion).

- When are hyaline casts the answer? à classically dehydration on USMLE.

- When are waxy casts the answer? à end-stage renal disease (not HY, but student occasionally asks).

- When are RBC casts the answer? à glomerulonephritis.

- When are WBC casts the answer? à pyelonephritis classically.

- Wtf is Tamm-Horsfall protein? à most common protein found in urine; not found in plasma;

produced by tubular cells (classically thick ascending limb); no other real significance; more just the

name pops up in Qbank rarely and student will ask about it.

- What is interstitial nephropathy? à aka interstitial nephritis, or tubulointerstitial nephritis, where

there are WBCs (eosinophils) in the urine; patient may or may not present with a fine maculopapular

rash à almost always due to beta-lactams or cephalosporins, but can also be caused by NSAIDs

(would be considered a type of analgesic nephropathy, which usually presents as interstitial nephritis

or renal papillary necrosis, but can also be just generalized edema in someone on NSAIDs).

- 62M + 6 weeks nafcillin for MSSA endocarditis + WBCs on U/A; Dx? à interstitial nephropathy.

MEHLMANMEDICAL.COM 36
MEHLMANMEDICAL.COM

- 35F + uses ibuprofen daily for headaches + rash + eosinophils in urine; what do you see on biopsy? à

NBME answer = “inflammatory infiltrates in the interstitium” = interstitial nephropathy.

- What is analgesic nephropathy? à renal damage due to NSAIDs à can be renal papillary necrosis,

interstitial nephritis, or generalized edema from reduced renal blood flow. If renal papillary necrosis,

will present with dark urine from ischemia. If interstitial nephritis, will present with eosinophils in

urine as described previously (patient may not necessarily have a rash).

- 80F + taking high doses of naproxen for 6 weeks for osteoarthritis + peripheral edema; mechanism of

the edema? à USMLE has some sort of obsession with naproxen. OA should be treated with

acetaminophen before NSAIDs, but patients will often erroneously / unknowingly self-medicate with

high doses of NSAIDs to relieve pain (seen on FM shelves). Mechanism for edema à USMLE answer =

“increased renal retention of sodium,” or “decreased renal excretion of sodium.”

- Why would analgesic nephropathy cause increased renal retention of sodium? à NSAID knocks out

COX à decreased prostaglandin synthesis à decreased diameter of afferent arterioles à decreased

renal blood flow à PCT attempts to increase water reabsorption to compensate for perceived low

volume status à PCT accomplishes this by increasing Na reabsorption à water follows sodium à

fluid retention + edema. This mechanism is HY particularly on FM shelves.

- Patient with analgesic nephropathy; USMLE Q asks mechanism in terms of NSAID’s ability to inhibit

constriction/dilation at afferent vs efferent arterioles; answer = “vasodilating prostaglandins at

afferent arteriole” (i.e., NSAID inhibits this).

- When is renal papillary necrosis the answer? à classically in sickle cell; can also from drugs like

NSAIDs; urine will be dark à renal papillae in the medulla receive less blood flow, so small changes in

the microvascular supply can lead to sloughing + necrosis of the medulla. You’ll be able to contrast

this disorder from diffuse cortical necrosis and acute tubular necrosis because the latter two

conditions, in the context of ischemia, are associated with massive, acute events.

- First answer for Tx of all kidney stones on USMLE à hydration / fluids.

- Tx for calcium oxalate stones? à fluids, then alkalinize urine (citrate); prevent recurrence with

thiazide (decrease urinary Ca).

MEHLMANMEDICAL.COM 37
MEHLMANMEDICAL.COM

- Tx for calcium phosphate stones? à fluids; prevent recurrence with thiazide (decrease urinary Ca).

Calcium phosphate stones form at higher pH, so do not alkalinize; CaOxalate stones form at lower pH,

which is why citrate helps.

- Tx for struvite (ammonium magnesium phosphate) stones? à fluids; acidify urine with ammonium

chloride; surgery (because they’re often huge).

- Tx for uric acid stones? à fluids + alkalinize urine with citrate; treat the underlying gout; avoid

uricosurics such as probenecid indefinitely; avoid low-dose aspirin.

- Tx for cystine stones? à fluids; alkalinize urine with citrate.

- Who gets calcium oxalate stones? à classically those with intestinal malabsorptive disorders such as

Crohn, Celiac, CF à malabsorption of fats means increased intraintestinal binding of fat to calcium à

less calcium available to bind oxalate intraluminally à more oxalate is absorbed through intestinal

wall. CaOxalate stones can also be seen in ethylene glycol (AntiFreeze) poisoning and

hypervitaminosis C.

- Who gets struvite stones? à UTIs caused by urease (+) organisms. Proteus spp. is classic. Stones are

coffin lid-shaped.

- Who gets uric acid stones? à generally uric acid over-producers such as in Lesch-Nyhan syndrome or

tumor lysis syndrome; alcoholics (USMLE wants under-excretion as mechanism via which EtOH causes

gout, but EtOH also can increase production); stones are radiolucent on non-contrast CT.

- Who get cystine stones? à patients with cystinuria caused by inability to absorb COLA (cysteine,

ornithine, lysine, arginine) à two cysteines (-SH) joined via disulfide bond make one cystine (-S—S-).

- Any factoid to know about cystine stones? à AR disorder; generally young adults in USMLE vignette;

nitroprusside urine test yields a magenta color; hexagon-shaped stones.

- What do I need to know about renal tubular acidosis type I (RTA I)? à decreased ability of the DCT to

secrete protons (urine pH is >5.5) à leads to metabolic acidosis; patient does not have hyperkalemia.

Can be associated with bilateral renal parenchymal stones (i.e., not within the lumina but instead in

the actual meat of the kidney); can be seen in Sjogren syndrome.

- What do I need to know about renal tubular acidosis type II (RTA II)? à decreased ability of the PCT

to reabsorb bicarb (urine pH is >5.5) à associated with carbonic anhydrase inhibitors and Fanconi

syndrome (expired tetracyclines). Patient does not have hyperkalemia.

MEHLMANMEDICAL.COM 38
MEHLMANMEDICAL.COM

- What do I need to know about renal tubular acidosis type IV (RTA IV)? à either hyporeninemic

hypoaldosteronism or aldosterone insensitivity at the kidney (urine pH <5.5) à patient will present

with lab values resembling Addison disease, but the etiology will be chronic renal failure, not adrenal.

For instance, 68F diabetic with biochemistry resembling Addison = RTA IV, not Addison à patient will

have low serum sodium, high potassium, low bicarb (metabolic acidosis). Patient is hyperkalemic, in

contrast to RTA I and II.

- USMLE Q asks “decreased absorption” vs “no change” for Fanconi syndrome with respect to glucose,

amino acids, phosphate, bicarb à answer = decreased absorption for all four.

- Explain RAAS system. à reduced renal perfusion (low blood volume, renal artery stenosis,

fibromuscular dysplasia) à juxtaglomerular cells secrete renin à renin cleaves angiotensinogen

(produced by liver) in the plasma into angiotensin I à AT I goes to lung where it’s cleaved by ACE into

AT II à AT II causes vasoconstriction in systemic arterioles + at the efferent arterioles of the kidney; it

also goes to the zona glomerulosa in the adrenal cortex, where it upregulates aldosterone synthase,

thereby converting 11-deoxycorticosterone into aldosterone.

- Where does aldosterone act in the kidney? à cortical collecting duct; binds to intracytosolic receptor,

causing upregulation of basolateral Na/K-ATPase pumps (1, 2, 3 à for every 1 ATP utilized, 2 K

secreted into tubular cell from blood; 3 Na reabsorbed from tubular cell into blood) à this causes

decreased intra-tubular cell Na à creates a favorable high-low gradient from urine into the tubular

cell à ENaC is upregulated secondarily on apical membrane to allow Na to move from urine into

tubular cell.

- Another important point about renin secretion? à USMLE wants you to know that beta-1 activity

causes renin release. Sometimes you’ll get a Q where a patient has low blood volume and they’ll ask

which is true, and the answer is “increased renal sympathetic activity.”

- Where does vasopressin (ADH) act in the kidney? à medullary collecting duct à inserts aquaporins

on apical membrane à increased free water reabsorption.

- Urine + serum findings in SIADH à concentrated urine (high osmolality) + low serum sodium (<135

mEq/dL).

- Urine + serum findings in diabetes insipidus à dilute urine (low osmolality) + high serum sodium

(>145 mEq/dL).

MEHLMANMEDICAL.COM 39
MEHLMANMEDICAL.COM

- Which drugs cause nephrogenic DI? à lithium + demeclocycline.

- Urine + serum findings in psychogenic polydipsia (patient drinks too much) à dilute urine (low

osmolality) + low serum sodium (<135 mEq/dL).

- In DI, comment on the urine osmolality relative to plasma (i.e., hypotonic, isotonic, or hypertonic) for

PCT, JGA, medullary collecting duct à PCT is always isotonic regardless of renal condition; JGA is

always hypotonic regardless of renal condition (joins the early-DCT with the afferent arteriole);

medullary collecting duct hypotonic. In SIADH, the latter would merely be hypertonic.

- What is the JGA? à juxtaglomerular apparatus contains JGC that secrete renin.

- What is the macula densa? à modified smooth muscle cells of the DCT that sense changes in serum

sodium, thereby triggering the JGC to secrete renin; if volume status is low, PCT is attempting to

reabsorb more water to compensate; in order to do so, it reabsorbs more Na, where water follows

Na. This means that more distally, less sodium will still be left in the urine. The macula densa will

sense that as a signal that the PCT is reabsorbing it because the patient is fluid deficient à triggers

JGA to secrete renin.

- What do I need to know about mannitol? à osmotic diuretic that acts by retaining water within the

nephron lumina; USMLE likes proximal straight tubule (short segment after the PCT) as the answer if

it shows you a drawing and is very specific; if proximal straight tubule isn’t shown or labeled, choose

thin descending limb over PCT. Avoid in heart failure because administration transiently increases

intravascular volume à increased preload going to RA à increased myocardial oxygen demand.

Mannitol is used to treat increased intracranial pressure (pseudotumor cerebri) in some patients after

intubation + hyperventilation have been instituted (decreased CO2 à decreased cerebral perfusion).

- What do I need to know about acetazolamide and the kidney? à carbonic anhydrase inhibitor that

acts to prevent bicarb reabsorption à causes a transient metabolic acidosis that can be used to

counteract the respiratory alkalosis secondary to altitude sickness (kidney cannot physiologically

compensate fast enough so the drug forces the metabolic acidosis).

- What do I need to know about Loops? à Furosemide is standard first-line agent à does not decrease

mortality in heart failure à used to fluid-unload in patients with generalized edema or symptomatic

pulmonary edema à inhibits the Na/K/2Cl symporter on the apical membrane of the thick-ascending

limb à causes hypokalemia notably, but is also associated with contraction alkalosis (RAAS kicks up

MEHLMANMEDICAL.COM 40
MEHLMANMEDICAL.COM

and secretes protons distally when volume status is low because of the Loop); also causes increased

urinary calcium and magnesium; classically ototoxic, especially when combined with aminoglycosides

like gentamicin; furosemide is a sulfa drug, so patients with sulfa allergy can go on ethacrynic acid

instead (not a low-yield drug; memorize that). Bumetanide is an alternative to furosemide in some

patients. In general, be aware that loop diuretics are the most effective fluid-unloaders of all of the

diuretics.

- How are calcium + magnesium reabsorbed in the thick ascending limb? à paracellular (between the

cells).

- What is Bartter syndrome? à disease that presents as though the patient is indefinitely on loop

diuretic (associated with JGC hyperplasia).

- What do I need to know about thiazides? à hydrochlorothiazide (HCTZ) is standard drug à inhibits

the Na/Cl symporter on the apical membrane of the early-DCT à associated with decreased urinary

calcium and magnesium (in contrast to loops) à known to cause hyperGLUC (hyperglycemia, -

lipidemia, -uricemia, -calcemia) à avoid in diabetics + patients with Hx of gout; can be used to

prevent recurrent calcium nephro-/ureterolithiasis. Thiazides may be used to Tx HTN in patients who

do not have cardiovascular disease or diabetes (in these patient groups, use ACEi or ARB initially

instead). Chlorthalidone is an alternative to HCTZ.

- What is Gitelman syndrome? à disease that presents as though the patient is indefinitely on thiazide

diuretic.

- What do I need to know about amiloride and triamterene? à simply that they’re ENaC inhibitors and

are potassium-sparing (meaning they don’t cause kaliuresis).

- What do I need to know about spironolactone? à aldosterone receptor antagonist used in heart

failure after the patient is already on ACEi (or ARB) and a beta-blocker. Can also be used in patients

with Conn syndrome / primary hyperaldosteronism. USMLE loves asking spironolactone as the answer

in patients who are already on furosemide who need another diuretic (furosemide fluid-unloads the

best but causes potassium wasting, whereas spironolactone is potassium-sparing, so adding the latter

on top of furosemide can limit potassium losses). Also very HY to know that spironolactone causes

gynecomastia due to its anti-androgenic effects. Eplerenone is an alternative to spironolactone that

MEHLMANMEDICAL.COM 41
MEHLMANMEDICAL.COM

carries much lower risk of gynecomastia. These drugs decrease mortality in heart failure (HY; in

contrast to furosemide, which does not decrease mortality and is used for symptoms only).

- What do I need to know about tolvaptan/conivaptan? à vasopressin (ADH) receptor antagonists that

can be used in SIADH (e.g., due to small cell lung cancer).

- What are the three main effects of parathyroid hormone (PTH) at the kidney?

o Upregulates apical Ca2+ reabsorption in the late-DCT (increases Ca reabsorption).

o Decreases phosphate reabsorption in the PCT.

o Upregulates 1-alpha-hydroxylase in the PCT à increases conversion of inactive 25-OH-D3 to

active 1,25-(OH)2-D3 à active vitamin D3 then goes to small bowel to increase both Ca +

PO4 absorption; active D3 also goes to bone to increase mineralization of osteoid into

hydroxyapatite.

- What kind of PTH-related pathology is seen in renal disease? à secondary hyperparathyroidism is

caused by chronic renal failure à kidney cannot filter PO4 or reabsorb Ca à decreased serum Ca +

increased serum PO4 à increased PTH secretion.

- Comment on vitamin D in relation to renal disease? à patient will have decreased 1,25-(OH)2-D3 and

increased 24,25-(OH)2-D3 (the latter is an inactive storage form that is created by the shunting of 25-

OH-D3; USMLE asks about 24,25 sometimes). Student then asks, “I don’t get it though. If vitamin D

deficiency is associated with decreased Ca + decreased PO4, and there’s vitamin D deficiency in

chronic renal failure, why are the electrolytes in the CRF decreased Ca + increased PO4?” à answer

is: the renal disease always wins; yes, there’s decreased PO4 absorption in the small bowel, but

there’s also decreased filtration at the kidney, and the latter wins in terms of the arrows.

- What is renal osteodystrophy? à any bone problem caused by renal disease (i.e., due to PTH or

vitamin D-related effects).

- Patient with chronic renal disease has a pseudofracture of the hip; Dx? à osteomalacia à

“pseudofracture” is pathognomonic for vitamin D deficiency; in this case, the pseudofracture is just a

type of renal osteodystrophy that may be seen.

- Patient has renal failure + friction rub heard on auscultation of the chest; Dx + Tx? à uremic

pericarditis (a type of fibrinous pericarditis) à USMLE Tx is immediate hemodialysis.

MEHLMANMEDICAL.COM 42
MEHLMANMEDICAL.COM

- What is the biochemical / acid-base disturbance seen in renal disease? à sodium is variable;

potassium high; calcium low; phosphate high; bicarb low; pH low; CO2 low (compensation).

- What do I need to know about EPO and the kidney? à merely that the kidney secretes it in response

to hypoxemia à high EPO in lung disease (causes secondary polycythemia); low EPO in polycythemia

vera (primary bone marrow over-production of various cell lines due to JAK2 mutation; EPO is

suppressed because RBCs are high). EPO is also often elevated in renal cell carcinoma (RCC).

- Smoker + hypercalcemia + polycythemia; Dx? à RCC (paraneoplastic EPO + PTHrp).

- Smoker + flank pain + red urine; Dx? à RCC.

- Most common type of RCC? à clear cell carcinoma (histo shows large clear cells on Step 1).

- Notable disease associated with RCC? à Von-Hippel Lindau (VHL) à RCC can be bilateral. VHL is AD,

chromosome 3, and also associated with retinal + cerebellar hemangioblastomas; caused by

constitutive activation of hypoxia-inducible factor.

- Renal cancer in children? à Wilms tumor.

- 3M + painless flank mass + no other info; Dx? à Wilms tumor.

- 6M + painless flank mass + seizure + MRI of brain shows periventricular nodules; Dx? à Tuberous

sclerosis with renal angiomyolipoma.

- Anything else about Wilms tumor? à increased incidence in horseshoe kidney in Turner syndrome.

- Horseshoe kidney key point? à not only increased risk of Wilms tumor, but the kidney gets caught

under the IMA. USMLE Step 1 likes this factoid for some dumb reason.

- What is WAGR complex? à Wilms tumor, Aniridia, Genitourinary malformation, Retardation; caused

by WT1 gene mutation.

- What is Denys-Drash syndrome? à Wilms tumor + male pseudohermaphroditism (secondary sex

characteristics are female; primary sex characteristics are male); caused by WT1 gene mutation.

- What is Beckwith-Wiedemann syndrome? à Wilms tumor + neonatal hypoglycemia + macrosomia /

macroglossia + hemihypertrophy +/- omphalocele; caused by WT2 gene mutation.

- Comment on AR vs AD polycystic kidney disease. à ARPKD is chromosome 6 and a pediatric

condition; it’s associated with hepatic fibrosis. ADPKD is chromosome 16 and presents in 30s or 40s;

associated with HTN and circle of Willis saccular (berry) aneurysms.

- Most common extra-renal location for cysts in ADPKD? à liver.

MEHLMANMEDICAL.COM 43
MEHLMANMEDICAL.COM

- What’s more important in management of ADPKD, serial BP checks or MRI angiogram circle of Willis?

à answer = serial BP checks à renal cysts compress microvasculature, causing surges in RAAS.

- Comment on systemic sclerosis and the kidney? à systemic sclerosis (diffuse type) can cause renal

crises with malignant HTN.

- 68M + diabetic + Hx of intermittent claudication + CABG + gradually increasing BP past two years; Dx?

à renal artery stenosis.

- Cause of renal artery stenosis? à atherosclerosis. They will always give RAS in a patient who has

cardiovascular disease.

- 23F + high aldosterone + high renin + high BP; Dx? à fibromuscular dysplasia à this is not renal

artery stenosis; if you say RAS, that means atherosclerosis. FMD is a tunica media proliferative

pathology resulting in RAS-like presentation; it’s seen classically in young women, whereas RAS will be

patients with CVD.

- How to Dx FMD? à MR angiography (HY on the USMLE) à NBME has actually said the buzz /

colloquial phrase “string of beads” appearance seen on MR angiography.

- Patient with high BP is given an ACEi or ARB and gets a spike in either renin or creatinine; Dx? à FMD

if young patient; RAS if patient with CVD. USMLE loves this point and it shows up on Steps 1 and 2CK.

If you have FMD or RAS, renal blood flow is already reduced, so the kidney has a reduced capacity to

remain within physiologic range for GFR à giving an ACEi or ARB will reduce filtration fraction and

lead to a clinically significant reduction in GFR à renin + Cr go up.

- Hx of many pregnancies + downward movement of vesicourethral junction à stress incontinence.

- Tx of stress incontinence à pelvic floor exercises (Kegel); if insufficient à mid-urethral sling.

- Hyperactive detrusor or detrusor instability à urge incontinence.

- Need to run to bathroom when sticking key in a door à urge incontinence.

- Incontinence in multiple sclerosis patient or perimenopausal à urge incontinence.

- Tx of urge incontinence à oxybutynin (muscarinic cholinergic antagonist) or mirabegron (beta-3

agonist).

- Incontinence + high post-void volume (usually 3-400 in question; normal is <50 mL) à overflow

incontinence.

MEHLMANMEDICAL.COM 44
MEHLMANMEDICAL.COM

- Incontinence in diabetes à overflow incontinence due to neurogenic bladder.

- Tx for overflow incontinence in diabetes à bethanecol (muscarinic cholinergic agonist).

- Incontinence in BPH à overflow incontinence due to outlet obstruction à eventual neurogenic

bladder.

- Tx for overflow incontinence in BPH à insert catheter first; then give alpha-1 blocker of 5-alpha-

reductase inhibitor; then TURP if necessary.

- Innervation of external urethral sphincter à pudendal nerve (somatic; voluntary).

- Innervation of internal urethral sphincter à hypogastric nerves (sympathetic).

- Innervation of detrusor muscle à pelvic splanchnic nerves (parasympathetic; S2-4).

- What is Brenner tumor? à ovarian tumor with bladder (transitional cell) epithelium.

- Exquisitely tender prostate on digital rectal exam à prostatitis.

- Prostatitis Tx à ciprofloxacin (fluoroquinolone à DNA gyrase / topoisomerase II inhibitor).

- Costovertebral angle tenderness + fever à pyelonephritis.

- Costovertebral angle tenderness + granular casts à pyelonephritis (correct, super-weird; NOT acute

tubular necrosis; this is on 2CK NBME).

- Tx for pyelonephritis à ciprofloxacin, OR ampicillin + gentamicin.

- Is chronic pyelonephritis ever an answer? à usually young patient, i.e., 3-4-year-old, who has

recurrent bouts of acute pyelo à need to know recurrent acute pyelo is what causes chronic pyelo.

For Step 1 level, they will show an image of a small, scarred kidney and give you the above

description, then the answer is simply “vesicoureteral reflux” as the mechanism. They might also ask

what you see on biopsy; answer = “tubular atrophy.” Ultrasound shows “broad scars with blunted

calyces.” The phrase “thyroidization of the kidney” is buzzy and more Qbank, not NBME. You need to

walk away knowing that chronic pyelo will produce scarred renal calyces in someone who’s had

recurrent acute pyelo.

- Tx for simple UTI à nitrofurantoin is classic answer (need not be cystitis in pregnant women; this is

listed as the correct answer in many 2CK-level NBME/CMS Qs); TMP/SMX is also classic combo.

- Q asks best initial Mx to prevent UTIs à answer = postcoital voiding. If unsuccessful, NBME wants

“postcoital nitrofurantoin prophylaxis” next; if not listed, choose “daily TMP/SMX prophylaxis.” The

MEHLMANMEDICAL.COM 45
MEHLMANMEDICAL.COM

latter sounds incredibly wrong, and I agree, sounds outrageous, but it’s correct on one of the obgyn

CMS forms and everyone gets it wrong.

- 22F + Sx of dysuria for 6 months + anterior vaginal wall pain + U/A completely normal + afebrile; Dx?

à chronic interstitial cystitis à answer on one of the obgyn forms. Must have at least 6 weeks of UTI-

like Sx without any identifiable pathology.

- Tx for asymptomatic bacteriuria à if pregnant, must Tx. If not pregnant, do not Tx. Exceedingly HY for

2CK.

- Why must Tx asymptomatic bacteriuria in pregnancy? à progesterone slows ureteric peristalsis,

increasing the risk of pyelo.

- 82F falls down stairs; urine dipstick shows 2+ blood and 3-4 RBCs/hpf on LM; Dx? à rhabdomyolysis

à need to know that false (+) blood on urine dipstick is rhabdomyolysis (dipstick cannot differentiate

myoglobinuria from actual RBCs).

- 22F has Sx of UTI + U/A completely normal; next best step? à test for chlamydia/gonorrhea (answer

on one of the newer obgyn forms).

- 25M + mucopurulent urethral discharge + culture grows nothing; Dx? à chlamydia (obligate

intracellular).

- Tx for chlamydia and gonorrhea on the Step à always cotreat with IM ceftriaxone PLUS either oral

azithromycin or oral doxycycline. Since chlamydia will not be picked up on culture, must always treat

for it. Some debate that if gonorrhea is present, then you’d see the gram (-) diplococci and therefore

don’t need to give ceftriaxone if not detected, but two points: 1) cultures for gonococcus are

notoriously insensitive (often need to do urethral, anal, and throat cultures to maximize chance of

picking it up), and 2) all NBME/CMS Qs always have cotreatment as the answer. In the rare even that

ceftriaxone isn’t listed as an answer choice, just choose the combo where one of the drugs is the

cephalosporin.

- Most common type of bladder cancer? à transitional cell carcinoma.

- Most specific cause of bladder cancer? à industrial (aniline) dyes.

- Most common cause of bladder cancer? à smoking.

- Type of bladder cancer seen with Schistosoma haematobium infection à squamous cell carcinoma.

MEHLMANMEDICAL.COM 46
MEHLMANMEDICAL.COM

- What is Potter sequence? à renal agenesis leading to oligohydramnios à causes pulmonary

hypoplasia in fetus.

- Most common genitourinary (GU) abnormality in neonatal males? à posterior urethral valves (PUV)

à thin membrane(s) in the urethra preventing the outflow of urine).

- Male neonate + suprapubic mass + no passage of urine; Dx? à PUV.

- Neonatal male with PUV; next best step in Dx? à voiding cystourethrogram. Tx must do surgery.

- 34M falls on balance beam + saddle injury + blood at urethral meatus; Dx + next best step in Mx? à

urethral injury; do retrograde urethrogram.

- 16M + skiing accident + bruising and/or pain over a flank + red urine; Dx + next best step in Mx? à

kidney injury; must do CT with contrast, not ultrasound. This is exceedingly HY on surgery forms and

sounds wrong (i.e., “why would you do CT with contrast in someone with suspected renal injury if

contrast can cause nephrotoxicity?”) à great question, but CMS/NBME/USMLE still want CT with

contrast, so don’t pick ultrasound.

- 16M + skiing accident + bruising and/or pain over a flank + no gross blood of urine (i.e, urine not red);

next best step in Mx? à urinalysis to look for blood (super HY) à if urinalysis shows blood, do CT

scan to check for renal injury. If U/A negative, renal injury unlikely, so choose “no further

investigations indicated,” not CT.

- 16M + skiing accident + bruising and/or pain over a flank + urinalysis is normal; next best step? à no

further investigations indicated.

- USMLE Q gives you creatinine of 1.0 mg/dL in 65M, then tells you 10 years later his Cr is 1.2 mg/dL;

Dx? à normal aging à creatinine clearance naturally decreases with age. There is no definitive cutoff

for USMLE Qs, but the student should be aware that Cr of 1.5 or greater is always pathologic, and at

Cr of 2.0, >90% of nephron function has already been lost.

- USMLE Q says oophorectomy is being performed + ureter is damaged; during ligation of which vessel

did the damage to the ureter occur? à answer = ovarian artery.

- What is water under the bridge? à ureter travels under the uterine artery in women and vas

deferens in men.

- Kid + bloody diarrhea + petechiae + red urine; Dx? à hemolytic uremic syndrome (HUS) caused by

EHEC O157:H7 or Shigella à HUS = triad of thrombocytopenia + schistocytosis (microangiopathic

MEHLMANMEDICAL.COM 47
MEHLMANMEDICAL.COM

hemolytic anemia) + renal insufficiency; toxin will inhibit ADAMTS13 in afferent arterioles + cause

endothelial damage à platelet consumption + clumping (thrombocytopenia) à RBCs shear as they

pass by the platelet clumps (schistocytosis).

- How does HUS contrast with TTP? à TTP is caused by a mutation that results in defective ADAMTS13,

or antibodies against ADAMTS13, resulting in the inability to cleave vWF multimers à platelet

clumping à similar progression as HUS. One of the points of contrast is that TTP is not toxin-induced,

and TTP also tends to be a pentad of the HUS findings + fever + neurologic signs.

- What is hepatorenal syndrome? à liver failure causing renal failure; rare Dx but the correct answer

USMLE wants is “normal renal biopsy.” However recent literature has suggested there is a

phenomenon called “glomerular tubular reflux” seen in this condition.

- Endocarditis following a genitourinary procedure (e.g., TURP) or catheterization; organism? à USMLE

wants enterococcus.

MEHLMANMEDICAL.COM 48
MEHLMANMEDICAL.COM

YouTube
@mehlmanmedical

Instagram
@mehlman_medical

MEHLMANMEDICAL.COM 49
MEHLMANMEDICAL.COM

MEHLMANMEDICAL
HY RENAL

All material is copyrighted and the property of mehlmanmedical.

Copyright © mehlmanmedical

MEHLMANMEDICAL.COM 50
MEHLMANMEDICAL
HY PEDIATRICS
MEHLMANMEDICAL.COM

YouTube
@mehlmanmedical

Instagram
@mehlman_medical

MEHLMANMEDICAL.COM 2
MEHLMANMEDICAL.COM

HY Pediatrics

Purpose of this review is not to be a superfluous 750-page pediatrics textbook; the purpose is to increase your USMLE and

Peds shelf scores via concise factoid consolidation.

- 5F + 1-year Hx of chronic nonproductive cough + all asthma Txs not effective + “CXR shows no

abnormalities except for a linear consolidation in the right middle lobe”; Dx? à Peds NBME =

bronchiectasis à students says wtf? (because normally bronchiectasis is “cups and cups” of foul-

smelling sputum in CF, TB, or COPD) à Dx is called “right middle lobe syndrome” (Google it) and can

present as bronchiectasis in peds characterized by nonproductive cough and linear consolidation on

CXR. You don’t have to agree that it’s HY, but that doesn’t change the fact that it’s on the Peds NBME.

- 7F + seizure-like episode + during episode, ECG shows P waves at 80bpm but not QRS complexes +

normal sinus rhythm resumes after 20 seconds + patient fully alert after one minute; Dx? à answer

on Peds NBME = Adam-Stokes attack à not true seizure disorder as per EEG; arrythmia leads to

hypoxia of brainstem à seizure-like fits ensue.

- 8M + low-pitched vibratory murmur heard throughout cardiac cycle + loudest at left upper sternal

border when sitting + murmur disappears when supine and neck rotated; Dx? à answer on Peds

NBME = venous hum à benign/innocent pediatric murmur that will disappear as child grows; caused

by turbulent venous return in SVC and jugular veins.

- 2M + fever 101F + diffuse crackles across both lungs + CXR shows diffuse interstitial infiltrates +

ongoing Hx of thrush, failure to thrive, and frequent diarrhea + mother used drugs during pregnancy;

what’s the next best step in diagnosis? à answer = “silver stain of bronchoalveolar fluid”; this is

pneumocystis jirovecii pneumonia (PJP) in child with AIDS secondary to maternal drug use

(presumably IV); NBME will ask any number of Qs on this, but important take-home point is

bronchoalveolar lavage is done after the CXR; PJP is also notably bilateral diffuse infiltrates with a

ground-glass appearance; if NBME gives you lobar presentation, it’s not PJP (usually S. pneumo).

- 6-month-old boy + runny nose for 2 days + respiratory stridor + temp 101F; what is the most likely

location of the pathology? à answer = larynx; Dx is laryngotracheobronchitis (croup) caused by

parainfluenza virus; classically “seal-like, barking” cough in school-age child; Tx = supportive, or

nebulized racemic epinephrine if severe.

MEHLMANMEDICAL.COM 3
MEHLMANMEDICAL.COM

- 10-month-old + fever 101F + bilateral wheezes; Dx? à answer = “community-acquired viral infection”

à RSV bronchiolitis; Tx = supportive (ribavirin and palivizumab are almost always wrong).

- 8M + from immigrant family + sitting forward in tripod position + drooling; next best step? à answer

= intubation; Dx = epiglottitis à Haemophilus influenzae type B (patient not vaccinated); if Q asks for

antibiotic, give ceftriaxone; for close contacts, give rifampin.

- 8M + recently convalesced from URTI + now has stridor + fever 101F; Dx? à answer = bacterial

tracheitis à usually S. aureus; Dx w/ bronchoscopy; Tx with third-gen cephalosporin + aminopenicillin

(e.g., IV flucloxacillin).

- 8-month-old girl + stridor that improves with neck extension; Dx? à answer = vascular ring à weird

but HY diagnosis for peds à aberrant embryologic development where the aorta and/or surrounding

vessels form a ring around the esophagus and/or trachea.

- 8-month-old girl + stridor that improves when prone or upright; Dx? à laryngomalacia à most

common cause of stridor in peds à soft cartilage of upper larynx collapses during inhalation.

- 3F + 2-wk Hx of cough and nasal congestion + snoring loudly past 6 months + P/E shows she breathes

predominantly through her mouth + 1/6 holosystolic murmur and loud S2 + CXR shows cardiomegaly

+ increased pulmonary vascular markings + echo shows RV hypertrophy and mild tricuspid regurg;

what is the most appropriate long-term Mx for this patient? à answer = adenoidectomy and

tonsillectomy à sounds weird, but HY for Peds shelf à can cause obstructive lung disease with cor

pulmonale à loud S2 and increased pulmonary vascular markings suggest pulmonary HTN; tricuspid

regurg can be seen sometimes in pulmonary HTN.

- 2F + stridor + laryngoscopy shows small growths of larynx; Dx? à answer = HPV 6/11 à laryngeal

papillomatosis.

- 9M + grey pseudomembrane seen in oropharynx; Tx? à answer = diphtheria antitoxin.

- 12M + steatorrhea + recurrent URTIs + new-onset stridor; why the new-onset stridor? à answer =

nasal polyps (common in cystic fibrosis); steatorrhea due to exocrine pancreas insufficiency.

- 7F + painless midline neck lump that moves upward when swallowing; Q asks what would be seen on

biopsy of the lesions à answer = thyroid tissue; Dx = thyroglossal duct cyst.

- 7F + painless lateral neck mass over sternocleidomastoid; Dx? à answer = brachial cleft cyst.

MEHLMANMEDICAL.COM 4
MEHLMANMEDICAL.COM

- Neonate born by forceps delivery + crooked neck + 1.5cm mass palpated in left side of neck; Dx? à

answer = “sternomastoid injury.”

- 8F + hypothyroidism + dysphagia; Dx? à answer = lingual thyroid (ectopic thyroid location); 4-7x

more common in girls; associated with hypothyroidism.

- 1F + Hct 26% + first six months of life on commercial formula + past 6 months of 2% cows milk and

table food + smear shows pale RBCs; Dx? à answer = iron deficiency anemia à infants (especially

age < 1 year) fed more than 24 ounces of cows milk daily at risk for iron deficiency anemia.

- 5F + pale conjunctivae + low Hb + low MCV + WBCs and platelets normal + low red cell distribution

width (RDW); Dx? à answer = “decreased synthesis of globin chains”; thalassemia has low RDW,

whereas iron deficiency anemia has high RDW (thalassemia à defective Hb leads to RBCs that are

uniformly small; IDA creates non-uniformity of RBC size where some can escape marrow at normal

size à high RDW); target cells are HY finding on smear (giveaway); next best step = hemoglobin

electrophoresis.

- 4M + itchy head + small oval areas of alopecia on scalp; most likely causal organism? à answer =

Trichophyton tonsurans à Dx is tinea capitis (cradle cap) à Tx with oral griseofulvin for patient only;

reduce risk by not sharing hats (three Qs on the peds forms right there).

- 17M + lives in Florida + hypopigmented areas on shoulders and torso; Tx? à answer = topical

selenium; Dx is tinea versicolor (Malassezia furfur).

- 7M + one-week Hx of low-grade fever and fatigue + 3-day Hx of rash and swelling starting at ankles

and spreading upward + ankles tender + exam shows “several palpable petechiae and confluent

purpuric areas over lower extremities”; Dx? à answer = Henoch-Schonlein purpura à can present

with tetrad of 1) palpable purpura (and apparently petechiae), 2) IgA nephropathy (red urine), 3)

abdominal pain (mesenteric adenitis), and 4) arthralgias; classically following viral infection, however

2CK vignettes will often omit mentioning the infection altogether; self-limiting.

- 4-month-old girl + asplenia + dextrocardia; which antibiotic should be used for prophylaxis? à

answer = penicillin (most important for Strep pneumo due to asplenia); Dx is Ivemark syndrome

(dextrocardia/situs inversus + hypoplasia of various organ systems).

MEHLMANMEDICAL.COM 5
MEHLMANMEDICAL.COM

- 3-month-old female + sepsis + Q asks for which antibiotic to give; answer = cefotaxime à give third-

generation cephalosporin for sepsis; for young children (generally <6 years), use cefotaxime over

ceftriaxone. Q on 2CK NBME 8 has 6M with sepsis and answer is ceftriaxone (cefotaxime isn’t listed).

- 12-month-old female + sickle cell + missed dose of penicillin prophylaxis + now has sepsis; which Abx

to give? à answer = cefotaxime; wrong answers are penicillin and ceftriaxone.

- Neonate with APGARs good at birth + 3-10 days later has BP 60/35 in upper extremities and

unobtainable in lower extremities + O2 sats 98 mmHg but nails appear dusky + 3/6 holosystolic

murmur at left sternal border; Q asks what’s responsible for current presentation? à answer =

“closure of ductus arteriosus” à Dx = preductal coarctation (the type seen in neonates); CoA can be

associated with tricuspid regurgitation and other cardiac abnormalities. Dusky nailbeds don’t equate

to cyanosis if O2 sats are normal.

- 7-day-old neonate + not cyanotic + 4/6 holosystolic murmur at left sternal border + murmur was not

present (or soft) at birth; what best explains the current presentation? à answer = “decreased

pulmonary vascular resistance”; this is a VSD with L à R shunt; pulmonary vessels have gradually

opened up over first week of life as ductus arteriosus closes; the decrease in pulmonary vascular

resistance causes ¯ right-heart pressure à enables a favorable L à R pressure gradient for the VSD

murmur to become more salient; lack of cyanosis means there is no R à L shunt + Eisenmenger

wouldn’t occur till later in life anyway.

- 5-day-old neonate + not cyanotic + 3/6 holosystolic murmur at left sternal border + murmur was not

present (or soft) at birth; what best explains the child’s presentation at birth? à answer = “increased

pulmonary vascular resistance”; lungs still hadn’t opened up so VSD L à R pressure gradient was not

yet conducive to salient murmur; Peds shelf asks both versions of this question (i.e., increased vs

decreased pulmonary vascular resistance as answers).

- 14F + Down syndrome + polycythemia + moderate cyanosis and digital clubbing + no murmur on

cardio exam but loud S2 + echo shows large VSD and dilated main pulmonary artery; mechanism for

polycythemia? à answer on Peds NBME = pulmonary artery hypertension”; Dx is Eisenmenger (large

VSD with reversal R à L); large VSDs may present without murmur; pulmonary vessels constrict to

compensate for high preload from previous L à R VSD shunt; constriction leads to hypoxia and

MEHLMANMEDICAL.COM 6
MEHLMANMEDICAL.COM

secondary polycythemia with high EPO; should also be noted that Down syndrome is associated with

endocardial cushion defects (AVSD > VSD > ASD; Step 1).

- Neonate + pulmonary valvular stenosis; Dx? à answer = Noonan syndrome; second most common

syndrome associated with congenital heart disease after Down.

- Neonate <34 weeks gestation + dyspnea; Dx? à neonatal respiratory distress syndrome (NRDS;

hyaline membrane disease); healthy lecithin (dipalmitoyl phosphatidylcholine) to sphingomyelin ratio

is >2; ratio is reduced in NRDS.

- Neonate born at 39 weeks gestation via C-section + RR of 70 (normal 40-60) + CXR shows bilateral

mild hyperinflation and prominent perihilar interstitial markings ; Dx? à answer = transient

tachypnea of the newborn à TTN is the answer when the vignette “sounds like NRDS but the kid is

term”; seen in C-sections and fast vaginal deliveries in term neonates; mechanism is delayed

absorption of the fetal lung fluid by pulmonary lymphatics; Tx is supportive.

- Neonate born vaginally at 43 weeks gestation + cyanotic + pO2 is 26mmHg on 100% oxygen + CXR

normal + echo shows normal cardiac anatomy with a right-to-left shunt across the foramen ovale;

what is the mechanism for the child’s condition? à answer = “failure of pulmonary vasodilation” à

Dx = persistent fetal circulation (aka persistent pulmonary hypertension of the newborn).

- Neonate born at 26 weeks + required oxygen in ICU for several weeks + is now on home oxygen; child

is at increased risk for what? à answers = bronchopulmonary dysplasia, retinopathy of prematurity,

germinal matrix bleed à all can be seen in neonates on oxygen therapy.

- 6-hour-old newborn born at term + cyanosis of upper and lower extremities but no circumoral

cyanosis + O2 sats normal + not in acute distress; next best step? à answer = “placement of the

newborn under warming lights” à Dx = acrocyanosis à benign peripheral cyanosis that improves

with warming; lack of circumoral cyanosis means no central cyanosis (i.e., heart/lungs OK).

- Newborn with slightly blue upper and lower extremities + placed under warming lights; next best

step? à answer = “tactile stimulation and oxygen therapy.” So for acrocyanosis: place under warming

lights then do tactile stimulation + oxygen therapy.

- Three ways VSD sounds on Peds shelf?

o Holosystolic murmur at left sternal border PLUS either parasternal heave or palpable thrill.

MEHLMANMEDICAL.COM 7
MEHLMANMEDICAL.COM

o Holosystolic murmur at left sternal border PLUS left atrial enlargement (L à R shunt leads to

increased preload back to LA in addition to the systemic preload).

o Holosystolic murmur at left sternal border PLUS diastolic rumble (volume overload of LA).

- 2-month-old boy + not cyanotic + to-and-fro murmur; Dx? à answer = “extracardiac left-to-right

shunt”; Dx = PDA.

- What are the four ways PDA sounds on Peds shelf?

o Continuous, machinery-like murmur (easy; the one everyone knows).

o Pan-systolic-pan-diastolic (if it’s continuous, then it’s present throughout cardiac cycle).

o To-and-fro murmur (student says wtf? à HY; also on NBME 6 for 2CK, where the Q rides on

you specifically knowing the description).

o Bounding pulses (normally refers to aortic regurg, but also a rare descriptor for PDA à

blood leaves aorta quickly L à R); in this scenario, the Q will say there’s a continuous

murmur with bounding pulses; in contrast, AR is a decrescendo holodiastolic murmur.

- 2-month-old boy + fixed splitting of S2; Dx? à answer = patent foramen ovale (ASD).

- Heart problem in neonate of mom with SLE? à congenital third-degree heart-block.

- 3M + elfin-like facies + hypercalcemia + well-developed verbal skills; Dx? à answer = William

syndrome.

- Heart problem in William syndrome? à supravalvular aortic stenosis.

- Bicuspid aortic valve; most likely to cause which murmur? à aortic stenosis.

- Mid-systolic (crescendo-decrescendo) murmur + gets worse with Valsalva? à HOCM.

- Mid-systolic (crescendo-decrescendo) murmur + no change or softens with Valsalva? à aortic

stenosis.

- Myxomatous degeneration of mitral valve? à mitral valve prolapse (MVP).

- Mid-systolic click? à MVP.

- Marfan or Ehlers-Danlos syndrome; heart murmur(s)? à MVP or aortic regurg.

- Late-peaking systolic murmur with ejection click? à another way they describe aortic stenosis.

- 2-week-old girl + HR of 240 + QRS duration of 50ms + no P waves + awake and alert + ice pack to the

face does not change the HR; next best step? à answer = adenosine; Dx is paroxysmal

MEHLMANMEDICAL.COM 8
MEHLMANMEDICAL.COM

supraventricular tachycardia (PSVT); complexes are needle-shaped (short QRS); in contrast,

ventricular tachycardia has wide complexes (>120ms).

- 10-month-old girl + sore throat for 2 days + fever 101F + S3 gallop heard on auscultation; what best

explains these findings? à answer = myocarditis (rheumatic fever); should be noted that PSGN will

take 1-2 weeks to occur after Group A Strep infection, but NBME has RF occurring as early as 2 days

after the Strep pharyngitis; type II hypersensitivity via molecular mimicry with Group A Strep M

protein (Step 1); Tx with penicillin.

- 8M + sore throat + new-onset tic; Dx? à answer = PANDAS (Pediatric Autoimmune Neuropsychiatric

Disorder Associated with Streptococcus) à Group A Strep infection can precipitate OCD, Tourette,

ADHD; answer = “check Streptolysin O titers”; student says wtf? à sounds weird when you first hear

about it, but this is actually HY for Psych shelf, Peds shelf, and 2CK.

- 7F + facial grimaces past 5 months + no other motor findings or abnormal sounds + mental status

normal; next best step in Mx? à answer = “schedule a follow-up examination in 3 months” à Dx =

provisional tic disorder à 1/5 children experience some form of tic disorder; most common ages 7-

12; usually lasts less than a year; “watch and wait” approach recommended. Provisional tic disorder

used to be called transient tic disorder; the name was changed because a small % go on to develop

chronic tics.

- 11-month-old boy + red tongue + maculopapular body rash + fever 100.5F; Dx? à answer = Scarlet

fever; Group A Strep; give penicillin to prevent RF; tongue colloquially referred to as Strawberry.

- 4F + maculopapular body rash + fever + white spots visualized on buccal mucosa; Dx? à answer =

rubeola (measles).

- 4F + maculopapular body rash + fever + suboccipital and/or post-auricular lymphadenopathy; Dx? à

answer = German measles (rubella).

- 4M + swollen lower face bilaterally + irritable + stiff neck; Dx? à answer = mumps; POM à Parotitis,

Orchitis, Meningitis; rash not classically associated.

- 4F + spiking fever followed by a rash + no other findings; Dx? à roseola (HHV-6); all you need to

remember is the “spiking fever followed by a rash.”

MEHLMANMEDICAL.COM 9
MEHLMANMEDICAL.COM

- 17F + itchy maculopapular rash on back + pink 2-cm ellipse on lower back; next best step in Mx? à

answer = reassurance and observation à Dx = pityriasis rosea à caused by HHV-6 and HHV-7; self-

limiting and lasts 5 weeks on average.

- 6M + Q shows you two pics: one of a circular rash (not a target) on the forearm; the other pic is a Bell

palsy (asymmetric smile); Q asks you to pick the antibiotic à answer = amoxicillin; Dx = Lyme disease;

rash need not be target (erythema chronicum migrans); don’t give doxycycline to kids age <8.

- 3M + rapid breathing for 1 hour + hyper-resonance on right side of chest + CXR shows overexpansion

of right lung; next best step in Mx? à answer = “endoscopic examination of the patient’s airway”

(bronchoscopy for foreign body aspiration); this Q is exceedingly HY for Peds shelf; easier Qs will

mention a 14-month-old crawling around on the floor beforehand.

- 2M + writing movements observed in sleep by parents + MRI of head shows periventricular nodules;

what else is most likely to be seen in this patient in the future? à answer = cardiac rhabdomyoma; Dx

= tuberous sclerosis (TSC) à periventricular nodules are the “tubers”; can also see renal

angiomyolipoma, adenoma sebaceum (angiofibromas in a butterfly distribution, along the nasal

bridge and nasolabial folds), subungual fibromas (nailbed fibromas); writhing movements in sleep =

seizures = classic first presenting feature.

- 17M + ataxia + cerebellar vascular lesion excised + red urine; Dx? à answer = von Hippel-Lindau à

causes cerebellar and retinal hemangioblastomas; can eventually lead to renal cell carcinoma (which

may become bilateral).

- 6F + axillary and groin freckling + small growth seen in the eye by mom? Dx? à neurofibromatosis

type I (NF1) à Lisch nodules (iris hamartomas); optic glioma; axillary + groin freckling; neurofibromas

(cutaneous lumps); café au lait spots (hyperpigmented macules); pheochromocytoma.

- 2F + congenital cataracts + mom has Hx of meningioma removed; Dx? à NF2 à congenital cataracts

can be seen; meningioma; classically bilateral acoustic schwannomas.

- 16M + violaceous papules in a temporal distribution + Hx of seizure; Dx? à Sturge-Weber; Port wine

stain birthmark classic, but violaceous papules is a newer presentation on USMLE.

- What are phakomatoses? à neurocutaneous disorders = TSC, VHL, NF1/2, Sturge-Weber; all

autosomal dominant, but SW is de novo.

MEHLMANMEDICAL.COM 10
MEHLMANMEDICAL.COM

- Neonate + large facial hematoma + echo shows aortic coarctation; Dx? à answer = PHACES syndrome

à Posterior fossa anomalies, Hemangiomas (classically large facial hemangioma), Arterial anomalies,

Cardiac anomalies, Eye anomalies, Sternal defects.

- Neonate + becomes blue during breastfeeds + cries and becomes pink when detaching from breast +

examination shows a pupil that appears to “bleed” into the periphery of the iris; Dx? à answer =

CHARGE syndrome à Coloboma (hole) of the eye, Heart defects, Atresia of the choanae, Renal

defects, Genitourinary defects, Ear abnormalities.

- 3F + brought to ED 30 mins after syncopal episode + mouth turned blue during temper tantrum + fell

to floor, shook, and became limp + after a few seconds she regained normal activity + exam shows

alert, active, well-hydrated patient; Dx? à answer = breath-holding spell.

- 4M + pale conjunctivae + 2/6 systolic murmur + Hb 6.4 g/dL + WBCs 50,000 + platelets 10,000; Dx? à

answer = acute lymphoblastic leukemia (ALL) till proven otherwise; murmur is flow murmur (from low

Hb à low O2 delivery à HR increases to compensate; this is extremely HY for peds).

- Leukemia in a kid; Dx until proven otherwise? à answer = ALL.

- 6M + just diagnosed with ALL + parents don’t want him to know + you’re now alone with the kid and

he asked you if he’s going to die; what do you say? à answer = “Have you talked to your parents

about this?”

- 6M + ALL + facial flushing and/or positive Pemberton sign; Dx? à answer = TALL (T cell ALL) à thymic

lesion can lead to SVC-like syndrome.

- 3F + 10-day Hx of coughing spells followed by vomiting; Dx? à answer = Pertussis; post-tussive

vomiting and hypoglycemia sometimes seen; coughing may also be described as “paroxysms.”

- 14M + WBCs 32,000 (85% lymphocytes) + two-week Hx of paroxysmal cough followed by vomiting;

Dx? à answer = pertussis. Student says wtf? à pertussis can cause absurdly elevated WBC count

that is almost all lymphocytes and resembles ALL.

- 17F + 8-month Hx of hirsutism + 1-yr Hx of menses occurring at 45-day intervals (prior was 28-day) +

increasingly moody and depressed according to mom + striae over abdomen; next best step? à

answer on Peds NBME = “measurement of morning fasting serum cortisol concentration”; students

frequently say wtf at this answer but it’s what the Peds NBME wants; wrong answers are

measurements of prolactin and testosterone.

MEHLMANMEDICAL.COM 11
MEHLMANMEDICAL.COM

- 12F + recent viral infection + lethargic + polyuria + polydipsia + high serum and urinary glucose; what

is the biochemical / acid-base disturbance most expected? à answer = high serum K (hyperkalemia)

but low total body K; low Na; low bicarb (metabolic acidosis); low pH; low CO2 (compensation).

- 12F + DKA + HR 56 + BP 146/88 + headache and confusion; Q asks cause of the mental status change;

answer = cerebral edema; hyperglycemia leads to osmotic changes with water moving out of cells à

cerebral edema (serious complication of DKA) à give IV normal saline first before adding insulin.

- 12-month-old female + fever + 8-10 watery stools daily + low BP + high HR + low bicarb; why the low

bicarb? à answer = lactic acidosis à any scenario with decreased perfusion (oxygen delivery) to

muscle leads to increased anaerobic respiration; this is HY for any type of shock – septic, cardiogenic,

hypovolemic (can also be seen with severe diabetes insipidus and pulmonary embolism).

- 3F + fell of tricycle two weeks ago + polyuria + polydipsia + high serum sodium + high urinary output;

Dx? à answer = diabetes insipidus à head trauma can cause both central DI as well as SIADH; DI =

high serum sodium and low urinary osmolality; SIADH = low serum sodium + high urinary osmolality.

- 3F + two-week Hx of fever, pallor, and decreased appetite + mouth ulcers + RBCs, WBCs, and platelets

all low; Dx? à answer = aplastic anemia (presumably from Parvo, but can also be other viruses); next

best step = immediate IV antibiotics for neutropenic fever (any time you have low neutrophils in the

setting of fever; if WBCs are low, then neutrophils also low; in addition, mouth ulcers on USMLE =

hugely HY presentation of neutropenia).

- 4F + treated for three weeks with TMP/SMX + now has isolated neutropenia; cause of neutropenia?

à answer = TMP/SMX à just be aware this can cause neutropenia.

- 3F + fever + RBCs, WBCs, platelets all low; IV antibiotics are administered; next best step in Dx? à

answer = bone marrow aspiration (sounds overly invasive, but it’s what they want to Dx the aplastic

anemia); there’s a UWorld Q where “Parvo IgM/IgG titers” is an answer, but it’s in an adult daycare

worker with a lacy body rash and no drop in her hematologic cell lines.

- 4M + hypoplastic thumbs + RBCs, WBCs, platelets all decreased; Dx? à answer = Fanconi anemia; AR

aplastic anemia characterized by absent or hypoplastic thumbs or radii.

- 4M + triphalyngeal thumbs + low RBCs; Dx? à answer = Diamond-Blackfan anemia; pure red cell

aplasia characterized by triphalyngeal thumbs.

MEHLMANMEDICAL.COM 12
MEHLMANMEDICAL.COM

- 4F + immunodeficiency + fairer skin than siblings + giant granules seen in phagocytes; Dx? à answer =

Chediak-Higashi syndrome; phagolysosomal fusion disorder with disruption of microtubule function;

melanosome function also impaired.

- 2M + immunodeficiency + delayed separation of umbilical cord; Dx? à answer = leukocyte adhesion

deficiency à deficiency of LFA-1/CD18 integrin.

- 2M + viral, fungal, bacterial, protozoal infections since birth + absent thymic shadow + scanty/absent

lymph nodes/tonsils; Dx? à answer = severe-combined immunodeficiency (SCID) à XR type is most

common and due to common gamma-chain mutation or IL-2 receptor deficiency; AR is due to

adenosine deaminase (ADA) deficiency; Tx is bone marrow transplant; absent thymic shadow means T

cell deficiency; scanty/absent lymphoid tissue means B cell deficiency.

- 2M + bacterial infections from ~6 months of life + scanty/absent lymph nodes/tonsils; Dx? à answer

= “deficiency of humoral immunity” (Bruton X-linked agammaglobulinemia); it must be pointed out

that a Q on one of the newer Peds forms has Bruton as the answer for a kid who has bacterial

infections since birth (which throws a dagger in the classic contrast with SCID); therefore the stronger

emphasis must be that Bruton is bacterial only, whereas SCID is all types of infections (viral, fungal,

bacterial, protozoal); Tx = “monthly infusion of immune globulin.”

- 2M + mental retardation + fairer skin than siblings + deficiency of hydroxylase enzyme; how could this

patient’s presentation have been prevented? à answer = “routine newborn screening” via heel-prick

test at birth for phenylketonuria (PKU; autosomal recessive; deficiency of phenylalanine hydroxylase)

à tyrosine becomes essential; phenylalanine must be avoided in diet; malignant PKU =

tetrahydrobiopterin (THB) deficiency (cofactor for phenylalanine hydroxylase).

- 8-month-old girl + oropharyngeal candidiasis + Hx of parainfluenza infection + 3/6 harsh systolic

murmur at left sternal border; what does she have a deficiency of? à answer = T lymphocytes à

possibly DiGeorge syndrome in this case (tetralogy of Fallot with murmur being a mix of VSD and

pulmonary stenosis); T lymphocyte deficiency will cause viral, fungal and protozoal infections; if the

USMLE wants B cell deficiency as part of answer, they’ll mention bacterial infections.

- 8-day-old newborn + truncus arteriosus + serum calcium of 7mg/dL + no thymic shadow; Dx? à

answer = hypoparathyroidism (DiGeorge; 3rd and 4th pouch agenesis à 3rd = thymus and inferior

parathyroids; 4th = superior parathyroids).

MEHLMANMEDICAL.COM 13
MEHLMANMEDICAL.COM

- 16F + 3-day Hx of pain and pressure over left cheek + Hx of strep pneumonias at age 6 and 10 +

seasonal allergies; Dx? à answer = “impaired humoral immunity” or “deficiency of mucosal

immunoglobulin”; Dx is IgA deficiency (one of the highest yield presentations for not just the Peds

shelf but also USMLE); sore cheek = sinusitis; presents as recurrent sinopulmonary infections; also

associated with Hx of Giardia infection, autoimmune diseases (e.g., vitiligo), and atopy (dry cough in

winter [cough-variant asthma], hay fever in spring, eczema in summer); anaphylaxis with blood

transfusion is “too easy” for most 2CK IgA deficiency Qs but will rarely show up, yes.

- 3M + recurrent OM + recently underwent placement of tympanostomy tube + mom HIV negative; Dx?

à answer = “antibody deficiency” à IgA deficiency.

- 6M + recurrent Staph infections + diarrhea; Dx? à answer = “neutrophil oxidation burst” à chronic

granulomatous disease (NADPH oxidase deficiency) à infections with catalase (+) organisms (Staph

infections are signature complication); SPACES à Staph, Psuedomonas, Aspergillus, Candida, E. coli,

Serratia; Aspergillus most common fungal infection seen in CGD; 2CK has Q where child gets Serratia

sepsis as part of CGD.

- 8F + fever + purpuric lesions over trunk and extremities + brother died of fulminant meningococcemia

four years ago; Dx? à answer “complement system immunodeficiency” à terminal complement

deficiency (C5-9) is associated with recurrent Neisseria infections (gonococcal and meningococcal).

- 14M + fever + stiff neck + non-blanching purpura on abdomen + BP of 60/35 + IV fluids and

norepinephrine have limited effect; Tx? à answer = hydrocortisone; Dx = Waterhouse-Friderichsen

syndrome; hemorrhagic necrosis of adrenal cortices secondary to meningococcal septicemia; non-

blanching rash in the setting of meningitis = meningococcus; hydrocortisone is the answer because

cortisol is deficient in this setting à cortisol normally needed to upregulate alpha-1 receptors on

arterioles, thereby permitting NE and E to do their job; that’s why NE has limited effect.

- 17F + Hx of cutaneous candida infections since childhood + 2-year Hx of type I diabetes mellitus + Hx

of autoimmune thyroiditis; Dx? à answer = chronic mucocutaneous candidiasis à answer = “T cell

dysfunction” = “impaired cell-mediated immunity” (both answers); although candida infections can be

seen in diabetes, the durations don’t match up here.

MEHLMANMEDICAL.COM 14
MEHLMANMEDICAL.COM

- 1-month-old male + diffuse white plaques in oral cavity that bleed when scraped + popular

erythematous rash with satellite lesions over the diaper area; mechanism? à answer = “decreased T

lymphocyte activity” à chronic mucocutaneous candidiasis.

- 8M + super high IgM on lab report; what’s the mechanism? à answer = deficiency of CD40 ligand on

T cells (can’t activate CD40 on B cells to induce isotype class switching) à Dx = Hyper IgM syndrome.

- 6F + recurrent Staph abscesses + eczematoid skin lesions + abnormal dentition; Dx? à answer =

hyper IgE syndrome (Job syndrome) à FATED à abnormal Facies, staphylococcal cold Abscesses,

retained primary Teeth, hyper IgE, Dermatologic findings (e.g., eczematoid lesions).

- 12M + eczematoid skin lesion on forehead + nosebleeds + Hx of infections; Q asks which cell is

dysfunctional à answer = T cell (on student’s 2CK, not Step 1); Dx = Wiskott-Aldrich syndrome (XR)

à classic triad of eczematoid skin lesions + thrombocytopenia + immunodeficiency.

- 3F + dilated superficial blood vessels on face + wobbly gait; mechanism? à answer = failure of

double-stranded DNA break repair” à Dx = ataxia-telangiectasia.

- 8M + Q shows you a pic of a red dot on the tongue or nailbed; Dx? à answer = hereditary

hemorrhagic telangiectasia (Osler-Weber-Rendu); autosomal dominant; HY tangent outside peds:

should be noted that this can lead to pulmonary arteriovenous fistulae in adults with high-output

cardiac failure (NBME for Step 1 has 45M + epistaxis since childhood + shows pic of red dot on tongue

+ high ejection fraction; answer = “pulmonary arteriovenous fistula” [due to HHT]).

- 17M + 3-wk Hx of lymphadenopathy, fever, and loose stools + hepatosplenomegaly + low RBCs and

WBCs + high IgM and IgG; Dx? à answer = HIV infection; wrong answers are CVID, IgA deficiency,

SCID, DiGeorge, Bruton (so you can eliminate to get there); CVID is can rarely occur as young as

adolescence but Dx is with IgG and IgA >2SD less than the mean (IgG high in this stem).

- Mother is HIV (+); what should be given to neonate following C-section? à answer = “A 6-week

course of zidovudine (AZT) within 12 hours of delivery”; some students argue this is outdated, but it’s

the answer on Peds NBME form 1; no newer Q exists yet for this.

- 6F + pruritic rash in intertriginous areas of elbows and knees + often appears with episodes of

wheezing and respiratory distress; Tx? à answer on NBME = topical triamcinolone (steroid); Dx is

atopic dermatitis (eczema) as part of atopy; should be noted “emollients” (lotion; should be oil-based)

are tried before topical steroids if both are listed.

MEHLMANMEDICAL.COM 15
MEHLMANMEDICAL.COM

- 17M + 12-month Hx of intermittent swelling of face/lips, chest, and arms + during episodes has

abdominal pain and diarrhea; next best step in Dx? à answer = “measurement of C1 esterase

inhibitor concentration”; not C1 esterase; C1 esterase inhibitor; Dx = hereditary angioedema; acute

Tx = C1 esterase inhibitor administration; danazol also used (induces liver to produce more C1EI).

- 3F + brown blood; Dx? à answer = congenital methemoglobinemia due to deficiency of cytochrome

reductase B5.

- 10M + ate new batch of sausage from his father, who is a butcher + high methemoglobin level; Tx? à

answer = “administration of methylene blue”; can also add vitamin C to regimen.

- 9-month-old boy + “can only sit with support” + “bangs two blocks together but does not scribble” +

“says ‘mama’ and ‘dada’ but not specifically to his parent”; Q wants “normal” or “delayed” for Gross

motor, Fine motor, and Language development; answer = Delayed Gross motor + Normal fine motor

and language development. Highest yield points regarding milestones (bolded mandatory ones):

o Gross motor:

§ Head up at 3 months (can support his/her own head unassisted).

§ Turn over front to back at 4-5 months.

§ Sits up at 6 months.

§ Crawls by 9 months.

§ Stands/walks alongside sofa at 10-11 months.

§ Walks at 12-15 months.

o Fine Motor:

§ Pincer grasp at 9 months.

§ Stacks 3, 6, 9 blocks at age 1, 2, 3, respectively.

§ Copies circle at 3-4 years.

§ Copies square at 4-5 years.

§ Copies triangle at 5-6 years.

o Language development:

§ “Mama” and “dada” by 1 year.

§ Recognizes own name by 1 year.

§ 2-word sentences at age 2.

MEHLMANMEDICAL.COM 16
MEHLMANMEDICAL.COM

§ 900 words at age 3.

o Miscellaneous:

§ Imaginary friends are normal at age 4-6 (Peds shelf likes this; no this is not

psychosis).

§ Repeated questioning about death is normal age 8-10 (on Peds shelf).

§ Parallel play age 2-3 (kids play side by side); cooperative play age 3-4.

- 10-month-old boy + grade IV GERD; Q asks for what surgical Mx to alleviate Sx; answer =

fundoplication.

- 12-hour-old newborn + excessive oral secretions and coughing after first feed + Sx resolved after

suctioning + pregnancy was characterized by polyhydramnios; next best step in Mx? à answer =

insertion of a nasogastric tube à Dx = tracheoesophageal fistula.

- 2-month-old boy + non-bloody non-bilious vomiting with feeds + increasing in amount and force; Dx?

à pyloric stenosis à associated with first-born males + oral erythromycin use (motilin receptor

agonist; Tx for chlamydia ophthalmia neonatorum); another Peds NBME Q wants you to know pyloric

stenosis is “single developmental defect,” implying it’s almost always not part of a broader syndromic

constellation; Dx with abdominal ultrasound; Tx = myomectomy; biochemical / acid-base disturbance

= low K, low Cl, high bicarb; detail such as “olive-shaped” abdominal mass too easy for 2CK but not

unheard of.

- 2-month-old boy + bilious vomiting + Down syndrome; Dx? à answer = duodenal atresia; do AXR

which shows double bubble sign à answer = “failure of canalization of proximal bowel”; bear in mind

that annular pancreas can be alternative cause of bilious vomiting + double bubble sign (not Down

syndrome).

- 2-month-old boy + bilious vomiting + chronic constipation; Dx? à answer = Hirschsprung; also

associated with Down syndrome; failure of neural crest cell migration distally; answer sometimes =

“rectal manometry” as next best step in Dx; notable that this can cause bilious vomiting.

- Acid-base / biochemical disturbance in vomiting? à hypochloremic, hypokalemic metabolic alkalosis.

- 18-month-old girl + intermittent abdominal pain for 24 hours + mass palpated in RLQ + fecal occult

blood test is positive; next best step in diagnosis? à answer = air-contrast enema (or contrast, or air,

etc.); imaging modalities such as USS (target sign) may be performed, but on the NBME, the answer

MEHLMANMEDICAL.COM 17
MEHLMANMEDICAL.COM

for how to Dx intussusception is always enema (Dx and therapeutic); presentation for intussusception

will almost always be under the age of 2 and be characterized by an “intermittent” nature to the

child’s presentation (i.e., intermittent crying; intermittently bringing the legs to the chest;

intermittently squatting; intermittently vomiting).

- 7-day-old boy + bilious vomiting + blood in stool + abdo slightly distended + bowel sounds decreased

+ digital rectal exam shows gross blood + lateral x-ray of upper GI tract with barium contrast shows

corkscrew appearance; Dx? à answer = “failure of normal bowel rotation” à congenital midgut

volvulus à diagnosed with upper-GI series (x-ray with contrast swallow).

- 1-month-old boy + bilious vomiting + normal rectal tone + fecal occult blood positive; Dx? à answer =

midgut volvulus (Peds NBME form 2); intussusception and Hirschsprung are wrong answers.

- 18-month-old girl + intermittent abdominal pain for 24 hours + fecal occult blood test is negative +

AXR shows dilated loops of small bowel and air-fluid levels; Dx? à answer = “volvulus” à congenital

midgut volvulus à originally I conjured that the negative fecal occult blood is the reason this is

volvulus (i.e., “sounds like intussusception but FOB is negative”), but the above 18-month-old from

Peds form 2 has positive FOB for midgut volvulus, so the point of distinction must be these specific

AXR findings, which would not be seen for intussusception à in other words, if vignette sounds like

intussusception but AXR shows dilated loops of small bowel and air-fluid levels, answer = midgut

volvulus, not intussusception, and FOB isn’t specific for either.

- 9M + blood in stool + Tc99 uptake scan confirms Dx; what’s the mechanism for patient’s

presentation? à answer = “heterotopic gastric tissue”; NBME has Qs with Meckel presenting even in

adults (do not fixate on age 2).

- 3F + failure to thrive + 3-month Hx of steatorrhea + did not pass meconium till 3 days of life;

mechanism for failure to thrive? à answer = “exocrine pancreatic insufficiency”; this is frequently

how the Peds shelf will refer to cystic fibrosis (can also refer to chronic pancreatitis in adults).

- 4-month-old girl with 5-cm melanoma-appearing lesion on thigh; Dx? à answer = congenital

melanocytic nevus (most common type of birthmark; benign).

- 4-month-old girl with 3-cm “bruise” on her back + vignette doesn’t sound like abuse; Dx? à answer =

blue nevus (Mongolian spot); more common in Asian descent; melanocytes don’t migrate to basal

layer + get trapped in dermis.

MEHLMANMEDICAL.COM 18
MEHLMANMEDICAL.COM

- Neonate + spongy 1-cm red lesion on the chest; Dx? à strawberry hemangioma.

- Strawberry hemangioma Tx? à don’t treat; will grow slightly then regress spontaneously over a few

years.

- Neonate + large vascular lesion on the leg + thrombocytopenia; Dx? à Kasabach-Merritt syndrome

(aka hemangioma with thrombocytopenia) à this is on the pediatric 2CK forms three times asked in

different ways; students always say wtf and I have to explain that, yes, it’s weird, but it’s HY for some

magical reason; this is not a strawberry hemangioma and requires surgical Tx.

- 1-month-old male + 12x8cm vascular lesion on buttocks since birth + examination shows scattered

petechiae and bruises; what best explains these findings? à answer = “platelet sequestration.” I’ve

memorized this from the NBMEs à similar to splenomegaly, which can cause thrombocytopenia from

sequestration within the red pulp, the implication that the large vascular lesion of KMS is that

platelets simply get caught within it.

- What is Potter sequence? à renal agenesis leading to oligohydramnios à causes pulmonary

hypoplasia in fetus (amniotic fluid needed for lung development).

- 12-hour-old male + abdomen is distended with a midline mass; Dx? à answer = posterior urethral

valves à most common genitourinary (GU) abnormality in neonatal males à thin membrane(s) in

the urethra preventing the outflow of urine.

- 2-day-old male + urinary dribbling + decreased urinary output + 6-cm suprapubic mass; Dx? à answer

= posterior urethral valves; two different Qs on different Peds forms.

- 3-month-old boy + fever + pyuria + few bacteriuria; next best step in Dx? à answer = renal

ultrasound; Dx most likely pyelonephritis (few bacteria typical on U/A despite pyuria); must do renal

and bladder ultrasound for all kids age 2-24 months who have febrile UTI à screens for congenital

genitourinary tract abnormalities.

- 6-month-old boy + one-day Hx of fever + pyuria + E.coli grown from urine culture + Abx initiated +

renal ultrasound shows no abnormalities; next best step? à answer = voiding cystourethrogram

(check for posterior urethral valves).

- What is phimosis vs paraphimosis? à phimosis = inability to retract the foreskin; paraphimosis =

inability to reduce (put back) the foreskin.

- Neonatal male with PUV; next best step in Dx? à voiding cystourethrogram. Tx must do surgery.

MEHLMANMEDICAL.COM 19
MEHLMANMEDICAL.COM

- 34M falls on balance beam + saddle injury + blood at urethral meatus; Dx + next best step in Mx? à

urethral injury; do retrograde urethrogram.

- 16M + skiing accident + bruising and/or pain over a flank + red urine; Dx + next best step in Mx? à

kidney injury (renal contusion); must do CT with contrast, not ultrasound. This is exceedingly HY on

surgery forms and sounds wrong (i.e., “why would you do CT with contrast in someone with

suspected renal injury if contrast can cause nephrotoxicity?”) à great question, but

CMS/NBME/USMLE still want CT with contrast, so don’t pick ultrasound.

- 16M + skiing accident + bruising and/or pain over a flank + no gross blood of urine (i.e, urine not red);

next best step in Mx? à urinalysis to look for blood (super HY) à if urinalysis shows blood, do CT

scan to check for renal injury. If U/A negative, renal injury unlikely, so choose “no further

investigations indicated,” not CT.

- 16M + skiing accident + bruising and/or pain over a flank + urinalysis is normal; next best step? à no

further investigations indicated.

- 3M + ran into corner of table + 1-day Hx of increasingly severe abdominal pain + vitals normal +

examination shows 2x2cm mass in the midline + serum lipase and amylase both elevated; next best

step? à answer = CT of abdomen à Dx = traumatic pancreatitis.

- Kid + bloody diarrhea + petechiae + red urine; Dx? à hemolytic uremic syndrome (HUS) caused by

EHEC O157:H7 or Shigella à HUS = triad of thrombocytopenia + schistocytosis (microangiopathic

hemolytic anemia) + renal insufficiency; toxin will inhibit ADAMTS13 in afferent arterioles + cause

endothelial damage à platelet consumption + clumping (thrombocytopenia) à RBCs shear as they

pass by the platelet clumps (schistocytosis).

- How does HUS contrast with TTP? à TTP is caused by a mutation that results in defective ADAMTS13,

or antibodies against ADAMTS13, resulting in the inability to cleave vWF multimers à platelet

clumping à similar progression as HUS. One of the points of contrast is that TTP is not toxin-induced,

and TTP also tends to be a pentad of the HUS findings + fever + neurologic signs.

- 30-month-old boy + fever 101F + loose, bloody stools + tonic-clonic seizure; Dx? à answer =

shigellosis on Peds NBME; wrong answers are HUS and “idiopathic seizure disorder.”

- 14F + lower abdominal cramps and intermittent knee pain for 6 weeks + 6-10 bloody stools per day +

WBCs and platelets elevated + arthrocentesis is normal; Dx? à answer = ulcerative colitis (UC) à IBD

MEHLMANMEDICAL.COM 20
MEHLMANMEDICAL.COM

frequently associated with arthritis; high WBCs characteristic of autoimmune flares; thrombocytosis

can be seen in infections and/or autoimmune flares; next best step in Dx = colonoscopy; Tx is initially

with 5-ASA compounds (mesalamine) and steroids; colectomy can be curative in severe cases.

- 16M + mouth ulcer + bloody diarrhea; Dx? à answer = Crohn disease à mouth to anus; mouth ulcers

are HY.

- HY differences between Crohn and UC?

o Crohn à transmural (can lead to perianal fistulae); granulomatous (non-caseating); skip

lesions (string sign on enema); cobblestoning (appearance on colonoscopy); “mouth to

anus,” but classically terminal ileum; increased risk of colorectal cancer (CRC) if colon

affected (lower risk compared to UC overall); erythema nodosum and anterior uveitis

common.

o UC à not transmural; not granulomatous; loss of haustra (lead-pipe appearance on enema);

pseudopolyps (appearance on colonoscopy); spares the anus; distribution is rectum-

ascending; greater risk of CRC compared to Crohn; associated with pyoderma gangrenosum

(crater-appearing lesion on [usually] limb with necrotic debris); associated with primary

sclerosing cholangitis more than Crohn.

- 7F + vignette shows pic of hyperpigmentation around the lips; Dx? à answer = “hamartomatous

polyps” à Dx = Peutz-Jeghers syndrome à perioral melanosis + hamartomatous colonic polyps à

peds form has “colonoscopy” as the answer in a 7F with PJ syndrome for next best step in Dx.

- 8F + recently finished course of antibiotics + bloody diarrhea; Dx? à answer = pseudomembranous

colitis (C. difficile) à presentations across NBMEs are both bloody and watery.

- 12M + epistaxis + bleeding time 9 minutes + platelet count 50,000; Dx? à immune thrombocytopenic

purpura (ITP); mechanism = antibodies against GpIIb/IIIa following viral infection (2CK Qs will often

not mention preceding viral infection for such diagnoses).

- 12M + epistaxis + bleeding time 9 minutes + platelet count 50,000; next best step? à answer =

corticosteroids, then IVIG, then splenectomy; Qbank has a Q floating around where answer is

observation only (apparently for mild cutaneous findings only in peds), but on NBME the answer is

always steroids (you don’t just watch the kid; you give him/her fucking steroids; not harmful).

MEHLMANMEDICAL.COM 21
MEHLMANMEDICAL.COM

- 17F + heavy periods + occasional epistaxis + finger takes a while to stop bleeding after being cut +

normal PT + normal aPTT + normal platelet aggregation studies; Dx? à answer = von Willebrand

disease (vWD); autosomal dominant; student says, “Wait, I thought aPTT and bleeding time are both

elevated in vWD. Why would aPTT be normal here?” à BT is always increased, as the main function

of vWF is to bridge GpIb on platelets to underlying collagen (so platelet dysfunction is guaranteed),

but the stabilizing effects on factor VIII are ancillary, not primary, which is why aPTT is normal in

about half of questions; it’s to my observation that most vWD Qs will have aPTT around the upper

limit of normal (NR 25-40s; where the Q might say aPTT is 38, or 43, seconds; only one Q I have seen

where BT was 60s for vWD); vWD will always have a mix of a platelet problem (cutaneous; generally

mild – i.e., epistaxis, petechiae) and a clotting factor problem (more severe – i.e., excessive bleeding

after tooth extraction; menorrhagia); platelet aggregation studies are normal because vWD doesn’t

have anything to do with GpIIb/IIIa, which mediate aggregation between platelets, not adherence to

underlying collagen; Tx is with desmopressin (DDAVP; stimulates production and release of vWF by

endothelial cells).

- 5M + hemarthrosis; Dx till proven otherwise? à answer = hemophilia A or B (A way more common,

so choose this if both listed and no way to differentiate); both X-linked recessive; isolated increase in

aPTT à hemophilia A is VIII deficiency; hemophilia B (Christmas disease) is factor IX deficiency;

hemarthrosis classic for hemophilia but rare in wVD (factor deficiency creates greater impediment of

clotting than the auxiliary deficit seen in vWD); Tx is with DDAVP and factor VIII replacement in

hemophilia A; DDAVP cannot be used for hemophilia B (factor IX replacement only).

- 5M + hemophilia A + Hx of several Txs of factor VIII replacement + Txs having increasingly reduced

ability to help + aPTT is now >120s; Dx? à answer = antibodies against factor VIII (occurs with

repeated Txs).

- 5F + hemophilia A; Q asks you directly how this is possible; answer = lyonization (skewed X-

inactivation); you will never see an XR disorder in a female unless the explicit purpose of the question

is lyonization.

- 12M + epistaxis + bleeding time 9 minutes + platelet count 50,000 + most effective way to decrease

recurrence? à answer = splenectomy. Student says, “Wait, I thought we give steroids first.” Yeah,

you’re right, but splenectomy is still most effective at decreasing recurrence.

MEHLMANMEDICAL.COM 22
MEHLMANMEDICAL.COM

- 10F + tonic-clonic seizure + 2-wk Hx of fever and joint pain + Hb, WBCs, and platelets all down +

Coombs test positive; Dx? à answer = lupus; cell lines are down due to anti-hematologic cell line

antibodies (i.e, this is not aplastic anemia; answer = “increased peripheral destruction”); Tx =

steroids.

- 15F + arthritis + malar rash + urine positive for blood and protein; which of the following would help

determine the course of management or this patient? à answer = renal biopsy à guides the

management of lupus.

- 3F + viral infection + fever + tonic-clonic seizure; Dx? à answer = febrile seizure; fever can precipitate

idiopathic seizure in 2-4% of children ages 6 months – 5 years; there is about a two-fold risk

progression to epilepsy compared to general population; Tx is with benzodiazepine.

- 10-month old boy + jerking movements of the limbs + EEG shows chaotic high-amplitude spikes with

no recognizable pattern; Dx? à answer = West syndrome (infantile spasms) à triad of spasms +

hypsarrhythmia on EEG (no recognizable pattern with high amplitude spikes) + developmental

regression; international definition of the diagnosis requires two out of three; starts age 3-12 months;

seen in 1-5% of Down syndrome kids; Tx = ACTH, prednisolone, or vigabatrin; ACTH is thought to act

by two mechanisms: 1) stimulates steroid (cortisol) release; 2) direct (cortisol-independent) effect on

melanocortin receptors.

- 13M + tonic-clonic seizure + 4-month Hx of hypnagogic/hypnopompic jerking of left arm + uncle has

epilepsy; Dx? à answer = juvenile myoclonic epilepsy; genetic with unclear inheritance pattern;

characterized by myoclonic jerks (usually hypnagogic and/or hypnopompic) that progress to tonic-

clonic seizures after several months; age of onset is usually 10-16, but can also start in adulthood; Tx

is valproic acid.

- 4F + few-month Hx of near-daily seizures + seizures typically occur while she’s sleeping + has started

putting objects in her mouth and making less eye contact + seizures not responding to anti-epileptic

meds; Dx? à answer = Lennox-Gastaut syndrome à severe childhood-onset epilepsy characterized

by near-daily seizures and cognitive decline (hyperoralism is a sign of cognitive regression [babies put

things in their mouths]); poor prognosis, with 5% mortality rate in childhood; 80-90% persistence of

seizures into adulthood.

- Child <5 years + watery diarrhea à rotavirus; classically seen < age 5.

MEHLMANMEDICAL.COM 23
MEHLMANMEDICAL.COM

- Child <5 years + watery diarrhea + 12-year-old brother and parents have similar Sx; Dx? à answer =

Norwalk virus, not rotavirus, because rest of family wouldn’t get rotavirus.

- 4M + watery diarrhea two weeks ago + now has heavier diarrhea and bloating and flatulence with

meals; Dx? à secondary lactose intolerance caused by sloughing of brush border following

gastroenteritis (rotavirus).

- 8M + bloating and flatulence with dairy products; next best step in diagnosis? à answer = hydrogen

breath test, or testing for decreased stool pH; Dx = lactose intolerance.

- 2F + iron deficiency anemia + bloating + diarrhea + eating table food; Dx? à answer = Celiac disease;

can cause iron deficiency anemia; Dx with Ab screen (anti-endomysial/-gliadin; anti-tissue

transglutaminase IgA).

- 5F + positive screen for Celiac Ab; next best step? (answers are either duodenal biopsy, or no further

studies indicated) à answer = duodenal biopsy (shows flattened villi); sounds wrong, but it’s what

NBME wants.

- “Can you explain that milk protein allergy stuff? Like the cow milk, soymilk, hydrolyzed formula

stuff.”

o Peds NBME / 2CK wants you to know that best way to decrease chance of developing allergy

to food protein à answer = “exclusively breastfeed for the first 6 months.” Milk protein

allergy is one type of food protein allergy.

o Milk protein allergy affects 2-3% of infants and young children; usually self-resolves, but

0.4% of adults retain the allergy.

o 10-15% of infants with cow milk protein allergy demonstrate allergic crossover with

soymilk protein.

o Dx can be made with skin prick and patch tests.

o Treatment for baby/infant with milk or soy protein allergy on USMLE is “switch to hydrolyzed

formula” à proteins processed into fragments à FDA labels hydrolyzed formulas as

hypoallergenic.

o Different types of milk protein allergy:

§ Food protein-induced enterocolitis syndrome (FPIES)

MEHLMANMEDICAL.COM 24
MEHLMANMEDICAL.COM

• Very severe; incessant vomiting and diarrhea within hours of consumption;

some may experience shock due to dehydration.

§ Food protein-induced allergic proctocolitis (FPIAP)

• Milder; characterized by intermittent bloody stools.

• This is how cow milk allergy frequently presents on Peds NBME.

§ Food protein-induced enteropathy (FPE)

• Chronic diarrhea that resolves when milk protein removed from diet.

- 2-month-old boy + started on cow milk-based formula last week + 1-wk Hx of vomiting and diarrhea

hours after feeds; next best step in Mx? à answer = “switch to hydrolyzed formula”; Dx = FPIES.

- 6-week-old girl + on cow milk-based formula for first 4 weeks of life + switched to soy milk-based

formula 2 weeks ago + 2-wk Hx of stool streaked with blood and mucous + no vomiting; next best step

in Mx? à answer = “switch to formula with hydrolyzed casein”; Dx = FPIAP.

- Bloody diarrhea + appendicitis-like pain (pseudoappendicitis) in a child à Yersinia enterocolitica.

- 3F + finished 10-day course of Abx + fever 101F + LLQ tenderness + fecal occult blood is positive; Dx?

à C. difficile; Tx = vancomycin. Two important points: 1) C. diff causes bloody stool frequently on 2CK

forms; 2) pseudoappendicitis seen with Y. enterocolitica is indeed RLQ (terminal ileitis / mesenteric

adenitis), not LLQ.

- 16-month-old male + Giardia infection + failure to thrive; Q asks mechanism for failure to thrive;

answer = “malabsorption of fats and carbohydrates”; Tx Giardia infection with metronidazole.

- 16M + painful testes + fever + positive cremasteric reflex; Dx? à answer = epididymitis

- Most common organism causing epididymitis? à Chlamydia in sexually active younger males; E. coli

in elderly males. This is also the same for prostatitis. If the vignette tells you no organisms grow on

culture, choose Chlamydia.

- 16M + acutely painful testes + negative cremasteric reflex; Dx? à answer = testicular torsion; do

Doppler ultrasound followed by surgical detorsion.

- 6M + painful testis + superior pole shows blue/black discoloration + bowel sounds are decreased +

abdomen is rigid; Dx? à answer = strangulated hernia, not testicular torsion à answer = “operative

management.”

MEHLMANMEDICAL.COM 25
MEHLMANMEDICAL.COM

- 6M + painful testis + superior pole shows blue dot + cremasteric reflex is intact; Dx? à answer =

torsion of appendix testis à this is on the new peds form but is fair game for FM à torsion of

appendix testis is different from testicular torsion; the latter presents with negative cremasteric

reflex; but torsion of appendix testis causes “blue dot sign.”

- 8-month-old boy + unilateral testicular enlargement + light shone to it causes transillumination;

answer = “persistent processus vaginalis” (hydrocele).

- Tx for hydrocele? à observe until the age of one as most spontaneously resolve; this is almost always

the answer; after the age of one, surgical management can be considered.

- Cryptorchidism; Tx? à observe; don’t do orchiopexy until at least age 1; most spontaneously descend

in first year of life.

- 3M + hard nodule on testis; Dx? à yolk sac tumor (endodermal sinus tumor) à serum AFP may be

elevated.

- 3M + hard nodule on testis + serum AFP + beta-hCG are elevated; Dx? à answer = mixed germ cell

tumor (embryonal cancer), not yolk sac tumor (yolk sac tumor is only high AFP; in mixed germ cell,

beta-hCG is also elevated).

- 14M + heaviness and/or bogginess of testes; Dx? à varicocele à one FM shelf Q literally says “bag of

worms” (normally this is so buzzy that we’d say this wouldn’t show up on an actual form, but it does,

so it must be mentioned here) à Dx with Doppler ultrasound à elective surgical intervention may be

performed to reduce risk of sub-fertility due to increased testicular temperature.

- 14M + dragging sensation of left testis + soft tissue mass at top of left testis that disappears when

supine; patient is at increased risk of what? à answer = infertility; Dx = varicocele.

- 16M + exam for school sports + both testes descended + painless mass in scrotum that can be

“milked” into the abdomen through the inguinal ring; Dx? à answer = indirect inguinal hernia.

- 8-month-old boy has undescended testis; Tx? à answer = observation until at least the age of 1; do

not do orchiopexy before age 1 for USMLE purposes.

- Tx for scabies? à answer = topical permethrin.

- Tx for pediculosis (lice)? à answer = topical permethrin.

- Tx for acne:

MEHLMANMEDICAL.COM 26
MEHLMANMEDICAL.COM

o Topical retinoids first (i.e., topical tretinoin; NOT oral isotretinoin); cause photosensitivity

(rash); also used for photoaging; mechanism is decreasing sebum production; topical

tretinoin (not oral isotretinoin) is not a teratogen and does not have any effect on pregnancy

or male sperm.

o Benzoyl peroxide used second; often coadministered with topic retinoids; mechanism is the

killing of bacteria.

o Topical clindamycin.

o Oral tetracycline; causes photosensitivity (blistering).

o Oral isotretinoin; must do beta-hCG in women; recommend barrier contraception even if on

OCP; can cause elevations in LFTs; can cause dyslipidemia; main complaint is dry skin +

peeling; takes several weeks to really start working but ultra-effective according to most

patients; can be commenced earlier in patients with severe nodulocystic acne; works by

diffusely shutting of sebum production.

- 3M + large calves + uses arms in order to crawl up off the floor; mechanism for disorder? à answer =

defective cystoskeletal connection to extracellular matrix à Dx = Duchenne muscular dystrophy (XR);

caused by frameshift mutation; Becker is less severe form presenting in adolescence or early-

adulthood; Becker not caused by frameshift (i.e., missense, etc.).

- 4F + intermittent cramping past several months + difficulty relaxing grip when squeezing physician’s

fingers; Dx? à answer = myotonic dystrophy à CTG trinucleotide repeat (TNR) expansion.

- 13M + 3-month Hx of decreasing muscle coordination + impaired speech + high-arched feet; Dx? à

answer = Friedreich ataxia à GAA TNR expansion; development of cardiomyopathy in >90%; pes

cavus (high-arched feet) a classic finding; in contrast, pes planus (flat feet) is Marfan syndrome.

- 14F + tripping and incoordination past 10 months + high-arched feet + hammer toes + weakness and

wasting of hand and foot muscles + deep tendon reflexes absent at ankles + vibration and

proprioception decreased on lower extremities + motor nerve conduction studies decreased in lower

extremities with prolonged distal motor latencies; Dx? à answer = Charcot-Marie-Tooth disease (aka

hereditary motor and sensory neuropathy); mechanism is defective protein production for myelin

sheaths and nerve fibers; hammer toes classic; pes cavus also seen.

MEHLMANMEDICAL.COM 27
MEHLMANMEDICAL.COM

- 2M + “scissoring” of the legs + increased tone in all extremities; Dx? à cerebral palsy; posited to have

peripartum fetal hypoxia as precipitating cause; spastic cerebral palsy accounts for 708-80% of cases.

- 10F + violaceous erythema of upper eyelids + scaly knuckles and finger joints; Dx? à answer =

dermatomyositis à asked on Peds NBME; heliotrope rash + mechanic’s hands; Gottron papules and

shawl rash can also be seen; Dx with electromyography and anti-Jo1/-Mi2 (same as polymyositis);

definitive Dx is muscle biopsy; Tx acute flares with steroids.

- 15M + 5’11” + plays soccer + knee pain; Dx? à Osgood-Schlatter à inflammation of patellar ligament

at the tibial tuberosity; occurs in fast-growing, active teenagers; USMLE wants “repeated avulsion

microfractures” as an answer.

- Kid + recurrent knee redness, warmth, pain + fever à Juvenile rheumatoid arthritis (JRA; called Still

disease if there are extra-articular manifestations like fever and rash).

- Kid + recurrent joint pain +/- high ESR +/- rash à JRA.

- Kid + sore throat two days ago + high ESR + Hx of intermittent knee pain + presents today with knee

pain + afebrile; Dx? à JRA à infection can be a precipitating factor for a flare.

- Tx for JRA? à NSAIDs + corticosteroids.

- 7F + 1-yr Hx of occasional fever and knee pain + low Hb + MCV 75; Dx? à anemia of chronic disease

(AoCD) secondary to JRA à low MCV seen AoCD in various Qs on 2CK NBMEs (resources classically

say normocytic [80-100]).

- 6M + viral infection + now has hip pain +/- fever; Dx? à answer = toxic synovitis (aka transient

synovitis), not septic arthritis à inflammation of the synovial lining of hip joint; Tx is supportive.

- 6M + suspected JRA + red, hot, painful knee à must do arthrocentesis to rule out septic arthritis. If

the vignette sounds like classic transient synovitis (affects hip, not knee), you do not need to do an

arthrocentesis.

- 3F + recurrent joint pain + fever + rash; Q says “in addition to naproxen, which of the following is the

next best step in Mx?” à answer = slit-lamp exam à indicated annually (high risk of anterior uveitis).

- 5F + 2-day Hx of limp and left hip pain + a week ago had watery stools and a temp of 100F + pain with

weight-bearing and movement + no swelling or erythema; Tx? à answer = ibuprofen (toxic synovitis).

- 17F had kickboxing tournament last weekend + knee is red, warm, tender à arthrocentesis à septic

arthritis.

MEHLMANMEDICAL.COM 28
MEHLMANMEDICAL.COM

- 15F + severe pain of sternoclavicular joint + fever + arthrocentesis yields thick, yellow fluid + gram

stain shows gram-negative diplococci; next best step? à answer = “culture of the aspirate fluid”;

student says “wtf why? You’ve already determined the organism” à determine sensitivities; but

empirically she would receive IM ceftriaxone and oral azithromycin or doxycycline; if septic, do IV

therapy.

- 3M + fever + bone pain of the tibia + Tc99 bone scan shows uptake in diaphysis; Dx? à answer =

Ewing sarcoma à presents like osteomyelitis with bone pain and fever; t(11;22); onion-skinning on

histo; blue cells.

- 16M + soccer tournament yesterday + fever + high WBCs + bone pain + Tc99 bone scan shows uptake

in the metaphysis; Dx? à answer = osteomyelitis; uptake in metaphysis, not diaphysis (Ewing).

- 1M + white-eye reflex; Q asks what child is at increased risk for later in life; answer = osteosarcoma;

this is a HY Q à children with hereditary retinoblastoma are also prone to osteosarcoma (generally

age 15-25 years); Q might also say a 15-year-old with Hx of enucleation from childhood cancer has a

bone mass à osteosarcoma.

- 14M + on knees helping dad with plumbing under kitchen sink for several hours + knee pain + joint

shows no effusion + no fever; Dx? à answer = prepatellar bursitis.

- 4M + increased bilirubin following viral infection or drug; blood smear shows degmacytes;

mechanism? à answer = decreased NADPH production; Dx = G6PD deficiency (XR); degamacytes =

bite cells; Heinz bodies (denatured/oxidized Hb) also seen in RBCs.

- 4M + hemolytic anemia due to enzyme deficiency + G6PD not listed as answer; Dx? à answer =

pyruvate kinase deficiency (second most common cause of hemolytic anemia secondary to enzyme

deficiency, after G6PD).

- 7M + viral infection or took drug + spherocytes seen on smear + Coombs test positive; Dx? à answer

= hemolytic anemia (caused by drug or infection); in contrast, if Coombs negative, answer =

hereditary spherocytosis.

- 3M + Hx of pathologic jaundice treated with phototherapy for 5 days + father had splenectomy when

younger + Coombs test negative + high reticulocyte count (10%); what would a blood smear show? à

answer = “normocytic and normochromic”; Dx = hereditary spherocytosis; autosomal dominant; Tx

ongoing hemolysis with splenectomy.

MEHLMANMEDICAL.COM 29
MEHLMANMEDICAL.COM

- 8F + viral infection + appears pale + spleen enlarged + low Hct + spherocytes on smear; patient is at

increased risk for what? à answer = cholelithiasis due to pigment stones.

- Kid with suspected JRA has sore knee à must do arthrocentesis to rule out septic arthritis.

- 3F + 3-month Hx of leg pain predominantly in calves + occurs at night and wakes her from sleep +

exacerbated by daily activity + relieved by acetaminophen + vitals normal + P/E normal; Dx? à

answer on Ped NBME = growing pains (weird Dx you need to know).

- Patient has “knock-knees” (i.e., knees touch); Dx? à genu valgum.

- 9F + both legs bowed + small child and/or malabsorptive disease (i.e., Crohn, CF); Dx? àanswer =

rickets (vitamin D deficiency) à will also cause low calcium, low phosphate, high PTH, high ALP

(because PTH high).

- 9F + both legs bowed + parents noticed bowing since she started to walk + recently bowing worse in

right leg + x-ray while standing shows collapse of the medial aspect of the metaphysis of proximal

tibia + rest of vignette describes healthy, thriving patient; Dx? à answer = tibia vara (Blount disease);

wrong answer is rickets; should be noted that bowing is physiologic age < 2 years; tibia vara.

- 11F + spina bifida + paraplegic and wheelchair-bound + swelling and pain in thigh for two days +

afebrile; next best step? à answer = “x-ray of lower extremity”; fracture may indicate child abuse.

- 4-month-old + “clicking/clunking” on physical exam à (+) Ortolani and Barlow maneuvers à primary

hip dysplasia (congenital hip dysplasia) à once these are positive, the next best step is ORTHO

REFERRAL if it is listed à referral always sounds wrong, but this is the correct answer if it’s listed; if

it’s not listed, do ultrasound if under 6 months, or x-ray if over 6 months. Tx is with abduction harness

(Pavlik harness; looks frog-leg-like).

- Newborn girl + palpable clunk when the hip is abducted, flexed, and lifted forward; what is the most

likely mechanism of the disease? à answer = “shallow, poorly developed acetabulum” (congenital

hip dysplasia).

- 5-8-year-old boy with painful limp; no other risk factors; x-ray shows contracted capital epiphysis; Dx?

à Legg-Calve-Perthes (idiopathic avascular necrosis); the word “contracted” wins over “capital

epiphysis” à this is a Q on one of the NBME forms where everyone selects slipped capital femoral

epiphysis (SCFE), but it’s Legg-Calve-Perthes; Tx = hip replacement.

MEHLMANMEDICAL.COM 30
MEHLMANMEDICAL.COM

- 5-8-year-old boy with painful limp + sickle cell disease; Dx? à avascular necrosis (but not Legg-Calve-

Perthes, because LCP is idiopathic).

- 5-8-year-old boy + painful limp + x-ray is negative + bone scan confirms diagnosis; answer? à USMLE

wants you to know that x-ray can be negative initially in avascular necrosis, but bone scan or MRI can

also pick it up.

- 11-13-year-old overweight boy with a painful limp; Dx? à SCFE; Tx = surgical pinning.

- 13M + painful limp + walks with antalgic gait + vignette mentions zero about his weight; mechanism?

à answer = “displacement of the epiphysis of the femoral head” à SCFE.

- 2-year-old boy running + playing with 8-year-old sister + they were holding hands and he fell + now he

holds arm pronated by his side; Dx? à nursemaid’s elbow à radial head subluxation.

- Tx for nursemaid’s elbow à hyperpronation OR gentle supination (both are correct answers; only one

will be listed).

- Kid falls on outstretched arm + pain over anatomical snuffbox; Dx + next best step in Mx? à scaphoid

fracture à do x-ray.

- Kid falls on outstretched arm + pain over anatomical snuffbox + x-ray is negative; next best step in

Mx? à thumb-spica cast à x-ray is often negative in scaphoid fracture; must cast to prevent

scaphoid avascular necrosis à re-x-ray in 2-3 weeks.

- 14F + low T3/T4 + high TSH; Q asks what you’d see on biopsy of thyroid; answer on Peds NBME =

lymphocytic infiltration (Hashimoto, aka chronic lymphocytic thyroiditis); no idea why this is asked

specifically on the Peds form but it is.

- 17F + BMI 33 + irregular periods; Dx? à answer = anovulation; insulin resistance à abnormal GnRH

pulsation à high LH/FSH à insufficient FSH means follicle isn’t adequately primed by the time the LH

spike triggers ovulation à follicle retained as cyst; add hirsutism à Dx = polycystic ovarian syndrome

(PCOS; must have 11 cysts bilaterally as per Amsterdam criteria); should be noted that on Peds form

5 the answer for the location of the pathology in PCOS is ovary, not hypothalamus.

- 17F + polyuria + polydipsia + high BMI; Dx? à answer = type II diabetes (yes, in a 17-year-old).

- 10F + polyuria + polydipsia + sore throat two weeks earlier; most likely infective Dx? à answer =

coxsackie B virus à can precipitate type I diabetes mellitus via molecular mimicry à antibodies

MEHLMANMEDICAL.COM 31
MEHLMANMEDICAL.COM

against six-oligopeptide viral sequence cross-react with beta-islet cell glutamic acid decarboxylase 65

(anti-GAD65 Abs).

- 10F + vesicles in posterior oropharynx; Dx? à answer = herpangina (coxsackie A) à can also cause

hand-foot-mouth disease (vesicles at these locations).

- Young child with normal free T4 and low total T4 à thyroid-binding globulin deficiency (opposite of

pregnancy).

- Young child + large belly + large tongue + hypotonia à cretinism (congenital hypothyroidism).

- Young child + large belly + family feeding him rice gruel; Dx? à answer = “protein-calorie

malnutrition” à kwashiorkor.

- Young child in third-world country + severe wasting; Dx? à marasmus (total calorie malnutrition).

- Nocturnal enuresis; when is it pathologic? à after age 5.

- 6M + nocturnal enuresis; next best step? USMLE / NBME / shelf wants the following order:

o Behavioral answer first; e.g., spend more time with child; decrease overt stressors as much

as possible.

o If the above not an answer, do star chart (positive reinforcement therapy; i.e., don’t wet the

bed and get a star; get 5 stars for extra dessert; 100 and we go to Disneyland).

o If star chart not listed or already attempted, next answer is enuresis alarm.

o Medications like imipramine and desmopressin are always wrong; water deprivation after

5pm is also always wrong.

o Students mess these Qs up because they’ll see enuresis alarm as correct on one form, but on

a different form it’s star chart, so know the above order.

- How to differentiate viral from bacterial upper respiratory tract infection (URTI)? à CENTOR criteria

o If 0 or 1 point, the URTI is unlikely to be bacterial (i.e., it’s likely to be viral). If 2-4 points,

chance is much greater that URTI is bacterial.

o 1) Absence of cough (i.e., no cough = 1 point; if patient has cough = 0 points).

o 2) Fever.

o 3) Tonsillar exudates.

o 4) Lymphadenopathy (cervical, submandibular, etc.).

MEHLMANMEDICAL.COM 32
MEHLMANMEDICAL.COM

- There is a version of the criteria that includes age, but on the USMLE it can cause you to get questions

wrong. So just use the simplified above four points.

o If 0-1 point, answer = “supportive care”; or “no treatment necessary”; or “warm saline

gargle” (same as supportive care); or “acetaminophen.” Latter is answer for 3M with viral

URTI + fever on Peds NBME form 2.

o If 0-2 points, next best step = “rapid Strep test.” If rapid Strep test is negative, answer =

throat culture, NOT sputum culture.

o While waiting on the throat culture results, we send the patient home with amoxicillin or

penicillin for presumptive Strep pharyngitis.

o If child is, e.g., 12 years old, and develops a rash with the beta-lactam, answer = beta-lactam

allergy.

o If the vignette is of a 16-17 year-old who has been going on dates recently (there will be no

confusion; the USMLE will make it clear), the answer = EBV mononucleosis; therefore do a

heterophile antibody test (Monospot test).

o EBV is the odd virus out that usually presents with all four (+) CENTOR criteria.

o This is why it’s frequently misdiagnosed as Strep pharyngitis. It is HY to know that beta-

lactams given to patients with EBV may cause rash via a hypersensitivity response to the Abx

in the setting of antibody production to the virus. EBV, in a patient who does not receive

Abx, can cause a mild maculopapular rash. But the rash with beta-lactam + EBV causes a

more intense pruritic response generally 7-10 days following Abx administration on the

extensor surfaces + pressure points.

- 9-month-old boy + fever for five days + edema of dorsa of hands + cervical lymphadenopathy +

cracked, fissured lips + redness of oral mucosa and conjunctivae; Tx? à answer = high-dose aspirin +

IVIG; Dx = Kawasaki disease; never give aspirin to kid (Reye syndrome) unless Dx is Kawasaki; other

features are palms and soles desquamation + coronary artery aneurysms.

- “Waiter tip” position in kid à upper brachial plexus injury (C5/6) à Erb-Duchenne palsy.

- Claw hand à lower brachial plexus à C8/T1 à Klumpke palsy.

- Asthma (outpatient) à albuterol (short-acting beta-2 agonist; SABA) inhaler for immediate Mx à if

insufficient, start low-dose ICS (inhaled corticosteroid) preventer à if insufficient, maximize dose of

MEHLMANMEDICAL.COM 33
MEHLMANMEDICAL.COM

ICS preventer à if insufficient, add salmeterol inhaler (long-acting beta-2 agonist; LABA); in other

words:

- 1) SABA; then

- 2) low-dose ICS; then

- 3) maximize dose ICS; then

- 4) LABA.

- That initial order is universal. Then you need to know last resort is oral corticosteroids, however they

are most effective. In other words:

- 12M has ongoing wheezing episodes + is on albuterol inhaler; next best step? à add low-dose ICS.

- 12M has ongoing wheezing episodes + is on albuterol inhaler; what’s most likely to decrease

recurrence à oral corticosteroids (student says “wtf? I thought you said ICS was what we do next and

oral steroids are last resort” Yeah, you’re right, but they’re still most effective at decreasing

recurrence. This isn’t something I’m romanticizing; this is also assessed on the FM NBME forms.

- After the LABA and before the oral steroids, any number of agents can be given in any order – i.e.,

nedocromil or cromolyn sodium, zileuton, montelukast, zafirlukast.

- MOA of nedocromil and cromolyn sodium? à mast cell stabilizers.

- MOA of zileuton? à lipoxygenase inhibitor (enzyme that makes leukotrienes from arachidonic acid).

- MOA of the -lukasts ?à leukotriene LTC, D, and E4 inhibitors. LTB4 receptor agonism is unrelated and

induces neutrophilic chemotaxis (LTB4, IL-8, kallikrein, platelet-activating factor, C5a, bacterial

proteins [Step 1]).

- 16M goes snowboarding all day + takes pain reliever for sore muscles afterward + next day develops

wheezing out on the slopes again; what’s going on? à took aspirin + this is Samter triad (now

cumbersomely known as aspirin-exacerbated respiratory disease [AERD]) à triad of aspirin-induced

asthma + aspirin hypersensitivity + nasal polyps). Just to be clear, other NSAIDs can precipitate

Samter triad, but the literature + USMLE will make it explicitly about aspirin.

- 16M takes aspirin + gets wheezing; what are we likely to see on physical exam? à answer on USMLE

= nasal polyps.

- 8M + non-productive cough worse at night + SoB during the day + CXR shows mild hyperinflation;

next best step in Dx? à answer on NBME = spirometry à Dx = asthma.

MEHLMANMEDICAL.COM 34
MEHLMANMEDICAL.COM

- 8M + Hx of asthma + recent dysphagia to solids + upper endoscopy shows many concentric rings; Dx?

à answer = eosinophilic esophagitis à dysphagia and/or weight loss in patient with asthma;

endoscopy shows “trachealization” of the esophagus; biopsy shows dense eosinophil infiltrates.

- 6F + attended summer camp for one month + comes home with peach-colored papules with central

umbilication; Dx? à answer = molluscum contagiosum.

- 16M + large red, non-vesicular rash on the chin; Q asks for Dx based on image; answer = impetigo

(don’t go chasing HSV or tinea faciei).

- 11M + 7-day-Hx of yellow crusties around his lips + red urine; what’s the mechanism? à answer =

“antigen-antibody immune complex” (PSGN) caused by Strep impetigo (be aware that S. aureus

exceeds Group A Strep for both bullous and non-bullous impetigo).

- 11M + yellow crusties on face; Tx? à answer = topical mupirocin (USMLE will not force you to choose

between topical mupirocin and oral dicloxacillin / cephalexin for bullous impetigo; I point this out

because more extensive impetigo can be managed with oral Abx, but NBME wants mupirocin; if the

latter is not listed, choose oral dicloxacillin or oral cephalexin; these orals have MSSA coverage;

amoxicillin and penicillin do not).

- Beefy red, well-demarcated skin plaque; Dx? à erysipelas à Group A Strep (S. pyogenes) >>> S.

aureus.

- More diffuse pink skin lesion + tenderness + fever à cellulitis à S. aureus exceeds S. pyogenes.

- Tx of erysipelas + cellulitis à oral dicloxacillin or oral cephalexin.

- Most common cause of otitis media? à Strep pneumoniae.

- Tx of otitis media? à oral amoxicillin only.

- Tx of recurrent OM à amoxicillin/clavulanate.

- When to do tympanostomy tube? à three or more OM in 6 months, or 4 or more in a year.

- Most common cause of otitis externa? à Pseudomonas.

- Tx of otitis externa? à topical ciprofloxacin + hydrocortisone drops.

- 17M + on the crew team + repeated water exposure; how best to prevent otitis externa? à

prophylactic alcohol-acetic acid drops; in contrast, carbamide peroxide drops are for cerumen (wax)

buildup.

MEHLMANMEDICAL.COM 35
MEHLMANMEDICAL.COM

- 8M + Hx of ear infection one month ago + slightly reduced hearing in one ear + otoscopy shows fluid

behind the tympanic membrane + afebrile and well-appearing; Dx + Tx? à answer = otitis media with

effusion (serous otitis media); fluid accumulation seen occasionally in middle ear after resolution of

OM (need not be recurrent OM); Tx is observation, as will usually self-resolve in 4-8 weeks.

- 10-month-old female + fussy + fever + no movement of left tympanic membrane on pneumatic

otoscopy; Dx? à answer = otitis media; the wrong answer is otitis media with effusion; immobility of

tympanic membrane = most sensitive finding for otitis media – i.e., if a Q tells you the tympanic

membrane is mobile, the Dx is not OM.

- 7M + otoscopy reveals squamous proliferation seen behind tympanic membrane; Dx + Tx? à

cholesteatoma; will gradually grow and invade inner ear, causing irreversible hearing loss; Tx is

surgical excision.

- 2F + fever 103 + tugging on pinna + pinna is displaced upward and outward + tenderness of mastoid

process; next best step? à answer = “CT of the temporal bone”; sounds incredibly wrong to do a CT

on a kid in this scenario, but this is the answer on the Peds NBME; mastoiditis (malignant otitis

externa) is sometimes associated with a temporal bone fluid collection that must be drained to

prevent brain abscess; MRI or CT must be done; x-ray is the wrong answer; fluid collection must be

drained as best answer; IV broad-spectrum antibiotics are then administered.

- 30-month-old male + knows 100 words but does not use them in phrases + parents concerned he’s

not developing verbally as well as older siblings did + other milestones met appropriately; next best

step? à answer = audiometry à hearing deficits are common cause of language delay.

- 6M + bilateral pneumonia + no other risk factors; Dx? à answer = Mycoplasma.

- 6M + lobar pneumonia + no other risk factors; Dx? à answer = Strep pneumoniae.

- 6M + lobar pneumonia + CXR shows interstitial markings; Dx? à answer = Mycoplasma (the word

“interstitial” wins over lobar distribution).

- 2F + lobar pneumonia + no other risk factors + Strep pneumo not listed as an answer choice; Dx? à

answer = Staph aureus (Peds NBME).

- 5F + recent convalescence from influenza infection + now has bacterial pneumonia; organism? à

answer = Staph aureus (answer following influenza infection).

- 17F + painless lateral neck mass + mediastinal mass; Dx? à Hodgkin lymphoma.

MEHLMANMEDICAL.COM 36
MEHLMANMEDICAL.COM

- 8M + puffiness around eyes and ankles; vignette gives you no other information; Dx? à answer =

minimal change disease (lipoid nephrosis) à 2CK-level Qs will often not mention anything about

preceding viral infections à classically periorbital and pedal edema +/- ascites; urine does not contain

blood as this is nephrotic syndrome; LM shows nothing; EM shows effacement of podocytic

processes.

- 8M + nephrotic syndrome + fever 103F + abdominal fluid wave + diffuse abdominal pain; next best

step? à answer = abdominal paracentesis à Dx = spontaneous bacterial peritonitis secondary to

ascites caused by [probably] minimal change disease; do gram stain of peritoneal fluid + check for

>250 WBCs/hpf; Tx with third-gen cephalosporin (no hard rule, but proclivity for cefotaxime age <6;

ceftriaxone >6).

- 44M + Hodgkin lymphoma + nephrotic syndrome; Dx? à answer = minimal change disease; I’m

including this in this peds discussion because this is HY and everyone gets it wrong; “Wait wtf? MCD is

peds; how does that make sense for 44M? Erratum?” à MCD is also answer for nephrotic syndrome

in Hodgkin.

- 8M + sickle cell + nephrotic syndrome; Dx? à answer = focal segmental glomerulosclerosis (FSGS).

- 8M + sickle cell + dark urine; Dx? à answer = renal papillary necrosis.

- 12F + high BUN and Cr + epistaxis + low Hb + normal platelet count; Dx? à answer = “acquired

platelet dysfunction” (uremic platelet dysfunction) à qualitative, not quantitative, platelet disorder

caused by high BUN decreasing platelet efficacy; low EPO is wrong answer; low Hb due to epistaxis,

not low EPO. If low EPO is the answer (i.e., anemia of chronic disease secondary to chronic renal

failure), then the patient wouldn’t have epistaxis.

- 12M + red urine 1-3 days after upper respiratory tract infection (URTI) à IgA nephropathy, not PSGN;

can also get IgA nephropathy from GI infections.

- 12M + red urine 1-2 weeks after URTI or skin infection à PSGN à can get it from Group A Strep skin

infections

- 10F + sore throat two weeks ago + red urine + peripheral edema + HTN; what’s the mechanism for the

HTN? à answer = “volume overload” à microvascular disturbance from the glomerulonephritis à

RAAS stimulation.

MEHLMANMEDICAL.COM 37
MEHLMANMEDICAL.COM

- 6F + red urine + abdo pain + arthralgias + violaceous lesions on buttocks + thighs; Dx? à Henoch-

Schonlein purpura; red urine = IgA nephropathy à HSP is tetrad of 1) IgA nephropathy, 2) palpable

purpura, 3) arthralgias, 4) abdo pain.

- 16F + motor vehicle accident (MVA) + bruising/pain over flank + red urine; next best step? à answer

= CT of abdomen with contrast to evaluate for renal injury; sound wrong, as you’d say “Really? A CT

with contrast for suspected renal injury? Yeah. Ultrasound is the wrong answer.

- 16F + MVA + bruising/pain over flank + urine shows no gross blood + urinalysis performed for what?

à answer = urine blood à presence of blood suggests renal injury in this setting.

- 16F + MVA + bruising/pain over flank + urine shows no gross blood + urinalysis shows no blood; next

best step? à answer = “no further diagnostic studies indicated” à if accident + bruising/pain over

flank, first look for gross blood; if positive, go straight to CT; if negative, do urinalysis looking for

blood; if positive, do CT; if negative, no further studies indicated. This concept is HY on surgery

NBMEs but also shows up on Peds form.

- 17F + HTN + low K + high bicarb; next best step? à answer = MR angiogram of renal arteries; Dx =

fibromuscular dysplasia; another answer = “increased renin production.”

- 13F has never had a period + has suprapubic mass + nausea + vomiting; next best step in Mx? à

answer = do beta-hCG à she’s pregnant; this is HY. Correct, girls can get pregnant without ever

having had a period à must rule out.

- 14F has massive unilateral breast mass + mom is freaking out bc her sister died of breast cancer à

answer = follow-up in six months à virginal breast hypertrophy is normal during puberty.

- 15M has unilateral mass behind his nipple +/- tenderness of it à answer = reassurance à physiologic

gynecomastia of puberty (higher androgens are aromatized to estrogens).

- 14F + Tanner stage 3; which of the following is true? à answer = menarche is imminent à USMLE

asks this Q straight up and it’s exceedingly HY and frequent.

- 17F + often really pad period pain + needs to miss class sometimes because it’s so bad + physical

exam is normal à answer = primary dysmenorrhea = prostaglandin hypersecretion (PGF2alpha) à

give NSAIDs.

- 17F + 2-day Hx of right-sided pelvic pain + vitals WNL + beta-hCG negative + USS shows 3.5cm simple

cyst; next best step in Mx? à answer on Peds NBME = observation (should be noted OCPs is not an

MEHLMANMEDICAL.COM 38
MEHLMANMEDICAL.COM

answer here; on one of the Obgyn NBME forms, observation is wrong; correct answer is “oral

contraceptive pills and re-ultrasound in 6 weeks) à therefore, choose the latter over observation if

both listed (and no contraindications to OCPs); if only observation is listed, go with that.

- 14F + never had menstrual period + one-wk Hx of constant, severe pelvic pain + 6-month Hx of

intermittent pelvic pain + BP of 90/50 + bluish bulge in upper vagina; Dx? à hematometra à

imperforate hymen with blood collection in the uterus à vagal response causes low BP à Tx =

cruciate incision of the hymen.

- 14F + never had menstrual period + 6-month Hx of intermittent pelvic pain + BP normal + bluish bulge

in upper vagina; Dx? à hematocolpos à blood collection in the vaginal canal, but not backed up to

the uterus like hematometra à Tx = cruciate incision of the hymen.

- 16F + vagina ends in blind pouch + no cervix or uterus + absent/scanty pubic and axillary hair + Tanner

stage 4; Dx? à answer = androgen insensitivity syndrome à next best step in Mx = karyotyping

(46XY).

- 16F + vagina ends in blind pouch + no cervix or uterus + coarse pubic and axillary hair + Tanner stage

4; Dx? à answer = Mullerian agenesis à next best step in Mx = karyotyping (46XX).

- 15M + 4’10” + normal hormone levels + bone parents normal height; next best step? à answer = get

bone age à if bone age < chronologic age, Dx = constitutional growth delay (growth curve shifted to

the right; will eventually become average height); if bone age = chronologic age, patient has genuine

short stature. Variant of constitutional growth delay Q is: rather than saying patient’s bone age is low,

Q will say he is Tanner stage 2 (implying he hasn’t yet gone through puberty so will catch up).

- 16F + 4’11” + Tanner stage 2 + scattered nevi + webbed neck + hasn’t yet had menstruation + bone

age = chronologic age; Dx? à answer = Turner syndrome (45XO); genuine short stature in Turner; LH

and FSH are both high; can achieve eventual pregnancy via IVF (surrogate); webbed neck = cystic

hygroma (lymphatic insufficiency).

- 15-month-old boy + 3 months ago was 25th %tile for length and weight + today is still 25th %tile for

length but 10th %tile for weight; next best step in evaluation? à answer = “recommend a diet diary

with calorie counts.”

- 13F + went running outside in hot weather + very exhausted + body temp 102.2F + pulse 100; Dx? à

answer = heat exhaustion (high body temp + no end-organ damage/failure); in contrast, heat stroke =

MEHLMANMEDICAL.COM 39
MEHLMANMEDICAL.COM

high body temp + end-organ damage; acanthocytes in blood smear = liver failure (HY for heat stroke);

acanthocytes not limited to abetalipoproteinemia.

- Renal cancer in children? à Wilms tumor.

- 3M + painless flank mass + no other info; Dx? à Wilms tumor à answer on peds form =

“ultrasonography of the abdomen.”

- 6M + painless flank mass + seizure + MRI of brain shows periventricular nodules; Dx? à Tuberous

sclerosis with renal angiomyolipoma.

- Anything else about Wilms tumor? à increased incidence in horseshoe kidney in Turner syndrome.

- Horseshoe kidney key point? à not only increased risk of Wilms tumor, but the kidney gets caught

under the IMA. USMLE Step 1 likes this factoid for some dumb reason.

- What is WAGR complex? à Wilms tumor, Aniridia, Genitourinary malformation, Retardation; caused

by WT1 gene mutation.

- What is Denys-Drash syndrome? à Wilms tumor + male pseudohermaphroditism (secondary sex

characteristics are female; primary sex characteristics are male); caused by WT1 gene mutation.

- Neonate born at 4800g + macroglossia + omphalocele + visceromegaly; child is most likely to develop

what? à answer = hypoglycemia à Beckwith-Wiedemann syndrome? à Wilms tumor + neonatal

hypoglycemia + macrosomia / macroglossia + hemihypertrophy +/- omphalocele; caused by WT2

gene mutation.

- 1M + 2-month Hx of increasing weakness and difficulty feeding + examination shows profound

hypotonia and tongue fasciculations + absent reflexes; Dx? à answer = “progressive infantile spinal

muscular atrophy” à characterized by decreased reflexes, muscle weakness and decreased tone, and

tongue fasciculations.

- Comment on AR vs AD polycystic kidney disease. à ARPKD is chromosome 6 and a pediatric

condition; it’s associated with hepatic fibrosis. ADPKD is chromosome 16 and presents in 30s or 40s;

associated with HTN and circle of Willis saccular (berry) aneurysms.

- 4F + third case of pyelonephritis + high creatinine; most likely cause of high Cr? à “congenital urinary

tract obstruction” or “vesicoureteral reflux”; Dx is chronic pyelonephritis à caused by recurrent acute

pyelonephritis à chronic pyelo is characterized by “thyroidization of kidney” + blunting and scarring

of renal calyses.

MEHLMANMEDICAL.COM 40
MEHLMANMEDICAL.COM

- 9F + Hx of recurrent pyelonephritis and simple UTIs + BP of 160/100; next best step in Mx? à answer

= “measurement of serum urea nitrogen and creatinine concentrations” à likely chronic

pyelonephritis leading to renal scarring + disruption of renal microvasculature + upregulation of RAAS.

- 6M + dancing eyes + HTN + lesion visualized in posterior mediastinum on CXR; Dx? à neuroblastoma;

students says wtf? à can occur anywhere in the median sympathetic chain, although classically intra-

abdominal. Dancing eyes = opsoclonus-myoclonus syndrome.

- Neuroblastoma; how to Dx? à answer on NBME is “urinary homovanillic acid (HVA) and

vanillylmandelic acid (VMA); mIBG scan may also be used; n-myc gene.

- 5M + 4-day Hx of increased shortness of breath + CXR shows mass in superior mediastinum +

thoracentesis yields 400mL serosanguinous fluid; most likely cause of pleural effusion? à answer =

“malignant pleural effusion” à T cell ALL (mediastinal mass leading to SVC-like syndrome).

- Pathologic jaundice in peds = must have at least one of the following:

o Any jaundice on the first day of life (first 24 hours of life), period = pathologic.

o Jaundice present after one week if term, or after two weeks if preterm = pathologic.

o Total bilirubin >15 mg/dL.

o Direct bilirubin >10% of total bilirubin, even if total bilirubin is <15 mg/dL.

o Rate of change of increase in bilirubin >0.5 mg/dL/hour.

- Tx for pathologic jaundice? à choose “phototherapy” first, followed by “exchange transfusion”; some

literature makes a case for IVIG after phototherapy, but USMLE has exchange transfusion as correct,

without listing IVIG.

- Neonate + total bilirubin of 14 mg/dL + direct bilirubin of 13 mg/dL; Dx + next best step in Mx + Tx? à

answer = biliary atresia; next best step = liver biopsy; Tx = liver transplant. Notice that total bilirubin is

OK in this particular scenario but direct is severely pathologic because >10% of total (i.e., >1.4 mg/dL).

- Neonate + total bilirubin of 20 mg/dL, with all of it unconjugated + continually worsens despite Mx;

Dx? à answer = Crigler-Najjar à absence of UDP-glucuronosyltransferase (conjugating enzyme); Tx =

liver transplant; type II of the disease is deficiency, rather than absence of enzyme, and can be Mx

with phenobarbital, which leads to a reduction in bilirubin.

- Difference between breastmilk jaundice vs breastfeeding jaundice?

MEHLMANMEDICAL.COM 41
MEHLMANMEDICAL.COM

o Breastmilk jaundice = due to beta-glucuronidase in breast milk, which leads to de-

conjugation of intestinal bilirubin + increased enterohepatic circulation –> jaundice that

starts on day 3-5 and peaks at 2-3 weeks –> Tx = stop breastfeeding for ~48 hours (and do

bottle feeding), which leads to a rapid decrease in bilirubin; once breastfeeding is resumed,

bilirubin might rise, but not back to pathologic levels.

o Breastfeeding jaundice = insufficient feeding (e.g., failure of suckling, etc.) + decreased milk

intake leads to reduced intestinal clearance of bilirubin –> increased enterohepatic

circulation –> jaundice that peaks at 3-5 days –> Tx = formula feeding (fluid + caloric

supplementation).

- Neonate born with jaundice (pathologic) + mom is O+ blood type + neonate is A or B blood type; Dx?

à hemolytic disease of the newborn (ABO type) à mothers with O blood type will have fractional

IgG (instead of all IgM) against A and B antigens à crosses placenta à fetal hemolysis à severity

highly variable.

- Neonate born with jaundice (pathologic) + born from 2nd pregnancy of Rh negative mom; Dx? à

answer = hemolytic disease of the newborn (Rh type) à presumably mother made antibodies against

fetal Rh antigen from prior pregnancy following mixing of circulations.

- Neonate + ambiguous genitalia + hypoglycemia + high potassium + low blood pressure; Dx? à answer

= congenital adrenal hyperplasia (CAH) due to 21 hydroxylase deficiency à low glucose because

cannot make cortisol or 11-deoxycortisol; high potassium because cannot make aldosterone or

corticosterone; ambiguous genitalia because aldosterone and cortisol precursors (pregnenolone and

progesterone, plus 17-OH types) are shunted to DHEA-S and androstenedione production;

androstenedione can be converted peripherally into testosterone; DHEA-S is adrenal-specific

androgen. 21 hydroxylase deficiency is most common cause of CAH.

- Neonate + ambiguous genitalia + potassium not high + BP normal + DHEA-S elevated; Dx? à CAH due

to 11 hydroxylase deficiency à patient can still make corticosterone (precursor to aldosterone) and

11-deoxycortisol (precursor to cortisol), so still has mineralo- and glucocorticoid synthesis, despite not

full-blown aldosterone and cortisol.

- Neonate + potassium low + glucose low + DHEA-S low; Dx? à CAH due to 17 hydroxylase deficiency

à potassium wasting from high aldosterone; cannot effectively make zona fasciculata (cortisol layer)

MEHLMANMEDICAL.COM 42
MEHLMANMEDICAL.COM

and reticularis (androgen layer) precursors and products; pregnenolone shunted purely to

aldosterone in zona glomerulosa.

- 7F + progressive fatigue past 6 months + low BP + darkening of skin; Q asks “most effective test to

determine diagnosis and management?” à answer on Peds NBME = “plasma cortisol”; Dx = Addison.

On IM NBME the answer is “ACTH stimulation test” for next best step in adult with Addison.

Literature says random plasma cortisol >25mcg/dL excludes Addison. It may be presumed that

random cortisol may be performed prior to the ACTH stimulation test, since the latter will entail serial

measurements (i.e., over time) of serum cortisol anyway.

- Normal vaccine schedule by timeline:

o At birth:

§ Hepatitis B

o At 2, 4, 6 months (3 doses total):

§ Hepatitis B (new 2020 guidelines: skip at 4 months; so give at birth, 2 months, and 6

months)

§ Strep pneumo PCV13

§ Rotavirus (oral; live-attenuated)

§ Polio (Salk; IM killed)

§ TDaP (tetanus, diphtheria, pertussis)

§ Haemophilus influenzae type B

o At 1 year:

§ Strep pneumo (4th dose)

§ Haemophilus influenzae type B (4th dose)

§ MMRV (mumps, measles, rubella, varicella; 1st dose)

§ Hepatitis A (1st dose)

o At 18 months:

§ Hepatitis A (2nd dose)

§ TDaP (4th dose)

o At 4-6 years:

§ TDaP (5th dose)Polio (4th dose)

MEHLMANMEDICAL.COM 43
MEHLMANMEDICAL.COM

§ MMRV (2nd dose)

o Age 9-45:

§ HPV (2 doses if first given age 9-14; 3 doses if first given age 15+)

o School age 11-13:

§ TDaP (6th dose)

§ Meningococcal (1st dose)

o School age 16:

§ Meningococcal (2nd dose)

- Vaccines by organism:

o HepB: birth, 2 + 6 months (3 doses)

o Rotavirus (oral; live-attenuated): 2, 4, 6 months (3 doses)

o Polio (Salk; IM killed): 2, 4, 6 months; 4-6 years (4 doses)

o Strep pneumoniae PCV13: 2, 4, 6 months; 1 year (4 doses)

o Haemophilus influenzae type B: 2, 4, 6 months; 1 year (4 doses)

o Tetanus, diphtheria, pertussis (TDaP): 2, 4, 6 months; 18 months; 4-6 years; 11-13 years (6

doses)

o Hepatitis A: 1 year; 18 months (2 doses)

o Mumps, measles, rubella, varicella (MMRV): 1 year; 4-6 years (2 doses)

o HPV: 9-45 years (2 doses if first given age 9-14; 3 doses if first given age 15+)

o Meningococcal: 11-13 years; 16 years (2 doses)

- Special notes on vaccines:

o Influenza

§ Two options: either killed (IM) or live-attenuated (intranasal)

§ Only give in fall or winter (if they say, e.g., April, the answer is don't give)

§ Give IM killed starting at age 6 months, then every year (annually)

§ Live-attenuated may be given annually in the fall or winter only to non-pregnant,

non-immunocompromised persons age 2-49

o Strep pneumoniae (additional administrations)

§ Give one dose of PCV13 to all persons age 65, then PPSV23 6-12 months later

MEHLMANMEDICAL.COM 44
MEHLMANMEDICAL.COM

§ To patients with asplenia, sickle cell, and cochlear implants, give PPSV23 6-12

months after last dose of PCV13.

- Neonate + diffuse pink body rash + desquamation of palms and soles; mechanism? à answer =

exotoxin à Dx = Staphylococcal scalded skin syndrome due to exfoliative toxin (produced by ~5% of

community staph).

- Neonate born at home + umbilical cord cut with kitchen knife and tied with twine + trismus; Dx? à

answer = tetanus; Tx = antitoxin; trismus = lockjaw (HY “vocab word” for USMLE); don’t confuse this

with tenesmus, which is the intractable feeling of needing to defecate; C. tetani classically enters

through umbilical stump.

- Neonate born at home + bleeding from umbilical stump; BT, aPTT, PT? à answer = vitamin K

deficiency à BT normal; PT and aPTT both elevated; vitamin K deficiency due to sterile bowel in the

absence of injection at birth.

- 6M + crops of vesicles on the trunk at different stages of healing; Dx? à answer = varicella (VZV) à

presumably not previously vaccinated; know this description.

- Neonate dies in utero + mother owns a deli; Dx? à granulomatosis infantiseptica (severest form of

Listeriosis); deli meats and soft cheeses are sources during pregnancy.

- Neonate born at term + fever + high WBCs (65% neutrophils) + CSF shows trace cells (90%

lymphocytes); Dx? à answer = Group B Strep (GBS; Strep agalactiae) sepsis; students can get

confused over the CSF findings, but this apparently is a normal finding; 65% neutrophils is slight left-

shift for bacterial infection.

- Neonate + GBS sepsis; Tx? à answer = ampicillin + gentamicin; wrong answer is ceftriaxone +

vancomycin.

- 6M + diagnosed with ALL + on chemotherapy + vesicles beneath the scapula; Dx? à herpes zoster

(immunosuppressed; you need to know pediatric shingles “is a thing” – i.e., it exists).

- Neonate + microphthalmia + limb deformities + zig-zag skin lesions; Dx? à congenital varicella

syndrome.

- Neonate + intracranial/periventricular calcifications + hydrocephalus + chorioretinitis; Dx? à answer

= congenital toxoplasmosis (know this triad).

MEHLMANMEDICAL.COM 45
MEHLMANMEDICAL.COM

- Neonate + patent ductus arteriosus +/- deafness +/- cataracts; Dx? à answer = congenital rubella

syndrome.

- Neonate + saddle nose + saber shins + Hutchinson teeth +/- deafness +/- cataracts; Dx? à answer =

congenital syphilis.

- Neonate + intracranial calcifications + deafness +/- hepatosplenomegaly +/- blueberry muffin rash;

Dx? à answer = congenital CMV; calcifications are non-specific, but the triad is not for toxo, so you

can rule that out; deafness very HY for CMV, although also not specific; CMV diagnoses from vignette

after you’ve eliminated the others, as per the descriptions for toxo, syphilis, rubella, and varicella.

- 16F + sexually active; what do we do – Pap smears and/or STI checks? à Don’t do Pap smears. Start

them at age 21 regardless of age of onset of sexual activity. However, yes, check for chalmydia and

gonorrhea starting from age of sexual onset. Also make sure she has HPV vaccine if not received

already.

- 16F + mucopurulent discharge + adnexal pain; Tx? à answer = IM ceftriaxone, PLUS either oral

azithromycin (one-off stat dose) or oral doxycycline (BID for a week) à always cotreat for chlamydia

and gonorrhea; if patient is septic, must give IV Abs (they ask this contrast on the NBME); Dx here is

pelvic inflammatory disease (PID).

- 17F + completely asymptomatic + male partner recently tested positive for gonorrhea; tests are

ordered for chlamydia and gonorrhea; do we treat her or not treat her? à answer = “intramuscular

ceftriaxone and oral doxycycline”; the wrong answer is “no treatment indicated because the patient is

asymptomatic.”

- 16M + redness of both eyes + rash over extremities + mucopurulent urethral discharge; Dx? à

answer = reactive arthritis à due to chlamydia; gonococcus does not cause reactive arthritis.

- 16M + sexually active + left knee pain + fever + S. aureus not listed as answer; Dx? à answer =

gonococcal arthritis.

- 16M + polyarthritis + positive Finkelstein test + cutaneous papules over the wrist; Dx? à answer =

gonococcal arthritis à will present either as monoarthritis of large joint, such as the knee, or as a

triad of polyarthritis, tenosynovitis, and cutaneous papules.

MEHLMANMEDICAL.COM 46
MEHLMANMEDICAL.COM

- 16F + thin, grey, watery discharge + Dx is made by wet mount; Dx? à answer = bacterial vaginosis

(Gardnerella); clue cells seen on wet mount (squamous cells covered in bacteria); KOH whiff test with

fishy odor also classic; Tx with topical metronidazole gel.

- 16F + yellow-green discharge + Dx made by wet mount; Dx? à answer = trichomonas vaginalis à wet

mount shows flagellated protozoa; strawberry cervix (punctate hemorrhages) or erythematous

vaginal canal might be seen; Tx with topical metronidazole for patient and partner (high risk of

reinfection).

- 16F + vesicular lesions on vulva + stinging/burning pain; Q asks “What is the most likely trajectory of

these lesions?” à answer = “gradually disappear over one week” à can agree or disagree with this

exact timeline, but it’s on the NBME; wrong answer is “gradually worsen then disappear over three

weeks”; Tx is clearly with oral acyclovir (or oral valacyclovir, but NBME will often have “oral acyclovir”

as answer); topical acyclovir is the wrong answer (even if co-prescribed in real life, USMLE wants oral

over topical if both listed).

- 17M + painless papule that ruptures into painless crater on penile base; next best step? à answer =

darkfield microscopy à Dx = primary syphilis; VDRL/RPR can be performed but sometimes negative in

primary syphilis; answer on one NBME form = “spirochete.”

- 17M + sexually active + maculopapular rash on upper back + palms and soles rash; next best step? à

answer = VDRL/RPR, followed by fluorescent treponemal antibody; Dx = secondary syphilis.

- 18M + positive Romberg sign + sexually active; Dx? à answer on Neuro NBME = neurosyphilis à

sounds wrong at first because you say, “Wait, really? Neurosyphilis in someone as young as 18?” But

neurosyphilis can occur at any stage of syphilis, and often appears during secondary. Positive

Romberg refers to tabes dorsalis. Can also present with Argyll-Robertson pupil (accommodates but

doesn’t react).

- 17M + went to Uganda over summer vacation + has painful crater on base of penile shaft; Dx? à

answer = haemophilus ducreyi à Tx = azithromycin.

- 17M + skin-colored papules on penile shaft; Tx? à answer = podophyllum resin à Tx for HPV (this is

also the answer on the Obgyn NBME for vulvar condylomata acuminata).

- 3F + purulent bloody discharge on underwear; Dx? à answer = vaginal foreign body.

MEHLMANMEDICAL.COM 47
MEHLMANMEDICAL.COM

- 17F + BMI 28 + erythematous and itchy vulva + no mention of discharge but Q asks for organism; Dx?

à answer = candidiasis; classically curd-like, white discharge, but one 2CK-level Q doesn’t mention

this; Tx = topical nystatin first or oral fluconazole (topical nystatin usually tried first).

- 7M + 6-wk Hx of headaches and difficulty walking + bilateral papilledema; Dx? à answer =

medulloblastoma à need to know this is cerebellar, not medullar; medulloblastoma of cerebellar

vermis à truncal ataxia; morning vomiting classic, but papilledema = increased intracranial pressure.

- 4M + morning vomiting + MRI shows infratentorial lesion that’s mixed hyper- and hypointense; Dx?

à pilocytic astrocytoma à mixed solid and cystic grossly à most common peds primary brain tumor.

- 4M + upward-gaze palsy; Dx? à Paranaud syndrome due to pinealoma.

- 4M + bitemporal hemianopsia + MRI shows calcification; Dx? à answer = craniopharyngioma à most

common pituitary tumor in peds (prolactinoma in adults); some sources argue it’s not a true pituitary

tumor since it’s derived from Rathke pouch (roof of primitive pharynx); can calcify and have motor-oil

appearance; frequently recurs following surgery.

- 6M + Q shows you image where kid looks like he got hit by a softball on his eye; Dx? à answer =

orbital cellulitis à must give IV antibiotics; in contrast, pre-septal cellulitis is less severe and can be

treated with orals.

- 6M + painful bump on eyelid; Dx? à hordeolum (aka stye); S. aureus infection; Tx with warm

compresses.

- 6M + painless bump on eyelid; Dx? à chalazion (blocked oil duct); Tx with warm compresses.

- 6M + painless bump on eyelid + warm compresses not working; Dx? à dermoid cyst of eyelid; Tx is

surgical.

- Comment on neonatal conjunctivitis (ophthalmia neonatorum):

o Chemical (first 24 hours of life)

§ Frequently caused by silver nitrate eyedrops (gonococcal prophylaxis), although this

is more historical and has been replaced by topical erythromycin drops (most states

require by law).

§ Erythromycin (and silver nitrate) drops are only prophylaxis against gonococcus;

they are not prophylaxis against chlamydia.

MEHLMANMEDICAL.COM 48
MEHLMANMEDICAL.COM

§ Best way to prevent chlamydial and gonococcal ophthalmia neonatorum is effective

Tx in the mother prior to delivery.

o Gonococcal (2-5 days of life)

§ Purulent discharge; Tx with IM cefotaxime (choose over ceftriaxone for neonates).

o Chlamydial (5-14 days of life)

§ Watery or purulent discharge.

§ Can classically lead to chlamydia trachomatis D-K pneumonia (i.e., not chlamydia

pneumoniae); can drain through nasolacrimal duct down into the nasopharynx and

lungs; diffuse crackles auscultated 1-2 weeks after the eye infection.

§ Tx with oral erythromycin (topical not only less effective, but also does not account

for any organism having entered the nasopharynx and/or lungs).

- 4M + goes to daycare + red, itchy eyes; Dx? à answer = adenovirus (most common cause of viral

conjunctivitis).

- 4M + goes to daycare + red, itchy eyes; the organism most likely responsible can also cause what? à

answer = hemorrhagic cystitis (red urine) à adenovirus.

- 6M + playing with firecracker + pain and tearing of eye + ocular movements full + visual acuity 20/20;

next best step? à answer = “fluorescein instillation of eye”; Dx = corneal abrasion; other vignette will

say kid playing in sandbox or near dad in metal shop; corneal abrasion shows up green on fluorescein

staining (normal areas are blue); cornea heals super-fast + patient will normally have incessant

tearing and scratchy sensation for a day and then wake up next morning just fine.

- 6M + itchy eye + vesicles present infraorbitally + fluorescein staining of eye shows dendritic pattern;

Dx? à answer = herpes keratitis; similar Dx is herpes zoster (VZV) ophthalmicus

(immunocompromised if child).

- 6M + being treated with chemo for ALL + vesicles around ear + Bell palsy; Dx? à herpes zoster oticus,

aka Ramsey-Hunt syndrome type II (VZV).

- Newborn + cystic dilation of 4th ventricle + absent cerebellar vermis; Dx? à Dandy-Walker syndrome.

- Newborn + myelomeningocele + downward herniation of cerebellar vermis; Dx? à Arnold-Chiari

malformation (term only applies to Chiari type II).

MEHLMANMEDICAL.COM 49
MEHLMANMEDICAL.COM

- 18M + bilateral loss of pain and temperature sensation; Dx? à syringomyelia (can be seen secondary

to syrinx from Chiari type I [less severe and presents later than type II]).

- Newborn + prominent occiput + micrognathia + overlapping fingers; Dx? à answer = Edward

syndrome (Trisomy 18).

- 8-month-old boy + 3rd-centile for weight + slanted palpebral fissures + epicanthal folds + single palmar

crease + thin upper lip with a “fish mouth” appearance + indistinct nasal philtrum; Dx? à answer on

Psych NBME = fetal alcohol syndrome (FAS), not Down syndrome; everyone says wtf about this

question, so what I tell my students is: if Q sounds like Down syndrome but they mention anything

about the philtrum (i.e., long, smooth, indistinct, etc.), the answer is FAS, not Down.

- 6M + IQ of 60 + small for age + born to female age 41 + no other information given; Q asks: most

likely cause of mental retardation? à answer = Down syndrome, not FAS; although FAS is most

common cause of MR overall, two points: 1) if they want FAS, they’ll mention the philtrum as per

above, and 2) most common cause of MR over the age of 40 is Down, not FAS.

- 8M + prominent jaw + protruding ears + IQ of 65; most likely explanation? à “Fragile site on the X

chromosome” à Fragile X (CGG TNR disorder).

- 7M + prominent ears + flattened nasal bridge + long philtrum + low IQ; Dx? à FAS, not Fragile X; the

Q mentions the philtrum.

- 4F + wringing of the hands + putting objects in her mouth + less eye contact; Dx? à answer = Rett

syndrome; only seen in girls; hyperoralism may reflect cognitive regression (babies put everything in

their mouths).

- 5M + talks to imaginary friends; Dx? à age-appropriate behavior.

- Newborn + multiple joint contractures; Dx? à answer = arthrogryposis (just be aware of this).

- Newborn + clubbed feet; Tx? à answer = serial casting in order to reposition feet (followed by minor

surgical Tx).

- 16M + belligerent + nystagmus; Tx? à answer = lorazepam (benzo for PCP toxicity).

- 16M + mutism + constricted pupils; Dx? à answer = PCP intoxication; student says “Wtf? I thought

there was belligerence / bellicosity?” This is one of the presentations on the NBME; if you don’t

believe me, Google “Mutism constricted pupils PCP.”

MEHLMANMEDICAL.COM 50
MEHLMANMEDICAL.COM

- 16M + found on floor in school bathroom + sluggish + vitals and pupils normal; Dx? à answer =

butane toxicity (inhalant toxicity); student had this on actual USMLE.

- 14M + ataxia + cognitive decline; Dx? à glue toxicity, not EtOH.

- 14M + dilated pupils + tachycardia; Tx? à answer = benzo (cocaine).

- 3M + microcytic anemia + one-month Hx of poor coordination, anorexia, irritability, and apathy; what

could have prevented this? à answer = “elimination of lead from the child’s home”; Tx = succimer for

kids (if lead levels >44 ug/dL); lead can cause microcytic anemia.

- 14M + dilated pupils + visual hallucinations + staying up all night; Tx? à answer = benzo

(amphetamine).

- 2M + blisters on tongue/mouth + next best step? à answer = “fiberoptic endoscopy” à Dx? =

exposure to drain cleaner.

- 16F + found by mom 20 minutes ago + consumed bottle of aspirin in suicide attempt + lethargic; what

is the acid-base disturbance she has? à answer = mixed metabolic acidosis-respiratory alkalosis à

the presumption is we don’t know how long ago she ingested + the lethargy might imply the acidosis

has taken effect (acutely, salicylate toxicity causes isolated respiratory alkalosis); another Q wants you

to select the literal values for mixed: normal Na, Normal K, normal Cl, low bicarb, low CO2, normal or

low pH (the metabolic acidosis will eventually “win” and the patient will become severely acidotic,

but rarely a Q might give you a pH in the lower end of normal range if patient is in transition).

- Tx for aspirin toxicity? à answer = bicarb à increases excretion via urinary alkalinization à cannot

reabsorb the oxalate (deprotonated, negative charge) form through the tubular walls.

- 16F + long Hx of depression + taking new medication + large pupils + dry skin + ECG shows QT

changes; Tx? à answer = sodium bicarb à Tx for TCA toxicity à causes dissociation of TCA from

myocardial sodium channels à TCA is basic, not acidic (in contrast to salicylates), so bicarb actually

decreases urinary excretion, but the mechanism as an antidote is different; TCAs cause CCC à coma,

convulsions, cardiotoxicity + triad of anticholinergic, anti-alpha-1-adrenergic, and anti-H1-

histaminergic side-effects.

- Difference between caput succedaneum and cephalohematoma?

o Caput succedaneum is poorly defined soft tissue edema on the scalp; caused by pressure of

fetal scalp against cervix during parturition, leading to transient decreased blood flow and

MEHLMANMEDICAL.COM 51
MEHLMANMEDICAL.COM

reactive edema; crosses suture lines; can be purplish in color similar to cephalohematoma

(i.e., don’t use color to distinguish); complications rare; disappears in hours to few days.

o Cephalohematoma is well-defined, localized, fluctuant swelling; caused by subperiosteal

hemorrhage (answer on Peds NBME = “blood under periosteum of parietal bone”); does not

cross suture lines; may be associated with underlying skull fracture, clotting disorders,

jaundice; disappears in weeks to months.

- Classic five findings that are seen in neonates of diabetic moms?

o Hypoglycemia à high neonatal insulin secondary to high maternal glucose crossing placenta;

insulin does not cross the placenta; if they tell you the neonate has hypoglycemia and then

ask for the mechanism, the answer is “increased insulin secretion”; hypoglycemia is also the

answer if the Q describes the neonate as jittery.

o Hypocalcemia à answer if they mention QT interval changes.

o Hypomagnesemia à just be aware this is possible; hypomagnesemia can be a cause of

hypocalcemia and/or hypokalemia refractory to supplementation (i.e., non-responsive to

supplementation); this is more established in adult alcoholics who are dietary Mg deficient

but can theoretically be applied to pediatrics.

o Hyperbilirubinemia à increased risk for jaundice and kernicterus.

o Polycythemia à increased RBC turnover is normal in neonates as fetal Hb declines and adult

Hb rises; physiologic jaundice Q will often give Hct in the low-50s% as normal; polycythemia

in neonates is defined as venous Hct >65%.

o Should be noted that babies born to diabetic moms are often large (>4000g; macrosomia).

- Glycogen storage diseases (2CK- / Peds shelf-sufficient detail):

o Type I (von Gierke)

§ Deficiency of glucose-6-phosphatase; autosomal recessive (AR).

§ Lactic acidosis, hypoglycemia, jaundice, hepatomegaly.

o Type III (Cori)

§ Deficiency of alpha-1,6-gluocosidase; AR.

§ No lactic acidosis (in contrast to von Gierke).

o Type II (Pompe)

MEHLMANMEDICAL.COM 52
MEHLMANMEDICAL.COM

§ Deficiency of alpha-1,4-glucosidase; AR.

§ Cardiomyopathy.

o Type V (McArdle)

§ Deficiency of myophosphorylase (glycogen muscle phosphorylase); AR.

§ Severe cramping after intense exercise, but normal serum lactate.

§ Patient usually adolescent or adult.

§ Can lead to rhabdomyolysis + increased serum CK.

- Lysosomal storage diseases (2CK- / Peds shelf-sufficient detail):

o Gaucher (AR)

§ Deficiency of glucocerebrosidase (glucosylceramidase) + buildup of

glucocerebroside.

§ This is the answer if they mention avascular necrosis of the hip in the context of a

lysosomal storage disease.

o Fabry (XR)

§ Deficiency of alpha-galactosidase A + buildup of ceramide trihexoside.

§ Angiokeratomas (clusters of violaceous-reddish spots on trunk, limbs, etc.).

§ Cardiomyopathy and/or renal insufficiency.

o Tay-Sachs (AR)

§ Deficiency of hexosaminidase A + buildup of GM2 ganglioside.

§ Cherry-red spot on macula + no hepatosplenomegaly.

§ Question tends to mention neuronal degeneration and/or vison loss age <2.

o Niemann-Pick (AR)

§ Deficiency of sphingomyelinase + buildup of sphingomyelin.

§ Cherry-red spot on macula + yes, hepatosplenomegaly (Niemann-Pick is a longer

name than Tay-Sachs, and hepatosplenomegaly is a long word, therefore Niemann-

Pick is the one that is associated with it).

o Krabbe (AR)

§ Deficiency of galactocerebrosidase (galactosylceramidase) + buildup of

galactocerebroside + psychosine.

MEHLMANMEDICAL.COM 53
MEHLMANMEDICAL.COM

§ Globoid cells + optic atrophy.

o Metachromatic leukodystrophy (AR)

§ Deficiency of arylsulfatase A + buildup of cerebroside sulfate.

§ Neurodegeneration.

o Hurler (AR)

§ Deficiency of alpha-L-iduronidase + buildup of glycosaminoglycans (GAGs; dermatan

and heparan sulfate).

§ Clouded corneas (HY).

§ Gargoyle facies, stridor, neurodegeneration.

o Hunter (XR)

§ Deficiency of iduronate sulfatase + buildup of GAGs.

§ Milder form of Hurler.

§ No clouded corneas.

§ Neurodegeneration.

o I-cell disease

§ Deficiency of N-acetylglucosamine-1-phosphate transferase (this enzyme name

shows up in a stem on the newer Step 1 NBMEs but you do not need to know it for

2CK), leading to inability to make mannose-6-phosphate at the Golgi.

§ Coarse facial features + restricted joint movement.

§ Golgi is always the answer if they ask you a cellular location for the disease, even

though this is considered a lysosomal storage disease.

- Collagen disorders (2CK- / Peds shelf-sufficient detail):

o Type I

§ Osteogenesis imperfecta (OI) à will be the answer for fractures at different stages

of healing after you’ve ruled out child abuse; if you can rule out OI as well, then the

answer is osteopetrosis (Albers-Schonberg).

§ Type I needed for bone and late-wound healing (scar becomes white).

o Type II

§ Stickler syndrome à congenital deafness + collagen II disorder = Sticker.

MEHLMANMEDICAL.COM 54
MEHLMANMEDICAL.COM

§ Type II needed for intervertebral disc + cartilage + vitreous humor.

o Type III

§ Ehlers-Danlos (ED) à hyperflexible joints + hyperextensible skin; can be associated

with mitral valve prolapse and/or aortic regurg and/or dissection; cerebral berry

aneurysms; ED is associated with many collagen types, but USMLE wants type III.

o Type IV

§ Alport syndrome (XR, not XD, on NBME 18 or 19 for Step 1) à ear and/or eye

problems + red urine = Alport; actual mutation in type IV collagen gene (vs

Goodpasture in adults, which is Abs against type IV collagen).

- Sugar disorders (2CK- / Peds shelf-sufficient detail):

o Both galactose and fructose disorder vignettes may carry the descriptor of “reducing sugars”

in the urine. Galactose disorders are detected by heel prick test at birth. Avoid breastfeeding

for galactose disorders (lactose = galactose + glucose). Avoid table sugar and honey for

fructose disorders.

o Classic galactosemia

§ Deficiency of galactose-1-phosphate uridyl transferase.

§ Failure to thrive, hepatosplenomegaly, cataracts.

§ Associated with E. coli sepsis in neonates.

o Duarte galactosemia

§ Partial deficiency of galactose-1-phosphate uridyl transferase.

§ Not as bad as classic galactosemia.

§ Much more common than classic galactosemia.

o Galactokinase deficiency

§ Cataracts from early age if not detected (galactose, via aldose reductase, goes to

galactitol à strong osmotic pull into lens of eye à cataracts. Mechanism similar to

adults with diabetic retinopathy (glucose à sorbitol, via aldose reductase).

§ Not as bad as Duarte or classic galactosemias.

o Essential fructosuria

§ Deficiency of fructokinase.

MEHLMANMEDICAL.COM 55
MEHLMANMEDICAL.COM

§ Often asymptomatic, but reducing sugars seen in urine incidentally.

§ No Tx necessary.

o Hereditary fructose intolerance

§ Deficiency of aldolase B.

§ Failure to thrive, hypoglycemia, jaundice.

§ Tx with avoidance of fructose-containing foods.

- Child abuse findings for USMLE? à spiral fractures (rotational/twisting force applied to bone);

posterior rib fractures (squeezing); circular burns (cigarettes); burns sparing flexor regions (from being

dipped in hot water, child flexes limbs to decrease exposed surface area); retinal detachment /

hemorrhages + subdural hematoma (shaken baby syndrome); avoidance of eye contact / quiet.

Important summary of CSF findings (see next page):

- This first chart is relatively qualitative and sufficient for the USMLE. The second chart is more

quantitative/expansive in case you’re interested; all values are derived extensively from the

literature.

MEHLMANMEDICAL.COM 56
MEHLMANMEDICAL.COM

MEHLMANMEDICAL.COM 57
MEHLMANMEDICAL.COM

YouTube
@mehlmanmedical

Instagram
@mehlman_medical

MEHLMANMEDICAL.COM 58
MEHLMANMEDICAL.COM

MEHLMANMEDICAL
HY PEDIATRICS

All material is copyrighted and the property of mehlmanmedical.

Copyright © mehlmanmedical

MEHLMANMEDICAL.COM 59
MEHLMANMEDICAL
HY HEME/ONC
MEHLMANMEDICAL.COM

YouTube
@mehlmanmedical

Instagram
@mehlman_medical

MEHLMANMEDICAL.COM 2
MEHLMANMEDICAL.COM

HY Heme/Onc

The purpose of this document is not to be a 500-page textbook with every low-yield and superfluous detail catered to; the

aim is to infuse you with the highest yield info for the USMLEs.

- 32F + low MCV + low Hb + smear is shown below; Dx?

o Answer = iron deficiency anemia (IDA); smear shows pale RBCs (i.e., with central pallor).

- 32F + low MCV + low Hb + smear is show below; Dx?

o Answer = thalassemia; smear shows target cells; these may be seen in other DDx, however

on USMLE thalassemia is by far the highest yield association.

- 32F + low MCV + low Hb; next best step in Mx? à answer = “check serum iron and ferritin.”

- 32F + low MCV + low Hb + low Fe + low ferritin; Dx? à answer = IDA.

- 32F + low MCV + low Hb + normal Fe + normal ferritin; Dx? à answer = thalassemia.

- 32F + normal MCV + low Hb + low Fe + normal ferritin; Dx? à answer = anemia of chronic disease

(AoCD).

- 32F + rheumatoid arthritis + low MCV + low Hb + low Fe + normal ferritin; Dx? à answer = AoCD;

student says, “Wait, but I thought MCV was supposed to be normal in AoCD.” à plenty of 2CK-level

Qs give low MCV for AoCD. That’s you’re value point.

- 32F + low MCV + low Hb + high red cell distribution width (RDW); Dx? à answer = IDA.

- 32F + low MCV + low Hb + low/normal RDW; Dx? à answer = thalassemia.

- 32F + low MCV + low Hb + high transferrin binding capacity; Dx? à answer = IDA.

MEHLMANMEDICAL.COM 3
MEHLMANMEDICAL.COM

- 32F + low MCV + low Hb + low/normal transferrin binding capacity; Dx? à AoCD.

- 32F + low MCV + low Hb + no improvement with iron supplementation; Dx + next best step in Mx? à

answer = thalassemia; next best step = hemoglobin electrophoresis.

- Mechanism for thalassemia? à decreased production of one type of hemoglobin chain (i.e., if ¯ alpha

chain production, then Dx is alpha thalassemia; same for beta, respectively).

- Why is RDW low or normal in thalassemia but high in IDA? à decreased heme chain synthesis results

in RBCs that are uniformly small (red cell distribution width is ¯ because all resultant RBCs are small);

in IDA, the microcytosis is non-uniform, so some RBCs are small; others are larger; the result is ­ RDW

in IDA.

- 32F + pregnant + completely asymptomatic till this point + low MCV + low Hb + starts taking prenatal

vitamin supplement + three weeks later still has low MCV and low Hb; next best step? à answer =

hemoglobin electrophoresis. Dx = alpha thalassemia silent, which refers to one alpha mutation (out of

a possible four) à asymptomatic apart from microcytic anemia on FBC.

- 32F + pregnant + Hx of fatigue + low MCV + low Hb + normal iron and ferritin; Dx? à answer = alpha

thalassemia trait (two mutations); patient will present similar to mild/moderate IDA but have normal

serum iron and ferritin.

- 8F + severe anemia + hepatosplenomegaly + normal iron and ferritin + Hb electrophoresis shows

tetrameric beta-hemoglobin (b4); Dx? à answer = hemoglobin H disease (three alpha mutations).

- 32F + pregnant + fetus dies in utero + fetal blood sampling shows tetrameric gamma-hemoglobin (c4);

Dx? à hemoglobin Barts (four alpha mutations); fatal in utero.

- 6M + low MCV + low Hb + hepatosplenomegaly + HbA2 6% + following x-ray of head; Dx?

MEHLMANMEDICAL.COM 4
MEHLMANMEDICAL.COM

o Answer = beta-thalassemia major; almost always a child; “chipmunk facies/skull” and

hepatosplenomegaly occur due to ­­ extramedullary hematopoiesis.

- 6M + low MCV + low Hb + normal iron and ferritin + hepatosplenomegaly + HbA2 6%; Tx? à answer =

serial blood transfusions + iron chelation therapy (e.g., deferoxamine) for beta-thalassemia major;

serial blood transfusions done to Tx thalassemia result in iron overload; do not confuse this type of

iron overload with that of hereditary hemochromatosis, which is instead managed with serial

phlebotomy, not chelation therapy. The iron overload due to serial transfusions is called secondary

hemochromatosis (transfusional siderosis).

- 10F + receiving transfusions for beta-thalassemia major; Q asks: to avoid iron overload, measurement

of which of the following is most sensitive in assessing patient’s iron stores; answer = ferritin; wrong

answer are iron saturation and transferrin.

- 22F + low MCV + low Hb + fatigue + HbA2 6%; Dx? à beta-thalassemia minor; usually adult.

- “What’s the deal with the HbA2. What does that mean?” à highest yield point for beta-thalassemia

major and minor is that HbA2 (a2d2; alpha2-delta2) is increased on hemoglobin electrophoresis.

Normal adult Hb is HbA1 (a2b2); fetal Hb is HbF (a2c2; alpha2-gamma2).

- Normal MCV? à 80-100 fL.

- Normal Hb? à according to literature, in men + elderly it’s 13.0-17.5 g/dL; menstruating females

12.0-17.5 g/dL. Below these thresholds, patient has anemia. Above, patient has polycythemia.

- Normal Hct? à Men and elderly: 42-52%; women: 36-46%.

- Normal WBC? à 4-11,000 / uL.

MEHLMANMEDICAL.COM Iron deficiency anemia Thalassemia


Hb ¯ ¯
MCV ¯ ¯
Serum iron ¯ Normal
Ferritin ¯ Normal
RDW ­ (HY) ¯/Normal (HY)
Blood smear Pale RBCs Target cells
Hb electrophoresis Normal a: normal; b: ­ HbA2 (HY)
What happens if we give iron? Improvement No improvement (HY)
Tx Iron Transfusions if severe

MEHLMANMEDICAL.COM 5
MEHLMANMEDICAL.COM

- 72M + fatigue + smear shows pale RBCs + Hb 9.4 g/dL; most likely cause? à answer = GI blood loss

(IDA) à must think diverticular bleed, colorectal cancer, and angiodysplasia causing IDA in elderly

patient with fatigue; 2CK-level Qs will jump straight to colonoscopy as the answer.

- 65M + pain in fingertips for 3 weeks + facial plethora + splenomegaly + Hb 20.2 g/dL + WBCs 14,500

with normal differential + normal platelets + O2 sats 94% on room air; Dx + Tx? à answer =

polycythemia vera (PCV); Tx = phlebotomy.

- Mechanism of PCV? à JAK2 mutation causing “proliferation of bone marrow stem cells.”

Erythropoietin (EPO) is decreased because it is suppressed. Although oxygen sats should be as close

to 100% as possible, patients generally hold up fine with sats >94%.

- 48F + pruritis after a shower + high WBCs + Hb 19.5 g/dL; Dx? à answer = PCV; pruritis after a shower

is a classic finding à reflect basophilia; WBC count can be normal or elevated in PCV.

- 50F smoker + Hx of COPD + Hb 18.5 g/dL; Dx + mechanism? à answer = secondary polycythemia à

increased EPO due to low oxygen tension (e.g., in COPD, CF, etc.) à mechanism is “proliferation of

erythroid precursors” à this is because the high EPO results in an elevation of only RBCs; in PCV, O2

sats would be 94% or greater and WBCs and/or platelets may be elevated.

- 55M smoker + red urine + polycythemia + hypercalcemia + unknown biopsy specimen is shown; Dx?

o Answer = renal cell carcinoma (RCC) à can cause paraneoplastic secondary polycythemia

due to EPO secretion as well as hypercalcemia due to PTHrp secretion (latter also squamous

cell of lung); biopsy shows clear cell carcinoma (most common variant of RCC; HY biopsy

finding for Step 1).

- Normal bleeding time (BT)? à answer = 2-7 minutes.

- Normal platelet count (PC)? à answer = 150-450,000/uL.

- Two main ways ITP presents on USMLE? à 1) school-age kid with viral infection followed by epistaxis

and/or bruising/petechiae; 2) woman 30s-40s with random bruising and elevated BT / low platelets.

MEHLMANMEDICAL.COM 6
MEHLMANMEDICAL.COM

- 34F + bruises + BT 9 minutes; Dx? à answer = idiopathic/immune thrombocytopenic purpura (ITP).

- 34F + bruises + BT 6 minutes; Dx? à answer = domestic abuse (on FM shelf).

- 34F + viral infection followed by petechiae + epistaxis; which would confirm the Dx (answers are

either ­ bleeding time or ¯ platelet count) à answer = ¯ platelet count; both are seen, but an

isolated ¯ platelet count in the setting of the clinical picture is how to confirm Dx of ITP.

- 12M + runny nose for 4 days + epistaxis; Dx? à answer = ITP à often follows viral infection.

- 12M + no mention of any type of infection + epistaxis + BT 8 minutes; Dx? à answer = ITP à student

says, “Wait, but you just said it follows a viral infection.” à USMLE need not mention viral infections,

and they often don’t. Especially on 2CK-level NBME Qs, for conditions associated with viral infections

like ITP, deQuervain subacute granulomatous thyroiditis, minimal change disease, IgA nephropathy,

etc., the Q won’t even mention viral infection; you just need to know the presentation. Just think:

most people with COVID-19 are/were asymptomatic right?

- 12M + cough + coryza + epistaxis; mechanism for this patient’s condition? à answer = antibodies

against glycoproteins IIb/IIIa on platelets (type II hypersensitivity); GpIIb/IIIa mediate platelet

aggregation (not adhesion, which is GpIa, not IIb/IIIa); there is often a genetic susceptibility to ITP.

This is asked on 2CK material as well.

- 12M + epistaxis + platelet count 50,000; next best step in management? à answer = steroids.

- 12M + epistaxis + platelet count 50,000 + steroids not effective; next best step in management? à

answer = IVIG.

- 12M + epistaxis + platelet count 50,000; what’s the most effective way to decrease recurrence? à

answer = splenectomy; students says, “Wait, I thought you just said steroids were what we do first.”

à Yeah, you’re right, but the Q asks what’s most effective in decreasing recurrence, so splenectomy

is correct. USMLE will sometimes ask next best step vs what’s most effective.

o Two points to note about ITP Tx:

§ 1) some literature has suggested no Tx is necessary in select pediatric cases where

the presentation is limited to mild cutaneous findings. However on the USMLE,

steroids is always the answer for immediate management.

§ 2) Literature suggests sometimes IVIG may be given before steroids (i.e., extremely

low platelet counts). But on the USMLE, steroids are always first, not IVIG.

MEHLMANMEDICAL.COM 7
MEHLMANMEDICAL.COM

- 12M + viral infection + low neutrophils; Dx? à answer = viral-induced neutropenia (not ITP).

- 12M + viral infection + low neutrophils + fever; next best step in Dx + Mx? à answer = neutropenic

fever (febrile neutropenia) à medical emergency; must give immediate IV broad-spectrum

antibiotics; patient has possible infection but no way to defend against it.

- 12M + low neutrophils + fever + antibiotics are administered; what could help restore this patient’s

neutrophil count? à answer = granulocyte-macrophage colony-stimulating factor (GM-CSF);

molgramostim is commonly used as the GM-CSF agent.

- 8M + viral infection + low Hb + low WBCs + low platelets; Dx? à answer = aplastic anemia (viral

induced; likely Parvo-B19); aplastic anemia = Dx when all cell lines are down due to decreased bone

marrow production).

- 32F + malar rash + arthritis + Hb 9 + WBCs 3000 + platelets 90,000; mechanism for this hematologic

presentation? à answer = “increased peripheral destruction” (i.e., autoantibodies); “decreased bone

marrow production” is the wrong answer in SLE; lupus is associated with the development of anti-

hematologic cell line antibodies, usually against platelets (i.e., it’s common to see isolated

thrombocytopenia in SLE), but antibodies can form against WBCs and RBCs as well. What makes this

difficult is that a ¯ in all cell lines (i.e., RBCs, WBCs, platelets) looks like aplastic anemia (i.e., such as

with Parvo), but with SLE, the finding is due to antibodies, not defective bone marrow production. In

contrast, if this were viral-induced, then yes, that is aplastic anemia, and the answer is “decreased

bone marrow production.”

- 44F + hospitalized and treated for overactive thyroid + Hct 45% + WBCs 1500 (neutrophils 5%;

lymphocytes 95%) + PC 250,000; Dx? à answer = drug-induced neutropenia à propylthiouracil and

methimazole both can cause agranulocytosis (neutropenia); other HY drugs for agranulocytosis are

ganciclovir (for CMV), clozapine (anti-psychotic), methotrexate (DMARD), ticlopidine (anti-platelet).

- 3M + absent thumb on the left hand + ¯ Hb + ¯ WBCs + ¯ platelets; Dx? à answer = Fanconi anemia;

autosomal recessive aplastic anemia; presents with absent or hypoplastic thumbs or radii.

- 3M + triphalangeal thumb + ¯ RBCs; Dx? à answer = Diamond-Blackfan anemia; pure RBC aplasia

that presents with triphalangeal thumb.

- Cancer associated with pure RBC aplasia? à answer = thymoma.

MEHLMANMEDICAL.COM 8
MEHLMANMEDICAL.COM

- 8M + viral infection + low Hb + low WBCs + low platelets + fever; next best step? à answer =

immediate IV antibiotics à even though viral-induced aplastic anemia, the patient still has a

neutropenic fever, so this is a medical emergency.

- 8M + viral infection + low Hb + low WBCs + low platelets; what’s the next best step in Dx? à answer =

bone marrow aspiration (to confirm decreased bone marrow production consistent with aplastic

anemia); sounds overkill, but it’s the answer.

- 26F + daycare worker + coryza + lacy rash on legs and trunk + all immunizations up to date + afebrile +

RBCs, WBCs, and platelets all normal; next best step in Dx? à answer = “check Parvovirus IgM titers”

à in the absence of aplastic anemia, this is the answer if daycare worker presents with rash.

- 44F + undergoing chemotherapy + low RBCs + low WBCs + low platelets; next best step in Dx? à

answer = bone marrow aspiration (chemo-induced aplastic anemia).

- 44F + undergoing chemotherapy + all cell lines down + temperature 101.8F; next best step? à

answer = immediate IV broad-spectrum antibiotics; this is chemo-induced aplastic anemia, but there’s

a neutropenic fever.

- “Do we have to know the clotting cascade?” à To understand many heme disorders, yes, you should

know it:

MEHLMANMEDICAL.COM 9
MEHLMANMEDICAL.COM

- “Can you explain PT and PTT in relation to extrinsic and intrinsic pathways?”

o PT reflects the functioning of the extrinsic pathway; PTT reflects intrinsic. In other words, if

PT alone is high, then the extrinsic pathway is fucked up. If PTT alone is high, then the

intrinsic pathway has a problem. If both PT and PTT are elevated, then the common pathway

has an issue.

- Normal prothrombin time (PT)? à answer = 10-15 seconds.

- Normal activated partial thromboplastin time (aPTT; PTT)? à answer = 25-40 seconds.

o aPTT is a slightly more sensitive version of PTT, but on USMLE they are used interchangeably.

- 55F + no Hx of bleeding problems + coagulation testing prior to CABG shows ­ PTT + normal PT +

normal BT; what’s most likely to be abnormal in this patient? à answer = USMLE answer is kallikrein

formation. Patients with ¯ factor XII (Hageman factor) are asymptomatic; factor XII converts pre-

kallikrein into kallikrein.

- Neonate + born at home + bleeding from umbilical site; what are the arrows for BT, PT, and aPTT? à

answer = normal BT; ­ PT; ­ aPTT; Dx = vitamin K deficiency. Born at home = not given vitK injection.

- 40M + on warfarin for prosthetic valve placed years ago + receiving broad-spectrum antibiotics for

infection; Q asks why required warfarin dosage over next several weeks would ¯; answer = “vitamin K

deficiency caused by depletion of normal gut flora.”

- What does vitamin K do? à cofactor for enzyme called gamma-glutamyl carboxylase à gamma-

carboxylates + activates clotting factors II, VII, IX, and X, as well as anti-clotting proteins C and S. Since

factors II (prothrombin) and X are in the common pathway, PT and PTT are both elevated in vitamin K

deficiency. Factor VII is in the extrinsic pathway; factor IX in the intrinsic pathway. Protein C functions

to inactivate factors Va and VIIIa (activated clotting factors V and VIII) back to their inactive form.

Protein S is merely a cofactor for protein C.

- How does warfarin relate to vitamin K? à Warfarin inhibits vitamin K epoxide reductase, which is the

enzyme that recycles vitamin K to its active form. That means less vitamin K can act as a cofactor for

gamma-glutamyl carboxylase. Notice the enzymes are different. Therefore warfarin leads to

decreased activation of clotting factors II, VII, IX, and X, as well as anti-clotting proteins C and S.

MEHLMANMEDICAL.COM 10
MEHLMANMEDICAL.COM

- “If proteins C and S are anti-clotting, and warfarin inhibits their activation, why would warfarin be an

anticoagulant then?” à protein C has a super-short half-life, so the effect of warfarin in the first few

days actually results in a hypercoagulable state, where protein C is ¯ but the actual clotting factors

are still present in greater amounts; this is why heparin is necessary as a bridging agent – i.e., patients

commenced on warfarin require simultaneous commencement of heparin for a few days.

- What does heparin do? à activates antithrombin III à leads to inactivation of prothrombin (factor II)

and factor X.

- How do you reverse warfarin? à vitamin K (slow; takes several days); if patient is actively bleeding or

requires surgery (i.e., fast reversal), answer = fresh frozen plasma.

- How do you reverse heparin? à protamine sulfate.

- Anything important about the structures of heparin vs warfarin? à heparin is a large, acidic, anionic

molecule and therefore does not cross the placenta; protamine, which is a positively charged cation,

can bind to and chelate it. Warfarin is a small lipophilic molecule that can cross the placenta, and is

therefore teratogenic.

- 34F + DVT + PC 220,000 + PT 13 seconds + PTT 36 seconds + heparin commenced + now PC is 130,000;

mechanism? à answer = drug-related antibodies à heparin-induced thrombocytopenia (HIT) à type

II hypersensitivity à antibodies form against the heparin-platelet factor 4 complex.

- Tx for HIT à stop heparin and give direct-thrombin inhibitor (e.g., dabigatran, lepirudin); warfarin is

the number-one wrong answer.

- How does a platelet problem present? à epistaxis or generally mild cutaneous findings (i.e.,

petechiae, bruising).

- How does a clotting factor problem present? à menorrhagia; excessive bleeding after tooth

extraction; hemarthrosis; excessive bleeding after circumcision (neonatal boys).

- 17F + Hx of epistaxis + sometimes clots in her menstrual pads; Dx till proven otherwise? à answer =

von Willebrand disease (vWD) à always presents with combination of platelet problem + clotting

factor problem. Clots with menses signify heavy bleeding (i.e., menorrhagia).

- Inheritance pattern of vWD? à autosomal dominant (high-yield).

- Mechanism of von Willebrand factor? à bridges GpIb on platelets to underlying collagen + vascular

endothelium à mediates platelet adhesion (don’t confuse this with platelet aggregation, which is

MEHLMANMEDICAL.COM 11
MEHLMANMEDICAL.COM

when platelets stick together via GpIIb/IIIa); this leads to increased bleeding time. vWF also has

secondary/ancillary function of stabilizing factor VIII in plasma à in vWD, PTT is elevated in ~1/2 of

question stems (students tend to erroneously memorize PTT as always ­, but this will get you

questions wrong).

- 17F + Hx of epistaxis + Hx of excessive bleeding with wisdom teeth removal + BT 9 minutes + PT 12

seconds + PTT 44 seconds; Dx? à answer = vWD à BT always ­; PTT ­ in only ~half of questions.

- 17F + Hx of epistaxis + cut her finger a month ago that took a long time to stop bleeding + PT normal +

PTT normal + platelet aggregation studies normal; Dx? à vWD à the ­ BT is reflected by the cut on

the finger taking a while to stop bleeding; PTT need not be elevated; normal platelet aggregation

studies simply mean that GpIIb/IIIa are functioning properly, but vWD relies on GpIa, which is

adhesion, not aggregation.

o When students get above vignette wrong, it’s because they don’t realize PTT is normal in

about half of vWD vignettes and/or they forget vWF mediates platelet adhesion, not

aggregation.

- 19M + petechiae + normal platelet count + BT 9 minutes + PTT 42 seconds; Tx for this patient’s

condition? à answer = desmopressin (DDAVP) increases vWF synthesis and release.

- 16F + took aspirin in suicide attempt + ¯ Hb + blood in stool; Q asks what finding would be expected

to be abnormal in this patient? à answer = bleeding time (­­ because aspirin inhibits COX1/2 à ¯

thromboxane A2 production in platelets à ¯ platelet function); the wrong answer is ¯ platelet count

(platelet count doesn’t change with aspirin/NSAID use); fibrin degradation products is also a wrong

answer (this is ­ in DIC and pulmonary embolism).

- 53M + coronary artery disease + asks physician about celecoxib + physician is reluctant to prescribe

because of increased risk of MI with celecoxib; Q asks why there is ­ risk; answer = “inhibition of

prostacyclin (PGI2) formation without inhibition of thromboxane A2 in platelet”; celecoxib is a COX2

selective inhibitor.

- 8M + hemarthrosis + PTT 90 seconds; Dx? à answer = hemophilia A or B à isolated ­ in PTT.

- Hemarthrosis in school-age boy; Dx till proven otherwise? à answer = hemophilia A or B.

MEHLMANMEDICAL.COM 12
MEHLMANMEDICAL.COM

- Hemarthrosis in school-age boy + no way to differentiate between A and B based on the vignette; Dx?

à answer = hemophilia A (way more common than B); for some reason, this epidemiologic point is

assessed on the Step in this fashion.

- 8-day-old neonatal male + excessive bleeding with circumcision; Dx? à answer = hemophilia A or B.

- Inheritance pattern of hemophilia A and B? à X-linked recessive (high-yield).

- Mechanism for hemophilia A and B? à answer = deficient production of factors VIII (A) and IX (B).

Rarely, they can be caused by antibodies against the factors.

- Tx for hemophilia A? à answer = IV desmopressin (DDAVP) and/or factor VIII replacement.

- Tx for hemophilia B? à answer = factor IX replacement only.

- 15M + Hx of receiving factor VIII replacement therapy for hemophilia A + becoming increasingly less

effective with time + PTT is 160 seconds; Dx? à answer = antibodies against factor VIII (almost always

due to repeated replacement of factor VIII).

- 13F + PTT 90 seconds + analysis shows deficiency of factor IX; Q asks the mechanism via which this is

possible à answer = lyonization (skewed X-inactivation); the USMLE Q will never give a female with

an X-linked recessive disorder unless the explicit point of the Q is X-inactivation/lyonization. That is,

never assume, “hmm well maybe there’s lyonization here…” If the USMLE wants that, they’ll ask it.

- 23M + BT 12 minutes + thrombocytopenia + smear shows giant platelets + failure of platelet

agglutination with ristocetin cofactor; Dx + mechanism? à answer = Bernard-Soulier disease;

mechanism = deficiency of platelet GpIb (mediates adhesion); platelets can be giant for some magical

reason.

- “Wait what’s that ristocetin assay thing. I’ve seen that before.” à all you need to know is that it will

cause platelet agglutination, but in Bernard-Soulier disease and vWD, the assay is negative – i.e., the

platelets don’t agglutinate. The test measures the binding of vWD to platelet GpIb, so clearly

deficiency of either will yield a negative test.

- 23M + BT 12 minutes + normal platelet count + ristocetin cofactor assay yields agglutination; Dx +

mechanism? à answer = Glanzmann thrombasthenia; mechanism = deficiency of platelet

glycoproteins IIb/IIIa (mediate aggregation, not adhesion). Student says, “Wait, but if it’s an isolated

increase in BT, why can’t this just be ITP then?” à Because platelet count is normal; in ITP, there’s

MEHLMANMEDICAL.COM 13
MEHLMANMEDICAL.COM

always thrombocytopenia. I’ve seen this Dx on the NBMEs show up as just “thrombasthenia,” where

you’re like “Wtf is thrombasthenia?” But that just refers to Glanzmann.

- 32F + Hx of Crohn + low Hb + MCV 90; Dx? à answer = AoCD à seen in autoimmune disease (i.e., RA,

IBD, SLE), organ failure (i.e., renal, liver), chronic infection (e.g., hepatitis B/C).

- Mechanism of AoCD? à inflammatory state leads to ­ IL-6 production by liver à ­ hepcidin

production by liver à ¯ ferroportin acivity à ¯ iron release by gut enterocytes and general cellular

stores à ¯ iron transport in blood à ferritin levels are normal but serum iron is ¯. Transferrin is also

¯, resulting in ¯ total iron binding capacity (TIBC). This means even though iron is low in the blood,

since transferrin is also low, there’s still ¯ binding capacity for iron overall. In contrast, in IDA, TIBC is

high (i.e., transferrin goes ­ to compensate for low serum iron; in AoCD, transferrin secretion is

suppressed despite ¯ serum iron).

o Student says, “But wait, isn’t this similar to thalassemia, where serum iron is decreased but

ferritin is normal?” à Good question, but in thalassemia, 1) they won’t give you a vignette

with autoimmune disease, organ failure, or chronic illness, and 2) they’ll often throw in

target cells, ¯ RDW, or Hb electrophoresis findings.

- “What about MCV and AoCD?” à classically normal MCV in anemia of chronic disease (normocytic

anemia), but some 2CK NBME Qs have ¯ MCV à student says, “Wait how is this AoCD? Isn’t MCV

supposed to be normal in AoCD?” à my response: “Yes, you’re right, but various 2CK have it ¯.” à

So your take-home regarding AoCD should be: classically normal, but can absolutely be ¯ on USMLE.

- 6F + Hx of multiple episodes of sore joints + fever of 102F + salmon-pink rash over body + high ESR +

Hb of 9 g/dL + MCV 72; Dx? à anemia of chronic disease secondary to juvenile rheumatoid arthritis

(JRA; aka juvenile idiopathic arthritis; JIA) à if Q gives you low MCV in AoCD, the vignette will be

overwhelmingly obvious for an autoimmune disease + you can eliminate the other answers.

- 44F + chronic alcoholism + abdominal fluid wave + low Hb + MCV 84; Dx? à AoCD.

- 68M + hasn’t been to doctor in years + Hb 8.6 + Hct 25% + MCV 90 + normal RDW + normal iron +

normal ferritin + normal transferrin saturation + creatinine 2.9; Dx + Tx? à AoCD caused by chronic

renal insufficiency. Tx = EPO.

MEHLMANMEDICAL.COM 14
MEHLMANMEDICAL.COM

- Tx of AoCD? à answer = supportive care / treat the underlying condition; EPO is the answer only if

renal failure is the etiology; if renal failure is not the underlying Dx, EPO is the wrong answer.

- 12F + chronic renal failure + epistaxis for past two weeks + platelets 200,000/uL + Hb 9.5 g/dL; Q asks

the mechanism for the epistaxis; answer = “acquired platelet dysfunction”; Dx = uremic platelet

dysfunction à high blood urea nitrogen (BUN) in renal failure causes a qualitative dysfunction of

platelets, where they merely don’t do their job; there is no quantitative issue (i.e,. platelet count is

normal).

o Students often choose “erythropoietin deficiency,” which is the wrong answer. Anemia of

chronic disease can be seen in renal failure secondary to decreased EPO, yes, but AoCD in

and of itself doesn’t cause epistaxis; epistaxis is seen with platelet problems; Hb is merely

down because there’s IDA from the epistaxis.

- 82F + back pain + M-protein spike showing IgG kappa + epistaxis; why the epistaxis? à uremic

platelet dysfunction secondary to renal failure from multiple myeloma (renal amyloidosis).

- “Can you tell me the highest yield points about multiple myeloma?”

o Cancer of plasma cells à monoclonal expansion, where the bone marrow has ­­ plasma

cells (>10%) originating from a single plasma cell.

o Plasma cells secrete non-functional immunoglobulin light-chains; these are mostly IgG;

serum protein electrophoresis is the next best step in the Dx of multiple myeloma, which

will show the ­­ IgG kappa and lamda; this ­­ in IgG is called an M-protein spike; this has

nothing to do with IgM; don’t confuse that.

o Since the IgG light chains are proteins and are present in ­­ amounts in the blood, they are

prone to deposit in tissues, leading to amyloidosis (abnormal protein deposition in tissues);

multiple myeloma is most common cause of renal and cardiac amyloidosis (this is all over the

NBMEs); any renal/cardiac Dx in MM, answer = renal or cardiac amyloidosis.

o The IgG light chains show up in the urine à Bence-Jones proteinuria.

o The IgG light chains cause RBCs to stick together à Roulette formation + ­­ ESR (the

Roulettes are heavy, so the RBCs deposit at a faster rate).

MEHLMANMEDICAL.COM 15
MEHLMANMEDICAL.COM

o The neoplastic plasma cells can cause ­ cytokine activity at bone à causes lytic lesions (e.g.,

“Pepperpot skull”) + back pain (spinal lytic lesions) à lysis of bone causes hypercalcemia.

“Pepper pot skull”

o Smear in multiple myeloma will show plasma cells (blue cells below) with “clockface
chromatin,” which is the appearance ascribed to the nuclei (purple below).

- “What are the highest yield points for Waldenstrom macroglobulinemia?”

o Cancer of plasmacytoid cells à “oid” means looks like but ain’t. So the cells look like plasma

cells, but they ain’t plasma cells. For instance,

§ Fibrinoid necrosis in polyarteritis nodosa à necrosis looks like fibrin, but it’s not.

§ Patients with MEN2B have Marfanoid body habitus à they look like they have

Marfan syndrome, but they don’t.

o Hyperviscosity syndrome à can present as headache, blurry vision, pain in the tips of the

fingers, and/or Raynaud phenomenon.

§ Correct, Raynaud occurs in things other than CREST syndrome. And these

hyperviscosity Sx are also seen in polycythemia.

o There is an IgM M-protein spike; in contrast, MM has an IgG M-protein spike; once again, the

M in M-protein spike has zero to do with IgM; it’s just what we call the immunoglobulin

spike. Students fuck this up a lot, which is why I have to be extra redundant explaining this.

MEHLMANMEDICAL.COM 16
MEHLMANMEDICAL.COM

o Unlike MM, there is no hypercalcemia; there is no Bence-Jones proteinuria; ESR need not be

elevated.

- 82F + back pain + hypercalcemia + high ESR; Dx? à multiple myeloma.

- 82F + back pain + hypercalcemia + Hx of breast cancer 15 years ago; Dx? à metastatic malignancy.

- 64M + back pain + calcium normal + high BUN & Cr + low serum Na + high serum K + low serum bicarb

+ x-ray shows lytic lesions of spine + epistaxis + low Hb + high MCV; Dx? à multiple myeloma causing

renal amyloidosis, resulting in uremic platelet dysfunction and renal tubular acidosis type IV

(insensitivity to aldosterone, or hyporeninemic hypoaldosteronism à Addisonian biochemistry but

the problem is due to the kidney, not the adrenal glands); student says, “Wait, but why is calcium

normal?” à Great question; probably because renal failure causes low Ca, but MM causes high Ca, so

the patient could theoretically be normocalcemic in MM if he/she also has renal failure (one NBME Q

has normocalcemia in MM with renal failure); epistaxis due to uremic platelet dysfunction; low Hb

due to nosebleeds; MCV can be elevated in multiple myeloma.

- 50F + serum protein electrophoresis shows M-spike + bone marrow biopsy shows <10% plasma cells;

Dx? à answer = monoclonal gammopathy of undetermined significance (MGUS); 1-2% chance per

year of progressing to multiple myeloma.

- 28F + just gave birth + two minutes after separation of placenta gets shortness of breath and

tachycardia + bleeding from IV + catheter sites; Dx? à answer = disseminated intravascular

coagulation (DIC) secondary to amniotic fluid embolism.

- “Can you explain DIC?” à intractable clotting cascade activation in the setting of manifold triggers

such as trauma, sepsis, amniotic fluid embolism, Tx of AML with release of Auer rods into circulation

à USMLE wants you to know the arrows in DIC (high-yield):

o ­ BT, PT, aPTT, D-dimer, plasmin activity; ¯ fibrinogen, platelets, clotting factors.

o Fibrinogen is converted to fibrin, which is why it decreases.

o Plasmin breaks down fibrin, so more fibrin means more plasmin is upregulated in an attempt

to dissolve the excess fibrin production.

o D-dimer = fibrin degradation products; since more fibrin is being broken down, D-dimer ­.

MEHLMANMEDICAL.COM 17
MEHLMANMEDICAL.COM

o “Bleeding from catheter/IV sites” is 9 times out of 10 synonymous with DIC. However on one

2CK-level surgery NBME Q, dilutional thrombocytopenia secondary to ­ blood transfusions

presents the same.

- 40F + has surgery + requires 22 packs of RBCs transfused during the surgery + afterwards has bleeding

from catheter/IV sites + DIC not listed as an answer; Dx? à answer = thrombocytopenia; student

says, “Wtf? I thought that description is DIC.” à packed RBCs don’t contain platelets à ­­ acute

transfusions à dilutional thrombocytopenia + can present with DIC-like picture in vignette. Weird I

know, but you learn something new every day.

- 32F + SLE + DVT + PTT of 60 seconds; Dx? à answer = antiphospholipid syndrome (APLS) à APLS is

the answer when the patient has thromboses despite a paradoxically high aPTT (you’d normally

expect bleeding diathesis with high aPTT). This is because phospholipid is necessary for proper

functioning of the clotting cascade, so the in vitro PTT test doesn’t work as well (therefore ­), but the

antibodies cause clotting factor activation in vivo. APLS has numerous causes: you can have APL

antibodies in the setting of SLE (we just happen to call these Abs lupus anticoagulant), but we can also

have Abs against beta-2-microglobulin and cardiolipin, unrelated to SLE. Patients with APLS can

sometimes have a false-positive VDRL screening test for syphilis.

- Important coagulopathies? à FVL, prothrombin gene mutation, antithrombin deficiency, protein C/S

deficiency.

- 65M + warfarin-induced skin necrosis; patient most likely has what? à answer = protein C deficiency

à short half-life of protein C, which is depleted by warfarin, leads to ­­ hypercoagulable state in

someone innately deficient.

- 23M + thrombosis + no mention of ­ PTT; Dx (FVL, prothrombin mutation, protein C/S deficiency are

not listed)? à answer = antithrombin deficiency; wrong answer is antiphospholipid syndrome

(presumably for APLS the vignette will mention ­ PTT if male patient, or recurrent miscarriage in

female).

- 13M + bloody diarrhea + afebrile + low platelets + low Hb + red urine + smear is shown; Dx?

MEHLMANMEDICAL.COM 18
MEHLMANMEDICAL.COM

o Answer = hemolytic uremic syndrome (HUS); presents with triad of:

o 1) thrombocytopenia;

o 2) hemolytic anemia with schistocytes (above smear); and

o 3) renal insufficiency with or without hematuria.

- Mechanism of HUS? à E. coli (EHEC O157:H7) and Shigella both secrete toxins (Shiga-like toxin and

Shiga toxin, respectively) that cause inflammation of renal microvasculature à ADAMTS13 protein

inactivation à failure of cleavage of vWF multimers à platelet adherence to vascular endothelium

cannot be as readily reversed à platelet aggregations protrude into vascular lumen causing shearing

of RBCs flying past à fragmentation of RBCs (schistocytes, aka helmet cells).

- 34F + epistaxis + platelet count 80,000 + low Hb + muscle strength 3/5 on left side of body +

temperature 101.2F; Dx? à answer = thrombotic thrombocytopenic purpura (TTP) à presents with

- “Can you explain TTP? I always mix that up with ITP.” à TTP is thrombotic thrombocytopenic

purpura; caused by antibodies against, or a mutation in, a protein called ADAMTS13, which is a

metalloproteinase that breaks down vonWillebrand factor multimers. The presentation will classically

be a pentad of 1) thrombocytopenia; 2) schistocytosis; 3) renal insufficiency; 4) fever; 5) neurologic

signs. The combination of thrombocytopenia + schistocytosis = microangiopathic hemolytic anemia

(MAHA).

- 34F + epistaxis + platelet count 80,000 + low Hb + muscle strength 3/5 on left side of body +

temperature 101.2F; Dx? à answer = TTP; at first you’re like, “Is this a stroke? What’s going on

here?” This is how it presents on the NBME. The low hemoglobin is due to the fragmentation of the

RBCs leading to schistocytosis.

MEHLMANMEDICAL.COM 19
MEHLMANMEDICAL.COM

- “What do you mean fragmentation of RBCs leading to schistocytosis?” à schistocytes are caused by

the fragmentation/shearing of RBCs intravascularly. TTP is one of the common causes. The others are

hemolytic uremic syndrome (HUS), DIC, HELLP syndrome in pregnancy, and mechanical hemolysis

(prosthetic heart valves).

- “Can you explain HUS? And I confuse it sometimes with TTP.” à therefore the combination of

thrombocytopenia + schistocytosis (often seen in a Q as a combo of low platelets + low Hb + normal

WBC count) should set off alarm bells for HUS and TTP. Because this process occurs in

microvasculature, this is why the combo of thrombocytopenia and schistocytosis is referred to as

microangiopathic hemolytic anemia (MAHA). Other HY points:

o Whereas TTP presents with a pentad of 1) thrombocytopenia; 2) schistocytosis; 3) renal

insufficiency; 4) fever; 5) neurologic signs, HUS presents with just the first three. Therefore:

o TTP = HUS + fever and neurologic signs.

o The mechanisms are different, as discussed above, but presentation-wise, that’s an easy way

to remember it.

o HUS will classically be pediatric + bloody diarrhea (EHEC or Shigella); TTP will usually be an

adult who presents with fever and stroke-like Sx + who also has the triad you’d expect to see

in HUS. The tricky thing about TTP is that episodes can be triggered by various etiologies,

such as viral infection, but the vignette will not give bloody diarrhea.

o Tx for HUS is supportive with intravenous fluids; Tx for TTP is plasmapheresis.

- 63M + fullness in LUQ + low Hb + high uric acid + bone marrow aspiration shows dry tap; Dx? à

answer = myelofibrosis à “massive splenomegaly” often seen in NBME (fullness of LUQ); Hb can be

low from myelophthisic anemia (crowding of bone marrow leading to anemia); myelofibrosis due to

JAK2 mutation (same as PCV). Student says, “Wait but why high uric acid? Isn’t that gout?” à can be

seen sometimes with increased cell turnover, including RBCs (precursors have nucleic acid).

- 54F + massive splenomegaly + smear is shown; Dx?

MEHLMANMEDICAL.COM 20
MEHLMANMEDICAL.COM

- Answer = myelofibrosis à dacrocytes = teardrop-shaped RBCs à HY for myelofibrosis; key terms are:

“massive splenomegaly,” “dry tap,” “teardrop RBCs”).

- What does red pulp vs white pulp of spleen do? à red pulp is where senescent RBCs are

phagocytosed; white pulp contains ~50% of the body’s reservoir of macrophages.

- 3F + African-American + painful hands + HR120 + 2/6 mid-systolic murmur; Dx? à answer = sickle cell

crisis; dactylitis (inflammation of the fingers) is one of the most common presentations, especially in

pediatrics; benign flow/functional murmurs can be seen with high HR, especially in peds.

- Inheritance pattern of sickle cell? à answer = autosomal recessive (HY); carrier status (one mutation)

is referred to as sickle cell trait, which is less severe than sickle cell anemia (two mutations).

- Mutation in sickle cell? à answer = glutamic acid à valine on the beta-chain.

- How to Dx sickle cell? à answer = hemoglobin electrophoresis à glutamic acid is negatively charged;

valine is neutral; this means HbS is more non-polar à does not migrate as far on Hb electrophoresis

(gel goes from - à +) because it is less attracted to the + end of the gel.

- When does sickling notably occur in sickle cell? à dehydration + increased acidity.

Sickle cells

- 4M + sickle cell + Q asks which of the following best describes the molecular basis for sickling in this

patient à answer = “gain of stabilizing hydrophobic interactions in the deoxygenated form of

hemoglobin S” à valine is more hydrophobic than glutamic acid.

MEHLMANMEDICAL.COM 21
MEHLMANMEDICAL.COM

- 5F + sickle cell; Q asks mechanism for sickling; answer = beta chain slips into a complimentary

hydrophobic pocket on the alpha chain.

- 3F + takes penicillin prophylaxis; why? à decreases Strep pneumo infections; penicillin prophylaxis

indicated until age 5, in addition to PCV13 and PPSV23 Strep pneumo vaccines.

- Why autosplenectomy in sickle cell + what vaccines are needed? à sickling in red pulp leads to

microinfarcts à any sickle cell patient needs vaccines for S. pneumo, H. influenzae type B, and

Neisseria meningitidis.

- Why are those vaccines needed? à with splenectomy (or autosplenectomy), increased risk of

infection with encapsulated organisms, which require opsonization and phagocytosis for clearance à

spleen white pulp is important site of phagocytosis (tangential: IgG and C3b are immune system’s

main opsonins).

- One-year-old girl + missed dose of penicillin prophylaxis a few days ago + now has fever of 103F + HR

100, RR 22, low BP; what antibiotic do we give? à answer = cefotaxime; penicillin and ceftriaxone are

wrong answers; community-acquired sepsis (patient need not have sickle cell) often treated with

third-generation cephalosporin; give cefotaxime < age 6; give ceftriaxone > age 6.

- 8M + sepsis; Abx Tx? à answer = ceftriaxone.

- 5F + sickle cell + red urine; Dx? à answer = renal papillary necrosis; most common cause of nephritic

syndrome in sickle cell.

- 5F + sickle cell + renal problem + no blood in urine; Dx? à answer = focal segmental

glomerulosclerosis (FSGS); most common cause of nephrotic syndrome in sickle cell.

- Osteomyelitis organism in sickle cell? à answer = Salmonella.

- 3F + sickle cell + foot pain for three weeks + fever of 103F + high WBCs; Dx? à answer =

osteomyelitis; acute sickling crisis is wrong answer; difficult Q, as low-grade fever, leukocytosis, and

pain over many weeks can be seen in sickling crisis, however fevers >101F suggest infection;

osteomyelitis may present with pain over many weeks.

- 16F + sickle cell; what is this patient at increased risk of? à answer = cholelithiasis; USMLE loves this

one à ­ RBC turnover à ­ unconjugated bilirubin production à ­ risk of pigmented gall stones

(calcium bilirubinate).

MEHLMANMEDICAL.COM 22
MEHLMANMEDICAL.COM

- Tx for sickle cell + what’s the mechanism? à answer = hydroxyurea; mechanism = ­ production of

fetal hemoglobin (HbF); hydroxyurea is a ribonucleotide reductase inhibitor à inhibits pyrimidine

synthesis.

- “Why do those with sickle cell trait and anemia have ­ resistance to malaria?” à RBC lifespan is

reduced due to ­ RBC turnover, so intra-erythrocytic lifecycle of the protozoan is disrupted.

- How to Dx malaria? à answer = thick and thin blood smears.

- Which type of malaria is the “worst?” à answer = Plasmodium falciparum because it causes cerebral

malaria (microthrombosis and hemorrhage).

- Malaria prophylaxis? à chloroquine, mefloquine.

- MOA of chloroquine/mefloquine? à inhibit Plasmodium heme polymerase.

- 20F + went to Africa + took chloroquine prophylaxis + got malaria anyway; why? à answer =

chloroquine resistance; wrong answer is medical noncompliance. Chloroquine resistance is HY.

- 20F + has malaria + treated appropriately + several weeks later has a resurgence of the malaria; why?

à answer = “presence of extra-erythrocytic form of organism” à P. vivax and ovale cause

hypnozoites, which are a latent intrahepatic form of the disease.

- 20F + has P. vivax + treated with primaquine; why? à answer = “primaquine kills hypnozoites.”

- 20F + comes back from Africa with hemolytic disease + smear is shown; Dx?

o Answer = malaria; smear shows ring form of malaria.

- 20F + lives in Connecticut + has a hemolytic disease; smear is shown; Dx?

MEHLMANMEDICAL.COM 23
MEHLMANMEDICAL.COM

o Answer = Babesia; smear shows ring form.

o Babesia causes a hemolytic disease similar to malaria but the patient will not have left the

USA. This type of Q can be tricky because both malaria and Babesia can have a similar-

appearing ring form. Bottom line:

§ Hemolytic disease + ring form on smear + patient went to Africa, South America, or

Asia; answer = malaria.

§ Hemolytic disease + ring form on smear + patient never left the United States;

answer = Babesia, not malaria.

- 22F + comes back from Africa with hemolytic disease + smear is shown; Dx?

o Answer = malaria; smear shows schizont form.

- 22F + lives in Connecticut + has a hemolytic disease; smear is shown; Dx?

o Answer = Babesia; smear shows characteristic Maltese cross.

- “What is HbC?” à just another type of Hb disorder where glutamic acid à lysine (not valine); lysine is

positively charged, so it migrates the least far on Hb electrophoresis, as it is most attracted to the –

charge at the origin (gel goes from - à +).

MEHLMANMEDICAL.COM 24
MEHLMANMEDICAL.COM

- What variables shift the Hb-O2 curve to the right? à answer = ­ temperature, ­ CO2, ­ H+ (i.e., ­

acidity), ¯ HCO3-, ­ 2,3-BPG.

- What does a Hb-O2 curve shift to the right mean? à answer = increased unloading of oxygen at

tissues.

- Does anything shift the curve to the left? à notably fetal hemoglobin (HbF) has left-shifted curve.

- Why is HbF (a2c2) left-shifted compared to adult hemoglobin (HbA1; a2b2)? à The beta chain on

adult hemoglobin has a charged histidine that forms an ionic bond with 2,3-BPG; this histidine is

replaced with an uncharged serine on the gamma chain of HbF that does not bind 2,3-BPG.

- “Why can deoxygenated blood carry more CO2 for a given pCO2 than oxygenated blood?” à answer

= “deoxyhemoglobin is a better buffer of hydrogen ions than oxyhemoglobin.” Student immediately

says wtf? à At peripheral tissues, CO2 production is ­; this diffuses into the RBC, combines with

water to make H2CO3, which then equilibrates to bicarb and a proton. The proton hops onto the

deoxygenated hemoglobin – i.e., the deoxyhemoglobin acts as a buffer for protons in the blood. At

the same time, the bicarb moves out of the RBC into the plasma, and chloride moves into the RBC to

balance charge. This is referred to as chloride shift. Key points:

o At the tissues, CO2 moves into RBCs.

o CO2 + H2O « H2CO3 « HCO3- + H+.

o HCO3- leaves the RBC. Cl- moves into RBC to balance charge. H+ hops onto deoxy-Hb.

o Therefore most CO2 in the blood is carried as bicarbonate in the plasma. Students tend to

erroneously pick “bicarb in the RBC” because it sounds weird so they think it’s right. But the

answer is bicarb in plasma.

MEHLMANMEDICAL.COM 25
MEHLMANMEDICAL.COM

o At the lungs, this process reverses, where H+ hops off the Hb, Cl- leaves the RBC, HCO3-

enters the RBC. HCO3- + H+ « H2CO3 « CO2 + H2O. CO2 then leaves the RBC and is

exhaled.

- 43M + works as a butcher + recent fatigue + low O2 sats; Dx? à answer = methemoglobinemia; can

be caused by nitrates used to preserve meats.

- Mechanism of methemoglobinemia? à Fe on hemoglobin is normally 2+ charge state (ferrous); it

becomes oxidized to Fe3+ (ferric), which does not bind O2 as well à Hb becomes desaturated.

- 28F + goes hiking and drinks mountain water + “brown blood”; Dx? à answer = methemoglobinemia

caused by nitrates/nitrates (yes, both can cause it) found in mountain/river water; “brown blood” is

seen in methemoglobinemia; in contrast “cherry red blood” (or lips) is seen in CO poisoning.

- 3F + “brown blood”; mechanism for her disease? à answer = “congenital methemoglobinemia

caused by deficiency of cytochrome B5 reductase.”

- Tx for methemoglobinemia? à answer = IV methylene blue + vitamin C.

- Pulse oximetry finding in methemoglobinemia? à answer = low (80s%).

- 34M + fatigue + moved into a new house in winter with an old ventilator; Tx? à answer = hyperbaric

oxygen for CO poisoning.

- 34M + light-headedness + was hanging out on moored boat while the engine was running; Dx? à CO

poisoning.

- Mechanism for CO poisoning? à CO has ­­ higher affinity for Hb than O2, so maximal O2 binding to

Hb is impaired.

- Pulse oximetry finding in CO poisoning? à answer = pulse oximetry is normal is CO poisoning à

standard pulse oximeters can only read the gas bound to Hb, period; they can’t distinguish whether

it’s O2 or CO; specialized CO oximeters do exist that can distinguish.

- What is normal pulse oximetry finding? à 98-100% saturation. However values as low as 94% can be

considered acceptable.

- 34M + new-onset fatigue + lives in house in winter with new ventilator + also just purchased second-

hand refrigerator + pulse oximetry 94%; Q asks “What in this patient’s house is causing his

condition?” à answer = ventilator à Dx is CO poisoning; the mention of the refrigerator in the above

Q is a distractor (for those thinking cyanide toxicity, the association is not classic); student says, “Wait,

MEHLMANMEDICAL.COM 26
MEHLMANMEDICAL.COM

I thought you just said pulse oximetry is normal in CO poisoning. Isn’t normal 98-100%?” à Yes, but

Qs have been known to have Hb saturation as low as 94% in CO poisoning.

- Tx for CO poisoning? à answer = hyperbaric oxygen.

- How does CO affect the Hb-O2 curve? à down-shifts it.

- 34M + housefire + confusion + burned upholstery + O2 sats normal; Dx? à answer = cyanide toxicity;

classically caused by inhaling fumes from burned upholstery.

- 34M + BP of 250/130 + confusion; Dx? à hypertensive encephalopathy. You say, “What? How does

that relate to heme/onc?” Stay with me here:

- 34M + BP of 250/130 + no confusion; sodium nitroprusside is administered then develops confusion;

Dx? à answer = cyanide toxicity, not hypertensive encephalopathy; CN toxicity can be caused by

sodium nitroprusside. Dumb trick, but on USMLE.

- Mechanism for CN toxicity? à binds to cytochrome oxidase and “inhibits transfer of electrons to

molecular oxygen”; that is: it interferes with the electron-transport chain.

- Tx for cyanide toxicity? à answer = amyl nitrite (answer on NBME) à the nitrites cause

methemoglobinemia; cyanide binds readily to Fe3+, forming cyanmethemoglobin, thereby releasing

the CN from cytochrome oxidase. Sodium thiosulfate and hydroxocobalamin are also used.

- 4F + leukocytes 78,000/uL (87% lymphocytes) + low Hb + low platelets + lymphadenopathy; Dx? à

answer = acute lymphoblastic leukemia (ALL) à pretty much always the answer for leukemia in peds

à normal leukocyte count is 4-11,000/uL à high leukocytes in peds with lymphocyte-predominant

spread should scream ALL. Leukemia is usually B cell.

ALL; lymphoblasts appear smooth and relatively uniform.

- 6M + WBCs 56,000 (mostly lymphocytes) + flushing of the face; Dx? à T cell ALL (TALL) à if SVC-like

syndrome is seen due to thymic lesion, answer is T cell, not B cell, variant.

MEHLMANMEDICAL.COM 27
MEHLMANMEDICAL.COM

- 3F + one-month Hx of fatigue + WBCs 3500 + low Hb + low platelets + lymphadenopathy; next best

step in Dx? à answer = bone marrow aspiration à Dx = ALL. Student says, “I thought you just said

though that WBCs would be high.” à can rarely have normal or low leukocyte counts; one NBME Q

has vignette resembling aplastic anemia + lymphadenopathy à answer is bone marrow aspiration,

which is confirmatory for ALL. The non-acute presentation suggests leukemia over viral-induced

aplastic anemia.

- 3M + trisomy 21 + pancytopenia + examination of bone marrow will show what? à answer = excess

lymphoblasts à increased risk of ALL in Down syndrome (and AML type VII, but in peds the answer is

almost always ALL).

- 20M + lymphadenopathy + low platelets + smear is shown; Dx?

o Answer = acute myelogenous leukemia (AML) à image of Auer rods is exceedingly HY for

the Step; Auer rods are composed of myeloperoxidase, which is a blue-green heme-

containing pigment; Tx of AML leads to lysis of cells à Auer rods released into circulation

and precipitate DIC.

- Tx of the leukemia + increased serum uric acid levels; Dx? à answer = tumor lysis syndrome; need to

know the arrows à answer = ­ potassium; ¯ bicarb; ¯ calcium; ­ phosphate; ¯ CO2; variable sodium;

­ uric acid. Xanthine oxidase inhibitors can help prevent (i.e., allopurinol, febuxostat); do not give

these agents if 6-mercaptopurine or azathioprine are being used in Mx (require XO for breakdown).

- 56M + ­­ WBCs + t(15;17); Dx + Tx? à answer = acute promyelocytic leukemia (AML type M3); Tx is

all-trans retinoic acid (vitamin A).

- 82F + fever 103F + gram (+) diplococci on sputum sample + WBCs 82,000 (87% lymphocytes); Dx +

next best step in management? à answer = chronic lymphocytic leukemia (CLL); next best step à

MEHLMANMEDICAL.COM 28
MEHLMANMEDICAL.COM

NBME answer = “quantitative immunoglobulin assay” à apparently ordered for those with chronic

infections when IgG is low (can be seen in leukemia). HY points:

o Whenever I ask students about this type of vignette, they’ll always say it’s Strep pneumo

causing pneumonia. But if this were the Dx in isolation, WBC count should only be about 11-

20,000 at most; if the patient is septic, maybe upwards of 25-30,000. So when you see WBCs

>30,000, you really need to say, “Oh shit ok, that’s leukemia as the underlying Dx. And this

patient just happens to have an infection due to immunodeficiency.”

o In addition, Strep pneumo causes an extracellular bacterial infection, so the shift should be

toward neutrophils (normal range ~55-60%, where bacterial shift would be ~65-90%), so if

the shift is toward lymphocytes, that should scream ALL or CLL. ALL would be the answer for

peds, however, not adults.

o Important you’re tangentially aware that pertussis can cause WBCs >30,000 with a

lymphocyte shift; resembles ALL in peds; always a wtf finding when you first learn of it; but

just say: “Ok, super high WBCs with lymphocyte shift à ALL, CLL, or pertussis.”

- 55M + high WBCs + Coombs test positive + smear is shown; Dx?

o Answer = CLL; smear shows smudge cells; CLL sometimes associated with autoimmune

hemolytic anemia (usually warm), hence a (+) Coombs test may be seen.

o “Warm? What?” à Relax. Warm vs cold autoimmune hemolytic anemia (AIHA) means

you’ve got either IgG (warm) or IgM (cold; Mmm ice cream) against RBCs.

o Cold AIHA (aka cold agglutinin disease) is seen sometimes with mycoplasma or CMV

infection, where the patient can have IgM antibodies against RBCs and a hemolytic anemia

(e.g., CMV infection + low Hb).

MEHLMANMEDICAL.COM 29
MEHLMANMEDICAL.COM

o Positive Coombs test = the patient has IgM or IgG antibodies against RBCs à whatever RBC

issue the patient has, it’s due to antibodies.

- “Wait, can you explain Coombs test real quick?” à direct vs indirect types; direct Coombs is taking

the patient’s RBCs and seeing if they agglutinate in vitro using various laboratory antibodies; if the

RBCs agglutinate, this means there were antibodies attached to their surface, and the patient did in

fact have antibody-mediated hemolytic anemia; indirect Coombs is taking the patient’s plasma and

seeing if it induces laboratory RBCs to agglutinate; if agglutination occurs, then the patient had

antibodies in his/her plasma and we know that he/she did in fact have antibody-mediated hemolysis.

- 61F + high WBCs + CD5 and CD23 positivity + positive Coombs test; Dx? à answer = CLL; leukemic

cells in CLL can be CD5 and/or CD23 positive (on retired Step 1 NBME).

- 44F + WBCs 14,500 + metamyelocytes and myelocytes seen on FBC + smear is shown; Dx?

o Answer = CML; smear shows “motley mix” of many different types of cells (HY image).

- 44F + WBCs 32,000 + metamyelocytes and myelocytes seen on FBC + urinalysis shows nitrites and

leukocyte esterase + smear is shown; Dx?

o Answer = leukemoid reaction; smear shows neutrophilia consistent with infection (UTI in this

case); answer can also be written on NBME as “reactive granulocytosis.”

MEHLMANMEDICAL.COM 30
MEHLMANMEDICAL.COM

o Leukemoid reaction = increased release of leukocytes from bone marrow reserve pool. WBC

count will be >30k. There is NBME Q where it’s 32k.

- 44F + WBCs 14,500 + metamyelocytes and myelocytes seen on FBC + decreased leukocyte ALP; Dx? à

answer = chronic myelogenous leukemia (CML); metamyelocytes and myelocytes are extremely HY

for CML; leukocyte ALP is also decreased.

- 44F + WBCs 32,000 + metamyelocytes and myelocytes seen on FBC + increased leukocyte ALP; Dx? à

answer = leukemoid reaction (inflammatory process; usually infection); the other HY condition that,

like CML, can present with metamyelocytes and myelocytes, however leukocyte ALP is increased.

- 44F + WBC 180,000 (50% neutrophils) + t(9;22); Dx? à answer = CML à Philadelphia chromosome à

t(9;22) bcr/abl à oncogenic tyrosine kinase.

- Tx of CML? à answer = imatinib.

- Important side-effect of imatinib? à answer = fluid retention (edema); tangentially, apart from

imatinib, know that dihydropyridine calcium channel blockers (nifedipine, amlodipine, etc.) also cause

fluid retention/edema.

- 54M + lytic bone lesions + electron microscopy is shown with cells that are CD1a positive; Dx?

o Answer = Langerhan cell histiocytosis; characteristic tennis racquet-shaped cells are called

Burbeck granules.

- 17M + fever + tonsillar exudates + cervical lymphadenopathy + confluent ulcerations seen in posterior

oropharynx; this pathogen can also cause what? à answer = hemolytic anemia; patient has

mononucleosis; 90% of the time, it’s caused by EBV; but 10% is CMV; linear (confluent) ulcers = CMV;

CMV is a known cause of cold AIHA.

MEHLMANMEDICAL.COM 31
MEHLMANMEDICAL.COM

- 9M + African-American + abdominal mass growing left of umbilicus + fever + night sweats + weight

loss + cytogenetic analysis reveals t(8;14); Dx? à answer = Burkitt lymphoma; students says, “Wait, I

thought it was supposed to be a jaw lesion.” à can be intra-abdominal or jaw.

- 9M + African-American + unilateral jaw swelling + t(2;8) + biopsy of lesion is shown; Dx?

o Answer = Burkitt; image is classic “starry sky” appearance; blue cells are lymphocytes; clear

cells are macrophages.

o NBME Q for Step 1 points to a macrophage on the starry sky histo and asks what cellular

process is occurring; answer = apoptosis à macrophages are called tingible (not tangible)

body macrophages à contain lots of phagocytosed cells at various stages of apoptosis.

o USMLE Step 1 assesses t(2;8), t(8;14), and (8;22) for Burkitt; resources tend to only focus on

t(8;14); easy way to remember: you can see that chromosome 8 is involved in all of them.

- 55M + fever + night sweats + weight loss + fluid wave in abdomen + abdominal paracentesis yields

300mL of milky fluid; Dx? à chylothorax secondary to intra-abdominal Burkitt lymphoma.

- Gene involved in Burkitt + what is it? à c-myc à transcription factor.

- 26M + waxing and waning neck mass over one-year period + t(14;18); Dx? à answer = follicular

lymphoma; may present with waxing/waning disease.

- Gene involved in follicular lymphoma + what is it? à answer = bcl-2 à anti-apoptotic molecule.

- 41F + lymphoma + t(11;14); Dx? à answer = mantle cell lymphoma.

- 9F + WBC 3,500 + smear shows WBCs with cytoplasmic projections that stain positive for acid-

resistant acid phosphatase; Dx? à answer = hairy cell leukemia; can have low WBC count.

- 72M + acute-onset highly aggressive B cell lymphoma; Dx? à diffuse-large B cell lymphoma (DLBCL);

most common non-Hodgkin lymphoma (NHL) in adults; highly aggressive.

MEHLMANMEDICAL.COM 32
MEHLMANMEDICAL.COM

- “What does non-Hodgkin vs Hodgkin mean?” à Hodgkin is a type of lymphoma that has

pathognomonic Reed-Sternberg cells on lymph node biopsy (owl-eye appearance); RS cells are CD15

and CD30 positive; some resources say NHL has B-symptoms (fever, night sweats, weight loss)

whereas in Hodgkin they are more rare, but on the USMLE, B-symptoms can occur in either; EBV can

also cause either; Hodgkin may present with contiguous spread, where affected areas are in close

proximity, whereas NHL can spread more haphazardly. NHL just refers to any lymphoma that doesn’t

have Reed-Sternberg cells.

Reed-Sternberg cells

- “Do I need to know the different types of Hodgkin lymphoma?” à highest yield points:

o Nodular sclerosing variant is more common in women.

o More Reed-Sternberg cells = poorer prognosis.

§ Leukocyte-rich Hodgkin = high lymphocytes + low RS cells (better prognosis).

§ Leukocyte-deplete Hodgkin = low lymphocytes + high RS cells (poor prognosis).

- 44M + Hx of Hodgkin disease + nephrotic syndrome; what’s the renal Dx? à answer = minimal

change disease (MCD); student says, “Wtf? But I thought that’s pediatrics.” à 9 times out of 10, yes,

it follows viral infection in a kid, but it is also associated with Hodgkin lymphoma in adults.

- 16F + painless lateral neck mass + a mediastinal mass; Dx? à answer = Hodgkin disease

- 16F + painless lateral neck mass + hepatomegaly; Dx? à answer= Hodgkin disease à classic vignette

presents as painless lateral neck mass (doesn’t wax and wane like follicular NHL) + either a

mediastinal mass or hepatomegaly; the mediastinal mass is not a thymoma; it’s mediastinal

lymphadenopathy.

MEHLMANMEDICAL.COM 33
MEHLMANMEDICAL.COM

- 49M + jaundice + high ALP + pancreatic enzymes normal + weight loss + painful erythematous areas

on arms and legs; Dx? à pancreatic head adenocarcinoma causing migratory thrombophlebitis

(Trousseau sign of malignancy).

- 50M + hepatitis C + purpura on arms and legs + joint pain + low complement C4; Dx? à answer =

cryoglobulinemia.

- “Wtf is a cryoglobulin?” à cryoglobulins are immunoglobulins that precipitate at “cold” temperatures

(i.e., <37C); cryoglobulinemia can be caused by malignancy as well as chronic infections (HepC, HIV).

C4 can be decreased due to activation of the complement cascade, notably C4. Do not confuse

cryoglobulinemia with cold agglutinin disease, which is aka cold autoimmune hemolytic anemia.

- 22M + vegetarian/vegan + ­ MCV + smear is shown; Dx?

o Answer = dietary B12 deficiency; smear shows hypersegmented neutrophil.

- “What is a hypersegmented neutrophil?” à seen in folate (B9) or B12 deficiency; ¯ DNA synthesis

results in neutrophils with >3 segments to the nuclei. Essentially if you see these on a smear, right

away you should be thinking B9 or B12 deficiency; patient will also have ­ MCV.

- 22M + vitiligo + vegetarian + ­ MCV + hypersegmented neutrophils; Dx? à answer = pernicious

anemia causing B12 deficiency (one autoimmune disease à ­ risk of others [polyglandular

syndromes]).

- 22M + vitiligo + vegetarian + ­ MCV + hypersegmented neutrophils; next best step? à answer =

check serum antibodies against parietal cells and intrinsic factor.

- 27F + strict vegetarian diet for 5 years + ¯ Hb + ¯ Hct + ¯ WBCs + ¯ platelets + no other info given; Q

asks, her nutrient deficiency significantly impairs which of the following cellular processes; answer =

DNA synthesis (B12 deficiency); wrong answer is heme production (instead B6 deficiency).

MEHLMANMEDICAL.COM 34
MEHLMANMEDICAL.COM

- 22M + Hx of epilepsy + ­ MCV + hypersegmented neutrophils; Dx? à B9 deficiency à anti-epileptic

meds (i.e., valproic acid, phenytoin, carbamazepine) cause ¯ intestinal folate absorption.

- 44M + alcoholic + ­ MCV + no hypersegmented neutrophils + serum methylmalonic acid and

homocysteine levels normal + smear is shown; Dx?

o Answer = alcohol-induced sideroblastic anemia à alcohol can cause ­ MCV. But since

there’s no impairment of DNA synthesis, there are no hypersegmented neutrophils; alcohol

can merely disrupt the heme synthesis pathway, where MCV can sometimes ­.

o Methylmalonic acid (or methylmalonyl-CoA) is ­ in B12 deficiency; homocysteine is ­ in both

B9 and B12 deficiencies.

- “What is sideroblastic anemia?” à condition characterized by normal iron levels but merely the

inability to incorporate the iron into heme à results in RBC precursors (nucleated erythroblasts)

containing peri-nuclear iron-laden macrophages that stain blue with Prussian blue stain (“ringed

sideroblasts”). The visible iron aggregates are known as Pappenheimer bodies.

- “Do I need to know the heme synthesis pathway?” à Annoying, but yes. The heme pathway

disorders are all over the NBMEs for Step 1.

MEHLMANMEDICAL.COM 35
MEHLMANMEDICAL.COM

- Cause of sideroblastic anemia? à Usually X-linked recessive condition due to deficiency of d-

aminolevulinic acid synthase (d-ALA), but can also be acquired, most commonly due to alcohol.

- Classic hematologic parameters in sideroblastic anemia? à ­ serum iron + ­ ferritin + ­ transferrin

saturation + ¯ Hb; MCV can be normal/¯ in XR form or ­ MCV in acquired forms.

o Bottom line: remember ­ iron across the board + MCV can be variable.

- 43M + daily alcohol use + ­ serum iron + ­ ferritin + ­ transferrin saturation + ¯ Hb + ­ MCV; Dx? à

answer = alcohol-induced sideroblastic anemia à alcohol can cause ­ MCV (non-megaloblastic,

unlike B9/B12 deficiencies, which cause megaloblastic).

- 23F + abdominal pain + pink urine + ­ urinary porphobilinogen + ­ d-ALA; Dx? à answer = acute

intermittent porphyria à heme synthesis disorder caused by deficiency of porphobilinogen

deaminase à classically associated with “Port wine-colored urine” (but vignettes can just say pink or

red), abdominal pain, and ­ urinary porphobilinogen; some patients can also have neurologic

findings.

- 19F + weakness of legs + decreased reflexes + severe abdominal pain + persistent vomiting + Port-

wine-colored urine; Q asks, urine studies are most likely to show ­ what? à answer = porphobilogen;

Dx is acute intermittent porphyria.

MEHLMANMEDICAL.COM 36
MEHLMANMEDICAL.COM

- Inheritance pattern of acute intermittent porphyria? à autosomal dominant (on Step 1 NBME).

- 44F + abdominal pain + no mention of urinary findings + paresthesias + alcohol seems to precipitate

episodes; Dx? à answer = acute intermittent porphyria; can be exacerbated by alcohol; one 2CK-level

neuro Q doesn’t mention anything about pink/red urine but mentions neurologic findings.

- Tx for acute intermittent porphyria? à answer = glucose infusion (acutely ¯ heme synthesis); can also

give hematin + heme arginate).

- 34F + recurrent episodes of blistering of face and arms over many years + ­ serum ALT and AST + ­

total serum porphyrin + ­ urine uroporphyrin III; Dx? à answer = porphyria cutanea tarda à heme

synthesis disorder caused by deficiency of uroporphyrinogen III decarboxylase à ­ urinary

uroporphyrin; causes “tea-colored urine” and photosensitivity (i.e., blistering).

- Tx for porphyria cutanea tarda? à answer = ¯ alcohol use (can precipitate Sx) + ¯ sun exposure.

- 40M + episodes of blistering from sun + ­ urine uroporphyrin III; Q asks which compound serves as

the precursor to uroporphyrin in this patient à answer = succinyl-CoA à required for the initiation of

heme synthesis (glycine not listed but by all means also correct).

- 29M from Albania + positive PPD test + negative CXR + started on monotherapy for condition +

develops paresthesias months later; which other finding might be seen in this patient? à answer =

impairment of heme synthesis à patient being treated with isoniazid (INH) for latent TB à INH

causes vitamin B6 deficiency if not supplemented à presents as neuropathy and/or seizures. Vitamin

B6 is needed for the first step of heme synthesis.

- 44M + hunter + recent cognitive decline + microcytic anemia + wrist drop + ­ d-ALA + ­ RBC

protoporphyrin; Dx? à answer = lead poisoning à inhibits ferrochelatase (causes ­ RBC

protoporphyrin) and d-ALA dehydratase (­ d-ALA); lead poisoning classically causes microcytic anemia

(HY finding, especially in adults; vignette is not always going to say kid eating paint chips in family’s

new house); and neuropathy (e.g., wrist drop, foot drop).

- 2M + cognitive decline after family moves into new house; what does the smear show?

MEHLMANMEDICAL.COM 37
MEHLMANMEDICAL.COM

o Answer = basophilic stippling (RNA precipitates) seen in lead poisoning.

- Tx for lead toxicity? à Tx not indicated unless serum levels >44 ng/dL (weirdly specific, but asked on

2CK/3); give dimercaprol or EDTA in adults; succimer is often used for children.

- How does iron toxicity present? à GI bleeding HY on USMLE; can also cause ­ anion-gap metabolic

acidosis (Iron and INH are the I in MUDPILES).

- 32M + red urine sometimes when waking in the morning; Dx? à answer = paroxysmal nocturnal

hemoglobinuria (PNH).

- Mechanism for PNH? à increased complement-mediated hemolysis caused by deficiency of CD55/59

+ deficiency of GPI anchor, which protect RBCs from complement-mediated breakdown.

- Tx for PNH? à eculizumab à monoclonal Ab against complement protein C5.

- Inheritance pattern for hereditary spherocytosis? à answer = autosomal dominant.

- Mechanism for spherocytosis? à answer = deficiency of ankyrin, spectrin, and/or band proteins à

results in cytoskeletal disruption and smaller, more spherical RBCs à Qs will sometimes merely have

“cytoskeleton” as the answer.

- Notable hematologic parameter in spherocytosis? à answer = ­ MCHC (mean corpuscular

hemoglobin concentration) à only time on USMLE you’ll see this variable ­, however do not choose

this for Qs asking you how to Dx spherocytosis. USMLE also wants you to know the spherocytes in

hereditary spherocytosis are normochromic, normocytic.

- How to Dx spherocytosis? à answer = osmotic fragility test; if negative answer = eosin-5-maleimide

test; the latter is a newer flow cytometry test and is now showing up; osmotic fragility test is the next

best step if you’re forced to pick between the two, but it can miss up to 25% of cases.

- Tx for hereditary spherocytosis? à answer = splenectomy for those with moderate-severe anemia;

the spleen normally clears out the spherocytes, thereby enlarging and also causing chronic anemia.

MEHLMANMEDICAL.COM 38
MEHLMANMEDICAL.COM

- 12M + viral infection + spherocytes seen on blood smear + Coombs test positive; Dx? à answer =

hemolytic anemia; wrong answer is hereditary spherocytosis; student immediately says, “Omg

erratum!” It’s not. Chill the fuck out for two seconds. You can get spherocytes in drug-/infection-

induced hemolytic anemia à autoantibodies cross-reacting with RBCs (type II hypersensitivity); the

key is seeing that the Coombs test is positive in drug-/infection-induced spherocytosis because the

patient has antibodies against RBCs; in contrast, hereditary spherocytosis has zero to do with

antibodies; it’s a cytoskeletal problem; so of course the Coombs test is negative. Bottom line: yes, you

can get spherocytes in things other than spherocytosis, namely drug-/infection-induced AIHA.

- 12M + viral infection + spherocytes seen on blood smear + Coombs test negative; Dx? à answer =

hereditary spherocytosis.

- 8M + chronic anemia + father had Hx of splenectomy + smear is shown; inheritance pattern?

o Answer = autosomal dominant; smear shows spherocytes. They love this detail about one of

the parents having had splenectomy. I’ve seen students incorrectly answer sickle cell for this

Q; sickle cell causes autosplenectomy; it isn’t treated with splenectomy.

- 40M + recent splenectomy; Q asks, what does the smear show here?

o Answer = Howell-Jolly bodies within RBCs, which are nuclear remnants; normal finding post-

splenectomy.

MEHLMANMEDICAL.COM 39
MEHLMANMEDICAL.COM

- Neonate + pathologic jaundice + father had Hx of splenectomy for chronic anemia; Dx? à answer =

hereditary spherocytosis.

- 11F + fatigue + Hb 6.5 + MCV 90 + reticulocyte count 9% (NR 0.5-1.5% of RBCs) + mother underwent

splenectomy as a youth for “low blood” and recently had a cholecystectomy; what’s the most likely

mechanism for this patient’s condition? à answer = deficiency of erythrocyte spectrin.

- “When is reticulocyte count elevated?” à normal range is 0.5-1.5% of RBCs (according to NBME); if

high, indicates hemorrhage, hemolytic anemia, or ­ RBC turnover (i.e., hereditary spherocytosis;

enzyme deficiencies; transfusion reactions; thalassemia when accompanied by severe anemia); if

normal or low, then iron deficiency; aplastic anemia; thalassemia not accompanied by severe anemia.

- 18-month-old girl + scleral icterus + pallor + hepatosplenomegaly + Hb 5.6 g/dL + bilirubin 3 mg/dL

(normal ~1.0) + smear shows severe hypochromia + nucleated erythrocytes + microcytosis + DNA

analysis shows mutation in beta-globin gene; what’s the Dx + what arrows do you expect for HbF,

HbA2/HbA1 ratio, reticulocyte count; answer = ­ HbF, ­ HbA2/HbA1 ratio, ­ reticulocyte count; Dx is

beta-thalassemia accompanied by severe anemia + RBC turnover.

- 12M + treated for infection + smear is shown; Dx?

o Answer = glucose-6-phosphate dehydrogenase (G6PD) deficiency; left smear shows

degmacytes (“bite cells”); right smear shows Heinz bodies.

o Bite cells are pathognomonic for G6PD deficiency à spleen phagocytoses part, but not all, of

RBC, resulting in characteristic “bitten” appearance on smear. Heinz bodies are precipitated /

oxidized hemoglobin seen toward the periphery of the RBC.

- Mechanism for G6PD deficiency? à G6PD needed to make NADPH, which acts as a reducing agent

(i.e., counteracts oxidation) to protect RBCs à if ¯ NADPH à ­ oxidation of RBCs à RBCs prone to

destruction in setting of stressors such as infection, drugs (i.e., dapsone), or foods (i.e., fava beans).

MEHLMANMEDICAL.COM 40
MEHLMANMEDICAL.COM

- Inheritance pattern of G6PD deficiency? à answer = X-linked recessive (HY).

- Neonatal girl + pathologic jaundice + hemolytic disease due to enzyme deficiency; Dx? à answer =

pyruvate kinase deficiency à second-most common cause of hereditary hemolytic anemia due to an

enzyme deficiency (after G6PD deficiency); since G6PD is XR, you know in a girl it can’t be the answer.

- 3F + failure to thrive + blood smear and enterocyte biopsy are both shown; Dx?

o Answer = abetalipoproteinemia; blood smear shows acanthocytes (spur cells); biopsy of

enterocytes shows large, clear fat droplets due to malabsorption (apo-B48 needed for

absorption from bowel).

- 82F + found in house unconscious during summer day + body temperature is 105F + blood smear

shows acanthocytes; Dx? à answer = liver failure; abetalipoproteinemia is the wrong answer; student

says wtf? à heat stroke = end-organ failure secondary to hyperthermia; heat exhaustion = fatigue,

but no end-organ failure secondary to hyperthermia; acanthocytes (aka spur cells) can be seen in

both liver failure and abetalipoproteinemia; USMLE loves liver failure secondary to heat stroke as a

cause of acanthocytosis.

- 2M + SCID + requires blood transfusion for severe anemia; what kind of blood products are most

appropriate? à answer = irradiated packed red blood cells.

MEHLMANMEDICAL.COM 41
MEHLMANMEDICAL.COM

- “Do I need to know transfusion reactions?” à Annoying but yes. HY point are as follows:

- What are the important ADP2Y12 receptor blockers? à clopidogrel, prasugrel, ticagrelor, ticlopidine.

- What are the important GpIIb/IIIa inhibitors? à abciximab, eptifibatide, tirofiban.

- Drugs that are both anti-platelet agents and vasodilators? à dipyridamole + cilostazol; both mixed

cAMP and cGMP phosphodiesterase inhibitors.

- Drugs that are direct-thrombin inhibitors? à bivalirudin, lepirudin, dabigatran, argatroban.

- When are these notably the answer? à once again, for Tx of HIT.

- MOA of fondaparinux? à indirect factor Xa inhibitor (activates antithrombin, causing factor Xa

inhibition).

- MOA of apixaban? à direct factor Xa inhibitor.

- Important fibrinolytic? à tPA à used for ischemic stroke within 3-4.5 hours. Streptokinase also

fibrinolytic.

- Drugs that are anti-fibrinolytics? à tranexamic acid, aminocaproic acid (can help reverse tPA).

- MOA of methotrexate? à reversible, competitive inhibitor of dihydrofolate reductase; first-line

DMARD for RA.

MEHLMANMEDICAL.COM 42
MEHLMANMEDICAL.COM

- Side-effects of methotrexate? à pulmonary fibrosis, hepatotoxicity, neutropenia.

- How to mitigate toxicity of methotrexate? à folinic acid (leucovorin rescue), not folic acid.

- MOA of 5-fluorouracil (5-FU)? à thymidylate synthase inhibitor.

- MOA of 6-mercaptopurine (6-MP)? à PRPP amidotransferase inhibitor (purine synthesis inhibitor).

- MOA of azathioprine? à metabolized into 6-MP à then inhibits purine synthesis.

- MOA of hydroxyurea? à ribonucleotide reductase inhibitor (pyrimidine synthesis inhibitor).

- MOA of mycophenolate mofetil? à inhibits IMP dehydrogenase (inhibits purine synthesis).

- Microtubule inhibitors? à colchicine, -bendazoles (mebendazole, albendazole), vincristine,

vinblastine, taxanes (paclitaxel/docetaxel), griseofulvin.

- Which microtubule inhibitor is the odd one out? à paclitaxel/docetaxel à hyperstabilize

microtubules; the others inhibit formation.

- Toxicity of vincristine? à neurotoxic.

- MOA of bleomycin? à causes free radical formation.

- Toxicity of bleomycin? à pulmonary fibrosis.

- MOA of doxorubicin (Adriamycin) / daunorubicin? à DNA intercalators + cause free radicals.

- Toxicity of doxorubicin/daunorubicin? à dilated cardiomyopathy.

- How to mitigate the toxicity of doxorubicin/daunorubicin? à dexrazoxane à chelates free radicals.

- MOA of erlotinib? à EGFR tyrosine kinase inhibitor.

- Notable use of erlotinib? à non-small cell lung cancer (for some reason USMLE cares).

- MOA of cetuximab? à monoclonal Ab against EGFR.

- MOA of imatinib? à bcr/abl tyrosine kinase inhibitor for CML à causes fluid retention (edema).

- MOA of tamoxifen/raloxifene? à selective estrogen-receptor modulators (SERMs).

- Important distinction between tamoxifen and raloxifene? à both are antagonists at ER receptors on

breast tissue and are agonists on bone, but only tamoxifen is partial agonist on endometrium (­ risk

of endometrial cancer with tamoxifen).

- MOA of trastuzumab (Herceptin)? à monoclonal Ab against HER-2/neu (ERBB2).

- Toxicity of trastuzumab? à cardiotoxicity.

- MOA of irinotecan/topotecan? à topoisomerase I inhibitors.

- MOA of etoposide/teniposide? à topoisomerase II inhibitors.

MEHLMANMEDICAL.COM 43
MEHLMANMEDICAL.COM

- MOA of cyclophosphamide? à guanine N7 alkylating agent.

- Major side-effect of cyclophosphamide? à hemorrhagic cystitis due to metabolite called acrolein.

- How to mitigate toxicity of cyclophosphamide? à Mesna (contains thiol -SH group).

- MOA of tacrolimus? à antagonist at FK506 receptor à decreases intracellular calcineurin à

decreases IL-2 transcription à decreases T cell function.

- Important side-effects of tacrolimus à type II diabetes, nephrotoxicity.

- MOA of cyclosporin? à antagonist at cyclophilin receptor à decreases intracellular calcineurin à

decreases IL-2 transcription à decreases T cell function.

- Important side-effects of cyclosporin à nephrotoxicity, hypertension, gingival hyperplasia,

hypertrichosis.

- MOA of sirolimus à antagonist at mTOR à does not decrease intracellular calcineurin à decreases

responsiveness to IL-2 à decreases T cell function.

- Important side-effect of sirolimus à dyslipidemia; notably not nephrotoxic.

o Cyclosporin + tacrolimus à ¯ calcineurin + ¯ IL-2 transcription + nephrotoxic.

o Sirolimus à no change calcineurin + ¯ responsiveness to IL-2 + not nephrotoxic.

- Important points about cisplatin à causes oto- and neurotoxicity; all over 2CK-level neuro shelf

exams as causing “toxic neuropathy” (chemo-induced neuropathy).

- How to mitigate toxicity of cisplatin? à saline infusion (NaCl, were the Cl- helps), followed by

amifostine.

- MOA of rituximab? à CD20 inhibitor on B cells.

MEHLMANMEDICAL.COM 44
MEHLMANMEDICAL.COM

YouTube
@mehlmanmedical

Instagram
@mehlman_medical

MEHLMANMEDICAL.COM 45
MEHLMANMEDICAL.COM

MEHLMANMEDICAL
HY HEME/ONC

All material is copyrighted and the property of mehlmanmedical.

Copyright © mehlmanmedical

MEHLMANMEDICAL.COM 46
MEHLMANMEDICAL
HY PSYCH
MEHLMANMEDICAL.COM

YouTube
@mehlmanmedical

Instagram
@mehlman_medical

MEHLMANMEDICAL.COM 2
MEHLMANMEDICAL.COM

HY Psych

Purpose of this review is not to be an unabridged, superfluous, 700-page psych textbook with absolutely every detail

catered to; the purpose is to increase your USMLE and Psych shelf scores via concise factoid consolidation.

- 12M + sore throat + new-onset tic; Dx? à PANDAS (Pediatric Autoimmune Neuropsychiatric Disorder

Associated with Streptococcus); Group A Strep infection can lead to tic, ADHD, OCD; question on

student’s USMLE asked for “anti-streptolysin O titer” as the answer to help assess etiology in kid with

a tic starting after a sore throat.

- 7F + facial grimaces past 5 months + no other motor findings or abnormal sounds + mental status

normal; next best step in Mx? à answer = “schedule a follow-up examination in 3 months” à Dx =

provisional tic disorder à 1/5 children experience some form of tic disorder; most common ages 7-

12; usually lasts less than a year; “watch and wait” approach recommended. Provisional tic disorder

used to be called transient tic disorder; the name was changed because a small % go on to develop

chronic tics.

- 65M + given IV methylprednisolone for temporal arteritis + develops confusion + visual hallucinations;

Dx? à answer = “corticosteroid-induced psychotic disorder”; astute student says, “I thought that

happens with high doses over longer periods of time.” à response: yeah, but Psych NBME has it

occurring after a one-off dose in a patient. Bottom line is: be aware that glucocorticoid psychosis is

tested.

- 8M + 1-year Hx of eye-blinking + facial grimacing + throat-clearing; most appropriate pharmacologic

therapy? à Psych shelf answer = risperidone; can Tx with antipsychotics, alpha-2 agonists (clonidine),

CBT.

- 82M + confusion + on various meds; Dx? à various answers on NBME are: “discontinuation of

anticholinergic medications”; “discontinuation of diphenhydramine” (1st gen H1 blocker);

“discontinuation of amitriptyline”; “discontinuation of doxepin”; “discontinuation of desipramine” (all

TCAs) à TCAs, 1st generation H1 blockers, and 2nd generation antipsychotics (atypicals) all cause a

triad of side-effects:

o Anti-cholinergic (anti-muscarinic)

o Anti-alpha-1-adrenergic

MEHLMANMEDICAL.COM 3
MEHLMANMEDICAL.COM

o Anti-H1-histaminergic

- “What do you mean by anticholinergic effects of meds?” à Start with knowing that DUMBBELSS is a

mnemonic for cholinergic effects: Diarrhea, Urination, Miosis (pupillary constriction), Bradycardia,

Bronchoconstriction, Excitation (neuromuscular), Lacrimation, Salivation, Sweating à so by anti-

cholinergic effects, it’s just the opposite of DUMBBELSS: constipation, urinary retention, mydriasis,

tachycardia, bronchodilation not seen (M3 agonism can bronchoconstrict, but dilation is sympathetic

beta-2-regulated), Flaccidity not seen, xerophthalmia (dry eye), xerostomia (dry mouth), anhidrosis.

- “What do you mean by anti-alpha-1-adrenergic?” à alpha-1 normally constricts peripheral arterioles

à anti-alpha-1 effects can precipitate orthostatic hypotension + fainting.

- “What do you mean by anti-H1-histaminergic effects?” à sedation.

- 8M + develops visual hallucinations after starting on over-the-counter cold med provided by his

mother; Dx? à anticholinergic delirium caused by diphenhydramine or dextromethorphan (anti-

tussive opioid).

- 82M + urinary hesitancy + interrupted stream + taking amitriptyline; next best step? à discontinue

amitriptyline.

- 25M + started on new psych med + is now hot and dry; Dx? à anticholinergic effects of TCA.

- 35F + family member Dx with breast cancer + says she can’t sleep; Q asks which med to give à

answer on Psych NBME = clonazepam à benzo OK for acute Tx of insomnia; never prescribe

chronically.

- 35M + recently divorced + stressed + insomnia; which med to give: answers are SSRI, antipsychotic,

and zolpidem; answer = zolpidem à non-benzo zolpidem may be used short-term for insomnia. Take

home point is be aware both benzo + non-benzo can be used acutely for insomnia.

- 52F + 8-week Hx of progressive confusion and memory loss + myoclonus; Dx? à Creutzfeldt-Jakob

syndrome (prion disease).

- 65M + visual hallucinations + bradykinesia + gradual cognitive decline; Dx? à Lewy body dementia.

- 65M + pulls his pants down when guests come over to the house + apathy; Dx? à frontotemporal

dementia (Pick disease) à characterized by apathy, disinhibition, personality change.

- 65M + Parkinsonism + axial dystonia; Dx? à answer = progressive supranuclear palsy (student had

this on the USMLE).

MEHLMANMEDICAL.COM 4
MEHLMANMEDICAL.COM

- 65M + wet, wobbly, wacky + Parkinsonism; Dx? à normal pressure hydrocephalus (NPH) à urinary

incontinence, ataxia, cognitive dysfunction; key point to make here is the Parkinsonism. Wiki it

yourself, you’ll see.

- Mechanism for incontinence in NPH? à answer = “failure to inhibit the voiding reflex.”

- Area of the brain affected in Huntington? à answer on shelf = caudate nucleus.

- 22M + hyperoralism + hyperphagia + docility; Dx? à Kluver-Bucy syndrome à bilateral amygdala

lesions.

- Cause of Kluver-Bucy syndrome? à most often due to HSV encephalitis.

- 49M + Down syndrome + forgetfulness; which part of brain is affected? à answer = nucleus basalis of

Maynert à high-density of cholinergic neurons (basal forebrain) à affected in Alzheimer (early-onset

in Down).

- Where in the brain is there high amount of norepinephrine production? à locus coeruleus (pons).

- Where in the brain are there high amounts of serotonergic neurons? à raphe nucleus (medial

reticular formation).

- Pharm Tx for Alzheimer? à acetylcholinesterase inhibitors first (donepezil, galantamine,

rivastigmine); sometimes Q will ask for mechanism, and answer = “cholinergic”; for more advanced

disease try NMDA (glutamate) receptor antagonist (memantine).

- 74M + MMSE 22/30 + avoids eye contact + weight loss + low mood; DX and Tx? à pseudodementia

à Tx = sertraline (SSRI), not donepezil.

- Main way to differentiate pseudodementia from dementia? à pseudodementia is depression that

presents as cognitive decline; vignette may describe weight loss or gain, avoidance of eye contact,

low mood, and/or tearing up during interview; vignette may also mention poor performance on the

reverse serial 7s of the MMSE, or the patient is slow drawing a clockface but can rapidly complete it

once prompted (apathy); pseudodementia presents as apathy on MMSE; in contrast, patients with

true dementia try on the MMSE.

- How to differentiate normal aging from Alzheimer on Psych shelf? à biggest point is that patients

who complain or are concerned about their own cognitive decline do not have Alzheimer; classic

example is 68F who frequently says she walks into rooms and can’t remember why she went in there

+ says she accidentally left the burner on in the kitchen last week and had an argument with her adult

MEHLMANMEDICAL.COM 5
MEHLMANMEDICAL.COM

daughter about it à answer = normal aging, not dementia à patient herself is concerned /

complaining, so answer is not dementia on USMLE.

- Other notable causes of reversible cognitive decline? à hypothyroidism, B12 deficiency,

neurosyphilis, neuro Lyme.

- 53M + BMI 25 + mostly quiet during interview + total cholesterol 300 mg/dL + hepatic AST slightly

elevated + HR 60; Dx + next best step in Mx + Tx? à hypothyroidism à check serum TSH à give

levothyroxine (T4); hypothyroidism can cause dysthymia, high cholesterol, and elevated hepatic

transaminases.

- 48F + BMI 26 + cholesterol elevated + HR 55 + creatine kinase (CK) elevated; Dx? à hypothyroidism

à check serum TSH; hypothyroid myopathy can cause proximal muscle weakness + elevated serum

CK.

- 81F + cooks own meals since husband passed away + seems depressed + various non-acute

neurologic findings; next best step? à assess suicide risk (this answer is basically always correct if it’s

listed).

- 81F + memory decline; next best step after assessing suicide risk? à Mini-Mental State Exam

(MMSE).

- 81F + cooks own meals since husband passed away + seems depressed + various non-acute

neurologic findings + MMSE is 25/30 + no suicidal ideation; next best step? à check serum B12 à

subacute combined degeneration (SCD) = pattern of neurologic dysfunction seen in B12 deficiency.

- Which three spinal tracts are involved in SCD? à lateral corticospinal, spinocerebellar, dorsal columns

à just remember that the spinothalamic is not involved.

- 68M + started on various medications 8 weeks ago for low ejection fraction heart failure + now has

depressed mood; most likely cause? à depression caused by beta-blocker.

- Nocturnal enuresis; when is it pathologic? à after age 5.

- 6M + nocturnal enuresis; next best step? USMLE / NBME / shelf wants the following order:

o Behavioral answer first; e.g., spend more time with child; decrease overt stressors as much

as possible.

MEHLMANMEDICAL.COM 6
MEHLMANMEDICAL.COM

o If the above not an answer, do star chart (positive reinforcement therapy; i.e., don’t wet the

bed and get a star; get 5 stars for extra dessert; 100 and we go to Disneyland).

o If star chart not listed or already attempted, next answer is enuresis alarm.

o Medications like imipramine and desmopressin are always wrong; water deprivation after

5pm is also always wrong..

- Students mess these Qs up because they’ll see enuresis alarm as correct on one form, but on a

different form it’s star chart, so know the above order.

- Patient wants to commit suicide; next best step? à admit to hospital involuntarily.

- Acutely suicidal patient + you need to admit him/her; what do you say? à “how would you feel about

entering hospital?”

- 10M + family comes from rural Asian town + family says child episodes in which he is “possessed” and

has episodes of spitting up blood + parents seem well-supportive; next best step in Dx? à EEG is

correct; the wrong answer is contacting child protective services; tongue or cheek biting during a

seizure can lead to post-ictal blood in saliva; students hate this type of question, but Psych shelf asks

Qs assessing sensitivity to cultural beliefs.

- 33M + robbed at gunpoint two days ago + now is mute and not responding to questions; Tx? à

answer = benzo (lorazepam) à catatonia secondary to adverse stimulus.

- 39F + >6-month Hx of multiple worries (i.e., career, marriage, kids going to college, etc.) + no overt

mood or psychotic Sx; Dx? à generalized anxiety disorder (GAD) à Dx is >6 months of general

worries without lack of specific stressor.

- Tx of GAD? à cognitive behavioral therapy (CBT) and/or SSRI; second-line pharm agent is buspirone,

which is a serotonin receptor agonist; USMLE wants you to know buspirone + its mechanism, but also

remember that it’s not first-line for GAD; SSRIs are.

- 22F + not performing well in classes since breaking up with boyfriend 3 months ago; sleeps well; no

weight loss; has low mood; Dx? à adjustment disorder à Dx is socio-occupational dysfunction

(school, work, social life) secondary to specific stressor, but patient must not have mood or psychotic

disorder, otherwise Dx is, e.g., major depressive disorder (MDD), or bipolar, etc. Some Psych shelf

questions will have as answers, e.g., “Adjustment disorder with depressed mood,” or “Adjustment

MEHLMANMEDICAL.COM 7
MEHLMANMEDICAL.COM

disorder with anxious mood,” but in these vignettes the patient won’t have actual MDD or a true

psychotic disorder.

- 82F + MDD + refuses to eat + catatonia; next best step? à answer on Psych shelf = electroconvulsive

therapy (ECT); some ECT indications are: catatonia, pregnancy, refusal to eat or drink, imminently

suicidal, treatment resistance, Hx of ECT response, psychotic features present.

- 33M + ingested substance + high RR + now has calcium oxalate urolithiasis; Dx? à ethylene glycol

toxicity à causes calcium oxalate nephrolithiasis; high RR is due to respiratory compensation for

high-anion gap metabolic acidosis (MUDPILES).

- 16M + found on floor in school bathroom + brought into hospital + sluggish + pupils and vitals normal;

which drug/substance did he do? à answer = butane (inhalant toxicity); this is HY!

- 14M + cognitive decline over a few months + ataxia; which drug/substance did he do? à answer =

glue, not alcohol à 14 is too young to get alcohol-induced cerebellar ataxia.

- 22M takes a drug + gets nystagmus + bellicosity (wants to fight) à answer = PCP.

- 22M takes a drug + gets mutism + has constricted pupils à answer = PCP. Fucking weird but it’s on

the psych NBME for 2CK. If you don’t believe me, you can Google “pcp mutism constricted pupils.”

- 22M + sluggish patient + has angiogram performed (for some reason) that shows decreased cerebral

blood flow; drug that was taken? à answer = cocaine; this question is on USMLE. Cocaine is known to

cause vasoconstriction, i.e., placental abruption, coronary vasospasm; apparently that extends to

cerebral blood flow on the exam.

- 16F + injected conjunctivae + has paranoia; Dx + Tx? à marijuana intoxication; observation.

- Tx for OCD on shelf? à answer = SSRI (i.e., sertraline).

- 23M + hearing voices for 6 weeks + staying up all night for 4 weeks; Dx? à answer = schizoaffective

disorder à Dx is >2 weeks of psychotic disorder in the absence of mood disorder.

- 23M + vignette sounds like he has schizophrenia + no mention of mood disorder + answer is

schizoaffective; why? à this is snapshot of the patient in the psychosis-only phase of schizoaffective

(asked on Psych shelf, where you need to eliminate the other answers, e.g., bipolar, cyclothymia, etc.,

in order to answer correctly).

MEHLMANMEDICAL.COM 8
MEHLMANMEDICAL.COM

- What are the important timeframes for schizophrenia vs schizophreniform vs brief psychotic

disorder? à brief psychotic disorder is < 1 month; schizophreniform is 1-6 months; schizophrenia is

>6 months of psychosis.

- If brain imaging performed in schizophrenic patient, what would be seen? à answer = enlargement

of the third and lateral ventricles.

- Major characteristic in psych vignettes that suggests psychotic disorder? à auditory hallucinations; in

contrast, visual hallucinations are non-specific and seen frequently in drug use (alcohol,

amphetamines, marijuana, etc.).

- Criteria for MDD: SIGECAPS (at least 5 of 9): Sleep disturbance, Interest (loss of), Guilt, Energy (loss

of), Concentration (loss of), Appetite (change in), Psychomotor disturbance (e.g., headaches), Suicidal

ideation. Bear in mind USMLE vignettes will often not give you 5 out of 9 SIGECAPS in all questions;

sometimes the vignette will be as simple as mentioning weight loss/gain in elderly patient who’s

teary-eyed.

- What is bipolar I vs II? à bipolar I is worse than II; both will have cycling episodes of mania and

depression, but bipolar I will often tell you there’s Hx of hospitalization, or losing one’s job, friends, or

relationships (socio-occupational dysfunction), or spending lots of money; bipolar II will tell you

patient keeps stable job and/or family life and has never been hospitalized.

- Tx for bipolar? à lithium or valproic acid to start.

- Adverse effects of lithium and valproic acid? à lithium causes Ebstein anomaly in pregnancy

(atrialization of right ventricle in fetus); also tremor and thyroid dysfunction; valproic acid notoriously

causes neural tube defects.

- What is cyclothymia vs dysthymia? à cyclothymia is >2 years of swinging low + elevated moods, but

never meets the thresholds for bipolar + never has true depressive episode. Dysthymia is >2 years of

low moods not ever meeting thresholds for MDD.

- When is delusional disorder the answer? à one, fixed, non-bizarre delusion + no other mood or

psychotic Sx; presentation will often be patient with mistrust + suspects coworkers and neighbors are

attempting to undermine her work or are stealing from her. If the vignette says anything about

MEHLMANMEDICAL.COM 9
MEHLMANMEDICAL.COM

“aliens,” “the heavens,” “the lord,” etc., the delusion is bizarre and the Dx is psychosis (i.e.,

schizophrenia), not delusional disorder.

- 59F + metastatic cancer + in pain + crying + “wants to die”; Q asks most likely reason for wanting to

die; answer = “inadequate pain control”; “major depression” is wrong answer; must address pain

management in cancer patients.

- 35F + chronic pain in arm since MVA last year + says to physician “I’m realizing I’ll be like this forever.”

Question wants most appropriate response; answer = “have you been feeling like just giving up?” à

must assess suicide risk.

- 42F + 3-month Hx of insomnia + discomfort while lying in bed; next best step in management? à

check serum iron and ferritin levels; student says wtf? à restless leg syndrome is most often caused

by iron deficiency.

- 42F + 3-month Hx of insomnia + discomfort while lying in bed + serum iron and ferritin are normal;

next best step in management? à D2 agonist – i.e., pramipexole or ropinirole, etc.

- Patient with restless leg syndrome is at increased risk for what disease later in life? à answer on

USMLE = Parkinson disease (if D2 agonist can Tx, then lack of dopamine transmission may be etiology

in some patients).

- 58M + loses consciousness while shaving + tilt-table test shows no abnormalities; Dx? à NBME

wants “carotid sinus hypersensitivity” as answer. If tilt-table test (+), answer = vasovagal syncope.

- 45F + fundoscopy shows hard exudates + cotton wool spots + scattered hemorrhages; Dx? à diabetic

retinopathy.

- Medication that can cause tardive dyskinesia that is not an antipsychotic? à answer =

metoclopramide (D2 antagonist); can also prolong QT interval and cause hyperprolactinemia.

- Anti-depressant med causing seizures? à bupropion.

- Other HY factoids about bupropion: also used for smoking cessation; never give in electrolyte

disturbance or eating disorder patients because of seizure risk; does not cause sexual dysfunction

(unlike SSRIs which can cause anorgasmia); bupropion is a reuptake inhibitor preferentially for NE and

dopamine over serotonin.

- 18M + enters emergency department; which blood parameter would indicate recent alcohol

intoxication? à elevation of serum GGT.

MEHLMANMEDICAL.COM 10
MEHLMANMEDICAL.COM

- Frontal lobe injury in car accident; NBME asks which deficit is most likely to ensue; answer =

conceptual planning.

- 56M + 3-day Hx of cutting from 12 beers a day down to 4; develops tremulousness; Tx? à

chlordiazepoxide (delirium tremens); classic vignette is guy has surgery + two days later has

tachycardia, tremulousness, and hallucinations (alcoholic hallucinosis).

- 32M + fear of flying; Dx + Tx? à specific phobia; Tx with benzo.

- 32M + fear of public speaking (glossophobia); Dx + Tx? à social phobia, not specific phobia; Tx =

atenolol (propranolol also OK).

- 25F + fear of flying + must fly soon; Tx? à lorazepam à specific phobia.

- Psych NBME form has both vignettes, practically identical, with the same answer choices:

o 27M + asthma + very anxious about speech he needs to make soon; Tx? à Psych NBME

wants lorazepam, not propranolol (because asthma patient).

o 27M + very anxious about speech he needs to make soon; Tx? à Psych NBME wants

propranolol, not lorazepam.

- 56M + alcoholism + acutely intoxicated + B1 is administered; the latter decreases what most

significantly? à Neuro shelf wants “anterograde amnesia” as the answer; mnemonic for Wernicke =

A COW à Ataxia, Confusion, Ophthalmoplegia, Wernicke.

- Antipsychotic medication started + muscle rigidity + no fever; Dx + Tx? à acute dystonia, not

neuroleptic malignant syndrome (because no fever); Tx with benztropine (muscarinic receptor

antagonist) or diphenhydramine (1st gen H1 blocker, which has strong anti-muscarinic side-effects).

- Antipsychotic medication started + abnormal eye movements + stiff neck; Dx? à acute dystonia

(oculogyric crisis + torticollis).

- Antipsychotic medication started + muscle rigidity + fever; Dx + Tx? à neuroleptic malignant

syndrome; give dantrolene (inhibits ryanodine channel).

- Antipsychotic med + restlessness; Dx + Tx? à akathisia; Tx with propranolol. Psych shelf has Q where

patient says: “I feel as though I am going to jump out of my skin!” à answer = “adverse effect of

prochlorperazine.”

MEHLMANMEDICAL.COM 11
MEHLMANMEDICAL.COM

- Antipsychotic med + bradykinesia; Dx + Tx? à drug-induced Parkinsonism; Tx with amantadine or

propranolol.

- Antipsychotic med + abnormal tongue movements; Dx + Tx? à tardive dyskinesia; stop antipsychotic

+ switch to atypical à answer on Psych shelf for one Q is “discontinue haloperidol and switch to

risperidone.”

- Mechanism for tardive dyskinesia? à answer on Psych NBME = “increased sensitivity of dopamine

receptors.”

- 8-month-old boy + 3rd-centile for weight + slanted palpebral fissures + epicanthal folds + single palmar

crease + thin upper lip with a “fish mouth” appearance + indistinct nasal philtrum; Dx? à answer on

Psych NBME = fetal alcohol syndrome (FAS), not Down syndrome; everyone says wtf about this

question, so what I tell my students is: if Q sounds like Down syndrome but they mention anything

about the philtrum (i.e., long, smooth, indistinct, etc.), the answer is FAS, not Down.

- 6M + IQ of 60 + small for age + born to female age 41 + no other information given; Q asks: most

likely cause of mental retardation? à answer = Down syndrome, not FAS; although FAS is most

common cause of MR overall, two points: 1) if they want FAS, they’ll mention the philtrum as per

above, and 2) most common cause of MR over the age of 40 is Down, not FAS.

- 8M + prominent jaw + protruding ears + IQ of 65; most likely explanation? à “Fragile site on the X

chromosome” à Fragile X (CGG TNR disorder).

- 7M + prominent ears + flattened nasal bridge + long philtrum + low IQ; Dx? à FAS, not Fragile X; the

Q mentions the philtrum.

- 4F + wringing of the hands + putting objects in her mouth + less eye contact; Dx? à answer = Rett

syndrome; only seen in girls; hyperoralism may reflect cognitive regression (babies put everything in

their mouths).

- 5M + talks to imaginary friends; Dx? à age-appropriate behavior.

- 7F + preoccupation with death + fear of dying + constantly asking parents about dying; Dx? à age-

appropriate behavior. Weird, but the NBME will do what it wants.

- 40F + headaches + abdo pain + mild weight loss + fatigue + 5/9 SIGECAPS not met + no mention of low

mood; Dx? à NBME wants somatization disorder, not MDD; this Q is a little challenging, as mere

MEHLMANMEDICAL.COM 12
MEHLMANMEDICAL.COM

weight loss/gain + low mood will often be MDD, particularly in elderly; somatization disorder Dx is

recurring, multiple, current somatic complaints; Tx = CBT.

- 59M + wife says he has bizarre behavior at night where he jumps out of bed and runs back and forth

across the room punching the air + says he does not recall such behavior but remembers bad dreams;

Dx? à answer = REM sleep behavior disorder à incomplete or absent REM muscle atonia; tends to

occur in older adult men; if recurrent, may indicate onset of neurodegenerative disorder like

Parkinson disease.

- 8M + gets out of bed at night and tries to leave the house; when his mom tries to stop him, he

violently tries to pull away from her; once he got out of the house and woke up outside while in the

middle of an episode; he has no recollection of the episodes; Dx? à sleep terror disorder; tends to

occur in pre-adolescent boys.

- 23M + used synthetic heroin + diffuse stiffness + drooling; Dx? à MPTP-induced Parkinsonian

syndrome à affects substantia nigra; MPTP-containing powder is sometimes sold as “synthetic

heroin.”

- 29M + temperature of 107.7F + Hx of MDD + normal HR, RR, BP + WBCs; next best step in Mx? à

Psych shelf wants “obtain rectal temperature under supervision”; fraudulent temperature / factitious

fever is a type of factitious disorder (patient seeks primary gain – i.e., medical attention; willing to

undergo invasive procedures); rectal temperature is most accurate way to measure temperature.

- 42M prisoner + can’t feel his foot + pulses and reflexes normal; Dx + Tx? à malingering (secondary

gain – i.e., money, drugs, time away from prison; not willing to undergo invasive procedures) à “no

treatment indicated.”

- Serum abnormalities seen in bulimic patient? à increased serum amylase + hypokalemic,

hypochloremic metabolic alkalosis.

- 18F + repeated purging + BMI 17; Dx? à anorexia, not bulimia à if BMI low, answer is anorexia.

- 18F + anorexia; what electrolyte disturbance is most likely? à hypokalemia.

- 18F + anorexia + BMI of 14 + reintroduced to foods; what electrolyte must we notably look out for?

à hypophosphatemia (refeeding syndrome).

- 26M + lost in the woods for three weeks + BMI 27 + reintroduced to foods; what electrolyte must we

notably look out for? à hypophosphatemia (refeeding syndrome).

MEHLMANMEDICAL.COM 13
MEHLMANMEDICAL.COM

- Amenorrhea in patient with anorexia; why? à decreased GnRH pulsation à decreased LH + FSH; Q

wants “¯ FHS, ¯ estrogen” as the answer; in contrast, premature ovarian failure, Turner syndrome,

and menopause have “­ FHS, ¯ estrogen” as the answer.

- Anorexia in patient with edema; mechanism for edema? à answer = decreased serum albumin (yes,

this is straight from the Psych NBME, no idea why).

- 45M + has repeated thoughts of harming his son + finds the thoughts absolutely outrageous and

disturbing + says he would never do such a thing; Dx + Tx? à obsessive-compulsive disorder (OCD) à

obsessions are the thoughts; compulsions are the actions; OCD can be just obsessions without

actions; Tx is CBT and/or SSRI.

- 33M + witnessed construction accident at work where many people were injured + found by police in

bowling alley parking lot talking to himself + unable to respond to questions about his identity; Dx? à

answer = dissociative fugue (dissociative disorder); fugue = amnesia for personal identity + travel.

- 16F + has mid-systolic click + episode of hyperventilation and chest pain and sense of impending

doom; Dx à panic attack = acute episode; recurrent episodes = panic disorder. Psych shelf will often

try to make vignette sound cardiac; sometimes “mitral valve prolapse” will be listed as a wrong

answer; student will say, “but there’s a mid-systolic click,” which is true, but the answer is still panic

attack/disorder; MVP is most common murmur in population + almost always benign finding.

- Tx of panic attack vs panic disorder? à for panic attack, answer = breathe into paper bag (sounds

wrong as an answer but is correct if listed); if this is not listed, choose benzo. For panic disorder, Tx =

SSRI.

- Fainting in panic disorder; why? à answer = “decreased cerebral perfusion,” or “cerebral

hypoperfusion”; hyperventilation à decreased CO2 à causes decreased cerebral blood flow.

- 56M + depressed mood + sleep apnea; Dx? à answer = mood disorder due to a general medical

condition.

- 22F + hyperphagia + hypersomnolence + improved mood with pleasurable events; Dx + Tx? à

atypical depression; Tx = SSRI, not MAOi; MAOi (e.g., phenelzine) are considered highly efficacious but

carry dangerous side-effects, so SSRIs remain first-line.

MEHLMANMEDICAL.COM 14
MEHLMANMEDICAL.COM

- Patient eats aged cheese + red wine + slice of pepperoni pizza (fuck now I want pizza) + takes

phenelzine for atypical depression + gets BP of 220/100; Dx + Tx? à answer = tyramine crisis; MAOi

prevent the breakdown of tyramine, a naturally occurring catecholamine in some foods; tyramine

prevents the reuptake of endogenous catecholamines; Tx for tyramine crisis on Psych shelf = alpha-1

blocker (phentolamine).

- 22M + schizophrenia + poor adherence to medications; best med to give to Tx? à haloperidol

decanoate.

- 26F in Australia picking peaches + pinpoint pupils; Q is “how could this have been prevented?” à

answer = “wearing gloves,” not “use of facemask.” Apparently organophosphate poisoning is

acquired through skin, not droplets.

- Tx for organophosphate poisoning? à answer = atropine before pralidoxime. Retired NBME 15 or 16

for Step 1 has both as answers, and atropine is correct.

- 57M + trouble with intercourse with his wife + has nocturnal erections; Dx? à secondary erectile

dysfunction (psychological) à if nocturnal tumescence is intact, he’s physiologically fine.

- 57M + recently divorced + now sleeping with new women + cannot ejaculate during sex + achieves

erection during sex with no problem; Dx? à performance anxiety.

- Tx for premature ejaculation? à answer = SSRI.

- 44M + comes in dressed all in yellow + high energy; Dx? à histrionic personality disorder.

- 44F + sexual toward doctor + high energy; Dx? à histrionic personality disorder

- 16F + slice marks on wrists and thighs + Hx of two prior broken engagements; Dx + Tx? à borderline

personality disorder; parasuicidal behavior common (suicidal gestures, but not true attempts at

suicide); Tx = dialectal behavioral therapy (DBT).

- Defense mechanism in borderline personality disorder? à splitting: “all doctors are bad; all nurses

are good.”

- 26M + works at plastics factory + quiet/loner; Dx? à schizoid personality disorder (ego-syntonic).

- 47F + highly sensitive to rejection + poor self-esteem; Dx? à avoidant personality disorder (ego-

dystonic).

- 26M + “wild thinking” + no auditory hallucinations; Dx? à schizotypal personality disorder.

- 8M + causes frequent disruption at school and at home; Dx? à ADHD; Tx = methylphenidate.

MEHLMANMEDICAL.COM 15
MEHLMANMEDICAL.COM

- 16M + disruptive in class + numerous suspensions from school + caught stealing at the mall; Dx? à

conduct disorder à pattern of law-breaking + must be under age 18; in contrast, a patient with

oppositional defiant disorder does not break the law.

- 27M + cheated on the Bar Exam + fired from job at 23 for stealing + arrested for drunk driving in high

school; Dx? à antisocial personality disorder; key detail is: must break the law; must be older than

age 18 and must have had conduct disorder prior to age 15 à in other words, just because an adult

breaks the law does not mean he or she has ASPD. Another important point is that “anti-social”

means “law-breaking,” not “not social.” Students tend to erroneously define anti-social as avoidant.

- 60M + recovering from a recent MI; he is at high risk of which of the following? à MDD à common

following major adverse events (e.g., trauma, serious Dx); give sertraline for post-MI MDD.

- 60M + recent MI + asks when he can resume sexual intercourse; answer = as soon as he feels ready;

wrong answer is “wait at least two weeks.”

- Is abuse reportable? à child + elderly abuse: yes. Domestic abuse: no. Answer for latter is to provide

supportive care + as much information as possible about what she can do if she feels unsafe.

- 72M + wife passed away 5 months ago + sometimes hears her voice at night; Dx? à normal

bereavement.

- 72M + wife passed away 7 months ago + still grieves; Dx? à pathological grief (normal grief is <6

months).

- 72M + wife passed away 3 months ago + 5kg weight loss + cries + guilt; Dx? à MDD (weight loss/gain

is huge indicator MDD in elderly).

- Patient with MDD; which hormone is increased in serum? à Psych shelf answer = cortisol.

- 22M + 1mm pupils + RR of 8 + stuporous; Dx + Tx? à Dx = Opioid/heroin overdose; Tx = naloxone.

- Different between obsessive-compulsive disorder (OCD) and obsessive-compulsive personality

disorder (OCPD)? à OCD is ego-dystonic (patient doesn’t want/like the thoughts/actions); OCPD is

ego-syntonic (patient is content / not looking to modify behavior).

- Tx for diabetic neuropathic pain? à answer = TCA (i.e., amitriptyline). Second-line is gabapentin

- Tx for herpetic / post-herpetic neuralgia (i.e., from shingles)? à gabapentin

- 82M diabetic + neuropathic pain + already taking carbamazepine + gabapentin to no avail; next best

step? à switch the meds to nortriptyline (a TCA) à student then asks, “Wait, I thought you said TCAs

MEHLMANMEDICAL.COM 16
MEHLMANMEDICAL.COM

are first-line. Why does this Q have the guy on those two meds then?” à two points: 1) we don’t like

giving TCAs to elderly because of their anticholinergic and anti-alpha-1 side-effects, so this vignette

happen to try other agents first, but if you’re asked first-line, always choose TCA; and 2) if we do give

a TCA to an elderly patient, we choose nortriptyline because it carries fewer adverse effects.

- How to differentiate cluster headache from trigeminal neuralgia? à cluster will be a male 20s-40s

with 11/10 lancinating pain behind the eye waking him up at night (he may pace around the room

until it goes away); details such as lacrimation and rhinorrhea are too easy and will likely not show up

on the shelf. In contrast, trigeminal neuralgia will be 11/10 lancinating pain behind the eye (or along

the cheek / jaw if V2 or V3 branches affected; it’s when V1 is affected that this diagnoses are more

readily confused) à TN is brought on by a minor stimulus such as brushing one’s hair or teeth, or a

gust of wind.

- Tx and prophylaxis for cluster headache? à Tx = 100% oxygen; prophylaxis = verapamil.

- Tx and prophylaxis for trigeminal neuralgia? à Tx = goes away on its own because it lasts only

seconds; prophylaxis = carbamazepine.

- Tx and prophylaxis for migraine? à Tx = NSAID, followed by triptan (triptans are NOT prophylaxis;

they are for abortive therapy only after NSAIDs); prophylaxis = propranolol.

- 32M + diffuse headache relieved by acetaminophen + sleep; Dx? à answer = tension-type headache;

Tx = rest + taper caffeine (if taking it).

- Other HY uses for propranolol?

o Migraine prophylaxis (FM form gives patient with HTN + migraine; answer = propranolol)

o Akathisia (with antipsychotic use)

o Thyroid storm (decreases peripheral conversion of T4 to T3)

o Essential tremor (bilateral resting tremor in young adult; autosomal dominant; patient will

self-medicate with EtOH, which decreases tremor); also the answer on Psych shelf for

lithium-induced tremor.

o Hypertension + idiopathic tremor (i.e., tremor need not be essential if patient has HTN à

answer on FM form is “beta-adrenergic blockade” for the HTN Tx).

o Esophageal varices prophylaxis (patients at risk of bleeds)

o Hypertrophic obstructive cardiomyopathy (increases preload à decreases murmur)

MEHLMANMEDICAL.COM 17
MEHLMANMEDICAL.COM

o Social phobia

- 44F + diplopia + dysphagia + eyelid ptosis; all worsen throughout the day; Dx? à myasthenia gravis

(MG).

- 44F + proximal muscle weakness + able to perform upward gaze without a problem for 60 seconds;

Dx? à Lambert-Eaton (LE) syndrome (if cannot perform, answer = MG).

- Vignette where Dx is either MG or LE but it’s not listed; answer? à “neuromuscular junction.”

- 44F + difficulty getting up from chair but is able to after several attempts; Dx? à LE.

- Mechanism of MG vs LE? à MG = antibodies against postsynaptic acetylcholine receptors; LE =

antibodies against presynaptic voltage-gated calcium channels.

- MG can sometimes be paraneoplastic syndromes of which cancer? à MG from thymoma (do chest

imaging to check for thymoma after Dx of MG; if thymoma present + removed, this cures the MG). Up

to 80% of patients with thymoma + MG have anti-MUSK Abs (muscle-specific kinase).

- LE can sometimes be a paraneoplastic syndrome of which cancer? à small cell lung cancer.

- How to Dx MG vs LE? à If both are listed, choose antibodies over Tensilon (edrophonium) test.

- What is edrophonium à short-acting acetylcholinesterase inhibitor.

- How do MG vs LE perform with edrophonium? à MG improves drastically; LE less change.

- Tx of MG? à give an acetylcholinesterase inhibitor (i.e., pyridostigmine).

- 6M + ECG shows miscellaneous arrhythmia + seizure-like episode; Dx? à Adam-Stokes attack à not

true seizure disorder as per EEG; arrythmia leads to hypoxia of brainstem à seizure-like fits ensue.

- 75M + episodes of loss of consciousness (LoC) for 2 years + tonic-clonic-like episodes + becomes pale

and sweaty + Hx of MI; Dx? à answer = “syncope” on the NBME (convulsive syncope).

- Mechanism of narcolepsy? à answer = deficiency of orexin (hypocretin).

- Diagnosis of narcolepsy? à answer = polysomnography.

- What is cataplexy? à loss of muscle tone usually in response to emotional stimulus (e.g., laughter) à

seen in narcolepsy.

- Tx of nacrolepsy = stimulants (i.e., modafinil).

- Polysomnography findings in narcolepsy? à REM latency: ¯; sleep latency: ¯.

- What is simple vs complex seizure? à simple = no LoC; complex = LoC; patient staring off into space

not aware of surroundings = LoC.

MEHLMANMEDICAL.COM 18
MEHLMANMEDICAL.COM

- What is partial vs generalized seizure? à partial = affecting one part of the brain; generalized =

involves the wholes of both cerebral hemispheres.

- What is myoclonic seizure? à jerks in muscle or group of muscles; no LoC.

- Patient with MDD has fluoxetine discontinued + tranylcypromine commenced one week later +

patient develops temp of 105F + HR 110 + RR 25; Dx? à serotonin syndrome; will show up on Psych

shelf as simply “drug-drug interaction”; can occur when combining SSRIs with St John Wort, or

notably when commencing a MAOi too soon after being on another serotonergic medication.

- Tx of serotonin syndrome? à answer = cyproheptadine (serotonin receptor antagonist).

- Difference between serotonin syndrome and carcinoid syndrome? à serotonin syndrome is from

drug-drug interactions and notably causes hyperpyrexia (high fever), tachycardia, and tachypnea;

carcinoid syndrome is a result of carcinoid tumors (usually small bowel, appendiceal, or bronchial)

secreting serotonin and causes flushing, diarrhea, abdominal pain, and bronchoconstriction.

- How to Dx + Tx carcinoid syndrome? à Dx with urinary 5-HIAA (5-hydroxyindole acetic acid); Tx with

octreotide, among other agents.

- Patient witnesses terror attack + has felt emotionally numb for two years since + sometimes

disturbed sleep; next best step? à answer = “provide information about the ranges of reactions to

trauma.”

- 64M + Parkinson disease + 2-month Hx of paranoid behavior; Dx + Tx? à Dx is Parkinson disease

psychosis (PDP); Tx on shelf = quetiapine (can Tx with quetiapine or clonidine).

- 45M + survived plane crash two weeks ago + wakes up screaming in middle of night reliving the

event; Dx? à answer = acute stress disorder, not post-traumatic stress disorder (PTSD); acute stress

disorder is < 1 month; PTSD is > 1 month. Treatment for both is CBT.

- Tx for TCA toxicity? à answer = sodium bicarb à causes dissociation of drug from myocardial sodium

channels.

- 25M + schizophrenic non-responsive to many meds + started on new drug + mouth ulcers; Dx? à

agranulocytosis secondary to clozapine; stop drug immediately; must do granulocyte checks

frequently when first commencing this agent.

- 16M + BMI 31 + snores loudly + morning headaches + 3-5 beats of jerk nystagmus with lateral gaze;

Dx? à answer = “sleep-related hypoventilation”; nystagmus is weird finding, but it’s on the Psych

MEHLMANMEDICAL.COM 19
MEHLMANMEDICAL.COM

NBME; morning headaches are common in sleep-related hypoventilation; can do polysomnography to

diagnose.

YouTube
@mehlmanmedical

Instagram
@mehlman_medical

MEHLMANMEDICAL.COM 20
MEHLMANMEDICAL.COM

MEHLMANMEDICAL
HY PSYCH

All material is copyrighted and the property of mehlmanmedical.

Copyright © mehlmanmedical

MEHLMANMEDICAL.COM 21
MEHLMANMEDICAL
HY FAMILY MEDICINE
MEHLMANMEDICAL.COM

YouTube
@mehlmanmedical

Instagram
@mehlman_medical

MEHLMANMEDICAL.COM 2
MEHLMANMEDICAL.COM

HY Family Medicine

- Patient has pain in the shoulder when raising the arm to paint a fence; Dx? à subacromial bursitis

(caused by repetitive overhead motion)

- Patient has pain in the shoulder when raising the arm above the head; subacromial bursitis isn’t

listed; Dx? à answer = rotator cuff injury

- Patient has pain in the shoulder lying on his or her side in bed; Dx? à rotator cuff injury

- Positive Gerber lift-off test à subscapularis injury à place dorsum of hand against lower back so

palm faces posteriorly; examiner applies pressure into palm + asks patient to move hand à if pain,

subscapularis injury

- Positive “empty can” or “full can” test; Dx? à supraspinatus injury à shoulder is abducted to 90

degrees; then downward pressure is applied; elicits pain when patient attempts to resist

- Resistance to lateral rotation of shoulder elicits pain; Dx? à infraspinatus or teres minor injury

- “Pitcher injury”? à infraspinatus injury; but if a pitcher has positive full or empty can test, use your

head à still supraspinatus injury

- Patient has elbow pain after leaning on elbow for long periods; Dx? à olecranon bursitis

- Patient has pain in lateral forearm with extension of elbow against resistance; Dx + Tx? à answer =

lateral epicondylitis (tennis elbow); Tx = forearm strap

- Patient has pain in medial forearm with flexion of elbow against resistance; Dx + Tx? à answer =

medial epicondylitis (golfer elbow); Tx = forearm strap

- Pain on lateral wrist in breastfeeding woman à deQuervain tenosynovitis à answer = immediate

steroid injection into the wrist. Dx with Finkelstein test (place thumb in palm of hand; then wrap four

digits over thumb; then ulnar deviate; pain at lateral wrist with ulnar deviation is + test)

- Lump on the dorsum of hand alongside a tendon; painless; slightly mobile; Dx? à ganglion cyst; Tx =

needle drainage

- Wrist fracture + posterior displacement of radius; Dx? à Colles fracture (“dinner fork deformity”)

- Wrist fracture + anterior displacement of radius; Dx? à Smith fracture

- Proximal ulnar fracture + anterior displacement of radial head; Dx? à Monteggia fracture

- Radial shaft fracture + displacement of distal radioulnar joint; Dx? à Galeazzi fracture

MEHLMANMEDICAL.COM 3
MEHLMANMEDICAL.COM

- Fracture of forearm in child with “bending” of the bone? à greenstick fracture à bone is soft so part

of bone remains intact

- Fracture in child abuse à spiral fracture (from rotational force/twisting of limb); also posterior rib

fractures

- Patient has hip pain lying on his or her side in bed?; Dx? à trochanteric bursitis

- Patient has lateral hip pain when palpated, when abducting against resistance, or when standing on

that foot; Dx? à greater trochanteric syndrome (gluteus medius or minimus tendonopathy)

- Patient has “locking” or “catching” of the knee; Dx? à meniscal tear

- Patient has pain in the medial knee; Dx? à anserine bursitis

- Patient has pain in the lateral knee; Dx? à iliotibial band syndrome

- 44F + pain worse in knee when going down/up stairs or when sitting for long periods of time +

crepitus + BMI 39; Dx? à answer = patellofemoral syndrome (patellofemoral pain syndrome); next

best step in Mx = “strengthening exercises for quadriceps muscles” + RICE (rest, ice, compression,

elevation).

- 25F + pain in anterior knee on the inferior kneecap + plays basketball + pain initially worse while

playing but past few weeks hurts when done playing as well; Dx? à answer = patellar tendonitis

(Jumper’s knee); next best step in Mx = “strengthening exercises for quadriceps muscles” + RICE.

- 15M + 5’11” + plays soccer + knee pain; Dx? à Osgood-Schlatter à inflammation of patellar ligament

at the tibial tuberosity; occurs in fast-growing, active teenagers; USMLE wants “repeated avulsion

microfractures” as an answer

- Patient has knee pain after spending long periods of time on her knees painting; Dx? à prepatellar

bursitis

- 32M + pain in anterior knee + fever 100.5F + joint effusion not present; Dx? à septic bursitis

- Any patient with red, warm, tender knee; next best step in Mx + Dx? à joint aspiration

(arthrocentesis); septic arthritis till proven otherwise

- 6M + viral infection + now has hip pain +/- fever; Dx? à answer = toxic synovitis (aka transient

synovitis), not septic arthritis à inflammation of the synovial lining of hip joint; Tx is supportive.

MEHLMANMEDICAL.COM 4
MEHLMANMEDICAL.COM

- 6M + suspected JRA + red, hot, painful knee à must do arthrocentesis to rule out septic arthritis. If

the vignette sounds like classic transient synovitis (affects hip, not knee), you do not need to do an

arthrocentesis.

- 5F + 2-day Hx of limp and left hip pain + a week ago had watery stools and a temp of 100F + pain with

weight-bearing and movement + no swelling or erythema; Tx? à answer = ibuprofen (toxic synovitis).

- Biggest risk factor for septic arthritis à abnormal joint architecture

- Pt groups most likely to get SA à prosthetic joints, RA/OA, recent intense exercise/joint trauma;

peds (JRA)

- Pt group most likely to get SA à those with prosthetic joints (can’t be more abnormal than fake joint)

- Pt with OA or RA has red, warm, tender knee à do arthrocentesis (septic arthritis)

- 17F had kickboxing tournament last weekend + knee is red, warm, tender à arthrocentesis (SA)

- Kid + recurrent knee redness, warmth, pain + fever à Juvenile rheumatoid arthritis (JRA; Still disease)

- Kid + recurrent joint pain +/- high ESR +/- rash à JRA

- Kid + recurrent joint pain + anemia à JRA (anemia of chronic disease)

- Kid with suspected JRA has sore knee à must do arthrocentesis to rule out septic arthritis

- Patient has “knock-knees” (i.e., knees touch); Dx? à genu valgum

- Child has bow legs; Dx? à genu varum à can be seen in rickets

- 9F + both legs bowed + parents noticed bowing since she started to walk + recently bowing worse in

right leg + x-ray while standing shows collapse of the medial aspect of the metaphysis of proximal

tibia + rest of vignette describes healthy, thriving patient; Dx? à answer = tibia vara (Blount disease);

wrong answer is rickets; should be noted that bowing is physiologic age < 2 years; tibia vara.

- Patient has lateral thigh pain; Dx? à meralgia paresthetica à due to lateral femoral cutaneous

nerve entrapment

- 18F + anorexia + runs long distances + has foot pain; Dx? à metatarsal stress fracture

- 25M + wakes up with heel pain + gradually improves throughout the morning; Dx? à plantar fasciitis

- 29M + pain in ball of the foot; Dx? à metatarsalgia à overuse injury / from jumping or sports

- 44F + frequently wears high-heel shoes + painful lump on the underside of her foot between her third

and fourth toes; Dx? à answer = Morton neuroma à benign growth of nerve tissue between the 2nd

MEHLMANMEDICAL.COM 5
MEHLMANMEDICAL.COM

and 3rd, or 3rd and 4th, metatarsal heads; usually from chronic irritation from high-heel shoes; Mulder

sign is replication of Sx when the metatarsal heads are compressed together.

- 42M + diabetes + decreased range of motion of the shoulder in all directions; Dx? à adhesive

capsulitis, aka “frozen shoulder” à Tx = range-of-motion exercises / physiotherapy

- 4-month-old + “clicking/clunking” on physical exam à (+) Ortolani and Barlow maneuvers à primary

hip dysplasia (congenital hip dysplasia) à once these are positive, the next best step is ORTHO

REFERRAL if it is listed à referral always sounds wrong, but this is the correct answer if it’s listed; if

it’s not listed, do ultrasound if under 6 months, or x-ray if over 6 months. Tx is with abduction harness

(Pavlik harness; looks frog-leg-like)

- 5-8-year-old boy with painful limp; no other risk factors; x-ray shows contracted capital epiphysis; Dx?

à Legg-Calve-Perthes (idiopathic avascular necrosis); the word “contracted” wins over “capital

epiphysis” à this is a Q on one of the NBME forms where everyone selects slipped capital femoral

epiphysis (SCFE), but it’s Legg-Calve-Perthes; Tx = hip replacement

- 5-8-year-old boy with painful limp + sickle cell disease; Dx? à avascular necrosis (but not Legg-Calve-

Perthes, because LCP is idiopathic)

- 5-8-year-old boy + painful limp + x-ray is negative + bone scan confirms diagnosis; answer? à USMLE

wants you to know that x-ray can be negative initially in avascular necrosis, but bone scan or MRI can

also pick it up

- 11-13-year-old overweight boy with a painful limp; Dx? à SCFE; Tx = surgical pinning

- Tissue mass in palm of hand + bent fingers; Dx? à Dupuytren contracture à seen in alcoholism,

diabetes, Norwegian descent, and epilepsy. Tx is surgical

- 2-year-old boy running + playing with 8-year-old sister + they were holding hands and he fell + now he

holds arm pronated by his side; Dx? à nursemaid’s elbow à radial head subluxation

- Tx for nursemaid’s elbow à hyperpronation OR gentle supination (both are correct answers; only one

will be listed)

- Kid falls on outstretched arm + pain over anatomical snuffbox; Dx? à scaphoid fracture

- Kid falls on outstretched arm + pain over anatomical snuffbox; next best step in Mx? à x-ray

MEHLMANMEDICAL.COM 6
MEHLMANMEDICAL.COM

- Kid falls on outstretched arm + pain over anatomical snuffbox + x-ray is negative; next best step in

Mx? à thumb-spica cast à x-ray is often negative in scaphoid fracture; must cast to prevent

scaphoid avascular necrosis à re-x-ray in 2-3 weeks

- When is figure-of-8-strap the answer? à clavicular fracture

- What part of the clavicle gets fractured easiest? à middle-third

- First Tx for carpal tunnel syndrome in patient who can’t stop offending activity (e.g., office worker) à

wrist splint first; then triamcinolone (steroid) injection into the carpal tunnel; do not select anything

surgical as it’s always wrong on the USMLE; NSAIDs are a wrong answer and not proven to help

- 32F + paresthesias in thenar region of hand +/- hand weakness + sensation intact over dorsum of

hand; next best step in Dx? à NBME answer = “Electrophysiological testing”; call it weird, but it’s

what they want. Examination findings such as Tinel sign, Phalen maneuver, Flick test are insufficient

for diagnosis.

- Tx for cubital tunnel syndrome à elbow splint

- What is cubital tunnel syndrome à ulnar nerve entrapment at elbow à presents similarly to carpal

tunnel syndrome but just in an ulnar distribution and involves the forearm.

- What is Guyon canal syndrome à ulnar nerve entrapment at the wrist à hook of hamate fracture or

chronic handle bar impaction in avid cyclists

- Vegan + they ask for nutrient deficiency + B12 is not listed; what’s the answer? à FM shelf wants

calcium as the answer (normally get from dairy + fish); B9 (folate) is wrong because we get that from

dark green leafy vegetables

- 82F + tea and toast diet for past 6 months; MCV is elevated; is the nutrient deficiency B9 or B12? à

FM shelf answer = B9 (folate) deficiency à stores deplete within six months à “tea and toast” used

to be classic for vitamin C deficiency, but the shelf uses this colloquialism for folate deficiency. If

scurvy is the answer, they will say the patient “appears ill” and has bleeding from gums or around hair

follicles (perifollicular hemorrhages)

- 16M + painful testes + fever + positive cremasteric reflex; Dx? à answer = epididymitis

- Most common organism causing epididymitis? à Chlamydia in sexually active younger males; E. coli

in elderly males. This is also the same for prostatitis. If the vignette tells you no organisms grow on

culture, choose Chlamydia

MEHLMANMEDICAL.COM 7
MEHLMANMEDICAL.COM

- 16M + acutely painful testes + negative cremasteric reflex; Dx? à answer = testicular torsion; do

Doppler ultrasound followed by surgical detorsion

- 6M + painful testis + superior pole shows blue/black discoloration + bowel sounds are decreased +

abdomen is rigid; Dx? à answer = strangulated hernia, not testicular torsion à answer = “operative

management”

- 6M + painful testis + superior pole shows blue dot + cremasteric reflex is intact; Dx? à answer =

torsion of appendix testis à this is on the new peds form but is fair game for FM à torsion of

appendix testis is different from testicular torsion; the latter presents with negative cremasteric

reflex; but torsion of appendix testis causes “blue dot sign”

- 8-month-old boy + unilateral testicular enlargement + light shone to it causes transillumination;

answer = “persistent processus vaginalis” (hydrocele)

- Tx for hydrocele? à observe until the age of one as most spontaneously resolve; this is almost always

the answer; after the age of one, surgical management can be considered

- 3M + hard nodule on testis; Dx? à yolk sac tumor (endodermal sinus tumor) à serum AFP may be

elevated

- 3M + hard nodule on testis + serum AFP + beta-hCG are elevated; Dx? à answer = mixed germ cell

tumor (embryonal cancer), not yolk sac tumor (yolk sac tumor is only high AFP; in mixed germ cell,

beta-hCG is also elevated)

- 22M + heaviness and/or bogginess of testes; Dx? à varicocele à one FM shelf Q literally says “bag of

worms” (normally this is so buzzy that we’d say this wouldn’t show up on an actual form, but it does,

so it must be mentioned here) à Dx with Doppler ultrasound à elective surgical intervention may be

performed to reduce risk of sub-fertility due to increased testicular temperature

- 8-month-old boy has undescended testis; Tx? à answer = observation until at least the age of 1; do

not do orchiopexy before age 1 for USMLE purposes.

- Tx for scabies? à answer = topical permethrin

- Tx for pediculosis (lice)? à answer = topical permethrin

- Tx for acne:

o Topical retinoids first (i.e., topical tretinoin; NOT oral isotretinoin); cause photosensitivity

(rash); also used for photoaging; mechanism is decreasing sebum production; topical

MEHLMANMEDICAL.COM 8
MEHLMANMEDICAL.COM

tretinoin (not oral isotretinoin) is not a teratogen and does not have any effect on pregnancy

or male sperm

o Benzoyl peroxide used second; often coadministered with topic retinoids; mechanism is the

killing of bacteria

o Topical clindamycin

o Oral tetracycline; causes photosensitivity (blistering)

o Oral isotretinoin; must do beta-hCG in women; recommend barrier contraception even if on

OCP; can cause elevations in LFTs; can cause dyslipidemia; main complaint is dry skin +

peeling; takes several weeks to really start working but ultra-effective according to most

patients; can be commenced earlier in patients with severe nodulocystic acne; works by

diffusely shutting of sebum production

- 22F + pain radiating down one arm; Dx? à answer = cervical disc herniation.

- 68M + pain in the neck + MRI shows degenerative changes; Dx? à cervical spondylosis.

- 58F + long-standing Hx of RA + bilateral paresthesias in the upper limbs; Dx? à atlantoaxial

subluxation secondary to RA à answer = cervical spine MRI.

- When to give a statin on the USMLE? à Guidelines from 2019 ACC/AHA:

o Anyone age 20-39 with an LDL > 190 mg/dL

o Anyone age 40-75 with an LDL > 70 mg/dL

o Age >75 à assessment of risk status + clinician-patient discussion are recommended before

commencing or discontinuing a statin

o Diabetics <40 if LDL > 100 mg/dL; hypertension, smoking Hx, CKD, albuminuria, FHx of CVD

in first degree relative (sibling or parent), HDL <40 mg/dL.

o Anyone age <19 with familial hypercholesterolemia

o Balance of risk factors contributes to an ASCVD risk score that determines intensity of statin

administered to the patient (not assessed on USMLE); what the USMLE cares about is you

knowing whether or not to give a statin period.

- First drug to start in T2DM à metformin

- Side-effect of metformin à lactic acidosis

MEHLMANMEDICAL.COM 9
MEHLMANMEDICAL.COM

- 40F + T2DM + on glyburide + HbA1c = 9.6%; what do we do? à answer = switch to metformin (beta-

islet cells are burned out if sulfonylurea isn’t working).

- 40F + T2DM + on glyburide + metformin + HbA1c = 9.6^; what do we do? à answer = switch to

intermediate-acting insulin (needs insulin if oral hypoglycemics aren’t working).

- 40F + T2DM + on glyburide + HbA1c 5.9% + BP 138/82 + creatinine of 1.0; what do we do? à answer

= start ACEi (e.g., enalapril) à start an ACEi if BP > 130/80 (either #) or evidence of protein in the

urine.

- 40F + T2DM + on glyburide + HbA1c 6.7% + LDL is 112 mg/dL; what do we do? à answer = commence

statin to bring LDL under 70 mg/dL (old guidelines said <100).

- How to Dx TB?

o PPD skin test is performed first diagnostically. If history of BCG vaccine, do interferon-gamma

release assay (IGRA) instead. Do not do IGRA in addition to PPD.

o If PPD is negative, repeat after one week. If negative again, no further studies indicated.

Repeats performed within 1 week may cause a false (+) secondary to a "booster reaction."

o If IGRA is negative, no further studies indicated.

o If PPD or IGRA is positive, do CXR. Do not repeat positive PPD tests.

o If CXR is negative, treat for latent TB / give TB prophylaxis. On the USMLE, "treatment for

latent TB" and "administer TB prophylaxis" mean the same thing.

o If CXR is positive, treat for active TB.

- How to manage a positive PPD test?

o Measure induration only. Erythema does not count.

- 5+ mm

o Recent contact with people with active TB

o HIV + status

o Organ transplant recipients

o Chronic prednisone use (>15mg/day for >1 month); anti-TNF-α agent use

o Findings consistent with TB on CXR

- 10+ mm

o Immigrant status (Western countries not included)

MEHLMANMEDICAL.COM 10
MEHLMANMEDICAL.COM

o IV drug users

o Healthcare workers; prison workers; homeless shelter personnel

o TB laboratory personnel

o Children under 4 years of age

- 15+ mm

o Everyone

- How to Tx latent TB?

o 9 months INH + pyridoxine (vitamin B6) - The USMLE Steps 1 and 2CK assess this as the

answer.

o 4 months rifampin

o 3 months INH + rifapentine + pyridoxine

o Vitamin B6 must be given with INH to prevent vitamin B6 deficiency.

- Tx of active TB

o Rifampin, INH, pyrazinamide, ethambutol (RIPE) for 2 months, followed by RI alone for 4

more months (6 months total)

o And of course add pyridoxine (annoying that it sounds similar to pyrazinamide)

- Travel + self-limiting watery or brown/green diarrhea à Traveler diarrhea = ETEC HL or HS toxin

- Bloody diarrhea + poultry consumption à Campylobacter jejuni or Salmonella spp.

- Abx (clindamycin, beta-lactam, cephalosporin) + diarrhea à C. difficile

- Dx of C. diff à stool AB toxin test, not stool culture

- Fever of 104 + abdo distension in C diff à toxic megacolon à laparotomy

- Tx of C. diff à vancomycin, not metronidazole (updated guidelines as of Feb 2018)

- Bloody diarrhea + travel à Entamoeba histolytica

- Tx of E. histolytica à metronidazole + iodoquinol; can give paromomycin

- Close quarters or military barracks or cruise ship + watery diarrhea à Norwalk virus

- Child <5 years + watery diarrhea à rotavirus

- Few organisms causing bloody diarrhea à Shigella

- Bloody diarrhea + appendicitis-like pain (pseudoappendicitis) in a child à Yersinia enterocolitica

MEHLMANMEDICAL.COM 11
MEHLMANMEDICAL.COM

- Bloody diarrhea + reactive arthritis in an adult à Y. enterocolitica, Campylobacter, Shigella,

Salmonella

- Diarrhea + Guillain-Barre syndrome à Campylobacter

- GBSyndrome CSF? à albuminocytologic dissociation (high protein + normal cells)

- GBSyndrome Dx? à electromyography + nerve conduction studies (on NBME)

- Cardiac ischemia + need to evaluate à ECG stress test first-line

- Cardiac ischemia + abnormal baseline ECG (e.g., BBB) à Echo stress test (need normal ECG to do ECG

stress test)

- Cardiac ischemia + patient can’t exercise à dobutamine + ECG/echo

- ECG shows diffuse ST-segment elevations à pericarditis

- Pericarditis Tx à NSAID, or steroid, or colchicine

- Central chest pain worse when supine; better when leaning forward à pericarditis

- Lateral chest pain after viral infection + increased CK à pleurodynia (intercostal muscle spasm)

- ST-segment depressions in the anterior ECG leads à posterior MI

- Electrical alternans on ECG à pericardial tamponade / pericardial effusion

- Pulsus paradoxus (drop in systolic BP >10 mm with inspiration) à cardiac tamponade or severe

asthma

- Beck triad à hypotension + muffled heart sounds + JVD à pericardial tamponade

- Tx of tamponade à pericardiocentesis or pericardial window

- Tamponade à do echo before pericardiocentesis if both listed (even though sounds wrong, on 2CK

NBME)

- Community-acquired pneumonia (CPP) + bilateral CXR infiltrates à Mycoplasma

- CPP + lobar pattern (right-lower lobe consolidation + dullness to percussion) à Strep pneumo

- CPP + lobar pattern, but they say “interstitial” in the vignette description à Mycoplasma, not S.

pneumo

- Empiric Tx for CPP à azithromycin

MEHLMANMEDICAL.COM 12
MEHLMANMEDICAL.COM

- Tx for CPP is pt on Abx past three months à fluoroquinolone over azithro

- Pneumonia in CF patient <10 years à S. aureus exceeds Pseudomonas

- Pneumonia in CF patient >10 years à Pseudomonas exceeds S. aureus.

- Pneumonia after influenza infection à USMLE wants S. aureus

- Pneumonia + rabbits à F. tularensis

- Pneumonia + cattle à Coxiella (Q fever)

- Pneumonia + birds à Chlamydia psittaci

- Tx for aspirin toxicity à bicarb (increased excretion through urinary alkalinization)

- Tx for diabetic neuropathic pain? à answer = TCA (i.e., amitriptyline). Second-line is gabapentin

- Tx for herpetic / post-herpetic neuralgia (i.e., from shingles)? à gabapentin

- 82M diabetic + neuropathic pain + already taking carbamazepine + gabapentin to no avail; next best

step? à switch the meds to nortriptyline (a TCA) à student then asks, “Wait, I thought you said TCAs

are first-line. Why does this Q have the guy on those two meds then?” à two points: 1) we don’t like

giving TCAs to elderly because of their anticholinergic and anti-alpha-1 side-effects, so this vignette

happen to try other agents first, but if you’re asked first-line, always choose TCA; and 2) if we do give

a TCA to an elderly patient, we choose nortriptyline because it carries fewer adverse effects.

- How to differentiate cluster headache from trigeminal neuralgia? à cluster will be a male 20s-40s

with 11/10 lancinating pain behind the eye waking him up at night (he may pace around the room

until it goes away); details such as lacrimation and rhinorrhea are too easy and will likely not show up

on the shelf. In contrast, trigeminal neuralgia will be 11/10 lancinating pain behind the eye (or along

the cheek / jaw if V2 or V3 branches affected; it’s when V1 is affected that this diagnoses are more

readily confused) à TN is brought on by a minor stimulus such as brushing one’s hair or teeth, or a

gust of wind.

- Tx and prophylaxis for cluster headache? à Tx = 100% oxygen; prophylaxis = verapamil.

- Tx and prophylaxis for trigeminal neuralgia? à Tx = goes away on its own because it lasts only

seconds; prophylaxis = carbamazepine.

- Tx and prophylaxis for migraine? à Tx = NSAID, followed by triptan (triptans are NOT prophylaxis;

they are for abortive therapy only after NSAIDs); prophylaxis = propranolol.

MEHLMANMEDICAL.COM 13
MEHLMANMEDICAL.COM

- 32M + diffuse headache relieved by acetaminophen + sleep; Dx? à answer = tension-type headache;

Tx = rest + taper caffeine (if taking it).

- Other HY uses for propranolol?

o Migraine prophylaxis (FM form gives patient with HTN + migraine; answer = propranolol)

o Akathisia (with antipsychotic use)

o Thyroid storm (decreases peripheral conversion of T4 to T3)

o Essential tremor (bilateral resting tremor in young adult; autosomal dominant; patient will

self-medicate with EtOH, which decreases tremor).

o Hypertension + idiopathic tremor (i.e., tremor need not be essential if patient has HTN à

answer on FM form is “beta-adrenergic blockade” for the HTN Tx).

o Esophageal varices prophylaxis (patients at risk of bleeds)

o Hypertrophic obstructive cardiomyopathy (increases preload à decreases murmur)

o Social phobia

- Important point about Graves? à proptosis/exophthalmos + pretibial myxedema are specific; in

other words, there are numerous causes of hyperthyroidism (e.g., toxic multinodular goiter, toxic

adenoma, etc.), but only Graves will cause the eye findings

- Tx for ophthalmopathy in Graves à steroids

- Why do the eye findings occur in Graves? à glycosaminoglycan deposition in/around extra-ocular

muscles

- High BP + fever + increased CK à thyroid storm (Tx with PTU + propranolol)

- What is the role of potassium iodide (KI) in hyperthyroid Tx? à shuts off gland production (Wolff-

Chaikoff effect) à answer in person exposed to nuclear fallout or radioiodine vapors in laboratory

- Hashimoto parameters à high TSH, low T3, low T4, decreased iodine uptake

- Mechanism for Hashimoto à antibodies against thyroperoxidase + thyroglobulin; anti-microsomal

- Histo of Hashimoto à lymphocytic infiltrate (easy to remember bc the non-eponymous name for

Hashimoto is chronic lymphocytic thyroiditis)

- Tx for Hashimoto = levothyroxine (synthetic T4)

- 50M + low mood + BMI 26 à Hashimoto

- Proximal muscle weakness in Hashimoto + increased serum CK à hypothyroid myopathy

MEHLMANMEDICAL.COM 14
MEHLMANMEDICAL.COM

- 45M + decreased ability to get up from chair unassisted + HR of 60 à Hashimoto

- 45M + high cholesterol + high hepatic AST + HR of 55 à Hashimoto (hypothyroidism can cause

bradycardia, high cholesterol, and high AST [the latter is weird, correct])

- Thyroid cancer in Hashimoto à thyroid lymphoma (autoimmune diseases à increased risk of

lymphoma)

- 22M + viral infection + very tender thyroid à subacute granulomatous thyroiditis (de Quervain)

- De Quervain parameters à triphasic à causes hyper-, then hypo-, then rebounds to euthyroid state

à most important point is that iodine uptake is DECREASED in de Quervain + drug-induced

thyroiditis, even if patient is in a hyper-phase.

- 22M + very tender thyroid + HR of 88 + tremulousness + heat intolerance à low TSH, high T3, high

T4, decreased iodine uptake (in contrast to Graves, which is painless and uptake is high)

- Tx for subacute thyroiditis à aspirin first, not steroids; steroids may be used later

- Drugs causing thyroiditis à lithium + amiodarone

- Surreptitious thyrotoxicosis àself- injection of thyroxine à low TSH, high T3, high T4, small thyroid

gland with decreased uptake

- Injection of triiodothyroinine (T3) à TSH will go down, T3 goes up (clearly), T4 does not go up

because T3 isn’t converted to T4; only T4 is converted to T3

- Injection of thyroxine à TSH will go down (negative feedback), T4 goes up (clearly), T3 goes up (due

to peripheral conversion), reverse T3 also goes up

- What is reverse T3? à an inactive form of T3; T4 is converted peripherally into T3 (active) and reverse

T3 (inactive)

- Anything else I need to know about reverse T3? à it’s increased in euthyroid sick syndrome à times

of stress/surgery/illness à cortisol increases à cortisol decreases conversion of T4 to active T3, so

more T4 is converted to reverse T3 à parameters in euthyroid sick syndrome: normal TSH, normal

T4, low T3, high reverse T3

- What is subclinical hypothyroidism à high TSH but normal T3 + T4 (don’t confuse with ESS)

- Subclinical hypothyroidism Tx à don’t treat unless TSH >10 (normal is 0.5-5), Hashimoto Abs are

present, or patient is pregnant à they ask this info on 2CK + Step 3

- Want to check thyroid function, what’s the first thing to order à TSH

MEHLMANMEDICAL.COM 15
MEHLMANMEDICAL.COM

- Want to check thyroid function, what’s the first thing to order à free T4

- What is free T4 à most thyroid hormone is protein-bound and inactive; free T4 tells you definitively

whether the patient has thyroid derangement or not

- Pregnancy and thyroid à estrogen causes increased thyroid-binding globulin production by the liver

à mops of T4 à less free T4 à less negative feedback at hypothalamus + anterior pituitary à TSH

goes up transiently to compensate à more T4 made à free T4 rebounds to normal but now total T4

is high à parameters you need to know for pregnancy: normal TSH + high total T4 + normal free T4 +

normal free T3 + high thyroid-binding globulin

- Hyperthyroidism in pregnancy à LH, FSH, TSH, hCG all share same alpha-subunit; their beta-subunits

differ; some women have increases sensitivity of TSH receptor to alpha-subunit, so high hCG in early

pregnancy can stimulate thyroid gland and cause transient hyperthyroidism

- Hashimoto in pregnancy à increase pregnant woman’s dose of levothyroxine by 50%

- Graves in pregnancy à avoid methimazole in first trimester (teratogenic; causes aplasia cutis

congenita) à give PTU in first-trimester à in second + third trimesters, switch from PTU to

methimazole (methimazole no longer teratogenic later in pregnancy + PTU is heavily hepatotoxic)

- Pt being treated for Graves + mouth ulcers à agranulocytosis (neutropenia) caused by methimazole

or PTU.

- Young child with normal free T4 and low total T4 à thyroid-binding globulin deficiency (opposite of

pregnancy)

- Young child + large belly + large tongue + hypotonia à cretinism (congenital hypothyroidism)

- Most common cause of cretinism à iodine deficiency

- Most common cause of hyperthyroidism in elderly à toxic multinodular goiter

- Evaluation of thyroid cancer, first step? à palpation of thyroid gland (on FM 2CK form as answer)

- If thyroid nodule present, then check TSH; if TSH normal or high à answer = FNA, not USS; if TSH low,

do radioiodine uptake scan; thyroid cancer is cold, not hot, which is why no FNA with low TSH

- Tx for shingles (not the pain; the actual shingles)? à answer = oral acyclovir (one of the forms writes

“oral acyclovir” rather than oral valacyclovir, but both are fine) à however if patient is on

chemotherapy / is immunocompromised, give IV acyclovir

MEHLMANMEDICAL.COM 16
MEHLMANMEDICAL.COM

- Patient older than 50 + temporal headache + high ESR; Dx + Tx? à temporal arteritis à give

immediate IV methylprednisolone (steroids) then do biopsy temporal artery (never biopsy first)

- Patient with temporal arteritis + jaw pain; why the jaw pain? à can cause temporomandibular joint

claudication

- Patient with temporal arteritis has muscle pain; Dx? à polymyalgia rheumatica (PMR)

- Tx of PMR? à oral steroids

- How to differentiate PMR from polymyositis? à both can have high ESR; PMR tends to have “muscle

pain + stiffness” with preserved muscle strength; polymyositis will present with proximal muscle

weakness +/- pain and stiffness (the idea is: PMR more likely to have pain + stiffness, but NO muscle

weakness; polymyositis less likely to have pain + stiffness, but WILL have proximal muscle weakness)

- Difference between polymyositis and dermatomyositis? à dermatomyositis has heliotrope rash

(around the eyes; don’t confuse with malar rash) +/- shawl rash +/- Gottron papules (violaceous

papules on the knuckles) +/- mechanics’ hands (rough/scaly palms)

- Dx of polymyositis/dermatomyositis? à anti-Jo1 and anti-Mi2 first; if positive, do confirmatory

muscle biopsy

- Mechanism of hyponatremia in heart failure? à answer = “baroreceptor-mediated ADH release” à

heart doesn’t pump as well à decreased systolic impulse à stretch of carotid sinus baroreceptors à

increased ADH release. This is the same autoregulation mechanism that will cause decreased

parasympathetic activity via vagus nerve.

- Nocturnal enuresis; when is it pathologic? à after age 5

- 6M + nocturnal enuresis; next best step? USMLE / NBME / shelf wants the following order:

o Behavioral answer first; e.g., spend more time with child; decrease overt stressors as much

as possible

o If the above not an answer, do star chart (positive reinforcement therapy; i.e., don’t wet the

bed and get a star; get 5 stars for extra dessert; 100 and we go to Disneyland)

o If star chart not listed or already attempted, next answer is enuresis alarm

o Medications like imipramine and desmopressin are always wrong; water deprivation after

5pm is also always wrong.

MEHLMANMEDICAL.COM 17
MEHLMANMEDICAL.COM

o Students mess these Qs up because they’ll see enuresis alarm as correct on one form, but on

a different form it’s star chart, so know the above order

- 72M + intermittent claudication + absent distal pulses + Hx of coronary artery bypass grafting + high

BP that’s been gradually increasing past two years; Dx? à renal artery stenosis

- 32F + high BP + high aldosterone/renin à fibromuscular dysplasia (tunica media proliferation in renal

arteries) à this is not renal artery stenosis à if you say “renal artery stenosis,” that means

atherosclerosis

- Increased creatinine following medication administered to someone with renal artery stenosis; what

was the drug? à ACEi or ARB

- Tx for RAS + FMD à initially medical therapy with cautious use of ACEi or ARB; definitive is renal

angioplasty + stenting; FMD is not curable

- How to differentiate viral from bacterial upper respiratory tract infection (URTI)? à CENTOR criteria

o If 0 or 1 point, the URTI is unlikely to be bacterial (i.e., it’s likely to be viral). If 2-4 points,

chance is much greater that URTI is bacterial.

o 1) Absence of cough (i.e., no cough = 1 point; if patient has cough = 0 points).

o 2) Fever.

o 3) Tonsillar exudates.

o 4) Lymphadenopathy (cervical, submandibular, etc.).

- There is a version of the criteria that includes age, but on the USMLE it can cause you to get questions

wrong. So just use the simplified above four points.

o If 0-1 point, answer = “supportive care”; or “no treatment necessary”; or “warm saline

gargle” (same as supportive care)

o If 2-4 points, next best step = “rapid Strep test.” If rapid Strep test is negative, answer =

throat culture, NOT sputum culture

o While waiting on the throat culture results, we send the patient home with amoxicillin or

penicillin for presumptive Strep pharyngitis

o If child is, e.g., 12 years old, and develops a rash with the beta-lactam, answer = beta-lactam

allergy

MEHLMANMEDICAL.COM 18
MEHLMANMEDICAL.COM

o If the vignette is of a 16-17 year-old who has been going on dates recently (there will be no

confusion; the USMLE will make it clear), the answer = EBV mononucleosis; therefore do a

heterophile antibody test (Monospot test)

o EBV is the odd virus out that usually presents with all four (+) CENTOR criteria

o This is why it’s frequently misdiagnosed as Strep pharyngitis. It is HY to know that beta-

lactams given to patients with EBV may cause rash via a hypersensitivity response to the Abx

in the setting of antibody production to the virus. EBV, in a patient who does not receive

Abx, can cause a mild maculopapular rash. But the rash with beta-lactam + EBV causes a

more intense pruritic response generally 7-10 days following Abx administration on the

extensor surfaces + pressure points

- Tx for TCA toxicity à sodium bicarb à dissociates drug from myocardial sodium channels

- Ataxia, confusion, ophthalmoplegia à Wernicke encephalopathy (B1 deficiency)

- Retrograde amnesia + confabulation in alcoholic à Korsakoff psychosis

- Wernicke-Korsakoff syndrome à mammillary bodies

- Hx of many pregnancies + downward movement of vesicourethral junction à stress incontinence

- Tx of stress incontinence à pelvic floor exercises (Kegel); if insufficient à mid-urethral sling

- Hyperactive detrusor or detrusor instability à urge incontinence

- Need to run to bathroom when sticking key in a door à urge incontinence

- Incontinence in multiple sclerosis patient or perimenopausal à urge incontinence

- Tx of urge incontinence à oxybutynin (muscarinic cholinergic antagonist) or mirabegron (beta-3

agonist)

- Incontinence + high post-void volume (usually 3-400 in question; normal is <50 mL) à overflow

incontinence

- Incontinence in diabetes à overflow incontinence due to neurogenic bladder

- Tx for overflow incontinence in diabetes à bethanechol (muscarinic cholinergic agonist)

- Incontinence in BPH à overflow incontinence due to outlet obstruction à eventual neurogenic

bladder

MEHLMANMEDICAL.COM 19
MEHLMANMEDICAL.COM

- Tx for overflow incontinence in BPH à insert catheter first; then give alpha-1 blocker of 5-alpha-

reductase inhibitor; then TURP if necessary

- Exquisitely tender prostate on digital rectal exam à prostatitis

- Prostatitis Tx à ciprofloxacin (fluoroquinolone à DNA gyrase / topoisomerase II inhibitor)

- Costovertebral angle tenderness + fever à pyelonephritis

- Costovertebral angle tenderness + granular casts à pyelonephritis (correct, super-weird; NOT acute

tubular necrosis; this is on 2CK NBME)

- Tx for pyelonephritis à ciprofloxacin, OR ampicillin + gentamicin

- Saddle anesthesia + urinary retention à cauda equina syndrome

- Perianal anesthesia + urinary retention or incontinence à conus medullaris syndrome

- Gradual-onset dementia + no sensory or motor dysfunction à Alzheimer

- Mini-mental state exam score low + patient tries to do well à Alzheimer

- Mini-mental state exam score low + patient is apathetic / takes long to perform tasks / does poorly on

reverse serial 7s à depression (pseudodementia)

- Patient complains about memory loss à normal aging, not Alzheimer

- First-line Tx for Alzheimer à donepezil (cholinesterase inhibitor à increases cholinergic

transmission); can also give galantamine or rivastigmine

- NMDA receptor (glutamate receptor) antagonist used in Alzheimer Tx à memantine

- Step-wise dementia/decline and/or sensory/motor disturbance à vascular dementia

- Hx of hypertension + dementia + sensory/motor disturbance à vascular dementia

- Visual hallucinations + Parkinsonism + dementia à Lewy body dementia

- Apathy + disinhibition + personality change + dementia à frontotemporal dementia (Pick disease)

- Urinary incontinence + ataxia + CNS dysfunction (wet, wobbly, wacky) à Normal-pressure

hydrocephalus

- Wet, wobbly, wacky + Parkinsonism à still NPH

- Parkinsonism in young patient à Wilson disease till proven otherwise

MEHLMANMEDICAL.COM 20
MEHLMANMEDICAL.COM

- Parkinsonism in older patient à Parkinson disease

- Parkinsonism + axial dystonia à progressive supranuclear palsy

- Tx of UTI à TMP/SMX or nitrofurantoin

- Tx of cystitis à nitrofurantoin (need not be pregnant)

- Waiter tip position in kid à upper brachial plexus injury à C5/6 à Erb-Duchenne palsy

- Claw hand à lower brachial plexus à C8/T1 à Klumpke palsy

- Pronated arm + wrist-drop à radial nerve injury

- Midshaft fracture of humerus à Radial nerve injury

- Supracondylar fracture of humerus à median nerve injury

- Surgical neck of humerus fracture à axillary nerve injury

- Medial epicondylar injury à ulnar nerve injury

- Weakened biceps + loss of sensation of lateral forearm à musculocutaneous nerve injury

- Paresthesias + pain following burn or casting à compartment syndrome

- Compartment syndrome Dx à measure compartment pressure

- Compartment syndrome Tx à fasciotomy of uncast

- Guy lifts heavy box à severe lower back pain + muscle spasm + no radiculopathy à lumbosacral

strain only

- Lumbosacral strain diagnosis à DO NOT x-ray

- Lumbosacral strain Tx à NSAIDs + exercise as tolerated; bedrest is the WRONG answer

- Guy lifts heavy box à severe lower back pain + radiculopathy à herniated disc; yes, x-ray.

- Point tenderness over a vertebra in older woman à osteoporosis (compression fracture)

- Point tenderness over a vertebra in younger patient on steroids à osteoporosis (compression

fracture)

- Point tenderness over a vertebra in patient with autoimmune disease à recognize patient is on

steroids à osteoporosis (compression fracture)

- Point tenderness over a vertebra in IV drug user à epidural abscess

MEHLMANMEDICAL.COM 21
MEHLMANMEDICAL.COM

- “Step-off” of one vertebra relative to another à spondylolisthesis

- Back pain worse in the morning and gets better throughout day in male 20s-40s à ankylosing

spondylitis

- Bamboo spine à ankylosing spondylitis

- Dx of AS à x-ray of sacroiliac joints

- Back pain worse when standing or walking for long periods of time à lumbar spinal stenosis

- Radiculopathy down an arm à cervical disc herniation

- Bilateral paresthesias in the arms in rheumatoid arthritis patient à atlantoaxial subluxation

- Bilateral paresthesias in the arms in rheumatoid arthritis patient à MRI of spine to Dx atlantoaxial

subluxation

- Prior to surgery in rheumatoid arthritis patient à cervical CT or flexion/extension x-rays of cervical

spine to check for atlantoaxial subluxation

- Back pain in elderly patient with hypercalcemia à multiple myeloma or metastases

- Back in pain in patient with history of other type of cancer à metastases

- Suspected spinal mets à MRI

- Metastases to long bones in prostate cancer à osteoblastic (Dx with bone scan); spine do MRI

- High hemoglobin +/- pruritis after shower +/- plethora +/- splenomegaly à polycythemia vera

- High hemoglobin + low EPO à polycythemia vera

- Pruritis after shower à basophilia

- High hemoglobin + lung disease / low pO2 à secondary polycythemia (high EPO)

- Polycythemia + hypercalcemia + smoker + red urine à RCC (paraneoplastic EPO + PTH-rp)

- Blurry vision or Raynaud or pain in fingers or headache à hyperviscosity syndrome

- Hereditary spherocytosis à AD, ankyrin or spectrin or band protein deficiency; Tx = splenectomy

- Treatment for ITP à steroids, then IVIG, then splenectomy

- Tx for hereditary hemochromatosis à serial phlebotomy

- Tx for secondary hemochromatosis (transfusional siderosis) à chelation therapy (deferoxamine)

MEHLMANMEDICAL.COM 22
MEHLMANMEDICAL.COM

- Viral infection + tinnitus + vertigo +/- neurosensory hearing loss à labyrinthitis

- Viral infection + vertigo à vestibular neuritis

- Tx for multiple sclerosis flares à IV steroids (IV methylprednisolone)

- Given to MS patients between flares à interferon-beta

- Tx for spasticity in MS à baclofen (GABA-B receptor agonist)

- Incontinence in MS à urge (hyperactive detrusor, as mentioned earlier)

- New-onset murmur + fever à endocarditis till proven otherwise

- Empiric Tx for endocarditis à vancomycin or ampicillin/sulbactam, PLUS gentamicin

- Beta-lactam (e.g., nafcillin) or cephalosporin + rash + renal issue à interstitial nephritis

- Interstitial nephritis à WBCs in the urine (eosinophils)

- Fixed splitting of S2 à atrial septal defect

- Holosystolic murmur at left sternal border PLUS either parasternal heave or palpable thrill à VSD

- Holosystolic murmur at left sternal border PLUS diastolic rumble à also VSD

- To-and-fro murmur à PDA (on 2CK NBME)

- Pan-systolic-pan-diastolic murmur à PDA

- Continuous, machinery-like murmur à PDA

- Congenital rubella syndrome à PDA

- Heart problem in neonate of mom with SLE à congenital third-degree heartblock

- Heart problem in William syndrome à supravalvular aortic stenosis

- Bicuspid aortic valve à aortic stenosis

- Mid-systolic (crescendo-decrescendo) murmur + gets worse with Valsalva à HOCM

- Mid-systolic (crescendo-decrescendo) murmur + no change or softens with Valsalva à aortic stenosis

- Myxomatous degeneration of mitral valve à mitral valve prolapse

- Marfan or Ehlers-Danlos syndrome à MVP or aortic regurg

- Rheumatic heart disease acutely (onset of Group A Strep infection) à mitral regurg

- Rheumatic heart disease later on (years after infection) à mitral stenosis

MEHLMANMEDICAL.COM 23
MEHLMANMEDICAL.COM

- Mid-systolic click à MVP

- Late-peaking systolic murmur with ejection click à another way they describe aortic stenosis

- Bounding pulses + massively wide pulse pressure à aortic regurg

- Brisk upstroke + precipitous downstroke of pulse à aortic regurg

- Syncope + angina + dyspnea (SAD) à aortic stenosis

- Dyspnea in second trimester of pregnancy à mitral stenosis

- Dyspnea late in pregnancy à peripartum cardiomyopathy

- Screening at age 50 à mammogram (every two years) + colonoscopy (every ten years)

- Colon cancer in first-degree relative (sibling or parent) à start at age 40 or ten years before diagnosis

in relative, whichever is earlier, and do every 5 years.

- Breast imaging (if performed) à ultrasound only under age 30; over age 30 do mammogram +/-

ultrasound

- Dysphagia to solids and liquids at the same time to start à says neurogenic cause à achalasia

- Dysphagia to solids that progresses to solids and liquids à esophageal cancer

- Halitosis +/- gurgling sound when swallowing +/- regurgitation of undigested food à Zenker

- Zenker + achalasia initial Dx modality à barium swallow

- After barium swallow is done and shows bird’s beak appearance à monometry to confirm Dx of

achalasia

- Pt with Hx of GERD + dysphagia à straight to endoscopy to rule out cancer

- Diabetic pt with new-onset GERD à diabetic gastroparesis

- Diabetic pt with new-onset GERD à give metoclopramide, not PPI

- Diabetic gastroparesis before giving med à endoscopy first to rule out physical obstruction

- Endoscopy negative for diabetic gastroparesis à do gastric-emptying scintigraphy

- Bulimia nervosa or anorexia à never give buproprion (seizure risk)

- Electrolyte abnormality in anorexia à hypokalemia

- Most common cause of death in anorexia à arrhythmia from hypokalemia

MEHLMANMEDICAL.COM 24
MEHLMANMEDICAL.COM

- Refeeding syndrome à worry about hypophosphatemia

- Tx of anorexia + depression à mirtazapine (alpha-2 antagonist); stimulates appetite

- Amenorrhea in anorexia à low FSH + low estrogen (hypogonadotropic)

- Premature ovarian failure + Turner syndrome + menopause à high FSH (low inhibin) + low estrogen

- Trichotillomania (eating one’s hair) + GI symptoms à gastric bezoar (hair ball)

- Tx for trichotillomania? à SSRI.

- Kid with high lymphocytes à ALL or pertussis (weird bc bacterial, but lymphocytes often >30k)

- Dry cough in winter à cough-variant asthma (1/3 of asthmatics only have cough)

- Young African American woman + dry cough + normal CXR à asthma (activation of mast cells), not

sarcoidosis

- Young African American woman + dry cough + nodularity on CXR à sarcoidosis (noncaseating

granulomas)

- Electrolyte disturbance in sarcoid à hypercalcemia à LOW PTH + high calcium

- Hypercalcemia in sarcoid, why? à epithelioid (activated) macrophages produce 1-alpha hydroxylase,

thereby activating vitamin D3

- Increased calcium in sarcoid à means decreased calcium in feces (bc D3 increased small bowel

absorption)

- Tx for sarcoid à steroids

- Outpatient Tx of asthma à SABA, then low-dose ICS, then maximize dose of ICS, then LABA, then use

any number of drugs (e.g., mast cell stabilizers, anti-leukotriene, etc.), then oral steroids last resort

- Kid with asthma on SABA inhaler + not effective + next best step? à ICS (fluticasone)

- Kid with asthma on SABA inhaler + most effective way to decrease recurrence? à oral steroids (not

next best step, but certainly most effective)

- 40s male + hematuria + hemoptysis à Goodpasture syndrome

- Antibodies in Goodpasture à Anti-GBM (anti-collagen IV)

- Dx of Goodpasture à antibodies first, but confirmatory is renal biopsy showing linear

immunofluorescence

MEHLMANMEDICAL.COM 25
MEHLMANMEDICAL.COM

- Hematuria + hemoptysis + “head-itis” (mastoiditis, sinusitis, otitis, nasal septal perforation) à

Wegener

- New name for Wegener à granulomatosis with polyangiitis

- Dx of Wegener à c-ANCA (anti-PR3; anti-proteinase 3)

- Asthma + eosinophilia à Churg-Strauss

- New name for CS à eosinophilic granulomatosis with polyangiitis

- Dx of CS à p-ANCA (anti-MPO; anti-myeloperoxidase)

- Hematuria in isolation + p-ANCA in serum à microscopic polyangiitis (MP)

- Teenage girl with Hx of cutaneous candida infections since childhood à chronic mucocutaneous

candidiasis

- MCC à answer = T cell dysfunction = impaired cell-mediated immunity on the USMLE

- Bacterial + fungal + protozoal + viral infections since birth à SCID

- Bacterial infections since age 6 months à Bruton

- Bacterial infections only since birth à Bruton (rare as hell to say from birth, but it’s on new 2CK

NBME)

- SCID XR variant à common gamma-chain mutation (IL-2 receptor deficiency)

- SCID AR variant à adenosine deaminase deficiency

- Bruton mechanism à tyrosine kinase mutation

- Hyper IgM syndrome à deficiency of CD40 ligand on T cell (can’t activate CD40 on B cell to induce

isotype class switching)

- Greasy, scaly scalp + itchy + papules + adult à seborrheic dermatitis

- Tx for SD à azole or selenium shampoo

- Tx for tinea capitis à oral griseofulvin for patient only

- How to decrease risk of tinea capitis à avoidance of sharing of hats

- Tx of onychomycosis (nailbed fungus) à oral terbinafine

- Tx of tinea pedis à topical terbinafine or topical azole

MEHLMANMEDICAL.COM 26
MEHLMANMEDICAL.COM

- Tx of tinea corporis (ring worm) à topical azole (clotrimazole or miconazole)

- Tx of cutaneous candida à oral azole

- Tx of oropharyngeal candida à nystatin mouthwash

- Tx of esophageal candidiasis à oral azole, not nystatin mouthwash

- Tx of vaginal candidiasis à topical nystatin before trying oral azole

- Odynophagia (painful swallowing) in immunocompromised pt à esophageal candidiasis till proven

otherwise

- CNS fungal infection or fungemia (rigors/chills) à amphotericin B

- Cryptococcal meningitis à amphotericin B + flucytosine, then do fluconazole taper

- Simple fungal pneumonia à fluconazole

- Sporothrix schenckii (rose thorn + finger papule) à itraconazole

- Hypopigmentation on upper back / trunk à tinea versicolor (Malassezia furfur)

- Tx of tinea versicolor à topical selenium

- Most common cause of impetigo à S. aureus now exceeds S. pyogenes

- Tx of impetigo à topical mupirocin

- Beefy red, well-demarcated skin plaque à erysipelas

- Most common cause of erysipelas à Group A Strep (S. pyogenes) >>> S. aureus

- More diffuse pink skin lesion + tenderness + fever à cellulitis

- Most common cause of cellulitis à S. aureus exceeds S. pyogenes

- Tx of erysipelas + cellulitis à oral dicloxacillin or oral cephalexin

- Give killed IM influenza vaccine when? à Every year in fall/winter only; start from 6 months of age

- Killed IM Influenza vaccine safe in pregnancy? à Yes, give anytime to pregnant women

- Live-attenuated intranasal influenza vaccine guidelines? à Only give age 2-49 to non-pregnant, non-

immunocompromised persons

- Vaccines at age 2, 4, 6 months: HepB, Polio Salk, Pneumo PCV13, DPT, HiB, rotavirus (also give HepB

at birth)

MEHLMANMEDICAL.COM 27
MEHLMANMEDICAL.COM

- HPV vaccine when à age 9-45

- Mom’s HepB status unknownà give neonate HepB vaccine; only give immunoglobulin if mom comes

back +

- MMR à first dose at 12-15 months; second dose age 4-6 years

- Varicella à one dose between 12-18 months

- Age 65 or older à give Pneumo PCV13 followed by PPSV23 6-12 months later

- Asplenia or sickle cell à PCV13 + PPSV23 + HiB + Meningococcal

- Young adult + non-smoker + has emphysema + relative died of hepatic cirrhosis à alpha-1 anti-

trypsin deficiency

- CREST syndrome lung pathology? à can cause pulmonary fibrosis à pulmonary hypertension

- Restrictive lung disease à normal or increased FEV1/FVC

- Obstructive lung disease à decreased FEV1/FVC

- Why is FEV1/FVC normal or high in restrictive? à radial traction on outside of airways is sticky (keeps

airways from closing)

- Apex to base lung changes when sitting/standing à both ventilation + perfusion increase apex to

base

- Most common cause of otitis media à Strep pneumo

- Tx of otitis media à oral amoxicillin only

- Tx of recurrent OM à amoxicillin/clavulanate

- When to do tympanostomy tube à three or more OM in 6 months, or 4 or more in a year

- Most common cause of otitis externa à Pseudomonas

- Tx of otitis externa à topical ciprofloxacin + hydrocortisone drops

- Prevention of OE in someone with constant water exposure (e.g., crew team) à alcohol-acetic acid

drops

- Tx of earwax buildup à carbamide peroxide drops

- Low hematocrit + low MCV + low transferrin + low TIBC + transferrin saturation normal or low à

anemia of chronic disease

MEHLMANMEDICAL.COM 28
MEHLMANMEDICAL.COM

- Low hematocrit + low MCV + high transferrin + high TIBC + transferrin saturation super-low à iron

deficiency anemia

- Low hematocrit + low MCV + increased red cell distribution width (RDW) à iron deficiency anemia

- Low hematocrit + low MCV + decreased RDW à thalassemia

- Low hematocrit + low MCV + low iron + low ferritin à iron deficiency

- Low hematocrit + low MCV + low iron + normal or high ferritin à anemia of chronic disease.

- Low hematocrit + low MCV + low iron + normal ferritin in pregnant woman on iron supplements à

thalassemia

- Microcytic anemia that doesn’t improve with iron supplementation à thalassemia

- Dx of thalassemia à hemoglobin electrophoresis

- Low hematocrit + normal MCV + low iron + normal or high ferritin à anemia of chronic disease

- Tx of anemia of chronic disease if renal failure is cause à answer = EPO

- Tx of anemia of chronic disease if renal failure not cause (IBD, RA, SLE, etc.) à CANNOT give EPO; Tx

underlying condition.

- High BP + smoker + TIA or stroke or retinal artery occlusion. How to best decrease stroke risk à

Answer = lisinopril, not smoking cessation

- Normotensive old pt + TIA or stroke or retinal artery occlusion à atrial fibrillation

- Hypertensive pt + stroke à do carotid duplex ultrasound

- Normotensive pt + stroke à do ECG; if ECG normal à Holter monitor

- High BMI female + irregular menstrual cycles à anovulation

- Anovulation + hirsutism à PCOS

- Anovulation. Cause USMLE wants? à insulin resistance à causes abnormal GnRH pulsation

- Why hirsutism in anovulation à abnormal GnRH pulsation causes high LH/FSH ratio

- Why high LH/FSH ratio important in anovulation/PCOS à ovulation stimulated when follicle not

ready à no ovulation (anovulation) à follicle retained as cyst

- What’s LH do? à Stimulates theca interna cells (females) and Leydig cells (males) to make androgens

MEHLMANMEDICAL.COM 29
MEHLMANMEDICAL.COM

- What’s FSH do? à Stimulates granulosa cells (females) and Sertoli cells (males) to make aromatase;

also primes follicles

- Tx for PCOS à if high BMI, weight loss first always on USMLE

- Tx for PCOS if they ask for meds and/or weight loss already tried à OCPs (if not wanting pregnancy);

clomiphene (if wanting pregnancy)

- PCOS increases risk of what à endometrial cancer (unopposed estrogen)

- Tx of prostate cancer à flutamide + leuprolide together (if they force a sequence, choose F then L).

- Tx of acute gout à indomethacin (NSAID) first on USMLE; then steroids, then colchicine

- Tx of acute gout if indomethacin + steroids not listed à colchicine

- Tx of acute gout in pt with renal insufficiency or Hx of renal transplant à steroids

- Tx of chronic gout (decrease recurrence) à allopurinol or febuxostat (xanthine oxidase inhibitors)

- Never give which drug to pt with Hx of uric acid stones or over-producer à probenecid (uricosuric)

- What are rasburicase / pegloticase à urate oxidase analogues à cleave uric acid directly

- Young kid + self-mutilation + red-orange crystals in diaper à Lesch-Nyhan syndrome (X-linked)

- Lesch-Nyhan enzyme à HGPRT deficiency

- Crystal type in pseudogout à calcium pyrophosphate deposition disease

- Two main causes of pseudogout à primary hyperparathyroidism + hemochromatosis

- Two ways pseudogout presents à monoarthritis of large joint (i.e., knee) or osteoarthritis-like

presentation in someone with primary hyperparathyroidism or hemochromatosis

- 32M + dark skin on forearms + increased fasting glucose; Dx? à hemochromatosis (bronze diabetes)

- Same male + painful hands + x-ray shows DIP involvement. Joint pain Dx? à pseudogout

- Tx of pseudogout à same as gout acutely; Tx underlying condition for chronic

- Biggest risk factor for osteoarthritis à obesity

- Tx of osteoarthritis à weight loss; if normal BMI à acetaminophen before NSAIDs

- Patient with OA taking naproxen (NSAID) + peripheral edema à increased renal retention of sodium

MEHLMANMEDICAL.COM 30
MEHLMANMEDICAL.COM

- Patient taking NSAID + edema; why? à NSAID decreases renal blood flow à PCT increases Na

reabsorption to compensate for perceived low volume status à water follow sodium

- Tx of rheumatoid arthritis à Two-armed: symptom-relief + disease-modifying anti-rheumatic drugs

(DMARDs)

- Symptom-relief for RA à NSAID first, then steroids (these do symptoms only; do not slow disease

progression)

- DMARDs for early RA à always methotrexate first; if insufficient, add another DMARD (sulfasalazine

or leflunomide); if insufficient add anti-TNF-alpha agent

- Methotrexate MOA à dihydrofolate reductase inhibitor

- Methotrexate side-effects à pulmonary fibrosis + hepatotoxicity + mouth ulcers (neutropenia)

- Sulfasalazine MOA à metabolized into sulfapyridine + mesalamine in the gut by bacteria

- Mesalamine is 5-ASA absorbed as the Tx for RA; only NSAID considered to be DMARD

- Leflunomide MOA à dihydroorotate dehydrogenase inhibitor (pyrimidine synthesis)

- Most specific Abs in RA à anti-CCP (cyclic citrullinated peptide), not RF (rheumatoid factor)

- X-ray of hands in RA vs OA à Only OA has DIPs involved; RA is PIPs + MCPs

- Symmetry in RA vs OA à RA is symmetrical; OA is not

- Most common presentation finding in SLE à arthritis (>90%)

- Woman 20s-40s + arthritis + thrombocytopenia à SLE

- Woman 20s-40s + arthritis + mouth ulcer + circular skin lesions à SLE

- Malar rash + low RBCs + low WBCs + low platelets; mechanism for low cell lines? à increased

peripheral destruction (antibodies against hematologic cells lines seen in SLE; isolated

thrombocytopenia most common)

- Tx of SLE flare à steroids

- Tx of lupus nephritis à mycophenolate mofetil

- Tx of discoid lupus à hydroxychloroquine

- Most specific Abs for SLE à anti-Smith (RNP), not anti-dsDNA

- Which Abs go up in acute SLE flares à anti-dsDNA (and C3 goes down)

MEHLMANMEDICAL.COM 31
MEHLMANMEDICAL.COM

- Drug-induced lupus Abs à anti-histone

- Drugs that cause DIL à Mom is HIPP à Minocycline, Hydralazine, INH, Procainamide, Penicillamine

- Viral infection + all three cell-lines are down à viral-induced aplastic anemia

- Viral-induced aplastic anemia; next best step in Dx? à bone marrow aspiration

- Viral-induced aplastic anemia; mechanism? à defective bone marrow production (contrast with SLE)

- Viral infection + low platelets à ITP (immune thrombocytopenic purpura)

- Woman 30s-40s with random bruising at different stages of healing à (also ITP; first rule out abuse)

- Mechanism of ITP à Abs against GpIIb/IIIa on platelets

- Dx of ITP à answer = low platelet count; don’t choose increased bleeding time

- ITP Tx à steroids first, then IVIG, then splenectomy

- ITP episode à next best step in management à steroids

- ITP episode à most effective way to decrease recurrence à splenectomy (not first-line, but most

effective)

- Family Hx of heme condition treated with splenectomy à hereditary spherocytosis (autosomal

dominant)

- Bleeding time meaning? à platelet problem

- PT and aPTT meaning? à clotting factor problem

- Heme findings in ITP à increased BT, normal PT, normal aPTT

- Heme findings in hemophilia à increased aPTT; bleeding time and PT are normal

- Cause of hemophilia à X-linked recessive; hemophilia A (factor VIII def); hemophilia B (factor IX def)

- Tx of hemophilia A à desmopressin for hemophilia A (increases VIII release); then give factor VIII

- Tx of hemophilia B à give factor IX

- Classic hemophilia presentation à hemarthrosis in school-age boy; bleeding after circumcision in

neonate

- Inheritance pattern of vWD à AD

- Heme findings in vWD à bleeding time always high; PT always normal; aPTT elevated half the time

MEHLMANMEDICAL.COM 32
MEHLMANMEDICAL.COM

- What is main function of vWF? à bridges platelet GpIb to underlying collagen (adhesion, not

aggregation)

- What is secondary function of vWF à stabilizes factor VIII in plasma (that’s why aPTT only half time

increased)

- vWD presentation à always one platelet problem + one clotting factor problem

- Platelet problem? à epistaxis, bruising, petechiae à generally mild and cutaneous

- Clotting factor problem à menorrhagia, excessive bleeding with tooth extraction, hemarthrosis (but

hemarthrosis very very rare in vWD; it is seen in hemophilia)

- vWD treatment à desmopressin à increases release of vWF

- Vitamin K deficiency heme parameters? à Increased PT + aPTT; bleeding time normal

- Cause of vitamin K deficiency in adults à chronic Abx knock out colonic flora

- Cause of sickle cell à glutamic acid to valine mutation on beta-chain

- HY drugs that cause agranulocytosis à clozapine, ganciclovir, propylthiouracil, methimazole,

methotrexate, ticlopidine

- How will agranulocytosis (neutropenia) present on USMLE? à mouth ulcers + fever

- Tx for febrile neutropenia / neutropenic fever à immediate broad-spectrum IV Abx

- Broad-spectrum Abx example? à Pipericillin/tazobactam; cefepime + vancomycin

- What is ganciclovir used for? à Tx of CMV (DNA polymerase inhibitor)

- PTU and methimazole are used for what? à Tx of Graves

- Familial thyroid cancer à medullary (even if they mention nothing else related to MEN 2A/2B); apple-

green birefringence on Congo red stain due to amyloid deposition; serum calcitonin high

- Calcitonin mechanism of action à inhibits osteoclast activity (not the opposite of PTH; in other

words, doesn’t put calcium back into bone; it merely caps the Ca that can resorb out of the bone)

- Most common thyroid cancer à papillary; extends lymphatogenously; has papillary structure and

psammoma bodies on LM; don’t worry about buzzywordy things like Orphan Annie nuclei

MEHLMANMEDICAL.COM 33
MEHLMANMEDICAL.COM

- Follicular carcinoma à literally just thyroid follicles on biopsy; will be a cold nodule, like any other

type of thyroid cancer (for instance, if you see follicles but it’s a hot nodule w/ increased uptake,

that’s a toxic adenoma, rather than follicular thyroid cancer); spreads hematogenously

- Hashimoto + thyroid cancer + no other histo description à thyroid lymphoma à autoimmune

diseases increase the risk of non-Hodgkin lymphoma

- MEN1 à pituitary, pancreas, parathyroid (MEN1 gene; chromosome 11)

- MEN2A à parathyroid, medullary thyroid carcinoma, pheochromocytoma

- MEN2B à medullary thyroid carcinoma, pheochromocytoma, mucosal neuromas, Marfanoid body

habitus (“oid” means looks like but ain’t)

- Riedel thyroiditis à fibrosis of thyroid à can extend into adjacent structures, e.g., the esophagus,

and resemble anaplastic carcinoma

- 17F + painless lateral neck mass + mediastinal mass; Dx? à Hodgkin lymphoma

- 42M + painless lateral neck mass + hepatomegaly; Dx? à Hodgkin lymphoma

- Electrolyte abnormality in Cushing syndrome à hypokalemia (chronically high glucocorticoid can

cause potassium wasting distally in the kidney similar to aldosterone)

- Pt has tachy + diaphoresis + diarrhea after drug à serotonin syndrome (tramadol; MOA too soon

after stopping SSRI)

- Pt has tachy + diaphoresis + diarrhea + tricuspid valve lesion à carcinoid syndrome

- Cause of carcinoid syndrome à usually small bowel or appendiceal tumor that has metastasized to

liver (if not metastasized, liver can process serotonin derivatives it receives); can also be due to

bronchogenic carcinoid; tumors are S-100 positive and of neural crest origin

- Dx of serotonin + carcinoid syndromes à urinary 5-hydroxyindole acetic acid (5-HIAA)

- Tx of serotonin syndrome à remove offending agents + administer cyproheptadine (serotonin

receptor antagonist)

- Tx of carcinoid syndrome à Tx underlying condition

- Asthma (outpatient) à albuterol (short-acting beta-2 agonist; SABA) inhaler for immediate Mx à if

insufficient, start low-dose ICS (inhaled corticosteroid) preventer à if insufficient, maximize dose of

MEHLMANMEDICAL.COM 34
MEHLMANMEDICAL.COM

ICS preventer à if insufficient, add salmeterol inhaler (long-acting beta-2 agonist; LABA); in other

words:

- 1) SABA; then

- 2) low-dose ICS; then

- 3) maximize dose ICS; then

- 4) LABA.

- That initial order is universal. Then you need to know last resort is oral corticosteroids, however they

are most effective. In other words:

- 12M has ongoing wheezing episodes + is on albuterol inhaler; next best step? à add low-dose ICS

- 12M has ongoing wheezing episodes + is on albuterol inhaler; what’s most likely to decrease

recurrence à oral corticosteroids (student says “wtf? I thought you said ICS was what we do next and

that oral steroids are last resort” Yeah, you’re right, but they’re still most effective at decreasing

recurrence. This isn’t something I’m romanticizing; this distinction is assessed on the FM NBME forms.

- After the LABA and before the oral steroids, any number of agents can be given in any order – i.e.,

nedocromil or cromolyn sodium, zileuton, montelukast, zafirlukast.

- MOA of nedocromil and cromolyn sodium à mast cell stabilizers

- MOA of zileuton à lipoxygenase inhibitor (enzyme that makes leukotrienes from arachidonic acid)

- MOA of the -lukasts à leukotriene LTC, D, and E4 inhibitors. LTB4 receptor agonism is unrelated and

induces neutrophilic chemotaxis (LTB4, IL-8, kallikrein, platelet-activating factor, C5a, bacterial

proteins)

- 16M goes snowboarding all day + takes pain reliever for sore muscles afterward + next day develops

wheezing out on the slopes again; what’s going on? à took aspirin + this is Samter triad (now

cumbersomely known as aspirin-exacerbated respiratory disease [AERD]) à triad of aspirin-induced

asthma + aspirin hypersensitivity + nasal polyps). Just to be clear, other NSAIDs can precipitate

Samter triad, but the literature + USMLE will make it explicitly about aspirin.

- 16M takes aspirin + gets wheezing; what are we likely to see on physical exam? à answer on USMLE

= nasal polyps.

MEHLMANMEDICAL.COM 35
MEHLMANMEDICAL.COM

- “Wait I don’t understand. Why would aspirin cause asthma?” à arachidonic acid can be shunted

down either the cyclooxygenase or lipoxygenase pathways; if you knock out COX irreversibly by giving

aspirin (or reversibly with another NSAID), more arachidonic acid will be shunted down the

lipoxygenase pathway à more leukotrienes à more bronchoconstriction

- Kid has Hx of AERD; physician considers agent to decrease his recurrence of Sx à zileuton, or -lukasts

(both are correct; and only one will be listed).

- Kid has Hx of AERD; what agent is most likely to decrease his recurrence of Sx à oral steroids (sounds

wrong, but once again, you need to know oral steroids are most effective for preventing asthma,

period; this is exceedingly HY, especially on family medicine forms). We simply don’t want to give

them because of their nasty side-effects (Cushing syndrome).

- Any weird asthma Txs? à omalizumab à monoclonal antibody against IgE à used for intractable,

severe asthma unresponsive to oral steroids + in patients who have eosinophilia + high IgE levels (I

asked a pulmonologist about this drug years ago when I was in MS3 and he said he was managing

1000 patients with asthma and just three were on omalizumab).

- Acute asthma Mx (emergencies) à most important piece of info straight-up is: USMLE wants you to

know that inhaled corticosteroids (ICS) have no role in acute asthma management. First thing we do

is give oxygen (any USMLE Q that shows depressed O2 sats, answer is always O2) + nebulized

albuterol (face mask with mist); IV steroids are then administered. The Mx algorithm is more

complicated, but that is what you need for the USMLE.

- Acid-base disturbance in asthma? à respiratory alkalosis à low O2, low CO2, high pH, normal bicarb

- “Wait, why the low CO2? Aren’t you not able to breathe?” à low CO2 is due to high respiratory rate;

even if your bronchioles are constricted + filled with secretions, CO2 can diffuse really quickly; in

contrast, O2 diffuses slowly and requires healthy airways; that’s why with a high RR, O2 and CO2 are

both low (O2 can’t get in, but CO2 can still get out); 19 times out of 20 on the USMLE, if your

respiratory rate is high, CO2 is low.

- “19 times out of 20? Then what’s the exception.” à I’ve seen COPD questions where the patient will

have a RR of 28 but a super-high CO2, and the answer is chronic respiratory acidosis + acute

respiratory acidosis (acute on chronic) à in the event of emphysema, where you literally have

reduced surface area for gas exchange, even if your RR is high, CO2 has no way of diffusing out.

MEHLMANMEDICAL.COM 36
MEHLMANMEDICAL.COM

- “Wait, why is bicarb normal in acute asthma attack? Shouldn’t it go low to compensate if CO2 is low?”

à not enough time for bicarb to change; takes a minimum of 12-24 hours for renal elimination to

have an effect on serum levels; this is why in altitude sickness, where CO2 is low (due to high RR bc of

lower atmospheric O2), azetazolamide (carbonic anhydrase inhibitor) can be given to increase bicarb

loss in the PCT of the kidney to essentially force a metabolic acidosis to compensate.

- 12M + acute asthma episode + given O2 + nebulized albuterol + IV steroids + his acid-base

disturbance is as we talked about above à after 30 minutes, new values are: low O2, normal CO2,

normal pH, normal bicarb; why? à he’s getting tired à low O2 means he should still be

hyperventilating, so for CO2 and pH to have normalized means his RR is decreasing à answer on

USMLE = intubate. When O2 and CO2 are both initially down, that’s called a type I respiratory failure;

then eventually it will invert, where this patient will have a respiratory acidosis with low O2, high CO2,

low pH, normal bicarb (type II respiratory failure when O2 and CO2 are the opposite).

- 12M + red urine 1-3 days after upper respiratory tract infection (URTI) à IgA nephropathy, not PSGN;

can also get IgA nephropathy from GI infections

- 12M + red urine 1-2 weeks after URTI or skin infection à PSGN à can get it from Group A Strep skin

infections

- 6F + red urine + abdo pain + arthralgias + violaceous lesions on buttocks + thighs; Dx? à Henoch-

Schonlein purpura; red urine = IgA nephropathy à HSP is tetrad of 1) IgA nephropathy, 2) palpable

purpura, 3) arthralgias, 4) abdo pain

- 13F has never had a period + has suprapubic mass + nausea + vomiting; next best step in Mx? à

answer = do beta-hCG à she’s pregnant; this is HY. Correct, girls can get pregnant without ever

having had a period à must rule out

- 14F has massive unilateral breast mass + mom is freaking out bc her sister died of breast cancer à

answer = follow-up in six months à virginal breast hypertrophy is normal during puberty

- 15M has unilateral mass behind his nipple +/- tenderness of it à answer = reassurance à physiologic

gynecomastia of puberty (higher androgens are aromatized to estrogens)

- Girl is Tanner stage 3; which of the following is true? à answer = menarche is imminent à USMLE

asks this Q straight up and it’s exceedingly HY and frequent

MEHLMANMEDICAL.COM 37
MEHLMANMEDICAL.COM

- 17F + really pad period pain + physical exam is normal à answer = primary dysmenorrhea =

prostaglandin hypersecretion (PGF2alpha) à give NSAIDs

- Tx for hypertension? à if patient has pre-diabetes, diabetes, or any cardiovascular/cerebrovascular

disease of any kind à answer = ACEi or ARB first. These agents decrease morbidity and mortality in

these patient groups. If patient has none of the above (i.e., your typical fat American middle-age male

who’s a little overweight but otherwise just has essential hypertension), the answer = HCTZ or

dihydropyridine CCB. You might think that’s really weird (i.e., “why not just give an ACEi or ARB

anyway to anyone if they’re good for morbidity/mortality?”), but the basis is: you’re not going to live

to 120 just because you start taking a statin when it’s not indicated; well the same is true here:

there’s no evidence of further improvement or morbidity/mortality in pts without the above risk

factors if started on ACEi or ARB). This knowledge about how to Tx HTN is HY for FM shelves in

particular

- 32F + pedal + forearm edema after commencing anti-hypertensive agent; Dx? à answer = fluid

retention / edema caused by dihydropyridine CCB (e.g., nifedipine) à really HY side-effect of d-CCBs!

- Side-effects of thiazides à hyperGLUC à hyperglycemia, -lipidemia, -uricemia, calcemia

- Whom should you never give thiazides to? à prediabetics or diabetics à will push people into type II

DM and make current DMs worse. One of the worst/frequent pharmacologic mistreatments. Also

don’t give to pts with Hx of gout (bc of hyperuricemia risk)

- Diabetic pt on HCTZ for HTN à take them the fuck off the thiazide and put them on an ACEi or ARB.

- Important use of thiazide apart from HTN management in select patients à decreased risk of nephro-

/ ureterolithiasis (stones) because they cause hypocalciuria (and hence hypercalcemia)

- 72F + 6-month Hx of small painless papule from a chickenpox scar on her chin; Dx? à answer =

Marjolin ulcer (squamous cell carcinoma) à SCC growing from previous scar or burn site

MEHLMANMEDICAL.COM 38
MEHLMANMEDICAL.COM

YouTube
@mehlmanmedical

Instagram
@mehlman_medical

MEHLMANMEDICAL.COM 39
MEHLMANMEDICAL.COM

MEHLMANMEDICAL
HY FAMILY MEDICINE

All material is copyrighted and the property of mehlmanmedical.

Copyright © mehlmanmedical

MEHLMANMEDICAL.COM 40
MEHLMANMEDICAL
HY DERMATOLOGY
MEHLMANMEDICAL.COM

YouTube
@mehlmanmedical

Instagram
@mehlman_medical

MEHLMANMEDICAL.COM 2
MEHLMANMEDICAL.COM

HY Dermatology

This document is not designed to be a long-winded, 350-page dermatology textbook that caters to superfluous

details that will never be tested. The focus here is to be as concise as possible with HY factoids in order to

increase your score on the USMLE.

- Student Q showed 10M with scalp lesion similar to below, then the Q asked for the treatment:

o Answer = oral griseofulvin for patient only (also on FM NBME form); wrong answer = “oral

griseofulvin for patient and classmates”; Dx is tinea capitis; note alopecia and circular/scaly

appearance of lesion; cause is dermatophytes (i.e., Microsporum; Trichophyton).

o Q on different NBME asks how to prevent; answer = “avoidance of sharing of hats”; “use of

medicated shampoo” is wrong answer.

- 24M + itchy patches and greasy scales along the hairline; Q asks for the diagnosis:

o Answer = seborrheic dermatitis (dandruff); treatment = topical selenium or ketoconazole

shampoo; does not cause circular area of alopecia as with tinea capitis; more common in

adults (tinea capitis more common in children); cause is inflammatory response to over-

colonization with Malassezia yeast.

o High prevalence in HIV patients; sudden onset in MSM à answer = do HIV test.

MEHLMANMEDICAL.COM 3
MEHLMANMEDICAL.COM

- 46F + has three dogs at home; the following lesion from her forearm is shown; Q asks treatment:

o Answer = topical miconazole or clotrimazole; diagnosis = tinea corporis (ringworm); Q will

often mention dogs or use of yoga mats at the gym.

- 35F + BMI of 55 + type II diabetes + red, moist 8x12-cm ellipse under right breast; Q asks biggest risk

factor for her condition? à answer = insulin resistance; obesity is wrong answer; diagnosis is

cutaneous Candida; treat with oral fluconazole.

- 27F + white, cheese-like discharge per vaginum; Q asks what oral treatment she needs; answer =

fluconazole; some students say, “Wait, I thought we use topical nystatin” à either oral fluconazole or

topical nystatin can be used; there’s an NBME Q for Step 1 where they specify “oral” treatment;

fluconazole is correct and nystatin isn’t listed.

- 32M + fever 101 F + red, itchy, scaly area between his 1st and 2nd toes + the redness/scaling extends

up dorsum of foot and onto ankle; Q asks most likely causal organism for his fever; answer = Staph

aureus; Trichophyton is wrong answer; diagnosis is Staph cellulitis superinfection over tinea pedis;

Staph can cause the fever; unlikely for tinea pedis in isolation to cause fever.

o Tx for tinea pedis on USMLE is topical terbinafine or -azole (i.e., clotrimazole/miconazole).

- 40F + diabetic foot ulcer; sterile probe to base of lesion is likely to show what? à correct answer on

new NBME exam = “polymicrobial”; wrong answers are Staph aureus and Pseudomonas. This is an

extremely important Q from NBME because people have long debated Staph vs Pseudomonas for

diabetic foot ulcers.

- 6M + puncture wound on foot 3 weeks ago + continues to have warmth, redness, and pain on

palpation; Q asks most likely organism (polymicrobial not listed) à answer = Pseudomonas on new

CMS Peds form; Staph aureus is wrong. Apparently implication is osteomyelitis has occurred due to

MEHLMANMEDICAL.COM 4
MEHLMANMEDICAL.COM

non-healing / continued pain. If osteomyelitis occurs from a plantar puncture wound, choose

Pseudomonas.

- 24M + excoriated rash on groin and inner ankle + rash on ankle was successfully treated with topical

clotrimazole a few weeks ago, but rash has reappeared + is on groin; what’s the mechanism? à

answer = “autoinfection” (i.e., he is scratching/re-infecting himself); wrong answers are related to

immunodeficiency. Call it stupid, but it’s on 2CK NBME form. You need to know “excoriations” mean

the patient has been scratching.

- 60M + farmer + thickened yellow nailbed of left big toe; Dx + Tx? à answer = onychomycosis (fungus

of nails); Tx = oral terbinafine (if not listed, choose griseofulvin).

- 17F + Candida skin infections since childhood + 2-year Hx of type I diabetes mellitus + 1-yr Hx of

autoimmune thyroiditis; Q asks mechanism for patient’s condition à answer = “deficiency of cell-

mediated immunity”; diagnosis is chronic mucocutaneous candidiasis; USMLE wants you to know this

is a T cell problem; autoimmune conditions go together (i.e., increased risk of one à increased risk of

another); this also applies to immunodeficiencies in relation to autoimmunity (e.g., IgA deficiency also

associated with atopy and vitiligo); although Candida infection risk increased with diabetes, the

infections in this patient far precede the diabetes Dx.

- 31M + gardener + has presentation shown below; Q asks the mechanism for this patient’s condition:

o Answer = lymphangitis; diagnosis is lymphocutaneous sporotrichosis (Sporothrix schenckii);

wrong answers are phlebitis, arteritis; treatment is oral itraconazole. Students early in their

prep should know that Sporothrix is classically papule on the finger caused by rose thorns;

this presentation is usually too easy for real USMLE though.

MEHLMANMEDICAL.COM 5
MEHLMANMEDICAL.COM

o Exam can also give Sporothrix as guy who goes hiking and scratches his face with a stick à

gets papule on the cheek that ruptures into oral cavity + causes draining sinus tract; answer =

sporotrichosis; wrong answer = craniofacial Actinomyces.

- 19M + plays soccer and goes to beach; has condition in image shown below; what is treatment?

o Answer = topical selenium; diagnosis is tinea versicolor (Malassezia furfur); fungus causes

degradation of fatty acids within the skin leading to hypopigmentation; this image is all over

the NBMEs for Steps 1 and 2.

- 2-month-old girl + red papules in groin area and intergluteal cleft; family has Hx of asthma; what’s the

diagnosis? à answer = Candida (diaper rash); not atopic dermatitis; the latter can occur in babies but

is more often on trunk, dorsa of hands, and face. Treat Candida diaper rash with topical -azoles or

nystatin.

- 48M + IV drug user + treated for 6 weeks in hospital on broad-spectrum antibiotics; intertriginous red

rash is seen; organisms are cultured as purple-budding organisms; diagnosis? à answer = Candida;

broad-spectrum antibiotics à increased risk of Candida infections.

- 42M + fever 100.8 F + diffuse, pink lesion shown on leg below; Q asks most appropriate treatment:

MEHLMANMEDICAL.COM 6
MEHLMANMEDICAL.COM

o Answer = oral dicloxacillin or cephalexin; diagnosis is cellulitis (infection of the dermis and

hypodermis [subcutaneous fat]); Staph aureus exceeds Strep pyogenes (Group A Strep) as

causal organism; must give beta-lactamase-resistant beta-lactam in the methicillin class (i.e.,

dicloxacillin, flucloxacillin) or first-generation cephalosporin (i.e., cephalexin, cephazolin), or

Augmentin (amoxicillin-clavulanate); amoxicillin and penicillin alone are wrong answers; 90%

of community Staph (i.e., MSSA) produces beta-lactamase, so amoxicillin and penicillin alone

will not work if Staph is the cause.

- 35M + fever 100.5 F + leg has lesion shown + Q asks most likely causal organism:

o Answer on NBME = Strep pyogenes (Group A Strep); diagnosis is erysipelas (infection of

upper dermis and superficial lymphatics); Group A Strep eclipses Staph aureus for erysipelas;

looks worse than cellulitis but is more superficial / “not as bad”; has characteristic “fiery red”

appearance and may appear well-demarcated with raised edges. Although Group A Strep >

Staph for erysipelas, Tx is same as cellulitis (oral dicloxacillin, cephalexin, or Augmentin)

because Staph can still cause it. Penicillin alone can be used for Strep pharyngitis.

- 7M + presentation shown; Q asks for the treatment:

MEHLMANMEDICAL.COM 7
MEHLMANMEDICAL.COM

o Answer = topical mupirocin; diagnosis is impetigo (school sores); Staph aureus exceeds

Group A Strep for both bullous and non-bullous types (bullous generally implies Staph); if

orals given, use dicloxacillin or cephalexin, but USMLE loves topical mupirocin for impetigo.

- 16M + cellulitis + BP of 80/40; Q asks which immunologic receptor(s) is/are bound in this patient’s

condition à answer = MHC-II and T-cell receptor; diagnosis is toxic shock-like syndrome caused by

exotoxin A (erythrogenic toxin) of Strep pyogenes (Group A Strep); mechanism is similar to Staph

aureus superantigen, TSST, which bridges MHC-II on macrophages and TCR, causing cytokine release

from macrophages.

o Toxic shock syndrome Qs will mention low BP in someone with cotton nasal packing or

tampons à answer is “MHC-II and TCR” bound by TSST of Staph aureus.

o In contrast, cellulitis causing shock, the answer will be Strep pyogenes (Staph will not be

listed) due to its exotoxin A.

- Neonate + diffuse pink body rash + desquamation of palms and soles; Q asks for molecular target of

the toxin in this condition; answer = desmosomes (hold adjacent keratinocytes together); diagnosis is

Staphylococcal scalded skin syndrome; epidermolytic exotoxin cleaves desmoglien-1 in desmosomes;

(+) Nikolsky sign (sloughing of the skin with friction).

- 12F + fever + sore throat + red tongue + pink maculopapular body rash; Dx + Tx? à scarlet fever

caused by Strep pyogenes; presents with “strawberry tongue” and salmon-pink body rash; Tx with

penicillin to prevent rheumatic heart disease.

- 14M + fever + rapid, irregular, jerking movements of limbs + following rash on legs as shown; Dx + Tx?

MEHLMANMEDICAL.COM 8
MEHLMANMEDICAL.COM

o Answer = erythema marginatum as seen in rheumatic fever; treatment is penicillin; erythema

marginatum is annular (ring-like) and serpiginous (serpent-/snake-like); movements are

Sydenham chorea.

- 10M + “yellow crusties” on his forearm for the past week + red urine; what’s the diagnosis? à answer

= post-streptococcal glomerulonephritis (PSGN); USMLE can write answer as “proliferative

glomerulonephritis” or “acute glomerulonephritis.”

o Proliferative glomerulonephritis = PSGN. Do not confuse this with diffuse proliferative

glomerulonephritis (SLE), or membranoproliferative glomerulonephritis (HepC, malignancy).

- 19M + burned leg playing with firecrackers + while in hospital develops infection of burn site that has

a yellow color; Dx? à answer = Staph aureus (golden staph); wrong answer is Pseudomonas (blue-

green color due to pyocyanin).

- 25F + breastfeeding + red, cracked, fissured nipple; Dx + Tx? à answer = mastitis; usually caused by

Staph aureus; Tx is oral dicloxacillin + continue breastfeeding through the affected breast.

- 25F + not breastfeeding + upper, outer quadrant non-fluctuant, warm, tender, red mass; Dx? à

answer = mastitis; wrong answer is abscess; mastitis = non-fluctuant; abscess = fluctuant; this is on

2CK obgyn CMS form; mastitis need not affect the nipple in breastfeeding woman.

- 25F + recently stopped breastfeeding + tender, fluctuant mass lateral to the nipple; patient is afebrile;

mass is not warm or red; Dx? à answer = galactocele (milk retention cyst); if abscess, they will say

red/warm +/- fever.

- 42F + inverted nipple + patient is worried because family Hx of breast cancer; Dx? à answer = ductal

ectasia; benign condition; as name implies, simply dilation/widening of lactiferous duct.

- 65F + red, eczematoid-appearing nipple + mass palpable beneath nipple; Dx? à answer = Paget

disease of breast; often associated with underlying ductal carcinoma in situ.

- 65F + peau d’orange of left breast + erythematous; Dx? à answer = inflammatory breast cancer;

peau d’orange appearance due to Coopers ligaments of breast (on NBME).

- 17M + presents as per image shown; what’s the diagnosis? (answers are either Propionibacterium

acnes or tinea faciei):

MEHLMANMEDICAL.COM 9
MEHLMANMEDICAL.COM

o Answer = acne; Propionibacterium acnes; not difficult, but I’ve seen enough students select

tinea faciei.

o First-line Tx for acne on USMLE is topical retinoids (i.e., topical tretinoin; not oral

isotretinoin; latter is only for severe acne). Topical retinoids (vitamin A) inhibit sebum

production; they cause photosensitivity and desquamation (peeling).

o Topical benzoyl peroxide is second-line for acne (although often co-administered with topical

tretinoin). It clears pores and kills bacteria.

o Topical clindamycin can be used if topical retinoids and benzoyl peroxide are insufficient; if

topical antibiotic is insufficient, oral tetracycline is used; the latter causes blistering

photosensitivity.

o Last resort is oral isotretinoin; must do beta-hCG (pregnancy test) before commencement

due to teratogenicity; oral isotretinoin does not cause problems with sperm in men; topical

retinoids in both men and women do not cause teratogenicity.

- 20F + being treated for acne with both topical + oral medications; her forehead is shown below; what

is the most appropriate recommendation for this patient?

MEHLMANMEDICAL.COM 10
MEHLMANMEDICAL.COM

o Answer = “avoidance of sun exposure”; above rash is classic photosensitivity caused by

topical retinoids; tetracycline photosensitivity tends to be blistering; do not choose answers

such as “avoidance of spicy/sweet foods” for acne questions.

- 16M + face shown in following image; Dx + Tx?

o Answer = Dx is tinea faciei; Tx = topical -azoles (clotrimazole, miconazole).

- 3M + fever of 103 F + stiff neck + low BP; following image of patient’s leg is shown. What is the most

likely causal organism?

o Answer = gram-negative diplococci; Dx is meningitis + characteristic non-blanching rash

caused by Neisseria meningitidis. Low BP can be endotoxic shock, but student should bear in

mind Waterhouse-Friderichsen syndrome is often asked; give hydrocortisone to increase BP

after normal saline is administered.

- 23M + 2-day Hx of soreness of left knee and right elbow + positive Finkelstein test on right hand +

cutaneous papules visualized on right wrist; Dx? à answer = gonococcal arthritis; will present one of

two ways on USMLE; 1) monoarthritis of large joint, such as the knee; 2) polyarthritis + tenosynovitis

(e.g., deQuervain) + cutaneous papules/vesicopustules. The USMLE will sometimes just have “gram-

negative diplococcus” as the answer.

- 28F + recently immigrated to US from India + rose spots on abdomen + severe constipation + fever

104 F + question asks how this condition is acquired; answer = “ingestion of fecal-contaminated

MEHLMANMEDICAL.COM 11
MEHLMANMEDICAL.COM

food”; diagnosis is typhoid (Salmonella typhi); humans are the reservoir; classically causes rose spots

on the abdomen + prostration (patient is lying supine + in pain) + either constipation or diarrhea; do

not confuse with the food poisoning Salmonella species (typhimurium and enteritidis), which

classically are acquired from poultry or turtles.

- 40F + penetrating trauma to thigh one week ago + skin has black appearance and crepitus on exam;

the most likely causal organism can also cause what? à answer = watery diarrhea; Dx is gas gangrene

due to Clostridium perfringens, resulting in subcutaneous emphysema (crunching of skin due to

underlying CO2 gas); this is due to production of lecithinase (phospholipase); C. perfringens also

causes watery diarrhea.

- 56F + poorly controlled diabetes + Pseudomonal sepsis + following image is shown; Dx?

o Answer = ecthyma gangrenosum; uncommon cutaneous infection seen in patients with

severe Pseudomonal infections.

- 23F + works as postal worker + horticulturist + has many pet birds; following image is shown + Q

describes lesion as an eschar with surrounding edema; Dx?

o Answer = Bacillus anthracis (anthrax); cutaneous anthrax can present classically with a black,

eschar lesion; the gram (+) rod increases cAMP and produces edema factor.

MEHLMANMEDICAL.COM 12
MEHLMANMEDICAL.COM

- 28F gives birth to stillborn neonate + she ate soft cheeses and deli meat while pregnant + stillborn has

diffuse granulomas on body; which of the following best describes the most likely causal organism? à

answer = gram (+) rods; Listeria can cause granulomatosis infantiseptica (severe intrauterine infection

often resulting in fetal demise + diffuse cutaneous granulomas).

- 39M + bilateral pneumonia + skin ulcer on back of hand (image shown) + low-grade fever + has many

pet rabbits; what’s the diagnosis?

o Answer = Francisella tularensis; cutaneous tularemia can present as ulcerative lesions; can

also cause bilateral atypical pneumonia; rabbits are classic source.

- 54M + type II diabetic + red rash under right axilla + rash appears bright coral red under Woods lamp

(image shown); Dx?

o Answer = erythrasma, caused by Corynebacterium minutissimum; skin infection of

intertriginous areas that shines/glows coral red under Woods lamp (holy shit Coral red, I

goin’ to Cairns, Australia now, but let’s not procrastinate).

- 40M + painful erythematous lesions in her axillae; an image is shown below; what’s the best

treatment for this patient?

MEHLMANMEDICAL.COM 13
MEHLMANMEDICAL.COM

o NBME answer = “surgical excision of lesions”; diagnosis is hidradenitis suppurativa, a skin

condition characterized by painful abscesses and sinus tracts, most commonly in the groin,

axilla, and under the breasts; frequently affects apocrine glands; cause is a combination of

genetic and environmental factors (idiopathic); NBME answer is surgical excision of lesions.

- 17M + athlete + strong body odor when finished with sports; Q asks what type of gland is responsible

à answer = apocrine; Types of glands:

o USMLE wants you to know apocrine glands are found in the groin and axilla and are

responsible for body odor. Part of the plasma membrane buds off with the substance

secreted by the cell. “Apocrine metaplasia,” is a term sometimes applied to fibrocystic

change of the breast (as well as “blue dome cysts” and “sclerosing adenosis”).

o Eccrine (merocrine) glands are simple sweat glands found all over the body. The secreted

substance is merely exocytosed from the cell.

o Sebaceous glands (holocrine) are those attached to hair follicles (not found on palms/soles).

The cell disintegrates when it secretes the substance.

- 34F + diabetic + frequently uses hot tub + image shown below; Dx?

MEHLMANMEDICAL.COM 14
MEHLMANMEDICAL.COM

o Answer = hot tub folliculitis; most frequently caused by Pseudomonas; increased risk of

Pseudomonal infections in diabetics (e.g., otitis externa).

§ Folliculitis = inflammation around hair follicle.

§ Furuncle (boil) = abscess involving single hair follicle (usually Staph aureus).

§ Carbuncle = cluster of boils (involving 2+ hair follicles) coalescing into one cutaneous

cavity of pus.

§ Abscess = collection of pus not specifically involving hair follicle.

§ For 2CK surg Qs, drain cutaneous pus collections then leave open to the air by

stuffing with sterile gauze + pulling out slowly daily.

- 62M + poorly controlled diabetes + black skin on perineum + patient is hemodynamically stable and

ABCs addressed; what’s the next best step in management? à answer = debridement of necrotic

tissue; diagnosis is Fournier gangrene (rare perineal/scrotal gangrene seen in older male diabetics);

Clostridium perfringens is most common culprit.

- 43F + recent penetrating trauma to leg + necrotic tissue that on examination appears to spread along

fascial planes; what’s the most likely diagnosis? à answer = necrotizing fasciitis; Tx = IV antibiotics +

debridement of necrotic tissue.

- 27F + painful 1-cm lump in labia majora; what’s the most likely causal organism? à answer =

polymicrobial; diagnosis is Bartholin gland cyst/abscess; uncomplicated cysts à treatment = warm

compresses or sitz bath; overt abscess à Tx answer = drainage.

- 8M + fever for 5 days + palms and soles desquamation + edema of dorsa of hands + cervical

lymphadenopathy + injection of conjunctiva and lips; Dx + Tx? à answer = Kawasaki disease; 5+ days

of fever is HY; Tx = aspirin + IVIG (never give aspirin to kids for other purposes because of Reye

syndrome).

- 24F + lives in Connecticut + went hiking five days ago + rash of wrists and ankles + rash migrates in

toward chest; Dx + Tx? à answer = Rocky Mountain spotted fever (Rickettsia rickettsii); classically

palms and soles rash, but USMLE has also said wrists + ankles; student should be aware rash is

centripetal (i.e., starts at palms/soles, or wrists/ankles, and moves inward to trunk); treatment is

doxycycline for anyone age 9 and older; pregnant women and children 8 and younger receive other

agents, such as amoxicillin.

MEHLMANMEDICAL.COM 15
MEHLMANMEDICAL.COM

- 6M + vesicular eruption on bottoms of feet, palms, and periorally; most likely causal organism? à

answer = “RNA virus; non-enveloped; non-segmented”; diagnosis is hand-foot-mouth disease caused

by Coxsackie A virus; this virus can also cause herpangina (posterior oropharyngeal vesicles).

- 24M + recently returned home from military service + fever of 101 F + cervical lymphadenopathy +

posterior oropharyngeal vesicles + painful vesicles inside the lip (nothing on outside of mouth/lips); Q

asks next best step in diagnosis? à answer = “PCR testing of vesicles”; viral culture is wrong answer;

diagnosis is HSV1/2; do not confuse with herpangina; HSV primary infection will classically present

with fever and lymphadenopathy.

- 54F + image shown below; Q asks most likely organism in terms of viral structure (i.e., DNA vs RNA;

enveloped vs non-enveloped; circular vs linear):

o Answer = DNA, enveloped, linear; Dx is HSV1/2; herpes labialis.

- 40F + works as dentist + image as shown below; what’s the MOA of the treatment?

o Treatment = DNA polymerase inhibitor (causes “chain termination”); acyclovir (or

valacyclovir); Dx is herpetic whitlow, which is an HSV1/2 infection of the finger, often caused

by inoculation from a cold sore; increased prevalence in dentists/hygienists.

- 40M + gardener + photo is shown; this is due to what type of toxicity?

MEHLMANMEDICAL.COM 16
MEHLMANMEDICAL.COM

o NBME answer = arsenic; Mees lines are white lines seen on fingernails in arsenic toxicity;

arsenic is present in small amounts in fertilizers, which causes plants to flourish; gardeners at

increased risk; arsenic can also cause palms/soles rash (arsenical keratosis).

- 8M + recent paranasal sinus infection + now presents with fever 103 F + painful, swollen eye as shown

below; Dx + Tx?

o Dx is orbital cellulitis (infection involving tissues posterior to orbital septum); Staph aureus

most common cause; rare sequela of adjacent infection, e.g., from the paranasal sinus;

medical emergency; requires IV antibiotics; preseptal cellulitis is a less severe version of

orbital cellulitis.

- 30F + painful, red bump on upper eyelid; image shown; Dx + Tx?

MEHLMANMEDICAL.COM 17
MEHLMANMEDICAL.COM

o Dx is hordeolum (stye); Staph aureus infection of sweat gland or oil duct; treat with warm

compresses.

- 44M + painless bump on eyelid; image shown; Dx + Tx?

o Dx is chalazion; blocked oil duct; not an infection; treat with warm compresses. 2CK in

particular likes hordeolum vs chalazion.

- 24M + painless ulcer on his penile shaft; Dx + Tx? à answer = primary syphilis (chancre sore

described); organism is Treponema pallidum; diagnose with darkfield microscopy to visualize

spirochetes; Tx with penicillin.

o Do not confuse with chancroid, which is painful and caused by Haemophilus ducreyi.

- 24M + Hx of unprotected intercourse + rash as shown in image below; KOH prep is negative; Q asks

what is most likely to confirm diagnosis:

o Answer on USMLE = fluorescent treponemal antibody(FTA); Dx is secondary syphilis showing

characteristic maculopapular/nodular body rash; palms and soles are classically affected in

secondary syphilis but Q need not mention it as per above; secondary syphilis is diagnosed

with serology; Q need not mention Hx of chancre from primary syphilis.

o VDRL/RPR ordered before FTA, but latter more specific. Former can be falsely positive in

patients with antiphospholipid syndrome (classically SLE patients with lupus anticoagulant).

MEHLMANMEDICAL.COM 18
MEHLMANMEDICAL.COM

o USMLE Q can also show you picture of wart-like lesions on the genitals (condylomata lata) +

tell you there’s palms/soles rash, then answer = Treponema pallidum; this is also secondary

syphilis presentation.

- 24M + Hx of unprotected intercourse + recently immigrated to US from Egypt + presents with below

image of lesion on forehead + 2/6 decrescendo holo-diastolic murmur auscultated on exam; the most

likely causal organism bears taxonomy most similar to which of the following:

o Answer = Leptospira interrogans; diagnosis in image above is gumma of tertiary syphilis;

syphilis is a spirochete, as are Leptospira, Borellia burgdorferi (Lyme disease), and Borrelia

recurrentis (relapsing fever). Tertiary syphilis can cause ascending aortitis (murmur is AR).

- 7M + from Massachusetts + family has pet dog + has rash as shown below; Dx + Tx?

o Answer = Lyme disease; rash is erythema chronicum migrans (classic target rash); the rash

need not be a target on USMLE; it can merely be circular with no clearing; but the target is

classic; USMLE can give two side by side images: 1) circular rash on limb that is not a target;

2) Bells palsy à student needs to infer this is Lyme disease even though rash isn’t a target.

Treatment in this kid is amoxicillin, not doxycycline.

MEHLMANMEDICAL.COM 19
MEHLMANMEDICAL.COM

o Treatment is doxycycline for most cases of Lyme; doxycycline is not given to children age 8

and younger or to pregnant women (causes teeth discoloration); if pregnant or age 8 and

younger, give amoxicillin.

o Be aware ceftriaxone can be given for severe Lyme that is disseminated, causing cardiac or

cognitive dysfunction.

- 24F + pregnant + Bells palsy + target rash; Q asks for treatment (answers are steroids, doxycycline,

ceftriaxone) à answer = ceftriaxone; doxycycline not given to pregnant women or children under age

8; student says, “Wait, but I thought you just said ceftriaxone is for severe Lyme; her presentation is

simple” à Yes, I agree, ceftriaxone is classically for severe Lyme, but in this Q, they force you to

choose it because we can’t give doxy; you need to be flexible in some cases (example on NBME).

- 25M + sleeps with pet dog + tick found on the dog + circular rash on arm + blood smear shows

phagocytes with intracellular berry cluster organisms; Dx + Tx? à answer = Ehrlichiosis (Ehrlichia

chaffeensis); bacterium spread by Ixodes tick (same as Lyme disease, Babesia, and Anaplasma); will

not cause target rash; “berry cluster organisms” or intracellular “morulae” may be seen.

- 40M + living in homeless shelter for past 4 months + itchy hands + image shown below topical

antifungals were attempted but not effective; Q asks for the treatment:

o Answer = topical permethrin; Dx is scabies (Sarcoptes scabiei); lesions classically described as

“linear burrows”; can become superinfected with Staph aureus.

o Disseminated scabies can occur in HIV patients; Tx is oral ivermectin.

o Topical permethrin is also treatment for pediculosis (lice).

- 19F + recent travel through southern US + stayed at dodgy AF motel on the side of the highway next

to a Denny’s and a Sonic + itchy lesions on arm as shown in image below; Q asks for organism:

MEHLMANMEDICAL.COM 20
MEHLMANMEDICAL.COM

o Answer on NBME = bed bugs; caused by an insect called Cimex.

- 6M + family recently immigrated to US from Albania + diffuse maculopapular rash + photograph of

buccal mucosa is shown below; Dx + Tx?

o Dx = measles (rubeola); Tx is supportive; image shows Koplik spots (pathognomonic whiteish

lesions on buccal mucosa); immigrant Hx on USMLE sometimes implies unvaccinated status.

- 6M + diffuse maculopapular rash + tenderness at base of occiput and behind the ears; Dx + Tx? à Dx

is rubella (German measles); Tx is supportive; suboccipital and post-auricular lymphadenopathy are

characteristic.

- 4M + fever + image shown below; what is the most likely causal organism (DNA vs RNA; enveloped vs

non-enveloped; segmented vs non-segmented)?

MEHLMANMEDICAL.COM 21
MEHLMANMEDICAL.COM

o Dx is mumps à causes POM à Parotitis, Orchitis, Meningitis; virus is RNA, enveloped, non-

segmented.

- Neonate + intracranial calcifications + hepatomegaly + rash as shown in image below; no murmurs; Q

asks most likely causal organism:

o Answer = congenital cytomegalovirus (CMV); image shows blueberry muffin rash; no

murmurs implies not rubella (causes patent ductus arteriosus).

- 5M + fever 2-3 days ago + brought in by mother to emergency with appearance as shown below; the

most likely causal organism can also cause what?

o Answer = aplastic anemia; Dx = Parvovirus B19 (Fifth disease); image shows classic “slapped

cheek” appearance; next best step in Dx = Parvovirus B19 IgM titers; if IgM titers are not

listed, choose bone marrow biopsy; increased risk of aplastic anemia in sickle cell. Once child

has developed the red cheeks, he/she has immunologically cleared the illness (i.e., if they

turn it into a communications style Q, tell parents to chill the fuck out / Relax because the

child has cleared the virus).

- 26F + works at daycare center + fever + rash as shown in image below; next best step in diagnosis?

MEHLMANMEDICAL.COM 22
MEHLMANMEDICAL.COM

o Answer = Parvovirus IgM titers; Parvo classically viral exanthem (rash) in adults.

- 17M + high school wrestler + Hx of atopy + has burning, painful rash on trunk + lymphadenopathy +

fever; image shown below; Q asks what’s the treatment?

o Answer = oral acyclovir (or valacyclovir); diagnosis is eczema herpeticum à HSV1/2 infection

superimposed on eczema; often self-inoculated from touching cold sore then cracked skin of

eczema; herpes infection can present with fever, lymphadenopathy, and tingling, burning,

and painful rash (herpetic neuralgia).

- 3M + undergoing chemotherapy for ALL + has rash as shown below; what’s the diagnosis?

MEHLMANMEDICAL.COM 23
MEHLMANMEDICAL.COM

o Answer = shingles; yes, pediatric shingles “is a thing”; patient is immunocompromised due to

chemotherapy.

o Shingles is aka “herpes zoster” and is caused by varicella zoster virus (VZV); herpes zoster is

not a virus name; herpes zoster literally is just another name for shingles, which is caused by

VZV (human herpes virus 3; HHV3), not HSV1/2 (HHV1/2).

- 84M + sudden-onset left-sided Bells palsy + lesions around the ear as shown below; what’s the

diagnosis?

o Answer = herpes zoster oticus (VZV); shingles of facial nerve; can be associated with Bells

palsy.

o Be aware shingles lesions can appear non-vesicular and black on USMLE.

o New FM form for 2CK wants you to know shingles vaccine indicated at age 60.

- 18F + immigrated to US from Mexico in elementary school + presents with crops of pruritic vesicles on

the trunk at different stages of healing; Dx? à answer = varicella (chickenpox); immigrant status

implies unvaccinated status; “crops of vesicles at different stages of healing” = classic VZV.

- 2M + fever of 103-4 F for 3 days; fever abruptly subsides, followed by rash as shown below; patient’s

vaccination Hx is up to date; what’s the diagnosis?

MEHLMANMEDICAL.COM 24
MEHLMANMEDICAL.COM

o Answer = roseola (HHV6); described as “spiking fever followed by a rash”; child will have high

fever for 2-3 days, followed by a rash.

- 22F + no past medical history + afebrile + itchy rash on back for past week as shown below; diagnosis?

o Dx = pityriasis rosea; caused by HHV6 or 7; self-limiting; starts as Herald patch (larger pink

ellipse above), usually on the back or trunk, then spreads upward onto the shoulder blades

(“Christmas tree distribution”); USMLE will show you image and expect you can make spot-

diagnosis.

- 50M + Hx of IV drug use + violaceous skin lesions as shown below; what’s the diagnosis?

o Answer = Kaposi sarcoma; usually caused by HHV 8 (Kaposi sarcoma-associated herpes virus)

in immunocompromised patients (i.e., AIDS, chemotherapy); Kaposi sarcoma are tumorous

lesions of vascular-lymphatic origin.

- 12M + undergoing chemotherapy for ALL + skin lesions as shown in image below; Q asks you to pick

the organism that causes it, but you see answers are all bacteria (i.e., HHV8 not listed):

MEHLMANMEDICAL.COM 25
MEHLMANMEDICAL.COM

o Answer = Bartonella henselae; Dx is bacillary angiomatosis; bacterium causes cat scratch

disease but is also known to cause Kaposi-sarcoma like lesions in immunocompromised

patients.

- 7F + has pet cat + papules on hand + lymph node biopsy shows granulomatous inflammation +

organism can be visualized using silver stain; Dx? à answer = cat scratch disease.

- 6M + went to pool party a week ago + has lesions on trunk as shown in image below; what’s the most

likely causal organism?”

o Answer = poxvirus (molloscum contagiosum); largest DNA virus; classically skin-colored or

reddish papules with central umbilication.

- 9M + comes to your clinic in Brazil + black ulcer on arm + vision loss + patient is successfully treated

with ivermectin; what’s the most likely diagnosis? à answer = Onchocerciasis (Onchocerca volvulus);

nematode (roundworm helminth) transmitted by black fly; second most common cause of blindness

worldwide after trachoma (Chlamydia A-C).

- 9M + lives in South America + swollen eyelid + shortness of breath on exertion + S3 heart sound on

examination; what’s the most likely organism? à answer = Trypanasoma cruzi (Chagas disease);

protozoan; unicellular eukaryote; spread by Reduviid bug (“kissing bug” because bite is painless); can

cause dilated cardiomyopathy and achalasia; swollen eyebrow is referred to as Romaña sign.

MEHLMANMEDICAL.COM 26
MEHLMANMEDICAL.COM

- 40F + traveled to Middle East for one month + has lesion on hand as shown below + elevated liver

enzymes + pancytopenia; the organism is transmitted by sandfly; what’s the most likely diagnosis?

o Answer = leishmaniasis; Leishmania donovani; protozoan; unicellular eukaryote; can cause

visceral disease known as kala azar, which is associated with ­ LFTs and pancytopenia.

- 39M + works at aquarium + presents with lesions as shown below; what’s the diagnosis?

o Answer = Mycobacterium marinum; causes red ulcers/blisters on hand/arm of those who are

exposed to aquatics. Do not choose Staph aureus if they specifically say aquarium /

waterpark in the question.

- 7M + received renal transplant last year + has preauricular reddish lesion shown below; Dx?

MEHLMANMEDICAL.COM 27
MEHLMANMEDICAL.COM

o Answer on USMLE = Mycobacterium avium intracellulare (MAI); can cause lymphadenitis

with classic reddish/violaceous lesion on the neck or preauricularly; MAI classically causes

lung disease in older women (Lady Windermere syndrome) and AIDS patients.

- 45M + recently immigrated to US from Libya with adult children + hypoesthesia of hands to pain and

temperature + nodularity of fingers and nose; the most likely causal organism most likely bears what

characteristic? à answer = “temperature sensitivity”; Dx = leprosy (Mycobacterium leprae); grows at

cooler temperatures; can cause neuropathy and disfiguration of face (leonine facies) and limbs.

- 17M + Hx of Celiac disease + lesions on elbows as shown in below photo; what is most likely to be

seen on skin biopsy?

o Answer = IgA deposition at dermal papillae; diagnosis is dermatitis herpetiformis (not actual

herpes, in contrast to eczema herpeticum; don’t confuse these conditions); this is a

cutaneous eruption that can occur in patients with Celiac.

- 35M + Hx of ulcerative colitis + lesion on forearm as shown in below photo; what’s the diagnosis?

o Answer = pyoderma gangrenosum; described as ulcer with necrotic debris; rare cutaneous

manifestation in IBD (usually ulcerative colitis).

MEHLMANMEDICAL.COM 28
MEHLMANMEDICAL.COM

- 32F + African-American + high serum calcium + image shown; Q asks what kind of hypersensitivity this

refers to:

o Answer = type III hypersensitivity; diagnosis is erythema nodosum secondary to sarcoidosis;

erythema nodosum is a panniculitis (inflammation of subcutaneous fat); can be caused by

autoimmune diseases like sarcoidosis and Crohn, as well as part of serum sickness due to

medications (e.g., sulfa).

- 16M + episodes of bloody stool over past two years + perianal abscesses/fistulae seen on physical

examination; Dx? à Crohn disease; increased risk since transmural inflammation in Crohn.

- 46M + pain during defecation + perianal skin tag visualized on physical exam + patient won’t allow

rectal exam due to exquisite pain; Dx? à anal fissure; usually posterior in the midline; NBME answer

is Sitz bath.

- 16M + painful 2-cm mass located at superior aspect of gluteal cleft; Dx + Tx? à answer = pilonidal

cyst/abscess; Tx = surgical closure.

- 20F + receives IM ceftriaxone + oral azithromycin for pelvic inflammatory disease; 3-5 days later she

has red rash in area of injection; diagnosis? à answer = Arthus reaction; type III hypersensitivity; can

be distinguished from type I (immediate) because type III takes a few days to appear.

o 20F + receives IM ceftriaxone + gets polyarthritis a few days later; Dx? à answer = serum

sickness; type III hypersensitivity.

- 34F with image shown below; Q asks which other condition is most immunologically similar:

MEHLMANMEDICAL.COM 29
MEHLMANMEDICAL.COM

o Image shows malar rash of SLE; answer = serum sickness, Arthus reaction, or erythema

nodosum; all four presentations are type III hypersensitivities (the correct answer is any type

III hypersensitivity listed in Q answer choices; the point is to know malar rash is due to

immune complexes, as are serum sickness, Arthus reaction, and erythema nodosum).

- 52F + increased serum creatine kinase + 3/5 strength of hips on physical exam; images as shown

below; what type of antibodies are classically seen in this condition?

o Answer = Anti-Jo1 antibodies; diagnosis is dermatomyositis; left image shows heliotrope

rash; often described as violaceous eyelids; do not confuse with malar rash of SLE; right

image shows shawl sign/rash; both dermatomyositis and polymyositis can have increased

serum CK and/or weakness on physical examination (in contrast, polymyalgia rheumatica will

have neither, and most often just pain + stiffness). Muscle biopsy shows T cell infiltrate.

- 40F + proximal muscle weakness + image shown below; Q asks for diagnosis:

o Dx = dermatomyositis; image shows Gottron papules; HY cutaneous finding; Dx is

dermatomyositis. Patients can also have “mechanics hands,” which are rough-surfaced /

scaly-appearing hands; do not go chasing fungal infections here.

MEHLMANMEDICAL.COM 30
MEHLMANMEDICAL.COM

- 6F + fever 102 F + pain, erythema, and warmth of left knee + Hx of several episodes of joint pains past

few years + Hb of 10.1 g/dL + MCV 72; diagnosis? à answer = juvenile rheumatoid arthritis; HY for

Peds; patients are susceptible to septic arthritis (as with this patient), but will often have Hx of several

episodes of non-septic joint pain; anemia of chronic disease common (MCV can be low; I’ve seen this

on multiple 2CK NBME/CMS Qs, where MCV is low, not normal); salmon pink body rash classic, but

only seen in maybe one-third of NBME/CMS Qs (as per my guesstimation).

- 25F + hands shown in image below; Q asks melanocyte # and melanin production in following

condition (i.e., ­, ¯, or no change):

o Answer = ¯ melanocyte #; ¯ melanin production; diagnosis is vitiligo; T cell-mediated

destruction of melanocytes; can be associated with other autoimmune polyglandular

syndromes, as well as IgA deficiency.

- Neonate + milky white skin + blonde hair + pale blue eyes; siblings and parents have darker

complexion; Q asks for melanocyte # and melanin production; answer = normal melanocyte #; ¯

melanin production; diagnosis is albinism; can be associated with many conditions, including PKU,

Chediak-Hegashi, and tyrosinase deficiency.

- 18F + pale complexion + freckles on face; Q asks for melanocyte # and melanin production of the

freckles; answer = normal melanocyte #, ­ melanin production; medical term for freckle is ephelis

(plural = ephelides).

- 69F + lesion on zygoma as shown below; Q asks for for melanocyte # and melanin production:

MEHLMANMEDICAL.COM 31
MEHLMANMEDICAL.COM

o Answer = ­ melanocyte #; no change melanin production; diagnosis is lentigo (age spot;

plural = lentigines).

- 34M + dark complexion + skin sample mixed up in lab with fair-skinned individual; Q asks melanocyte

# and melanosome # in dark-skinned individual à answer = no change melanocyte #; ­ melanosome

#; melanosomes are organelles within melanocytes that produce melanin.

- 25F + Hx of gastroesophageal reflux + hands shown in image below; Q asks for what condition this

patient is most likely to develop:

o Answer = pulmonary hypertension (secondary to pulmonary fibrosis); diagnosis is limited

scleroderma (CREST syndrome); left image shows Raynaud phenomenon, which is color

change due to vascular spasm and reactive hyperemia; the right image shows sclerodactyly,

which is tightening of skin of the digits.

o Raynaud phenomenon is not limited to scleroderma and can be seen in conditions such as

SLE, hyperviscosity syndrome, or can be familial (erythromelalgia).

- 42F + dysphagia + cracked corners of the mouth; image of nail shown below; Q asks for what is most

likely to be seen in this patient:

MEHLMANMEDICAL.COM 32
MEHLMANMEDICAL.COM

o Answer = low MCV; Dx is Plummer-Vinson syndrome à esophageal webs (dysphagia) +

angular cheilosis (cracked corners of mouth) + iron deficiency anemia (severe cases can

present with koilonychia [spoon-shaped nails] or pica [eating ice, clay, starch]).

- 16F + blood in stool + arthritis + erythematous/silvery scaling lesions on forehead, above upper lip,

and on elbows; attempting to remove one of the skin lesions causes bleeding; diagnosis?

à answer = psoriasis; patient here also has IBD; HLA-B27 sometimes associated (PAIR à Psoriasis,

Ankylosing spondylitis, IBD, Reactive arthritis); Auspitz sign is bleeding of psoriatic scales with attempted

removal; psoriatic lesions classically on extensor areas but can be on face/forehead; Munro

microabscesses are collections of neutrophils in the skin in psoriasis; psoriatic arthritis in some patients

shows “pencil-in-cup” deformity on hand x-ray; treat with topical calcipotriene (vitamin D derivative),

topical steroid, or coal tar; if topicals not effective or patient has systemic psoriasis (arthritis), oral

methotrexate or acitretin can be given; the latter is a vitamin A derivative.

- 8F + excoriations visible in flexor creases + occasional dry cough worse in winter; image of elbow

shown below; Q asks what type of hypersensitivity is most likely responsible for this patient’s

presentation:

MEHLMANMEDICAL.COM 33
MEHLMANMEDICAL.COM

o Answer = type I (immediate); diagnosis is atopy; image shows eczema (atopic dermatitis);

patient has cough-variant asthma (1/3 of asthma patients only have dry cough, usually worse

in the winter or with exercise); treat with oil-based emollient and topical corticosteroids; if

steroids used >5-7 days continuously, thinning of the dermis may occur.

- 18F + history of eczema + area over elbow is red, inflamed, and oozing; 6-year-old sister recently had

weeping papules on face; Dx in the 18-year-old? à answer = Staph aureus or Group A Strep

superinfection over eczema; likely inoculated from younger sister’s impetigo.

- 22M + recently treated with azithromycin for chlamydial urethritis; forearm is shown in below photo;

Dx?

o Answer = urticaria (hives); type I hypersensitivity; can be precipitated by various allergens,

including pollen, pet dander, and drugs.

- 39M + went hiking + used sunscreen over body + has linear vesicles on legs; Q asks best way to

prevent this condition; Dx? à answer = “avoidance of contact with weeds”; diagnosis is contact

dermatitis due to poison ivy/sumac; linear vesicles is hugely HY descriptor for poison ivy/sumac;

sunscreen will be the answer if they say rash on dorsa of hands, arms, and face (i.e., it’s everywhere);

nickel will be the answer for contact dermatitis if they mention vesicles on the wrist in someone who

MEHLMANMEDICAL.COM 34
MEHLMANMEDICAL.COM

wears a watch; contact dermatitis is type IV hypersensitivity (T cell-mediated); rash will appear within

days of exposure to irritant + will take a few days to go away following removal of the irritant.

- 38F + recently treated with trimethoprim/sulfamethoxazole for simple UTI + develops sloughing skin

over her arms; image shown below; Dx?

o Answer = Stevens-Johnson syndrome à autoimmune type IV hypersensitivity (T cell

response) resulting in detachment of skin covering <10% surface area of body; Nikolsky sign

is (+). Toxic epidermal necrolysis is sloughing of >30% surface area; 10-30% is an

intermediate form; classically caused by sulfa drugs and anti-epileptics (lamotrigine).

- 24F + tattoo a couple days ago + image shown below; Q asks for Dx:

o Answer = contact dermatitis from tattoo ink; asked on USMLE.

24F + nurse + allergic to bananas + image of hands shown below; Q asks diagnosis:

MEHLMANMEDICAL.COM 35
MEHLMANMEDICAL.COM

o Answer = type IV hypersensitivity; Dx = latex allergy (contact dermatitis); patients who are

allergic to bananas are often allergic to latex; contact dermatitis secondary to medical

adhesives (i.e., bandages) can also occur, causing well-demarcated rash.

- 38F + chronic dry skin on the legs + scratches same area repeatedly; no other past medical history;

image shown below; Dx?

o Answer = lichen simplex chronicus; dry, excoriated skin due to repeated scratching; can be

seen in patients with many different conditions.

- 23M + painful vesicular lesions on the lip + fever + lymphadenopathy + presents with rash on arms as

shown in below photo; diagnosis?

MEHLMANMEDICAL.COM 36
MEHLMANMEDICAL.COM

o Answer = Erythema multiforme; immune complex-mediated rash (type III hypersensitivity)

with many etiologies; HSV1/2 infection is classic cause.

- 21M + blistering skin lesions + oral involvement + no mention of medications recently taken +

Nikolsky sign (+); Q asks for the molecular target of the antibodies seen in this condition? à answer =

desmosomes (desmoglein proteins), which mediate adjacent keratinocyte adhesion; diagnosis is

pemphigus vulgaris, which is a blistering autoimmune skin condition caused by antibodies against

desmosomes; oral involvement is common; Nikolsky sign is positive; immunofluorescence will show a

net-like pattern.

- 21M + blistering skin lesions + no oral involvement + no mention of medications recently taken +

Nikolsky sign negative; Q asks for the molecular target of the antibodies seen in this condition? à

answer = hemidesmosomes, which stabilize basal epithelial cells to the basement membrane;

diagnosis is bullous pemphigoid; less severe than pemphigus vulgaris; usually no oral involvement in

bullous pemphigoid; immunofluorescence will show a linear pattern.

- 21M + formation of skin blisters with minor trauma; Q asks for what type of cell-cell interaction is

disrupted in this patient à answer = “basal:suprabasal” à weird answer, but on NBME; diagnosis is

epidermolysis bullosa, which is due to mutations (not antibodies, as with the aforementioned PV and

BP) in keratin 5 and 14 of the dermal-epidermal junction; formation of blisters with trauma is not

Nikolsky sign; the latter is removal / sloughing of the skin with friction, not blister formation with

friction.

- 26M + oral + genital ulcers + high ESR; Dx? à answer = Behcet disease.

63M + patchy facial erythema that worsens with spicy foods and alcohol; slight pain of rash in cold weather;

image of patient is below; Q asks simply for diagnosis:

MEHLMANMEDICAL.COM 37
MEHLMANMEDICAL.COM

o Answer = rosacea; multifactorial etiology; topical metronidazole sometimes effective;

avoidance of certain triggers, like spicy food or alcohol.

o Rhinophyma is a severe bulbous enlargement of the nose that can occur in some patients

with rosacea.

- 46M + high serum calcium + CXR shows bilar lymphadenopathy + image of face is shown below; Dx?

o Answer = lupus pernio (a cutaneous manifestation of sarcoidosis, not SLE, despite the name).

- 26F + presents during winter + painful/itchy toes + topical antifungals not effective; Dx?

o Answer = perniosis (chilblains); painful inflammation of distal capillaries due to repeated

exposure to cold air, followed by immersion in hot water (i.e., from bath/shower).

o This is different from frostnip and frostbite. Frostnip is cold-exposed skin (effects quickly

reversible; no skin damage); frostbite is more severe and can result in damage such as

blistering and necrosis.

- 28F + occasional painful single mouth ulcers (image shown below); Dx + Tx?

MEHLMANMEDICAL.COM 38
MEHLMANMEDICAL.COM

o Dx = aphthous ulcers (aphthous stomatitis); no Tx necessary; not related to HSV1/2; etiology

is multifactorial; can be precipitated by allergens such as spice; T cell-mediated.

- 35F + mouth ulcers + fever + treated two days ago for hyperthyroidism in hospital; Dx? à answer =

drug-induced neutropenia (propylthiouracil or methimazole); neutropenia (agranulocytosis) can

present as mouth ulcers (mucositis); HY drugs are the thionamides, clozapine, ganciclovir,

methotrexate.

- 37M + occasional itchy bumps on hands; sometimes in ring-like pattern; Dx?

o Dx = granuloma annulare; caused by cutaneous T cell response; occurs in young, healthy

patients; may present as small bumps that progress to annular/ring-like pattern; no Tx.

- 45F + hepatitis C positive + purple, pruritic skin lesions; image shown below; Dx?

MEHLMANMEDICAL.COM 39
MEHLMANMEDICAL.COM

o Answer = lichen planus; classically described as “The Ps” à purple, pruritic, polygonal

papules; however Qs need not mention they’re pruritic; you just need to know hepatitis C +

red/purple skin lesions = lichen planus.

- 28F + thrombocytopenia + polyarthritis + chronic sores of the cheeks and scalp + anti-Smith antibody

positive; diagnosis? à answer = SLE; dermatologic component is discoid lupus, which is a severe

cutaneous presentation seen sometimes in SLE patients; sores/scarring of the face, scalp, and ears;

thrombocytopenia (and leuko-/erythropenia) seen in many SLE patients due to anti-hematologic cell

line antibodies; arthritis most common presenting feature in SLE.

- Neonate born to mother who took methimazole during first trimester; photo of child is shown; what’s

the diagnosis?

o Answer = aplasia cutis congenita; absence of skin on an area of scalp; can be caused by

teratogens such as methimazole; dumb/seemingly pedantic detail, I know, but it was on

student’s exam.

- 5M + second episode of edema of face, hands, and arms; patient is prescribed danazol; image of

patient is shown below; what is the most likely mechanism for this condition?

o Answer = deficiency of C1 esterase inhibitor (not C1 esterase alone); diagnosis is hereditary

angioedema; condition characterized by recurrent swelling of various bodily regions; danazol

(androgen receptor partial agonist) causes liver to produce more C1 esterase inhibitor.

MEHLMANMEDICAL.COM 40
MEHLMANMEDICAL.COM

- 79M + purpura on forearms and dorsa of hands + normal hematologic studies; Q asks most likely

cause for this condition; answer = normal age-related changes; diagnosis is senile purpura; increased

fragility of dermal collagen and blood vessels secondary to normal senescence.

- 65M + worked in construction; yellow, thickened, coarsely wrinkled skin of forearms; Q asks most

likely cause for this condition; answer = sun exposure; diagnosis is solar elastosis (UV light exposure).

- NBME Q asks best way to prevent sun damage; “avoidance of sun” not listed as answer (would be

correct if listed); Q gives answers such as SPF 15, SPF 30, etc.; correct answer = “wear protective

clothing”; sounds obvious, but I’ve seen numerous students choose SPF 30, thinking there’s a trick.

- 33F + third trimester of pregnancy + has itchy erythematous/violaceous rash on abdomen within

stretch marks; image is shown below; Dx + Tx?

o Diagnosis = pruritic urticarial papules and plaques of pregnancy (PUPPP); “weird diagnosis,”

but asked on 2CK for obgyn; cause is sporadic/multifactorial; oral antihistamines (2nd gen H1

blocker) are Tx.

- 33F + third trimester of pregnancy + itchy rash around umbilicus + stretch marks not involved; image

is shown below; Dx + Tx?

MEHLMANMEDICAL.COM 41
MEHLMANMEDICAL.COM

o Dx is gestational pemphigoid; does not involve stretchmarks (unlike PUPPP); cause is

sporadic/multifactorial; Tx is oral or topical steroids.

- 33F + third trimester of pregnancy + intense, diffuse pruritis, especially on palms + soles; no skin rash;

serum bile acids are elevated; Dx + Tx? à answer = intrahepatic cholestasis of pregnancy; Dx by

measuring increased serum bile acids; Tx is ursodeoxycholic acid (ursodiol).

- 12M + nosebleeds for the past week + petechial rash + bleeding time 9 minutes + platelet count

90,000/uL; Dx + Tx? à answer = idiopathic (immune) thrombocytopenic purpura (ITP); cause is

antibodies against GpIIb/IIIa on platelets (type II HS); Dx with decreased platelet count (answer on

NBME; increased bleeding time for Dx is wrong answer).

- 67M + lesion shown on nose in photo below; Q simply asks the diagnosis:

o Answer = basal cell carcinoma (BCC); on USMLE, classically pearlescent / slightly translucent;

talengiectasias common; borders can sometimes be described as “heaped up” or “rolled”;

may or may not be ulcerated.

o Treatment for skin cancers on cosmetically sensitive areas such as the eyelid or nose can be

managed with Mohs micrographic surgery (on NBME).

- 30F + atypical skin lesion on neck + biopsy shows “islands and nests of basophilic cells” (shown

below); Q asks for the diagnosis:

MEHLMANMEDICAL.COM 42
MEHLMANMEDICAL.COM

o Answer = BCC; histo shows “islands and nests of basophilic cells.”

- 42M + receives topical immunomodulator for confirmed BCC; what is the drug he received? à

answer = imiquimod; stimulates toll-like receptor-7 (TLR-7).

- 62F + farmer + lesions on forearm/hand shown below; Q asks diagnosis:

o Answer = actinic keratoses (aka solar keratoses); precursor to squamous cell carcinoma

(SCC); classically described as red/scaly lesions on forehead, ear, or arms of fisherman,

farmers, or construction workers. Cryotherapy is usual treatment.

- 74M + fisherman; forehead and ear are shown below; Q asks for the diagnosis of the ear lesion:

o Answer = squamous cell carcinoma (ear; right image); forehead shows actinic keratoses;

actinic keratoses can classically progress to SCC; patient has many actinic keratoses and was

likely at risk of developing SCC, as with the ear lesion; on USMLE, SCC will not have

telangiectasias or pearlescent/translucent appearance (as with BCC).

MEHLMANMEDICAL.COM 43
MEHLMANMEDICAL.COM

o Both SCC and BCC can be ulcerated and/or have rolled edges (I’ve seen rolled/ulcerated SCC

on NBME exam, even though rolled edges are textbook BCC description; patient had actinic

keratoses).

o Do not memorize BCC vs SCC as necessarily occurring in certain locations (i.e., upper vs lower

lip, etc. USMLE has zero regard for this stuff. You need to look for telangiectasias and pearly

appearance for BCC, versus absence for SCC).

- 60M + 2 packs cigarettes daily for 30 years + lesion on back; biopsy is shown below; Q asks Dx:

o Answer = SCC; histo shows keratin pearls (pink circles), which are classic for SCC; USMLE

wants you to know “keratin pearls + intercellular bridges” = SCC, the same way “islands and

nests of basophilic cells” = BCC.

o Smoking + immunodeficiency (e.g., HIV) are HY risk factors for SCC.

- 70F + chickenpox scar on chin since childhood + recent abnormal growth of the scar + biopsy confirms

neoplasia; Q asks for the diagnosis à answer = SCC à a Marjolin ulcer is an SCC that arises from a

prior scar or site of trauma/burn. This is an important factoid Dx for 2CK surgery as well.

- 68F + rough-surfaced grey/white lesion on labia majora; Dx + Tx? à answer = lichen sclerosus; must

do biopsy to rule out SCC before topical steroids; the latter are effective treatment. LS can also be

perineal.

- 65F + lesion on face shown below; Q asks for the diagnosis:

MEHLMANMEDICAL.COM 44
MEHLMANMEDICAL.COM

o Answer = keratoacanthoma; can be confused with SCC; described as dome-shaped with a

hyperkeratotic core, surrounded by a wall of inflamed skin; Tx is surgical excision.

- 23M + 1-cm lesion on arm as shown in photograph below + lesion has not changed recently + uncle

died of melanoma; what’s the next best step in management?

o Answer = observation; diagnosis is nevocytic nevus (benign mole); when choosing excisional

answers on USMLE, think ABCDE à Asymmetry, Border (irregular), Color (variegated),

Diameter (>1cm), Elevation.

- 45F + lesion on leg shown in photo below; what’s the next best step in management?

o Answer on NBME = excisional biopsy; suspected malignant lesions in non-cosmetically

sensitive areas (i.e., not on the head/neck) can simply be excised with narrow margins;

excision of additional tissue is important if margins are positive (i.e., entire lesion wasn’t

excised).

o “Full-thickness biopsy” answer on USMLE for suspected melanoma on back of neck (NBME Q

gives lesion similar to above on neck, with full-thickness biopsy as answer). Excision of lesion

MEHLMANMEDICAL.COM 45
MEHLMANMEDICAL.COM

is management for neck lesions if full-thickness biopsy confirms melanoma; do Mohs

micrographic surgery for facial lesions.

o Punch biopsy is a type of full-thickness biopsy; if Q asks, choose side of lesion for biopsy,

rather than middle of lesion.

o Students will sometimes ask about shave biopsy. I have never seen this assessed on NBME,

but literature says it can sometimes be used to remove superficial non-pigmented lesions

where the clinician does not suspect melanoma (performing shave biopsy on melanoma can

create problems for assessing depth, prognosis, and therapy).

- 45M + confirmed melanoma; Q asks which aspect most relates to prognosis; answer = depth of lesion.

Wrong answer is “lymphocytic infiltrate”; the latter is good for prognosis, as immunosurveillance

functions to suppress skin cancers.

- 32F + confirmed melanoma + receives aldesleukin; Q asks which cytokine this relates to à answer =

IL-2; aldesleukin is a recombinant interleukin-2 that can be used in Tx of melanoma and RCC; IL-2

normally functions to stimulate T cells.

- 30F + African-American + lesion on foot as shown below; what’s the diagnosis?

o Answer = acral lentiginous melanoma; melanoma of palms/soles (areas not usually exposed

to sun); more common in persons of African and Asian descent.

- 34F + Asian-American + fingernail shown in photo below; diagnosis?

MEHLMANMEDICAL.COM 46
MEHLMANMEDICAL.COM

o Answer = acral lentiginous melanoma; USMLE will show you image of ALM on the palm/sole,

or subungual.

- 29M + nailbed lesion + radiating pain incited by cold temperature; Dx? à answer = glomus tumor;

tumor of glomus body, which is type of modified smooth muscle cell in fingers/toes that assists in

thermoregulation.

- 67M + lesion on face as shown in photo below; biopsy shows malignant cells growing laterally along

the basement membrane without dermal invasion; Q asks diagnosis:

o Answer = lentigo maligña; considered melanoma in situ; starts as black/brown “stain” that

grows laterally within stratum basale without penetration; once it invades, it is called lentigo

maligña melanoma (Hutchinson melanotic freckle); the terminology can sound confusing

because lentigo maligña is still technically melanoma, albeit in situ, but once it invades it

takes on “melanoma” as the suffix. This is on NBME exam.

- 37F + confirmed melanoma + two other first-degree family members also had melanoma; Q asks for

which gene is most likely associated in this patient à answer = BRAF; proto-oncogene, not a tumor

suppressor; BRAF codes for BRAF serine-threonine kinase. You do not need to know vemurafenib for

USMLE; only reason I mention it here is because some students may ask about it.

- 83F + facial lesions shown in photo below; Q asks for diagnosis:

MEHLMANMEDICAL.COM 47
MEHLMANMEDICAL.COM

o Answer = seborrheic keratoses; sun exposure and old age are biggest risk factors; described

as “greasy,” or waxy, skin growths that appear like they can be “peeled off”; not malignant.

o Sign of Leser-Trélat = sudden, eruptive seborrheic keratoses secondary to underlying, visceral

malignancy (i.e., unrelated to age or sun).

- 23M + skin-colored, painless 4-mm growth on penile shaft; Q asks most likely viral etiology; answer =

human papillomavirus 6 or 11; cause condylomata acuminata (warts); HPV 6 and 11 can also cause

laryngeal papillomatosis (vocal cord growths) in pediatrics (due to vertical exposure from birth canal);

HPV 16 and 18 are HY strains for cervical/vaginal/penile SCC.

- 55M + fungating mass from the rectum; Q asks next best step in Dx? à answer = biopsy of the mass

(simple Q on surgery form); likely SCC from HPV 16/18.

- 55M + 3-month Hx of pencil-like stools + 3-cm fungating mass just inside anal verge; Q asks next best

step in management (biopsy of mass not listed) à answer = colonoscopy; wrong answer is “surgical

excision of mass” and chemo/radiotherapies. Apparently colonoscopy first needs to be done to

evaluate for extent of colonic involvement before definitive surgical management is employed.

- 15M + mouth shown in photo below; Q asks for what kind of polyps he most likely has:

o Answer = hamartomatous; Dx is Peutz-Jeghers syndrome; combination of perioral melanosis

(sophisticated way of saying hyperpigmentation around the mouth/lips) and hamartomatous

colonic polyps.

o Perioral melanosis also seen in Carney complex (cardiac myxoma in a kid, perioral melanosis,

endocrine hypersecretion). Not fucking with you. It’s on the USMLE.

- 6M + jaw lesion as shown in photo below; Q asks for molecular function of gene involved:

MEHLMANMEDICAL.COM 48
MEHLMANMEDICAL.COM

o Answer = transcription factor; Dx is Burkitt lymphoma; usually t(8;14) translocation (can be

2;8, and 8;22); c-MYC gene; codes for transcription factor; “starry sky” appearance on histo.

o Focus here isn’t heme/onc, but HY point: do not confuse with follicular lymphoma, which is

t(14;18) translocation, BCL-2 gene; codes for anti-apoptotic molecule; presents as

waxing/waning neck mass over 1-2 years.

- 49M + smoker + rough, white lesion on lateral aspect of tongue that does not scrape off; Dx? à

answer = leukoplakia; precancerous lesion to SCC; caused by smoking / chewing tobacco.

- 32M + HIV positive + white lesions on lateral tongue that do not scrape off; Q asks for the viral

etiology à answer = EBV; Dx is oral hairy leukoplakia; not precancerous; caused by hyperkeratosis;

one of the most common presentations of HIV (in addition to oropharyngeal candidiasis).

- 54M + IV drug-user + lesion on leg shown in photo below + biopsy shows malignant T cells with

cerebriform nuclei; Q asks which virus is most likely responsible:

o Answer = Human T cell lymphotropic virus (HTLV-1/2); diagnosis is mycosis fungoides, which

is a cutaneous T cell lymphoma; T cells have nuclei with classic “cerebriform” appearance;

Pautrier microabscesses are atypical T cells in epidermis; HTLV-1/2 is retrovirus (RNA,

enveloped) most similar to HIV; increased prevalence in IV drug-users and persons in Japan

and Caribbean; you do not need to be able to do a spot-diagnosis here; Q will pretty much

always give you enough descriptors in the vignette.

MEHLMANMEDICAL.COM 49
MEHLMANMEDICAL.COM

- 54M + Hx of IV drug-use + diffuse red body rash as shown in photo below + blood smear shows

leukemic T cells with cerebriform nuclei; Q asks for structure of viral etiology (i.e., RNA vs DNA;

enveloped vs non-enveloped):

o Answer = RNA, enveloped; virus is HTLV-1/2; diagnosis is Sezary syndrome; T cell leukemic

extension of mycosis fungoides; diffuse exfoliative erythroderma is classic.

- 82F + had gallbladder removed at age 58 + afebrile + abdomen shown below; Q asks next best step in

diagnosis?

o Answer = CT of abdomen with contrast; Dx is pancreatic cancer; image shows jaundice (yes, I

jacked up the saturation to make the patient yellow AF); remote cholecystectomy denotes

impossibility of choledocholithiasis; this Q asked on Surg NBME; if the Q tells you in the last

line that CT shows no abnormalities, next best step = ERCP à look for cholangiocarcinoma.

o Head of pancreas cancer impinges on common bile duct à jaundice with increased ALP and

direct bilirubin; pancreatic enzymes are normal; if gallstone pancreatitis, enzymes up.

- 40M + history of shooting groin pain + elevated serum glucose + red rash on abdomen; Q asks for

which pancreatic cancer type is the Dx à answer = glucagonoma à can cause ­ glucose + necrolytic

MEHLMANMEDICAL.COM 50
MEHLMANMEDICAL.COM

migratory erythema (rash). Patient has MEN1 (pancreas, parathyroid, pituitary); shooting groin pain =

ureterolithiasis; hyperparathyroidism à high Ca2+ à stones.

- 40M + flushing of face + diarrhea + low serum potassium; Dx? à answer = VIPoma; classically WDHA

syndrome à Watery Diarrhea, Hypokalemia, Achlorhydria; 2CK Surg Q gives facial flushing.

- 27F + 20 weeks’ gestation + scattered skin lesions on trunk and arms, as shown in photo below;

patient also has axillary/groin freckling; Q asks inheritance pattern:

o Answer = autosomal dominant; Dx is neurofibromatosis type I (NF1); image shows café au

lait spot; one of the phakomatoses (neurocutaneous disorders à NF1/2, VHL, TSC, Sturge-

Weber); NF1 à café au lait spots, neurofibromas, axillary/groin freckling, optic glioma,

pheochromocytoma; for some reason, USMLE likes NF1 in obgyn Qs, even though the

condition is genetic / unrelated to obgyn.

- 7M + lesions shown on face in image below; Q asks inheritance pattern:

o Answer = autosomal dominant; Dx is tuberous sclerosis; image shows adenoma sebaceum

(angiofibromas); one of the phakomatoses; intracranial/periventricular nodules (tubers),

adenoma sebaceum, cardiac rhabdomyoma, lymphangeoleiomyomatosis, subungual

fibromas, renal angiomyolipoma.

MEHLMANMEDICAL.COM 51
MEHLMANMEDICAL.COM

- 17M + Hx of epilepsy + physical examination shows violaceous papules in a temporal distribution; Dx?

à answer = Sturge-Weber; classically Port Wine-stain birthmark (nevus flammeus) in textbooks, but

USMLE can describe this as “violaceous papules in a temporal distribution”; associated with

leptomeningeal angioma (causing seizure) and glaucoma.

o Classic Port wine stain birthmark, as seen in Sturge-Weber; condition is not inherited and is

due to somatic mosaicism.

- 3M + deficiency of a-galactosidase A + buildup of ceramide trihexoside + heart/kidney issues + red

lesions on skin shown in image below; Q asks diagnosis:

o Answer = Fabry disease; lysosomal storage disease; X-linked recessive; image shows

angiokeratomas, which are dilated capillaries forming papules.

- 6M + skin lesions as shown in image below + polyostotic fibrous dysplasia + testes large for

gestational age; Dx?

MEHLMANMEDICAL.COM 52
MEHLMANMEDICAL.COM

o Answer = McCune-Albright syndrome à triad of “coast of Maine” café au lait spots (above

image), polyostotic fibrous dysplasia (bone is replaced by fibrous tissue), and endocrine

hypersecretion (classically precocious puberty).

- 3M + retained primary teeth + eczema + recurrent Staphylococcal abscesses; Dx? à answer = hyper-

IgE syndrome (Job syndrome) à FATED à coarse Facies, Staphylococcal Abscesses, retained primary

Teeth, hyper-IgE, Dermatologic abnormalities (eczema).

- 3M + delayed separation of umbilical cord at birth + recurrent skin infections without pus + biopsy

shows decreased neutrophils at sites of skin infection; Dx? à answer = leukocyte adhesion deficiency;

defective LFA-1/CD18 integrin.

- Neonatal male + family recently immigrated to US from China; back is shown in image below; Q asks

for next best step in management?

o Answer = schedule routine follow-up; wrong answer is contacting child protective services;

diagnosis is Mongolian spot (blue nevus), a form of benign birthmark where dermal

melanocytes fail to migrate superficially to stratum basale; often mistaken for child abuse.

- 22M + BMI 36 + neck shown in image below; what is most likely to be seen in this patient?

MEHLMANMEDICAL.COM 53
MEHLMANMEDICAL.COM

o Answer = hyperinsulinemia; image shows acanthosis nigricans (brown/black velvety skin,

usually along nape of neck); associated with hyperinsulinemia and type II diabetes; can also

be seen in patients with underlying, visceral malignancy (e.g., gastric adenocarcinoma).

- 42M + lantern jaw + increased hat/shoe size + high BP + image shown below; Q asks next best step in

diagnosis:

o Answer = measure serum insulin-like growth factor I (IGF-1); wrong answer is measure serum

growth hormone; diagnosis is acromegaly; GH causes liver to secrete IGF-1, which in turn

promotes growth of tissues; skin tags can be seen secondary to insulin resistance; growth

hormone excess causes insulin resistance (tangential, but for Step 1, choose ­ activity for

catabolic/gluconeogenic enzymes; ¯ activity for anabolic enzymes, since ¯ insulin effect).

- 3M + coloboma of the iris + ventricular septal defect + ear is shown below; Q asks what else is likely to

be seen in this patient?

MEHLMANMEDICAL.COM 54
MEHLMANMEDICAL.COM

o Answer = atresia of the choanae; Dx is CHARGE syndrome à Coloboma of the iris, Heart

defects, Atresia of the choanae (baby turns blue during breastfeeding and pink when crying;

Dx by inserting nasogastric tube), Retardation of growth/development, Genitourinary

anomalies, Ear anomalies (protruding ears with lack of earlobe).

- 7M + hole seen on preauricular aspect of ear bilaterally; patient is asymptomatic; Q asks treatment:

o Answer = no Tx necessary; preauricular pits are benign finding, usually not associated with

any congenital disorder; can become infected; surgical closure if frequent infections.

- Neonate + lesion superior to pinna in image shown below; Dx+ Tx?

o Answer = no treatment necessary; Dx is strawberry hemangioma; benign capillary tumor that

will grow slightly then regress spontaneously within a few years; no Tx necessary unless

causing functional impairment.

- 50F + photo of lesions on neck shown below; Dx + Tx?

MEHLMANMEDICAL.COM 55
MEHLMANMEDICAL.COM

o Answer = cherry hemangiomas; no Tx necessary; benign capillary tumors caused by UV light;

greater prevalence with increasing age. Students confuse with strawberry hemangioma in

peds.

- Neonate + tuft of hair seen on lower back; Dx? à answer = spina bifida occulta; NBME exam has one

answer written as “spinal dysraphism,” rather than spina bifida.

- 50M + long-standing Hx of ulcerative colitis managed with multiple medications; abdomen shown in

image below; Q asks patient is at increased risk for what?

o Answer = osteoporosis (many answers possible); Dx is Cushing syndrome (from exogenous

prednisone used in autoimmune disease); USMLE wants you to know purple striae are

classically Cushing (weakening of dermal collagen + capillary walls à micro bleeding into

skin).

o Cushing syndrome can also cause hyperpigmentation due to ­ ACTH in some patients,

including patients with small cell (i.e., need not be Cushing disease [anterior pituitary tumor

secreting ACTH]); NBME Q has hyperpigmentation in small cell patient.

- 41M + serum glucose 130 mg/dL + sore hands + hand x-ray shows Heberden nodes; physical exam

shows darkening of the skin of the forearms; Q asks next best step in diagnosis? à answer = check

serum ferritin; Dx is hereditary hemochromatosis; can present as “bronze diabetes”; diabetes due to

deposition of iron in tail of pancreas; hemosiderin deposition in skin causes hyperpigmentation

(hemosiderosis); arthritis is pseudogout, not osteoarthritis (two of the biggest risk factors for

pseudogout are hereditary hemochromatosis and primary hyperparathyroidism; can present as OA-

like presentation in someone with aforementioned conditions, or as monoarthritis of large joint, such

as the knee).

MEHLMANMEDICAL.COM 56
MEHLMANMEDICAL.COM

- 30M + 6-month-Hx of fatigue + serum potassium 6.0 mEq/L, sodium 137 mEq/L, bicarb 23 mEq/L +

eosinophils 23% + hyperpigmentation of forearms; Q asks which diagnostic test indicated à answer =

ACTH stimulation test; Dx is Addison disease (primary hypoadrenalism); sodium and bicarb often in

normal range in aldosterone derangement (especially HY on 2CK); adrenal insufficiency can cause

eosinophilia (all over 2CK); do not go chasing stool ova and parasites; hyperpigmentation due to lack

of cortisol negative feedback at hypothalamus and anterior pituitary à increased ACTH production

(precursor is POMC, which will become both ACTH and a-MSH; both ACTH and a-MSH can increase

pigmentation).

- Neonate + violaceous lesion on leg shown in photo below; platelet count 50,000/uL; Q asks

mechanism for thrombocytopenia:

o Answer = platelet sequestration; Dx is Kasabach-Merritt syndrome (aka infantile

hemangioma with thrombocytopenia); asked several times on 2CK Peds assessment; not

strawberry hemangioma; platelets can be sequestered within lesion; Tx is surgical.

- 3M + brought in by mother for pain in right arm after falling off swing on playground; x-ray shows

fracture of the humerus; photo of hand shown below; Q asks next best step?

MEHLMANMEDICAL.COM 57
MEHLMANMEDICAL.COM

o Answer = contact child protect services; photo shows classic circular appearance of cigarette

burn; can also be on face / resemble impetigo if at different stages of healing; humerus

fractures uncommon and can be from blunt-force trauma; child abuse classically spiral

fractures (rotational force), but emphasis in above Q is the cigarette burn.

- 22F + lesions on ear shown in image below + occurred following removal of ear piercings several

weeks ago; Dx?

o Answer = keloid scars; disorganized growth of collagen type I and III; scar grows beyond

boundaries of original wound (in contrast to hypertrophic scars that may resemble keloids

but do not grow beyond boundaries of original wound). Tx is surgical excision, although

recurrence is common. Benign.

- 22M + face shown in photo below; vitals normal; no past medical Hx; Dx?

o Answer = pseudofolliculitis barbae (razor bumps); increased prevalence in African descent;

curly beard hair grows back into the skin; Tx is to allow the beard to grow; USMLE Q will

show you image and just ask the spot-Dx.

- 65F + Hx of pain in buttocks/thighs when walking his dog + image of foot shown; what’s the most

likely cause of this finding?

MEHLMANMEDICAL.COM 58
MEHLMANMEDICAL.COM

o Answer = arterial insufficiency; image shows arterial ulcer; often punched-out in appearance,

well-demarcated; usually tops/bottoms of feet/toes; patients will frequently have Hx of

intermittent claudication, diabetes, CABG, etc. Arterial disease often associated with trophic

changes of legs demonstrating shiny skin with loss of hair.

- 50F + image shown below; what’s the most likely cause of this finding?

o Answer = chronic venous insufficiency; image shows venous ulcer; classically large, sloughy

lesion around the ankle / medial malleolus; hyperpigmentation common in venous disease

due to congestion and hemosiderin extravasation (“brawny edema”).

o USMLE wants you to be able to spot-diagnose arterial vs venous ulcers. Exceedingly HY for

2CK surgery Qs in particular.

- 54M + leg shown below; no other past medical Hx; Q asks for next best step in Dx + Tx?

MEHLMANMEDICAL.COM 59
MEHLMANMEDICAL.COM

o Image shows varicose veins; Dx à Duplex venous ultrasonography for venous disease /

valvular insufficiency; Tx is compression stockings; do not choose surgical interventions for

varicose veins on USMLE.

- 50M + 1-cm painful, palpable “cord” around the ankle / tracks up toward the knee; image is shown

below; Q asks for Tx:

o Answer on NBME = subcutaneous enoxaparin; Dx is superficial thrombophlebitis; similar to

DVT, but in more superficial vein; treated with heparin; wrong answer is compression

stockings; difficult Q since compression stockings common answer for venous disease, but if

patient has active venous occlusion (i.e., DVT or superficial thrombophlebitis), give heparin.

- 48M + heavy smoker + hands shown below + no other significant past medical history; Dx + Tx?

o Dx = thromboangiitis obliterans (Buerger disease); Tx = smoking cessation; condition is digital

gangrene in males who are heavy smokers; contrasts from gangrene due to diabetes, which

will be pedal.

MEHLMANMEDICAL.COM 60
MEHLMANMEDICAL.COM

- 66F + type II diabetes + HbA1c 9.8% + metoclopramide is one of her meds; image of foot shown

below; x-ray of the foot shows disorganization of the tarsals/metatarsals; Q asks most likely cause of

the lesion shown:

o Answer = “lack of appropriate joint sensation”; Dx = Charcot joint (neuropathic joint); patient

cannot feel her feet due to peripheral neuropathy; vignette description points away from

pure arterial ulcer (plus arterial ulcers are more distal/punched-out, rather than around the

ankle); metoclopramide implies patient has diabetic gastroparesis, hence is on the

prokinetic; gastroparesis implies advanced neuropathy; USMLE can give you Hx of

neurogenic/hypotonic bladder as well (i.e., on bethanechol).

- 62M + hepatitis C + decreased serum C4 + legs shown in image below; Dx?

o Answer = cryoglobulinemia (presenting as livedo reticularis) secondary to hepatitis C;

cryoglobulins are immune complexes that precipitate at cold temperatures (type III HS);

livedo reticularis is a mottled, reticulated vascular pattern that has many etiologies;

cryoglobulinemia is associated with decreased serum complement protein C4 (in contrast, C3

is sometimes down in SLE flares and Group A Strep infections).

MEHLMANMEDICAL.COM 61
MEHLMANMEDICAL.COM

- 65M + smoker + appearance shown in photos below; Dx?

o Answer = Pancoast tumor causing SVC syndrome; photos show Pemberton sign (facial

erythema due to diminished venous return when arms are raised above head).

o Children with ALL who have (+) Pemberton sign have T cell variant (thymic lesion); normally

ALL is B cell, without thymic lesion.

- 49M + alcoholism + image shown below; Q asks which vessel(s) is/are experiencing congestion:

o Answer = superficial epigastric veins; Dx is caput medusae; sometimes seen in severe portal

hypertension secondary to cirrhosis.

- 70F + chronic alcoholism + hands shown below; Q asks mechanism for this finding:

o Answer = “failure of the liver to degrade estrogen”; palmar erythema (above image), spider

angiomata, and gynecomastia are classic hyper-estrogenic findings seen in advanced liver

MEHLMANMEDICAL.COM 62
MEHLMANMEDICAL.COM

disease; Q can also have “hyperestrogenism” as the answer; if the latter isn’t listed,

“decreased testosterone” is also correct on one of the NBMEs.

- 38F + was told when she was younger she had a “click” in her heart + recent dental procedure + fever

of 103 F + 3/6 holosystolic murmur; finger is shown below; Q asks next best step in management:

o Answer = blood cultures; diagnosis is subacute endocarditis due to Strep viridans (Hx of

dental procedures in someone with valve abnormality; this patient has Hx of mitral valve

prolapse); do blood cultures before antibiotics; then do transesophageal echocardiogram

(TEE) to diagnose; murmur above is mitral regurg (preceding MVP would have been mid-

systolic click). Patients can also have Janeway lesions / Osler nodes in endocarditis.

- 70M + smoker + recently underwent AAA repair + foot shown below; Dx?

o Answer = cholesterol emboli; due to embolization of cholesterol plaques to distal

vasculature; Hx of AAA repair classic; atheroma launch off to distal arterioles/capillaries.

- 24M + snowboarding accident where he collided with tree + severely painful and distorted left thigh +

platelets are 50,000/uL; on examination, patient’s chest is shown below; Dx?

MEHLMANMEDICAL.COM 63
MEHLMANMEDICAL.COM

o Answer = fat embolism; patient has femoral shaft fracture à release of bone marrow fat

into blood à chelates platelets à thrombocytopenia; petechial rash on chest due to

occlusion of microvasculature by fat; thrombocytopenia in and of itself is not responsible for

petechial rash (for instance, this finding is not classic in ITP).

- 29M + laparotomy 3 months ago following gunshot wound + now has erythematous, tender nodule

along incision line + afebrile; patient is treated appropriately and light microscopy of lesion is shown

below; Dx?

o Answer = suture granuloma; immune reaction from residual foreign suture material; Tx is

removal of suture. NBME Q shows histo revealing foreign body surrounded by histiocytes

(activated macrophages).

- 49M + rash over axillae + alopecia + diminished smell and taste; Q asks which nutrient is deficient à

answer = zinc; classically causes anosmia + hypogeusia; can also cause alopecia; neonates with

congenital malabsorption (acrodermatitis enteropathica) can have rash on face.

- 82F + “appears ill” + bruises on forearms + bleeding around hair follicles and from gums; Dx? à

scurvy (vitamin C deficiency); perifollicular hemorrhages and oral mucosal bleeding are classic.

- 70M + image of neck shown below; Q asks for nutrient deficiency:

MEHLMANMEDICAL.COM 64
MEHLMANMEDICAL.COM

o Answer = niacin (vitamin B3); image shows Casal necklace; B3 deficiency causes pellagra à

3Ds à dementia, dermatitis, diarrhea; the photodermatitis is classically Casal necklace, but

can also be described as merely hyperpigmentation of the forearms in a patient with

increased bowel motions and delirium (one of the biggest risk factors for delirium is

underlying dementia).

- 50M + drinks plenty of alcohol and bags of nucleic acids; image of painful toe is shown below; Tx?

o Answer = indomethacin (NSAID), colchicine (microtubule inhibitor), or prednisone; diagnosis

is acute gout; chronic gout managed first-line with xanthine oxidase inhibitor (i.e., allopurinol

or febuxostat) between attacks.

- “What are the layers of the skin?”

o You might say, “Yo that’s weird you didn’t mention this first thing cuz it’s hyper-basic.” à

Reason I held off is because USMLE doesn’t directly assess, “What are the skin layers?”

MEHLMANMEDICAL.COM 65
MEHLMANMEDICAL.COM

- 25F + long time spent in sun + erythema over 70% of her body; Dx + Tx? à answer = first-degree burn

(sunburn); Tx = supportive care / no Tx necessary; do not select answers such as silver sulfadiazine,

etc.

o First-degree (superficial) burn à epidermis only à diffuse erythema + pain.

o Second-degree (superficial partial) à involves papillary dermis à blistering + pain.

o Second-degree (deep partial) à involves reticular dermis à white/leathery; painless.

o Third-degree (deep; full-thickness) à involves hypodermis à painless.

o Fourth-degree à involves underlying muscle, tendon, or bone.

o I have not seen silver sulfadiazine and triple antibiotic ointment as answers on NBME

assessments; literature is mixed on their use.

- 20M + body weight 75kg + 30% of body surface area third-degree burned in housefire; 2CK Surg Q

asks how much / what type of fluid should be given:

o Parkland formula for surgery is used to calculate fluid resuscitation over next 24 hours.

o Parkland formula = 4mL x (% surface body area) x (weight in kg).

o 4 x 30 x 75 = 9,000 mL = 9L Ringer Lactate or 0.9% normal saline over next 24 hours.

o Give first half in first 8 hours post-burn; give second half in subsequent 16 hours.

o Yes, there is a calculation Q on Surg NBME for 2CK.

- “What do I need to know about dumb molecular skin stuff? Like the connective proteins and stuff.”

MEHLMANMEDICAL.COM 66
MEHLMANMEDICAL.COM

o Tight junction is the answer if they ask what prevents movement of solute between cells

(i.e., prevents water from getting through your skin). They can also ask about this in relation

to gastrointestinal mucosa (i.e., person with IBD demonstrates radiolabeled substance in

blood after it was injected into bowel; however person without IBD does not have the

substance in the blood; why? à answer = loss of tight junction function in IBD patient. Tight

junctions contain proteins called claudins and occludins.

o Adherens junction is the answer if they ask about what connects the actin cytoskeleton of

adjacent cells. E-cadherins are essential proteins (E-cadherins compose adherens junctions);

E-cadherins are calcium-dependent.

o Desmosome is the answer for pemphigus vulgaris and staphylococcal scalded skin syndrome;

composed to desmoglein proteins.

o Gap junctions contain connexin proteins. Unrelated to skin, but I’ve seen this asked for

myocardial muscle electrical conduction (i.e., how is electrical synchrony accomplished?) à

answer = gap junctions. I’ve seen “connexin” as distractor answer on NBME, but never as

correct answer.

- USMLE Q asks which layer vitamin D synthesis starts in à answer = stratum basale.

o 7-dehydrocholesterol in stratum basale, via UV-B radiation, à cholecalciferol.

§ NBME Q tells you patient does not get sunlight and asks “synthesis of which

substrate is impaired?” à answer = cholecalciferol; 7-dehydrocholesterol is wrong

answer.

MEHLMANMEDICAL.COM 67
MEHLMANMEDICAL.COM

o Cholecalciferol goes to liver and takes on 25-hydroxylation, becoming 25-OH-D3 (calcidiol).

§ If patient has liver disease, 25-OH-D3 is ¯; answer can also be “decreased hepatic

hydroxylation.”

o 25-OH-D3 goes to the kidney and takes on the 1-hydroxylation, via 1a-hydroxylase, under

the action of parathyroid hormone à 1,25-(OH)2-D3 (calcitriol).

o 1,25-(OH)2-D3 then goes to the small bowel and increases absorption of Ca2+ and PO43-; it

also goes to bone and converts unmineralized osteoid à mineralized hydroxyapatite.

- “Do I need to know stuff like macule, patch, plaque, etc.? Various terminology?” à Yes, but this stuff

more just applies to terms thrown around in USMLE vignettes. In other words, you will not get asked

directly, “Is this a patch?” Or, “Is this a vesicle vs bulla?” Etc.

o Macule: flat lesion <1cm.

o Patch: flat lesion >1cm; classically café au lait spots in NF1.

o Papule: raised lesion <1cm; “maculopapular” à many viral rashes and drug reactions.

o Plaque: raised lesion >1cm; psoriasis.

o Vesicle: clear-fluid collection <5mm; HSV1/2, or shingles (VZV).

o Bulla: clear-fluid collection >5mm; bullous impetigo (S. aureus); pemphigus vulgaris, bullous

pemphigoid, epidermolysis bullosa.

- 26F + brought in by ambulance following motorcycle accident + BP 160/100 + HR 52 + respiratory rate

10; image of patient shown below; Q asks mechanism for patient’s hypertension:

o Answer = “increased intracranial pressure”; image shows Battle sign (bruising over mastoid

process seen in base of skull fracture); patient can also have racoon eyes, rhinorrhea, and

otorrhea in base of skull fracture; Cushing reflex = hypertension, bradycardia, and bradypnea

as a result of increased intracranial pressure.

MEHLMANMEDICAL.COM 68
MEHLMANMEDICAL.COM

- 79M + otoscopic view of ear canal shows excessive cerumen; Q asks what the diagnosis is à answer =

“normal aging”; cerumen = ear wax; topical carbamide peroxide can dissolve excessive cerumen.

- 26M + receding hair at the temporal regions and on the scalp; Q asks genetics à answer = polygenic /

multifactorial; Dx is androgenetic alopecia (male pattern baldness); due to DHT sensitivity; topical

minoxidil (Rogaine) may be attempted for Tx.

- 25M + 2nd degree burn on leg from playing with fireworks + heals; 3 months later, area of burn

appears darker than surrounding skin; Dx + Tx? à answer = post-inflammatory hyperpigmentation;

benign hyperpigmentation that can occur following inflammation (as the name implies); no Tx

necessary; usually self-resolving; if topicals used, hydroquinone or tretinoin often effective.

- 32F + on combined oral contraceptive pills for 12 years + slightly hyperpigmented skin on cheeks and

buttocks; Dx + Tx? à answer = melasma (chloasma); benign hyperpigmentation of skin usually due to

estrogen-containing OCPs or pregnancy; worsened with sun exposure; no Tx necessary; can consider

stopping OCPs.

- 44M + alcoholic + brought in from the snow in the winter; student Q showed pic of dude’s red feet

following rewarming and they asked what electrolyte (high or low) we’re most worried about à

answer = hyperkalemia à alcoholics susceptible to rhabdomyolysis à lysis of cells releases

potassium + myoglobin is nephrotoxic and can cause acute tubular necrosis and hyperkalemia.

- Neonate + white bumps on nose + occasional cough with feeds + family Hx of atopic dermatitis; image

shown below; Q asks Tx:

Answer = no Tx necessary; “exfoliative cleanser” and “topical low-dose corticosteroid” = wrong answers; Dx =

milia (“milk spots”); clogged eccrine ducts; common, benign finding in babies.

MEHLMANMEDICAL.COM 69
MEHLMANMEDICAL.COM

YouTube
@mehlmanmedical

Instagram
@mehlman_medical

MEHLMANMEDICAL.COM 70
MEHLMANMEDICAL.COM

MEHLMANMEDICAL
HY DERMATOLOGY

All material is copyrighted and the property of mehlmanmedical.

Copyright © mehlmanmedical

MEHLMANMEDICAL.COM 71
MEHLMANMEDICAL
HY SURGERY
MEHLMANMEDICAL.COM

YouTube
@mehlmanmedical

Instagram
@mehlman_medical

MEHLMANMEDICAL.COM 2
MEHLMANMEDICAL.COM

HY Surgery

- When is sitz bath the answer? à Tx of anal fissure à Q will say “in addition to stool softeners, what’s

the next best step in Mx?” à answer = sitz bath.

- When is anal fissure the answer? à Q will say physical exam shows of the perineum shows an ulcer in

the posterior midline of the anal canal +/- an adjacent edematous skin tag at the anal verge à highest

yield descriptor is if they say the rectal exam cannot be performed because of the exquisite pain and

increased anal sphincter tone.

- When is hydrocortisone enema the answer? à sometimes for IBD à bear in mind first-line Tx for IBD

is 5-ASA compounds (mesalamine), but I mention this because “hydrocortisone enema” is a common

answer choice on surg forms and students ask me when that’s the answer.

- When is anal manometry the answer? à to Dx Hirschsprung à they’ll say a child passes one string-

like stool every 4 days despite maximum laxative therapy + there is no stool in the rectal vault; then

they’ll ask for next best step in Dx? à anal manometry (pressure study) à and of course

confirmatory biopsy is most accurate, but anal manometry is done first.

- When is “placement of elastic bands” the answer? à Tx for hemorrhoids on USMLE.

- Cystic mass at top of gluteal cleft (by the tailbone); Dx + Tx? à pilonidal cyst (aka pilonidal abscess

according to the literature) à Tx = surgical drainage of abscess (incision + drainage just off the

midline).

- How do you Dx GERD? à answer = trial of PPIs (diagnostic + therapeutic).

- When is 24-hour pH monitoring the answer? à if patients have failed PPI trial but have suggestive

symptomatology, or if initial Sx of GERD are equivocal (i.e., no burning but chronic/nocturnal cough).

- When is endoscopic ultrasound the answer? à it almost never is, but it’s an occasional answer choice

on surg forms, so I’m mentioning it; may have utility in Dx pancreatic cancer if CT is negative.

- When is esophageal manometry the answer? à USMLE wants this for achalasia after a barium or

gastrografin (water-soluble contrast) swallow has been performed showing the bird’s beak

appearance.

- 42M + 3-year-Hx of GERD + CXR shows air-fluid level posterior to the cardiac silhouette; Dx? à hiatal

hernia.

MEHLMANMEDICAL.COM 3
MEHLMANMEDICAL.COM

- Above 42M; next best step in diagnosis? à barium swallow à will show proximal stomach herniating

through the esophageal hiatus.

- 42M w/ hiatal hernia suggested on barium swallow; next best step? à esophagogastroduodenoscopy

(answer on the NBME).

- High ALP + high direct bilirubin + high amylase or lipase à gallstone pancreatitis = choledocholithiasis

- High ALP + high direct bilirubin + high amylase or lipase + remote Hx of cholecystectomy à sphincter

of Oddi dysfunction (can’t be a stone cuz the gallbladder was removed ages ago)

- High ALP + high direct bilirubin + normal amylase or lipase in someone with recent cholecystectomy

à choledocholithiasis (retained stone in cystic duct that descended, but not distal to pancreatic duct

entry point)

- Dx and Tx of choledocholithiasis à ERCP

- High ALP + high direct bilirubin + normal amylase or lipase in someone with remote cholecystectomy

à pancreatic cancer

- Dx of pancreatic cancer à CT abdo with contrast

- High ALP + high direct bilirubin + normal amylase or lipase in someone with remote cholecystectomy

+ CT is negative à cholangiocarcinoma

- High ALP + high direct bilirubin + normal amylase or lipase + diffuse pruritis + high cholesterol à

primary biliary cirrhosis (PBC)

- High ALP + high direct bilirubin + normal amylase or lipase + autoimmune disease (in pt or family) à

PBC

- Dx of PBC à anti-mitochondrial Abx next best step; liver biopsy is confirmatory

- Recent cholecystectomy + fever + abdo pain à post-op bile leak

- High ALP + high direct bilirubin + normal amylase or lipase + CT shows cystic lesion in bile duct à

choledochal cyst à do simple excision of cyst (cholangiocarcinoma not cystic + CT can be negative)

- Imaging to view liver or pancreas à CT with contrast

- Imaging to view gallbladder à Ultrasound

- Imaging to view gallbladder in suspected cholecystitis only if USS negative à HIDA scan

MEHLMANMEDICAL.COM 4
MEHLMANMEDICAL.COM

- Imaging to view bile ducts à ERCP or MRCP (choose ERCP > MRCP if both listed)

- UC + high bilirubin + high ALP à primary sclerosing cholangitis

- 8-day-old neonate + jaundice + direct bilirubin 14 mg/dL + total bilirubin 15 mg/dL; Dx? à answer =

biliary atresia à super HY Dx.

- 8-day-old neonate + jaundice + direct bilirubin 14 mg/dL + total bilirubin 15 mg/dL; next best step in

diagnosis? à answer = straight-up liver biopsy à if positive, must do liver transplant.

- 79M + Hx of atrial fibrillation + severe, acute, diffuse abdo pain; Dx? à acute mesenteric ischemia

caused by mural thrombus embolizing to SMA or IMA.

- Above 79M; next best step in Mx? à mesenteric arteriography.

- Above 79M; Tx? à antibiotics (for necrotic bowel) then laparotomy (to remove necrotic bowel) à

they will tell you in last line of vignette that IV Abx are administered and then ask for the next step,

which is just laparotomy. It should be noted that the literature mentions various Txs like

embolectomy, but the USMLE wants resection of nonviable bowel as the answer.

- 52F + short episode of ventricular fibrillation + defibrillated + now has severe abdo pain; Dx? à acute

mesenteric ischemia due to ischemia caused by VF, not an embolus à antibiotics; CT if stable; if

unstable go straight to laparotomy.

- 55F diabetic + Hx of intermittent claudication + Hx of abdo pain 1-2 hours after eating meals; Dx? à

chronic mesenteric ischemia (CMI) caused by severe atherosclerosis of SMA or IMA (essentially

angina of the bowel).

- 55F diabetic + Hx of CABG + Hx of abdo pain 1-2 hours after eating meals; next best step in Dx? à

mesenteric arteriography (CMI).

- 55F diabetic + Hx renal artery stenosis + Hx of abdo pain 1-2 hours after eating meals; Tx? à

angioplasty + stenting (CMI) to restore blood flow.

- Patient with CMI who has a 2-day Hx of severe abdo pain + fever; Dx? à acute mesenteric ischemia

(acute on chronic due to a thrombosis; essentially akin to an “MI” of the bowel) à do mesenteric

arteriography to Dx; Tx with Abx + laparotomy to remove necrotic bowel.

- What is pectinate line? à separates upper 2/3 of the anal canal (part of hindgut; endoderm-derived)

from the lower 1/3 of anal canal (aka proctodeum, which is ectodermal).

MEHLMANMEDICAL.COM 5
MEHLMANMEDICAL.COM

- Lymphatic drainage above/below pectinate line? à above: internal iliac; below: superficial inguinal.

- Arterial supply above/below pectinate line? à above: superior rectal artery; below: middle/inferior

rectal arteries.

- Venous drainage above/below pectinate line? à above: superior rectal vein; below: middle/inferior

rectal veins.

- How does pectinate line relate to hemorrhoids? à above: internal hemorrhoids (painless); below:

external hemorrhoids (painful).

- Tx for hemorrhoids? à conservative first, i.e., fiber + exercise; if they want intervention, banding

(rubber band ligation) is the answer.

- How do you Dx congenital midgut volvulus? à upper-GI series (AXR + contrast follow-through of

esophagus, stomach, and duodenum with barium or gastrografin).

- 87F + coffee bean sign on AXR + obstipated; Dx? à sigmoid volvulus

- Tx for sigmoid volvulus? à answer on surgery NBME = “sigmoidoscopy-guided placement of rectal

tube”

- 22M + stressed studying for exams + yellow eyes + has had a few similar episodes in the past + is

otherwise healthy; Dx? à answer = Gilbert syndrome (proncouned jeel-BEAR).

- Mechanism for Gilbert? à decreased expression of glucuronosyltransferase (bilirubin conjugating

enzyme) in the liver à decreased ability to take up unconjugated bilirubin at the liver à jaundice.

- Tx for Gilbert? à reassurance; benign condition.

- Criteria for pathologic jaundice? à student says “I really need to know that?” Absolutely. HY on peds

shelves and 2CK. If any one or more of the following is positive, the etiology of the kid’s jaundice is

considered pathologic:

o Any jaundice present in first 24 hours of life, period.

o Any jaundice present after one week if term or two weeks if preterm.

o Total bilirubin >15 mg/dL.

o Direct bilirubin >10% of total bilirubin, even if total is under 15 mg/dL.

o (The one everyone forgets) Rate of change of increase of total bilirubin >0.5 mg/dL/hour.

- 16F + fever + high leukocytes + RLQ pain that migrated from epigastrium; Dx? à appendicitis (easy,

but so HY how can I not at least mention it classically) à USMLE wants you to know that migration is

MEHLMANMEDICAL.COM 6
MEHLMANMEDICAL.COM

because, initially, epigastric pain = visceral pain; RLQ pain = inflammation of parietal peritoneum.

Must do a pregnancy test if female + adnexal ultrasound to look for gyn causes, i.e., ruptured cyst,

etc. If male, go straight to laparoscopy. If rule out gyn cause in female, do laparoscopic removal.

Ultrasound + CT can be done, but false-negatives have led to rupture + death, so they don’t change

management if clinical suspicion is high, which is why pt goes straight to laparoscopy if under high

suspicion for appendicitis à if during surgery the appendix is normal, answer = still remove it.

- Mallory-Weiss tear vs esophageal varices HY point à MWT usually caused by vomiting/retching in

alcoholic + presents with a little bit of blood in the vomitus; varices present with LOTS of blood à

about half of patients with ruptured varix die.

- Mallory-Weiss tear vs Boerhaave à MWT is not transmural; Boerhaave is transmural à causes

subcutaneous emphysema (crepitus due to air under the skin).

- Tx of varix? à banding + octreotide.

- Prophylaxis for varix? à propranolol.

- Person who’s vomiting; what’s the biochemical disturbance? à hypokalemic hypochloremic

metabolic alkalosis à low K, low Cl, high pH, high bicarb, low H, anion gap normal (even though it’s

alkalosis, not acidosis, the USMLE will still ask an arrow for the anion gap here).

- 2-week-old male + forceful non-bilious vomiting; Dx? à hypertrophic pyloric stenosis.

- Dx of pyloric stenosis? à abdominal ultrasound to show olive-shaped hypertrophied pylorus.

- Tx of pyloric stenosis? à myomectomy

- Who gets pyloric stenosis? à first-born males (weird, but it’s on an old NBME) + neonates taking oral

erythromycin for chlamydial ophthalmia neonatorum (erythromycin is a motilin-receptor agonist).

- 2-week-old male + bilious vomiting; Dx? à duodenal atresia, annular pancreas, congenital midgut

volvulus, or Hirschsprung (correct, Hirschsprung can present with bilious vomiting).

- 2-week-old + Down syndrome + bilious vomiting + passed meconium ok; Dx? à duodenal atresia

- 2-week-old + Down syndrome + bilious vomiting + slow to pass meconium; Dx? à Hirschsprung

- How do you Dx duodenal atresia? à abdominal x-ray (AXR) showing double-bubble sign (very HY).

- How do you Dx Hirschsprung? à rectal manometry, followed by confirmatory rectal biopsy showing

absence of ganglion cells.

- Mechanism for Hirschsprung? à failure of migration of neural crest cells distally to the rectum.

MEHLMANMEDICAL.COM 7
MEHLMANMEDICAL.COM

- Failure to pass meconium at birth. Most likely cause overall? à cystic fibrosis.

- 18-month-old + intermittent abdominal pain + crying + blood in stool; Dx? à intussusception.

- 18-month-old + intermittent squatting + crying + FOBT positive; Dx? à intussusception.

- 18-month-old + occasionally brings legs to chest + vomits + FOBT positive; Dx? à intussusception.

- 18-month-old + occasionally brings legs to chest + vomits + FOBT negative; Dx? à volvulus à this is

congenital midgut volvulus.

- 18-month-old + occasionally brings legs to chest + vomits + FOBT negative

- Presentation sounds like intussusception but no blood per rectum à answer = congenital midgut

volvulus.

- Cause of intussusception? à >99% are in kids under age 2; caused by lymphoid hyperplasia due to

viral infection (e.g., rotavirus) or recent vaccination; if in adult (usually elderly), it is caused by

colorectal cancer.

- Dx and Tx of intussusception? à USMLE wants enema as the answer. Even though ultrasound can be

done which shows a target sign, the USMLE always wants enema. And it can be any type. I’ve seen

“air contrast enema”, “air enema,” “contrast enema” all as answers. I also had a student simply get

“water-soluble contrast enema” on the exam, which means gastrografin. Barium would refer to

regular contrast.

- For contrast swallows, when to do barium vs water-soluble (gastrografin)? à barium most of the

time; if at risk of aspiration, must do barium because aspiration of gastrografin will cause

pneumonitis. If patient has suspected esophageal perforation, do not do barium, as that will cause

mediastinitis; must do gastrografin in this case.

- Level of celiac trunk and main branches + what’s it supply? à T12; splenic artery, common hepatic

artery, left gastric artery; supplies foregut (mouth to duodenum at ampulla of Vater).

- Level of SMA + what’s it supply? à L1; supplies midgut (duodenum at ampulla of Vater until 2/3 distal

transverse colon); so for instance, the right colic and middle colic arteries come off SMA.

- Level of IMA + what’s it supply? à L3; supplies hindgut (2/3 distal transverse colon until the pectinate

line 2/3 distal on the anal canal); left colic artery comes off IMA.

- Renal + gonadal (testicular in men; ovarian in women) arteries come off of L2 most often.

- Abdominal aorta bifurcates into common iliacs at L4.

MEHLMANMEDICAL.COM 8
MEHLMANMEDICAL.COM

- Abdominal aortic aneurysm occurs in males over 55 who are ever-smokers à a one-off abdo

ultrasound is done in this patient group.

- Most common locations for atherosclerosis (in descending order) à abdominal aorta, coronary

arteries, popliteal arteries, carotid arteries.

- USMLE favorite question à “Which of the following is supplied by an artery of the foregut but is not

itself derived from the foregut?” à answer = spleen.

- Short gastric arteries come off which artery? à splenic.

- What’s the main arterial supply to the pancreas? à Arteria pancreatica magna (greater pancreatic

artery) à a branch of the splenic.

- Which nerve must be severed to remove cancer at gastroesophageal junction à answer = vagus (just

memorize it; it’s on a retired NBME).

- HY structures passing through diaphragm? à “I Ate 10 Eggs At 12.” à IVC T8; T10 Esophagus +

thoracic duct; Aortic hiatus (aorta, azygous vein, thoracic duct) at T12.

- Celiac disease important points? à gluten intolerance; gluten found in wheat, oats, rye, and barley,

so therefore will get Sx after eating, e.g., pasta (too easy for the vignette to mention though); causes

flattening of intestinal villi on biopsy (image HY); patients often present with iron deficiency anemia

(HY way to differentiate from lactose intolerance); Dx with Abs: anti-endomysial IgA (anti-gliadin

IgA), anti-tissue transglutaminase IgA à after you get positive Abs in Dx of Celiac, USMLE wants

duodenal biopsy to confirm (“no further studies indicated” is the wrong answer) à Tx = dietary

changes to avoid gluten.

- Dx of C. difficile? à answer = stool AB toxin test, not stool culture (exceedingly HY).

- Tx of C. difficile? à guidelines as of Feb 2018 say oral vancomycin first-line, not metronidazole à

apparently UW is updated on this too now à note that vanc is given orally à apart from C. diff, it’s

always given IV because it has terrible oral bioavailability, but in the case of C. diff, where we want

the drug confined to the lumen of the colon, that makes sense.

- Mechanism of colonic necrosis in C. diff colonic necrosis? à answer = “cytoskeletal disruption.”

- Patient is treated with vanc for C. diff but gets recurrence weeks later; why? à answer =

“regermination of spores.”

MEHLMANMEDICAL.COM 9
MEHLMANMEDICAL.COM

- C. diff + fever of 104F + tachy + diffuse abdominal pain; next best step in Mx? à AXR à look for toxic

megacolon à Tx w/ NPO (nothing by mouth), NG decompression + rectal tube (decompression) + Abx

(vancomycin or fidaxomicin) + steroids (if UC) + correct any electrolyte imbalances (sometimes low K)

à if patient doesn’t improve with conservative therapy, must do surgery (subtotal colectomy +

ileostomy); do not do a colonoscopy on a patient with toxic megacolon as this will cause perforation.

- Damage to which nerves can cause constipation? à answer = pelvic splanchnic (because these are

parasympathetic, which drive bowel function) à in contrast, hypogastric nerves = sympathetic

(internal anal sphincter); pudendal nerve = somatic (external anal sphincter).

- 45M + cirrhosis + fluid wave + fever + abdo pain; Dx? à spontaneous bacterial peritonitis (SBP).

- 69F diabetic + undergoing peritoneal dialysis + fever + abdo pain; Dx? à SBP.

- 8M + viral infection + pedal/periorbital edema + fluid wave + fever + abdo pain; Dx? à SBP à

minimal change disease (nephrotic syndrome) causing ascites.

- SBP Dx à paracentesis (peritoneal aspiration; don’t confuse with periocardiocentesis) + do gram-

stain + look for >250 WBCs per HPF.

- Tx for SBP à ceftriaxone.

- “What do I need to know about serum-ascites albumin gradient (SAAB)?”

o Equal to the (serum albumin) – (ascitic fluid albumin).

o If SAAB >1.1, Dx = transudative process (i.e., congestive heart failure, Budd-Chiari, cirrhosis

with portal hypertension) à all cause increased hydrostatic pressure in portal vein, so fluid

leakage into peritoneal cavity carries less protein.

o If SAAB <1.1, Dx = exudative process (i.e., infection [e.g., TB, SBP]), pancreatitis, malignancy),

or nephrotic syndrome (HY); if the serum protein is very low because of nephrotic

syndrome, even though the fluid leakage into the peritoneal cavity also carries very low

protein, the result is still a low SAAB.

- Woman 20s-50s + high cholesterol + diffuse pruritis + sister has rheumatoid arthritis; Dx? à primary

biliary cirrhosis à USMLE likes “autoimmune diseases go together” in patient (or family).

- How to Dx PBC? à anti-mitochondrial Abs first; if positive, liver biopsy is confirmatory.

MEHLMANMEDICAL.COM 10
MEHLMANMEDICAL.COM

- Neonate born 32 weeks gestation + fever + abdominal distension + feeding intolerance +

hematochezia; Dx? à necrotizing enterocolitis à bowel infection in premature neonates usually <32

weeks gestation.

- Dx of NE? à abdominal x-ray (AXR) visualizing pneumatosis intestinalis (air in the bowel wall), air in

the portal vein, or free air under the diaphragm.

- Tx of NE? à NPO (nil per os; nothing by mouth), NG decompression, broad-spectrum Abx; if necrotic

bowel, must do surgery.

- 49M + Hx of abdo pain after meal; now presents with sepsis + diffuse, acute abdo pain; Dx? à

ruptured viscus (duodenum).

- Above 49M; next best step in Dx? à answer = “x-ray of chest + abdomen” to look for air under the

diaphragm (confirms diagnosis); USMLE will never give you choice A) CXR; B) AXR, etc.; they’ll either

give you CXR alone, AXR alone, or both; one of the 2CK surgery NBMEs has both as the answer.

- 42M + dysphagia to solids + liquids + no other Hx; Dx? à achalasia à inability to swallow solids +

liquids together implies neurogenic etiology.

- 42M + EtOH Hx + dysphagia to solids that progresses to include liquids; Dx? à esophageal cancer

(SCC) à dysphagia to solids that progresses to solids + liquids = cancer until proven otherwise.

- HY points about esophageal cancer? à adenocarcinoma is distal 1/3 and is caused by GERD (obesity

à low LES tone à GERD à Barrett à adenocarcinoma); SCC is upper 2/3 of esophagus and is caused

by smoking + alcohol; can also be caused by webs, burns, chemicals, and achalasia (difficult, because

achalasia is LES so your thought is, “how can that cause SCC of upper 2/3?” à probably the dysphagia

causes increased esophageal irritation, which then becomes the risk factor for SCC).

- Mechanism for achalasia? à loss of NO-secreting neurons in myenteric plexus of LES à increased LES

tone à bird’s beak appearance on contrast swallow + increased tone on esophageal manometry;

cause is often idiopathic, but Chagas disease (Trypansoma cruzi) is a known infective cause (rare).

- How do you Dx achalasia? à USMLE wants barium (or gastrografin) swallow, then manometry, then

confirmatory biopsy, in that order. There is a Q on an NBME for 2CK where both barium and

manometry were listed, and the answer was barium swallow, not manometry.

- So when is manometry the answer for achalasia? à when they show you a pic of the bird’s beak from

the barium swallow already performed, so clearly the next best step is manometry. The USMLE will

MEHLMANMEDICAL.COM 11
MEHLMANMEDICAL.COM

sometimes show a graph of a manometry that’s been performed, and you’ll simply see that the

pressure is high at the LES à hence Dx = achalasia. The confirmatory / most accurate test is biopsy of

LES showing loss of neurons.

- Tx for achalasia à surgery first-line (endoscopic myotomy is superior to pneumatic dilatation); if

patient has high surgical risk, can use botulinum toxin as first-line therapy à if fails, dCCB or nitrates.

- 42M + overweight + halitosis + gurgling sound when drinking fluids + occasionally regurgitates

undigested food; next best step in Mx? à barium (or gastrografin) swallow; Dx = Zenker.

- Above 42M + Hx of GERD à go straight to endoscopy as the answer (cancer, not Zenker).

- Location + mechanism of Zenker? à false diverticulum just superior to the cricopharyngeus on the

posterior pharyngeal wall à USMLE answers are “increased oropharyngeal pressure” and

“cricopharyngeal muscle spasm.” They want you to know it is not a congenital weakness. Dysphagia is

a risk factor because this increases oropharyngeal pressure. I’ve noticed Zenker vignettes often

mention the patient is overweight, although the association is non-specific.

- What is Whipple triad? à means insulinoma à 1) hypoglycemia, 2) Sx of hypoglycemia (diaphoresis,

tachycardia, tremulousness), 3) relief of Sx with meals (or they’ll say it gets worse between meals).

- If patient has Whipple triad, what’s next best step in Mx? à check serum C-peptide levels à now this

is where I get you a point: if C-peptide is high, the wrong answer is CT abdo to look for insulinoma

because the Dx is not automatically insulinoma. Now you’re probably like, “Really? Wait, why? I’m

not following.” à if C-peptide high, answer = first check serum hypoglycemic levels à meaning,

some patients can surreptitiously take sulfonylureas (i.e., glyburide, etc.), which are insulin

secretagogues, so they’re C-peptide levels will be high. Only after the serum hypoglycemic screen is

negative do you consider CT abdo.

- 52F + 2 kids + BMI 28 + recurrent colicky epigastric pain; next best step in Dx? à ultrasound

(cholelithiasis).

- 45F + BMI 29 + BP 140/90 + high TGAs + low HDL + elevated fasting glucose + slightly elevated AST

and ALT; Dx? à NASH (non-alcoholic steatohepatitis) à caused by metabolic syndrome à Tx is

lifestyle modification to eradicate the metabolic syndrome.

MEHLMANMEDICAL.COM 12
MEHLMANMEDICAL.COM

- Above 45F + completely normal liver enzymes; Dx? à NASH à I’ve seen plenty of Qs where all labs

values they show you are completely normal + the only thing you’re left with is, “well she’s fat / has

metabolic syndrome,” and you eliminate the others to just say, “well, this is NASH.”

- First Tx for pancreatitis in general à USMLE wants triad of NPO (nil per os; nothing by mouth) + fluids

+ NG tube à if USMLE asks for imaging, do CT with contrast to look for fluid collections + degree of

necrosis à drain fluid collections percutaneously; if pseudoabscess forms and doesn’t regress,

answer = internal drainage via ERCP or EUS (endoscopic ultrasound); if frank pus (fat enzymatic

necrotic pancreas) >30%, must do necrosectomy (excision of necrotic pancreas) or aggressive

percutaneous drainage.

- 69M + LLQ pain + fever = diverticulitis à Dx with CT with contrast of abdomen à Tx w/ Abx

(metronidazole, PLUS fluoroquinolone or Augmentin; USMLE won’t ask you the exact Abx, but you

should be aware that metro covers anaerobes below the diaphragm) à never do a colonoscopy on

someone with suspected diverticulitis, as you may cause perforation. However, after the diverticulitis

is fully treated + cleared, patient will need a follow-up colonoscopy to rule out malignancy.

- Crohn GI findings? à mouth to anus; classically terminal ileum; frequently intermittent bloody

diarrhea if colonic involvement; skip lesions causing “string sign” on contrast studies; “cobblestone

mucosa”; transmural inflammation with non-caseating granulomas; perianal fistulae; B12 + fat-

soluble vitamin malabsorption.

- 25M + oral ulcer + bloody diarrhea; Dx? à Crohn

- Extra-intestinal manifestations of Crohn? à classically erythema nodosum (red shins; not a rash; this

is panniculitis, which is inflammation of subcutaneous fat); anterior uveitis (red eyes); oxalate nephro-

/ ureterolithiasis (decreased fat absorption à more binding of fat to calcium [saponification] à

therefore less calcium binds to oxalate à more oxalate absorbed à oxalate stones).

- Any weird factoid about Crohn? à sometimes patients (+) for anti-Saccharomyces cerevisiae Abs

- How to Dx Crohn? à USMLE wants colonoscopy

- Tx for Crohn? à USMLE wants NSAIDs (either sulfasalazine or mesalamine [5-ASA] will be listed)

before steroids à use immunosuppressants late (i.e., infliximab, azathioprine, etc.).

MEHLMANMEDICAL.COM 13
MEHLMANMEDICAL.COM

- Does Crohn share anything with UC? à Yes, bear in mind in real life, there is overlap between the

two diseases, so don’t pigeonhole things; think of these disease-associations as propensities rather

than as intransigent rules.

- 28M + lower back pain worse in morning and gets better throughout the day + mouth ulcer; Dx? à

Crohn disease (oral involvement only) + sacroiliitis (back pain Sx of ankylosing spondylitis).

- 20F + bloody diarrhea + sore joints + eczematoid plaque on forehead à IBD + psoriatic arthritis

- UC GI findings? à rectum-ascending (meaning, starts at rectum and ascends; does not involve anus;

Crohn of course is mouth to anus); bloody diarrhea; not transmural (mucosa + submucosa involved

only; unlike Crohn); no granulomas (unlike Crohn); lead pipe appearance of colon on contrast studies

(unlike “string sign” of Crohn); crypt abscesses (just memorize) à lead pipe means loss of haustra (so

the colon looks smooth from the outside; this is really HY!) à USMLE might also there are

“microabscesses within the crypts” on colonoscopy (meaning crypt abscesses in UC).

- Extra-intestinal manifestations of UC? à primary sclerosing cholangitis (beaded appearance of

common bile duct; can be p-ANCA positive); pyoderma gangrenosum (crater on the forearm with

necrotic debris); like Crohn, is associated with anterior uveitis + HLA-B27 associations.

- How to Dx UC? à USMLE wants colonoscopy.

- Tx for UC? à same as Crohn for USMLE purposes, but just be aware in severe cases colectomy is

performed.

- 65M + intermittent bloody diarrhea + now has fever of 104F + abdominal pain + high leukocytes; Dx?

à answer = toxic megacolon à Dx with AXR, not colonoscopy! à if you scope, patient will perforate

and die à AXR will show dilated bowel (e.g., one NBME Q says “12-cm cecum”); in general, know that

AXR is done when you’re looking for gas.

- Where do most colonic ischemic ulcers occur? à watershed areas à splenic flexure (watershed of

SMA and IMA) + sigmoidal-rectal junction (watershed of IMA and hypogastric artery).

- 72M + advanced CVD + bloody stool; Dx? à ischemic colitis (due to ischemic ulcer).

- 55M + BMI of 33 + vignette doesn’t mention diabetes + 3 months burning in throat à Dx = GERD à

Tx? = trial of PPIs (i.e., trial of omeprazole) for two weeks à relief of Sx is consistent with GERD as

the correct Dx.

MEHLMANMEDICAL.COM 14
MEHLMANMEDICAL.COM

- 55M + BMI of 33 + poorly controlled diabetes (type I or II) + 3 months of burning in throat à Dx =

diabetic gastroparesis, NOT GERD (woahhh crazy) à first pharm Tx = metoclopramide, not PPIs. If

metoclopramide not listed, choose erythromycin (motilin receptor agonist).

- Regarding gastroparesis, the USMLE vignette will make an explicit point about bad diabetic disease,

i.e., peripheral edema (renal insufficiency due to decreased oncotic pressure from albuminuria) +/-

cataracts (osmotic damage from intracellular sorbitol) +/- urinary retention (neurogenic bladder due

to osmotic denervation leading to hypocontractile detrusor) +/- they simply say HbA1C of 12%

(diabetes is 6.5% or greater; prediabetic is 6-6.49). There will be no question as to whether they want

gastroparesis vs simple GERD.

- If the vignette (more 2CK here) doesn’t ask straight-up which drug you choose and they ask for next

best step in Mx for suspected gastroparesis à first do endoscopy to rule out physical obstruction à if

negative, then do gastric-emptying scintigraphy (scintigraphic gastric-emptying assay) à if delayed

gastric emptying, first Tx = smaller meals; if insufficient, then do metoclopramide, then add

erythromycin.

- Ursodeoxycholic acid (Ursodiol) à naturally occurring bile acid given to patients with cholesterol

cholelithiasis as an alternative to cholecystectomy. On USMLE, Dx of cholelithiasis = abdominal

ultrasound, followed by Tx = cholecystectomy. Ursodiol can be given to select patients but is not

considered the universal next best step.

- Aluminum (antacid) à causes constipation à “Aluminimum amount of feces.”

- Magnesium (antacid) à causes diarrhea à this actually showed up as a case on 2CS (correct, CS).

- Calcium carbonate (antacid) à can cause rebound gastric acid hypersecretion + milk alkali syndrome

à hypercalcemia + metabolic alkalosis.

- What is Dumping syndrome? à caused by gastric bypass surgery, diabetes, or malfunctioning pyloric

sphincter, in which stomach contents following a meal enter the duodenum too quickly; there are

two types: early vs late à both show up in vignettes (without people even realizing they’re seeing a

Dumping syndrome Q).

- Early Dumping syndrome à 10-30 minutes after a meal à rapid entry of hyperosmolar gastric

contents into duodenum à osmotic expansion of small bowel lumen à diarrhea + bloating à on

USMLE, answer = “rapid emptying of hyperosomlar chyme into small bowel.”

MEHLMANMEDICAL.COM 15
MEHLMANMEDICAL.COM

- Late Dumping syndrome à 1-2 hours after meal à rapid absorption of carbohydrates through small

bowel wall à hyperglycemia à pancreas secretes lots of insulin à rebound hypoglycemia à USMLE

merely wants you to identify this in a vignette as Dumping syndrome. They might say Hx of gastric

bypass + now there’s a meal + patient gets diarrhea +/- hypoglycemia à Dx simply = Dumping

syndrome.

- What is Blind loop syndrome? à disturbance of normal floral balance in the small bowel due to

disruption of peristalsis (i.e., surgery / post-surgical ileus), but may also be caused by conditions like

IBD and scleroderma à leads to steatorrhea + B12 def + fat-soluble vitamin deficiencies à USMLE

merely wants you to be able to make the diagnosis from a vignette à Tx is with antibiotics

(doxycycline or fidaxomicin).

- 8M + bloody stool + perfectly healthy otherwise; Dx? à Meckel diverticulum; student says, “huh, I

thought that was age 2.” I agree with you. But there’s an NBME Q where the kid was 8, and the

answer was Meckel à bleeding due to “heterotopic gastric / pancreatic tissue.”

- How to Dx Meckel diverticulum? à Meckel scan (Tc99 uptake scan that localizes to diverticulum).

- Tx for Meckel à if asymptomatic, can leave alone; if symptomatic, surgical removal.

- 16F + fever + high leukocytes + RLQ pain that migrated from epigastrium; Dx? à appendicitis (easy,

but so HY how can I not at least mention it classically) à USMLE wants you to know that migration is

because, initially, epigastric pain = visceral pain; RLQ pain = inflammation of parietal peritoneum.

Must do a pregnancy test if female + adnexal ultrasound to look for gyn causes, i.e., ruptured cyst,

etc. If male, go straight to laparoscopy. If rule out gyn cause in female, do laparoscopic removal.

Ultrasound + CT can be done, but false-negatives have led to rupture + death, so they don’t change

management if clinical suspicion is high, which is why pt goes straight to laparoscopy if under high

suspicion for appendicitis à if during surgery the appendix is normal, answer = still remove it.

- Mallory-Weiss tear vs esophageal varices HY point à MWT usually caused by vomiting/retching in

alcoholic + presents with a little bit of blood in the vomitus; varices present with LOTS of blood à

about half of patients with ruptured varix die.

- Mallory-Weiss tear vs Boerhaave à MWT is not transmural; Boerhaave is transmural à causes

subcutaneous emphysema (crepitus due to air under the skin).

- Tx of varix? à banding + octreotide.

MEHLMANMEDICAL.COM 16
MEHLMANMEDICAL.COM

- Prophylaxis for varix? à propranolol.

- Most common cause of death due to fall or MVA? à traumatic rupture of the aorta (thoracic).

- Where does rupture of the aorta occur? à where the ligamentum arteriosum wraps around the top

of the descending arch à ligament is taut but arch is more mobile à leads to shearing.

- What will the NBME/USMLE Q say for traumatic rupture à MVA or fall followed by “widening of the

mediastinum on CXR.”

- 32M + MVA + widening of mediastinum on CXR; next best step in Dx? à aortic arteriography (aka

aortography).

- Tx for traumatic rupture? à if ascending arch: labetalol + surgery; if descending arch: labetalol only.

Sodium nitroprusside is the wrong answer.

- Traumatic rupture + low BP; next best step? à labetalol (decreases shearing forces, even with low BP

it’s the answer on the NBME).

- If patient is diagnosed with bicuspid aortic valve, next best step in Mx? à annual transthoracic echos

à if valve cross-sectional area falls below 1.0 cm2 then do aortic valve replacement; there’s a surgery

NBME Q where they say cross-sectional area is 0.8 cm2 and the answer is straight-up “aortic valve

replacement.”

- Most common cause of carotid plaques? à HTN à the strong systolic impulse from the heart pounds

the carotids --> endothelial damage --> atherosclerosis.

- 55M + BP 150/90 + TIA; next best step in Mx? à carotid duplex USS à the first thing you want to

think about is, "does this guy have a carotid plaque that has resulted in a clot embolizing to his brain."

- 80M + good blood pressure (e.g., 110/70) + stroke or TIA; next best step in Mx? à ECG à you want

to think, "Does he have atrial fibrillation with a LA mural thrombus that's now embolized to the

brain."

- 80M + good blood pressure (e.g., 110/70) + stroke or TIA + ECG shows sinus rhythm with no

abnormalities; next best step in Mx? à Holter monitor à when you first see this scenario you're

probably like, "Wait, the ECG is normal, so it's not AF?" à No, it is likely AF, but AF is often

paroxysmal, so in order to detect it in this scenario, the next best step is a Holter monitor (24-hour

MEHLMANMEDICAL.COM 17
MEHLMANMEDICAL.COM

wearable ECG). This means that later in the day when he sits down to have dinner and then pops into

AF, the Holter monitor will pick it up.

- What % of people over age 80 have AF? à 8% of people over age 80 have AF, which is why age is a

huge risk factor. In other words, if the vignette says the guy is 58, AF is probably less likely just based

on shear probability, regardless of hypertensive status." And, once again, knowing that AF is often

paroxysmal is really important.

- Age 50s-60s + high BP + TIA/stroke/retinal artery occlusion; next best step in Dx? à answer = carotid

duplex ultrasound to look for carotid plaques.

- Age >75 + good BP + TIA/stroke/retinal artery occlusion; answer = ECG to look for AF à if normal, do

Holter monitor to pick up paroxysmal AF.

- 55M + good BP + carotid bruit heard on auscultation; next best step in Mx? à answer = carotid

duplex ultrasound to look for carotid plaques à in this case, if they are obvious and explicit about the

suspected etiology of the stroke, TIA, or retinal artery occlusion, then you can just do the carotid

duplex ultrasound.

- How to Mx carotid plaques? à first we have to ask whether the patient is symptomatic or

asymptomatic. A bruit does not count as symptoms (that's a sign). Symptomatic means stroke, TIA, or

retinal artery occlusion. According to recent guidelines: carotid occlusion >70% if symptomatic, or

>80% if asymptomatic à answer = do carotid endarterectomy. Below these thresholds à answer =

medical management = statin, PLUS clopidogrel OR dipyridamole + aspirin. The USMLE will actually

not be hyper-pedantic about the occlusion %s (that’s Qbank). They'll make it obvious for you which

answer they want. They'll say either 90% à answer certainly = carotid endarterectomy, or they'll say

50% à answer = medical management only. There’s one NBME Q where they say a guy has a bruit

but is asymptomatic, and has 10 and 30% occlusion in the left vs right carotids, respectively, and he’s

already on aspirin + statin, and the answer is "maintain current regimen” à if he were symptomatic,

even with low occlusion, he’d certainly need statin, PLUS clopidogrel OR dipyridamole + aspirin.

- 68F + diabetic + diffuse, dull abdo pain 1-2 hours after meals; Dx? à chronic mesenteric ischemia due

to atherosclerosis of SMA or IMA, not duodenal ulcer (if they want the latter, they’ll say 29M from

Indonesia) à essentially stable angina of the bowel.

MEHLMANMEDICAL.COM 18
MEHLMANMEDICAL.COM

- 68F + Hx of intermittent claudication + CABG + abdo pain 1-2 hours after eating meals; Dx? à chronic

mesenteric ischemia.

- 78M + Hx of AF + acute-onset severe abdo pain “out of proportion to physical exam”; Dx? à acute

mesenteric ischemia due to embolus.

- 16F + Hx of severe anorexia + BMI of 14 + has episode of ventricular fibrillation due to hypokalemia +

now has severe abdo pain; Dx? à acute mesenteric ischemia due to episode of decreased blood flow

(should be noted that hypokalemia causing arrhythmia is most common cause of death in anorexia).

- 68F + diabetic + Hx of diffuse, dull abdo pain 1-2 hours after meals + now has 2-day Hx of severe abdo

pain out of proportion to physical exam; Dx? à acute on chronic mesenteric ischemia due to

ruptured atherosclerotic plaque (akin to an “MI” of the bowel).

- Dx of acute + chronic mesenteric ischemia? à USMLE answer = mesenteric angiography.

- Tx of acute mesenteric ischemia? à endarterectomy might be able to restore blood flow if caught in

time, but on the USMLE, they will say “IV antibiotics are administered; what’s the next best step in

Mx?” and the answer is just “laparotomy.”

- Tx of chronic mesenteric ischemia à endarterectomy to clear vessel.

- 32M + pneumothorax that does not resolve following placement of a chest tube; Dx? –> ruptured

bronchus or rupture of intrathoracic trachea.

- 32M + pneumothorax with a persistent air leak despite chest tube placement; Dx? –> ruptured

bronchus or rupture of intrathoracic trachea.

- 32M + contralateral tracheal deviation + low BP; Dx? –> tension pneumothorax; Tx = needle

decompression followed by chest tube. If out in the field (i.e., not in hospital), if there is penetrating

chest trauma, tape over the wound on three sides only, so air can exit but not enter. Tension

pneumothorax need not be associated with penetrating chest trauma, but this is one of the most

common etiologies.

- Mechanism of low BP in tension pneumothorax? –> answer = compression of venous structures (IVC).

- 25M + tall +/- uses cocaine + has dyspnea + air in pleural space; mechanism? –> rupture of subapical

bleb causing spontaneous pneumothorax; Tx = can technically observe if very small and patient

stable; however answer on USMLE will still be needle decompression followed by chest tube.

MEHLMANMEDICAL.COM 19
MEHLMANMEDICAL.COM

- 35F + C-section 24-48 hours ago + crackles at both lung bases; Dx? –> atelectasis; normally the fever

is within 24 hours, but a Q on one of the forms says “two days after surgery.”

- 48M + motor vehicle accident (MVA) + rib fractures + paradoxical breathing (i.e., chest wall moves

outward with exhalation and inward with inhalation); Dx? –> flail chest.

- 48M + MVA + has severe pain and/or bruising over the sternum; Dx? –> myocardial contusion –> do

an ECG + monitor in hospital due to high risk of arrhythmia. Get troponins.

- 72F + 6-month Hx of small painless papule from a chickenpox scar on her chin; Dx? à answer =

Marjolin ulcer (squamous cell carcinoma) à SCC growing from previous scar or burn site.

- 34F + painless lump on dorsal aspect of proximal hand; Dx? à ganglion cyst; Tx = needle drainage.

- 44F + frequently wears high-heel shoes + painful lump on the underside of her foot between her third

and fourth toes; Dx? à answer = Morton neuroma à benign growth of nerve tissue between the 2nd

and 3rd, or 3rd and 4th, metatarsal heads; usually from chronic irritation from high-heel shoes; Mulder

sign is replication of Sx when the metatarsal heads are compressed together.

MEHLMANMEDICAL.COM 20
MEHLMANMEDICAL.COM

YouTube
@mehlmanmedical

Instagram
@mehlman_medical

MEHLMANMEDICAL.COM 21
MEHLMANMEDICAL.COM

MEHLMANMEDICAL
HY SURGERY

All material is copyrighted and the property of mehlmanmedical.

Copyright © mehlmanmedical

MEHLMANMEDICAL.COM 22

You might also like